tier-1 exam - rda softech private limited

420
SSC CGL SOLVED PAPERS FOR TIER-1 EXAM 20 F General Intelligence & Reasoning F Quantitative Aptitude F English Comprehension F General Awareness

Upload: khangminh22

Post on 27-Jan-2023

0 views

Category:

Documents


0 download

TRANSCRIPT

yoursmahboob.w

ordpress.com

SSC CGLSOLVED PAPERS FOR

TIER-1 EXAM

20

F General Intelligence & ReasoningF Quantitative AptitudeF English Comprehension

F General Awareness

yoursmahboob.w

ordpress.com

• Head Office : B-32, Shivalik Main Road, Malviya Nagar, New Delhi-110017

• Sales Office : B-48, Shivalik Main Road, Malviya Nagar, New Delhi-110017

Tel. : 011-26691021 / 26691713

Typeset by Disha DTP Team

DISHA PUBLICATION

ALL RIGHTS RESERVED

© Copyright Publisher

No part of this publication may be reproduced in any form without prior permission of the publisher.

The author and the publisher do not take any legal responsibility for any errors or misrepresentations

that might have crept in. We have tried and made our best efforts to provide accurate up-to-date

information in this book.

For further information about the books from DISHA,Log on to www.dishapublication.com or email to [email protected]

yoursmahboob.w

ordpress.comCONTENTS

1. SSC CGL Solved Paper Morning Shift 16 May 2010 1-22

2. SSC CGL Solved Paper Evening Shift 16 May 2010 23-44

3. SSC CGL Solved Paper Morning Shift 19 June 2011 45-64

4. SSC CGL Solved Paper Evening Shift 26 June 2011 65-85

5. SSC CGL Solved Paper Morning Shift 1 July 2012 86-109

6. SSC CGL Solved Paper Evening Shift 1 July 2012 110-133

7. SSC CGL Solved Paper Morning Shift 8 July2012 134-155

8. SSC CGL solved paper Evening Shift 8 July 2012 156-177

9. SSC CGL Solved Paper Morning Shift 21 April 2013 178-201

10. SSC CGL Solved Paper Evening Shift 21 April 2013 202-227

11. SSC CGL Solved Paper Morning Shift 19 May 2013 228-252

12. SSC CGL Solved Paper Evening Shift 19 May 2013 253-275

13. SSC CGL solved paper Morning Shift 21 May 2013 276-298

14. SSC CGL solved paper 19 October 2014 299-324

yoursmahboob.w

ordpress.com

15. SSC CGL solved paper 09 August 2015 325-348

16. SSC CGL solved paper 16 August 2015 349-372

17. SSC CGL solved paper Morning Shift 8 Sept 2016 373-384

18. SSC CGL solved paper Morning Shift 9 Sept 2016 385-395

19. SSC CGL solved paper Morning Shift 10 Sept 2016 396-406

20. SSC CGL solved paper Morning Shift 11 Sept 2016 407-416

yoursmahboob.w

ordpress.comPART-A : GENERAL INTELLIGENCE & REASONING

Directions (Qs. 1-8) : In each of the followingquestions, select the related letters/word/number fromthe given alternatives.

1. ACEG : SUWY : : BDFH : ?(a) TVZX (b) RTZV(c) TVXZ (d) RTVZ

2.M N O: :: : ?AC AD AE

(a)P

AF(b)

QAB

(c)P

AC(d)

RAD

3. 5 : 27 : : 9 : ?(a) 83 (b) 81 (c) 36 (d) 18

4. 6 : 11 : : 11 : ?(a) 6 (b) 17 (c) 21 (d) 30

5. ABE : 8 : : KLO : ?(a) 37 (b) 39 (c) 38 (d) 36

6. Sty : Plg : : Byre : ?(a) Eagle (b) Cow(c) Tiger (d) Hem

7. Patrol : Security : : Insurance : ?(a) Money (b) Policy(c) Savings (d) Risk

8. ADBC : EHFG : : ILJK : ?(a) MOPN (b) MPNO(c) ORPQ (d) MPON

Directions (Qs. 9-14) : In each of the followingquestions, select the one which is different from theother three responses.

9. (a) 25, 36 (b) 144, 169(c) 100, 121 (d) 9, 64

10. (a) Heat (b) Light(c) Bulb (d) Electricity

11. (a) Wave (b) Current(c) Tide (d) Storm

12. (a) X (b) Y(c) H (d) D

13. (a) ZKXJ (b) CMAL(c) TGRF (d) FRTK

14. (a) ABJNM (b) QRTUZ(c) IXYOQ (d) WFGOP

15. Arrange the following words according to thedictionary ?1. Inventory 2. Involuntary3. Invisible 4. Invariable5. Investigate(a) 4, 2, 5, 3, 1 (b) 4, 5, 1, 3, 2(c) 2, 5, 4, 1, 3 (d) 4, 1, 5, 3, 2

16. Find the next two letters in the given series.B C E H L ? ?(a) XY (b) MN (c) QW (d) OP

17. Which one set of letters when sequentiallyplaced at the gaps in the given letter series shallcomplete it ?a_b_a__n_bb_abbn(a) abnabb (b) bnbban(c) bnbbna (d) babban

18. Find out a set of numbers amongst the four setsof numbers given in the alternatives, which is themost similar to the numbers given in the question.Given : (6, 30, 90)(a) 6, 42, 86 (b) 7, 42, 218(c) 6, 24, 70 (d) 8, 48, 192

19. Which one number is wrong in the given series ?126, 98, 70, 41, 14(a) 98 (b) 70 (c) 126 (d) 41

20. Arrange the following in the meaningful/logicalorder :1. Exhaust 2. Night3. Day 4. Sleep5. Work(a) 1, 3, 5, 2, 4 (b) 3, 5, 1, 4, 2(c) 3, 5, 1, 2, 4 (d) 3, 5, 2, 1, 4

Directions (Qs. 21-24) : In each of the followingquestions, a series is given with one term missing.Choose the correct alternative from the given onesthat will complete the series.

21. 3, 4, 7, 11, 18, 29, ?(a) 31 (b) 39(c) 43 (d) 47

SSC Combined Graduate Level (CGL) Solved PaperMORNING SHIFT 16 MAY, 2010

yoursmahboob.w

ordpress.com

2 SSC CGL SolvedPaper

22. AGMSY, CIOUA, EKQWC, ? IOUAG, KQWCI(a) GMSYE (b) FMSYE(c) GNSYD (d) FMYES

23. 975, 864, 753, 642, ?(a) 431 (b) 314 (c) 531 (d) 532

24. 8, 24, 12, ? 18, 54(a) 28 (b) 36 (c) 46 (d) 38

25. Ashok's mother was 3 times as old as Ashok 5years ago. After 5 years she will be twice as oldas Ashok How old is Ashok today ?(a) 10 years (b) 15 years(c) 20 years (d) 25 years

26. M is the son of P. Q is the grand daughter of Owho is the husband of P. How is M related to O?(a) Son (b) Daughter(c) Mother (d) Father

27. In a row of boys, Srinath is 7th from the left andVenkat is 12th from the right. If they interchangetheir positions, Srinath becomes 22nd from theleft. How many boys are there in the row ?(a) 19 (b) 31 (c) 33 (d) 34

28. From the given alternative words, select the wordwhich cannot be formed using the letters of thegiven word :Given : IMPASSIONABLE(a) IMPASSABLE (b) IMPOSSIBLE(c) IMPASSIVE (d) IMPASSION

29. Only one meaningful word can be formed byrearranging the letter of the given jumbled word.Find out that word.Given : MUSPOPAPOTIH(a) METAMORPHIC(b) PHILANTHROPIST(c) HIPPOCAMPUS(d) HIPPOPOTAMUS

30. Which number is wrong in the given series ?1, 9, 25, 50, 81(a) 1 (b) 25 (c) 50 (d) 81

31. A bus leaves Delhi with half the number of womenas men, At Meerut, ten men get down and fivewomen fget in. Now there are equal number ofmen and women. How many passengers boardedthe bus initially at Delhi ?(a) 36 (b) 45(c) 15 (d) 30

32. If the day before yesterday was Sunday, whatday will it be three days after the day aftertomorrow ?(a) Sunday (b) Monday(c) Wednesday (d) Saturday

Directions (33) : In the following question, onestatement is given followed by two assumptions I andII. You have to consider the statement to be true evenif it seems to be at variance from commonly knownfacts. You have to decide which of the givenassumptions, if any, follow from the given statement.

33. Statements : Politicians become rich by the votesof the people.Assumptions :I. People vote to make politicians rich.II. Politicians become rich by their virtue.(a) Only I is implicit(b) Only II is implicit(c) Both I and II are implicit(d) Both I and II are not implicit

Directions (34) : In the following question, twostatements P and Q are given followed by fourconclulions I, II, III and IV. You have to consider thetwo statements to be true even if they seem to be atvariance from commonly known facts. You have todecide which of the given conclusions, if any, followthe given statements.

34. Statements :P. All men are women.Q. All women are crazy.Conclusions :I. All men are crazy.II. All the crazy are men.III. Some of the crazy are menIV. Some of the crazy are women(a) None of the condusions follows(b) All the conclusions follow(c) Only I, III and IV follow(d) Only II and III follow

35. If HOSPITAL is written as 32574618 in a certaincode, how would POSTAL be written in that code?(a) 752618 (b) 725618(c) 725168 (d) 725681

36. Find the missing number from the given responses.173 (24) 526431 (18) 325253 (?) 471(a) 22 (b) 42 (c) 30 (d) 06

37. After interchanging ÷ and +, 12 and 18, whichone of the following equations becomes correct?(a) (90 × 18) + 18 = 60 (b) (18 + 6) ÷ 12 = 2(c) (72 ÷ 18) × 18 = 72 (d) (12 + 6) × 18 = 36

yoursmahboob.w

ordpress.com

3SSC CGL SolvedPaper

38. If SPARK is coded as TQBSL, what will be thecode for FLAME ?(a) GMBNF (b) GNBNF(c) GMCND (d) GMBMF

39. Find out the correct answer for the unsolvedequation on the basis of the given equationIf 6 * 5 = 91

8 * 7 = 16910 * 7 = 211

then 11 * 10 = ?(a) 331 (b) 993 (c) 678 (d) 845

40. A child is looking for his father, he went 90 metresin the east before turning to his right. He went 20metres before turning to his right again to lookfor his father at his uncle's place 30 metres fromthis point. His father was not there. From here hewent 100 metres to his north before meeting hisfather in a street. How far did the son meet hisfather from the starting point ?(a) 80 m (b) 100 m (c) 260 m (d) 140 m

41. In the following question, D stands for any of themathematical signs at different places, which aregiven as choices under each question. Select thechoice with the correct sequence of signs whichwhen substituted makes the question. Select thechoice with the correct sequence of signs whichwhen substituted makes the question as a correctequation.24 D 4 D 5 D 4(a) × + = (b) = × +(c) + × = (d) = + ×

42. What is the number missing from the third target?

51649

92989

15?

147

(a) 45 (b) 48 (c) 51 (d) 5443. In a classroom, there are 5 rows, and 5 children A,

B, C, D and E are seated one behind the other in5 seperate rows as follows :A is sitting behind C, but in front of B.C is sitting behind E, D is sitting in front of E.The order in which they are sitting from the firstrow to the last is(a) DECAB (b) BACED(c) ACBDE (d) ABEDC

44. Little wooden cubes each with a side of one inchare put together to form a solid cube with a sideof three inches. This big cube is then painted redall over on the outside. When the big cube isbroken up into the original little ones, how manycubes will have paint on two sides ?(a) 4 (b) 8 (c) 12 (d) 0

45. K is a place which is located 2 km away in thenorth-west direction from the capital P, R is anotherplace that is located 2 km away in the south-westdirection from K. M is another place and that islocated 2 km away in north-west direction fromR. T is yet another place that is located 2 kmaway in the south-west direction from M. In whichdirection is T located in relation to P?(a) South-west (b) North-west(c) West (d) North

46. Find out which of the diagrams given in thealternatives correctly represents the relationshipstated in the question.Sharks, Whales, Turtles

(a) (b)

(c) (d)

Directions (47) : In this question, among the fouranswer figures, which figure can be formed from thecut-pieces given below in the question figure.

47. Question Figure:

Answer Figures:

(a) (b) (c) (d)

yoursmahboob.w

ordpress.com

4 SSC CGL SolvedPaper

48. How many triangles are there in the followingfigure ?

(a) 11 (b) 13(c) 9 (d) 15

49. From the given answer figures, select the one inwhich the question figure is hidden/embeddedin the same direction.Question Figure:

Answer Figures:

(a) (b) (c) (d)

50. Which answer figure is the exact mirror image ofthe given question figure when the mirror is heldfrom the right at PQ?Question Figure:

P

QAnswer Figures:

(a) (b) (c) (d)

PART-B : GENERAL AWARENESS

51. A concave lens always forms an image which is(a) real and erect(b) virtual and erect(c) real and inverted(d) virtual and inverted

52. A vitamin requires cobalt for its activity. Thevitamin is(a) Vitamin B12 (b) Vitamin D(c) Vitamin B2 (d) Vitamin A

53. One of the constituents of tear gas is(a) Ethane (b) Ethanol(c) Ether (d) Chloropicrin

54. The modulus of rigidity is the ration of(a) longitudinal stress to longitudinal strain(b) Volume stress to volume strain(c) shearing stress to shearing strain(d) tensile stress to tensile strain

55. The propagation of sound waves in a gasinvolves(a) adiabatic compression and refraction(b) isothermal compression and rarefaction(c) isochoric compression and rarefaction(d) isobaric compression and rarefaction

56. An stomic clock is based on transitions in(a) Sodium (b) Caesium(c) Magnesium (d) Aluminium

57. Plasma membrane in eukaryotic celle is made up of(a) Phospholipid(b) Lipoprotein(c) Phospholpo-protein(d) Phospho-protein

58. Which one of the following is also called the'power plants' of the cell ?(a) Golgi body (b) Mitochondrion(c) Ribosome (d) Lysosome

59. What is the chemical name of vinegar ?(a) Citric acid (b) Acetic acid(c) Pyruvic acid (d) Malic acid

60. Which of the following is not a property of heavywater ?(a) Boiling point of heavy water is lower than

that of ordinary water(b) Density of heavy water is higher than that

of ordinary water(c) Freezing point of heavy water is higher than

that of ordinary water(d) It produces corrosion

yoursmahboob.w

ordpress.com

5SSC CGL SolvedPaper

61. In which of the following processes is energyreleased ?(a) Respiration (b) Photosynthesis(c) Ingestion (d) Absorption

62. Animals living in the three trunks are known as(a) Arboreal (b) Volant(c) Amphibious (d) Aquatic

63. Arrange the following in chronological order :1. Tughlaqs 2. Lodis3. Saiyids 4. Ilbari Turks5. Khiljis(a) 1, 2, 3, 4, 5 (b) 5, 4, 3, 2, 1(c) 2, 4, 5, 3, 1 (d) 4, 5, 1, 3, 2

64. The book titled 'The Indian War of Independence'was written by(a) Krishna Verma (b) Madame Cama(c) B.G. Tilak (d) V.D. Savarkar

65. Who was the founder of The Servants of IndiaSociety?(a) G..K. Gokhale (b) M.G. Ranade(c) B.G. Tilak (d) Bipin Chandra Pal

66. The term 'Caste' was derived from(a) Portuguese (b) Dutch(c) German (d) English

67. The term 'Greater India' denotes(a) Political unity (b) Cultural unity(c) Religious unity (d) Social unity

68. Formalised system of trading agreements withgroups of countries is known as(a) Trading blocks(b) Trade ventures(c) Trade partners(d) Trade organisations

69. Mahatma Gandhi was profoundly influenced bythe writings of(a) Bernard Shaw (b) Karl Marx(c) Lenin (d) Leo Tolstoy

70. Seismic sea waves which approach the coasts atgreater force are known as(a) Tides (b) Tsunami(c) Current (d) Cyclone

71. Depression formed due to deflating action ofwinds are called(a) Playas (b) Yardang(c) Ventifacts (d) Sand dunes

72. The land of maximum biodiversity is(a) Tropical (b) Temperate(c) Monsoonal (d) Equatorial

73. In input frequency of a full wave rectifier be n,then output frequency would be

(a)n2

(b) n

(c)3n2 (d) 2n

74. Heat transfer horizontally within the atmosphereis called(a) Conduction (b) Convection(c) Absorption (d) Advection

75. Indian Standard Time relates to(a) 75.5°E longitude (b) 82.5°E longitude(c) 90.5°E longitude (d) 0° longitude

76. Who is rightly called the "Father of Local SelfGovernment" in India ?(a) Lord Mayo (b) Lord Ripon(c) Lord Curzon (d) Lord Clive

77. The Directive Principles of State Policy wasadopted from the(a) British Constitution (b) Swiss Constitution(c) U.S. Constitution (d) Irish Constitution

78. Which is the second nearest star to the Earthafter the Sun?(a) Vega (b) Sirius(c) Proxima Centauri (d) Alpha Centauri

79. The two forms of democracy are(a) Parliamentary and Presidential(b) Direct and Indirect(c) Monarchical and Republican(d) Parliamentary and King

80. Which is an extra-constitutional body ?(a) Language Commission(b) Planning Commission(c) Election Commission(d) Finance Commission

81. The Prime Minister of India is(a) Elected (b) Appointed(c) Nominated (d) Selected

82. Which is not an All India Service ?(a) Indian Administration Service(b) Indian Police Service(c) Indian Foreign Service(d) Indian Forest Service

83. The forest in Sundarban is called(a) Scrub jungle (b) Mangrove(c) Deciduous forest (d) Tundra

84. Noise is measured in(a) Watt (b) REM(c) Centigrade (d) Decibel

yoursmahboob.w

ordpress.com

6 SSC CGL SolvedPaper

85. Who among the following captured his thirdconsecutive National Billiards title in the year2009 ?(a) Pankaj Advani (b) Devendra Joshi(c) Geet Sethi (d) Dhruv Sitawala

86. The messenger satellite launched by NASA is tostudy(a) Mercury (b) Venus(c) Satrun (d) Jupiter

87. What was the name of the ship that sank near theParadip Port in September, 2009 causing an oilspill ?(a) Red Rose (b) Black Rose(c) White Rose (d) Green Rose

88. Who among the following has been honouredwith the prestigious Dadasaheb Phalke Awardfor 2007 ?(a) Yash Chopra(b) Ustad Amjad Ali Khan(c) Manna Dey(d) A. Nageshwara Rao

89. An Intelligent terminal(a) has a microprocessor, but can-not be

programmed by the user(b) can process small data processing jobs, with

the use of a large CPU(c) interacts with the user in English(d) cannot take data from the user

90. Which one of the following is not a method ofestimating National Income ?(a) Expenditure method(b) Product method(c) Matrix method(d) Income method

91. The monetary policy is India is formulated by(a) Central Government(b) Industrial Financial Corporation of India(c) Reserve Bank of India(d) Industrial Development Bank of India

92. A short-term government security paper is called(a) Share (b) Debenture(c) Mutual fund (d) Treasury bill

93. WTO basically promotes(a) Financial support (b) Global peace(c) Unilateral trade (d) Multilateral trade

94. Under which market condition do firms haveexcess capacity?(a) Perfect competition(b) Monopolistic competition(c) Duopoly(d) Oligopoly

95. Price theory is also known as(a) Macro Economics(b) Development Economics(c) Public Economics(d) Micro Economics

96. At present, India is following(a) Fixed exchange rate(b) Floating exchange rate(c) Pegged up exchange rate(d) Pegged down exchange rate

97. Who among the following won the 'ICC Cricketerof the Year Award' for the eyar 2009 ?(a) M.S. Dhoni(b) Gautam Gambhir(c) Mitchell Johnson(d) Tillakaratne Dilshan

98. The bats can fly in the dark because(a) they can see the objects in darkness(b) they have weak legs and are likely to be

attacked by predators(c) they generate flashes of light(d) they generate ultrasonic sound waves

99. What changes will happen to a bow of ice andwater kept at exactly zero degree Celsius?(a) All ice will melt(b) All water will become ice(c) No change will happen(d) Only some ice will melt.

100. National Income is the(a) Net National Product at market price(b) Net National Product at factor cost(c) Net Domestic Product at market price(d) Net Domestic Product at factor cost

PART-C : NUMERICAL APTITUDE

101. The H.C.F. and L.C.M. of two numebrs are 8 and48 respectively. If one of the numbers is 24, thenthe other number is(a) 48 (b) 36(c) 24 (d) 16

102. The greatest number, which when subtractedfrom 5834, gives a number exactly divisible byeach of 20, 28, 32 and 35, is(a) 1120 (b) 4714(c) 5200 (d) 5600

103. The ninth term of the sequence 0, 3, 8, 15, 24, 35,.... is(a) 63 (b) 70(c) 80 (d) 99

yoursmahboob.w

ordpress.com

7SSC CGL SolvedPaper

104.0.125 0.027

0.25 0.15 0.09+

- + is equal to

(a) 0.3 (b) 0.5(c) 0.8 (d) 0.9

105. The sum of the series(1 + 0.6 + 0.06 + 0.006 + 0.0006 + ....) is

(a)213 (b)

113

(c)123

(d)223

106.0.009 0.036 0.016 0.08

0.002 0.0008 0.0002´ ´ ´

´ ´ is equal to

(a) 34 (b) 36(c) 38 (d) 39

107. A number, when divided by 114, leaves remainder21. If the same number is divided by 19, then theremainder will be(a) 1 (b) 2(c) 7 (d) 17

108. The square root of 0.09 is(a) 0.30 (b) 0.03(c) 0.81 (d) 0.081

109. The number 0.121212.... in the form pq is equal to

(a)4

11 (b)2

11

(c)433

(d)233

110. If 3 6 2x 13 3 3

5 5 5

- -æ ö æ ö æ ö=ç ÷ ç ÷ ç ÷è ø è ø è ø , then x is equal to

(a) –2 (b) 2(c) –1 (d) 1

111. Two numbers are in the ratio 3 : 4. Their L.C.M. is84. The greater number is(a) 21 (b) 24(c) 28 (d) 84

112. A drum of kerosene is 34

full. When 30 litres of

kerosene is drawn from it, it remains 7

12 full. The

capacity of the drum is(a) 120 l (b) 135 l(c) 150 l (d) 180 l

113. By what least number should 675 be multipliedso as to obtain a perfect cube number ?(a) 3 (b) 5(c) 24 (d) 40

114.1 1 1 1

1 11 111 11112 2 2 2

æ ö+ + +ç ÷è ø is equal to

(a) 1236 (b) 112342

(c) 618 (d) 617115. 0.001 is equal to

(a)1

1000 (b)1

999

(c)199 (d)

19

116.4.41 0.16

2.1 1.6 0.21´

´ ´ is simplified to

(a) 1 (b) 0.1(c) 0.01 (d) 10

117. If a and b are two odd positive integers, by whichof the following integers is (a4 – b4) alwaysdivisible ?(a) 3 (b) 6(c) 8 (d) 12

118. If a = 11 and b = 9, then the value of2 2

3 3a b ab

a b

æ ö+ +ç ÷

-è ø is

(a)12

(b) 2 (c)120

(d) 20

119. 256 256 144 144112

´ - ´ is equal to

(a) 420 (b) 400(c) 360 (d) 320

120. If a and b be positive integers such that a2 – b2 =19, then the value of a is(a) 19 (b) 20(c) 9 (d) 10

121. If the ratio of cost price and selling price of anarticle be as 10 : 11, the percentage of profit is(a) 8 (b) 10(c) 11 (d) 15

122. A manufacturer marked an article at Rs. 50 andsold it allowing 20% discount. If his profit was25% then the cost price of the article was(a) Rs. 40 (b) Rs. 35(c) Rs. 32 (d) Rs. 30

yoursmahboob.w

ordpress.com

8 SSC CGL SolvedPaper

123. A shopkeeper earns a profit of 12% on selling abook at 10% discount on the printed price. Theratio fo the cost price and the printed price of thebook is(a) 45 : 56 (b) 45 : 51(c) 47 : 56 (d) 47 : 51

124. By selling a bicycle for ` 2,850, Aa shopkeepergains 14%. If the profit is reduced to 8%, then theselling price will be(a) ` 2,600 (b) ` 2,700(c) ` 2,800 (d) ` 3,000

125. By selling an article, a man makes a profit of 25%of its selling price. His profit per cent is

(a) 20 (b) 25 (c)2163 (d)

1333

126. If A's income is 50% less than that of B's, then B'sincome is what per cent mroe than that of A?(a) 125 (b) 100(c) 75 (d) 50

127. 1.14 expressed as a per cent of 1.9 is(a) 6% (b) 10%(c) 60% (d) 90%

128. Two natural numbers are in the ratio 3 : 5 andtheir product is 2160. The smaller of the numbersis(a) 36 (b) 24 (c) 18 (d) 12

129. If 60% of A = 34

of B, then A : B is

(a) 9 : 20 (b) 20 : 9(c) 4 : 5 (d) 5 : 4

130. Two successive price increases of 10% and 10%of an article are equivalent to a single priceincrease of(a) 19% (b) 20%(c) 21% (d) 22%

131.3 x 3 x 23 x 3 x

+ + - =+ - -

then x is equal to

(a)5

12(b)

125

(c)57

(d)75

132. An equilateral triangle of side 6 cm has its cornerscut off to form a regular hexagon. Area (in cm2) ofthis regular hexagon will be(a) 3 3 (b) 3 6

(c) 6 3 (d)5 3

2

133. The length (in metres) of the longest rod that canbe put in a room of dimensions 10 m × 10 m × 5 m is(a) 15 3 (b) 15 (c) 10 2 (d) 5 3

134. If ̀ 1000 is divided between A and B in the ratio3 : 2, then A will receive(a) `400 (b) `500(c) `600 (d) `800

135. What must be added to each term of the ratio 7 :11, so as to make it equal to 3 : 4 ?(a) 8 (b) 7.5(c) 6.5 (d) 5

136. A sum of money at compound interest doublesitself in 15 years. It will become eight times ofitself in(a) 45 years (b) 48 years(c) 54 years (d) 60 years

137. Buses start from a bus terminal with a speed of 20km/hr at intervals of 10 minutes. What is the speedof a man coming from the opposite directiiontowards the bus terminal if he meets the buses atintervals of 8 minutes ?(a) 3 km/hr (b) 4 km/hr(c) 5 km/hr (d) 7 km/hr

138. If the circumference of a circle is decreased by50% then the percentage of decrease in its area is(a) 25 (b) 50(c) 60 (d) 75

139. At what rate per cent per annum will a sum of Rs.1,000 amount to ̀ 1,102.50 in 2 years at compoundinterest?(a) 5 (b) 5.5 (c) 6 (d) 6.5

140. What annual payment will discharge a debt of`6,450 due in 4 years at 5% per annum simpleinterest ?(a) `1,400 (b) `1,500(c) `1,550 (d) `1,600

141. The average of the first 100 positive integers is(a) 100 (b) 51(c) 50.5 (d) 49.5

142. In a family, the average age of a father and a motheris 35 years. The average age of the father, motherand their only son is 27 years. What is the age ofthe son ?(a) 12 years (b) 11 years(c) 10.5 years (d) 10 years

143. If 5 men or 7 women can earn ̀ 5,250 per day, howmuch would 7 men and 13 women earn per day ?(a) ` 11,600 (b) ` 11,700(c) ` 16,100 (d) ` 17,100

yoursmahboob.w

ordpress.com

9SSC CGL SolvedPaper

144. If A and B together can complete a piece of workin 15 days and B alone in 20 days, in how manydays can A alone complete the work ?(a) 60 (b) 45(c) 40 (d) 30

145. By walking at 34

of his usual speed, a man reaches

his office 20 minutes later than his usual time.The usual time taken by him to reach his office is(a) 75 minutes (b) 60 minutes(c) 40 minutes (d) 30 minutes

146. A can complete a piece of work in 18 days, B in 20days and C in 30 days, B and C together start thework and are forced to leave after 2 days. Thetime taken by A alone to complete the remainingwork is(a) 10 days (b) 12 days(c) 15 days (d) 16 days

147. A train, 300 m long, passed a man, walking alongthe line in the same direction at the rate of 3 km/hrin 33 seconds. The speed of the train is(a) 30 km/hr (b) 32 km/hr

(c)832

11 km/hr (d)

83511

km/hr

Directions (148-150) : The pie chart, given here,represents the number of valid votes obtained by fourstudents who contested election for school leadership.The total number of valid votes polled was 720.

Observe the chart and answer the questions based on it.

Yasin

60°80°

100°120°

Paramjeet

Vishwanath Sivaranarayan

148. What was the minimum number of votes obtainedby any candidate?(a) 100 (b) 110(c) 120 (d) 130

149. What was the winner ?(a) Sivaraman (b) Paramjeet(c) Yasin (d) Vishwanath

150. By how many votes did the winner defeat hisnearest rival ?(a) 40 (b) 45 (c) 48 (d) 50

PART-D : ENGLISH COMPREHENSION

Directions (151-155) : In the following questions,some of the sentences have errors and some have none.Find out which part of a sentence has an error. Thenumber of that part is your answer. Your answer is(d) i.e., No error.

151. He is is university professor (a)/ but of his threesons (b)/ neither has any merit. (c)/ No error (d)

152. After knowing truth, (a)/ they took the rightdecision (b)/ in the matter. (c)/ No error (d)

153. It is time you (a)/ decide on your next (b)/ courseof action. (c)/ No error (d)

154. He who has suffered most (a)/ for the cause, (b)/let him speak. (c)/ No error (d)

155. A cum of coffee (a)/ is an excellent complement(b)/ to smoked salmon. (c)/ No error (d)

Directions (156-160) : Sentences are given withblanks to be filled in with an appropriate word(s).Four alternatives are suggested for each question.Choose the correct alternative out of the four.

156. The hotel was not too expensive,_______(a) was it ? (b) wasn't it ?(c) is it ? (d) isn't it ?

157. Like humans, zoo animals must have a dentist_____ their teeth.(a) fill (b) filled(c) filling (d) to be filled

158. It was very kind of you to do the washing-up,but you ______ it.(a) didn't have to do(b) hadn't to do(c) mightn't have done(d) mustn't have done

159. He went _____ sea alone.(a) in (b) to (c) into (d) on

160. The _____ of our civilization from an agriculturalsociety to today's complex industrial world wasaccompanied by war.(a) adjustment (b) migration(c) route (d) metamorphosis

yoursmahboob.w

ordpress.com

10 SSC CGL SolvedPaper

Directions (161-165) : In the following questionschoose the word opposite in meaning to the givenword.

161. FLORID(a) Weak (b) Pale(c) Monotonous (d) Ugly

162. VERITY(a) Sanctity (b) Reverence(c) Falsehood (d) Rarity

163. PERSPICUITY(a) Vagueness (b) Dullness(c) Unfairness (d) Unwillingness

164. FERVENT(a) Inexcilable (b) Enduring(c) Dispassionate (d) Subdued

165. MEANDERING(a) Sliding (b) Slopping(c) Strained (d) Straight

Directions (166-170) : In the following questions outof the four alternatives, choose the one which bestexpresses the meaning of the given word.

166. LUXURIANT(a) Luxury loving (b) Lovely(c) Rich (d) Abundant

167. CANTANKEROUS(a) Cancerous (b) Ferocious(c) Quarrelsome (d) Fissiparous

168. ONUS(a) Sadness (b) Happiness(c) Responsibility (d) Criticism

169. DERISION(a) Humiliation (b) Embarrassment(c) Ridicule (d) Condemnation

170. TRITE(a) Commonplace (b) Clever(c) Brief (d) Impudent

Directions (171-175) : In the following questions apart of the sentence is bold. Below are givenalternatives to the bold part at (1), (2) and (3) whichmay improve the sentence. Choose the correctalternative. In case no improvement is needed, youranswer is (4).

171. Obviously he isn't cut up to be a good teacher.(a) cut out (b) cut in(c) cut for (d) No improvement

172. Power got with money is the most craved fortoday.(a) sought after (b) wished for(c) welcomed for (d) No improvement

173. The brown shirt wants washing.(a) has to wash(b) is in need of a wash(c) requries a wash(d) No improvement

174. You are asked to copy this letter word by word.(a) word for word (b) word with word(c) word to word (d) No improvement

175. The weak man is a slave to his sensuouspleasures.(a) sensory (b) sensual(c) secondary (d) No improvement

Directions (176-180): In the following questions, outof the four alternatives, choose the one which can besubstituted for the given words/sentence.

176. An underhand device resorted to in order tojustify misconduct(a) Subterfuge (b) Manoeurce(c) Stratagem (d) Complicity

177. Impossible to describe(a) Miraculous (b) Ineffable(c) Stupendous (d) Appalling

178. One who criticises popular beliefs which hethinks is mistaken or unwise(a) Philistine (b) Iconoclast(c) Imposter (d) Cannibal

179. Detaining and confining someone(a) Interruption (b) Interrogation(c) Interment (d) Omternment

180. Science of the races of mankind(a) Genealogy (b) Epistemology(c) Ethnology (d) Sociology

Directions (181-185) : In the following questions,groups of four words are given. In each group, oneword is correctly spelt. Find the correctly spelt word.

181. (a) collaborate (b) comemorate(c) colate (d) choclate

182. (a) circuiteous (b) clairvoyant(c) chivelery (d) cavelcade

183. (a) severety (b) sovereignity(c) superiorty (d) serenity

184. (a) cummulative (b) comemmorative(c) accummulative (d) accommodative

185. (a) benidiction (b) besmerch(c) beneficient (d) benevolence

yoursmahboob.w

ordpress.com

11SSC CGL SolvedPaper

Directions (186-190) : In the following questions,the 1st and the last sentences of the passage arenumbered 1 and 6. The rest of the passage is split intofour parts and named P, Q, R and S. These four partsare not given in their proper order. Read the sentenceand find out which of the four combinations is correct.Then find the correct answer.186. 1. Education in India had a glorious beginning.

P. But after the British rule, it faced manychanges.

Q. It went on for centuries with the same glory.R. English as the medium of instruction had a

very great response.S. One of the changes was the introduction of

English as the medium of instruction.6. As the Britishers left we had a complexity of

opinions regarding English(a) PQRS (b) QPSR(c) PQSR (d) SRPQ

187. 1. It is easy to criticize the people at the helm,for the slow progress in every field.

P. We are well aware that the intellectuals areleaving our country for better employmentopportunities.

Q. Then question remains unanswered becauseour country cannot show opportunities tothe intellectuals.

R. Then, what about their obligation to theMotherland?

S. First, we should ask ourselves as to what ishappening to the young intellectuals in India.

6. This situation of 'Brain-Drain' leads to avariety of problems.

(a) PSQR (b) RPSQ(c) PSRQ (d) SPRQ

188. 1. It is the responsibility of parents to teachthe young moral values in life.

P. Many children take advantage of theirparents busy schedule.

Q. This result in children's ignorane of socialvalues.

R. The reason behind it is that parents are quitebusy nowadays.

S. Nowadays parents spend very meagre timewith children.

6. As such, the society is going away from thevalue system.

(a) SRPQ (b) PQRS(c) SQRP (d) SPQR

189. 1. The man who does his duty without anyselfish desire for fruit may be called a sanyasias well as yogi.

P. The man who has achieved much evenness oftemper will be serene, because his mere thoughtsare changed with the strength of action.

Q. He would practise yoga. i.e., evenness oftemper, and cannot but perform action.

R. The root of the matter is that one should notallow his mind to fit from object to desire toanother and from that to a third.

S. But he who abstains from action altogetheris only an idler.

6. A yogi is one who is not at teached to hisobjects of sense or to action and whose mindhas ceased to roam restlessly.

(a) SRQP (b) RQPS(c) QRSP (d) PRSQ

190. 1. This was an important day for Alattook.P. It was a cold day, but Alatook would be warm.Q. For the first time he was going to hunt seals

alone.R. First he put on his fur-lined jacket.S. Then he put on mittens and boots of deerskin

to protect his hands and feet from the cold.6. Finally he picked up the gun he had cleaned

so carefully the day before.(a) PQRS (b) QPRS(c) PRSQ (d) QRPS

Directions (191-195) : In the following questions, asentence has been given in Active Vocie/PassiveVocie, Out of the four alternatives suggested, selectthe one which best expresses the same sentence inPassive/Active Voice.

191. They have made a film based on this novel.(a) A film was based on this novel and made.(b) A film have been made based on this novel(c) A film, based on this novel, has been made(d) A film has been based and made on this

novel.192. The people couldn't move me to the hospital and

the doctor operated on me at home.(a) I couldn't be moved to the hospital and was

operated on at home by the doctor.(b) I couldn't be moved to the hospital and I

had to be operated on at home.(c) I couldn't be mvoed to the hospital and I

was operated at home by the doctor.(d) I couldn't be moved to the hospital by the

people and operated on at home.

yoursmahboob.w

ordpress.com

12 SSC CGL SolvedPaper

193. Why did he deprive you of the membership?(a) Why you were deprived of the membership?(b) Why were you depreived of his membership

by him?(c) Why was he deprived of his membership?(d) Why were you deprived of your membership

by him?194. The news has been bought to us by him.

(a) He brought us the news(b) He has brought us the news(c) He was brought the news to us(d) We brought the news to him.

195. Not a word was spoken by the criminal inself-defence.(a) The criminal spoke not a word in

self-defence.(b) The criminal in self-defence spoke no word(c) The criminal did not speak a word in

self-defence(d) The criminal spoke in self-defence not a

word.

Directions (196-200) : In the following passage, youhave one brief passage with 5 questions followingthe passage. Read the passage carefully and choosethe best answer to each question out of the fouralternatives.

PASSAGEEvery profession of trade, every art and every sciencehas its technical vocabulary, the function of which ispartly to designate things or processes which haveno names in ordinary English and partly to securegreater exactness in nomenclature. Such specialdialects or jargons are necessary in technicaldiscussion of any kind. Being universally understoodby the devotees of the particular science or art, theyhave the precision of a amthematical formula. Besides,they save time, for it is much more economical to name

a process than to describe it. Thousands of thesetechnical terms are very properly include din everylarge dictionary, yet, as a whole, they are rather on tehoutskirts of the English language than actually withinits borders.

Different occupations, however, differ widely inthe character of their special vocabularies. In tradesand handicrafts and other vocations like farming andfishing that have occupied great numbers of men fromremote times, the technical vocabuulary is very old.An average man now uses these in his ownvocabularly. The special dialects of law, medicine,divinity and philosophy have become familiar tocultivated persons.196. Special words used in technical discussion

(a) may become part of common speech(b) never last long(c) should resemble mathematical formula(d) should be confined to scientific fields

197. The writer of this article is(a) a scientist (b) a politician(c) a linguist (d) a businessman

198. This passage is primarily concerned with(a) various occupations and professions(b) technical terminology(c) scientific undertakings(d) a new language

199. It is true that(a) various professions and occupations often

interchange words(b) there is always a non-technical word that

may be substituted for the technical word(c) the average man often uses in his own

vocabulary what was once technicallanguage not emant for him

(d) everyone is interested in scientific findings200. In recent years, there has been a marked increase

in the numebr of technical terms in thenomenclature of(a) Farming (b) Fishing(c) Sports (d) Government

yoursmahboob.w

ordpress.com

13SSC CGL SolvedPaper

1. (c) As,

A C E G B D F H

+1

+1

+1

+1

Similarly,

+1

+1

+1

+1

S U W Y T V X Z

2. (a) As,

M N O P: :: :

A C A D A E A F

+1 +1 +1

+1 +1 +13. (a) As, 5 × 5 + 2 = 27

Similarly, 9 × 9 + 2 = 834. (c) As, 6 × 2 – 1 = 11

Similarly, 11 × 2 – 2 = 205. (c) As, A + B + E Þ 1 + 2 + 5 = 8

Similarly,K + L + O Þ 11 + 12 + 15 = 38

6. (b) The resting place of pig is called Sty. Similarly,the resting place of cow is called Byre.

7. (d) In order to ensure security, police ordefence personnel patrol the area. Similarly,to cover risk, insurance is done.

8. (b) As,

+4+4

+4

+4

A D B C E H F G

Similarly,

+4

+4

+4

+4

I L J K M P N O

9. (d) Except the number pair 9, 64 in all othersperfect squares of two consecutivenumbers are given.25 36¯ ¯(5)2 (6)2

144 169¯ ¯(12)2 (13)2

100 121¯ ¯(10)2 (11)2

But,9 64¯ ¯(3)2 (8)2

10. (c) Bulb is an item while all others arephenomena.

11. (c) Except Tide, all other terms are related toboth air and water. But tide is a regular riseand fall in the level of sea, caused by theattraction of the moon and sun.

12. (b) The position number of Y in the Englishalphabet is an odd number.X Þ 24, Y Þ 25H Þ 8, D Þ 4

13. (d) Z K X J

–1

–2

–1

–2

C M A L

–1

–2

T G R F

–7

+14

F R T K

14. (c) Except in letter group IXYOQ, in all othersthere is only one vowel.In the letter group IXYOQ, there are twovowels.

HINTS & SOLUTIONS

yoursmahboob.w

ordpress.com

14 SSC CGL SolvedPaper

15. (d) Arrangement of words according to theDictionary :(4) Invariable

¯(1) Inventory

¯(5) Investigate

¯(3) Invisible

¯(2) Involuntary

16. (c) The pattern is as follows :1 2 3

4 5 6

B C E

H L Q W

+ + +

+ + +

¾¾® ¾¾® ¾¾®

¾¾® ¾¾® ¾¾®

17. (b) a b b n / a bb n / a bb n / abbn18. (d) 6 × 5 = 30, 30 × 3 = 90

8 × 6 = 48, 48 × 4 = 19219. (d) 126 98 70 42 14

–28 –28 –28 –28Therefore, the number 41 is wrong the series.

20. (c) Meaningful order of the words :3. Day ® 5. Work ® 1. Exhaust ® 2. Night® 4. Sleep

21. (d) 3 + 1 = 4 ; 3 + 4 = 7 ;4 + 7 = 11 ; 7 + 11 = 1811 + 18 = 29 ; 18 + 29 = 47

22. (a)2 2 2 2 2

2 2 2 2 2

2 2 2 2 2

2 2 2 2 2

2 2 2 2 2

A C E G I K

G I K M O Q

M O Q S U W

S U W Y A C

Y A C E G I

+ + + + +

+ + + + +

+ + + + +

+ + + + +

+ + + + +

¾¾® ¾¾® ¾¾® ¾¾® ¾¾®

¾¾® ¾¾® ¾¾® ¾¾® ¾¾®

¾¾® ¾¾® ¾¾® ¾¾® ¾¾®

¾¾® ¾¾® ¾¾® ¾¾® ¾¾®

¾¾® ¾¾® ¾¾® ¾¾® ¾¾®

23. (c)975 864 753 642 531

–111 –111 –111 –111

24. (b)8 24 12 36 18 54

×3 ¸2 ×3 ×3¸225. (b) Suppose the present age of Ashok is x years

and that of his mother is y years.5 years ago3 (x – 5) = (y – 5)Þ 3x – 15 = y – 5Þ 3z – y = 10 ...(i)5 years hence,2 (x + 5) = (y + 5)

Þ 2x + 10 = y + 5Þ 2x – y = – 5 ...(ii)From equations (i) and (ii)x = 15 years

26. (a) O is the husband of P. M is the son of P.Therefore , M is the son of O.

27. (c)S V

7th 22nd

12thTotal number of boys in the row

= 22 + 12 – 1 = 3328. (c) There is no ‘V’ letter in the given word.29. (d) Meaningful word Þ HIPPOPOTAMUS30. (c)

2 2 2 2 2

1 9 25 49 81

(1) (3) (5) (7) (9)

¯ ¯ ¯ ¯ ¯

Therefore, the number 50 is wrong in theseries.

31. (b) Suppose the number of women boarded thebus at Delhi is x.Therefore, the number of men = 2xAccording to question,2x – 10 = x + 5Þ 2x – x = 10 + 5\ x = 15Total number of passengers boarded thebus initially = 3x= 3 × 15 = 45

32. (a) Day before yesterday was Sunday.Therefore, today is Tuesday.Day after tomorrow will be Thursday.Thursday + 3 = Sunday

33. (d) The statement implies that politicians winelections by the votes of people. Therefore,niether of the assumptions is implicit in thestatement.

34. (c)

Men

WomenCrazy

Conclusion I : TrueConclusion II : FalseConclusion III : TrueConclusion IV : True

yoursmahboob.w

ordpress.com

15SSC CGL SolvedPaper

35. (b) As, H O S P I T A L

3 2 5 7 4 6 1 8¯ ¯ ¯ ¯ ¯ ¯ ¯ ¯

Therefore,P O S T A L

7 2 5 6 1 8

¯ ¯ ¯ ¯ ¯ ¯

36. (a) 1 + 7 + 3 + 5 + 2 + 6 = 244 + 3 + 1 + 3 + 2 + 5 = 18Therefore,2 + 5 + 3 + 4 + 7 + 1 = 22

37. (d) (12 + 6) × 18 = 36 Þ (18 ̧ 6) × 12 = 36Þ 3 × 12 36

38. (a)

+1

+1

+1

+1+1

S P A R K T Q B S L

Similarly,

+1

+1

+1

+1+1

F L A M E G M B N F

39. (a) As, 6 × 5 = 3030 × 3 + 1 = 918 × 7 = 5656 × 3 + 1 = 16910 × 7 = 7070 × 3 + 1 = 211Similarly,11 × 10 = 110110 × 3 + 1 = 331

40. (b)

A

F

D

E B

C

20 m

30 m60 m

80 m

20 m

30 m

\ Required distance = AF

( ) ( )2 280 60= +6400 3600 10000 100m+ = =

41. (*) Option (b)24 = 4 × 5 + 4Þ 24 = 20 + 4Option (d)24 = 4 + 5 × 4Þ 24 = 4 + 20Both option (b) and (d) are correct.

42. (b) As, 5 × 3 + 1 = 1616 × 3 + 1 = 499 × 3 + 2 = 2929 × 3 + 2 = 89Therefore,15 × 3 + 3 = 4848 × 3 + 3 = 147

43. (a) 1st Row Þ D2nd Row Þ E3rd Row Þ C4th Row Þ A5th Row Þ B

44. (b) First Layer

Second Layer

Third Layer

4 cubes each of the first and third layerswill have paint on two sides only.Therefore, total number of cubes havingpaint on two sides.= 4 × 2 = 8

45. (c)

T R P

M KN NE

E

SESSWW

NW

It is clear that T is located to the West of P.46. (c) Sharks belong to class pisces. Whale is a

mammal and Turtle belongs to class reptilia.

Sharks Whale

Trutle

yoursmahboob.w

ordpress.com

16 SSC CGL SolvedPaper

47. (c)

48. (b) A

B CD

E FG

The triangles are :DABC ; DABD ; DADC ; DAFC ;DFDC ; DAFB ; DFDB ; DFBC;DGBD ; DADE ; DGBE ; DFDG ;DDBE ;

49. (d)

50. (c)

51. (b)52. (a) Vitamin B-12, also called cobalamin, is a

water-soluble vitamin that has a key role inthe normal functioning of the brain andnervous system, and the formation of redblood cells.

53. (d) 54. (b) 55. (a) 56. (b) 57. (a)58. (b) Acetic acid, also known as ethanoic acid,

is an organic chemical compound bestrecognized for giving vinegar its sour tasteand pungent smell. It is one of the simplestcarboxylic acids and has the chemicalformula CH3COOH.

59. (b)60. (a) Boiling point of heavy water is lower than

that or ordinary water61. (a)

62. (a) Arboreal animals are creature who spendthe majority of their lives in trees. They eat,sleep and play in the tree canopy. Thereare thousands of species that live in trees,including monkeys, koalas, possums,sloths, various rodents, parrots, chameleons,geckos, tree snakes and a variety of insects.

63. (d) 64. (d)65. (d) The Servants of India Society was formed

in Pune, Maharashtra, on June 12, 1905 byGopal Krishna Gokhale, who left the DeccanEducation Society to form this association.

66. (a) 67. (b)68. (a) A trade bloc is a type of intergovernmental

agreement, often part of a regionalintergovermental organization, whereregional barriers to trade, (tariffs and non-tariff barriers) are reduced or eliminatedamong the participating states.

69. (d)70. (b) A tsunami, also known as a seismic sea

wave, is a series of waves in a water bodycaused by the displacement of a largevolume of water, generally in an ocean or alarge lake.

71. (b) 72. (a) 73. (a)74. (d) Advection is the transfer of heat or matter

by the flow of a fluid, especiallyhorizontally in the atmosphere or the sea.

75. (b)76. (b) The real benchmarking of the government

policy on decentralisation can, however, beattributed to Lord Ripon who, in his famousresolution on local self-government on May18, 1882, recognised the twin considerationsof local government: (i) administrativeefficiency and (ii) political education.

77. (d) 78. (c) 79. (a)80. (b) 81. (b) 82. (c)83. (b) The Sundarbans is a natural region

comprising southern Bangladesh and a partin the Indian state of West Bengal. It is thelargest single block of tidal halophyticmangrove forest in the world.

84. (d) 85. (a) 86. (a) 87. (a)88. (c) Prabodh Chandra Dey known by his stage

name Manna Dey, was an Indian playbacksinger. He debuted in the film Tamanna in

yoursmahboob.w

ordpress.com

17SSC CGL SolvedPaper

1942, by this song “Upar Gagan Bishal” andwent on to record more than 4,000 songsfrom 1942 to 2013. The Government of Indiahonoured him with the Padma Shri in 1971,the Padma Bhushan in 2005 and theDadasaheb Phalke Award in 2007.

89. (a) 90. (c)91. (c) The Reserve Bank of India is India’s central

banking institution, which controls themonetary policy of the indian rupee. Itcommenced its operations on 1 April 1935during the British Rule in accordance withthe provisions of the Reserve Bank of IndiaAct, 1934.

92. (d) T- bills are issued to meet short-termmismatches in receipts and expenditure.Bonds of longer maturity are called datedsecurities.

93. (d) 94. (b) 95. (d) 96. (b) 97. (b)98. (d) Bats are good at flying at night because

they use sound rather than sight tonavigate. Bats send pulses of soundthrough their mouths or noses, and thesepulses echo back outlining the objects inthe bats flight path. The ears of a bat arelarge and oddly constructed but they helpit to determine where the echoes are comingfrom.

99. (c) No change will happen.100. (b) National income is the total value a

country’s final output of all new goods andservices produced in one year.

101. (d) p × q = HCF × LCM

\ Second number 8 48 16

24´= =

102. (b) 2 20, 28, 32, 352 10, 14, 16, 355 5, 7, 8, 357 1, 7, 8, 7 1, 1, 8, 1

\ LCM = 2 × 2 × 5 × 7 × 8 = 1120\ Required number= 5834 – 1120 = 4714

103. (c) 0 + 3 = 33 + 5 = 88 + 7 = 1515 + 9 = 24

24 + 11 = 3535 + 13 = 4848 + 15 = 63

63 + 17 = 80104. (c) If 0.5 = a and 0.3 = b then,

3 3

2 2a bExpression

a ab b+=

- +

( )( )2 2

2 2a b a ab b a b

a ab b+ - += = +

- += 0.5 + 0.3 = 0.8

105. (a) 1 + 0.6 + 0.06 + 0.006 + 0.0006

+ ... = 1.666...= 1. 6

6 21 19 3

= =

106. (b) Expression

0.009 0.036 0.016 0.080.002 0.0008 0.0002

´ ´ ´=´ ´

9 32 16 82 8 2

´ ´ ´=´ ´

= 3 × 2 × 3 × 2 = 36107. (b) If the first divisor is a multiple of second

divisor.Then, remainder by the second divisor.\ Remainder = 21 ̧ 19 = 2

108. (a) 0.09 0.3 0.3 0.3= ´ =

109. (c)12 40.121212.... 0.1299 33

= = =

110. (c)3 6 2x 13 3 3

5 5 5

- -æ ö æ ö æ ö=ç ÷ ç ÷ ç ÷è ø è ø è ø

3 3 3 2x 13 3 3 35 5 5 5

- - -æ ö æ ö æ ö æ öÞ =ç ÷ ç ÷ ç ÷ ç ÷è ø è ø è ø è ø

0 3 2x 13 3 35 5 5

- -æ ö æ ö æ öÞ =ç ÷ ç ÷ ç ÷è ø è ø è øÞ 2x – 1 = –3Þ 2x = –3 + 1 = –2Þ x = –1

yoursmahboob.w

ordpress.com

18 SSC CGL SolvedPaper

111. (c) Let the numbers be 3x and 4x.\ Their LCM = 12x\ 12x = 84

84x 712

Þ = =

\ Larger number= 4x = 4 × 7 = 28

112. (d) Let the capacity of the drum be x litres.

3x 7x304 12

\ - =

3x 7x 304 12

Þ - =

9x 7x 3012-Þ =

x 306

Þ =

= x = 6 × 30 = 180 litres113. (b) 675 = 5 × 5 × 3 × 3 × 3 = 5

No to be multiplied

114. (a)1 1 1 11 11 111 11112 2 2 2

+ + +

= 1234 + 2 = 1236

115. (b)10.001

999=

116. (a)4.41 0.16

2.1 1.6 0.21´

´ ´

441 16 121 16 21

´= =´ ´

117. (c) a4 – b4 = (a2 + b2) (a + b) (a – b)\ Required number = (3 + 1) ( 3 – 1) = 8

118. (a)2 2

3 3a b ab

a b+ +-

( )( )2 2

2 2a b ab

a b a b ab

+ +=- + +

1a b

=-

1 111 9 2

= =-

119. (b) If 256 = a and 144 = b, then2 2a ba b--

[a – b = 256 – 144 = 112]

( )( )( )

a b a b a ba b+ -= = +-

= 256 + 144 = 400120. (d) a2 – b2 = 19

Þ 102 – 92 = 19Þ a = 10

121. (b) Gain = 11x – 10x = ̀ x

p 100p% 100p´\ = ´

x 100 1010x= ´ =

122. (c) Marked price = ̀ 50S.P. after discount = 80% of 50= ̀ 40If the CP of article be ̀ x, then

125 x 40100´ =

40 100x125´Þ = = ̀ 32

123. (a) Let the CP be ̀ 100.\ SP = ̀ 112If the marked price be ̀ x, then90% of x = 112

112 100x90´Þ = = ̀

11209

\ Required ratio

1120100 :9

=

= 900 : 1120 = 45 : 56124. (b) C.P. of bicycle

100 2850114= ´ =

`2500

S.P. for a profit of 8%

108 2500100= ´ =

`2700

yoursmahboob.w

ordpress.com

19SSC CGL SolvedPaper

125. (d) If the S.P. of article be `x,

then its CP = x x4- =

`

3x4

x4Gain% 100

3x4

\ = ´

100 133 %3 3

= =

126. (b) Required precentage

50 100100 50= ´

-= 100%

127. (c) Required percentage

1.14 100 60%1.9

= ´ =

128. (a) Let the numbers be 3x and 5x.\ 3x × 5x = 2160

2 2160x 144 12 123 5

Þ = = = ´´

Þ x = 12\ Smaller number= 3x = 3 × 12 = 36

129. (d)A 60 3B100 4´ = ´

3 3A B5 4

Þ ´ = ´

A 3 5 5 : 4B 4 3

Þ = ´ =

130. (c) Single equivalent percentage increase inprice

10 1010 10 % 21%100´æ ö= + + =ç ÷è ø

131. (b) Tricky apporoach

3 x 3 x 213 x 3 x

+ + - =+ - -

By componendo and dividendo,

2 3 x 2 1 32 12 3 x

+ +Þ = =--

Squaring both sides,

3 x 93 x

+ =-Þ 3 + x = 27 – 9xÞ 9x + x = 27 – 3 = 24

24 12x10 5

Þ = =

132. (c) A

B C

D E

F

GH

I

Side of the regular hexagon

1 6 2 cm3= ´ =

23 3Area of the hexagon a2

\ =

3 3 2 22

= ´ ´

6 3 sq. cm.=133. (b) Length of the longest rod

2 2 2a b c+ +

2 2 210 10 5= + +

225 15 metre= =134. (c) A’s share

3 1000 6005æ ö= ´ =ç ÷è ø

` `

135. (d) Let the required number be x.

7 x 311 x 4

+\ =+

Þ 28 + 4x = 33 + 3xÞ x = 33 – 28 = 5

yoursmahboob.w

ordpress.com

20 SSC CGL SolvedPaper

136. (a)TRA P 1

100æ ö= +ç ÷è ø

15Rate2 1 1100

æ ö= +ç ÷è øCubing on both sides, we have

45Rate8 1 1100

æ ö= +ç ÷è øRequired time = 45 years

137. (c) Distance coverd in 10 minutes at 20kmph =distance covered in 8 minutes at (20 + x) kmph

( )10 820 20 x60 60

Þ ´ = +

Þ 200 = 160 + 8xÞ 8x = 40

40x 5 kmph8

Þ = =

138. (d) Circumference = 2pr (one variable)

\ The decrease in area = 50 – 50 + 50 50

100´

= –75%

139. (a)TRA P 1

100æ ö= +ç ÷è ø

21102.50 r11000 100

æ öÞ = +ç ÷è ø

211025 r110000 100

æ öÞ = +ç ÷è ø

2 2105 r1100 100

æ ö æ öÞ = +ç ÷ ç ÷è ø è ø

r 1051100 100

Þ + =

r 105100 100

Þ =

Þ r = 5%

140. (b) Let the annual instalment be ̀ x.

x 3 5x100´ ´æ ö\ +ç ÷è ø

x 2 5 x 1 5x x x 6450100 100´ ´ ´ ´æ ö æ ö+ + + + + =ç ÷ ç ÷è ø è ø

115x 110x 105x x 6450100 100 100

Þ + + + =

Þ 115x + 110x + 105x + 100x= 6450 ×100Þ 430x = 6450 × 100

6450 100x 1500430

´\ = =`

141. (c) 1 + 2 + 3 + ......+ n ( )n n 1

2+=

\ Average of these numbers

\ Averagen 1

2+=

100 1 50.52

+= =

142. (b) Father + mother= 2 × 35 = 70 yearsFather + mother + son= 27 × 3 = 81 years\ Son’s age = 81 – 70 = 11 years

143. (d) 5 men º 7 women

7 497 men 7 women5 5

\ º ´ =

\ 7 men + 13 women

49 11413 women5 5

= + =

Now,

7 women 5250ºQ `

114 women5

\

5250 114 171007 5

º ´ =`

yoursmahboob.w

ordpress.com

21SSC CGL SolvedPaper

144. (a) (A + B)’s 1 day’s work 1

15=

B’s 1 day’s work 120

=

\ A’s 1 day’s work

1 1 4 3 115 20 60 60

-= - = =

\ A alone will do the work in 60 days.

145. (b)43 of usual time = Usual time + 20 minutes

1 rd of usual time3

\

= 20 minutesUsual time 20 3\ = ´

= 60 minutes146. (c) (B + C)’s 2 days’work

1 1 3 22 220 30 60

+æ ö æ ö= + =ç ÷ ç ÷è ø è ø

1 part6

=

Remaining work 1 51 part6 6

= - =

\ Time taken by A to complete this part ofwork

5 18 15 days6

= ´ =

147. (d) If the speed of the train be x kmph, thenrelative speed= (x – 3) kmph.

= (x – 3) × 5 m / sec8

( )

300 335x 3

18

\ =- ´

Þ 5400 = 33 × 5 (x – 3)Þ 360 = 11 (x – 3)Þ 11x – 33 = 360

393 8x 35 kmph11 11

Þ = =

148. (c) yasin got the minimum votes.360 720° ºQ

72060 60 120360

\ ° º ´ =

149. (a) Sivaraman got the maximum votes. i.e.

720 120 240 votes360

´ =

He was the winner.150. (a) Angles of the difference of votes of the

winner and the nearest rival = 120 – 100 =20°

360 720° ºQ

72020 20 40360

\ ° º ´ =

151. (c) Neither is used for two things. For morethan two things, none should be used.

152. (a) After knowing the truth will be a correctusage.

153. (b) It is time/It is high time is followed by theclause in simple past that shows presenttime. Hence, decided on your next shouldbe used.

154. (c) Replace let him speak by should be allowedto speak.

155. (d) 156. (a) 157. (a)158. (d) 159. (b) 160. (d)161. (b) The word Florid (Adjective) means : rosy;

gaudy; ornated; red; having too muchdecoration or detail.The word Pale (Adjective) means : light incolour; not strong or bright; having skinthat is almost white because of illness.Hence, the words florid and pale areantonymous.

162. (c) The word Verity (Noun) means : a belief orprinciple about life that is accepted as true;truth).Hence, the words verity and falsehood areantonymous.

yoursmahboob.w

ordpress.com

22 SSC CGL SolvedPaper

163. (a) The word Perspicuity (Noun) means :clarity.The word vagueness (Noun) means : noclarity in a person’s mind.Hence, the words perspicuity andVagueness are antonymous.

164. (c) The word Fervent (Adjective) means :having or showing very strong and sincerefeelings about something; ardent.The word Dispassionate (Adjective) means: not influenced by emotion; impartial.Hence, the words fervent and dispassionateare antonymous.

165. (d) The word Meandering (Adjective) means : notstraight ; curved ; a course that does notfollow a straight path.Hence, the words meandering and straightare antonymous.

166. (d) The word Luxuriant (Adjective) means :growing thickly and strongly; rich insomething that is pleasant or beautiful;abundant.

167. (c) The word Cantankerous (Adjective) means: bad tempered and always complaining.Hence, the words cantankerous andquarrelsome are synonymous.

168. (c) The word Onus (Noun) means : theresponsibility for something.

169. (c) The word Derision (Noun) means :rodicule; mockery; a strong feeling thatsomebody/something is ridiculous and notworth considering seriously.

170. (a) The word Trite (Adjective) means : dull andboring because it has been expressed somany times before; not original; banal; veryordinary and containing nothing that isinteresting or important.

Hence, the words trite and commonplaceare synonymous.

171. (a) Phrase ‘cut out’ means : to have the qualitiesand abilities needed for something.

172. (d)173. (c) requires a wash174. (a) word for word means : in exactly the same

words or when translated exactly equivalentwords.

175. (b) The word Sensual (adjective) means:connected with your physical feelings;giving pleasure to your physical senses;especially to sexual pleasures.

176. (b) 177. (b) 178. (b) 179. (d) 180. (c)181. (a) Correct spellings of other words are :

commemorate, collate and chocolate.182. (b) Correct spellings of other words are :

circuitous, chivalry and cavalcade.183. (d) Correct spellings of other words are :

severity, sovereignty and superiority.184. (d) Correct spellings of other words are :

cumulative, commemorative and accumulative.185. (d) Correct spellings of other words are :

benediction, besmirch and beneficent.186. (b) 187. (d) 188. (a) 189. (a) 190. (b)191. (c) A film, based on this novel, has been made192. (a) I couldn’t be moved to the hospital and

was operated on at home by the doctor.193. (d) Why were you deprived of your

membership by him ?194. (b)195. (c) The criminal did not speak a word in

self-defence.196. (c) 197. (c) 198. (b)199. (c) The average man often uses in his own

vocabulary what was once technicallanguage not meant for him

200. (d)

yoursmahboob.w

ordpress.com

23SSC CGL SolvedPaper

PART-A : GENERAL INTELLIGENCE & REASONING

1. Babu is Rahim's neighbour and his house is 200metres away in the north west direction. Josephis Rahim's neighbour and his house is located200 metres away in the south west direction. Gopalis Joseph's neighbour and he stays 200 metresaway in the south east direction. Roy is Gopal'sneighbour and his house is located 200 metresaway in the north east direction. Then where isthe position of Roy's house in relation to Babu's?(a) South east (b) South west(c) North (d) North east

2. A group of friends are sitting in an arrangementone each at the corner of an octagon. All are facingthe centre. Mahima is sitting diagonally oppositeRama, who is on Sushma's right. Ravi is next toSushma and opposite Girdhar, who is onChandra's left. Savitri is not on mahima's rightbut opposite Shalini. Who is on Shalini's right?(a) Ravi (b) Mahima(c) Girdhar (d) Rama

3. A cube has the following figures drawn on itsfive faces. The top surface is blank. The ellipse isbetween the cross and the triangle. The square ison the right of the triangle. The ellipse and thesquare are opposite to each other. Which face isthe circle on ?(a) On the top(b) Opposite to ellipse(c) Opposite to triangle(d) At the bottom

Directions (4-11) : In each of the following questions,select the related word/letters/ number/figure fromthe given alternatives.

4. Fox : Cunning : : Rabbit : ?(a) Courageous (b) Dangerous(c) Timid (d) Ferocious

5. Flexible : Rigid : : Confidence : ?(a) Diffidence (b) Indifference(c) Cowardice (d) Scare

6. AZCX : BYDW : : HQJO : ?(a) GRFP (b) IPKM(c) IPKN (d) GRJP

7. QIOK : MMKO : : YAWC : ?(a) USGA (b) UESG(c) VUES (d) SUEG

8.ABC BCD CDE: :: : ?

F I L

(a)DEF

O(b)

DEFN

(c)EDF

O(d)

DEFM

9. 1 : 8 : : 27 : ?(a) 37 (b) 47(c) 57 (d) 64

10. 24 : 126 : : 48 : ?(a) 433 (b) 192(c) 240 (d) 344

11. 987 : IHG : : 654 : ?(a) FDE (b) FED(c) EFD (d) DEF

Directions (12–18) : In each of the following questionsfind the odd word/letters/number/ figure from thegiven responses.

12. (a) Room (b) Chamber(c) Veranda (d) Cabin

13. (a) Mouth Organ (b) Electric Guitar(c) Keyboard (d) Sonata

14. (a) A (b) 1(c) D (d) E

15. (a) RNMP (b) JFEH(c) RPOQ (d) HDCF

16. (a) AbcdE (b) IfghO(c) ApqrL (d) UlmnE

17. (a) 6243 (b) 2643(c) 8465 (d) 4867

18. (a) 49 – 33 (b) 62 – 46(c) 83 – 67 (d) 70 – 55

19. From amongst the given alternatives, select theone in which the set of numbers is most like theset of numbers given below :(6, 14, 30)(a) 4, 16, 28 (b) 7, 12, 22(c) 6, 12, 22 (d) 5, 12, 20

SSC Combined Graduate Level (CGL) Solved PaperEVENING SHIFT 16 MAY 2010

yoursmahboob.w

ordpress.com

24 SSC CGL SolvedPaper

20. Which one of the given responses would be ameaningful order of the following words ?A. Family B. CommunityC. Member D. LocalityE. Country(a) C, A, D, B, E (b) C, A, B, D, E(c) C, A, B, E, D (d) C, A, D, E, B

21. Arrange the following words according to thedictionary :A. TORTOISE B. TORONTOC. TORPED D. TORUSE. TORSEL(a) B, E, C, A, D (b) B, E, C, D, A(c) B, C, E, A, D (d) B, C, E, D, A

22. Which set of letters when sequentially placed atthe gaps in the given letter series shall completeit ?__a__aaaba__ __ ba__ ab __(a) abaaaa (b) abaaba(c) aababa (d) ababaa

Directions (23-27): In each of the following questions,find the missing number/ letters/ figure from the givenresponses :

23. a, r, c, s, e, t, g, ___, ___(a) x, z (b) u, i(c) w, y (d) v, b

24. (?), PSVYB, EHKNQ, TWZCF, ILORU(a) BEHKN (b) ADGJM(c) SVYBE (d) ZCFIL

25. 0, 4, 18, 48, ? 180(a) 58 (b) 68(c) 84 (d) 100

26. 36, 28, 24, 22 ?(a) 18 (b) 19(c) 21 (d) 22

27. 7, 9, 13, 21, 37, ?(a) 58 (b) 63(c) 69 (d) 72

28. Select the number which does NOT belong tothe given series :232, 343, 454, 564, 676(a) 676 (b) 454(c) 343 (d) 564

29. A bus left with some definite number ofpassengers. At the first stop, half the passengersleft the bus and 35 boarded the bus. At the second

stop 15

the of the passengers left and 40 boarded

the bus. Then, the bus moved with 80 passengers

towards its destination without stopping anywhere. How many passengers were thereoriginally ?(a) 25 (b) 30(c) 40 (d) 50

30. If the day after tomorrow is Sunday, what daywas tomorrow's day before yesterday?(a) Friday (b) Thursday(c) Monday (d) Tuesday

31. A man is 3 years older than his wife and fourtimes as old as his son. If the son becomes 15years old after 3 years, what is the present age ofthe wife ?(a) 60 years (b) 51 years(c) 48 years (d) 45 years

32. X and Y are brothers. R is the father of Y. S is thebrother of T and maternal uncle of X. What is Tto R?(a) Mother (b) Wife(c) Sister (d) Brother

33. Suresh is 7 ranks ahead of Ashok in the class of39 students. If Ashok's rank is 17th from the last,what is Suresh's rank from the start ?(a) 16th (b) 23th(c) 24th (d) 15th

34. A word/set of letters given in capital letters isfollowed by four answer words. Out of these onlyone cannot be formed by using the letters of thegiven word/set of letters. Find out that word :INDETERMINATE(a) DETERMINE (b) RETINUE(c) REMINDER (d) RETINA

35. A group of alphabets are given with each beingassigned a numerical code. These have to beunscrambled into a meaningful word and thecorrect code so obtained may be indicated fromthe given responses ?R A H K S1 2 3 4 5(a) 5 1 2 3 4 (b) 5 4 2 1 3(c) 5 3 2 1 4 (d) 5 3 1 2 4

36. A statement is given followed by twoassumptions, (1) and (2). You have to considerthe statement to be true, even if it seems to be atvariance from commonly known facts. You are todecide which of the given assumptions candefinitely be drawn from the given statement.Indicate your answer.Statement : Theoretical education does not bringin economic advancement and it lends to a steadyloss of confidence and money in the country.

yoursmahboob.w

ordpress.com

25SSC CGL SolvedPaper

Assumptions :(1) There is close relationship between

development of confidence and economicdevelopment

(2) Theoretical education makes pricelesscontribution for development of confidence.

(a) Only 1 is implicit(b) Only 2 is implicit(c) Both 1 and 2 are implicit(d) Both 1 and 2 are not implicit

37. Two statements are given followed by fourconclusions, I, II, III and IV. You have to considerthe statements to be true, even if they seem to beat variance from commonly known facts. You areto decide which of the given conclusions candefinitely be drawn from the given statements.Indicate your answer.Statements :(A) No cow is a chair(B) All chairs are tables.Conclusions :I. Some tables are chairs.II. Some tables are cowsIII. Some chairs are cowsIV. No table is a cow(a) Either II or III follow(b) Either II or IV follow(c) Only I follows(d) All conclusions follow

38. If HONESTY is written as 5132468 andPOVERTY as 7192068, how is HORSE written ina certain code?(a) 50124 (b) 51042(c) 51024 (d) 52014

39. In a certain code SISTER is written as RHRSDQ.How is UNCLE written in that code ?(a) TMBKD (b) TBMKD(c) TVBOD (d) TMKBD

40. If 841 = 3,633 = 5,425 = 7 then 217 – ?(a) 6 (b) 7(c) 8 (d) 9

41. The following equations follow a common property.Find out the correct value to complete D :A = 51 (714) 14 :B = 61 (915) 15 :C = 71 (1136) 16 :D = 81 (?) 17(a) (1377) (b) (1378)(c) (1356) (d) (1346)

42. After interchanging ̧ and =, 2 and 3 which oneof the following statements becomes correct ?(a) 15 = 2 ̧ 3 (b) 5 ¸ 15 = 2(c) 2 = 15 ̧ 3 (d) 3 = 2 ̧ 15

43. 25 * 2 * 6 = 4 * 11 * 0Which set of symbols can replace * ?(a) x, –, x, + (b) +, –, ×, +(c) ×, +, ×, – (d) ×, +, +, ×

44. Find the missing number from the given responses:5 6 124 3 42 3 ?

18 27 96(a) 4 (b) 5(c) 3 (d) 6

45. Peter walked 8 kms, west and turned right andwalked 3 kms. The again he turned right andwalked 12 kms. How far is he from the startingpoint ?(a) 7 (b) 8(c) 4 (d) 5

46. Choose the correct figure that represents thegiven relation:Blue eye4s, females, doctors

(a) (b)

(c) (d)

47. Among the four answer figures, which figure canbe formed from the cutpieces given below in thequestion figure ?Questions Figure :

Answer Figures :

(a) (b)

(c) (d)

yoursmahboob.w

ordpress.com

26 SSC CGL SolvedPaper

48. If the mirror is placed on the line LM, then whichof the answer figures is the right image of thegiven question figure?Questions Figure :

L MAnswer Figures :

(a) (b)

(c) (d)

49. How many triangles are there in the followingfigure ?

(a) 20 (b) 24(c) 28 (d) 32

50. From the given answer figures, select the one inwhich the question figure is hidden/embedded.Questions Figure :

Answer Figures :

(a)

(b)

(c)

(d)

PART-B : GENERAL AWARENESS

51. Which of the following Indonesian regions wasa victim of massive earthquake in 2004 ?(a) Irian Jaya (b) Sumatra(c) Kalibangan (d) Java

52. The first non-stop air-conditioned 'DURANTO'train was flagged off between(a) Sealdah – New Delhi(b) Mumbai – Howrah(c) Bangalore – Howrah(d) Chennai – New Delhi

53. Which among the following agencies releasedthe report, Economic Outlook for 2009–10 ?(a) Planning Commission(b) PM's Economic Advisory Council(c) Finance Commission(d) Reserve Bank of India

54. India and U.S. have decided to finalise agreementsrelated to which of the following ?(a) Trade and Investment(b) Intellectual Property(c) Traditional Knowledge(d) All of the above

55. Which one of the following states does not formpart of Narmada River basin ?(a) Madhya Pradesh (b) Rajasthan(c) Gujarat (d) Maharashtra

56. Which of the following countries has recentlybecome the third largest market for Twitter ?(a) China (b) India(c) Brazil (d) Indonesia

57. The exchange of commodities between twocountries is referred as(a) Balance of trade (b) Bilateral trade(c) Volume of trade (d) Multilateral trade

yoursmahboob.w

ordpress.com

27SSC CGL SolvedPaper

58. Soil erosion on hill slopes can be checked by(a) Afforestation(b) Terrace cultivation(c) Strip cropping(d) Contour ploughing

59. Who coined the word 'Geography'?(a) Ptolemy (b) Eratosthenese(c) Hacataus (d) Herodatus

60. Which of the following is called the 'ecologicalhot spot of India'?(a) Western Ghats(b) Eastern Ghats(c) Western Himalayas(d) Eastern Himalayas

61. The art and science of map making is called(a) Remote Sensing (b) Cartography(c) Photogrammetry (d) Mapping

62. The age of the Earth can be determined by(a) Geological Time Scale(b) Radio-Metric Dating(c) Gravity method(d) Fossilization method

63. The monk who influenced Ashoka to embraceBuddhism was(a) Vishnu Gupta (b) Upa Gupta(c) Brahma Gupta (d) Brihadratha

64. The declaration that Democracy is a Government'of the people' by the people; for the people' wasmade by(a) George Washington(b) Winston Churchill(c) Abraham Lincoln(d) Theodore Roosevelt

65. The Lodi dynasty was founded by(a) Ibrahim Lodi (b) Sikandar Lodi(c) Bahlol Lodi (d) Khizr Khan

66. Harshvardhana was defeated by(a) Prabhakaravardhana(b) Pulakesin II(c) Narasimhasvarma Pallava(d) Sasanka

67. Who among the following was an illiterate ?(a) Jahangir (b) Shah Jahan(c) Akbar (d) Aurangazeb

68. Which Governor General is associated withDoctrine of Lapse ?(a) Lord Ripon (b) Lord Dalhousie(c) Lord Bentinck (d) Lord Curzon

69. India attained 'Dominion Status' on(a) 15th January, 1947 (b) 15th August, 1947(c) 15th August, 1950 (d) 15th October, 1947

70. Despotism is possible in a(a) One party state(b) Two party state(c) Multi party state(d) Two and multi party state

71. Marx belonged to(a) Germany (b) Holland(c) France (d) Britain

72. Which one of the following is the guardian ofFundamental Rights ?(a) Legislature (b) Executive(c) Political parties (d) Judiciary

73. Sarkaria Commission was concerned with(a) Administrative Reforms(b) Electoral Reforms(c) Financial Reforms(d) Centre-State relations

74. The speaker of the Lok-Sabha has to addresshis/her letter of resignation to(a) Prime Minister of India(b) President of India(c) Deputy Speaker of Lok Sabha(d) Minister of Parliamentary Affairs

75. A want becomes a demand only when it is backedby the(a) Ability to purchase(b) Necessity to buy(c) Desire to buy(d) Utility of the product

76. The terms 'Micro Economics' and "MacroEconomics" were coined by(a) Alfred Marshall (b) Ragner Nurkse(c) Ragner Frisch (d) J.M. Keynes

77. During periods of inflation, tax rates should(a) increase (b) decrease(c) remain constant (d) fluctuate

78. Which is the biggest tax paying sector in India ?(a) Agriculture sector (b) Industrial sector(c) Transport sector (d) Banking sector

79. "Economics is what it ought to be" – Thisstatement refers to(a) Normative economics(b) Positive economics(c) Monetary economics(d) Fiscal economics

80. The excess of price a person is to pay rather thanforego the consumption of the commodity iscalled(a) Price(b) Profit(c) Producer's surplus(d) Consumer's surplus

yoursmahboob.w

ordpress.com

28 SSC CGL SolvedPaper

81. Silver halides are used in photographic platesbecause they are(a) oxidised in air(b) soluble in hyposolution(c) reduced by light(d) totally colourless

82. Tetra ethyle lead (TEL) is(a) a catalyst in burning fossil fuel(b) an antioxidant(c) a reductant(d) an antiknock compound

83. Curie point is the temperature at which(a) Matter becomes radioactive(b) A metal loses magnetic properties(c) A metal loses conductivity(d) Transmutation of metal occurs.

84. The isotope used for the production of atomicenergy is(a) U-235 (b) U-238(c) U-234 (d) U-236

85. The acceleration due to gravity at the equator(a) is less than that at the poles(b) is greater than that at the poles(c) is equal to that at the poles(d) does not depend on the earth's centripetalacceleration

86. Which of the following is not a nucleon ?(a) Proton (b) Neutron(c) Electron (d) Positron

87. The material used in the manufacture of lead pencilis(a) Graphite (b) Lead(c) Carbon (d) Mica

88. Angle of friction and angle of repose are(a) equal to each other(b) not equal to each other(c) proportional to each other(d) None of the above

89. Processor's speed of a computer is measured in(a) BPS (b) MIPS(c) Baud (d) Hertz

90. 'C' language is a(a) Low level language(b) High level language(c) Machine level language(d) Assembly level language

91. What happens to a person who receives the wrongtype of blood?(a) All the arteries constrict(b) All the arteries dialates(c) The RBCs agglutinate(d) The spleen and lymphnodes deteriorate

92. NIS stands for(a) National Infectious diseases seminar(b) National Irrigation Schedule(c) National Immunisation Schedule(d) National Information Sector

93. If all bullets could not be removed from gun shotinjury of a man, it may cause poisoning by(a) Mercury (b) Lead(c) Iron (d) Arsenic

94. Ringworm is a ................ disease.(a) Bacterial (b) Protozoan(c) Viral (d) Fungal

95. Pituitary gland is situated in(a) the base of the heart(b) the base of the brain(c) the neck(d) the abdomen

96. Who discovered cement ?(a) Agassit (b) Albertus Magnus(c) Joseph Aspdin (d) Janseen

97. According to RBI's Report on the trend andprogress of banking, the Non-performing Assets(NPA's) in India for 2008-09 for Indian Banks in2008 have stood at(a) 2.3 per cent (b) 2.6 per cent(c) 3.5 per cent (d) 5.2 per cent

98. Window 7, the latest operating system fromMicrosoft Corporation has .............. Indianlanguages fonts.(a) 14 (b) 26(c) 37 (d) 49

99. TRIPS and TRIMS are the terms associated with(a) IMF (b) WTO(c) IBRD (d) IDA

100. A Presidential Ordinance can remain in force(a) For Three months(b) For six months(c) For nine months(d) Indefinitely

PART-C : NUMERICAL APTITUDE

101. In how many years will a sum of ̀ 800 at 10% perannum compound interest, compounded semi-annually becomes ̀ 926.10 ?

(a)112

(b)213

(c)123

(d)122

yoursmahboob.w

ordpress.com

29SSC CGL SolvedPaper

102. In a 100m race, Kamal defeats Bimal by 5 seconds.If the speed of Kamal is 18 Kmph, then the speedof Bimal is(a) 15.4 kmph (b) 14.5 kmph(c) 14.4 kmph (d) 14 kmph

103. A train, 240 m long crosses a man walking alongthe line in opposite direction at the rate of 3 kmphin 10 seconds. The speed of the train is(a) 63 kmph (b) 75 kmph(c) 83.4 kmph (d) 86.4 kmph

104. A boatman rows 1 km in 5 minutes, along thestream and 6 km in 1 hour against the stream. Thespeed of the stream is(a) 3 kmph (b) 6 kmph(c) 10 kmph (d) 12 kmph

105. A can compete 13

of a work in 5 days and B, 25

ofthe work in 10 days. In how many days both Aand B together can complete the work ?

(a) 10 (b)398

(c)485 (d)

172

106. 7 men can complete a piece of work in 12 days.How many additional men will be required tocomplete double the work in 8 days ?(a) 28 (b) 21(c) 14 (d) 7

107. One pipe fills a water tank three times faster thananother pipe. If the two pipes together can fill theempty tank in 36 minutes, then how much timewill the slower pipe alone take to fill the tank ?(a) 1 hour 21 minutes (b) 1 hour 48 minutes(c) 2 hours (d) 2 hour 24 minutes

108. In an examination, a student scores 4 marks forevery correct answer and loses 1 mark for everywrong answer. A student attempted all the 200questions and scored, in all 200 marks. Thenumber of questions, he answered correctly was(a) 82 (b) 80(c) 68 (d) 60

109. The average of odd numbers upto 100 is(a) 50.5 (b) 50(c) 49.5 (d) 49

110. If A's income is 25% less than B's income, byhow much percent is B's income more than thatof A?(a) 25 (b) 30

(c)1333 (d)

2663

111. (12 + 22 + 32 + ...... + 102) is equal to(a) 380 (b) 385(c) 390 (d) 392

112. The sixth term of the sequence 2, 6, 11, 17, ..... is(a) 24 (b) 30(c) 32 (d) 36

113. Two number are in the ratio 7 : 11. If 7 is added toeach of the numbers, the ratio becomes 2 : 3. Thesmaller number is(a) 39 (b) 49(c) 66 (d) 77

114.1 1 1 1

1 1 1 .... 13 4 5 25

æ ö æ ö æ ö æ ö- - - -ç ÷ ç ÷ ç ÷ ç ÷è ø è ø è ø è ø is equal to

(a)225 (b)

125

(c)19125

(d)1

325115. A number, when divided by 136, leaves remainder

36. If the same number is divided by 17, theremainder will be(a) 9 (b) 7(c) 3 (d) 2

116. Simplified form of

5/335

5x

5--é ùæ öê úç ÷ê úç ÷ê úè øë û

is

(a) x5 (b) x–5

(c) x (d)1x

117. A 4–digit number is formed by repeating a 2–digit number such as 1515, 3737, etc. Any numberof this form is exactly divisible by(a) 7 (b) 11(c) 13 (d) 101

118. (0.1 × 0.01 × 0.001 × 107) is equal to

(a) 100 (b)1

10

(c)1

100 (d) 10

119. It 1

2p 4p

+ = the value of 33

1p8p

+ is

(a) 4 (b) 5(c) 8 (d) 15

120. If p and q represent digits, what is the possiblemaximum value of q in the statement 5p9 + 327 +2q8 = 1114?(a) 9 (b) 8(c) 7 (d) 6

yoursmahboob.w

ordpress.com

30 SSC CGL SolvedPaper

121. The least among the fractions 15 19 24 34, , ,16 20 35 35

is

(a)3435 (b)

1516

(c)1920

(d)2425

122. 1.27 in the form pq is equal to

(a)127100 (b)

73100

(c)1411 (d)

1114

123.3.25 3.20 3.20 3.05

0.064´ - ´

is equal to

(a) 1 (b)12

(c)1

10(d) 10

124. Out of six consecutive natural numbers, if thesum of first three is 27, what is the sum of theother three ?(a) 36 (b) 35(c) 25 (d) 24

125. The H.C.F. and L.C.M. of two numbers are 12 and336 respectively. If one of the numbers is 84, theother is(a) 36 (b) 48(c) 72 (d) 96

126. The sum of two numbers is 36 and their H.C.Fand L.C.M. are 3 and 105 respectively. The sumof the reciprocals of two numbers is

(a)235 (b)

325

(c)435 (d)

225

127. If 'n' be any natural number, then by which largestnumber (n3 – n) is always divisible ?(a) 3 (b) 6(c) 12 (d) 18

128. If 1.5a = 0.04 b then b ab a-+

is equal to

(a)7377

(b)7733

(c)2

75 (d)752

129. How many perfect squares lie between 120 and300 ?(a) 5 (b) 6(c) 7 (d) 8

130.2 2(0.1) (0.01) 10.0001

ì ü-ï ï+í ýï ïî þ

is equal to

(a) 1010 (b) 110(c) 101 (d) 100

131. If there is a profit of 20% on the cost price of anarticle, the percentage of profit calculated on itsselling price will be

(a) 24 (b)2163

(c)183 (d) 20

132. If the cost price of 15 books is equal to the sellingprice of 20 books, the loss percent is(a) 16 (b) 20(c) 24 (d) 25

133. If an article is sold at 200% profit, then the ratioof its cost price to its selling price will be(a) 1 : 2 (b) 2 : 1(c) 1 : 3 (d) 3 : 1

134. If on a marked price, the difference of sellingprices with a discount of 30% and two successivediscounts of 20% and 10% is ` 72, then themarked price (in rupees) is(a) 3,600 (b) 3,000(c) 2,500 (d) 2,400

135. If an electricity bill is paid before due date, onegets a reduction of 4% on the amount of the bill.By paying the bill before due date a person got areduction of ` 13. The amount of his electricitybill was(a) ` 125 (b) ` 225(c) ` 325 (d) ` 425

136. Successive discounts of 10%, 20% and 30% isequivalent to a single discount of(a) 60% (b) 49.6%(c) 40.5% (d) 36%

137. The price of an article was first increased by 10%and then again by 20%. If the last increased pricebe ̀ 33, the original price was(a) ` 30 (b) ` 27.50(c) ` 26.50 (d) ` 25

138. If each side of a square is increased by 10%, itsarea will be increased by(a) 10% (b) 21%(c) 44% (d) 100%

yoursmahboob.w

ordpress.com

31SSC CGL SolvedPaper

139. The ratio of milk and water in mixtures of fourcontainers are 5 : 3, 2 : 1, 3 : 2 and 7 : 4 respectively,in which container is the quantity of milk, relativeto water, minimum ?(a) First (b) Second(c) Third (d) Fourth

140. Two numbers are in the ratio 1 : 3. If their sum is240, then their difference is(a) 120 (b) 108(c) 100 (d) 96

141. The ratio of income and expenditure of a personis 11 : 10. If he saves ` 9,000 per annum, hismonthly income is(a) ` 8,000 (b) ` 8,800(c) ` 8,500 (d) ` 8,250

142. If W1 : W2 = 2 : 3 and W1 : W3 = 1 : 2 then W2 :W3 is(a) 3 : 4 (b) 4 : 3(c) 2 : 3 (d) 4 : 5

143. A copper wire of length 36 m and diameter 2 mmis melted to form a sphere. The radius of thesphere (in cm) is(a) 2.5 (b) 3(c) 3.5 (d) 4

144. The ratio of the radil of two wheels is 3 : 4. Theratio of their circumferences is(a) 4 : 3 (b) 3 : 4(c) 2 : 3 (d) 3 : 2

145. If the length of a rectangle is increased by 10%and its breadth is decreased by 10%, the changein its area will be(a) 1% increase (b) 1% decrease(c) 10% increase (d) No change

146. In how many years will a sum of money double

itself at 16 %4

simple interest per annum ?

(a) 24 (b) 20(c) 16 (d) 12

147. A sum of ̀ 12,000, deposited at compound interestbecomes double after 5 years. How much will itbe after 20 years ?(a) ` 1,44,000 (b) ` 1,20,000(c) ` 1,50,000 (d) ` 1,92,000

Directions (148-150) : The pie chart, given here,shows the amount of money spent on various sportsby a school administration in a particular year.

Observe the pie chart and answer the questions basedon this graph.

Hockey100°

Tennis35°

Basket ball20°

Cricket160°

Football45°

148. If the money spent on football was ` 9,000 howmuch more money was spent on hockey than onfootball ?(a) ` 11,000 (b) ` 11,500(c) ` 12,000 (d) ` 12,500

149. If the money spent on football was ` 9,000, whatamount was spent on Cricket ?(a) ` 31,000 (b) ` 31,500(c) ` 32,000 (d) ` 32,500

150. If the money spent on football is ` 9,000, thenwhat was the total amount spent on all sports ?(a) ` 73,000 (b) ` 72,800(c) ` 72,500 (d) ` 72,000

PART-D : ENGLISH COMPREHENSION

Directions (151–155) : In the following questionssentences are given with blanks to be filled in withan appropriate word(s). Four alternatives aresuggested for each question. Choose the correctalternative out of the four.

151. The court ________ cognizance of the criminal'swords.(a) took (b) made(c) gave (d) allowed

152. ________ wins this civil war there will be littlerejoicing at the victory.(a) Whichever (b) Whoever(c) Whatever (d) Wherever

153. As he got older his belief in these principles didnot ________.(a) wither (b) shake(c) waver (d) dither

154. Everyone in this world is accountable to God________ his actions.(a) about (b) for(c) to (d) over

155. Your father used to be the principal of thiscollege______?(a) did he ? (b) does he ?(c) didn't he ? (d) doesn't he ?

yoursmahboob.w

ordpress.com

32 SSC CGL SolvedPaper

Directions (156–160) : In the following questions,choose the word opposite in meaning to the givenword.

156. JETTISON(a) accept (b) reward(c) preserve (d) consent

157. AMELIORATE(a) improve (b) depend(c) soften (d) worsen

158. GROTESQUE(a) natural (b) odd(c) whimsical (d) sinful

159. DEVIOUS(a) straight (b) obvious(c) simple (d) superficial

160. EVANESCENT(a) imminent (b) permanent(c) pervasive (d) immanent

Directions (161–165) : In the following questions,out of the four alternatives. Choose the one whichbest expresses the meaning of the given word.

161. DEBACLE(a) decline (b) downfall(c) discomfiture (d) degeneration

162. OSTRACISE(a) banish (b) belittle(c) beguile (d) besiege

163. PROPHYLACTIC(a) antagonistic (b) toxic(c) preventive (d) purgative

164. CODDLE(a) huddle (b) satisfy(c) protect (d) cheat

165. FLIMSY(a) funny (b) irrational(c) weak (d) partisan

Directions (166–170) : In the following questions, apart of the sentence is printed in bold. Below aregiven alternatives to the bold part at (a), (b) and (c)which may improve the sentence. Choose the correctalternative. In case no improvement is needed, youranswer is (d).

166. To get into the building I'll disguise as a reproter.(a) disguise to be(b) disguise as one(c) disguise myself(d) No improvement

167. He denied that he had not forged my signature(a) would not forge (b) had forged(c) did not forge (d) No improvement

168. If I had played well, I would have won the match.(a) I played well (b) I play well(c) I am playing well (d) No improvement

169. Since the records are missing the possibility ofpaying more than one compensation for the samepiece of land cannot be ruled aside.(a) out (b) off(c) away (d) No improvement

170. A callous system generates nothing but amisanthrope.(a) develops (b) induces(c) produces (d) No improvement

Directions (171–175) : In the following questions,out of the four alternatives, choose the one whichcan be substituted for the given words/ sentence.171. One who hides away on a ship to obtain a free

passage(a) Compositor (b) Stoker(c) Stowaway (d) Shipwright

172. Clues available at a scene(a) circumstantial (b) derivative(c) inferential (d) suggestive

173. An unexpected piece of good fortune(a) windfall (b) philanthropy(c) benevolence (d) turnstile

174. An emolument over and above fixed income orsalary(a) Honorarium (b) Sinecure(c) Perquisite (d) Prerogative

175. The animals of a particular region.(a) Flora (b) Museum(c) Zoo (d) Fauna

Directions (176–180) : In the following questions,groups of four words are given. In each group, oneword is correctly spelt. Find the correctly spelt word.176. (a) parapharnelia (b) parsimonious

(c) peccadilo (d) peadiatrics177. (a) measureable (b) manageable

(c) marriagable (d) manoevrable178. (a) tussel (b) tunnle

(c) tumble (d) trable179. (a) populus (b) pompuous

(c) prelious (d) presumptuous180. (a) impromptue (b) illustrious

(c) illusery (d) impetous

Directions (181–185) : In the following questions,the 1st and the last sentences of the passage arenumbered 1 and 6. The rest of the passage is split intofour parts and named P, Q, R and S. These four partsare not given in their proper order. Read the sentenceand find out which of the four combinations is correct.Then find the correct answer.

yoursmahboob.w

ordpress.com

33SSC CGL SolvedPaper

181. 1. The most vulnerable section of the societyare the students.

P. Revolutionary and new fledged ideas havea great appeal to them.

Q. Agitations may be non-violent methods ofprotest.

R. They cannot resist the charm of persuasion.S. They are to be taught that without discipline

they cannot get proper education.6. However if these become violent, the

antisocial elements get encouraged and theyput all proper working out of gear.

(a) PRSQ (b) RSQP(c) SRPQ (d) RPQS

182. 1. Venice is a strange city.P. There are about 400 odd bridges connecting

the islands of Venice.Q. There are no motor cars, no horses and no

buses there.R. These small islands are close to one another.S. It is not one island but a hundred islands.6. This is because Venice has no streets.(a) SRPQ (b) PSRQ(c) RQPS (d) QSRP

183. 1. One of the most terrible battles of theAmerican Civil War was fought in July 1863.At Gettysburg.

P. The chief speech on that occasion was givenby Edward Everett, a celebrated orator.

Q. Lincoln was asked to make a few remarks.R. In November of that year a portion of the

battlefield was dedicated as a final restingplace for men of both armies who died there.

S. Everett's speech lasted 2 hours. Lincoln'sfor 2 minutes; it was over almost before thecrowd realized that it had begun.

6. But the Gettysburg speech is now one ofthe world's immortal pieces of literature.

(a) SQRP (b) RPQS(c) PQRS (d) QPSR

184. 1. The teacher training agency in Englandhopes to make teaching one of the top threeprofessions.

P. They have also demanded that the campaignshould be matched by improved pay scales,work load and morale so as to avoidrecruitment problems with an aim to raisethe image of the teaching profession.

Q. A series of advertisements are now beingscreened showing famous people speakingabout teachers they remember and admire.

R. An amount of $100 million has been set asideto combat the shortage of applicants forteacher training.

S. Teacher Unions have welcomed thiscampaign.

6. It is high time for the Indian Governmentalso to think on similar lines and take stepsto lift up the sinking morale of the teachingprofession.

(a) QRPS (b) RPSQ(c) RQSP (d) QPSR

185. 1. Some say that failure is like toxic waste.P. I see failure more as a fertilizer.Q. Thinking about it pollutes and undermines

the attitudes needed for success.R. The seeds of success must be planted afresh.S. It can be used to enrich the soil of your mind.6. Turning failure into a fertilizer is

accomplished by using your errors as stepsin learning.

(a) SRQP (b) PQSR(c) SPQR (d) QPSR

Directions (186–190): In the following questions, asentence has been given in Active Voice/ PassiveVoice, Out of the four alternatives suggested, selectthe one which best expresses the same sentence inPassive/ Active Voice.

186. The agent had disclosed the secret before it wasevening.(a) The secret was disclosed by the agent before

it was evening(b) The secret had disclosed by the agent before

it had been evening(c) The secret had been disclosed by the agent

before it was evening(d) The secret was disclosed by the agent before

it had been evening.187. Surely the lost child must have been found by

now.(a) Surely must have found the lost child by now.(b) Surely someone must have found the lost

child by now(c) Surely now must have found the lost child(d) Now must have found the lost child surely.

188. We serve hot meals till 10.30 guests can order,coffee and sandwiches upto 11.30.(a) Hot meals are serving till 10.30 : coffee and

sandwiches are ordering by guests till 11.30.(b) Hot meals are being served till 10.30 : coffee

and sandwiches are being ordered till 11.30.(c) Hot meals are served till 10.30, coffee and

sandwiches may be ordered till 11.30.(d) Hot meals will be served till 10.30, coffee

and sandwiches will be ordered upto 11.30.

yoursmahboob.w

ordpress.com

34 SSC CGL SolvedPaper

189. Lie face-down; stretch your arms in front.(a) You are face down, arms are to be

outstretched(b) You should be lying face down, with arms

outstretched.(c) You should be lying face down; let arms

stretch out.(d) Let face be down; let arms be stretched out.

190. The Greeks expected to win the internationaltrophy.(a) It was expected that the Greeks would win

the international trophy.(b) The international trophy was expected to

be won by the Greeks.(c) It was expected that the Greeks will win the

international trophy.(d) It was expected by the Greeks that they

would win the international trophy.

Directions (191–195) : In the following questions,you have one brief passage with 5 questions followingthe passage. Read the passage carefully and choosethe best answer to each question out of the fouralternatives.

In May 1966, The World Health Organisation wasauthorised to initiate a global campaign to eradicatesmall pox. The goal was to eradicate the disease in onedecade. Because similar projects for malaria and yellowfever had failed, few believed that smallpox couldactually be eradicated, but eleven years after the initialorganisation of the campaign, no cases were reportedin the field.

The strategy was not only to provide massvaccinations, but also to isoate patients with activesmall-pox in order to contain the spread of the diseaseand to break the chain of human transmission. Rewardsfor reporting small-pox assisted in motivating thepublic to aid health workers. One by one, each small-pox victim was sought out, removed from contact withothers and treated. At the same time, the entire, villagewhere the victim had lived was vaccinated.

Today small pox is no longer a threat to humanity.Routine vaccinations have been stopped worldwide.191. Which of the following is the best title for the

passage ?(a) The World Health Organisation(b) The Eradication of Small-pox(c) Small-pox Vaccinations(d) Infectious Diseases

192. What was the goal of the campaign againstsmall-pox?(a) To decrease the spread of small-pox

worldwide.(b) To eliminate small-pox worldwide in ten

years.(c) To provide mass vaccinations against

small-pox worldwide.(d) To initiate worldwide projects for small-pox,

malaria and yellow fever at the same time.193. According to the paragraph what was the

strategy used to eliminate the spread ofsmall-pox?(a) Vaccination of the entire village(b) Treatment of individual victims.(c) Isolation of victims and mass vaccinations(d) Extensive reporting of out breaks

194. Which statement doesn't refer to small-pox?(a) Previous projects had failed.(b) People are no longer vaccinated for it.(c) The World Health Organisation mounted a

worldwide campaign to eradicate thedisease.

(d) It was a serious threat.195. It can be inferred that

(a) no new cases of small-pox have beenreported this year.

(b) malaria and yellow fever have beeneliminated.

(c) small-pox victims no longer die when theycontract the disease

(d) small-pox is not transmitted from one personto another.

Directions (196-200) : Some of the sentences haveerrors and some have none. Find out which part of asentence has an error. The number of that part is youranswer. If there is no error. your answer is (d).

196. Judge in him (a)/ prevailed upon the father (b)/and he sentenced his son to death. (c)/ Noerror (d).

197. Nine tenths (a)/ of the pillar (b)/ have rotted away.(c)/ No error (d).

198. One major reason (a)/ for the popularity oftelevision is (b)/ that most people like to stay athome. (c)/ No error (d).

199. Out efforts are (a)/ aimed to bring about (b)/ areconciliation. (c)/ No error (d).

200. Three conditions (a)/ critical for growing (b)/plants are soil, temperature, chemical balance oramount of moisture (c)/ No error (d).

yoursmahboob.w

ordpress.com

35SSC CGL SolvedPaper

HINTS & SOLUTIONS

1. (a) The direction diagram is as follows :

Babu

Rahim

200 m

200 m 200 m

200 m

Joseph

Gopal

Roy

NW

SW SE

NENorth

South

West East

The house of Roy is in South–Eastdirection with respect to the house ofBabu.

2. (a) Shalini Mahima

Chandra

Girdhar

SavitriRama

Sushma

Ravi

Ravi is to the right of Shalini.3. (d) According to the question,

× D

The circle as at the bottom.

4. (c) Here, animal-behaviour relationship hasbeen shown. Fox is characterised by itscunningness, Similarly, rabbit is consideredas timid.

5. (a) Flexible is antonym of Rigid. Similarly.Confidence is antonym. of Diffidence.

6. (c) As,

A Z C X B Y D W+1

–1

–1+1

Similarly,

H Q J O I P K N+1–1

–1+1

7. (b) As,

Q I O K M M K O–4

+4

+4–4

Similarly,

Y A W C U E S G–4

+4

+4–4

8. (a) As,

ABC : BCD : : CDE : DEF F I L O

+3 +3

yoursmahboob.w

ordpress.com

36 SSC CGL SolvedPaper

9. (d) As, (1)3 = 1; (2)3 = 8(3)3 = 27; (4)3 = 64

10. (d) 52 – 1 = 24; 53 + 1 = 126

72 – 1 = 48; 73 + 1 = 344

11. (b) As, 9 8 7

I H G¯ ¯ ¯

Similarly,

6 5 4

F E D¯ ¯ ¯

12. (c) Except Veranda, all others are surroundedby four walls.

13. (d) Except Sonata, all others are instruments.Sonata is a piece of music composed forone instrument or two.

14. (c) Except D, all others are Vowels.15. (c) Here,

4 1 3

4 1 3

2 1 2

4 1 3

R N M P

J F E H

R P O Q

H D C F

- - +

- - +

- - +

- - +

¾¾® ¾¾® ¾¾®

¾¾® ¾¾® ¾¾®

¾¾® ¾¾® ¾¾®

¾¾® ¾¾® ¾¾®

16. (c) Except in A p q r L, in all others the first andthe last letters given in Capital are Vowels.

17. (d) 6 + 2 = 8; 4 + 3 = 7 Þ 8 – 7 = 12 + 6 = 8; 4 + 3 = 7 Þ 8 – 7 = 18 + 4 = 12; 6 + 5 = 11 Þ 12 – 11 = 14 + 8 = 12; 6 + 7 = 13 Þ 12 – 13 = –1

18. (d) 49 – 33 = 16; 62 – 46 = 16; 83 – 67 = 16But, 70 – 55 = 15

19. (b) 6 + 8 = 14; 14 + (8 × 2) = 307 + 5 = 12; 12 + (5 × 2) = 22

20. (b) Meaningful order of the words3. Member ® 1. Family ® 2. Community® 4. Locality ® 4. Country

21. (c) Arrangement of words according toDictionary :3. Toranto ® 3. Torped ® 5. Torsel ® 1.Tortoise ® 4. Torus

22. (a) a a b a/ aaba/ a a ba / a ab a

23. (b) There are two alternating series :2 2 2 2a c e g i+ + + +¾¾® ¾¾® ¾¾® ¾¾®

1 1 1r s t u+ + +¾¾® ¾¾® ¾¾®Therefore, ? = ui

24. (b) The pattern is as follows :3 3 3 3P S V Y B+ + + +¾¾® ¾¾® ¾¾® ¾¾®3 3 3 3E H K N Q+ + + +¾¾® ¾¾® ¾¾® ¾¾®3 3 3 3T W Z C F+ + + +¾¾® ¾¾® ¾¾® ¾¾®

3 3 3 3I L O R U+ + + +¾¾® ¾¾® ¾¾® ¾¾®

Now, 4 4P T,E I+ +¾¾® ¾¾®Therefore, the first letter of the first termshould be

4E A-¾¾®

3 3 3 3A D G J M+ + + +¾¾® ¾¾® ¾¾® ¾¾®25. (d) The pattern is as follows :

0 4 18 48 100 180

+4 +14 +30 +52 +80

+10 +16 +22 +28

+6 +6 +626. (c) The pattern is as follows :

36 28 24 22 21

–8 –4 –2 –127. (c) The pattern is as follows :

7 9 13 21 37 69

+2 +4 +8 +16 +32

×2 ×2 ×2 ×228. (d) 232 + 111 = 343

343 + 111 = 454

454 + 111 = 565565 + 111 = 676The number 564 does not belong to theseries.

yoursmahboob.w

ordpress.com

37SSC CGL SolvedPaper

29. (b) Suppose there were x passengers initiallyNumber of passengers after first stop

= x

352

+

Number of passengers after second stop

= 4 x

355 2æ ö+ç ÷è ø + 40 = 80

Þ x (80 40)

35 52 4

-+ ´

Þ x

50 352= - = 15

\ x = 3030. (b) The day after tomorrow is Sunday.

Therefore, today is Friday.The day on tomorrow’s day beforeyesterday = Friday – 1 = Thursday

31. (d) Suppose the present age of son is x years.Therefore, present age of the father= 4x yearsAccording to question,x + 3 = 15\ x = 15 – 3 = 12 yearsThe present age of father= 4x = 4 × 12 = 48 years\ The present age of man’s wife= 48 – 3 = 45 years

32. (b) R is father of X and Y.S is maternal uncle of X and Y Consideringthe given options, it may be assumed thatT is wife of R.

33. (a) S A24th 17th

34. (b) There is no ‘U’ letter in the given word.35. (c) S H A R K

5 3 2 1 4

¯ ¯ ¯ ¯ ¯

36. (d) Neither (a) nor (b) is implicit in the statement.The statement does not indicate thatconfidence and economic development arerelated.

37. (*)

cowChair

Tables

Table

Conc I: TrueConc II: FalseConc III: FalseConc IV: False

or

38. (b) As, H O N E S T Y

5 1 3 2 4 6 8¯ ¯ ¯ ¯ ¯ ¯ ¯

and, P O V E R T Y

7 1 9 2 0 6 8¯ ¯ ¯ ¯ ¯ ¯ ¯

Therefore,

H O R S E

5 1 0 4 2¯ ¯ ¯ ¯ ¯

39. (a) As,

S I S T E R R H R S D Q–1

–1–1–1–1–1

Similarly,

U N C L E T M B K D–1

–1–1–1–1

40. (d) As, 84

= 2; 2 + 1 = 3

63

= 2 ; 2 + 3 = 5

42

= 2 ; 2 + 7 = 941. (a) As, A = 51 × 14 = 714

B = 61 × 15 = 915C = 71 × 16 = 1136\ D = 81 × 17 = 1377

yoursmahboob.w

ordpress.com

38 SSC CGL SolvedPaper

42. (b) 5 = 15 ̧ 343. (a) 25 × 2 – 6 = 4 × 11 + 0

Þ 50 – 6 = 44 + 0 , Þ 44 = 4444. (d) 5 + 4 = 9 and 9 × 2 = 18

6 + 3 = 9 and 9 × 3 = 2712 + 4 = 16 and ?

= 96 616

=

45. (d)

3 km

3 km

C

B

DE

A8 km

4 km12 km

AE = 2 2 2 2(AD) (DE) (3) (4)+ = +

= 9 16 25+ = = 5 km46. (d) Some blue eyed may be females and

vice-versa.Some females may be doctors andvice-versa.Some blue eyed. may be doctors andvice-versa.Some blue eyed females may be doctors.

Blue eyed Females

Doctors

47. (a)

48. (a) In water image upside becomes downside.

49. (c) A B

FG H I

C

DEThe triangles are:DFEB ; DCBD ; DFAG ; DFEG :DBCI ; DCDI ; DAFI ; DEFI ;DBGC; DDCG; DAGI ; DBIH :DAGB; DABI; DHAB; DHBI:DHGI; DHAG; DGEI; DGED:DIDE; DIDG; DJGI; DJDI:DJGE; DJDE; DAIE; DBGD:Thus, there are 28 triangles.

50. (a)

51. (b) 52. (a)53. (b) Economic Advisory Council to the Prime

Minister (PMEAC) is a non–constitutional,non–permanent and independent bodyconstituted to give economic advice to theGovernment of India, specifically the PrimeMinister. The council serves to highlightkey economic issues facing the country tothe government of India from a neutralviewpoint.

54. (d) 55. (b) 56. (c)57. (c) A bilateral trade is the exchange of goods

between two countries that facilitates tradeand investment by reducing or eliminatingtariffs, import quotas, export restraints andother trade barriers.

58. (b) Terrace cultivation, method of growingcrops on sides of hills or mountains byplanting on graduated terraces built intothe slope. Though labour–intensive, themethod has been employed effectively tomaximize arable land area in variable terrainsand to reduce soil erosion and water loss.

59. (b) 60. (a)61. (b) Cartography, the art and science of

graphically representing a geographical:area, usually on a flat surface such as amap or chart. It may involve thesuperimposition of political, cultural, orother non geographical divisions onto therepresentation of a geographical area.

yoursmahboob.w

ordpress.com

39SSC CGL SolvedPaper

62. (b)63. (b) Upagupta (c. 3rd Century BC) was a

Buddhist monk. According to some storiesin the Sanskrit text Ashokavadana, he wasthe spiritual teacher of the Mauryanemperor Ashoka.

64. (c) 65. (c) 66. (b)67. (c) The doctrine of lapse was an annexation

policy purportedly devised by LordDalhousie, who was the Governor Generalfor the East lndia Company in lndiabetween 1848 and 1856.

68. (b) 69. (b) 70. (a) 71. (a)72. (d) 73. (d) 74. (c) 75. (d)76. (d) Ragnar Anton Kittil Frisch was a

Norwegian economist and the co-recipientof the first Nobel Memorial Prize inEconomic Sciences in 1969 (with JanTinbergen). He is known for havingfounded the discipline of econometrics,and for coining the widely used term pairmacroeconomics /microeconomics in 1933.

77. (a) 78. (b)79. (a) Normative economics (as opposed to

positive economics) is a part of economicsthat expresses value or normativejudgments about economic fairness orwhat the outcome of the economy or goalsof public policy ought to be.

80. (c) 81. (c)82. (d) An antiknock agent is a gasoline additive

used to reduce engine knocking andincrease the fuel's octane rating by raisingthe temperature and pressure at which autoignition occurs.The typical antiknock agents in use are:Tetraelhyllead (Still in use as a high octane

83. (b) 84. (a) 85. (a)86. (d) A positron is a particle of matter with the

same mass as an electron but an oppositecharge. It is a form of antimatter because,when a positron encounters an electron,the two completely annihilate to yieldenergy.

87. (a) 88. (a)89. (b) Acronym for million instructions per

second. A old measure of acomputer'sspeed and power, MIPS measures roughlythe number of machine instructions that acomputer can execute in one second.

90. (b) 91. (c) 92. (d)93. (d) 94. (d) 95. (b)

96. (c) Joseph Aspdin was an English cementmanufacturer who obtained the patent forPortland cement on 21 October 1824.

97. (a) 98. (d)99. (b) The Agreement on Trade–Related Aspects

of Intellectual Property Rights (TRIPS) isan international agreement administeredby the World Trade Organization (WTO)that sets down minimum standards formany forms of intellectual property (IP)regulation as applied to nationals of otherWTO Members.The Agreement on Trade–RelatedInvestment Measures (TRIMS) recognizesthat certain investment measures canrestrict and distort trade. It states that WTOmembers may not apply any measure thatdiscriminates against foreign products orthat leads to quantitative restrictions, bothof which violate basic WTO principles.

100. (b)101. (a) Rate = 10% per annum = 5% half yearly

A = P TR1

100æ ö+ç ÷è ø

Þ 926.10 = 800 T51

100æ ö+ç ÷è ø

Þ T9261 21

8000 20æ ö=ç ÷è ø

Þ 3 T21 21

20 20æ ö æ ö=ç ÷ ç ÷è ø è ø

\ Time = 3 half years = 112

years

102. (c) Time taken by Kamal

100 2051818

= =´

\ Time taken by Bimal= 20 + 5 = 25

\ Bimal’s speed = 10025

= 4 m

= 4 18

kmph = 14.4 kmph.

yoursmahboob.w

ordpress.com

40 SSC CGL SolvedPaper

103. (c) Let train speed be xrelative speed = (x + 3) kmph

\ Time = Length of the train

Relative speed

Þ

24010 2401000

3600 (x 3) 1000(x 3)= =

+ +Þ x + 3 = 86.4Þ x = 83.4 kmph

104. (a) Speed of current

= 12

(Rate downstream – Rate upstream)

= 12

(12 – 6) kmph [Rate downstream]

= 1

605´ = 12 kmph = 3 kmph

105. (b) Total time taken by A = 15 days

Total time taken by B = 10 5

= 25 days

\ (A + B)’s 1 day’s work

= 1 1 5 3 8

15 25 75 75++ = =

\ the work will be completed in 75 398 8

=

days.106. (c) M1D1W2 = M2D2W1

Þ 7 × 12 × 2 = M2 × 8 × 1

Þ M2 = 7 12 2

8´ ´

= 21

\ No. of additional men = 21 – 7 = 14107. (d) If time taken by the pipe at faster rate to fill

the tank be x minutes then

1 1 1 3 1 1x 3x 36 3x 36

++ = Þ =

Þ 3x = 4 × 36Þ x = 48 minutes\ Time taken by the slower pipe= 48 × 3 = 144 minutes= 2 hours 24 minutes

108. (b) If the number of correct answers be x, thenx × 4 – 1. (200 – x) = 200Þ 4x – 200 + x = 200Þ 5x = 400

Þ x = 400

5 = 80

109. (b) Average of the first n natural odd numbers= nNumber of odd numbers upto 100 = 50= required average.No. of odd no upto 100 = 50Sum of 50 odd no = 502

Average = 50 50

50´

= 50.

110. (c) Required percentage

= 25 100 1

100 33 %100 25 3 3

´ = =-

111. (b) 12 + 22 + 32 + .... + n2

= n(n 1)((2n 1)

6+ +

\ 12 + 22 + 32 + .... + 102

= 10(10 1)(20 1)

6+ +

= 385

112. (c) 2 + 4 = 66 + 5 = 1111 + 6 = 1717 + 7 = 24

24 + 8 = 32113. (b) Let the numbers be 7x and 11x respectively.

\ 7x 7 2

11x 7 3+ =+

\ 22x + 14 = 21x + 21Þ x = 7\ Smaller number= 7x = 7 × 7 = 49

114. (a)1 1 1

1 1 1 ....3 4 5

æ ö æ ö æ ö- - -ç ÷ ç ÷ ç ÷è ø è ø è ø

1 11 1

24 25æ ö æ ö- -ç ÷ ç ÷è ø è ø

= 2 3 4 23 24 2

.....3 4 5 24 25 25´ ´ ´ ´ =

115. (d) If the first divisor be a multiple of thesecond divisor, then required remainder =remainder obtained by dividing the firstremainder (36) by the second divisor (17)= 2Q 17 is a factor of 136\ Remainder when 36 is divided by 17 = 2

yoursmahboob.w

ordpress.com

41SSC CGL SolvedPaper

116. (c) ( )55

5 3 / 5 3x-

-é ùê úê úê úë û

=

1 5 53 5 35x

-´ ´-æ ö

ç ÷ç ÷è ø

= 3 55 3x-

- ´ = x

117. (d) xyxy = xy × 100 + xy= xy (100 + 1) = 101 × xyHence, the number is exactly divisible by101.

118. (d) 0.1 × 0.01 × 0.001 × 107 = 10–6 × 107 = 10

119. (b) 2p + 1p = 4

Þ p + 12p = 2

\ 31p

2pæ ö

+ç ÷è ø

= 3

31 1 1

p 3.p. p2p 2p8pæ ö

+ + +ç ÷è ø

Þ 8 = p3 + 31 3 2

28p+ ´

Þ 3

31p

8p+ = 8 – 3 = 5

120. (c) 5 P 93 2 72 q 8

1 1 1 4

If p = 0, then q’s maximum value = 7

121. (b)1516

= 0.94; 1920

= 0.95

2425

= 0.96; 3435

= 0.97

122. (c) 1.27 = 27 3 14

1 199 11 11

= =

123. (d)3.20(3.25 3.05)

0.064-

= 3.20 0.20

0.064´

= 10

124. (a) 8 + 9 + 10 = 2711 + 12 + 13 = 36So, let 3 consecutive no x, x + 1, x + 2Next 3 consecutive no x + 3; x + 4, x + 5i.e. sum of last 3 consecutive no. is 9 morethan sum of first 3.= 27 + 9 = 36

125. (b) First number × second number= HCF × LCMÞ 84 × second number = 12 × 336\ Second number

= 12 336

84´

= 48

p × q = HCF × LCM

q = 12 336

84´

= 48

126. (c) Let the numbers be 3x and 3y.\ 3x + 3y = 36Þ x + y = 12 ...(i)and 3xy = 105 ...(ii)Dividing equation (i) by (ii), we have

x y 123xy 3xy 105

+ =

Þ 1 1 4

3y 3x 35+ =

127. (b) n3 – n = (n2 – 1)Þ n (n +1) (n – 1)For n = 2, n3 – n = 623 – 2 = 6i.e. n3 – n is always divisible by 6.

128. (a) 1.5a = 0.04 b

b 1.5a 0.04

=

By componendo and dividendo,b a 1.5 0.04 1.45 73b a 1.5 0.04 1.54 77- -

= = =+ +

129. (c) 112 = 121, 122 = 144, 132 = 169, 142 = 196152 = 225, 162 = 256, 172 = 289Square no above 120 = 121 of 11Square no below 300 = 289 of 17Total 11, 12, 13, 14, 15, 16, 17, i.e. 7 no.

yoursmahboob.w

ordpress.com

42 SSC CGL SolvedPaper

130. (d)0.01 0.0001 0.0099

1 10.0001 0.0001-

+ = +

= 99 + 1 = 100131. (b) If the CP = ̀ 100, then SP = ̀ 120 and gain =

` 20

Gain % = 20 100

120´

= 50 216 %3 3

=

132. (d) If the CP of each book be `1, thenSP of 20 books = `15CP of 20 books = `20

\ L% = 20 15

10020-´ = 25%

133. (c) Let CP = 100P = 200SP = CP + P = 300CPSP

= 100300

= 13

134. (a) Let the marked price be ̀ x.

\ In case I, SP = ` 70x100

Single discount equivalent to successivediscounts of 20% and 10%.

= 20 10

20 10 %100´æ ö+ -ç ÷è ø = 28%

\ S.P. in this case = ` 75x100

\ 72x 70x100 100- = ̀ 72

Þ 2x

100 = 72

\ x = 72 100

= ̀ 3600

135. (c) Let the amount of the bill be ̀ x.

\ 4x

100 = 13

Þ x = 1300

4 = ̀ 325

136. (b) Single equivalent discount for successivediscounts of 10% and 20%.

= 20 100

10 20 %100´æ ö+ -ç ÷è ø = 28%

Single equivalent discount for 28% and30%

= 28 30

28 30 %100´æ ö+ -ç ÷è ø = 49.6%

137. (d) Net increase percentage

= 20 10

10 20 %100´æ ö+ +ç ÷è ø = 32%

\ x × 132100

= 33

Þ x = 33 100

132´

= ̀ 25

138. (b) Increase percent in area

= 10 10

10 10 %100´æ ö+ +ç ÷è ø = 21%

139. (c) Milk in V1 = 58

= 0.625

Milk in V2 = 23

= 0.66

Milk in V3 = 35

= 0.6

Milk in V4 = 7

11 = 0.636

140. (a) Let the numbers be 3x and x.3x + x = 240Þ 4x = 240

Þ x = 240

4 = 60

\ Difference = 3x – x = 2x= 2 × 60 = 120

141. (d) Let the income of man be Rs. = 11x and hisexpenditure be ̀ 10x.\ Savings x = ` 9000

\ Monthly income of man = 11 9000

12´

=

` 8250let income = 11xexpenditure = 10xSaving = 11x – 10x = xx = 9000

Monthly income = 11x 11 900012 12

´=

= 8250

yoursmahboob.w

ordpress.com

43SSC CGL SolvedPaper

142. (a)1

2

W 2W 3

=

Þ 2

1

W 3W 2

= and 1

3

W 1W 2

=

\ 2 1 2

1 3 3

W W W 3 1 3W W W 2 2 4´ = = ´ =

143. (b) Volume of the wire = pr2h\ p × 0.1 × 0.1 × 3600 cm3

Þ 36p cm3

Volume cylinder = vol. sphere

Volume of the sphere = 34 R3p

= 36 p

Þ R3 = 36 3

= 27

\ R = 3 27 = 3 cm144. (b) Ratio of the circumferences

= Ratio of radii = 3 : 4

1

2

R 3R 4

=

1 1 1

2 2 2

C 2 r rC 2 r r

p= =p

= 34

145. (b) Required change in area210 10 x= 1% trick =

100 100- ´ -= -

Negative sign shows a decrease.Net change

10 – 10 – 10 10

100´

= –1%or 1% decreasing.

146. (c) T = SI 100

P R´´

(For double SI = P = x)

x 100= = 16 year.25x

4

´

´

147. (d) A = P TR1

100æ ö+ç ÷è ø

Þ 24000 = 12000 5R1

100æ ö+ç ÷è ø

Þ 2 = 5R1

100æ ö+ç ÷è ø

Þ 24 = 20R1

100æ ö+ç ÷è ø : = 16 times

i.e. The sum amounts to ̀ 192000.148. (a) Q 45° º ̀ 9000

\ 55° º 9000

5545´ °

° = ̀ 11000

149. (c) Q 45° º ̀ 9000

\ 160° º 9000

16045´ °

° = ̀ 32000

150. (d) Q 45° º ̀ 9000

\ 360° º 9000 360

45´ ° = ̀ 72000

151 (a) Idiom take cognizance of somethingmeans: to understand or considersomething to take notice of something.

152. (b)153. (a) The word wither means : to become less

or weaker.154. (b)155. (c) didn't he ?156. (a) The word Jettison (Verb) means : to throw

something: abandon; to reject an idea.Hence, the words jettison and accept areantonyms.

157. (d) The word Ameliorate (Verb) means : tomake something better.Hence, the words ameliorate and worsenare antonymous.

158. (a) The word Grotesque (Adjective) means :strange in a way that is unpleasant:extremely ugly, unusualHence, the words grotesque and naturalare antonyrnous.

159. (a) The word Devious (Adjective) means :behaving in a dishonest way: a route thatis not straight.Hence, the words devious and straight areantonymous.

yoursmahboob.w

ordpress.com

44 SSC CGL SolvedPaper

160. (b) The word Evanescent (Adjective) means:disappearing quickly from sight or memory.Hence, the words evanescent andpermanent and antonymous.

161. (b) The word Debacle (Noun) means : an eventor a situation that is a complete failure arecauses embarrassment.Hence, the words debacle and downfall aresynonymous.

162. (a) The word Ostracise (Verb) means : torefuse to let somebody he a member of asocial group: refuse, shun.Hence the words banish and ostracise aresynonymous.

163. (c) The word Prohylactic (Adjective) means :done or used in order to prevent a disease.Hence, the words prophylactic andpreventive are synonymous.

164. (b) The word Coddle (Verb) means: to treatsomebody with too much care andattention, pamper, cosset.Hence, the words coddle and satisfy aresynonymous.

165. (c) The word Flimsy (Ajective) means : badlymade and not strong enough: thin andeasily torn.Hence, the words flimsy and weak aresynonymous.

166. (c) 167. (b) 168. (d) 169. (a)170. (c) 171. (c) 172 (a) 173. (a)174. (a) 175. (d)176. (b) Correct spellings of other words are :

paraphernalia, peccadillo and paediatrics.177. (b) Correct spellings of other words are :

measurable : marriageable andmanauverable.

178. (c) Correct spellings of other words are :tussle, tunnel and treble.

179. (d) Correct spellings of other words are :populous, pompous and perilous.

180. (b) Correct spellings of other words are :impromptu : illusory and impetus.

181. (a) 182. (a) 183. (b)184. (c) 185. (d)186. (c) The secret had been disclosed by the

agent before it was evening.187. (b) Surely some one must have found the lost

child by now.188. (c) Hot meals are served till 10.30: coffee and

sandwiches may be ordered till 11.30.189. (d) Let face be down : let arms be stretched

out.190. (d) It was expected by the Greeks that they

would win the international trophy.191. (b) The Eradication of Small-pox192. (b) To eliminate smallpox world-wide in ten

years.193. (c) Isolation of victims and mass vaccinations194. (a) Previous projects had failed.195. (c) Small-pox victims no longer die when they

contract the disease196. (a) Sometimes Common Nouns are used as

Abstract Nouns as they express qualities.In this situation, we use 'the' before them.Hence. The Judge in him should be used.

197. (b) The structure of some sentences is :Indefinite number + of + NounIndefinite quantity + of + NounIn these sentences, the subject is one thatcomes after 'of'. Here, the word pillar issingular, hence, has rotted away should beused.

198. (c) Here, replace that most people like to stayat home by most of the people like to stayat home.

199. (b) The word aim takes preposition 'at'.Hence, at bringing about should be used.

200. (c) Here, plants are sell, temperature andchemical balance or amount of moistureshould be used.The output. Thus, the average fixed costcurve is a rectangular hyperbole.

yoursmahboob.w

ordpress.com

45SSC CGL SolvedPaper

PART-A : GENERAL INTELLIGENCE & REASONING

Directions (1-9) : In each of the following questions,select the related letters/word/numbers from the givenalternatives.

1. Fish : Scales : : Bear : ?(a) Feathers (b) Leaves(c) Fur (d) Skin

2. Writer : Pen : : ?(a) Needle :Tailor (b) Artist : Brush(c) Painter : Canvas (d) Teacher : Class

3. NUMERAL : UEALRMN : : ALGEBRA : ?(a) LRBAGEA (b) BARLAGE(c) LERAGBA (d) LERABGA

4. BDAC : FHEG : : NPMO:?(a) RQTS (b) QTRC(c) TRQS (d) RTQS

5. FGHI : OPQR : BCDE : ?(a) KLMJ (b) KLMN(c) IUVW (d) STUW

6. PNLJ : IGEC: : VTRP : ?(a) OMKI (b) RSTU(c) QSRC (d) RPOM

7. 17 : 60 : : 20 : ?(a) 57 (b) 69(c) 81 (d) 93

8. 6 : 64 : : 11 : ?(a) 144 (b) 169(c) 121 (d) 124

9. 123 : 4: : 726 : ?(a) 23 (b) 26(c) 14 (d) 12

Directions (10-15) : In each of the following questions,find the odd number / letters/ word from the givenalternatives.

10. (a) Flute (b) Violin(c) Guitar (d) Sitar

11. (a) Prod (b) Sap(c) Jab (d) Thrust

12. (a) JKOP (b) MNST(c) CABD (d) OPWX

13. (a) CAFD (b) TSWV(c) IGLJ (d) OMRP

14. (a) 162 (b) 405(c) 567 (d) 644

15. (a) 156 (b) 201(c) 273 (d) 345

16. In the following question, number of letters areskipped in between by a particular rule. Which ofthe following series observes the rule?(a) ABFGJK (b) ACFJOU(c) MPQSTV (d) ADFHJL

17. Find out the pair of numbers that does not belongto the group for lack of common property.(a) 11–115 (b) 10–90(c) 9–72 (d) 8–56

Directions : Which one of the given responses wouldbe a meaningful descending order of the following?

18. 1. Major 2. Captain3. Colonel 4. Brigadier5. Lt. General(a) 5, 4, 3, 1, 2 (b) 5, 1, 4, 2, 3(c) 4, 5, 1, 3, 2 (d) 3, 4, 2, 5, 1

Directions: Arrange the following words as per orderin the dictionary.

19. 1. Dissident 2. Dissolve3. Dissent 4. Dissolute5. Dissolution(a) 3, 1, 4, 5, 2 (b) 3, 2, 1, 4, 5(c) 3, 1, 4, 2, 5 (d) 3, 2, 4, 5, 1

20. Which one-set of letters when sequentiallyplaced at the gaps in the given letter series shallcomplete it?__cb__cab__baca__cba__ab(a) cabcb (b) abccb(c) bacbc (d) bcaba

Directions (21-24) : In each of the following questions,a series is given, with one/two terms(s) missing.Choose the correct alternative from the given onesthat will complete the series.

21. XYZCBAUVWFE __?__ __?__(a) DR (b) RS(c) DS (d) MN

SSC Combined Graduate Level (CGL) Solved PaperMORNING SHIFT 19 JUNE 2011

yoursmahboob.w

ordpress.com

46 SSC CGL SolvedPaper

22. reoc, pgme, nikg, lkii __ ? __(a) acef (b) jmgk(c) efgh (d) wxyz

23. 4, 196, 16, 169, ?, 144, 64(a) 21 (b) 81(c) 36 (d) 32

24. 8, 15, 36, 99, 288, __ ? __(a) 368 (b) 676(c) 855 (d) 908

25. Find the wrong number in the series.6, 9, 15, 22, 51, 99(a) 99 (b) 51(c) 22 (d) 15

26. In a row of girls, Kamala is 9th from the left andVeena is 16th from the right. If they interchangetheir positions, Kamla becomes 25th from the left.How many girls are there in the row?(a) 34 (b) 36(c) 40 (d) 41

27. Ravi has spent a quarter 14æ öç ÷è ø

of his life as a boy,,

one–fifth 15æ öç ÷è ø

as a youth, one-third 13æ öç ÷è ø

as man

and thirteen (13) years in old age. What is hispresent age?(a) 70 years (b) 80 years(c) 60 years (d) 65 years

28. Among her children, Ganga's favourites are Ramand Rekha. Rekha is the mother of Sharat, who isloved most by his uncle Mithun. The head of thefamily is Ram Lal, who is succeeded by his sonsGopal and Mohan. Gopal and Ganga have beenmarried for 35 years and have 3 children. What isthe relation between Mithun and Mohan?(a) Uncle (b) Son(c) Brother (d) No relation

29. Out of 100 families in the neighbourhood, 50 haveradios, 75 have TVs and 25 have VCRs. Only 10families have all three and each VCR owner alsohas a TV. If some families have radio only, howmany have only TV?(a) 30 (b) 35(c) 40 (d) 45

30. Suresh was born on 4th October 1999.Shashikanth was born 6 days before Suresh. TheIndependence Day of that year fell on Sunday.Which day was Shashikanth born?(a) Tuesday (b) Wednesday(c) Monday (d) Sunday

31. Five boys A, B, C, D. E are sitting in a park in acircle. A is facing South-West, D is facingSouth-East, B and E are right opposite A and Drespectively and C is equidistant between D andB. Which direction is C facing'?(a) West (b) South(c) North (d) East

32. At what time are the hands of clocks togetherbetween 6 and 7?

(a)83211

minutes past 6

(b)83411

minutes past 6

(c)83011

minutes past 6

(d)5327

minutes past 6

33. In a certain office, 13

of the workers are women,

12

of the women are married and 13

of the married

women have children. If 34

of the men are married

and 23

of the married men have children, then

what part of workers are without children?

(a)5

18(b)

49

(c)1118

(d)1736

34. If a man on a moped starts from a point and rides4 km South, then turns left and rides 2 km to turnagain to the right to ride 4 km more, towards whichdirection is he moving?(a) North (b) West(c) East (d) South

35. Ganesh cycles towards South–West a distanceof 8 m, then he moves towards East a distance of20 m. From there he moves towards North–East adistance of 8 m, then he moves towards West adistance of 6 m. From there he moves towardsNorth–East a distance of 2 m. Then he movestowards West a distance of 4 m and then towardsSouth– West 2 m and stops at that point. How faris be from the starting point?(a) 12 m (b) 10 m(c) 8 m (d) 6 m

yoursmahboob.w

ordpress.com

47SSC CGL SolvedPaper

36. From the given alternative words, select the wordwhich cannot be formed using the letters of thegiven word:'CONCENTRATION'(a) CONCERN (b) NATION(c) TRAIN (d) CENTRE

37. From the given alternative words, select the wordwhich can be formed using the letters of the givenword:'DETERMINATION'(a) DECLARATION (b) NATIONAL(c) TERMINATED (d) DEVIATION

38. If in a certain code HYDROGEN is written asJCJZYSSD, then how can ANTIMONY be writtenin that code?(a) CPVKOQPA (b) CRZQWABO(c) ERXMQSRC (d) G'I'ZOSU'I'E

39. If DELHI is coded as 73541 and CALCUTTA as82589662, then how can CALICUT be coded?(a) 5279431 (b) 5978013(c) 8251896 (d) 8543691

40. Two statements are given followed by fourinferences. Select the alternative which is mostappropriate.Statements:India is becoming industrialised. Pollution is aproblem associated with industrialisationInferences:I. All industrial centres are polluted.II. India is polluted.III. Polluted nations are industrialised.IV. India may become polluted.(a) All are appropriate(b) None is appropriate(c) Only IV is appropriate(d) Only II is appropriate

Directions (41–42) : In each of the followingquestions. Select the missing number from the givenresponses.

41. 2 7 97 3 49 8 ?126 168 216(a) 8 (b) 3(c) 6 (d) 36

42.?

33 17

9

(a) 60 (b) 68(c) 55 (d) 65

43. If '–' stands for '¸' '+' stands for '×' , '¸' for '–' and '×'for '+', which one of the following equations incorrect?(a) 30 – 6 + 5 × 4 ÷ 2 = 27(b) 30 + 6 – 5 ÷ 4 × 2 = 30(c) 30 × 6 ÷ 5 – 4 + 2 = 32(d) 30 ÷ 6 × 5 + 4 – 2 = 40

44. Some equations have been solved on the basisof a certain system. Find the correct answer forthe unsolved equation on that basis. If 9 * 7 = 32,13 * 7 = 120, 17 * 9 = 208, then 19 * 11 = ?(a) 150 (b) 180(c) 210 (d) 240

45. Forecast the Growth Rate for the year 1995 fromthe following data:

Years 1990 1991 1992 1993 1994 1995Growth 3.5 3.7 4.1 4.9 6.5 ?Rate

®

®

(a) 7.8 (b) 8.6(c) 9.7 (d) 9.9

46. Which of the following diagrams represents therelationship among Sun, Moon and Star?

(a) (b) (c) (d)

47. How many rectangles are there in the givendiagram?

(a) 4 (b) 7(c) 9 (d) 18

yoursmahboob.w

ordpress.com

48 SSC CGL SolvedPaper

Directions : From the given answer figures, selectthe one in which the question figure is hidden/embedded.

48. Question Figure:

Answer Figures:

(a) (b) (c) (d)

Directions : If a mirror is placed on the line MN, thenwhich of the answer figures is the correct image ofthe given question figure ?

49. Question Figure:

N

MAnswer Figures:

(a) (b) (c) (d)

Directions: A piece of paper is folded and cut as shownbelow in the question figures. From the given answerfigures, indicate how it will appear when opened.

50. Questions Figure:

Answer Figures :

(a) (b) (c) (d)

PART-B : GENERAL AWARENESS

51. When there is an official change in the exchangerate of domestic currency, then it is called:(a) Appreciation (b) Depreciation(c) Revaluation (d) Deflation

52. Inflation redistributes income and wealth infavour of:(a) Pensioners (b) Poor(c) Middle class (d) Rich

53. The fringe benefit tax was introduced in thebudget of(a) 2003-04 (b) 2004-05(c) 2005-06 (d) 2006-07

54. In the Budget estimates for 2011-12, an allocationof ̀ 400 crore has been made to bring in a greenrevolution in the East in the cropping system of:(a) Wheat (b) Rice(c) Jowar (d) Pulses

55. An announced by the Finance Minister in hisbudget speech on 28-2-2011, the income taxexemption limit for individuals other than seniorcitizens and women has been raised to:(a) `1,80,000 (b) `1,90,000(c) `2,00,000 (d) `2,50,000

56. Judicial review in the Indian Constitution is basedon:(a) Rule of Law(b) Due process of Law(c) Procedure established by Law(d) Precedents and Conventions

57. The Drafting of the Constitution was completedon:(a) 26th January, 1950(b) 26th December, 1949(c) 26th November, 1949(d) 30th November, 1949

58. Who was the President of the ConstituentAssembly?(a) Pt. Jawahar Lal Nehru(b) Sardar Patel(c) Dr. Rajendra Prasad(d) Dr. B.R. Ambedkar

59. Which innovative discussion process isintroduced by the Indian parliament to the WorldParliamentary systems?(a) Question hour(b) Zero hour(c) Resolutions(d) Presidential Speech

60. The judges of the Supreme Court retire at the ageof :(a) 60 years (b) 65 years(c) 62 years (d) 58 years

yoursmahboob.w

ordpress.com

49SSC CGL SolvedPaper

61. The Greek ambassador sent to ChandraguptaMaurya's Court was :(a) Kautilya (b) Seleucus Nicator(c) Megasthenes (d) Justin

62. Identify the European power from whom Shivajiobtained cannons and ammunition:(a) The French (b) The Portuguese(c) The Dutch (d) The English

63. The call of "Back to the Vedas" was given by:(a) Swami Vivekananda(b) Swami Dayanand Saraswati(c) Aurobindo Ghosh(d) Raja Ram Mohan Roy

64. Simon Commission was boycotted by thenationalist leaders of India because:(a) they felt that it was only an eyewash(b) all the members of the Commission were

English(c) the members of the Commission were biased

against India(d) it did not meet the demands of the Indians

65. Who among the following British personsadmitted the Revolt of 1857 as a national revolt?(a) Lord Dalhousie(b) Lord Canning(c) Lord Ellenborough(d) Disraeli

66. How much of the Earth's land surface is desert?(a) 1/10th (b) 1/5th

(c) 1/3rd (c) 1/6th

67. Which of the following is called the 'shrimp capitalof India'?(a) Mangalore (b) Nagapatnam(c) Kochi (d) Nellore

68. River Indus originates from:(a) Hindukush range (b) Himalayan range(c) Karakoram range (d) Kailash range

69. The lowest layer of the atmosphere is:(a) Stratosphere (b) Thermosphere(c) Troposphere (d) Mesosphere

70. The Konkan Railway connects:(a) Goa – Mangalore(b) Roha – Mangalore(c) Kanyakumari – Mangalore(d) Kanyakumari – Mumbai

71. Bark of this tree is used as a condiment–(a) Cinnamon (b) Clove(c) Neem (d) Palm

72. Who was the architect of North and South Blocksof the Central Secretariate in Delhi?

(a) Sir Edward Lutyens(b) Herbert Baker(c) Robert Tor Russell(d) Antonin Raymond

73. Saliva helps in the digestion of:(a) Fats (b) Starch(c) Proteins (d) Vitamins

74. The longest bone in the human body is:(a) Ulna (b) Humerus(c) Femur (d) Tibia

75. Red data book gives information about specieswhich are:(a) extinct (b) endangered(c) dangerous (d) rare

76. Which of the following is the smallest bird?(a) Pigeon (b) Parrot(c) Humming bird (d) House sparrow

77. The time period of a pendulum when taken to theMoon would:(a) remain the same (b) decrease(c) become zero (d) increase

78. The atmospheric air is held to the Earth by:(a) gravity(b) winds(c) clouds(d) rotation of the Earth

79. The function of ball hearings in a wheel is:(a) to increase friction(b) to convert kinetic friction into rolling friction(c) to convert static friction into kinetic friction(d) just for convenience

80. 'Shock–absorbers' are usually made of steel as it:(a) is not brittle(b) has lower elasticity(c) has higher elasticity(d) has no ductile property

81. The first computer made available for commercialuse was:(a) MANIAC (b) ENIAC(c) UNIVAC (d) EDSAC

82. A communication network which is used by largeorganizations over regional, national or globalarea is called:(a) LAN (b) WAN(c) MAN (d) VAN

83. Which of the following could be used as fuel inpropellant or rockets?(a) Liquid Hydrogen + Liquid Nitrogen(b) Liquid Oxygen + Liquid Argon(c) Liquid Nitrogen + Liquid Oxygen(d) Liquid Hydrogen + Liquid Oxygen

yoursmahboob.w

ordpress.com

50 SSC CGL SolvedPaper

84. The addition of gypsum to portland cement helpsin:(a) increasing the strength of cement(b) rapid setting of cement(c) preventing rapid setting of cement(d) reduction in the cost of cement

85. White lung disease is prevalent among theworkers of:(a) Paper industry (b) Cement industry(c) Cotton industry (d) Pesticide industry

86. lodoform is used as an:(a) antipyretic (b) analgesic(c) antiseptic (d) anaesthetic

87. An artificial ecosystem is represented by:(a) pisciculture tank (b) agricultural land(c) zoo (d) aquarium

88. The constituents of automobile exhaust that cancause cancer is are:(a) Oxides of nitrogen(b) Carbon monoxide(c) Polycyclic hydrocarbons(d) Lead

89. The optimum dissolved oxygen level (in mg/litre)required for survival of aquatic organisms is:(a) 4 – 6 (b) 2 – 4(c) 8 – 10 (d) 12 – 16

90. The world's only floating national park is situatedin:(a) Manipur (b) Kuala Lumpur(c) Bilaspur (d) Dispur

91. According to the latest 'Education DevelopmentIndex' based on a series of surveys conductedby the National University of Education, Planningand Administration and released in February,2011, the state with the highest DevelopmentIndex is:(a) Tamil Nadu (b) Kerala(c) Punjab (d) Gujarat

92. Indian Army's Operation 'Saiyam' was related to :1(a) Kashmir(b) Indo–China Border in the Central Region(c) North–East(d) Indo–Pak Border in Punjab and Rajasthan

93. In the Railway Budget for 2011–12, an outlay of`57,630 crore has been announced for addingnew lines with length of:(a) 1000 km (b) 1200 km(c) 1300 km (d) 1500 km

94. The strong earthquake–cum–tsunami which hasmoved Japan's main island by a few feet and hascaused the Earth's axis to wobble is expected tolead to the shortening of the day by:

(a) 1.6 microseconds(b) 2.3 microseconds(c) 3,1 microseconds(d) 3.4 microseconds

95. The 'Project Snow Leopard' to conserve theendangered species, launched by the UnionMinistry of Environment and Forests covers thestates of:(a) Jammu & Kashmir and Himachal Pradesh

only(b) Jammu & Kashmir, Himachal Pradesh and

Uttarakhand only(c) Jammu & Kashmir, Himachal Pradesh,

Uttarakhand and Arunachal Pradesh only(d) Jammu & Kashmir, Himachal Pradesh,

Uttarakhand, Arunachal Pradesh and Sikkim96. The 2010 FIFA World Cup Final was held at:

(a) Paris (b) Berlin(c) Johannesburg (d) London

97. Who of the following received– Sangeet NatakAkademi's Ustad Bismillah Khan Yuva Puruskarfor 2009 in 'Hindustani Vocal Music'?(a) Ornkar Shrikant Dadarkar(b) Ragini Chander Shekar(c) Abanti Chakravorty and Sukracharya Rabha(d) K. Nellai Manikandan

98. Which of the following folk/tribal dances isassociated with Uttar Pradesh?(a) Veedhi (b) Thora(c) Tamasha (d) Rauf

99. Which of the following books has been writtenby Atiq Rahimi?(a) Earth and Ashes (b) This Savage Rite(c) The Red Devil (d) Witness the Night

100. Who is the recepient of the Sahitya AkademiAward 2010 in Hindi Literature category?(a) Uday Prakash (b) Laxman Dubey(c) Nanjil Nandan (d) Mangat Badal

PART-C : NUMERICAL APTITUDE

101. 6 6 6 ...+ + + = ?(a) 2.3 (b) 3(c) 6 (d) 6.3

102. The square root of 3 23 2

æ ö+ç ÷-è ø is

(a) 3 2+ (b) 3 2-(c) 2 3± (d) 2 3-

yoursmahboob.w

ordpress.com

51SSC CGL SolvedPaper

103. The remainder when 321 is divided by 5 is(a) 1 (b) 2(c) 3 (d) 4

104. The value of

1 22 13 11

13 13 133

-

++

+

is

(a)38

109(b)

10938

(c) 1 (d)116109

105. The last digit of (1001)2008 + 1002 is(a) 0 (b) 3(b) 4 (d) 6

106. The value of 3 2 4 3 63 6 6 2 3 2

- ++ + +

is

(a) 4 (b) 0(c) 2 (d) 3 6

107. If x * y = (x + 3)2 (y –1), then the value of 5 * 4 is(a) 192 (b) 182(c) 2 (d) 356

108.2 2 2

2 2 2(0.05) (0.41) (0.073)

(0.005) (0.041) (0.0073)+ ++ +

is

(a) 10 (b) 100(c) 1000 (d) None of these

109. If 9 x 12 147= + , then x = ?(a) 2 (b) 3(c) 4 (d) 5

110. 3 1271343- is equal to

(a)59

(b)117-

(c)47

(d)2

17-

111. If the sum of two numbers be multiplied by eachnumber separately, the products so obtained are247 and 114. The sum of the numbers is(a) 19 (b) 20(c) 21 (d) 23

112. Find a number, one–seventh of which exceedsits eleventh part by 100.(a) 1925 (b) 1825(c) 1540 (d) 1340

113. The L.C.M. of three different numbers is 120.Which of the following cannot be their H.C.F.?(a) 8 (b) 12(c) 24 (d) 35

114. A number when divided by 49 leaves 32 asremainder. This number when divided by 7 willhave the remainder as(a) 4 (b) 3(c) 2 (d) 5

115. In an examination a student scores 4 marks forevery correct answer and loses 1 mark for everywrong answer. If he attempts all 75 questions andsecures 125 marks, the number of questions heattempts correctly is(a) 35 (b) 40(c) 42 (d) 46

116. The traffic lights at three different road crossingschange after 24 seconds, 36 seconds and 54seconds respectively. If they all changesimultaneously at 10 : 15 :00 AM, then at whattime will they again change simultaneously?(a) 10 : 16 : 54 AM (b) 10 : 18 : 36 AM(c) 10 : 17 : 02 AM (d) 10 : 22 : 12 AM

117. A can do a work in 12 days. When he had workedfor 3 days, B joined him. If they complete thework in 3 more days, in how many days can Balone finish the work?(a) 6 days (b) 12 days(c) 4 days (d) 8 days

118. A and B can complete a piece of work in 8 days, Band C can do it in 12 days, C and A can do it in 8days. A, B and C together can complete it in(a) 4 days (b) 5 days(c) 6 days (d) 7 days

119. X is 3 times as fast as Y and is able to completethe work in 40 days less than Y. Then the time inwhich they can complete the work together is(a) 15 days (b) 10 days

(c)172

days (d) 5 days

120. A copper wire is bent in the shape of a square ofarea 81 cm2. If the same wire is bent form of asemicircle, the radius (in cm) of the semicircle is

(Take 227

p = )

(a) 16 (b) 14(c) 10 (d) 7

yoursmahboob.w

ordpress.com

52 SSC CGL SolvedPaper

121. The volume (in m3) of rain water that can becollected from 1.5 hectares of ground in a rainfallof 5 cm is(a) 75 (b) 750(c) 7500 (d) 75000

122. A river 3 m deep and 40 m wide is flowing at therate of 2 km per hour. How much water (in litres)will fall into the sea in a minute?(a) 4,00,000 (b) 40,00,000(c) 40,000 (d) 4,000

123. A bicycle wheel makes 5000 revolutions in moving11 km. Then the radius of the wheel (in cm) is

(Take 227

p = )(a) 70 (b) 35(c) 17.5 (d) 140

124. The perimeter of a triangle is 40cm and its area is60 cm2. If the largest side measures 17cm, thenthe length (in cm) of the smallest side of thetriangle is(a) 4 (b) 6(c) 8 (d) 15

125. A shopkeeper allows a discount of 10% to hiscustomers and still gains. 20%. Find the markedprice of the article which costs ̀ 450.(a) ` 600 (b) ` 540(c) ` 660 (d) ` 580

126. What single discount is equivalent to twosuccessive discounts of 20% and 15%?(a) 35% (b) 32%(c) 34% (d) 30%

127. In a business partnership among A, B, C and D,the profit is shared as follows:A's share B's shareB's share C's share

= = C's share 1D's share 3

=

If the total profit is ̀ 4,00,000, the share of C is(a) ` 1,12,500 (b) ` 1,37,500(c) ` 90,000 (d) ` 2,70,000

128. A can contains a mixture of two liquids A and B inthe ratio 7 : 5. When 9 litres of mixture are drawnoff and the can is filled with B, the ratio of A andB becomes 7 : 9. Litres of liquid A contained bythe can initially was(a) 10 (b) 20(c) 21 (d) 25

129. What number should be added to or subtractedfrom each term of the ratio 17 : 24 so that itbecomes equal to 1 : 2?(a) 5 is subtracted (b) 10 is added(c) 7 is added (d) 10 is subtracted

130. The ratio of weekly incomes of A and B is 9 : 7and the ratio of their expenditures is 4 : 3. If eachsaves ̀ 200 per week, then the sum of their weeklyincomes is(a) ` 3,600 (b) ` 4,200(c) ` 4,800 (d) ` 5,600

131. Among three numbers, the first is twice thesecond and thrice the third. If the average of thethree numbers is 49.5, then the difference betweenthe first and the third number is(a) 54 (b) 28(c) 39.5 (d) 41.5

132. The mean of 50 numbers is 30. Later it wasdiscovered that two entries were wrongly enteredas 82 and 13 instead of 28 and 31. Find the correctmean.(a) 36.12 (b) 30.66(c) 29.28 (d) 38.21

133. If the selling price of 10 articles is equal to thecost price of 11 articles, then the gain percent is(a) 10 (b) 11(c) 15 (d) 25

134. While selling a watch, a shopkeeper gives adiscount of 5%. If he gives a discount of 6%, heearns ` 15 less as profit. What is the marked priceof the watch?(a) ` 1,250 (b) ` 1,400(c) ` 1,500 (d) ` 750

135. Krishna purchased a number of articles at ̀ 10 foreach and the same number for ̀ 14 each. He mixedthem together and sold them for `13 each. Thenhis gain or loss percent is

(a) Loss 18 %3

(b) Gain 28 %3

(c) Loss 28 %3

(d) Gain 18 %3

136. A trader bought two horses for `19,500. He soldone at a loss of 20% and the other at a profit of15%. If the selling price of each horse is the same,then their cost prices are respectively.(a) ` 10,000 and ̀ 9,500(b) ` 11,500 and ̀ 8,000(c) ` 12,000 and ̀ 7,500(d) ` 10,500 and ̀ 9,000

137. The cost price of an article is 40% of the sellingprice. What percent of the cost price is the sellingprice?(a) 140% (b) 200%(c) 220% (d) 250%

yoursmahboob.w

ordpress.com

53SSC CGL SolvedPaper

138. If 90% of A = 30% of B and B = 2x % of A, then thevalue of x is(a) 450 (b) 400(c) 300 (d) 150

139. When the price of sugar decreases by 10%, aman could buy 1 kg more for ` 270. Then theoriginal price of sugar per kg is(a) ` 25 (b) ` 30(c) ` 27 (d) ` 32

140. If the price of sugar is raised by 25%, find by howmuch percent a householder must reduce hisconsumption of sugar so as not to increase hisexpenditure?(a) 10 (b) 20(c) 18 (d) 25

141. If 30% of A is added to 40% of B, the answer is80% of B. What percentage of A is B?(a) 30% (b) 40%(c) 70% (d) 75%

142. A man can row 6 km/h in still water. If the speedof the current is 2 km/h, it takes 3 hours more inupstream than in the down–stream for the samedistance. The distance is(a) 30 km (b) 24 km(c) 20 km (d) 32 km

143. A student goes to school at the rate of 122

km/h

and reaches 6 minutes late. If he travels at thespeed of 3 km/h. he is 10 minutes early. Thedistance (in km) between the school and hishouse is(a) 5 (b) 4(c) 3 (d) 1

144. A sum of money placed at compound interestdoubles itself in 4 years. In how many years willit amount to four times itself?(a) 12 years (b) 13 years(c) 8 years (d) 16 years

145. The simple interest on a sum for 5 years is onefourth of the sum. The rate of interest per annumis(a) 5% (b) 6%(c) 4% (d) 8%

146. The difference between the compound interestand simple interest on ` 10,000 for 2 years is ̀ 25.The rate of interest per annum is(a) 5% (b) 7%(c) 10% (d) 12%

Directions (147 to 150) : The following graph showsthe demand and production of cotton by 5 companiesA, B, C, D and E. Study the graph and answer questions147 to 150.

Demand (in tonnes)

Production (in tonnes)

Companies

3000

3300

27002400

1800

1500

1500

600 12

00

900

A B C D E

147. What is the ratio of companies having moredemand than production to those having moreproduction than demand?(a) 2 : 3 (b) 4 : 1(c) 3 : 2 (d) 1 : 4

148. What is the difference (in tonnes) betweenaverage demand and average production of thefive companies taken together?(a) 320 (b) 420(c) 2100 (d) 1050

149. The production of company D is how many timesthat of the production of the company A?(a) 1.8 (b) 1.5(c) 0.5 (d) 0.4

150. The demand for company B is what percent ofthe demand for company C?(a) 1.5 (b) 2.5(c) 25 (d) 30

PART-D : ENGLISH COMPREHENSION

Directions (151–155) : In the following questions,some of the sentences have errors and some arecorrect. Find out which part of a sentence has anerror. The number of that part is the answer. If there isno error your answer is (d) i.e. No error.

151. The future of food companies (a) / seems quitesecure (b) / owed to ever–growing demand. (c) /No error. (d)

152. The vaccine (a)/ when hit the Indian market (b) /is dogged by controversy. (c) / No error. (d)

153. His son (a) / is working (b) / very hardly. (c) / Noerror. (d)

yoursmahboob.w

ordpress.com

54 SSC CGL SolvedPaper

154. Do you know that it was (a) / who has done (b) /this piece of beautiful work? (c) / No error. (d)

155. The company has ordered (a) / some (b) / newequipments. (c) / No error. (d)

Directions (156–160) : In the following questions,sentences are given with blanks to be filled in withan appropriate word. Our alternatives are suggestedfor each question. Choose the correct alternative outof the four.

156. The housewife _______ the cakes burning, andran to switch off the oven.(a) smell (b) smells(c) smelt (d) smelling

157. _______ an old legend, King Shirham lived inIndia.(a) In the event of (b) Due to(c) According to (d) In reference to

158. _______ you leave now, you will be late.(a) Until (b) Till(c) Unless (d) Although

159. The _______ were arrested for illegally huntingthe bears.(a) poachers (b) rangers(c) soldiers (d) villagers

160. If you had followed the rules, you _______disqualified.(a) will not be(b) would not be(c) will not have been(d) would not have been

Directions (161–165) : In the following questions,out of the four alternatives, choose the one whichbest expresses the meaning of the given word.

161. LOQUACIOUS(a) Talkative (b) Slow(c) Content (d) Unclear

162. VINDICTIVE(a) Imaginative (b) Accusative(c) Spiteful (d) Aggressive

163. INCLEMEN T(a) Selfish (b) Active(c) Unfavourable (d) Inactive

164. GENIAL(a) Cordial (b) Unselfish(c) Careful (d) Specific

165. ACCRUE(a) Accumulate (b) Accommodate(c) Grow (d) Suffice

Directions (166–170) : In the following questions,out of the four alternatives, choose the word oppositein meaning to the given word.

166. ACCORD(a) Disagreement (b) Welcome(c) Disrespect (d) Conformity

167. INFIRMITY(a) Employment (b) Indisposition(c) Strength (d) Weakness

168. FEASIBLE(a) Useful (b) Impractical(c) Uneven (d) Important

169. METICULOUS(a) Forgetful (b) Destructive(c) Careless (d) Flagrant

170. SYNTHETIC(a) Natural (b) Plastic(c) Cosmetic (d) Apathetic

Directions (171–175) : In the following questions,four alternatives are given for the meaning of thegiven Idiom/Phrase. Choose the alternative whichbest expresses the meaning of the Idiom/Phrase.

171. A bolt from the blue(a) a delayed event(b) an inexplicable event(c) an unexpected event(d) an unpleasant event

172. Cold comfort(a) absurdity (b) deception(c) slight satisfaction (d) foolish proposal

173. To be all at sea.(a) a family voyage(b) lost and confused(c) in the middle of the ocean(d) a string of islands

174. To take to one's heels(a) to walk slowly (b) to run away(c) to march forward (d) to hop and jump

175. To bite the dust(a) eat voraciously(b) have nothing to eat(c) eat roots(d) None of the above

Directions (176–180) : In the following questions, asentence or Bold part thereof is given which mayneed improvement. Alternatives as given at (a), (b)and (c) below, which may be a better option. In caseno improvement is needed, your answer is (d).

yoursmahboob.w

ordpress.com

55SSC CGL SolvedPaper

176. He has for good left India.(a) He has left for good India.(b) He has left India for good.(c) Good he has left India.(d) No improvement

177. We are credibly informed that the murderer hasgiven himself up.(a) We are informed that the murderer has

credibly given himself up.(b) We are informed that the murderer has given

credibly himself up.(c) We are informed that credibly the murderer

has given up himself,(d) No improvement

178. We generally select one of the most intelligentstudent of the school for this award.(a) one of the most intelligent students of the

school(b) one of the intelligent most students of the

school(c) one of the intelligent most student of the

school(d) No improvement

179. My friend lives in a nearby street whose name Ihave forgotten.(a) the name of which(b) which name(c) of which name(d) No improvement

180. He both won a medal and a scholarship.(a) He won a medal and a scholarship both.(b) Both he won a medal and a scholarship.(c) He won both a medal and a scholarship.(d) No improvement

Directions (181–185) : In the following questions,out of the four alternatives, choose the one whichcan be substituted for the given words/sentence.

181. A post with little work but high salary(a) Director (b) Trustee(c) Sinecure (d) Ombudsman

182. Something that causes death(a) Dangerous (b) Fatal(c) Brutal (d) Horrible

183. A person who writes decoratively(a) Calligrapher (b) Collier(c) Choreographer (d) Cartographer

184. Pertaining to cattle(a) Canine (b) Feline(c) Bovine (d) Verminous

185. To look at someone in an angry or threatening way(a) Glower (b) Gnaw(c) Gnash (d) Grind

Directions (186–190) : In the following questions,groups of four words are given. In each group, oneword is correctly spelt. Find the correctly spelt word.

186. (a) agnostik (b) accomplice(c) advercity (d) acrimonous

187. (a) dysentery (b) momentary(c) cemetary (d) comentary

188. (a) ebulient (b) jubilant(c) iminent (d) tolerant

189. (a) malaign (b) arraign(c) asigne (d) degine

190. (a) harrassment (b) embarrasment(c) fulfilment (d) denoument

Directions (191–200) : In the following passage someof the words have been left out. First read the passageover and try to understand what it is about. Then fillin the blanks with the help of the alternatives given.

PASSAGE (Q.Nos. 191–200)The Solar System has been a complicated wonder forthe astronomers. This is a (191) to which we may neverhave the exact answer. Man has wondered (192) theage of the Earth (193) ancient times. There were allkinds of (194) that seemed to have the (195). But mancould not begin to (196) about the questionscientifically until about 400 years (197). When it wasproved that the (198) revolved round the Sun and theEarth was a (199) of our Solar System, then scientistsknew where to (200).191. (a) problem (b) question

(c) matter (d) query192. (a) around (b) out

(c) about (d) on193. (a) since (b) during

(c) around (d) from194. (a) ideas (b) opinions

(c) stories (d) matters195. (a) solution (b) novel

(c) book (d) answer196. (a) read (b) think

(c) open (d) guess197. (a) now (b) time

(c) then (d) ago198. (a) Moon (b) time

(c) Earth (d) Mars199. (a) part (b) division

(c) opening (d) centre200. (a) end (b) begin

(c) think (d) work

yoursmahboob.w

ordpress.com

56 SSC CGL SolvedPaper

HINTS & SOLUTIONS

1. (c) The body of fish remains covered withscales externally. Similarly, the body of bearremains covered with fur.

2. (c) Here, Worker – Tool relationsh ip has beenshown. Writer uses pen for writting.Similarly, painter works on canvas.

3. (d) As, 1 2 3 4 5 6 7

N U M E R A L¯ ¯ ¯ ¯ ¯ ¯ ¯

It has been arranged as2 4 6 7 5 3 1

U E A L R M N¯ ¯ ¯ ¯ ¯ ¯ ¯

Similarly,

1 2 3 4 5 6 7

A L G E B R A¯ ¯ ¯ ¯ ¯ ¯ ¯

2 4 6 7 5 3 1

L E R A B G A¯ ¯ ¯ ¯ ¯ ¯ ¯

4. (d) As,

B D A C

–1

–1

F H E G

–1

–1

Similarly,

N P M O

–1

–1

R T Q S

–1

–1

5. (b) As,

F G H I : O P Q R

+1 +1+1 +1

+6

+1 +1Similarly,

B C D E : K L M N

+1 +1+1 +1

+6

+1 +1

6. (a) As,

P N L J :

+2 +2 +2 +2

+2 +2

I G E C

Similarly,

V T R P :

+2 +2 +2 +2

+2 +2

O M K I

7. (b) As, 17 × 3 + 9 = 51 + 9 = 60Similarly, 20 × 3 + 9 = 60 + 9 = 69

8. (b) As, (6 + 2)2 = (8)2 = 64Similarly, (11 + 2)2 = (13)2 = 169

9. (d) As, 123 Þ 12 ¸ 3 = 4Similarly, 726 Þ 72 ̧ 6 = 12

10. (a) Except Flute, all others are stringed musicalinstruments.

11. (b) Prod (Verb) means, ‘to push somebdy/something with a finger or a pointedobject’, ‘to encourage’, ‘to poke’.Prod (Noun) means ‘an act of prodding’,‘an act of reminding somebody to takeaction’.Sap (Verb) means ‘to make somebody/something weak or destroy gradually’.Sap (Noun) means ‘the liquid in a plantthat carrries food to all parts of it’, ‘a stupidperson who is easily tricked or treatedunfairly’.Jab (Verb) means ‘to push or poke atsomebody/something quickly androughly’.Jab (Noun) means ‘a sudden rough blow’.Thrust (Verb) means ‘to push something/somebody/oneself suddenly or violently’.Thrust (Noun) means ‘an act or movementof thrusting’.Therefore, Sap is different from others.

12. (b) Except MNST, all others are having avowel.

yoursmahboob.w

ordpress.com

57SSC CGL SolvedPaper

13. (b) TSWV does not have any vowel

C A F D ,

–2 –2 –2 –2

+1 +1

I G L J

and O M R P

–2 –2 –1 –1

+1 +2

T S W Vbut,

So, TSWV is odd one out.14. (d) As, 162 Þ 16 ¸ 2 = 8

405 Þ 40 ̧ 5 = 8,and 567 Þ 56 ̧ 7 = 8,but, 644 Þ 64 ¸ 4 = 16

15. (a) Except 156, all other numbers are oddnumbers.

16. (b) Only ACFJOU follows a particular rule

+2 +3 +4 +5 +6

A C F J O U

17. (a)115 9010.45; 9;11 10

= =

72 568; 79 8

= =

18. (a) Descending Order5. Lt. General

¯4. Brigadier

¯3. Colonel

¯1. Major

¯2. Captain

19. (a)3. Dissent

¯1. Dissident

¯4. Dissolute

¯5. Dissolution

¯2. Dissolve

20. (c) b cb / a ca / b c b / aca / bcb / a c a / b

21. (a) XYZ UVW R ST

CBA FE D

22. (b) Here, the pattern is as follows :

2 2 2 2

2 2 2 2

2 2 2 2

2 2 2 2

r p n l

e g i k

o m k i

c e g i

- - - -

+ + + +

- - - -

+ + + +

¾¾® ¾¾® ¾¾® ¾¾®

¾¾® ¾¾® ¾¾® ¾¾®

¾¾® ¾¾® ¾¾® ¾¾®

¾¾® ¾¾® ¾¾® ¾¾®

jmgk

23. (c) The pattern is as follows :4 = (2)2; 16 = (4)2; 36 = (6)2;64 = (8)2; 196 = (14)2; 169 = (13)2;144 = (12)2;

24. (c) The pattern is as follows :

8 15 36 99 288 855

+7 +21 +63 +189 +567

×3 ×3 ×3 ×3

25. (c) 6 + 3 = 9, 9 + 6 = 15 15 + 12 = 27 , 27 + 24= 51, 51 + 48 = 99

26. (c) Here,9th

K V16th

Total number of girls = 25 + 16 – 1 = 4027. (c) Suppose his present age is x years.

According to question

x x x4 5 3

+ + = x – 13

Þ 15x 12x 20x

60+ +

= x – 13

Þ 47x = 60x – 780Þ 60x – 47x = 780Þ 13x = 780

\ x = 78013

= 60 years

28. (d) Ram lal

Ganga Gopal Mohan

Ram Rekha Mithun

Sharat

Uncle

yoursmahboob.w

ordpress.com

58 SSC CGL SolvedPaper

29. (c) 25 have VCRs and each VCR owner alsohas a TV.Therefore, the TV owners who have notVCRs 75 – 25 = 50Now, 10 have all the three. Therefore,50 – 10 = 40 have only TV.

30. (b) Shashikant was born on 29th September1999.15th August, 1999 was Sunday.Days upto 29th September from 15 August16 + 29 = 45 days = 6 weeks 3 old daysSunday + 3 = Wednesday.

31. (d) A

EB

C

D

So, C is facing towards East.32. (a) Hands of clock will be together at

83211

minutes past 6.

There are 30 munute spaces between hourand minute hand at 6O’clock.The minute hand gains 55 minutes in60 mintues\ It will gain 30 minutes in

60 833 32

55 11´ = mintues

33. (c) Suppose total number of workers in theoffice = x

Number of woman workers = x3

\ Number of man workers

= x – x 3x x 2x3 3 3

-= =

Number of married woman workers =x 1 x3 2 6´ =

Number of married woman workers whohave children

= x 1 x6 3 18´ =

Number of married man workers

= 2x 3 x3 4 2´ =

Number of married man workers who have

children = x 2 x2 3 3´ =

Number of workers who have children

= x x3 18

+

= 6x x 7x

18 18+

=

Number of workers without children

= x – 7x 18x 7x 11 x18 18 18

-= =

34. (d) The direction diagram is as follows:Starting point

4 km

2 km 4 km

It is clearly shown that he is moving southdirection.

35. (b)

AH = 20 – (4 + 6) = 10m36. (d) There is only one ‘E’ in the given word.37. (c) There is no ‘C’ letter in the given word.

There is no ‘L’ letter in the given word.There is no ‘V’ letter in the given word.

38. (b) Difference is +2, + 4, +6, +8, +10, +12, +14,+16

39. (c) As, D E L H I

7 3 5 4 1¯ ¯ ¯ ¯ ¯

and C A L C U T T A

8 2 5 8 9 6 6 2¯ ¯ ¯ ¯ ¯ ¯ ¯ ¯

Therefore,

C A L I C U T

8 2 5 1 8 9 6¯ ¯ ¯ ¯ ¯ ¯ ¯

yoursmahboob.w

ordpress.com

59SSC CGL SolvedPaper

40. (c) Only conclusion IV is appropriate. The useof ‘term’ all in the Conclusion I amkes itinvalid. From a general statement, wecannot derive definite conclusion. Thereare many factors responsbile for pollution.Therefore, we cannot say that pollutednations are industrialised. As pollution isa problem associated with industrialisation,India may become polluted.

41. (c) ColumnwiseI. 2 × 7 × 9 = 126II. 7 × 3 × 8 = 168III. 9 × 4 × ? = 216

\ ? = 216 69 4

42. (d)65 9

+8

+32

33 17+16

43. (a) 30 – 6 + 5 × 4 ̧ 2 = 27Þ 30 ¸ 6 × 5 + 4 – 2 = 27Þ 25 + 4 – 2 = 2730 + 6 – 5 ̧ 4 × 2 = 30Þ 30 × 6 ¸ 5 – 4 + 2 = 30Þ 36 – 4 + 2 ¹ 30 30 × 6 ¸ 5 – 4 + 2 = 32Þ 30 + 6 – 5 ̧ 4 × 2 ¹ 32

44. (d) 9 + 7 = 16; 9 – 7 = 216 × 2 = 3213 + 7 = 20; 13 – 7 = 620 × 6 = 12017 + 9 = 26; 17 – 9 = 826 × 8 = 20819 + 11 = 30; 19 – 11 = 8

30 × 8 = 24045. (c) As, 3.5 + 0.2 = 3.7

3.7 + 0.4 = 4.14.1 + 0.8 = 4.94.9 + 1.6 = 6.5

6.5 + 3.2 = 9.7

46. (d) Sun is a star. Moon is a satellite.

Star

Sun

Moon

47. (d)A B C D

I H G F

EJK L

M

The rectangles are :ABKJ; JKHI; BCLK;KLGH; CDML; LMFG;ACGI; ACLJ; JLGI;BDFH; BDMK; KMFH;ADFI; ADMJ; JMFIABHI, BCGH and CDFG are squares.We know that every square is a rectanlge.But its reverse is not always true.Note : By option only its easy to analyze

48. (c) 49. (c) 50. (c) 51. (c) 52. (d)53. (c) 54. (b) 55. (a) 56. (c) 57. (c)58. (c) 59. (b) 60. (b) 61. (c) 62. (b)63. (b) 64. (b) 65. (d) 66. (b) 67. (d)68. (d) 69. (c) 70. (b) 71. (a) 72. (b)73. (b) 74. (c) 75. (b) 76. (c) 77. (d)78. (a) 79. (b) 80. (c) 81. (c) 82. (b)83. (d) 84. (c) 85. (d) 86. (c) 87. (d)88. (d) 89. (a) 90. (a) 91. (b) 92. (d)93. (c) 94. (a) 95. (d) 96. (c) 97. (a)98. (b) 99. (a) 100. (a)

101. (b) 6 6 6..... x+ + =6 = 3 × 2By trick = 3 answer

102. (a) Expression = ( 3 2)( 3 2)

+-

Rationalising the denominator,( 3 2)( 3 2)( 3 2)( 3 2)

+ +- +

= 2

2( 3 2) ( 3 2)3 2+ = +-

\ 23 2 ( 3 2)3 2

+ = +-

= 3 2+

yoursmahboob.w

ordpress.com

60 SSC CGL SolvedPaper

103. (c) 31 = 3; 32 = 9; 33 = 27; 34 = 81; 34 = 243i.e. unit’s digit is repeated after index 4.Remainder after dividing 21 by 4 = 1\ Unit’s digit in the expansion of (3)21 = 3\ Remainder after dividing by 5 = 3

104. (a) Expression

=

7 13 77 393 11 33

1 13 31 33 39 1 103

--=

+ ++ ++

=

38 3833 33

1 103 330 3 3310

=+ ++

=

3838 33 3833

99 10 33 109 10933

= ´ =+

105. (b) Last digit of (1001)2008 + 1002 = 1 + 2 = 3106. (b) Expression

= 3 2 4 3 63 6 6 2 3 2

- ++ + +

= 3 2( 6 3) 4 3( 6 2)

( 6 3)( 6 3) ( 6 2)( 6 2)- -

-+ - + -

+

6 3 2( 3 2) 3 2

+ -

= 3 2( 6 3) 4 3( 6 2)

6 3 (6 2)- -- +- -

6( 3 2)

3 2--

= 2( 6 3) 3( 6 2)- - - +

6( 3 2)-

= 12 6 18 6 18 12- - + + -= 0

107. (c) x « y = (x + 3)2 (y – 1)\ 5 « 4 = (5 + 3)2 (4 – 1)= 64 × 3 = 192

108. (b)2 2 2

2 2 2(0.05) (0.41) (0.073)

(0.005) (0.041) (0.0003)+ ++ +

2 2 2

2 2 2

(0.05) (0.41) (0.073)1 (0.05) (0.41) (0.073)

100

+ +

+ +

= 100109. (b) 9 x 3 2 2 3 7 7= ´ ´ + ´ ´

9 x 2 3 7 3 9 3Þ = + =x 3\ =

110. (b) = 3 3127 343 1271343 343

-- =

= 3

3 33

216 (6) 6 11343 7 7(7)

= = = -

111. (a) Let the numbers be x and y.\ x (x + y) = 247and y (x + y) = 114Þ x2 + xy = 247 and xy + y2 = 114On adding;x2 + xy + xy + y2 = 247 + 114Þ x2 + 2xy + y2 = 361Þ (x + y)2 = 192 Þ x + y = 19

112. (a) Let the number be x.

\ x x7 11- = 100

Þ 11x 7x

11 7-´

= 100

Þ 4x = 77 × 100

Þ x = 77 100

= 1925

113. (d) HCF must be a factor of LCM from option35 is not factor of 120.

114. (a) Here, the first divisor i.e. 49 is multiple ofsecond divisor i.e. 7.\ Required remainder = Remainderobtained on dividing 32 by 7 =4

115. (b) Let the number of correct answers be x.\ x + 4 – (75 – x) × 1 = 125Þ 4x – 75 + x = 125Þ 5x = 125 + 75 = 200

\ x = 200

5 = 40

yoursmahboob.w

ordpress.com

61SSC CGL SolvedPaper

116. (b) LCM of 24, 36 and 54 seconds= 216 seconds= 3 minutes 36 seconds\ Required time = 10 : 15 : 00 +3 minutes 36 seconds= 10 : 18 : 36 a.m.

117. (a) ATQ3 3 3A A B

+ + = 1

6 312 B

+ = 1

3 1B 2

=

B = 6 days

118. (c) (A + B)’s 1 day’s work = 18

(B + C)’s 1 day’s work = 1

12

(C + A)’s 1 day’s work = 18

On adding,2 (A + B + C)’s 1 day’s work

= 1 1 1 3 2 38 12 8 24

+ ++ + =

= 8 124 3

=

\ (A + B + C)’s 1 day’s work = 16

Hence, the work will be compelted in 6days.

119. (a) If X completes a work in x days, Y will dothe same in 3x days.\ 3x – x = 40 Þ x = 20\ Y will finish the work in 60 days.\ (X + Y)’s 1 days work

= 1 1 3 1 120 60 60 15

++ = =

\ Both together will complete the work in15 days.

120. (d) Side of a square

= 81 = 9 cm\ Length of the wire= 4 × 9 = 36 cm.

\ Perimeter of semi-circle = (p + 2)rwhere r = radius

Þ 22

2 r 367æ ö+ =ç ÷è ø

Þ 36

r 367

=

Þ r = 36 7

36´

= 7 cm.

121. (b) 1 hectare = 10000 sq. metre\ Area of the ground = 15000 sq. metre

\ Required volume = 15000 × 5

100= 750 m3

122. (b) Volume of water flowed in an hour= 2000 × 40 × 3 m3

= 240000 m3

\ Volume of water flowed in 1 minute.

= 240000

60 = 4000 m3

= 4000000 litre123. (b) Distance covered by wheel in one revolution

= Circumference of wheel

= 11000 115000 5

= m

= 11 1005´ cm = 220 cm

\ 2pr = 220

Þ 2 ×22 r7´ = 220

Þ r = 220 72 22´´

= 35 cm

124. (c) Smallest side of the triangle = x cm (let)\ Second side of triangle= 40 – 17 – x = 23 – x

Semi-perimeter, = s = 402

= 20

\ s(s a)(s b)(s c)- - - = 60

Þ 20(20 17)(20 x)(20 23 x)- - - + = 60Þ (20 – x) (x – 3) = 60Þ 20x – 60 – x2 + 3x = 60Þ x2 – 23x + 120 = 0Þ x2 – 15x – 8x + 120 = 0Þ x (x – 15) – 8 (x – 15) = 0Þ (x – 8) (x – 15) = 0Þ x = 8 or 15

yoursmahboob.w

ordpress.com

62 SSC CGL SolvedPaper

125. (a) Let the marked price of the article be ̀ x.

\ x × 90 450 120

100 100´

=

Þ 9x10

= 540

Þ x = 540 10

= ̀ 600

126. (b) Single equivalent discount

= xy

x y %100

æ ö+ -ç ÷è ø

= 20 15

20 15 %100´æ ö+ -ç ÷è ø = 32%

127. (c) A : B = 1 : 3B : C = 1 : 3 = 3 : 9C : D = 1 : 3 = 9 : 27\ A : B : C : D = 1 : 3 : 9 : 27Sum of ratios = 1 + 3 + 9 + 27 = 40\ C’s share of profit

= 940

× 400000 = ̀ 90000

128. (c) A = 7x litre, B = 5x litre (let)In 9 litres of mixure,

= 7x 9

12x´ =

214

litre

B = 5x 15

912x 4´ = litre

In new situation,

217x4

155x 94

-

- + =

79

Þ 28x 21 7

20x 15 36 9- =- +

Þ 252x – 189 = 140x + 147Þ 112x = 336 Þ x = 3\ Initial quantity of liquid A= 7x = 7 × 3 = 21 litre

129. (d) Let the number x be added

\ 17 x 124 x 2

+ =+

Þ 34 + 2x = 24 + xÞ 2x – x = 24 – 34Þ x = –10Hence, 10 should be subtracted.

130. (*) Let monthly income of A and B be 9x and 7xExpenditure = Income – SavingATQ9x 200 47x 200 3-

=-

27x – 6.00 = 28x – 800x = 200Sum = 200 × 16 = 3200

131. (a) Let the second number be x.\ First number = 2x

\ Third number = 2x3

\ 2x + x + 2x3

= 49.5 × 3

Þ 6x + 3x + 2x = 49.5 × 9 = 445.5

Þ 11x = 445.5 Þ x = 445511

= 4.05

\ Requried difference

= 2x – 2x 4x3 3

=

= 4 40.5

= 54

132. (c) Requried average

= 30 + (28 31 82 13)

50+ - -

= 36

3050

æ ö+ -ç ÷è ø = 30 – 0.72 = 29.28

133. (a) Let the C.P. of each article be ̀ 1.\ C.P. of articles = ` 10and S.P. of 10 articles = ` 11\ Profit percent

= 11 10

10010-´ = 10%

134. (c) Difference in discount = 1%

1 x100´ = 15

x = 1500

135. (a) Average cost of = 10 14

2+

= 12

QP = 13

P% = 13 12 100

12- ´ =

183

yoursmahboob.w

ordpress.com

63SSC CGL SolvedPaper

136. (b) The sum of cost prices of two articles is` x. One of them is sold at a loss of a% andother is sold at a gain of b% and their S.P.is same.\ C.P. of article sold at a loss of a%

= 100 b

x200 a b

- +

= 100 15

19500200 20 15

- +

= 115

19500195´ = ̀ 11500

\ C.P. of second article = ` 8000137. (d) Let the S.P. of the article = ` 100

\ C.P. = ̀ 40\ Required percentage

= 100

10040´ = 250%

138. (d)A 90 B 30

100 100´ ´

=

Þ 3A = B

Þ 3A = A × 2x

100Þ 300 = 2x Þ x = 150

139. (b) Let the original price of sugar be ̀ x/kg.

\ New price = ` 9x10

/kg

\ 270 2709x x10

- = 1

Þ 300 270 30

1 1x x x- = Þ =

Þ x = ̀ 30/kg

140. (b) Percentage decrease = 25

125× 100 = 20

141. (d) A × 30 B 40 B 80

100 100 100´ ´+ =

Þ A × 30 = B × 40

Þ A 40 4B 30 3

= = Þ B 3A 4

=

Þ B 3100 100A 4´ = ´ = 75%

142. (b) Let the required distance be x km.

\ x x

6 2 6 2-

- + = 3

Þ x x4 8- = 3

Þ 2x x

8-

= 3

Þ x = 3 × 8 = 24 km.143. (b) Let the required distance be x km.

x x 165 3 602

- =

Þ 2x x 45 3 15- =

Þ 6x 5x 4

15 15-

= Þ x = 4 km.

144. (c) A = P TR1

100æ ö+ç ÷è ø

Þ 2 = 1 4R1

100æ ö+ç ÷è ø

Þ 22 = 8R1

100æ ö+ç ÷è ø

= 8 yrs

145. (a) Simple Interest 1Principal 4

=

\ Rate = S.I. 100

Principal Time´´

= 1 1004 5´´

= 5 % per annum

146. (a) Difference = 2PR

10000

Þ 25 = 210000 R

10000´

Þ R = 5%147. (c) Requried ratio = 3 : 2

yoursmahboob.w

ordpress.com

64 SSC CGL SolvedPaper

148. (b) Required difference

= 3000 600 2400 1200 3300

5+ + + +æ ö

ç ÷è ø

1500 1800 900 2700 15005

+ + + +æ ö- ç ÷è ø= 2100 – 1680 = 420 tonnes

149. (a) Requried value = 27001500

= 1.8

150 (c) Required percentage

= 600

1002400

´ = 25%

151. (c) Owing to (preposition) means : because of.Hence, owing to ever-growing demand...should be used here.

152. (c) Here simple past (Passive) i.e. was doggedby controversy should be used.

153. (c) Here, very hard should be used. The wordhardly (Adverb) means : almost no; almostnot; almost none.The word hard means : putting a lot ofeffort or energy into an activity.

154. (b) Here, the antecedent of Relative Pronoun‘who’ is ‘I’ and hence who have doneshould be used as 'I' agrees with 'have'.

155. (c) The word equipment (Uncountable Noun)means : the things that are needed for aparticular purpose or activity.Its plural is not equipments.Hence, new equipment should be used here.

156. (c) 157. (c) 158. (a)159. (a) 160. (d)161. (a) The word Loquacious (Adjective) means :

talking a lot; talkative.162. (c) The word Vindictive (Adjective) means :

trying to harm or upset somebody orshowing that you want to, because youthink that they have harmed you; spiteful;revengeful.

163. (c) The word Inclement (Adjective) means :not pleasant; unfavourable; cold. wet etc.

164. (a) The word Genial (Adjective) means :friendly and cheerful; affable; cordial.

165. (a) The word Accrue (Verb) means : toincrease over a period of time; to allow asum of money or debts to grow over aperiod of time; accumulate.

166. (a) The word Accord (Noun) means : a formalagreement between two organisations,countries etc.Its antonym should be disagreement

167. (c) The word Infirmity (Noun) means :weakness or illness over a long period.Its antonym should be strength (Noun)which means : the quality of beingphysically strong, brave.

168. (b) The word Feasible (Adjective) means : thatis possible and likely to be achieved;practicable.Its antonym should be impractical.

169. (3) The word Meticulous (Adjective) means :paying careful attention to every detail;fastidious; thorough.

170. (a) The word Synthetic (Adjective) means :artificial; man-made.Its antonym should be natural.

171. (c) Idiom 'a bolt from the blue' means : an eventor a piece of news which is sudden andunexpected; a complete surprise.

172. (c) Cold Comfort (Noun) means : the fact thatsomething that would normally be gooddoes not make you happy because thewhole situation is bad.

173. (b) Idiom to be all at sea means : confused andnot knowing what to do.

174. (b) Idiom take to your heels means : to runaway from somebody/something.

175. (d) Idiom bite the dust means : to fail or to bedefeated or destroyed.

176. (b) He has left India for good.177. (d) Give yourself up to somebody means : to

offer yourself to be captured.178. (a) 179. (a) 180. (c) 181. (c)182. (b) 183. (a) 184. (c) 185. (a)186. (b) accomplice (agnostic; adversity;

acrimonious).187. (a) dysentery (cemetery; momentary;

commentary).188. (b) jubilant (ebullient; imminent; tolerant).189. (b) arraign (malign; assign; design).190. (c) fulfilment (harassment; embarrassment;

denouement).191. (b) 192. (c) 193. (a) 194. (c)195. (d) 196. (b) 197. (d) 198. (c)199. (a) 200. (b)

yoursmahboob.w

ordpress.comPART-A : GENERAL INTELLIGENCE & REASONING

1. If MEKLF is coded as 91782 and LLLJK as 88867,then how can IGHED be coded?(a) 97854 (b) 64521(c) 53410 (d) 75632

2. Two statments are given below followed by fouralternative inferences. Select the one which ismost appropriate.Statements :(i) All radios sold in that shop are of high

standard.(ii) Some of Murphy radios are sold in that shop.Inferences:1. All radios of high standard are

manufactured by Murphy company.2. Some of the Murphy radios are of high

standard.3. None of the Murphy radios is of high

standard.4. Some of the Murphy radios of high standard

are sold in that shop.(a) 1 and 2 inferences only(b) 2 and 4 inferences only(c) 1 and 3 inferences only(d) 1 and 4 inferences only

Directions: In Question nos. 3 and 4, select themissing number from the given responses :

3. 7 3 24 9 62 1 5

69 91 ?

(a) 58 (b) 51(c) 65 (d) 64

4.

?

216

8

64

(a) 343 (b) 512(c) 729 (d) 1000

5. The population of rats is increasing year afteryear in a village. Find out the missing populationfrom the following information:Years 1990 1991 1992 1993 1994 1995Population 4 8 16 ? 44 64(a) 22 (b) 32(c) 28 (d) 34

6. If – stands for division, + for multiplication, ̧ forsubtraction and × for addition, then which one ofthe following equations is correc?(a) 19 + 5 – 4 × 2 ̧ 4 = 11(b) 19 × 5 – 4 ̧ 2 + 4 = 16(c) 19 ¸ 5 + 4 – 2 × 4 = 13(d) 19 ¸ 5 + 4 + 2 ̧ 4 = 20

Directions : In Question nos 7 to 15, select the relatedletter/word/ number from the given alternatives.

7. Country : President : : State : ?(a) Chief Minister (b) Prime Minister(c) Speaker (d) Governor

8. Mirage : Desert : : ?(a) Sky : Illusion (b) Rainbow : Sky(c) Rain : Rainbow (d) Image : Mirror

9. TEKCAR : RACKET : : TCEJBO : ?(a) TCEOBJ (b) OBJECT(c) CEJBOT (d) REJECT

10. JIHK : PONQ : : WVUX : ?(a) KNML (b) RSTU(c) HIGJ (d) MLKN

11. UUWX : WWYZ : : OOQR : ?(a) OOPG (b) MMPQ(c) XXYZ (d) QQST

12. BIMN : CKPR : : CURD : ?(a) DWUH (b) WUHC(c) UHDW (d) HUVN

13. BCFE : HILK : : NORQ : ?(a) TXWU (b) TXUW(c) TUXW (d) TUWX

14. ? : 63 : : 08 : 26(a) 12 (b) 9(c) 18 (d) 15

15. 64 : ? : : 72 : 53(a) 44 (b) 54(c) 52 (d) 70

SSC Combined Graduate Level (CGL) Solved PaperEVENING SHIFT 26 JUNE 2011

yoursmahboob.w

ordpress.com

66 SSC CGL SolvedPaper

Directions : In question nos. 16 to 21, find the oddnumber/letters/ number pair from the givenalternatives.16. (a) Softball (b) Baseball

(c) Cricket (d) Basketball17. (a) Annoy (b) Distress

(c) Harass (d) Ravage18. (a) PRSU (b) SUWY

(c) HJLN (d) CEGI19. (a) YDWB (b) TKRI

(c) QNOM (d) HLFJ20. (a) Tomato (b) Potato

(c) Carrot (d) Onion21. (a) 125 (b) 789

(c) 236 (d) 34722. Find out the pair of numbers that does not belong

to the group for lack of common property.(a) 16 – 18 (b) 56 – 63(c) 96 – 108 (d) 86 – 99

23. In the following question, number of letters areskipped in between by a particular rule. Whichof the following observes the rule?(a) ACZXFG (b) CFXURI(c) CFIURX (d) CXFUIR

24. Which one of the given responses would be ameaningful order of the following?1. Sentence 2. Word3. Chapter 4. Phrase5. Paragraph(a) 4, 3, 1, 2, 5 (b) 2, 3, 5, 4, 1(c) 3, 5, 1, 4, 2 (d) 1, 3, 2, 4, 5

25. Arrange the following words as per order in thedictionary:1. Inhabit 2. Ingenious3. Inherit 4. Influence5. Infatuation(a) 1, 2, 3, 4, 5 (b) 5, 4, 1, 2, 3(c) 4, 5, 2, 1, 3 (d) 5, 4, 2, 1, 3

26. Which one set of letters when sequentiallyplaced at the gaps in the given letter series shallcomplete it?__ab __b __aba __ __ abab(a) a bb aa (b) bb aa b(c) ab aa b (d) a aa ba

Directions: In Question Nos. 27 to 30, a series isgiven, with one/two term missing. Choose the correctalternative from the given ones that will completethe series.27. 14, 19, 29, 49, 89, __?__

(a) 139 (b) 149(c) 159 (d) 169

28. BMRG, DLTF, FKVE, HJXD, __ ? __(a) JIZC (b) JZIB(c) GIFB (d) MOLC

29. 121, 144, 289, 324, 529, 576, __ ? __(a) 961 (b) 841(c) 900 (d) 729

30. 5, 21, 69, 213, 645, __ ? __(a) 1670 (b) 1941(c) 720 (d) 1320

31. Find out the questioned number.6 : 5 : : 8 : ?(a) 2 (b) 4(c) 6 (d) 10

32. In a row of trees, a tree is 7th from left end and14th from the right end. How many trees are therein the row?(a) 18 (b) 19(c) 20 (d) 21

33. Shan is 55 years old, Sathian is 5 years junior toShan and 6 years senior to Balan. The youngestbrother of Balan is Devan and he is 7 years juniorto him. So what is the age difference betweenDevan and Shan?(a) 18 years (b) 15 years(c) 13 years (d) 7 years

34. Vinod introduces Vishal as the son of the onlybrother of his father's wife. How is Vinod relatedto Vishal?(a) Cousin (b) Brother(c) Son (d) Uncle

35. There are 80 families in a small extension area. 20per cent of these families own as car each. 50 percent of the remaining families own a motor cycleeach. How many families in that extension do notown any vehicle?(a) 30 (b) 32(c) 23 (d) 36

36. If John celebrated his victory day on Tuesday,5th January 1965, when will he celebrate his nextvictory day on the same day?(a) 5th January 1970 (b) 5th January 1971(c) 5th January 1973 (d) 5th January 1974

37. Five girls M, N, O, P and Q are standing in a row.p is on the right of Q, N is on the left of Q, but ison the right of M, P is on the left of O. Who isstanding on the extreme right?(a) Q (b) N(c) O (d) P

yoursmahboob.w

ordpress.com

67SSC CGL SolvedPaper

38. Sita is elder than Swapna. Lavanya is elder thanSwapna but younger than Sita. Suvarna isyounger than both Hari and Swapna, Swapna iselder than Hari. Who is the youngest ?(a) Sita (b) Lavanya(c) Suvarna (d) Hari

39. After 9'O clock at what time between 9 p.m. and10 p.m. will the hour and minute hands, of a clockpoint in opposite direction?

(a) 15 min. past 9 (b) 16 min. past 9

(c) 416

11min. past 9 (d)

11711

min. past 9

40. One evening, Raja started to walk toward theSun. After walking a while, he turned to his rightand again to his right. After walking a while, heagain turned right. In which direction is hefacing?(a) South (b) East(c) West (d) North

41. Vivek and Ashok start from a fixed point. Vivekmoves 3 km north and turns right and then covers4 km. Ashok moves 5 km west and turns rightand walks 3 km. Now how far are they apart?(a) 10 km (b) 9 km(c) 8 km (d) 6 km

42. From the given alternative words, select the wordwhich cannot be formed using the letters of thegiven word:MISFORTUNE(a) FORT (b) TURN(c) SOFT (d) ROAM

43. From the given alternative words, select the wordwhich can be formed using the letters of the givenword:STRANGULATION(a) TRIANGLE (b) GARLAND(c) ROASTING (d) TRAUMA

44. If in a certain code, RAMAYANA is written asPYKYWYLY, then how MAHABHARATA canbe written in that code?(a) NBIBCIBSBUB (b) LZGZAGZQZSZ(c) MCJCDJCTCVC (d) KYFYZFYPYRY

45. Some equations have been solved on the basisof certain system. Find the correct answer for theunsolved equation on that basis.If 94 + 16 = 42, 89 + 23 = 78, then 63 + 45 = ?(a) 18 (b) 28(c) 38 (d) 48

46. Which one of the following diagrams representsthe correct relationship among 'Judge', 'Thief' and'Criminal'?

(a) (b)

(c) (d)

47. How many triangles are there in the given figure?

A B

C D(a) 16 (b) 14(c) 8 (d) 12

Directions: From the given answer figures, select theone in which the question figure is hidden/ embedded.

48. Question figure

Answer figures

(a) (b) (c) (d)

yoursmahboob.w

ordpress.com

68 SSC CGL SolvedPaper

Directions: if a mirror is placed on the line MN, thenwhich of the answer figures is the right image of thegiven figure ?49. Question figure

N

MAnswer figures

(b) (c) (d)

Directions: A piece of paper is folded and cut asshown below in the question figures. From the givenanswer figures, indicate how it will appear whenopened.

50. Question figure

(a) (b) (c) (d)Answer figures

(a) (b) (c) (d)

PART-B : GENERAL AWARENESS51. The common tree species in nilgiri hills is

(a) Sal (b) Pine(c) Eucalyptus (d) Teak

52. Which of the following statements on RailwayBudget for 2011–12 is correct?(a) There would be 10% increase in fares for

long distance train travel both byairconditioned and non–airconditionedclasses.

(b) There would be 15% increase in freight rateson all goods other than foodgrains.

(c) There would be 15% increase in passengerfares for all classes for long distance travelas well as in freights.

(d) There would be no increase in fares for bothsuburban and long distance train travel.

53. The nuclear reactors which were damagedheavily due to a strong Earthquake - cum–Tsunami that hit Japan on March 11, 2011 causingradiation leakage were at(a) Fukushima (b) Kyoto(c) Tokyo (d) None of the above

54. The maximum limit on poll expenditure forAssembly constituencies has been raised inFebruary, 2011 to(a) 16 lakhs (b) 18 lakhs(c) 20 lakhs (d) 25 lakhs

55. First Indian Prime Minister to visit Siachen hasbeen(a) Rajiv Gandhi(b) Inder Kumar Gujaral(c) Man Mohan Singh(d) None of the above

56. Which of the following books has been writtenby Kishwar Desai?(a) The Red Devil(b) Witness the Night(c) Tonight This Savage Rite(d) Earth and Ashes

57. Which of the following folk/tribal dances isassociated with Karnataka?(a) Yakshagana (b) Veedhi(c) Jatra (d) Jhora

58. Who of the following received Sangeet NatakAkademi's Ustad Bismillah Khan Yuva Puraskarfor 2009 in Theatre?(a) Omkar Shrikant Dadarkar(b) Ragini Chander Sarkar(c) Abanti Chakravorty & Sukracharya Rabha(d) K. Nellai Manikandan

59. Which of the following countries did not winany of the "FIFA World Cup" in 2002, 2006 &2010?(a) Brazil (b) Argentina(c) Spain (d) South Africa

60. Who invented vaccination for 'Small Pox'?(a) Sir Fredrick Grant Banting(b) Sir Alexander Fleming(c) Edward Jenner(d) Louis Pasteur

61. Who was the first Indian to become member ofBritish Parliament?(a) Bankim Chander Chatterjee(b) W.C. Bannerjee(c) Dadabhai Naoroji(d) None of the above

yoursmahboob.w

ordpress.com

69SSC CGL SolvedPaper

62. The purchase of shares and bonds of Indiancompanies by Foreign Institutional Investors iscalled(a) Foreign Direct Investment(b) Portfolio Investment(c) NRI Investment(d) Foreign Indirect Investment

63. BT seed is associated with(a) Rice (b) Wheat(c) Cotton (d) Oil seeds

64. The Headquarters of International AtomicEnergy Agency is in(a) Geneva (b) Paris(c) Vienna (d) Washington

65. In the Budget Estimates for 2011–12, anallocation of ̀ 400 crore has been made to bringin a green revolution in the East in the rice–basedcropping system of(a) Assam & West Bengal only(b) Assam, West Bengal, Orissa & Bihar only(c) Assam, West Bengal, Orissa, Bihar &

Jharkhand only(d) Assam, West Bengal, Orissa, Bihar,

Jharkhand, Eastern Uttar Pradesh &Chhatisgarh

66. In the Budget for 2011–12 presented by theFinance Minister on 28–2–2011, the income taxexemption limit for senior citizens (60 years toless than 80 years) has been increased to(a) ` 2.50 lakh (b) ` 2.60 lakh(c) `2.80 lakh (d) ` 3.00 lakh

67. If the Anglo–Indian community does not getadequate representation in the Lok Sabha, twomembers of the community can be nominated bythe(a) Prime Minister(b) President(c) Speaker(d) President in consultation with theParliament

68. For the election of President of India, a citizenshould have completed the age of(a) 25 years (b) 30 years(c) 35 years (d) 18 years

69. Who said, "A good citizen makes a good Stateand a bad citizen makes a bad State "?(a) Plato (b) Rousseau(c) Aristotle (d) Laski

70. A member of Parliament will lose his membershipof Parliament if he is continuously absent fromSessions for(a) 45 days (b) 60 days(c) 90 days (d) 365 days

71. In India, the Residuary Powers are vested with(a) Union Government(b) State Government(c) Both the Union Government and the StateGovernment(d) Local Government

72. Mention the place where Buddha attainedenlightenment(a) Sarnath (b) Bodh Gaya(c) Kapilavastu (d) Rajgriha

73. Coronation of Shivaji took place in(a) 1627 A.D. (b) 1674 A.D.(c) 1680 A.D. (d) 1670 A.D.

74. The System of Dyarchy was introduced in India in(a) 1909 (b) 1935(c) 1919 (d) 1945

75. The Editor of 'Young India' and 'Harijan' was(a) Nehru (b) Ambedkar(c) Mahatma Gandhi(d) Subash Chandra Bose

76. Who of the following attended all the ThreeRound Table Conferences?(a) B.R. Ambedkar (b) M.M. Malavia(c) Vallabhbhai Patel (d) Gandhiji

77. Which is the largest living bird on Earth?(a) Emu (b) Ostrich(c) Albatross (d) Siberian Crane

78. Rihand Dam Project provides irrigation to(a) Gujarat and Maharashtra(b) Orissa and West Bengal(c) Uttar Pradesh and Bihar(d) Kerala and Karnataka

79. The Headquarters of MCF (Master ControlFacility) – the nerve centre of the entire spacecraft operations in India is at(a) Hyderabad – Andhra Pradesh(b) Thumba – Kerala(c) Sriharikota – Andhra Pradesh(d) Hassan – Karnataka

80. Which is the longest irrigation canal in India?(a) Sirhind Canal (b) Yamuna Canal(c) Indira Gandhi Canal (d) East Kosi Canal

yoursmahboob.w

ordpress.com

70 SSC CGL SolvedPaper

81. Which one of the following minerals is found inMonazite sand?(a) Potassium (b) Uranium(c) Thorium (d) Sodium

82. In coriander, the useful parts are(a) roots & leaves(b) leaves & flowers(c) leaves & dried fruits(d) flowers & dried fruits

83. Which plant is called Herbal Indian Doctor?(a) Amla (b) Mango(c) Neem (d) Tulsi

84. The pH of human blood is(a) 7.2 (b) 7.8(c) 6.6 (d) 7.4

85. Which amongst the following is largest endocrinegland in the body?(a) Thyroid (b) Parathyroid(c) Adrenal (d) Pituitary

86. Which amongst the following is the largestmammal?(a) Elephant (b) Whale(c) Dinosaur (d) Rhinoceros

87. Which part becomes modified as the tusk ofelephant?(a) Canine (b) Premolar(c) Second incisor (d) Molar

88. Optical fibres are based on the phenomenon of(a) Interference(b) Dispersion(c) Diffraction(d) Total Internal Reflection

89. Now–a–days yellow lamps are frequently usedas street lights. Which of the following gases isused in these lamps?(a) Sodium (b) Neon(c) Hydrogen (d) Nitrogen

90. 'Mirage' is an example of(a) refraction of light only(b) total internal reflection of light only(c) refraction and total internal reflection of light(d) dispersion of light only

91. The phenomenon of light associated with theapprearance of blue colour of the sky is(a) Interference (b) Reflection(c) Refraction (d) Scattering

92. In which of the following areas, a spreadsheetsoftware is more useful?(a) Psychology (b) Publishing(c) Statistics (d) Message sending

93. A Group Ware is a(a) Hardware (b) Network(c) Software (d) Firmware

94. Lens is made up of(a) Pyrex glass (b) Flint glass(c) Ordinary glass (d) Cobalt glass

95. The element which is used for vulcanizing rubber(a) Sulphur (b) Bromine(c) Silicon (d) Phosphorus

96. Which of the following is responsible for the extrastrength of pyrex glass?(a) Potassium Carbonate (b) Lead Oxide(c) Borax (d) Ferric Oxide

97. The noble gas used for the treatment of cancer is(a) Helium (b) Argon(c) Krypton (d) Radon

98. Vasundhara Summit was held in(a) USA (b) UK(c) Brazil (d) Australia

99. 'Loktak' is a(a) Valley (b) Lake(c) River (d) Mountain Range

100. Which city receives the highest cosmic radiationamongst the following?(a) Chennai (b) Mumbai(c) Kolkata (d) Delhi

PART-C : NUMERICAL APTITUDE

101. X sells two articles for ̀ 4,000 each with no lossand no gain in the interaction. If one was sold ata gain of 25% the other is sold at a loss of

(a) 25% (b)218 %9

(c)216 %3

(d) 20%

102. 20% loss on selling price is what per cent loss onthe cost price?(a) 25% (b) 15%

(c)216 %3

(d)116 %3

103. If 90% of A = 30% of B and B = x% of A, then thevalue of x is(a) 800 (b) 300(c) 700 (d) 400

yoursmahboob.w

ordpress.com

71SSC CGL SolvedPaper

104. A reduction of 20% in the price of sugar enablesme to purchase 5 kg more for ̀ 600. Find the priceof sugar per kg before reduction of price.(a) ` 24 (b) ` 30(c) ` 32 (d) ` 36

105. The price of a commodity rises from ` 6 per kg to` 7.50 per kg. If the expenditure cannot increase,the percentage of reduction in consumption is(a) 15 (b) 20(c) 25 (d) 30

106. First and second numbers are less than a thirdnumber by 30% and 37% respectively. Thesecond number is less than the first by(a) 7% (b) 4%(c) 3% (d) 10%

107. Walking at 6th7

of his usual speed a man is 25

minutes too late. His usual time to cover thisdistance is(a) 2 hours 30 minutes(b) 2 hours 15 minutes(c) 2 hours 25 minutes(d) 2 hours 10 minutes

108. Walking at 5 km/hr a student reaches his schoolfrom his house 15 minutes early and walking at 3km/hr he is late by 9 minutes. What is the distancebetween his school and his house?(a) 5 km (b) 8 km(c) 3 km (d) 2 km

109. A sum of ̀ 12,000 deposited at compound interestbecomes double after 5 years. After 20 years, itwill become(a) ` 48,000 (b) ` 96,000(c) ` 1,90,000 (d) ` 1,92,000

110. Simple interest on a certain sum for 6 years is 925

of the sum. The rate of interest is

(a) 6% (b)1

6 %2

(c) 8% (d)18 %2

111. The difference between the compound interestand simple interest for the amount ` 5,000 in 2years is ` 32. The rate of interest is(a) 5% (b) 8%(c) 10% (d) 12%

112. The least number, which is to be added to thegreatest number of 4 digits so that the sum maybe divisible by 345, is(a) 50 (b) 6(c) 60 (d) 5

113. If 4 3 5 2

48 18++

= a + b 6, then the values of a

and b are respectively

(a)9 4

,15 15- (b)

3 4,

11 33

(c)9 2,

10 5(d)

3 4,5 15

114. If 17200 is divided by 18, the remainder is(a) 1 (b) 2(c) 16 (d) 17

115. If 2

43 756

++

+

x = 10, then the value of x is

(a)1276135

(b)536

(c) 4.35 (d) 9116. The unit digit in the sum of (124)372 + (124)373 is

(a) 5 (b) 4(c) 2 (d) 0

117. The value of 1 1 1

33 3 3 3 3

+ + ++ - is

(a) 3 3+ (b) 3(c) 1 (d) 0

118. If a * b = ab, then the value of 5 * 3 is(a) 125 (b) 243(c) 53 (d) 15

119. If 1 .5x = 0.04y, then the value of 2 2

2 22-

+ +y x

y xy x

is

(a)73077

(b)7377

(c)73

770(d)

7477

yoursmahboob.w

ordpress.com

72 SSC CGL SolvedPaper

120. If 0.03 0.3´ a = 0.3 × 0.3 × b value of ab

is

(a) 0.009 (b) 0.03(c) 0.09 (d) 0.08

121. If 131

9 3+ =x

, then the value of x is

(a)1439

9(b) 160

(c)1443

9(d) 169

122. The sum of two numbers is 24 and their productis 143. The sum of their squares is(a) 296 (b) 295(c) 290 (d) 228

123. Which one of the following will completely divide571 + 572 + 573?

(a) 150 (b) 160(c) 155 (d) 30

124. L.C.M. of two numbers is 120 and their H.C.F. is10. Which of the following can be the sum ofthose two numbers?(a) 140 (b) 80(c) 60 (d) 70

125. When 'n' is divisible by 5 the remainder is 2. Whatis the remainder when n2 is divided by 5?(a) 2 (b) 3(c) 1 (d) 4

126. A student was asked to divide a number by 6 andadd 12 to the quotient. He, however, first added12 to the number and then divided it by 6, getting112 as the answer. The correct answer shouldhave been(a) 124 (b) 122(c) 118 (d) 114

127. Four runners started running simultaneouslyfrom a point on a circular track. They took 200seconds, 300 seconds, 360 seconds and 450seconds to complete one round. After how muchtime they meet at the starting point for the firsttime?(a) 1800 seconds (b) 3600 seconds(c) 2400 seconds (d) 4800 seconds

128. 'x' number of men can finish a piece of work in 30days. If there were 6 men more, the work could befinished in 10 days less. The original number ofmen is(a) 6 (b) 10(c) 12 (d) 15

129. A work can be completed by P and Q in 12 days,Q and R in 15 days, R and P in 20 days. In howmany days P alone can finish the work?(a) 10 (b) 20(c) 30 (d) 60

130. A is thrice as good a workman as B and is,therefore, able to finish a piece of work in 60 daysless than B. The time (in days) in which they cando it working together is

(a) 22 (b)1222

(c) 23 (d)1234

131. A copper wire is bent in the form of square withan area of 121 cm2. It the same wire is bent in theform of a circle, the radius (in cum) of the circle is

(Take p = 227 )

(a) 7 (b) 10(c) 11 (d) 14

132. The areas of three consecutive faces of a cuboidare 12 cm2, 20 cm2 and 15 cm2, then the volume(in cm3) of the cuboid is(a) 3600 (b) 100(c) 80 (d) 60

133. Water is flowing at the rate of 5 km/h through apipe of diameter 14cm into a rectangular tankwhich is 50 m long, 44m wide. The time taken, inhours, for the rise in the level of water in the tankto be 7 cm is

(a) 2 (b)112

(c) 3 (d)122

134. The wheel of a motor car makes 1000 revolutionsin moving 440 m. The diameter (in metre) of thewheel is(a) 0.44 (b) 0.14(c) 0.24 (d) 0.34

yoursmahboob.w

ordpress.com

73SSC CGL SolvedPaper

135. The sides of a triangles are in the ratio 2:3:4. theperimeter of the triangle is 18cm. The area (in cm2)of the triangle is(a) 9 (b) 36(c) 42 (d) 3 15

136. Marked price of an article is `275. Shopkeeperallows a discount of 5% and he gets a profit of4.5%. The actual cost of the article is(a) 250 (b) 225(c) 215 (d) 210

137. The difference between a discount of 40% on`500 and two successive discounts of 36%, 4%on the same amount is(a) ` 0 (b) ` 2(c) ` 1.93 (d) ` 7.20

138. If x : y = 4 : 5, then(3x + y) : (5x + 3y) =(a) 3 : 5 (b) 5 : 3(c) 17 : 35 (d) 35 : 17

139. The ratio of the quantities of an acid and water ina mixture is 1 : 3. If 5 litres of acid is further addedto the mixture, the new ratio becomes 1 : 2. Thequantity of new mixture in litres is(a) 32 (b) 40(c) 42 (d) 45

140. The ratio between two numbers is 2 : 3. If eachnumber is increased by 4, the ratio between thembecomes 5 : 7. The difference between thenumbers is(a) 8 (b) 6(c) 4 (d) 2

141. Monthly incomes of A and B are in the ratio of 4: 3 and their expenses bear the ratio 3 : 2. Each ofthem saves ̀ 6,000 at the end of the month, thenthe monthly income of A is(a) ` 12,000 (b) ` 24,000(c) ` 30,000 (d) ` 60,000

142. The average of three consecutive odd numbersis 12 more than one third of the first of thesenumbers. What is the last of the three numbers ?(a) 15 (b) 17(c) 19 (d) Data inadequate

143. The average of 18 observations is recorded as124. Later it was found that two observations withvalues 64 and 28 were entered wrongly as 46 and82. Find the correct average of the 18observations.

(a)7

1119

(b) 122

(c) 123 (d)3

1377

144. If the cost price of 15 articles is equal to the sellingprice of 12 articles, find gain %(a) 20 (b) 25(c) 18 (d) 21

145. The cost price of an article is 64% of the markedprice. The gain percentage after allowing adiscount of 12% on the marked price is(a) 37.5% (b) 48%(c) 50.5% (d) 52%

146. A man purchased some eggs at 3 for ` 5 and soldthem at 5 for ̀ 12. Thus he gained ̀ 143 in all. Thenumber of eggs he bought is(a) 210 (b) 200(c) 195 (d) 190

Direction: The following graph shows the productionof cotton bales of 100 kg each in lakhs by differentstates A, B, C, D and E over the years. Study the graphand answer Question Nos. 147 to 150.

2003-2004

2005-2006

2006-2007

8

18

27

13

21 21

6

11

1517

8

13

510

7

282624222018161412108642

States

147. The production of State C in 2003–2004 is howmany times its production in 2005–2006?(a) 2.5 (b) 1.85(c) 1.5 (d) 0.4

148. In which State(s) is there a steady increase in theproduction of cotton during the given period?(a) A and B (b) B and D(c) A and C (d) D and E

yoursmahboob.w

ordpress.com

74 SSC CGL SolvedPaper

149. How many kg of cotton was produced by State Cdurig the given period?(a) 32,00,00,000 kg (b) 42,50,00,000 kg(c) 33,00,00,000 kg (d) 35,00,00,000 kg

150. The number of States for which the productionof cotton in 2005–2006 is less than or equal tothe preceding year is(a) 3(b) 2(c) 1(d) There is no such states

PART-D : ENGLISH COMPREHENSION

Directions: In question nos. 151 to 155, groups offour words are given. In each group, one word iscorrectly spelt. Find the correctly spelt word andmark your answer in the Answer Sheet.

151. (a) qestalt (b) imbrolios(c) ampasse (d) recondite

152. (a) hindrance (b) corespondence(c) insurence (d) assurence

153. (a) adversery (b) adultary(c) advisory (d) arbitary

154. (a) rogeu (b) colleague(c) diluge (d) atege

155. (a) malignant (b) impertinant(c) independant (d) neglegent

Directions: In the following passage (156–165),some of the words have been left out. First read thepassage over and try to understand what it is about.Then fill in the blanks with the help of the alternativesgiven. Mark your answer in the Answer Sheet.

Passage (Q no. 156–165)Almost every full moon night, the officials in Andamanand Nicobar Islands take part in a cautious ritual. Thetribesmen watch from a safe distance as the officials156 the island in a boat 157 gifts for them. Theislanders come 158 hesitantly only after the officials159 dumbed the coconuts brought 160 them ontothe beach and begin 161 away from their smallisland. On some nights the tribals even 162 upenough courage to swim upto a few feet 163 fromthe boat. The officials maintain that they do 164want to interfere with the way of life of the tribals. Theisland 165 heavily on the mainland for most goods.

156. (a) approach (b) depart(c) present (d) absent

157. (a) coming (b) leaving(c) carrying (d) relieving

158. (a) forward (b) backward(c) sides (d) upward

159. (a) has (b) have(c) had (d) be

160. (a) by (b) to(c) in (d) for

161. (a) watching (b) seeing(c) sailing (d) stopping

162. (a) leave (b) muster(c) come (d) present

163. (a) correctly (b) distantly(c) near (d) away

164. (a) no (b) neither(c) not (d) nor

165. (a) spy (b) rely(c) occupy (d) reply

Directions: In question nos 166 to 170, some of thesentences have errors and some are correct. Findout which part of a sentence has an error and morethe appropriate letter (a, b, c). If a sentence is freefrom errors, blacken the rectangle corresponding to(d) in the Answer Sheet.

166.A great many student

/(a)

havebeendeclared successful/ /

(b) (c)

Noerror(d)

167.We are going to launch

/(a)

this three crores project/(b)

within the next few months Noerror/

(c) (d)

168.I hope to go to shopping this weekend

/(a) (b)

if the weather permits Noerror/

(c) (d)

yoursmahboob.w

ordpress.com

75SSC CGL SolvedPaper

169.The lawyer asked if it was worth to take

/ /(a) (b)

thematter to court Noerror/

(c) (d)

170.After a carefully investigation

/(a)

we discovered/

(B)

that the house was infested with termites/

(c)

Noerror(d)

Directions: In question nos 171 to 175, Sentencesare given with blanks to be filled in with a givenwith blanks to be filled in with an appropriateword(s). Four alternatives are suggested for eachquestion. Choose the correct alternative out of thefour.

171. The company let me _______ time off work.(a) take (b) taking(c) to take (d) took

172. I assume _______ with me.(a) every one agreeing(b) that every one agrees(c) every one to agree(d) that every one to agree

173. _______ the rain forests is very important, if wedo not want the flora and fauna found there tobecome extinct.(a) Reserving (b) Destroying(c) Preserving (d) Maintaining

174. If I had helped him, he _______.(a) will not be drowned(b) would not be drowned(c) will not have drowned(d) would not have drowned

175. When will you head _______ your assignment?(a) in (b) back(c) down (d) into

Directions: In question nos. 176 to 180, out of thefour alternatives, choose the one which best expressesthe meaning of the given word and mark it in theAnswer Sheet.

176. Barren(a) good (b) wholesome(c) unproductive (d) profitable

177. Infamy(a) notoriety (b) glory(c) integrity (d) familiarity

178. Intrepid(a) hesitant (b) fearless(c) extrovert (d) rash

179. Prodigal(a) exclusive (b) productive(c) lavish (d) carefree

180. Perspicuous(a) relevant (b) precise(c) brief (d) clear

Dirctions: In question nos. 181 to 185, choose theword opposite in meaning to the given word andmark it in the Answer Sheet.

181. Liberty(a) serenity (b) slavery(c) serfdom (d) subordination

182. Disorderly(a) chaotic (b) organized(c) adjusted (d) arranged

183. Elevation(a) reduction (b) humiliation(c) depression (d) debasement

184. Glossy(a) dull (b) shining(c) weary (d) tired

185. Appropriate(a) dissimilar (b) incomparable(c) unsuitable (d) disparate

Directions : In question nos. 186 to 190, fouralternatives are given for the given Idiom/Phrase.Choose the alternative which best expresses themeaning of the Idiom/Phrase and mark it in theAnswer Book.

186. a damp squib(a) rainy weather(b) a disappointing result(c) a skirt in a laundry(d) None of the above

187. in cold blood(a) angrily (b) deliberately(c) excitedly (d) slowly

yoursmahboob.w

ordpress.com

76 SSC CGL SolvedPaper

188. to take someone for a ride(a) to give a ride to someone(b) to deceive someone(c) to be indifferent(d) to disclose a secret

189. to move heaven and earth(a) to cause an earthquake(b) to try everything possible(c) to pray to all Gods(d) to tavel in a rocket

190. to smell a rat(a) to smell foul(b) to see a rat(c) to chase a rat(d) to be suspicious

Directions : In question nos. 191 to 195, a sentence isgiven which/ a part of which may need improvement.Alternatives are given at (a), (b) and (c) below whichmay be a better option. In case no improvement isneeded your answer is (d). Blacken the.

191. What do you for go to school?(a) For what do you go to school?(b) What do you go for to school?(c) What do you go to school for?(d) No improvement

192. He pleased the directors and this completed hisreport in good time.(a) He pleased the directors in good time and

this completed his report.(b) He completed his report in good time and

this pleased the directors.(c) He pleased the directors and completed his

report and this in good time.(d) No improvement

193. The courtiers used to tell the King how efficientan administrator he was all day long.

(a) The courtiers all day long used to tell theKing how efficient an administrator he was.

(b) The courtiers used all day long to tell theKing how efficient an administrator he was.

(c) The courtiers used to tell the King all daylong how efficient an administrator he was.

(d) No improvement194. Every Saturday I go out for shopping

(a) for shops (b) to shopping(c) for shop (d) No improvement

195. We had a grand party and we enjoyed very much.(a) We had a grand party and enjoyed very

much.(b) We had a grand party to enjoy very much.(c) We had a grand party and we enjoyed

ourselves very much.(d) No improvement.

Directions: In question nos. 196 to 200, out of thefour alternatives, choose the one which can besubstituted for the given words/sentence.

196. An inscription on a tomb(a) espionage (b) epilogue(c) epitaph (d) elegy

197. Feeling inside you which tells you what is rightand what is wrong :(a) cleaverness (b) conscience(c) consciousness (d) fear

198. Release of a prisoner from jail on certain termsand condition(a) Parole (b) Parley(c) Pardon (d) Acquittal

199. Loss of memory(a) Ambrosia (b) Amnesia(c) Insomnia (d) Forgetting

200. To struggle helplessly(a) Flounder (b) Founder(c) Fumble (d) Finger

yoursmahboob.w

ordpress.com

77SSC CGL SolvedPaper

1. (c) As, M E K L F

9 1 7 8 2¯ ¯ ¯ ¯ ¯

and

L L L J K

8 8 8 6 7¯ ¯ ¯ ¯ ¯

Therefore,I G H E D

5 3 4 1 0¯ ¯ ¯ ¯ ¯

(Here, E = 1, F = 2, G = 3, So on.)2. (b) Since some of Murphy radios are sold in

that shop which sells high standard radios.Therefore, some of the Murphy radios areof high standard.

3. (c) Column wiseFirst Column,(7)2 + (4)2 + (2)2 = 49 + 16 + 4 = 69Second Column,(3)2 + (9)2 + (1)2 = 9 + 81 + 1 = 91Similarly, In third column,

(2)2 + (6)2 + (5)2 = 4 + 36 + 25 = 65

4. (b)

8 = (2)3? = (8)3

= 512

216 = (6)3 64 = (4)3

5. (c) The pattern is as follows :4 + 4 = 88 + 8 = 16

16 + 12 = 2828 + 16 = 4444 + 20 = 64

6. (c) -Þ ¸ +Þ ´¸Þ - ´ ¸ +

Option (c), 19 ̧ 5 + 4 – 2 × 4 = 1319 – 5 × 4 ̧ 2 + 4 = 1319 – 5 × 2 + 4 = 1319 – 10 + 4 = 1323 – 10 = 1313 = 13

7. (d) President is the head of Union Executiveof a India. Similarly, Governor is the headof State Executive.

8 (c) Mirage is an illusion caused by hot airconditions making one see something thatis not there, especially the appearance ofa sheet of water on a hot road or in a desert.Rainbow is an arch of seven coloursformed in the sky when the sun shinesthrough rain.

9. (b) The letters have been written in reverseorder.TEKCAR Þ RACKETSimilarly,TCEJBO Þ OBJECT

10. (d) First three letters are consecutive lettersbut in reverse order and the fourth lettercomes immediately after the first letter inthe English alphabetical series.

1 1 3

1 1 3

1 1 3

1 1 3

J I H K

P O N Q

W V U X

M L K N

- - +

- - +

- - +

- - +

¾¾® ¾¾® ¾¾®

¾¾® ¾¾® ¾¾®

¾¾® ¾¾® ¾¾®

¾¾® ¾¾® ¾¾®

11. (d) As, 2 2 2 2

U U W X

W W Y Z+ ¯ + ¯ + ¯ + ¯

Similarly, 2 2 2 2

Q Q S T

O O Q R+ ¯ + ¯ + ¯ + ¯

HINTS & SOLUTIONS

yoursmahboob.w

ordpress.com

78 SSC CGL SolvedPaper

12. (a) As,

B I M N C K P R+1

+2+3

+4

Similarly,

C U R D D W U H+1

+2+3

+4

13. (c) As,

B C F E

+ +1

: H I L K

–1+1

+2+2

Similarly,

N O R Q

+1

+2

: T U X W

–1+1

+2

+114. (d) 42 – 1 = 15

43 – 1 = 6332 – 1 = 833 – 1 = 26

15. (b) 7 + 2 = 9; 5 + 3 = 89 – 8 = 16 + 4 = 10; 5 + 4 = 910 – 9 = 1

16. (d) Basketball is different from others. InSoftball, Baseball and Cricket, bats andballs are used, Basketball involves onlyball.

17. (d) Ravage is the different from the otherwords.

18. (a) Here,2 1 2

2 2 2

2 2 2

2 2 2

P R S U

S U W Y

H J L N

C E G I

+ + +

+ + +

+ + +

+ + +

¾¾® ¾¾® ¾¾®

¾¾® ¾¾® ¾¾®

¾¾® ¾¾® ¾¾®

¾¾® ¾¾® ¾¾®

19. (c) Here,

Y D W B

–2

–2

T K R I

–2

–2

Q N O M

–1

–2

H L F J

–2

–220. (a) Except Tomato all others are grown

underground.21. (a) The number 125 is a perfect cube.

5 × 5 × 5 = 12522. (d) As 2 × 8 = 16; 2 × 9 = 18

7 × 8 = 56; 7 × 9 = 6312 × 8 = 96; 12 × 9 = 108

868

= 10.75; 999

= 11

Also, only (d) is coprime.23. (d) Only in option (d) the rule is followed

C 3F 6 (Forward)I 9X 3U 6 (Backward)R 9

®®®®®®

i.e.3 3 6 6 9 9C X F U I R

24. (c) Meaningful order of the words :3. Chapter

¯5. Paragraph

¯1. Sentence

¯4. Phrase

¯2. Word

yoursmahboob.w

ordpress.com

79SSC CGL SolvedPaper

25. (d) Arrangement of the words as perdictionary

5. Infatuation ¯

4. Influence ¯

2. Ingenious ¯

1. Inhabit ¯

3. Inherit

26. (d) a a b a b / a a b a b / a ab ab27. (d) The pattern is as follows :

14 19 29 49 89 169

+5 +10 +20 +40 +80

+5 +10 +20 +40

+5 +10 +20

+5 +10

+5

So, 169 will complete the series.28. (a) The pattern is as follows :

2 2 2 2

1 1 1 1

2 2 2 2

1 1 1 1

B D F H

M L K J

R T V X

G F E D

+ + + +

- - - -

+ + + +

- - - -

¾¾® ¾¾® ¾¾® ¾¾®

¾¾® ¾¾® ¾¾® ¾¾®

¾¾® ¾¾® ¾¾® ¾¾®

¾¾® ¾¾® ¾¾® ¾¾®

2 2 2 2

1 1 1 1

2 2 2 2

1 1 1 1

B D F H

M L K J

R T V X

G F E D

+ + + +

- - - -

+ + + +

- - - -

¾¾® ¾¾® ¾¾® ¾¾®

¾¾® ¾¾® ¾¾® ¾¾®

¾¾® ¾¾® ¾¾® ¾¾®

¾¾® ¾¾® ¾¾® ¾¾®

JIZC

So, JIZC will complete the series.29. (b) The pattern is as follows :

121 144 289 324 529 576 841

(11)2 (12)2 (17)2 (18)2 (23)2 (24)2 (29)2

® ® ® ® ® ® ®

So, 841 will complete the series.30. (b) The pattern is as follows :

21 – 5 = 16 Þ 16 × 3 = 4869 – 21 = 48 Þ 48 × 3 = 144213 – 69 = 144 Þ 144 × 3 Þ 432645 – 213 Þ 432 Þ 432 × 3 = 1296and

1941 – 645 = 1296

31. (c) As, 6 = 3 × 2 Similarly, 8 = 4 × 2

3 + 2 = 5 4 + 2 = 6 .32. (c) Total number of trees in the row = 14 + 7 –

1 = 2033. (a) Age of Shan = 55 years

Age of Sathian = 55 – 5 = 50 yearsAge of Balan = 50 – 6 = 44 yearsAge of Devan = 44 – 7 = 37 yearsDifference between the ages of Shan andDevan = 55 – 37 = 18 years.

34. (a) Wife of Vinod’s father means the motherof Vinod.Only brother of Vinod’s mother meansmaternal uncle of Vinod.Therefore, Vinod is cousin of Vishal.

35. (b) 20% of 80 = 20

80100´ = 16

50% of remaining

= (80 – 16) × 50

100 = 32

The families which do not own any vehicle= 80 – (32 + 16)= 80 – 48 = 32

36. (b) 5 January 1965 Þ Tuesday5 January 1966 Þ Wednesday5 January 1967 Þ Thursday5 January 1968 Þ Friday5 January 1969 Þ SundaySince, 1968 is a leap year.5 January 1970 Þ Monday5 January 1971 Þ Tuesday

37. (c) The arrangement is as follows :

M N Q P OLEFT

RIGHT

So, O is standing on the extreme right.38. (c) Sita > Swapna.......................... (i)

S SwS > Lavanya > Sw .......................... (ii)

LHari, Sw > Suvarna .......................... (iii) H SuSw > H .......................... (iv)From all the statements :S > L > Sw > H > Su

yoursmahboob.w

ordpress.com

80 SSC CGL SolvedPaper

39. (c) At 9 O’clock, the minute hand is 9 × 5 = 45minute – spaces behind the hour hand.Therefore, the minute hand will have togain 45 – 30 = 10 minute space over thehour hand.Q Gain of 55 minute spaces equal 60minutes.\ Gain of 15 minute spaces equals

= 60 180 4

15 1655 11 11´ = =

Therefore, hour and minute hands of aclock point in opposite direction after 9

O’clock at 416

11 minutes past 9.

40. (a) The direction diagram is as follows :N

S

EW

SunStartingpoint

So, raja is facing south direction.41. (b) The direction diagram is as follows :

B 4 kmC

3 km

A5 km

3 km

5 km

Ashok/Vivek

N

S

EW

Here, C’b = B’A = 5 kmSo, C’B + BC = 5 + 4 = 9 kmTherefore, they are 9 km fare aprt eachother.

42. (d) There is no ‘A’ letter in the given word.

43. (c) STRA N G ULA T I O N44. (d) As,

R A M A Y A N A2 2 2 2 2 2 2 2P Y K Y W Y L Y- ¯ - ¯ - ¯ - ¯ - ¯ - ¯ - ¯ - ¯

Similarly,

2 2 2 2 2 2 2 2 2 2 2

M A H A B H A R A T A

K Y F Y Z F Y P Y R Y- ¯ - ¯ - ¯ - ¯ - ¯ - ¯ - ¯ - ¯ - ¯ - ¯ - ¯

45. (c) 9 × 4 + 1 × 6 = 36 + 6 = 428 × 9 + 2 × 3 = 72 + 6 = 78Similarly

6 × 3 + 4 × 5 = 18 + 20 = 38

46. (c) Judge is different from both the thief andcriminal.The thief comes under the class criminal.

Judge

Criminal

Thief

47. (a) A E B

H FI

C DThe triangles are :D AIH; DAIE; DEIB; DBFI;DIHC; DIGC; DIGD; DDFI;DIAB; DIBD; DICD; DIAC;DBAC; DACD; DBDC; BDATotal triangles = 16

48. (c)

49. (b) 50. (c) 51. (c) 52. (d) 53. (a)54. (a) 55. (c) 56. (b) 57. (a) 58. (c)59. (d) 60. (c) 61. (c) 62. (d) 63. (c)64. (c) 65. (c) 66. (d) 67. (a) 68. (b)69. (c) 70. (b) 71. (a) 72. (b) 73. (b)74. (c) 75. (c) 76. (a) 77. (b) 78. (c)79. (d) 80. (c) 81. (c) 82. (c) 83. (a)84. (d) 85. (a) 86. (b) 87. (c) 88. (d)89. (a) 90. (c) 91. (d) 92. (c) 93. (c)94. (b) 95. (a) 96. (c) 97. (d) 98. (c)99. (b) 100. (a)101. (d) SP of both articles is same. Profit on one is

equal to loss on the other.If loss per cent be x, then

25 – x – 25x100

= 0

yoursmahboob.w

ordpress.com

81SSC CGL SolvedPaper

Þ 25 – x – x4

= 0 Þ 100 – 4x – x = 0

Þ 5x = 100Þ x = 20

102. (c) Let SP = 100Loss% on SP = 20%CP = 100 + 20 = 120

L% of cp = 20 2

100 16120 3´ =

103. (b) A × 90 B 30

100 100´

=

Þ A × 3 = BÞ A × x% = A × 3

Þ x

100 = 3 Þ x = 300

104. (b) Let CP = x, Total ̀ = 600, Sugar bought

= 600

x

ATQ 80x 600

5100 xé ù+ê úë û

= 600

480 + 4x = 6004x = 120x = 30

105. (b) Percentage increase

= 7.50 6 100

6- ´ = 25

\ Percentage decrease in consumption

= 25 100

125´ = 20%

106. (d) Let the third number = 100.First number = 70Second number = 63\ Required per cent

= 70 63 100

70- ´ = 10%

107. (a)2

1

S 6S 7

= 2

1

T 7T 6

=

\ 7x – 6x = 25x = 25 min

T1 6x = 25660´ hr

= 2hr 30 min

108. (c) Let the required distance be x km.

\ x x 243 5 60- =

Þ 5x 3x 2

15 5- = Þ

2x3

= 2

Þ 2x = 2 × 3 Þ x = 3 km

109. (d) A= P TR1

100æ ö+ç ÷è ø

Þ TA R1

P 100æ ö= +ç ÷è ø

Þ 2 = 5R1

100æ ö+ç ÷è ø

Þ 24 = 20R1

100æ ö+ç ÷è ø Þ 16 =

20R1100

æ ö+ç ÷è øHence, the principal will become 16 timesi.e.` (16 × 12000)

= ̀ 192000

110. (a) Rate = SI 100

Principal Time´´

= 9 10025 6´ = 6% per annum

111. (b) Difference of 2 years2rP

100æ öç ÷è ø

Þ 32 = 25000 r

10000´

Þ r2 = 32 10000

5000´

= 64

Þ r = 64 = 8%112. (b) The largest 4-digit number = 9999

345)9999(28690

30992760339

\ Required number = 345 – 339 = 6

yoursmahboob.w

ordpress.com

82 SSC CGL SolvedPaper

113. (d)4 3 5 2

48 18++

4 3 3 2 2 24 3 3 2

+ +Þ+

1 2 24 3 3 2

+Þ+

By Rationaizing

1 2 2(4 3 3 2)(4 3 3 2)(4 3 3 2)+ -+ -

1 8 6 1248 18+ --

1 8 6 1230

+ -

30 12 8 630

- +

18 18 6 a b 630 303 4 6 a b 65 15

3 4a , b5 15

+ = +

+ = +

= =

114. (a) Remainder when (x – 1)n is divided by x is(–1)n

\ (17)200 = (18 – 1)200

\ Remainder = (–1)200 = 1

115. (a) 2x 4330 76

++

+

= 10

2x 4 6337

Þ + ´+

2x 1024337

Þ + =+

Þ x + 2

111 2437+ = 10

Þ x + 2 37135´

= 10

Þ x + 74

135 = 10

Þ x = 10 – 74 1350 74

135 135-

= = 1276135

116. (d) 41 = 4; 42 = 16; 43 = 64; 44 = 256; 45 = 1024Remainder on dividing 372 by 4 = 0Remainder on dividing 373 by 4 = 1\ Required unit digit= Unit’s digit of the sum = 6 + 4 = 0

117. (b)1 1 133 3 3 3 3æ ö+ + -ç ÷è ø+ -

= 1 3 3 3 333 (3 3)(3 3)

æ ö- - -+ + ç ÷+ -è ø

= 1 2 33

9 33-+ +-

= 1 3 1 13 3 3

33 3 3+ - = + - =

118. (a) a * b = ab

\ 5 * 3 = 53 = 5 × 5 × 5 = 125119. (b) 1.5x = 0.04y

= x 0.04 4 2y 1.5 150 75

= = =

Þ y 75x 2

=

Now, 2 2

2 2y x

y 2xy x-

+ +

= 2(y x)(y x)

(y x)- +

+

=

y 1y x xyy x 1x

-- =+ +

=

75 1 73275 7712

-=

+

120. (*) 0.03 0.3 a´ ´= 0.3 × 0.3 bOn squaring,0.03 × 0.3 × a = 0.09 × 0.09 × b

Þ a 0.09 0.09b 0.03 0.3

´=´

= 0.9

yoursmahboob.w

ordpress.com

83SSC CGL SolvedPaper

121. (b)x 1319 3

+ =

Squaring on both sides,x 169

19 9

+ =

Þ x 169 160

19 9 9= - =

Þ x = 160122. (c) Let the two numbers be x and y.

\ x + y = 24and, xy = 143\ x2 + y2 = (x + y)2 – 2xy= (24)2 – 2 × 143= 576 – 286 = 290

123. (c) 571 + 572 + 573

= 571 (1 + 5 + 52)= 570 × 5 × 31= 571 × 155 which is exactly divisible by155.

124. (d) Let the numbers be 10x and 10y where xand y are prime to each other.\ LCM = 10 xyÞ 10xy = 120Þ xy = 12Posssible pairs = (3, 4) or (1, 12)\ Sum of the numbers = 30 + 40 = 70

125. (d) Required remainder = Remainder obtainedby dividing 22 by 5.Remainder = 4

126. (b) Let the number be x

\ x 12

6+

= 112

Þ x + 12 = 672Þ x = 672 – 12 = 660

\ Correct answer = 660 12

6+

= 110 + 12 = 122127. (a) Required time = LCM of 200, 300, 360 and

450 seconds = 1800 seconds.128. (c) m1d1 = m2d2

x (30) = (x + 6) 20Þ 2x + 12 = 3xÞ 3x – 2x = 12Þ x = 12 men

129. (c) (P + Q)’s 1 day’s work

= 1

12...(i)

(Q + R)’s 1 day’s work = 1

15...(ii)

(R + P)’s 1 day’s work = 1

120...(iii)

Adding all three equations, 2 (P + Q + R)’s1 day’s work

= 1 1 1 5 4 3

12 15 20 60+ +

+ + =

= 12 160 5

=

\ (P + Q + R)’s 1 day’s work = 1

10...(iV)

\ P’s 1 day’s work= Equation (iv) – equation (ii)

= 1 1 3 2 1

10 15 30 30-

- = =

\ P alone will complete the work in 30 days,130. (b) If a completes the work in x days, B will do

the same in 3x days.\ 3x – x = 60Þ 2x = 60Þ x = 30 and 3x = 90\ (A + B)’s day’s work

= 1 1 3 130 90 90

++ = =

4 290 45

=

\ A and B together will do the work in

45 1222 2

= days.

131. (a) Side of square = 121 = 11 cm\ Length of wire = 4 × 11 = 44 cm\ 2pr = 44

Þ 2 × 22

r7´ = 44

Þ n = 44 72 22´´

= 7 cm

132. (d) If the length, breadth and height of thecuboid be x, y and z cm respectively, thenxy = 12; yz = 20; zx = 15\ x2y2z2 = 12 × 20 × 15= 3600 cm6

\ n = xyz = 3600 = 60 cm3

133. (a) Water flowed by the pipe in 1 hr.= pr2h

yoursmahboob.w

ordpress.com

84 SSC CGL SolvedPaper

= 22 7 7

50007 100 100

´´ ´

´ metre3

= 77 m3

Volume of expected water in the tank

Þ 50 44 7

100´ ´

= 154 m3

\ Required time

= 15477

= 2 hr..

134. (b) Distance covered by wheel in onerevolution= Circumference of wheel

\ p × diameter = 440

1000

Þ 227

× diameter = 440

1000

Þ Diameter = 440 7

1000 22´

= 0.14 cm135. (d) Ratio = 2 : 3 : 4

= 4 : 6 : 8Perimeter = 18 cm

\ Semi-perimeter(s) = 4 6 8

2+ +

= 9

\ Area of triangle

= s(s a)(s b)(s c)- - -

= 9(9 4)(9 6)(9 8)- - -

= 9 5 3 1 3 15´ ´ ´ = sq. cm.136. (a) MP = 275

SP after Discount of 5% = 95

275100´

CP where P % of 4.5 = 100 95

275104.5 100

´ ´

= ̀ 250137. (d) Single equivalent discount for 36% and 4%

= 36 4

36 4100´æ ö+ -ç ÷è ø

= (40 – 1.44)% = 38.56%\ Required difference = 1.44% of 500

= 500 1.44

100´

= ̀ 7.20

138. (c)x 4y 5

=

\

x3 1y3x y

5x 3y x5 3y

æ ö+ç ÷è ø+ =

+ æ ö+ç ÷è ø

=

4 12 53 15 54 75 35

+´ +=

´ + =

1735

139. (d) Let the quantity of acid in original mixturebe x litre and that of water be 3x litre.

\ x 5 13x 2+ =

Þ 2x + 10 = 3x Þ x = 10\ Quantity of new mixture= 4x + 5 = 45 litres

140. (a) Let the numbers be 2x and 3x.

\ 2x 4 53x 4 7

+ =+

\ 15x + 20 = 14x + 28Þ x = 28 – 20 = 8 = Required Difference

141. (b) Let salary of A and B be = 4x and 3xexpenditure = income-salaryATQ4x 6000 33x 6000 2- =-

8x – 12000 = 9x – 18000x = 6000A’s salary = 4x = 4 × 6000 = 24000

142. (b) Let 3 consecutive odd no. be x, x + 2 andx + 4 ATQx x 2 x 4 112 x

3 3+ + + + = +

3x 6 x3 3+- = 12

= 2x + 6 = 36 x = 36 6

2-

= 15

last no = 15 + 4 = 19143. (b) Difference in observations

= 64 + 28 – 46 – 82 = –36\ Correct average

= 3612418- = 122

yoursmahboob.w

ordpress.com

85SSC CGL SolvedPaper

144. (b) Percentage profit

= 15 12 100

12- ´ = 25

145. (a) Marked price of article = ` 100 (let)\ C.P. of article = ` 64\ S.P. of article = ̀ 88\ Profit per cent

= 88 64

10064-´ = 37.5%

146. (c) Let he buy 15 eggs.\ CP of 15 15 eggs = ̀ 25\ SP of 15 eggs = ̀ 36\ Gain = 36 – 25 = ̀ 11Q ` 11 º = 15 eggs

\ ` 143 º 15 14311´

= 195 eggs.

147. (d) Required answer = 6

15 = 0.4

148. (c) It is obvious from the graph.149. (a) Total cotton production in state

C = (6 + 11 + 15) lac × 100 kg = 320000000 kg150. (b)151. (b) recondite (imbroglios, empasse, crystal)152. (a) hindrance (correspondence, insurance,

assurance)153. (c) advisory (adversary, adultery, arbitrary)154. (b) colleague (rogue, dialogue, allege)155. (a) malignant (impertinent: independent

negligent).156. (a) 157. (c) 158. (a) 159. (b) 160. (d)161. (c) 152. (b) 163. (d)164. (c) 165. (b)166. (a) Here, A great many students should be

used.167. (b) In hyphenated terms. singular Noun i.e.,

this three-crore project should be used.168. (a) Here. I hope to go shopping should be

used.169. (b) Here. Gerund i.e. If it was worth taking

should be used.170. (a) Here, Adjective i.e. After a careful

investigation should be used.171. (a) 172. (b) 173. (c) 174. (d) 175. (a)176. (c) The word Barren (Adjective) means: not

good enough for plants to grow on itinfertile unproductive.

177. (a) The word Infamy (Noun) means: the stateof being well known for something bad orevil an evil act notoriety.

178. (b) The word Intrepid (Adjective) means : verybrave not afraid of danger or difficultiesfearless.

179. (c) The word Prodigal (Adjective) moans : toowilling to spend money or waste time,energy or materials extravagant lavish.

180. (b) The word Perspicuous (Adjective) meansprecise clear and accurate)

181. (b) The word Liberty (Noun) means : freedomto live as you choose without too manyrestrictions from government or authority.The word slavery (Noun) means : state ofbeing a slave: a system of legally owninganother person and forcing to work forthem.

182. (d) The word Disorderly (Adjective) means :showing lack of control untidy deranged.Its antonym should be arranged.

183. (c) The word Elevation (Noun) means theheight of a place: the process of somebodygetting a higher or more important rankpart of a surface that is higher than theparts around it.The word Depression (Noun) means : thestate of feeling very sad and without hopepart of a surface that is lower than the partsaround it.

184. (a) The word Glossy (Adjective) meansshining, smooth and shiny.Its antonym should be dull.

185. (c) The word Appropriate (Adjective) means :suitable acceptable or correct for theparticular circumstances.Its antonym should be unsuitable.

186. (b) a disappointing result187. (a) angrily188. (b) to deceive someone189. (b) to try everything possible190. (d) to be suspicious191. (c) What do you go to school for ?192. (b) He completed his report in good time and

this pleased the directors.193. (c) The courtiers used to tell the King all day

long how efficient an administrator he was.shining: smooth and

194. (d) No improvement195. (c) We had a grand party and we enjoyed

ourselves very much.196. (c) epitaph .197. (b) conscience198. (a) Parole199. (b) Amnesia200. (a) Flounder

yoursmahboob.w

ordpress.com

86 SSC CGL SolvedPaper

PART-A : GENERAL INTELLIGENCE & REASONING

Directions : In questions no. 1 to 9, select the relatedletters/word number from the given alternatives.

1. 8 : 12 : : 6 : ?(a) 8 (b) 11(c) 5 (d) 7

2. 13 : 19 : : 21 : ?(a) 41 (b) 81(c) 141 (d) 14

3. Eagle : Swoops : : Duck : ?(a) waddles (b) floats(c) swims (d) flits

4. APPLE : 50 : : ORANGE : ?(a) 60 (b) 69(c) 61 (d) 63

5. Accommodation : Rent : : Journey : ?(a) Freight (b) Octroi(c) Fare (d) Expense

6. Fire : Smoke : : ?(a) Children : School (b) Cloud : Rain(c) Moon : Sky (d) Shoe : Polish

7. Grenade : Gun : : ?(a) Sister : Brother (b) Father : Mother(c) Man : Woman (d) Head : Brain

8. TSH : IRQ: : QPK : ?(a) LNO (b) LON(c) PWK (d) PON

9. AEZ : FPY : : BGX : ?(a) HWW (b) IYY(c) HTX (d) HYW

Directions : In questions no. 10 to 17, find the oddnumber/ letters/ word/ number pair from the givenalternatives.

10. (a) SP (b) NL(c) ZW (d) TQ

11. (a) Major (b) Colonel(c) Brigadier (d) Admiral

12. (a) Life Insurance Corporation.(b) New India Assurance Company Ltd.(c) United India Insurance Company Ltd.(d) National Insurance Company Ltd.

SSC Combined Graduate Level (CGL) Solved PaperMORNING SHIFT 1 JULY, 2012

13. (a) Hurdle (b) Disease(c) Barrier (d) Obstacle

14. (a) Mar (b) Remedy(c) Maim (d) Mutilate

15. (a) Socrates (b) Beethoven(c) Mozart (d) Bach

16. (a) (132, 5) (b) (125, 8)(c) (124, 7) (d) (112, 4)

17. (a) 6246 – 6296 (b) 7137 – 7267(c) 4344 – 4684 (d) 5235 – 5465

Directions : In questions no. 18 to 22, a series isgiven, with one term missing. Choose the correctalternative from the given ones that will completethe series.

18. YX, UTS, ONML, ? .(a) FEDCB (b) GFEDC(c) IHGFE (d) HGFED

19. DA, HE, LI, ? , TQ(a) PJ (b) PT(c) PM (d) PK

20. B E I N T ? .(a) X (b) Y(c) A (d) Z

21. AZ, CX, EV, ? .(a) HT (b) HU(c) GS (d) GT

22. D9Y, J27S, P81M, V243G, ? .(a) A324B (b) C729B(c) B729A (d) A729B

23. Which one set of letters when sequentiallyplaced at the gaps in the given letter series shallcomplete it?

_c_bd_cbcda_a_db_a(a) daabbc (b) bdbcba(c) adabcd (d) cdbbca

Directions : In questions no. 24 to 27, identify thewrong number in the series.

24. 9, 19, 40, 83, 170, 340(a) 83 (b) 40(c) 340 (d) 170

yoursmahboob.w

ordpress.com

87SSC CGL SolvedPaper

25. 21, 28, 33, 35, 37, 36(a) 21 (b) 36(c) 33 (d) 35

26. 5, 13, 29, 61, 120, 253(a) 120 (b) 253(c) 61 (d) 29

27. 0, 7, 28, 63, 124, 215(a) 28 (b) 215(c) 7 (d) 63

28. Some relationships have been expressed throughsymbols which are explained below:0 = greater thanD = not equal to × = not less than+ = equal tof = not greater thanÑ = less thana Ñ b Ñ c implies(a) a D b f c (b) a f b + c(c) a 0 b + c (d) a 0 b × c

29.2 34 5

= ?

(a) 14400 (b) 15600(c) 23040 (d) 17400

30. If PEAR is written as GFDN, how is REAP writtenin this code?(a) FDNG (b) NFDG(c) DNGF (d) NDFG

31. If 54 + 43 = 2, 60 + 51 = 10, then 62 + 72 = ?(a) 30 (b) 18(c) 20 (d) 9

32. If L denotes ×M denotes ¸ ; P denotes + ; Q denotes –then 16 P 24 M 8 Q 6 M 2 L 3 = ?(a) 10 (b) 9(c) 12 (d) 11

33. In this question, from the given alternatives selectthe word which cannot be formed by using theletters of the given word.APPROPRIATE(a) PIRATE (b) APPROVE(c) PROPER (d) RAPPORT

34. If FLATTER is coded as 7238859 and MOTHERis coded as 468159, then how is MAMMOTHcoded?(a) 4344681 (b) 4344651(c) 4146481 (d) 4346481

35. If 16 – 2 = 2, 9 – 3 = 0, 81 – 1 = 8, then what is 64 – 4?(a) 4 (b) 2(c) 6 (d) 8

Directions : In questions no. 36 to 37, select themissing number from the given responses.

36.7

4 35

824159

2 281 18

99

84 5

32

37

? 2314

(a) 20 (b) 25(c) 10 (d) 15

37. 10 1511812 12

104 1210 5 1318 20 ?

(a) 21 (b) 20(c) 23 (d) 22

38. Four aeroplanes of Airforce viz, A, B, C, D, startedfor a demonstration flight towards east. Afterflying 50 km planes A and D flew towards right,planes B and C flew towards left. After 50 km,planes B and C flew towards their left, planes Aand D also towards their left. In which directionsare the aeroplanes A, B, D, C respectively flyingnow?(a) North, South, West, East(b) South, North, West, East(c) East, West, West, East(d) West, East, West, East

39. Satish start from A and walks 2 km east upto Band turns southwards and walks 1 km upto C. AtC he turns to east and walks 2 km upto D. He thenturns northwards and walks 4 km to E. How far ishe from his starting point ?(a) 5 km (b) 6 km(c) 3 km (d) 4 km

yoursmahboob.w

ordpress.com

88 SSC CGL SolvedPaper

Directions: In questions no. 40 to 41, one/twostatements are given, followed by two conclusions Iand II. You have to consider the statements to be true,even if they seem to be at variance from commonlyknown facts. You have to decide which of the givenconclusions, if any follow from the given statement.

40. Statements :1. Temple is a place of worship.2. Church is also a place of worship.Conclusions :I. Hindus and Christians use the same place

for worship.II. All churches are temples.(a) Neither conclusion I nor II follows(b) Both conclusion I nor II follows(c) Only conclusion I follows(d) Only conclusion II follows

41. Statement :The human organism grows and developsthrough stimulation and action.Conclusions :I. Inert human organism cannot grow and

develop.II. Human organisms do not react to stimulation

and action.(a) Neither conclusion I nor II follows(b) Both conclusion I nor II follows(c) Only conclusion I follows(d) Only conclusion II follows

42. If the first four letters of a termHIPPNOWADIASM are written in reverse order,the next five letters are written without changingtheir order and then, the remaining letters areagain written in reverse order, then which letter isin the middle of the word?(a) O (b) W(c) A (d) I

43. In the following letter series how many times doPQR occur in such away that Q is in the middle ofP and R.Q M P N P Q RR O P Q N O P P Q R P M Q R O PQ R P P R R P Q R P(a) 5 (b) 6(c) 4 (d) 3

44. Volume of a sphere is equal to the volume of ahemisphere. If the radius of the hemisphere is

33 2 cm, then the radius of the sphere is equal to

(a) 39 2 cm (b) 36 2 cm(c) 27 cm (d) 3 cm

45. A sheet of paper has been folded as shown bythe question figures. You have to figure out fromamongst the four answer figures how it willappear when opened?Question figures :

Answer figures :

(a) (b) (c) (d)

46. Which of the answer figure is exactly the mirrorimage of the question figure if a mirror is placedon the line MN?Question figure :

B B

M

NAnswer figures :

B B

(a) (b) (c) (d)47. If SEARCH is coded as TFBSDI, how will PENCIL

be coded ?(a) RGPEN (b) LICNEP(c) QFODJM (d) QDMBHK

48. Which answer figure completes the form inquestion figure ?Question figures :

?

Answer figures :

(a) (b) (c) (d)

yoursmahboob.w

ordpress.com

89SSC CGL SolvedPaper

49. From the answer figures, select the one in whichthe question figure is hidden/embedded.Question figure :

Answer figures :

(a) (b) (c) (d)

50. A word is represented by only one set of numbersas given in any one of the alternatives. The setsof numbers given in the alternatives arerepresented by two classes of alphabets as intwo matrices given below. The columns and rowsof Matrix I are numbered from 0 to 4 and that ofMatrix II are numbered from 5 to 9. A letter fromthese matrices can be represented first by its rowand next by its column, e.g., 'A' can berepresented by 10, 33, etc., and 'H' can berepresented by 59, 78, etc. Similarly, you have toidentify the set for the word GUIDE.

Matrix-I

0 1 2 3 40 I E A O U1 A O U I E2 E I O U A3 O U E A I4 U A I E O

Matrix-II

5 6 7 8 95 F D B G H6 B G H F D7 D F G H B8 G H D B F9 H B F G D

(a) 85, 23, 21, 87, 32 (b) 58, 31, 12, 57, 41(c) 77, 13, 42, 99, 32 (d) 66, 31, 43, 78, 14

PART-B : GENERAL AWARENESS

51. The National Commission for Minorities wasconstituted in the year(a) 1990 (b) 1992(c) 1980 (d) 1989

52. The first Indian who was chosen as the SecretaryGeneral of Commonwealth is(a) Rakesh Verma (b) Gopalaswami(c) Krishna Murthy (d) Kamalesh Sharma

53. In which of the following systems of governmentis bi–cameralism an essential feature?(a) Federal system(b) Unitary system(c) parliamentary system(d) Presidential system

54. Kuldip Nayer, a journalist, was appointed as aHigh Commissioner in(a) Sri Lanka (b) Australia(c) UK (d) Pakistan

55. Which king is referred to as DevanampiyaPiyadassi (Beloved of the Gods) in theinscriptions?(a) Asoka(b) Harsha(c) Bindusara(d) Chandragupta Maurya

56. Socialism succeeds in achieveing(a) higher standard of living of the people(b) equal distribution of income in the society(c) higher individual welfare in the society(d) maximum social welfare in the society

57. Monopolist resorts to price discriminationdepending upon the(a) Elasticity of supply(b) Elasticity of demand(c) Law of demand(d) Law of supply

58. Economic planning is an essential feature of(a) Socialist economy(b) Capitalist economy(c) Mixed economy(d) Dual economy

59. The HYV programme in India is also called as(a) Traditional Agriculture(b) New Agricultural Strategy(c) White Revolution(d) Blue Revolution

yoursmahboob.w

ordpress.com

90 SSC CGL SolvedPaper

60. The National Policy for Empowerment of Womenwas adopted in the year(a) 2001 (b) 2005(c) 1991 (d) 1995

61. Ballots were first used in(a) Australia (b) USA(c) Ancient Greece (d) England

62. The Rashtriya Barh Ayog (RBA) is related with(a) Droughts and Floods(b) Poverty Alleviation(c) Floods(d) Disaster Management

63. Which of the following criteria is not used for theclassification of human races?(a) Nose (b) Hair(c) Eyes (d) Ear

64. Railway coaches are manufactured at(a) Jamshedpur (b) Chittaranjan(c) Perambur (d) Varanasi

65. Fertilization occurs normally in the(a) Cervix (b) Vagina(c) Fallopian tube (d) Uterus

66. People consuming alcohol in heavy quantitiesgenerally die of(a) liver or stomach cancer(b) weakening of heart muscles leading to

cardiac arrest(c) blood cancer(d) Cirrhosis

67. The organisms at the base of the grazingfood-chain are(a) Carnivores (b) Decomposers(c) Producers (d) Herbivores

68. Who among the following was credited with thedestruction of 'Chihalgani', a group of powerfulnobles?(a) Balban(b) Qutb-ud-din- Aibak(c) Iltutmish(d) Razia Sultan

69. Bombay was given away as dowry to the EnglishKing Charles II for marrying the Princess of(a) France (b) Portugal(c) Holland (d) Denmark

70. The Grand Trunk Road built by Shershahconnected Punjab with(a) Agra (b) East Bengal(c) Lahore (d) Multan

71. Name the Maratha Saint who was a contemporaryof Shivaji.(a) Saint Eknath (b) Saint Tukaram(c) Saint Dhyaneshwar (d) Namdev

72. The study of lake is called(a) Topology (b) Hydrology(c) Limnology (d) Potomology

73. A series of lines connecting places having a quakeat the same time are called(a) Homoseismal lines(b) Seismolines(c) Coseismal lines(d) Isoseismal lines

74. 'Lumen' is the unit of(a) Illuminance(b) Brightness(c) Luminous flux(d) Luminous intensity

75. The transfer of date from a CPU to peripheraldevices of computer is achieved through(a) interfaces (b) buffer memory(c) modems (d) computer ports

76. Which of the following items is not used in LocalArea Networks (LANs)?(a) Interface Card (b) Cable(c) Computer (d) Modem

77. The mass of 10 moles of water is(a) 90 g (b) 45 g(c) 18 g (d) 180 g

78. Vitamin A is rich in(a) Carrot (b) Lime(c) Beans (d) Rice

79. The high boiling point of water compared tohydrogen sulphide or hydrogen chloride is dueto(a) Dipole insulation(b) Van der Waal's attraction(c) Polar covalent bonding(d) Hydrogen bonding

80. Which of the following determines the chemicalproperties of an element?(a) Number of electrons(b) Number of neutrons(c) Number of protons(d) All of the above

81. The Central Drug Research Institute of India islocated at(a) Madras (b) Lucknow(c) Delhi (d) Bangalore

yoursmahboob.w

ordpress.com

91SSC CGL SolvedPaper

82. Which of the following cereals was among thefirst to be used by man?(a) Rye (b) Wheat(c) Barley (d) Oat

83. Which of the following wheat species are beingcultivated in India?(a) Club wheat (b) Durum wheat(c) Emmer wheat (d) Bread wheat

84. Which one of the following forces is a 'dissipativeforce'?(a) Electrostatic force (b) Magnetic force(c) Gravitational force (d) Frictional force

85. If a resistive wire is elongated, its resistance(a) decreases (b) remains constant(c) increases (d) All of the above

86. If a magnet has a third pole, then the third pole iscalled(a) defective pole (b) consequent pole(c) extra pole (d) arbirary pole

87. How many spokes are there in the Dharmachakraof the National Flag?(a) 14 (b) 18(c) 22 (d) 24

88. The latest official language of the U.N. is(a) Russian (b) Arabic(c) Chinese (d) Spanish

89. Srikrishna Committee Report, which was madepublic in 2011, is related to(a) rejuvenation of higher education(b) demand for a separate Telangana State(c) ragging in educational institutions(d) impeachment of Justice Dinakaran

90. The latest book 'Kurukshetra to Kargil' is writtenby(a) Suryanath Singh (b) Kunal Bhardwaj(c) Karan Singh (d) Kuldip Singh

91. The Educational Development Index (EDI)Report, released in 2011, is led by(a) Tamil Nadu (b) Puducherry(c) Kerala (d) Lakshadweep

92. Shunglu Committee, which submitted its reportin 2011, is related by(a) Commonwealth Games scandal(b) reforms in the Insurance sector(c) revamp of Defence management(d) management of Co-operative Sector

93. The 65th National Football Championship(Santosh Trophy 2011) was won by(a) Bengal (b) Punjab(c) Manipur (d) Goa

94. The organisation involved primarily withenvironmental planning is(a) CIFRI (b) ICAR(c) CSIR (d) NEERI

95. What would be the impact of global warming onmangrove forests?(a) They will grow more luxurious(b) Large areas of mangroves will be submerged(c) Their role as carbon sinks will become more

important(d) Both (a) and (c) above

96. The sweet taste of fruits is due to(a) Lactose (b) Fructose(c) Maltose (d) Ribose

97. The most endangered Asiatic top predator onthe edge of extinction is(a) Black Bear (b) Asiatic Lion(c) Siberian Tiger (d) Dhole

98. Analects is the sacred book of(a) Confucianism (b) Judaism(c) Shintoism (d) Taoism

99. The seat of Madhya Pradesh High Court islocated at(a) Gwalior (b) Indore(c) Bhopal (d) Jabalpur

100. The brightest planet is(a) Venus (b) Mercury(c) Jupiter (d) Mars

PART-C : QUANTITATIVE APTITUDE

101. P and Q are two points observed from the top ofa building 10 3 m high. If the angles of depressionof the points are complementary and PQ = 20 m,then the distance of P from the building is(a) 25 m (b) 45 m(c) 30 m (d) 40 m

102. If A and B are complementary angles, then thevalue ofsin A cos B + cos A sin B – tan A tan B + sec2 A –cot2 B is(a) 2 (b) 0(c) 1 (d) –1

103. The least value of 2 sin2 q + 3 cos2q is(a) 3 (b) 5(c) 1 (d) 2

104. A, O, B are three points on a line segment and Cis a point not lying on AOB. If ÐAOC = 40° andOX, OY are the internal and external bisectors ofÐAOC respectively, then ÐBOY is

yoursmahboob.w

ordpress.com

92 SSC CGL SolvedPaper

(a) 70° (b) 80°(c) 72° (d) 68°

105. If 4x = sec q and 4x

= tan q then 2

218 x

xæ ö-ç ÷è ø is

(a)1

16(b)

18

(c)12

(d)14

106. If 2 – cos2 q = 3 sin q cos q, sin q ¹ cos q thentan q is

(a)12

(b) 0

(c)23

(d)13

107. If sin q + cos q = 2 cos (90 – q), then cot q is

(a) 2 1+ (b) 0

(c) 2 (d) 2 –1108. If x sin3 q + y cos3 q = sin q cos q and x sin q = y

cos q, sin q ¹ 0, cos q ¹ 0, then x2 + y2 is

(a)12 (b)

12

(c) 1 (d) 2109. In the following figure, O is the centre of the circle

and XO is perpendicular to OY. If the area of thetriangle XOy is 32, then the area of the circle is

O

X Y

(a) 64 p (b) 256 p(c) 16 p (d) 32 p

110. The side BC of D ABC is produced to D. If ÐACD

= 108° and ÐB = 12

ÐA then ÐA is

(a) 36° (b) 72°(c) 108° (d) 59°

111. Two circles of radii 4 cm and 9 cm respectivelytouch each other externally at a point and acommon tangent touches them at the points Pand Q respectively. They the area of a squarewith one side PQ, is(a) 97 sq. cm (b) 194 sq. cm(c) 72 sq. cm (d) 144 sq. cm

112. Two tangents are drawn from a point P to a circleat A and B. O is the centre of the circle. If ÐAOP= 60°, then ÐAPB is(a) 120° (b) 90°(c) 60° (d) 30°

113. If each intetior angle is double of each exteriorangle of a regular polygon with n sides, then thevalue of n is(a) 8 (b) 10(c) 5 (d) 6

114. If the length of the side PQ of the rhombus PQRSis 6 cm and ÐPQR = 120°, then the length of QS,in cm, is(a) 4 (b) 6(c) 3 (d) 5

115. The angle formed by the hour–hand and theminute–hand of a clock at 2 : 15 p.m. is

(a)1272

° (b) 45°

(c)1222

° (d) 30°

116. Two sides of a triangle are of length 4 cm and 10cm. If the length of the third side is 'a' cm. then(a) a > 5 (b) 6 £ a £ 12(c) a < 5 (d) 6 < a < 14

117. If x = (0.08)2, y = 21

(0.08) and z = (1 – 0.08)2 – 1,

then out of the following the true relation is(a) y < x and x = z(b) x < y and x = z(c) y < z < x(d) z < x < y

118. In xy–plane, P and Q are two points havingco–ordinates (2, 0) and (5, 4) respectively. Thenthe numerical value of the area of the circle withradius PQ is(a) 16 p (b) 32 p(c) 14 p (d) 25 p

yoursmahboob.w

ordpress.com

93SSC CGL SolvedPaper

119. If 4

41

xx

+ = 23, then the value of 21x

xæ ö-ç ÷è ø will

be(a) 7 (b) –7(c) –3 (d) 3

120. The value of 6 {6 6 .......upto...}é ù+ + +ë û is

equal to(a) 3 (b) 10(c) 8 (d) 2

121. If 1xx

+ = 3, the value of 5

51

xx

+ is

(a) 123 (b) 126(c) 113 (d) 129

122. sec4 q – sec2 q is equal to(a) tan2 q – tan4 q (b) tan2 q + tan4 q(c) cos4 q – cos2 q (d) cos2 q – cos4 q

123. In DABC, AD is the median and AD = 12

BC. If

ÐBAD = 30°, then measure of ÐACB is(a) 90° (b) 45°(c) 30° (d) 60°

124. If 6 15 x 10´ = , then the value of x is(a) 3 (b) ± 3(c) 3 (d) 6

125.3 5 13

4 3 5+- -

+ is equal to

(a) 0 (b)32

(c)5

2(d) 5

126. If a + b + 1 = 0, then the value of (a3 + b3 + 1 – 3ab) is(a) 3 (b) 0(c) –1 (d) 1

127. In the xy–coordinate system, if (a, b) and (a + 3, b+ k) are two points on the line defined by theequation x = 3y – 7, then k = ?

(a)73

(b) 1

(c) 9 (d) 3

128. The average age of four boys, five years ago was9 years. On including a new boy, the presentaverage age of all the five is 15 years. The presentage of the new boy is(a) 14 years (b) 6 years(c) 15 years (d) 19 years

129. If the average of 39, 48, 51, 63, 75, 83, x and 69 is60, then the value of x is(a) 52 (b) 53(c) 50 (d) 51

130. The cost of a piece of diamond varies with thesquare of its weight. A diamond of ̀ 5,184 valueis cut into 3 pieces whose weights are in the ratio1 : 2 : 3. Find the loss involved in the cutting.(a) `3, 068 (b) `3, 088(c) `3, 175 (d) `3, 168

131. A discount of 30% on the marked price of a toyreduces its selling price by ̀ 30. What is the newselling price (in ̀ ) ?(a) 70 (b) 21(c) 130 (d) 100

132. The capacities of two hemispherical vessels are6.4 litres and 21.6 litres. The ratio of their innerradii is(a) 4 : 9 (b) 16 : 81(c) 2 : 3 (d) 2 : 3

133. Pipe A alone can fill a tank in 8 hours. Pipe Balone can fill it in 6 hours. If both the pipes areopened and after 2 hours pipe A is closed, thenthe other pipe will fill the tank in

(a) 6 hours (b)1

32

hours

(c) 4 hours (d)122

hours

134. If (a – b) = 3, (b – c) = 5 and (c – a) = 1, then the

value of 3 3 3a b c 3abc

a b c+ + -

+ + is

(a) 17.5 (b) 20.5(c) 10.5 (d) 15.5

135. The population of a town is 15000. If the numberof males increases by 8% and that of females by10%, then the population would increase to 16300.Find the number of females in the town.(a) 4000 (b) 6000(c) 3000 (d) 5000

yoursmahboob.w

ordpress.com

94 SSC CGL SolvedPaper

136. If ̀ 5,000 becomes ̀ 5,700 in a year's time, whatwill `7,000 become at the end of 5 years at thesame rate of simple interest?(a) `10,500 (b) `11,900(c) `12,700 (d) `7, 700

137. A thief is noticed by a policeman from a distanceof 200 m. The thief starts running and thepoliceman chases him. The thief and thepoliceman run at the rate of 10 km and 11 km perhour respectively. The distance (in metres)between them after 6 minutes is(a) 190 (b) 200(c) 100 (d) 150

138. 'A' sells an article to 'B' at a profit of 20% and 'B'sells it to 'C' at a profit of 25%. If 'C' pays `1200,the cost price of the article originally (in ̀ ) is(a) 700 (b) 600(c) 1,000 (d) 800

139. The number nearest to 75070 which is divisibleby 65, is(a) 75070 (b) 75075(c) 75010 (d) 75065

140. The number 20% more than 80 is(a) 36 (b) 30(c) 90 (d) 96

141. A tree is broken by the wind. If the top of the treestruck the ground at an angle of 30° and at adistance of 30 m from the root, then the height ofthe tree is(a) 25 3m (b) 30 3m

(c) 15 3m (d) 20 3m142. If cos A + cos2 A = 1, then sin2 A + sin4 A is equal

to

(a) 1 (b)12

(c) 0 (d) –1143. A farmer divides his herd of n cows among his

four sons, so that the first son gets one–half the

herd, the second one–fourth, the third son 15

and the fourth son 7 cows. Then the value of n is(a) 240 (b) 100(c) 180 (d) 140

144. By what least number should 675 be multiplied toobtain a number which is a perfect cube?(a) 7 (b) 8(c) 5 (d) 6

145. The least number which when divided by 35, 45,55 leaves the remainder 18, 28, 38 respectively is(a) 3448 (b) 3482(c) 2468 (d) 3265

Directions: In questions no. (146 to 150). The graphshows Income and Expenditure of a company. Studythe graph and answer the questions.

01020304050

3020

5040

60 60

30 3040

35

60

2000 2001 2002 2003 2004

IncomeExpenditure

YEARSRs

. in

Lank

s

146. The expenditure from 2002 to 2003 increased by

(a)133 %3

(b) 40%

(c) 10% (d) 20%147. The income in 2002 was equal to the expenditure

in the year(a) 2003 (b) 2004(c) 2000 (d) 2001

148. The profit was maximum in the year(a) 2003 (b) 2004(c) 2001 (d) 2002

149. The difference in profit between 2001 and 2002 is(a) `25 lakhs (b) No difference(c) `10 lakhs (d) `20 lakhs

150. The number of years in which the income exceedsthe average income is(a) three (b) four(c) one (d) two

PART-D : ENGLISH COMPREHENSION

Directions: In question no. 151 to 155, some parts ofthe sentences have errors and some have none. Findout which part of a sentence has an error. If a sentenceis free from error, then your rows in answer is (d), i.e.,no error.

151. You do not / look as / your brother/ No error (a) (b) (c) (d)

152. My elder brother / is six / foot high. / No error (a) (b) (c) (d)

yoursmahboob.w

ordpress.com

95SSC CGL SolvedPaper

153. Without no proof of your guilt / the only courseopen to me / (a) (b)/ is to dismiss the case. / No error (c) (d)

154. As we see it, / she appears to be unreasonable / (a) (b)anxious about pleasing her husband. / No error (c) (d)

155. The scissor is / lying on / the table. / No error (a) (b) (c) (d)

Directions : In question no. 156 to 160, sentencesare given with blanks to be filled in with anappropriate word(s). Four alternatives are suggestedfor each question. Choose the correct alternative outof the four as your answer.

156. The Union Budget is likely to be presented onFebruary 26, two days ahead of the ___________date.(a) critical (b) conventional(c) suitable (d) convenient

157. I am sorry ___________ the mistake.(a) from (b) with(c) for (d) at

158. He ___________ her that she would pass.(a) insured (b) ensured(c) assumed (d) assured

159. Your father ___________ worry. I'm a verycareful driver.(a) needn't (b) none(c) can't (d) doesn't

160. The ___________ chosen for construction ofthe building is in the heart of the city.(a) cite (b) slight(c) sight (d) site

Directions: In questions no. 161 to 165, out of thefour alternatives, choose the one which best expressesthe meaning of the given word as your answer.

161. Nexus(a) connection (b) distance(c) deficit (d) difference

162. Mammoth(a) straight (b) huge(c) wild (d) greedy

163. Hyperbole(a) expansion (b) imitation(c) decoration (d) exaggeration

164. Eulogy(a) apology (b) address(c) speech (d) praise

165. Menacingly(a) dangerously (b) threateningly(c) harmfully (d) hideously

Directions: In questions no. 166 to 170, choose theword opposite in meaning to the given word as youranswer.

166. Impeccable(a) faulty (b) tedious(c) flashy (d) boring

167. Amalgamate(a) separate (b) combine(c) assimilate (d) integrate

168. Zenith(a) climax (b) crisis(c) acme (d) nadir

169. Influx(a) reflex (b) deflection(c) effluent (d) exodus

170. Orderly(a) semitic (b) colic(c) democratic (d) chaotic

Directions: In questions no. 171 to 175, fouralternatives are given for the idiom / phraseunderlined in the sentence. Choose the alternativewhich best express the meaning of the idiom/phraseas your answer.

171. Ram is very calculative and always has an axe togrind.(a) has no result(b) works for both sides(c) has a private agenda(d) fails to arouse interest

172. The police looked all over for him but drew ablank.(a) did not find him (b) put him in prison(c) arrested him (d) took him to court

173. On the issue of marriage, Sarita put her foot down.(a) stood up (b) was firm(c) got down (d) walked fast

yoursmahboob.w

ordpress.com

96 SSC CGL SolvedPaper

174. His investments helped him make a killing in thestock market.(a) lose money quickly(b) plan a murder quickly(c) murder someone quickly(d) make money quickly

175. There is no gainsaying the fact that the countryis in difficulties.(a) ignoring (b) hiding(c) forgetting (d) denying

Directions: In questions no. 176 to 180, a part of thesentence is underlined. Below are given alternativesto the underliend part at (a), (b) and (c) which mayimprove the sentence. Choose the correct alternative.In case no improvement is needed, your answer is (d).

176. Sordid and sensational books tend to vitiate thepublic taste.(a) divide(b) distract(c) distort(d) No improvement

177. By studyding AIDS has engaged manyresearchers in the last decade.(a) Important study(b) Now that the study(c) The study of(d) No improvement

178. His Master's thesis was highly estimated and isnow being prepared for publication.(a) was highly discussed(b) was highly commended(c) is highly appraised(d) No improvement

179. No sooner had she realized her blunder than shebegan to take corrective measures.(a) then she began to take(b) than she began taking(c) when she began to take(d) No improvement

180. A good scholar must be precise and possessoriginality.(a) must be precise and original(b) must be possess precision and original(c) must be precision and possess originality(d) No improvement

Directions: In questions no. 181 to 185, out of thefour alternatives, choose the one which can besubstituted for the given words/ sentence.

181. One who loves books(a) Bibliophile (b) Bibliophagist(c) Bibliophoebe (d) Bibliographer

182. Speaking without preparation(a) Deliberate (b) Fluent(c) Loquacious (d) Extempore

183. Special trial of the Head of State by Parliament(a) Impingement (b) Infringement(c) Impeachment (d) Impediment

184. Someone able to use both hands with equal skill(a) Ambivalent (b) Amphibious(c) Ambiguous (d) Ambidextrous

185. Cure for all diseases(a) Curable (b) Panacea(c) Incurable (d) Curative

Directions: In questions no. 186 to 190, there arefour different words out of which one is correctly spelt.Find the correctly spelt word.

186. (a) pleintive (b) sustein(c) villain (d) alleince

187. (a) comissioner (b) commissionar(c) commisioner (d) commissioner

188. (a) aprentice (b) advertise(c) treatice (d) sencitive

189. (a) suprintendent (b) supirentendent(c) superintendent (d) superentendent

190. (a) symetry (b) symmitry(c) symatry (d) symmetry

Directions: In questions no. 191 to 200, you havetwo brief passages with 5 questions following eachpassage. Read the passages carefully and choose thebest answer to each question out of the four alternatives.

PASSAGE-I (191 – 195)Two years later, in November 1895, he signed his finalwill. He left the bulk of his fortune, amounting to about£1,75,000 to a trust fund administered by Swedish andNorwegian trustees. The annual interest shall beawarded as prizes to those persons who during theprevious year have rendered the greatest services tomankind. The interest shall be divided into five equalparts– now amounting to about £8,000 each– one ofwhich shall be awarded to the person who has made

yoursmahboob.w

ordpress.com

97SSC CGL SolvedPaper

the most important discovery or invention in the realmof physics, one to the person who has made the mostimportant chemical discovery or improvement, one tothe person who has made the most importantphysiological or medical discovery, one to the personwho has produced the most outstanding work ofliterature, idealistic in character, and one to the personwho has done the best work for the brotherhood ofnations, the abolition or reduction of standing armies,as well as for the formation or popularization of peacecongress.191. The said prize is awarded

(a) once in 5 years (b) every year(c) once in 4 years (d) once in 2 years

192. Which is the prize that is referred to in thepassage?(a) Nobel Prize(b) Magsaysay Award(c) Pulitzer Prize(d) Booker Prize

193. The number of prizes in the field of science are(a) Four (b) One(c) Three (d) Five

194. Total annual prize money amounts to(a) £8,000 (b) £1,750,000(c) £350,000 (d) £40,000

195. Prize is awarded for outstanding work in(a) Chemistry (b) Literature(c) Physics (d) All the above

PASSAGE-II (196–200)If an opinion contrary to your own makes you angry,that is a sign that you are subconsciously aware ofhaving no good reason for thinking, as you do. Ifsomeone maintains that two and two are five, or thatIceland is on the Equator, you feel pity rather thananger, unless you know so little of arithmetic orgeography that his opinion shakes your own contraryconviction.

196. If someone else's opinion makes us angry, it meansthat(a) we are subconsciously aware of having no

good reason for becoming angry(b) there may be good reasons for his opinion

but we are not consciously aware of them(c) our own opinion is not based on good

reason and we know this subconsciously(d) we are not consciously aware of any reason

for our own opinion197. "Your own contrary conviction" refers to

(a) the fact that you feel pity rather than anger(b) the opinion that two and two are four and

that Iceland is a long way from the Equator(c) the opinion that two and two are five and

that Iceland is on the Equator(d) the fact that you know so little about

arithmetic or geography198. Conviction means

(a) persuasion (b) disbelief(c) strong belief (d) ignorance

199. The writer says if someone maintains that twoand two are five you feel pity because you(a) have sympathy(b) don't agree with him(c) want to help the person(d) feel sorry for his ignorance

200. The second sentence in the passage(a) builds up the argument of the first sentence

by restating it from the opposite point ofview

(b) makes the main point which has only beenintroduced by the first sentence

(c) simply adds, a further point to the argumentalready stated in the first sentence

(d) illustrates the point made in the firstsentence

yoursmahboob.w

ordpress.com

98 SSC CGL SolvedPaper

HINTS & SOLUTIONS

1. (a) The relationship is x : (2x – 4)8 × 2 – 4 = 16 – 4 = 12Similarly, 6 × 2 – 4 = 12 – 4 = 8

2. (a) (1)2 = 1 ; (3)2 = 9 Þ 19Similarly, (2)2 = 4 ; (1)2 = 1Þ 41

3. (a) The movement of eagle is like swooping.Similarly, the movement of duck is calledwaddle.

4. (a) As,

Similarly,O R A N G E

5. (c) We pay rent for accommodation. Similarly,we pay fare for journey.

6. (b) Fire causes smoke. Smoke comes out whensomething is burnt in fire. Similarly, cloudcauses rain.

7. (d) Grenade and gun are fire arms. Similarly, headand brain are sensitive organs.

8. (b) As, T S H

I RQ

+1–2 –2

Similarly,

+1–2 –2Q P K

L O N9. (a) As, A E Z F P Y

+5+11–1

Similarly,B G X W W

+6+16–1

H

10. (b) SNZT

PLWQ

–3–2–3–3

11. (d) Major, Colonel and Brigadier are differentranks in the Indian Army. Admiral is thetopmost rank in the Indian Navy.

12. (a) Except Life Insurance Corporation, all othersare insurance companies for generalinsurance, i.e., for vehicles, property etc.

13. (b) Except disease, all other terms denoteobstruction, hindrance or interruption.

14. (b) Except remedy, all other terms denote lossof something.

15. (a) Socrates was a Greek philosopher. LudwingVan Beethoven was a German Composer andMusician. Bach was also a German Composer.WA Mozart was a Austrian Compose.

16. (d) 112 is completely divisible by 4.112 132 125 12428; 26.4; 15.625; 17.71

4 5 8 4= = = =

17. (a) The difference between the two numbers inthe number pair 6246 – 6296 is least.6296 – 6246 = 507267 – 7137 = 1304684 – 4344 = 3405465 – 5235 = 230

18. (b) The pattern is as follows :

YX U T S ONM L G F E D C

–3 –4 –5

–1 –1 –1 –1 –1 –1 –1 –1 –119. (c) The pattern is as follows :

+4D

A

H

E

L

I

P

M

T

Q+1

+4 +4 +4

+4+4+4

20. (c) The pattern is as follows :

21. (d) The pattern is as follows :

AZ

CX

EV

GT

+2–2 –2 –2

+2 +2

yoursmahboob.w

ordpress.com

99SSC CGL SolvedPaper

22. (c) The pattern is as follows :

D9Y

J27S

P81M

V

G

B729A

+6×3

–6

+6 +6 +6×3 ×3 ×3

–6 –6 –6

23. (c)

a c d b / d a c b / c d a b / a c d b / d a24. (c) 9 × 2 + 1 = 18 + 1 = 19

19 × 2 + 2 = 38 + 2 = 4040 × 2 + 3 = 80 + 3 = 8383 × 2 + 4 = 166 + 4 = 170170 × 2 + 5 = 340 + 5 = 340Therefore, the number 340 is wrong in theseries.

25. (d) 21 + 7 = 2828 + 5 = 3333 + 3 = 3536 + 1 = 3737 – 1 = 36Therefore, the number 35 is wrong in theseries.

26. (a) 5 + 8 = 1313 + 16 = 2929 + 32 = 6161 + 64 = 120125 + 128 = 253Therefore, the number 120 is wrong in theseries.

27. (b) 0 + 7 = 77 + 21 = 2828 + 35 = 6363 + 61 = 124124 + 87 = 215Therefore, the number 215 is wrong in theseries.

28. (a) a b cÑ ÑÞ a < b < cOption (a)a b c a b cor,D f Þ > £a < b £ cOption (b)a b c a b cf + Þ £ =Option (c)a 0 b + c Þ a > b = cOption (d)a 0 b × c Þ a > b ³ c

29. (a) 2 × 3 × 5 × 4 = 120120 × 120 = 14400

30. (b) As,P E A R

G F D NTherefore,R E A P

N F D G31. (d) 5 – 4 = 1; 4 – 3 = 1

1 + 1 = 26 – 0 = 6; 5 – 1 = 46 + 4 = 106 – 2 = 4; 7 – 2 = 54 + 5 = 9

32. (a) Þ ¸Þ ×

Þ + Þ –

L

P

M

Q16 P 24 M 8 Q 6 M 2 L 3 = ?Þ ? = 16 + 24 ¸ 8 – 6 ¸ 2 × 3Þ ? = 16 + 3 – 3 × 3Þ ? = 16 + 3 – 9 = 10

33. (b) There is no 'V' letter in the given word.34. (a) As,

F L A T T E R

7 2 3 8 8 5 9and,M O T H E R

4 6 8 1 5 9Therefore,M A M M O T H

4 3 4 4 6 8 135. (a) 16 Þ (2 + 2)2 = (4)2

9 Þ (3 + 0)2 = (3)2

81 Þ (1 + 8)2 = (9)2

Similarly, 64 Þ (4 + 4)2 = (8)2

36. (d) The product of two numbers in a sector isequal to the central number in the previoussector.3 × 5 = 158 × 3 = 247 × 2 = 145 × 3 = 15

yoursmahboob.w

ordpress.com

100 SSC CGL SolvedPaper

8 × 4 = 329 × 1 = 99 × 2 = 187 × 4 = 28

37. (c) First Column10 + 12 + 4 + 10 = 3636 182

=

Second Column

11 + 12 + 12 + 5 = 4040 202

= =

Third Column

15 + 8 + 10 + 13 = 46 46 232

= =

38. (c)

North

SouthA, B, C, D

50 km

B, C

B, C50 km

A,D

50 km

A, DA Þ East, B Þ West,C Þ West, D Þ East.

39. (a)

A B

C D

E

F

2 km

1 km

3 km

2 km

\ Required distance AE

( ) ( )2 2AF EF= +

2 2(4) (3)= + 16 9 25 5 km= + = =

40. (a) Temple and Church are places of worship. Itdoes not imply that Hindus and Christiansuse the same place for worship. Church isdifferent temple. Therefore, neitherConclusion I nor II follows.

41. (a) Growth and development of human organismis a continuous process. Some changes takeplace in human body now and then.Therefore, neither Conclusion I nor IIfollows.

42. (b) H I P P N O W A D I A S M

P P I H N O W A D M S A I

43. (c) QMPN PQR ROPQNOP PQR

MQRO PQR PPRR PQR P

44. (d) Volume of sphere 34 r3

= p

Volume of hemisphere 32 r3

= p

Now,

3 34 2r r3 3

p = p

or, ( )33 34 2r 3 23 3

=

or, 3 2 3r 27 23 4

= ´ ´ ´

\ r = 3 cm45. (b) Option (b) is correct answer.46. (d) Option (d) is the right mirror image.47. (c) As,

S E A R C H T F B S D I

+1+1+1+1+1+1

Similarly,

+1+1+1+1+1+1

P E N C I LQ F O D J M

yoursmahboob.w

ordpress.com

101SSC CGL SolvedPaper

48. (b)49. (d)

50. (a) G Þ 58 , 66, 77, 85 , 98

U Þ 04, 12, 23 , 31, 40

I Þ 00, 13, 21 , 34, 42

D Þ 56, 69, 75, 87 , 99

E Þ 01, 14, 20, 32 , 43

G U I D E

(a)

(b)

(c)

(d)

85 23 21 87 32

58 31 12 57 41

77 13 42 99 32

66 31 43 78 14

51. (b) The Union Government set up the NationalCommission for Minorities (NCM) under theNational Commission for Minorities Act,1992.

52. (d) Mr. Kamlesh Sharma, an Indian diplomat,became Commonwealth Secretary-Generalon 1 April 2008. He was appointed to thepost by Commonwealth Heads ofGovernment at their meeting in Kampala,Uganda, in November 2007.

53. (a) Some countries, Such as Argentina, Austria,Australia, Belgium, Brazil, Canada, Germany,India, Malaysia, Mexico, Pakistan, Russia,Switzerland and the United States, link theirbicameral systems to their federal politicalstructure.

54. (c) He was appointed High Commissioner toGreat Britain in 1990.

55. (a) King Asoka assumed the title DevanampiyaPiyadasi which means "Beloved-of-the-Gods, He who Looks on with Affection".

56. (b) Socialism is an economic systemcharacterised by social ownership and/orcannot of the means of production andcooperative management of the economy.

57. (b) The act of selling the same article, productunder a single control, at different prices todifferent buyers is known as price

discrimination. Information on the priceelasticity of demand can be used by businessas part of a policy of price discrimination(also known as yield management).

58. (a) Economic planning is an essential featureof socialism. The most prominent exampleof a planned economy was the economicsystem of the Soviet Union.

59. (b) The New Agricultural Strategy is based onconcentration of high-yielding varieties ofseeds and complementary inputs onselected water-as-sured areas.

60. (a) The National Policy for Empowerment ofWomen 2001 has as its goal bringing aboutadvancement, development and empowermentof women in all spheres of life through creationof a more responsive judicial and legal systemsensitive to women and mainstreaming agender perspective in the developmentprocess.

61. (a) First used in the Australian state of Victoriain 1857, the paper ballot listing all thecandidates was first known as "theAustralian ballot. "In 1889, New York becamethe first American state to use these ballots.

62. (c) The Government of India decide to set upthe Rashtriya Barh Ayog (National FloodCommission) in 1976 to evolve acoordinated, integrated and scientificapproach to the flood control problems inthe country.

63. (d) The term race or racial group usually refersto the concept of dividing humans intopopulations or groups on the basis ofvarious sets of characteristics. The mostwidely used human racial categories arebased on visible traits (especially skincolour, cranial or facial features and hairtexture), and self-identification.

64. (c) Passenger coaches are manufactured atthree principal places : Integral CoachFactory (ICF) at Perambur, Railway CoachFactory (RCF) at Kapurthala, And BharatEarth Movers Ltd. (BEML) at Bangalore.

65. (c) This process usually occurs in the upperthird of the fallopian tube of the woman.

66. (d) Cirrhosis is a condition in which the liverdoes not function properly due tolong-term damage. Cirrhosis is most

yoursmahboob.w

ordpress.com

102 SSC CGL SolvedPaper

commonly caused by alcohol, hepatitis B,hepatitis C, and non-alcoholic fatty liverdisease. Typically, more than two or threedrinks per day over a number of years isrequired for alcoholic cirrhosis to occur.

67. (c) The food webs we see are grazing foodchains since at their base are producerswhich the herbivores then graze on.

68. (a) Chihalgani was the group of most importantand powerful forty nobles or highly placedofficers in the court of Iltutmish. Balbandestroyed their clout.

69. (b) In 1661, Catherine of Braganza of Portugalbrought islands of Bombay to Charles II ofEngland as part of her marriage dowry.

70. (a) The road was initially built by Sher Shah toconnect Agra, his capital, with Sasaram, hishometown.

71. (b) Shivaji, the great Maratha King and founderof a nationalist tradition was contemporaryof Tukaram. Tukaram introduced Shivaji toRamdas for his spritual educataion.

72. (c) Limnology is the story of lakes and otherfreshwater basins.

73. (a) Homoseismal lines is the line on the Earth'ssurface connecting points where the seismicwave arrives, generated by an earthquake,at the same time.

74. (c) The lumen is the SI derived unit of luminousflux, a measure of the total "amount" ofvisible light emitted by a source.

75. (a) In computing, an interface is a sharedboundary across which two separatecomponents of a computer system exchangeinformation. The exchange can be betweensoftware, computer hardware,peripheraldevices, humans and combinations of these.

76. (c) Interface Card, Ethernet Cable and Routersare used in setting up a LAN. The routercan be plugged directly into the modem viaan Ethernet cable, and all other computersare eventually connected in some fashionto the route. In terms of LAN, a computercannot be said to be an item whichconstitutes this network.

77. (c) A single mole is set to the number of particlesfound in 12.000 grams of carbon-12. A moleof water has 6.022 x 1023 water molecules.One mole of water weighs 18.0152 grams.

78. (a) Carrot is rich in Vitamin A and it improveseyesight.

79. (d) The reason for this is the hydrogen bondingbetween neighboring water molecules.Because hydrogen bonding is a relativelystrong intermolecular force, high heat energyis required to break up the force.

80. (a) Valence electrons are important indetermining how an elements reactschemically with other elements. Since thevalence electrons are the electrons in thehighest energy level, they are the mostexposed of all the electrons, so they are theelectrons that get most involved in chemicalreactions.

81. (b) CDRI, is located at Lucknow was inuguratedin 1951 by the then Prime Minister of India,Jawahar lal Nehru.

82. (b) Wheat was the first cereal to be cultivatedby man. In several places in the Middle Eastit was sowed, tended and reaped soon after8000 BC. The people of Jericho are the firstknown to have lived mainly from thecultivation of crops.

83. (b) Durum wheat is the only tetraploid form ofwheat widely used today, and the secondmost widely cultivated wheat. This hardwheat is cultivated in clayey soil and ishighly sought after for its physicalcharacteristics.

84. (d) A dissipative force counteracts motion. Itsdirection is opposite to the direction of thevelocity vector. Dynamic friction is adissipative (non-conservative) force : itdissipates energy (mainly through heat andsound), and energy lost by moving in onedirection.

85. (c) Poisson strain is defined as the negative ratioof the strain in the traverse direction (causedby the contraction of the bar's diameter) tothe strain in the longitudinal direction. Asthe length increases and the cross sectionalarea decreases, the electrical resistance ofthe wire also rises.

86. (b) Consequent Poles are magnetic poles thatexist where the specimen has beensuccessively magnetized in differentsections to create more than two poles ; e.g.,two north poles with one south polebetween them.

yoursmahboob.w

ordpress.com

103SSC CGL SolvedPaper

87. (d) The spokes in the Ashok Chakra represent24 states as 24 were the number of states atthat time.

88. (b) In 1980, the General Assembly made Arabican official and working language of all itscommittees and subcommittees.

89. (b) A five member, committee was constitutedwith Justice Srikrishna as the chairman ofthe committee in March 2010 on the separateTelangana issue.

90. (d) The author is Brigadier (Retired) KuldipSingh. This book is an effort to understandnow the future will unfold in Asia 2030.

91. (c) The Ministry of Human ResourceDevelopment, through the NationalUniversity of Educational Planning &Administration, has developed anEducation development Index (EDI) to trackprogress of the States towards UniversalElementary Education (UEE). Kerala tops thecomposite EDI and Bihar is at 35th rank.

92. (a) The VK Shunglu Committee report inquiredinto the Commonwealth Games corruptioncases.

93. (a) The 65th National Football Championshipfor Santosh Trophy 2010-11 was held inAssam. Bengal beat Manipur 2-1 to win thetrophy.

94. (d) The National Environmental EngineeringResearch Insititute (NEERI) is a researchinstitute created and funded by Governmentof India. It was established in Nagpur in1958.

95. (d) Mangrove wetlands are possible sinks/sources for carbon dioxide and other relatedgreenhouse gases. Global warming maypromote expansion of mangrove forests tohigher latitudes. Elevated CO2 concentrationmay increase mangrove growth bystimulating photosynthesis or improvingwater use efficiency.

96. (b) Fructose, or fruit sugar, a is one of the threedietary monosaccharide, along with glucoseand galactose, which is absorbed directlyinto the bloodstream during digestion.

97. (d) The Dhole is on the edge of extinction. Alsocalled the Asiatic wild dog or Indian wilddog, it is a species of canid native to Southand Southeast Asia.

98. (a) The Analects of Confucius is an anthologyof brief passages that present the words ofConfucius and his disciples, describeConfucius as a man, and recount some ofthe events of his life. The Analects includestwenty books, each generally featuring aseries of chapter that encompass quotesfrom Confucius, which were compiled by hisdisciples after his death.

99. (d) The Court was established in Nagpur, butafter the reorganisation of states on1 November 1956, it was moved to Jabalpur.

100. (a) There are droplets of sulphuric acid and acidiccrystals in the atmosphere of Venus. Thesmooth surfaces of these droplets of sulfuricacid and crystals reflect light very well, whichis one reason why Venus is so bright.

101. (c)

BQ P

A

90 –°q q

a

h

In this type of Q just puth2 = ab

( ) ( )210 3 x x 20= +

300 = x (x + 20)10 (30) = x (x + 20)x + 20 = 30

102. (c) A + B = 90° Þ A = 90° – BÞ sin A = sin (90° – B) = cos BSimilarly,Þ cos A = sin B, tan A = cot B\ sin A . cos B + cos A. sin B – tan A. tanB+ sec2 A– cot2 B = cos2 B+ sin2 B – cotB.tanB + sec2A – tan2A= 1 – 1 +1 = 1

2 2[ tan B.cot B 1,sec A tan A 1]= - =Q

103. (d) 2sin2A q + 3 cos2 q = 2 sin2 q + 2cos2 q + cos2

q= 2 (sin2q + cos2q) + cos2q= 2 + cos2q\ Lest value = 2 + 0 = 2

2[ cos 0]q ³Q

yoursmahboob.w

ordpress.com

104 SSC CGL SolvedPaper

104. (a)

OX is the bisector of ÐAOC.\ ÐAOC = 2 ÐCOXOY is the bisector of ÐBOC.\ÐBOC = 2 ÐCOY\ÐAOC + ÐBOC= 2ÐCOY + 2ÐCOX = 180°Þ 2 (ÐCOX + ÐYOC) = 180°Þ ÐXOY = 90°\ÐAOX + ÐXOY + ÐBOY = 180°\ÐBOY = 180° – 90° – 20° = 70°

105. (c) 4x = sec q

secx4qÞ =

Again, 4 1 tantanx x 4

q= qÞ =

22

18 xx

æ ö\ -ç ÷è ø

2 2sec tan816 16

æ öq q= -ç ÷è ø

( )2 28 1sec tan16 2= q- q =

106. (a) 2 – cos2q = 3sinq. cosqDividing by cos2q

2 22 3sin .cos1

cos cosq q- =

q qÞ 2 sec2q – 1 = 3 tanqÞ 2 (1 + tan2q) – 1 = 3 tanqÞ 2 tan2q + 2 – 1 = 3 tanqÞ 2 tan2q – 3 tanq + 1 = 0Þ 2 tan2q – 2 tanq – tanq + 1= 0Þ 2 tan q (tan q – 1) – 1 (tan q – 1) = 0Þ (2 tanq – 1) (q – 1) = 0

1tan or 12

Þ q =

107. (d) sinq + cosq = ( )2 cos 90- qsinq + cosq = 2 sinqDivide eq. by sin q1 + cotq = 2cotq = 2 1-

108. (c) xsin3q + ycos3q = sinq.cosqÞ (x sin q) . sin2 q + (y cos q)cos2 q = sin q.cos qÞ xsinq . sin2q + xsin q. cos2q= sin q.cos qÞ x sin q (sin2q + cos2 q)= sin q.cos qÞ x = cos q\ x sinq = y cosqÞ cos q . sin q = y cos qÞ y = sin q\ x2 + y2 = cos2 q + sin2q = 1

109. (a)

X

O

YÐXOY = 90°;OX = OY = radices (r)\ D XOY is a right angled triangle.

( ) ( )1 OX OY 322

\ ´ ´ =

Þ r2 = 2 × 32 = 64

r 64 8\ = =\ Area of circle = pr2

= 64 p sq. units

110. (b) A

BC D

ÐACD = ÐABC + ÐBACA108 A2ÐÞ ° = +Ð

3 A 1082ÐÞ = °

108 2A 723´Þ Ð = = °

yoursmahboob.w

ordpress.com

105SSC CGL SolvedPaper

111. (d)P

Q

r1 r2

r1 + r2 = 13 cmr2 – r1 = 9 – 4 = 5 cmPQ

( ) ( )222 1dis tan ce between centres r r= - -

( )2 213 5 12 cm= - =\ Area of square = 12 × 12= 144 sq. cm.

112. (c)

A

P

B

O

In right Ds OAP and OPB,AP = PB, OA = OBOP = OP

OAP OBP\D @ D\ ÐAOP = ÐPOB and ÐAPO= ÐOPBFrom D AOP,ÐAPO = 180° – 90° – 60° = 30°\ ÐAPB = 2 × 30 = 60°

113. (d) Let exterior Ðbe = xinterior Ðbe = 2x

x + 2x = 180 3x = 180 x = 60°

no of side n 360=60

6=

114. (b) S R

O

90°

P Q

ÐPQO 1 PQR 602

= = °

From DPOQ,ÐOPQ = 180° – 90° – 60° = 30°

sin OPQ OQPQ

=

Þ OQ = PQ sin 30°

16 32

= ´ =

\ QS = 2 × 3 = 6 cm115. (c) Angle traced by hour hand in an hour = 30°

1 9Angle traced In2 i.e. hours4 4

\

9 135304 2

°= ´ ° =

Angle traced by minute hand in 60 minutes= 360°\ Angle traced in 15 minutes

360 15 9060

= ´ = °

\ Required angle 13590

2°= ° -

45 1222 2

°= =

116. (d) The sum of any two sides of a triangle isgreater than third side and their differenceis less than third side.10 – 4 < a < 10 + 46 < a < 14

117. (d) x = (0.08)2

y ( )21 10000 156.25

640.08= = =

Z = (1 – 0.08)2 – 1= 1 + (0.08)2 – 2 × 0.08 – 1= (0.08)2 – 2 × 0.08Clearly, z < x < y

118. (d) Y

O

Q

X

(5, 4)

(2, 0)P

yoursmahboob.w

ordpress.com

106 SSC CGL SolvedPaper

( ) ( )2 2PQ 5 2 4 0= - + -

9 16 5= + =\ Area of circle = pr2

= 25 p sq. units

119. (d) 44

1x 23x

+ =

22

21x 2 23

xæ öÞ + - =ç ÷è ø

22

21x 23 2 25

xæ öÞ + = + =ç ÷è ø

22

1x 5x

\ + =

22

21 1x x 2x x

æ ö\ - = + -ç ÷è ø

= 5 – 2 = 3

120. (a) x 6 6 6 ............= + + + ¥

On squaring,

2x 6 6 6 ..........= + + + ¥Þ x2 = 6 + xÞ x2 – x – 6 = 0Þ x2 – 3x + 2x – 6 = 0Þ x (x – 3) + 2 (x –3) = 0Þ (x – 3) (x + 2) = 0Þ x = 3 because x 2¹ -by trick 3 2 6

it is 6 isanswer´ =

+ \Q

121. (a)1x 3x

+ =

On squaring,21x 9

xæ ö+ =ç ÷è ø

22

1x 9 2 7x

Þ + = - =

Again, 31x 27

xæ ö+ =ç ÷è ø

33

1 1x 3 x 27xx

æ öÞ + + + =ç ÷è ø

33

1x 27 3 3 18x

Þ + = - ´ =

2 32 3

1 1x xx x

æ öæ ö\ + +ç ÷ç ÷è øè ø= 7 × 18

55

1 1x x 126xx

æ öÞ + + + =ç ÷è ø

55

1x 126 3 123x

Þ + = - =

122. (b) sec4 q – sec2 q= sec2 q (sec2 q – 1)= (1 + tan2 q) (1 + tan2 q –1)= tan2 q + tan4 q

123. (d) A

B CD

BD = DC = ADÐBAD = 30°From D ABD,ÐBAD = 30°\ ÐABD = ÐBAD = 30°\ ÐADB = 180° – 2 × 30° = 120°\ ÐADC = 180° – 120° = 60°\ AD = DCÞ ÐDAC = ÐACD = 60°

124. (a) 6 15 x 10´ =

2 3 3 5 x 10Þ ´ ´ ´ =

2 5 3 x 10Þ ´ ´ =

3 10 x 10Þ =Þ x = 3

yoursmahboob.w

ordpress.com

107SSC CGL SolvedPaper

125. (b) ( )( )1 3 5

3 5 3 5 3 5-=

+ + -

3 5 3 59 5 4- -= =-

3 5 3 534 4

+ -\ - -

12 3 5 3 54

- - - +=

6 34 2

= =

126. (b) If a + b + c = 0then a3 + b3 + c3 – 3abc = 0

127. (b) Points (a, b) and [(a + 3), (b + k)] will satisfythe equation x – 3y + 7 = 0.\ a – 3b + 7 = 0 ......(i)and a + 3 – 3 (b + k) + 7 = 0Þ a + 3 – 3b – 3 k + 7 = 0Þ a – 3b + 7 + 3 – 3 k = 0Þ 3 – 3k = 0 Þ 3k = 3

[ ]3k 1 a 3b 7 03

Þ = = - + =Q

128. (d) Sum of the present ages of four boys= 9 × 4 + 20 = 56 yearsSum of the present ages of five boys= 15 × 5 = 75 years\ Present age of new boy= 75 – 56 = 19 years

129. (a) 39 + 48 + 51 + 63 + 75 + 83 + x + 69 = 60 × 8Þ 428 + x = 480Þ x = 480 – 428 = 52

130. (d) If the weight of a piece of diamond be 6 xunits, thenOriginal price a (6x)2 = 36kx2

\ 36. kx2 = 5184 .....(i)Again,New price = k (x2 + 4x2+ 9x2)= 14 kx2

14 5184 201636

´= = `

\ Loss = 5184 – 2016= ̀ 3168

131. (a) 30% 30ºQ `\ 100% º ̀ 100\ New S.P. = 100 – 30= ̀ 70

132. (d) 31

32

2 r 6.432 21.6r3

p=

p

3 331

32

r 64 4 2216 6 3r

æ ö æ öÞ = = =ç ÷ ç ÷è ø è ø

1

2

r 2r 3

Þ =

133. (d) Part of the tank filled by both pipes in twohours

1 128 6

æ ö= +ç ÷è ø

3 4 7224 12+æ ö= =ç ÷è ø

Remaining part = 7 51

12 12- =

Time taken by B in filling the remaining part5 5 16 2 hours

12 2 2= ´ = =

134. (a) a3 + b3 + c3 – 3abc= (a + b + 3)

= ( ) ( ) ( )2 2 21 (a b c) a b b c c a2

é ù+ + - + - + -ë û

3 3 3a b c 3abca b c

+ + -\+ +

( ) ( ) ( )2 2 21 a b b c c a2

é ù= - + - + -ë û

( )1 9 25 12

= + +

35 17.52

= =

135. (d) If the number of females be x, then, numberof males = 15000 – x

10 8x (15000 x)100 100

\ ´ + - ´

yoursmahboob.w

ordpress.com

108 SSC CGL SolvedPaper

= 16300 – 15000Þ 10x + 120000 – 8x= 1300 × 100Þ 2x = 130000 – 120000= 10000Þ x = 5000

136. (b) Interest = 5700 – 5000 = ̀ 700

700 100Rate 14%5000 1

´\ = =´

Case II,Interest

Principal Time Rate100´ ´

=

7000 5 14 4900100´ ´ =`

Amount = 7000 + 4900 = ̀ 11900137. (c) Relative speed = 11 – 10 = 1 kmph

Distance covered in 6 minutes

1000 6metre 100 metre60

= ´ =

\ Remaining distance= 200 – 100 = 100 metre

138. (d) Effective profit percent

20 2520 25100

´æ ö= + +ç ÷è ø= 50%

\ Original cost price

100 1200 800150

= ´ = `

139. (b) 65)75070(115465

10065

35732532026060

\ Required number= 75070 + (65 – 60) = 75075

140. (d) Required number

80 120 96100´= =

141. (c)

A

B

C

D30

30°

AB = treeBC = broken part\ BC = CDAD = 30 metreFrom DACD,

tan 30°ACAD

=

1AC AD3

Þ = ´

30 10 3metre3

= =

CD = AC sin 30°

110 3 5 3 BC2

= ´ = =

\ AB = AC + BC

10 3 5 3 15 3metre= + =142. (a) cos A = 1 – cos2 A = sin2A

\ sin2A + sin4A = sin2A + cos2A= 1

143. (d) According to the question,

n n n 7 n2 4 5

+ + + =

10n 5n 4n 7 n20

+ +Þ + =

19n 19n7 n n 720 20

Þ + = Þ - =

n 7 n 20 7 14020

Þ = Þ = ´ =

144. (c) 675 = 5 × 5 × 3 × 3 × 3= 33 × 52

\ Required number = 5

yoursmahboob.w

ordpress.com

109SSC CGL SolvedPaper

145. (a) 35 – 18 = 1745 – 28 = 1755 – 38 = 17i.e., difference between the divisor andcorreseponding remainder is same.LCM of 35, 45 and 55 = 3465\ Required number= 3465 – 17 = 3448

146. (a) Required percentage increase40 30 100100

30 3-= ´ =

133 %3

=

147. (a) Income of company in 2002= ̀ 40 lakhsExpenditure of company in 2003= ` 40 lakhs

148. (b) Profit of company in 2004= ` 25 lakhs

149. (c) Required difference= 20 – 10 = ̀ 10 lakhs

150. (a) Average income of company30 50 40 60 60

5+ + + +=

240 48 lakhs5

= =`

The incomes of company in years 2001, 2003and 2004 were greater than ̀ 40 lakhs.

151. (b) Here, look like should be used. Like is anAdjective which is used as Preposition.

152. (c) Here foot tall should be used.153. (a) Here, with no proof of your guilt/ without

any proof of your guilt should be used.154. (b) Here, she appears to be unreasonable

(Adverb) should be used because anAdverb modifies an Adjective.

155. (a) Here, The scissors are should be used.156. (b) 157. (c) 158. (d) 159. (a) 160. (d)161. (a) The word Nexus (Noun) means : a

complicated series of connections betweendifferent things; connection.

162. (b) The word Mammoth (Adjective) means :extremely large; huge.

163. (d) The word Hyperbole (Noun) means : a wayof speaking or writing that makes somethingsound better, more exciting than it really is;exaggeration.

164. (d) The word Eulogy (Noun) means : speech orpiece of writing praising somebody/something very much : praise.

165. (b) The word Menacingly (Adverb) means :threateningly; seeming likely to cause youharm or danger.

166. (a) The word Impeccable (Adjective) means :without mistakes or fault; perfect.Hence, its antonym should be faulty.

167. (a) The word Amalgamate (Verb) means :merge; to put two or more things to getherso that they form one; assimilate.Its antonym should be separate whichmeans : to divide into different parts orgroups; to move apart.

168. (d) The word Zenith (Noun) means : the highestpoint; peak; the time when something isstrongest and most successful.The word Nadir (Noun) means : the worstmoment of a particular situation.

169. (d) The word Influx (Noun) means : the fact of alot of people, money or things arrivingsomewhere.The word Exodus (Noun) means : a situationin which many people leave a place at thesame time.

170. (d) The word Orderly (Adjective) means :arranged or organised in a neat, careful andlogical way; tidy; behaving well.The word Chaotic (Adjective) means : in astate of complete confusion and lack oforder.

171. (c) Idiom have an axe to grind means : to haveprivate reasons for being involved insomething or for arguing for a particularcause.

172. (a) Idiom draw a blank means : to get noresponse or result.

173. (b) Idiom put your foot down means : to be verystrict in opposing what somebody wishesto do; to drive faster.

174. (d) Idiom make a killing means : to make a lotof money quickly.

175. (d) Idiom Gainsay (verb) means : to disagree; todeny.

176. (b) 177. (c) 178. (b) 179. (d) 180. (a)181. (a) 182. (d) 183. (c) 184. (d) 185. (b)186. (c) 187. (d) 188. (b) 189. (c) 190. (d)191. (b) 192. (a) 193. (c) 194. (a) 195. (d)196. (c) 197. (a) 198. (c) 199. (d) 200. (d)

yoursmahboob.w

ordpress.com

110 SSC CGL SolvedPaper

PART-A : GENERAL INTELLIGENCE & REASONING

Directions: In questions no. 1 to 9, select the relatedletter/word/number from the given alternatives:

1. Zoology : Animal :: Psychology : _?_(a) Animal(b) Human-being(c) Animal and human-being(d) Plant

2. CFIL : ABCD :: _?_ : WXYZ(a) ZYXW (b) DCBA(c) JHPS (d) XURO

3. L × M : 12 × 13 :: U × W : _?_(a) 21 × 23 (b) 24 × 26(c) 9 × 11 (d) 12 × 23

4. Man : Mammal : : _?_(a) Liberty : Literate (b) Hail : Snow(c) Native : Inhabitant (d) Offspring : Family

5. 6 : 2 :: 8 : _?_(a) 5 (b) 1(c) 3 (d) 7

6. CFIL : XURO :: ORUX : _?_(a) MJFC (b) ROLI(c) RITO (d) LIFC

7. 1 : 8 :: 4 : _?_(a) 32 (b) 64(c) 512 (d) 128

8. Life starts : Embryo :: Life ends : _?_(a) Death (b) Old age(c) Dead body (d) Illness

9. EVFU : TGSH :: IRJQ : _?_(a) PKLO (b) KWLX(c) PKOL (d) OLPK

Directions: In questions no. 10 to 20, find the oddnumber/letters/number pair from the given alternatives.

10. (a) 72 (b) 81(c) 93 (d) 66

11. (a) (125, 27) (b) (64, 216)(c) (216, 02) (d) (343, 01)

12. (a) Division (b) Addition(c) Subtract (d) Multiplication

13. (a) 47 (b) 17(c) 27 (d) 37

14. (a) 5270 – 2936 (b) 186 – 69(c) 168 – 570 (d) 1001 – 100

15. (a) 8 5 3 6 2 (b) 6 3 8 5 2(c) 5 2 6 3 8 (d) 2 8 7 5 1

16. (a) VEIIDD (b) IVEF(c) VEENS (d) EINN

17. (a) Arunachal Pradesh(b) Jammu and Kashmir(c) Haryana(d) Himachal Pradesh

18. (a) Size (b) Number(c) Form (d) Weight

19. (a) Fish (b) Frog(c) Tortoise (d) Crab

20. (a) Board (b) Commission(c) Team (d) Agenda

Directions: In questions no. 11 to 24, a Series is given,with one term missing. Choose the correct alternativefrom the given ones that will complete the series.

21. RAZ SBY TCX UDW VEV _?_(a) ZAT (b) WFU(c) FWU (d) XGX

22. AAC BBD CCE DDF EEG F _?_(a) DG (b) FG(c) GH (d) FH

23. B I P _?_ D(a) U (b) W(c) S (d) R

24. DF, GJ, KM, NQ, RT, _?_(a) UX (b) UW(c) YZ (d) XZ

Directions: In questions no. 25 to 26, which one setof letters when sequentially placed at the gaps in thegiven letter series shall complete it?

25. a_ba_c_aad_aa_ea(a) babbb (b) babbd(c) babbc (d) bacde

26. aa_aa bb_b_aa_aa bb_bb.(a) bbbaa (b) bbbba(c) aabbb (d) babba

SSC Combined Graduate Level (CGL) Solved PaperEVENING SHIFT 1 JULY, 2012

yoursmahboob.w

ordpress.com

111SSC CGL SolvedPaper

27. If 64 ̧ 14 = 5, 92 ̧ 31 = 7, 26 ̧ 11 = 6, then 56 ̧ 22= _?_(a) 11 (b) 39(c) 7 (d) 36

28. It was Shriram’s and Sreedevi’s 12th WeddingAnniversary. Shriram said “When we got married,Sreedevi was 3/4th of my age, but now she is 5/6thof my age”. What actually are their present ages?(a) Shriram 38, Sreedevi 32(b) Shriram 36, Sreedevi 30(c) Shriram 30, Sreedevi 24(d) Shriram 40, Sreedevi 34

29. Find the wrong number in the series from the givenalternatives.17, 36, 53, 68, 83, 92(a) 92 (b) 53(c) 68 (d) 83

30. If P denotes ¸, Q denotes ×, R denotes +, and Sdenotes –, then, 1 8 Q 1 2 P 4 R 5 S 6 = _?_(a) 95 (b) 53(c) 51 (d) 57

31. If DEAF is equal to 32, what will be LEAF = _?_(a) 56 (b) 48(c) 50 (d) 52

32. If 25 ̧ 5 = 15, 30 ¸ 6 = 20, then 35 ̧ 7 = _?_(a) 75 (b) 20(c) 50 (d) 25

33. In a certain code, “CERTAIN” is coded as“XVIGZRM”, “SEQUENCE” is coded as“HVJFVMXV”. How would “REQUIRED” becoded?(a) FJIVWVIR (b) VJIFWTRV(c) WVJRIFVI (d) IVJFRIVW

Directions: In question no. 34, from the givenalternatives select the word which cannot be formedusing the letters of the given word.

34. ‘CONSTITUTIONAL’(a) CONSULT (b) LOCATION(c) TUTION (d) TALENT

35. If 33 + 45 = 30, 90 + 26 = 40, then 30 + 45 = —?(a) 15 (b) 14(c) 16 (d) 18

Directions: In question no. 36, select the missingnumber from the given responses.

36.7

8

42

88

4 39

6

15

97

1 ?

(a) 6 (b) 2(c) 3 (d) 4

37. 7 6 92 8 44 3 ?

36 42 26(a) 5 (b) 2(c) 3 (d) 4

38. Ram and Sham start walking in oppositedirections. Ram covers 6 kms and Sham 8 kms.Then Ram turns right and walks 8 kms and Shamturns left and walks 6 kms. How far each is fromthe starting point?(a) 11 kms (b) 8 kms(c) 9 kms (d) 10 kms

39. Sherly starting from a fixed point goes 15 mtoward North and then after turning to his righthe goes 15 m. Then he goes 10, 15 and 15 metresafter turning to his left each time. How far is hefrom his starting point?(a) 15 metres (d) 5 metres(c) 10 metres (d) 20 metres

Directions: In questions no. 40 to 41, two statementsare followed by two conclusions numbered I and II.Which one of the four alternatives is correct?40. Statement:

I. All teachers are aged.II. Some women are teachers.Conclusion:I. All aged are womenII. Some women are aged.(a) Both conclusion I and II follow(b) Only conclusion I follows(c) Only conclusion II follows(d) Neither conclusion I nor II follows

41. Statement:I: All skaters are good swimmers.II: All good swimmers are runners.

yoursmahboob.w

ordpress.com

112 SSC CGL SolvedPaper

Conclusion:I: Some runners are skaters.II: Some skaters are good swimmers.(a) Neither conclusion I nor II follows(b) Only conclusion I follows(c) Only conclusion II follows(d) Both conclusion I and II follow

42. How many 9’s are followed by and preceeded bynumbers divisible by 2?8 9 6 5 3 5 9 6 8 3 4 9 6 5 2 6 9 7 3 7 2 9 4 1 3 7 9 4 17 3 4 9 8 4 5 3 9 7 6 1 5 3 1 9 5 7 4 2 9 6 8 5 3 2 9 57 4 8 9 4 5 1(a) 12 (b) 6(c) 8 (d) 10

43. If LUXOR is coded as 30, then GUILDS will becoded as?(a) 40 (b) 36(c) 38 (d) 24

44. In the following list of English alphabets, onealphabet has not been used. Identify the same.X N F A P S R W L T M D E X M G BC X Q J L O P V R C Q J Z O H S GO D I P T S M R A B E F G N U N E(a) V (b) I(c) K (d) J

45. If Alphabets are serially numberd, one of theanswers given below has not a meaningful wordhidden in it. Identify the answer.(a) 5, 18, 5, 8, 1, 3, 5, 20 (b) 5, 1, 3, 5, 20, 8, 18(c) 18, 5, 8, 1, 3, 5, 20 (d) 20, 5, 8, 1, 3, 5, 18

Directions: In question no. 46, which answer figurewill complete the question figure?

46. Question figure

?

Answer figures

(a) (b) (c) (d)

47. From the given answer figures, select the one inwhich the question figure is hidden/embeddedQuestion figure

Answer figures

(d)(a) (b) (c)

Directions: In question no. 48, if a mirror is placedon the line MN, then which of the answer figures isthe right image of the given figure?

48. Question figure

M N

Answer figures

(a) (b) (c) (d)

49. A square sheet of paper has been folded andpunched as shown below. You have to figureout from amongst the four response figures, howit will appear when opened?Question figure

Answer figures

(a) (b) (c) (d)

yoursmahboob.w

ordpress.com

113SSC CGL SolvedPaper

Directions: In question no. 50, a word is representedby only one set of numbers as given in any one of thealternatives. The sets of numbers given in thealternatives are represented by two classes ofalphabets as in two matrices given below. Thecolumns and rows of Matrix I are numbered from 0 to4 and that of Matrix II are numbered from 5 to 9. Aletter from these matrices can be represented first byits row and next by its column, e.g., ‘T’ can berepresented by 31, 76 etc. and ‘S’ can be representedby 14, 99 etc. Similarly, you have to identify the setfor the word STAR.

50.0 1 2 3 4

0 G V E A C1 R O N G S2 M N E S I3 O T I T A4 N S N E P

Matrix - I

5 6 7 8 90 R E O N G1 N P V E S2 M T I O N3 E A I C O4 N T A R S

Matrix - II

(a) 99, 31, 86, 98 (b) 23, 76, 33, 98(c) 14, 87, 98, 97 (d) 69, 96, 03, 56

PART-B : GENERAL AWARENESS

51. Which is NOT a correct statement?(a) Phenols are acidic(b) In benzene all the atoms lie in one plane(c) Methylated spirit contains only methanol(d) Dilute solutions contain less amount of solute

52. The infective stage of Malaria is:(a) Gametocyte (b) Ring stage(c) Sporozoite (d) Merozoite

53. The treaty of Versailles restored Alsace-Lorraineto:(a) Italy (b) Britain(c) France (d) Belgiun

54. The Asokan Edicts were deciphered first by:(a) Sir John Marshall (b) Sir William Jones(c) Charles Wilkins (d) James Princep

55. Which of the following is meant for the ex-situconservation of various species?(a) Sperm bank (b) Blood bank(c) Germplasm bank (d) Herbarium

56. An algae type ocean deposit is:(a) Neritic remains (b) Diatom Ooze(c) Pteropod Ooze (d) Pelagic deposits

57. Photosynthetic vesicle found in bacteria is called a:(a) Mesosome (b) Chromatophore(c) Genophore (d) Pneumatophore

58. What type of mirror is used in a view findingmirror of a vehicle?(a) Convex mirror(b) Plane mirror(c) Concave mirror(d) Paraboloidal mirror

59. What is m-commerce?(a) machine commerce(b) mobile commerce(c) money commerce(d) marketing commerce

60. Who said that the Directive Principles of StatePolicy are just like “a cheque on bank payable atthe convenience of the bank”.(a) Pandit Nehru (b) K. T. Shah(c) B. R. Ambedkar (d) N. G. Ranga

61. Wheat, Barley, Lemon, Orange, rye and pearlmillet belong to:(a) the same plant family(b) two plant families(c) three plant families(d) four plant families

62. Who favoured the Artic Home thoery of theAryans?(a) Pargiter (b) A. C. Das(c) B. G. Tilak (d) Jacobi

63. A plant known only in cultivation having arisenunder domestication is referred to as:(a) Scion (b) Cultigen(c) Cultivar (d) Clone

64. The proposal for the creation of new All-IndiaServices can be considered only:(a) if majority of State Legislatures make such

demand

yoursmahboob.w

ordpress.com

114 SSC CGL SolvedPaper

(b) if Lok Sabha passes a resolution by two-thirds majority

(c) if the Rajya Sabha passes a resolution bytwo-thirds majority

(d) None of the above65. Pyroligneous acid obtained from wood contains:

(a) 10% Formaldehyde (b) 10% Acetic acid(c) 10% Formic acid (d) 10% ethanol

66. Union Carbide India Ltd. manufacturedessentially:(a) Heavy water (b) Petrochemicals(c) Fertilizers (d) Leather goods

67. The iron and steel plant in Bihar is at:(a) Visakhapatnam (b) Bokaro(c) Burnpur (d) Vijay Nagar

68. Who was the teacher of Gautama Buddha?(a) Panini (b) Alara Kalama(c) Kapila (d) Patanjali

69. Ram Sharan Sharma, who died in 2011 was aneminent(a) Chemist (b) Economist(c) Indologist (d) Archaeologist

70. The 34th National Games were held in 2011 in:(a) Uttarakhand (b) Kerala(c) Karnataka (d) Jharkhand

71. “Eye for an eye and tooth for a tooth” is theguiding principle of:(a) Attributive theory of Justice(b) Retributive theory of Justice(c) Deterrent theory of Justice(d) Reformative theory of Justice

72. Low cost housing is an example for:(a) Mixed wants (b) Social wants(c) Private wants (d) Merit wants

73. Torah is the scared book of:(a) Zoroastrianism (b) Confucianism(c) Taoism (d) Judaism

74. The 98th Indian Science Congress was held in2011 at:(a) Bengaluru (b) Bhopal(c) Chennai (d) Bhubaneshwar

75. Drying oils contain a fairly large proportion of:(a) Unsaturaled fatty acids(b) Fats(c) Proteins(d) Saturated fatty acids

76. Consumption for the sake of enjoying socialacknowledgement is called:(a) Rational consumption(b) Social consumption

(c) Conspicuous consumption(d) Demonstration consumption

77. The red, orange and yellow colours of leaves aredue to:(a) Carotenoids (b) Aldehydes(c) Tannins (d) Lignins

78. Which bank was the first to introduce ATMs tothe world?(a) Hong Kong Bank(b) Standard Chartered Bank(c) Bank of America(d) Citi Bank

79. We receive sunlight on earth surface. What typeof light beams are these?(a) Random (b) Parallel(c) Converging (d) Diverging

80. The state which has registered the highestpopulation growth rate according to 2001 censusis:(a) Kerala (b) Uttar Pradesh(c) Nagaland (d) Sikkim

81. Earth is a very big magnet. In which directiondoes it magnetic field extend?(a) west to east (b) north to south(c) south to north (d) east to west

82. Of the following economists, whom do youconsider to be the Master of “Partial Analysis”?(a) Leon Walras (b) Alfred Marshall(c) J. M. Keynes (d) Lionel Robbins

83. The authority to specify which castes shall bedeemed to be scheduled castes rests with the:(a) Commissioner for Scheduled Castes and

Tribes(b) Prime Minister(c) President(d) Governor

84. Polar-bears hold cures for:(a) Type II diabetes (b) Osteoporosis(c) Breast-cancer (d) Kidney failure

85. Which colour/colours of light has the highestvelocity through vacuum?(a) Blue (b) Red(c) Green (d) All of the above

86. The ultimate source of energy in a hydroelectricpower station is:(a) solar energy(b) the potential energy of water(c) the kinetic energy of water(d) the electro-chemical energy of water

yoursmahboob.w

ordpress.com

115SSC CGL SolvedPaper

87. Mamta Sharma was appointed in 2011 as thechairperson of:(a) National Commission for Minorities(b) National Commission for Protection of Child

Right(c) National Commission for women(d) National Commission for BCs

88. India making ‘Double Taxation AvoidanceAgreements’ (DTAA) with other countries forthe promotion of:(a) Bilateral trade(b) External commercial borrowings(c) Foreign direct investments(d) Foreign institutional investment

89. The seat of Kerala High Court is located at:(a) Kottayam(b) Thiruvananthapuram(c) Kollam(d) Ernakulam

90. The disease that kills more people than lungcancer as a consequence of air pollution is:(a) chronic bronchitis (b) asthma(c) emphesema (d) heart attack

91. The most densely populated state in India is:(a) Kerala (b) Uttar Pradesh(c) West Bengal (d) Tamil nadu

92. Brain drain has been caused by:(a) failure to recognise talent in the originating

country.(b) the lure of high living standards(c) lack of employment opportunities(d) socio-economic instability

93. Human Development index was formulated by:(a) ASEAN (b) IBRD(c) UNDP (d) UNCTAD

94. The biggest planet in the solar system is:(a) Venus (b) Jupiter(c) Saturn (d) Uranus

95. Peninsular India has the following zonal soiltypes:(a) Red and yellow soil(b) Forest soil(c) Saline soil (d) Alluvial soil

96. The prose collection of the vedic poems are:(a) Samhitas (b) Upanishads(c) Aranyakas (d) Brahmanas

97. The study of population is known as.(a) Demography (b) Climatology(c) Petrology (d) Hydrology

98. Which of the following pairs is correctlymatched?(a) Milk of lime–sodium sulphate(b) Glauber’s salt–calcium sulphate(c) Salt petre–potassium nitrate(d) Gypsum–calcium hydroxide

99. Who is the author of the book ‘Pakistan: Beyondthe Crisis State’?(a) Khuram Iqbal (b) Maleeha Lodhi(c) Amir Mir (d) M. J. Akbar

100. The first speaker of Lok Sabha was:(a) S. Radhakrishnan(b) M. Ananthasayanam Ayyangar(c) Sardar Hukum Singh(d) G. V. Mavlankar

PART-C : QUANTITATIVE APTITUDE

101. If cot A + cosec A = 3 and A is an acute angle,then the value of cos A is:(a) 4/5 (b) 1(c) 1/2 (d) 3/4

102. A three-digit number 4a3 is added to anotherthree-digit number 984 to give the four digitnumber 13b7 which is divisible by 11. Then thevalue of (a + b) is:(a) 11 (b) 12(c) 9 (d) 10

103. In a right-angled triangle ABC, ÐB is the rightangle and AC = 2 5 cm. If AB – BC = 2 cm, thenthe value of (cos2A – cos2C) is:(a) 2/5 (b) 3/5(c) 6/5 (d) 3/10

104. A boy standing in the middle of a field, observesa flying bird in the north at an angle of elevationof 30° and after 2 minutes, he observes the samebird in the south at an angle of elevation of 60°.If the bird flies all along in a straight line at aheight of 50 3 m, then its speed in km/h is:(a) 4.5 (b) 3(c) 9 (d) 6

105. The perimeter of an isosceles, right-angledtriangle is 2p unit. The area of the same triangle is:

(a) ( ) 23 2 2 p sq.unit-

(b) ( ) 22 2 p sq.unit+

(c) ( ) 22 2 p sq.unit-

(d) ( ) 23 2 p sq.unit-

yoursmahboob.w

ordpress.com

116 SSC CGL SolvedPaper

106. DABC and DDEF are similar and their areas berespectively 64 cm2 and 121 cm2. If EF = 15.4 cm,BC is:(a) 12.3 cm (b) 11.2 cm(c) 12.1 cm (d) 11.0 cm

107. If G is the centroid of DABC and AG = BC, thenÐBGC is:(a) 75° (b) 45°(c) 90° (d) 60°

108. If tan(x + y) tan (x – y) = 1, then the value oftan x is:(a) 3 (b) 1

(c) 1/2 (d)13

109. In a partnership business, A invests 1 th6

of the

capital for 16

of the total time, B invests 14

of the

capital for 14

of the total time and C, the rest of

the capital for the whole time. Out of a profit of ̀19,400, B’s share is:(a) ` 2000 (b) ` 1200(c) ` 1600 (d) ` 1800

110. A jar contains a mixture of two liquids A and B inthe ratio 4 : 1. When 10 litre of the mixture isreplaced with liquid B, the ratio becomes 2 : 3.The volume of liquid A present in the jar earlierwas:(a) 20 lt (b) 10 lt(c) 16 lt (d) 15 lt

111. If (5x2 – 3y2) : xy = 11 : 2, then the postive value ofx/y is:(a) 7/2 (b) 5/2(c) 3/2 (d) 5/3

112. By decreasing 15° of each angle of a triangle, theratios of their angles are 2 : 3 : 5. The radianmeasure of greatest angle is:(a) 11p/24 (b) p/12(c) p/24 (d) 5p/24

113. The least value of 4 cosec2a + 9sin2a is:(a) 14 (b) 10(c) 11 (d) 12

114. The greatest number that will divide 19, 35 and59 to leave the same remainder in each case is:(a) 9 (b) 6(c) 7 (d) 8

115. The average temperature of Monday, Tuesdayand Wednesday was 30°C and that of Tuesday,Wednesday and Thursday was 33°C. If thetemperature on Monday was 32°C, then thetemperature on Thursday was:(a) 33°C (b) 30°C(c) 41°C (d) 32°C

116. If 3 2a3 2

-=+

, 3 2b3 2

+=-

, then the value of

2 2a bb a

+ is:

(a) 900 (b) 970(c) 1030 (d) 930

117. The next term of the series– 1, 6, 25, 62, 123, 214, _____ is:(a) 345 (b) 143(c) 341 (d) 343

118. O is the circum centre of the triangle ABC withcircumradius 13 cm. Let BC = 24 cm and OD isperpendicular to BC. Then the length of OD is:(a) 7 cm (b) 3 cm(c) 4 cm (d) 5 cm

119. If ax + by = 6, bx – ay = 2 and x2 + y2 = 4, then thevalue of (a2 + b2) would be:(a) 10 (b) 2(c) 4 (d) 5

120. The area of the largest triangle that can be inscribedin a semicircle of radius x in square units is:(a) 4x2 (b) x2

(c) 2x2 (d) 3x2

121. D and E are the mid-points of AB and AC ofDABC; BC is produced to any point P; DE, DPand EP are joined. Then,

(a)1PED ABC4

D = D (b) DPED = DBEC

(c) ADE BECD = D (d) BDE BECD = D

122. If 1a 1a

+ = , then the value of a3 is:

(a) – 2 (b) 2(c) – 1 (d) 4

123. The mean of 19 observation is 24. If the mean ofthe first 10 observations is 17 and that of the last10 observations is 24, find the 10th observation.(a) 65 (b) 37(c) –46 (d) 53

yoursmahboob.w

ordpress.com

117SSC CGL SolvedPaper

124. A watch is sold at a profit of 30%. Had it beensold for ` 80 less, there would have been a lossof 10%. What is the cost price of rupees?(a) 150 (b) 200(c) 400 (d) 800

125. A train overtakes two persons who are walkingin the same direction in which the train is running,at the rate of 2 kmph and 4 kmph and passesthem completely in 9 and 10 seconds respectively.The length of the train (in metres):(a) 72 (b) 45(c) 54 (d) 50

126. The length of the common chord of two circlesof radii 15 cm and 20 cm whose centres are 25 cmapart is (in cm):(a) 20 (b) 24(c) 25 (d) 15

127. If a commission of 10% is given on the markedprice of a work, the publisher gains 20%. If thecommission is increased to 15%, the gain presentis:

(a) 15% (b)216 %3

(c)113 %3 (d)

115 %6

128. If 5 3 5 32 x5 3 5 3

+ -= -- +

, then the value of

x is:(a) 6 (b) 30(c) 15 (d) 15

129. If x 2 3= + , then the value of 1xx

+ is:

(a) 3 (b) 6(c) 2 6 (d) 6

130. If 12 men or 18 women can reap a field in 14 days,then working at the same rate, 8 men and 16women can reap the same field in:(a) 9 days (b) 5 days(c) 7 days (d) 8 days

131. By selling 9 articles for a rupee, a man incurred aloss of 4%. To make a gain of 44%, the number ofarticles to be sold for a rupee is:(a) 5 (b) 3(c) 4 (d) 6

132. The simplied value of2sin A 1 cos A sin A1

1 cos A sin A 1 cos A+- + -

+ - is:

(a) cosA (b) 0(c) 1 (d) sinA

133. If a3 – b3 = 56 and a – b = 2, then the value of(a2 + b2) is:(a) – 10 (b) – 12(c) 20 (d) 18

134. If tanq – cotq = a and cosq – sinq = b, then thevalue of (a2 + 4) (b2 – 1)2 is:(a) 4 (b) 1(c) 2 (d) 3

135. Area of the trapezium formed by x-axis; y-axisand the lines 3x + 4y = 12 and 6x + 8y = 60 is:(a) 37.5 sq. unit (b) 31.5 sq. unit(c) 48 sq. unit (d) 36.5 sq. unit

136. Area of the triangle formed by the graph of the line2x – 3y + 6 = 0 along with the coordinate axes is:(a) 1/2 sq. units (b) 3/2 sq. units(c) 3 sq. units (d) 6 sq. units

137. If (a2 – b2) sinq + 2ab cosq = a2 + b2, then thevalue of tanq is

(a) ( )2 21 a b2ab

+ (b) ( )2 21 a b2

-

(c) ( )2 21 a b2ab

- (d) ( )2 21 a b2

+

138. Prabhat look a certain amount as a loan from abank at the rate of 8% p.a. simple interest andgave the same amount to Ashish as a loan at therate of 12% p.a. If at the end of 12 years, he madea profit of ` 96 in the deal, then the originalamount was:(a) ` 3356 (b) ` 1000(c) ` 2000 (d) ` 3000

139. AB is a diameter of a circle with centre O. CD is achord equal to the radius of the circle. AC andBD are produced to meet at P. Then the measureof ÐAPB is:(a) 120° (b) 30°(c) 60° (d) 90°

140. R and r are the radius of two circles (R > r). If thedistance between the centre of the two circlesbe d, then length of common tangent of twocircles is:

(a) 2r d2 - (b) ( )2d R r2 - -

(c) ( ) 2R r d2- - (d) 2R d2 -

yoursmahboob.w

ordpress.com

118 SSC CGL SolvedPaper

141. P is a point outside a circle and is 13 cm awayfrom its centre. A secant drawn from the point Pintersect the circle at points A and B in such away that PA = 9 cm and AB = 7 cm. The radius ofthe circle is:(a) 5.5 cm (b) 5 cm(c) 4 cm (d) 4.5 cm

142. If 1a 0

a 2+ =

+ , then the value of

( )( )

33

1a 2a 2

+ ++

is:

(a) 2 (b) 6(c) 4 (d) 3

143. If a is a positive acute angle and 2sina + 15cos2a= 7, then the value of cota is:(a) 3/4 (b) 4/3

(c)5

2(d)

25

144.1 876542 876544

876543 876543+ ´

´ is equal to

(a) 3 (b) 0(c) 1 (d) 2

145. The perimeters of two similar triangle DABC andDPQR are 36 cm and 24 cm respectively. If PQ =10 cm, then AB is:(a) 25 cm (b) 10 cm(c) 15 cm (d) 20 cm

Directions for question no. 146 to 150 : The numberof mobile simcards in 4 states are given in multiplebar diagrams. Study the diagram and answer thequestion 146 to 150.

0123456789

1011121314151617181920212223

Gujarat Tamil Nadu Assam Kerala

No.

of M

obile

Sim

card

ow

ners

(in

lakh

s)

BSNL

Airtel

Aircel

146. In Assam, the ratio of Aircell simcard and Airtelsimcard sold is:(a) 3 : 2 (b) 2 : 5(c) 5 : 2 (d) 2 : 3

147. In which state are there the largest number ofowners of Airtel simcard?(a) Tamil Nadu (b) Gujarat(c) Kerala (d) Assam

148. Average of simcard sold in the four states inlakhs is(a) 30.25 (b) 40.5(c) 35 (d) 33.75

149. The range of BSNL simcard sold in the 4 statesin lakhs is:(a) 12 (b) 15(c) 14 (d) 13

150. Of all the simcards sold in all the four states, thenumber of simcards sold in Gujarat is (approx)(a) 40% (b) 38%(c) 35% (d) 42%

PART-D : ENGLISH COMPREHENSION

Directions for question no. 151 to 155 : In thefollowing questions, some parts of the sentences haveerrors and some have none. Find out which part of asentence has an error. If a sentence is free from error,blacken the rectangle corresponding to (d) in theAnswer Sheet.

151. World is producing enough / (a)

for every citizen but still there is hunger andmalnutrition

(b)and it is continuing year after year. / No error.

(c) (d)152. The N.C.C. commandant along with his cadets /

(a)are going to Delhi /

(b)to participate in the Republic Day Parade. / Noerror.

(c) (d)153. He did not succeed / to get the job

(a) (b)though he tried his level best / No error.

(c) (d)154. Many of the famous / advertising offices /

(a) (b)are located at Madison Avenue. / No error.

(c) (d)

yoursmahboob.w

ordpress.com

119SSC CGL SolvedPaper

155. Nature has denied us / the power of closing our ears/ (a) (b)which she gave in respect of our eyes. / No error.

(c) (d)

Directions for question no. 156 to 160 : In thefollowing questions. sentences are given with blanksto be filled in with an appropriate word(s). Fouralternatives are suggested for each question. Choosethe correct alternative.

156. The ladies black purse, which is on sale has abeautiful ________ carved on it.(a) motif (b) patch(c) layout (d) schematic

157. Who is the person you ________ at the cinemalast night?(a) were recognising (b) recognised(c) have recognised (d) had recognised

158. As you sow ________ shall you reap.(a) when (b) as(c) like (d) so

159. He complemented her ________ new dress.(a) for (b) of(c) on (d) about

160. It took him a long time ________ the candidate’sapplication.(a) to considering(b) to consider and weigh(c) considering weighing(d) to consider and to weigh

Directions : In question nos. 161 to 165, out of thefour alternatives, choose the one which best expressesthe meaning of the given word and mark it in theAnswer sheet.

161. Annexure(a) retirement (b) commencement(c) attachment (d) development

162. Errand(a) energy (b) task(c) mistake (d) blunder

163. Bequeath(a) give (b) disclose(c) scold (d) surround

164. Nonchalant(a) imaginary (b) casual(c) neutral (d) formal

165. Forbearance(a) deliverance (b) patience(c) extravagance (d) relevance

Directions : In question nos. 166 to 170, choose theword opposite in meaning to the given word andmark it in Answer Sheet.

166. Amenable(a) acquiescent (b) distrustful(c) inattentive (d) unwilling

167. Conspicuous(a) blatant (b) definite(c) obvious (d) obsure

168. Reproof(a) approbation (b) apposition(c) condemnation (d) appropriation

169. Niggard(a) avaricious (b) extravagant(c) generous (d) miserly

170. Exotic(a) conventional (d) poor(c) inexpensive (d) indigenous

Directions : Four alternatives are given for the Idiom/phrase underlined in the sentence. Choose thealternative which best expresses the meaning of theIdiom/phrase and mark it in the Answer-Sheet.

171. His speech has taken the wind out of my sails.(a) made my words or actions ineffective(b) made me depressed(c) made me think for the future(d) made me remember my past

172. There is no point in discussing the new projectwith him as he always pours cold water on anyideas.(a) puts off (b) dislikes(c) disapproves of (d) postpones

173. Regadless of what her parents said, she wantedto let her hair down that night.(a) really enjoy (b) wash her hair(c) comb her hair (d) work till late

174. I jumped out of my skin when the explosionhappened.(a) was in panic (b) was excited(c) was nervous (d) was angry

175. She didn’t realize that the clever salesman wastaking her for a ride.(a) trying to trick her(b) taking her in a car(c) pulling her a long(d) forcing her to go with him

yoursmahboob.w

ordpress.com

120 SSC CGL SolvedPaper

Directions : In question no. 176 to 180, a part of thesentence is underlined. Below are given alternativesto the underlined part at (a), (b), (c) which mayimprove the sentence. Choose the correct alternative.In case no improvement is needed your answer is (d).

176. It took her a long time to get past her failure inthe medical examination.(a) through (b) over(c) by (d) No improvement

177. The boy wanted to ask his father for money, butwaited for a propitious occasion.(a) protective (b) prophetic(c) prospective (d) No improvement

178. I did not agree with him; he appeared to be sobigoted for me to concur.(a) much (b) very(c) too (d) No improvement

179. As soon as she noticed the workmen, she askedthem what they have been doing.(a) have done (b) had been(d) are doing (d) No improvement

180. He was asleep before the mother tucked him off.(a) through (b) away(c) in (d) No improvement

Directions : In question no. 181 to 185, out of thefour alternatives choose the one which can besubstituted for the given words/sentence.

181. A raised place on which offerings to a God are made.(a) rostrum (b) church(c) altar (d) mound

182. Something that cannot be explained(a) unthinkable (b) impregnable(c) mysterious (d) inexplicable

183. A written declaration made on oath in thepresence of a magistrate(a) affidavit (b) dossier(c) voucher (d) document

184. A person who thinks only about himself’ andnot about others’ needs:(a) egomaniacal (b) egoistic(c) egotistic (d) egocentric

185. A guide-post pointing out the way for a place(a) finger-post (b) lamp-post(c) checkpost (d) lastpost

Directions : In question no. 186 to 190, four wordsare given in each question, out of which only oneword is correctly spelt. Find the correctly spelt word.

186. (a) digresion (b) digrestion(c) digression (d) degression

187. (a) presumpchous (b) presumtous(c) presumptuous (d) presomptous

188. (a) equalibirium (b) equilibrium(c) equilibriam (d) equilibirium

189. (a) vaterinerian (b) veteinarian(c) vetarinerian (d) veterinerian

190. (a) marrytime (b) marytime(c) maritime (d) meritime

Directions : In question no. 191 to 200, you havetwo brief passages with 5 questions following eachpassage. Read the passages carefully and choosethe best answer to each question out of the fouralternatives.

PASSAGE-I“People very often complain that poverty is a greatevil and that it is not possible to be happy unless onehas a lot of money. Actually, this is not necessarilytrue. Even a poor man, living in a small hut with noneof the comforts and luxuries of life, may be quitecontented with his lot and achieve a measure ofhappiness. On the other hand, a very rich man, livingin a palace and enjoying everything that money canbuy, may still be miserable, if, for example, he doesnot enjoy good health or his only son has taken toevil ways. Apart from this, he may have a lot ofbusiness worries which keep him on tenterhooks mostof the time. There is a limit to what money can buyand there are many things which are necessary for aman’s happiness and which money cannot procure.

Real happiness is a matter of the right attitudeand the capacity of being contented with whateveryou have is the most important ingredient of thisattitude”.191. The phrase “on tenterhooks” means:

(a) in a state of thoughfulness(b) in a state of anxiety(c) in a state of sadness(d) in a state of forgetfulness

192. It is true that:(a) money alone can give happiness(b) money always gives happiness(c) money seldom gives happiness(d) money alone cannot give happiness

193. A rich man’s life may become miserable if he:(a) has evil son, bad health and business

worries(b) does not enjoy good health(c) has business worries(d) has business worries and his only son has

taken to evil ways

yoursmahboob.w

ordpress.com

121SSC CGL SolvedPaper

194. Which of the following is the most appropriatetitle to the passage?(a) Poverty, a great evil(b) The key of happiness(c) Contentment, the key of happiness(d) Money and contentment

195. Which of the following statement is true?(a) Only a poor but contented man can be

happy(b) A poor but contented man can never by

happy(c) A poor but contented man can be happy(d) A poor but contented man is always happy

PASSAGE- IIThe problem of water pollution by pesticides can beunderstood only in context, as part of the whole towhich it belongs - the pollution of the totalenvironment of mankind. The pollution entering ourwaterways comes from many sources, radioactivewastes from reactors, laboratories and hospitals;fallout from nuclear explosions; domestic wastes fromcities and towns; chemical wastes from factories. Tothese is a added a new kid of fallout - the chemicalsprays applied to crop lands and gardens, forestsand fields. Many of the chemical agents in thisalarming melange initiate and augment the harmfuleffects of radiation, and within the groups ofchemicals themselves there are sinister and little -understood interactions, transformations andsummations of effect.

Ever since the chemists began to manufacturesubstances that nature never invented, the problemof water purification have become complex and thedanger to users of water has increased. As we haveseen, the production of these synthetic chemicals inlarge volume began in the 1940’s. It has now reachedsuch proportion that an appalling deluge of chemicalpollution is daily poured into the nation’s waterways.

When inextricably mixed with domestic and otherwastes discharged into the same water, these chemicalssometimes defy detection by the methods in ordinaryuse by purification plants. Most of them are socomplex that they cannot be identified. In rivers, areally incredible variety of pollutants combine toproduce deposits that sanitary engineers can onlydespairingly refer to as “gunk”.196. All the following words mean ‘chemicals’ except:

(a) sands (b) substances(c) pesticides (d) deposits

197. The main argument of paragraph 1 is:(a) that there are sinister interaction in the use

of chemicals(b) that there are numerous reasons for

contamination of water supplies(c) that there are many dangers from nuclear

fallout(d) that pesticides are dangerous

198. The word ‘gunk’ in the last line refers:(a) to the waste products deposited by sanitary

engineers(b) to the debris found in rivers(c) to unidentifiable chemicals found in water(d) to the domestic water supplies

199. Water pollution can only be understood:(a) in relation to world contamination(b) by the whole human race(c) in context(d) in relation to the number of pesticides that

exist200. Water contamination has become serious:

(a) since water pollution was difficult to assess(b) since nature has taken a hand in pollution(c) since chemists began to use new substances(d) since businessmen authorised the use of

chemicals.

yoursmahboob.w

ordpress.com

122 SSC CGL SolvedPaper

HINTS & SOLUTIONS

1. (d) Zoology is the scientific study of animals.Similarly, Psychology is study of the mindand how it functions. It is the study ofhuman and animals behaviour.

2. (*) C F I L A B C D

–2

–6–4

–8Similarly,

–2

–6–4

–8

Y B E H W X Y Z

3. (a) L × M =12 × 13Position Number in the English alphabetseries.Similarly,U × W = 21 23´

4. (d) Man is a mammal. Man belongs to the classMammal. Similarly, offspring is a part of family.

5. (c) As, 46 2 4; 22

- = =Similarly,

68 2 6; 32

- = =

6. (d) C F I L

Pairs of Opposite Letters.Similarly,

7. (a) As, 1 × 8 = 8Similarly,4 × 8 = 32

8. (c) Life starts with the formation of embryo.When life ends dead body is left.

9. (a) E « V F « UPairs of Opposite Letters

U T–1

T « G S « HPairs of Opposite LettersI « R J « QPairs of Opposite Letters

–1Q PP « K O « LPairs of Opposite Letters

10. (b) 81 is a perfect square.11. (c) Except in (216, 02), in all others both the

numbers are perfect cubes.12. (c) Except Subtract, all others are Nouns. The

Noun for subtract (Verb) is Subtraction.13. (c) 27 is a perfect cube. All other numbers are

Prime Numbers.14. (b) 1 + 8 + 6 = 15 and 6 + 9 = 1515. (d) Except in the number 28751, in all others the

digits are the same.16. (a) IVEF Þ FIVE

VEENS Þ SEVENEINN Þ NINEVEIIDD Þ DIVIDE

17. (a) Jammu and Kashmir, Haryana and HimachalPradesh are northern States of India.Arunachal Pradesh is estern-most States ofIndia.

18. (b) Number is different from the others.19. (b) Frog is an amphibian.20. (d) Except Agenda, all other denotes a group

of persons.21. (b) The pattern is :

R

A

Z

S

B

Y

T

C

X

U

D

W

V

E

V

W

F

U

+1

–1

+1 +1 +1 +1

+1 +1 +1 +1 +1

–1 –1 –1 –1

22. (d) AA BB CC

DD EE FF23. (b)

B I P W D+7 +7 +7 +7

yoursmahboob.w

ordpress.com

123SSC CGL SolvedPaper

24. (a)

D G K N R U

F J M Q T X

+3 +4 +3 +3+4

+4 +4 +4+3 +3

25. (d) a b b a / a c c a / a d d a / a e e a

26. (d) a a b a a / b b a b b / a a b a a / b b a b b27. (c) 6 + 4 = 10; 1 + 4 = 5; 10 – 5 = 5

9 + 2 = 11; 3 + 1 = 4; 11 – 4 = 72 + 6 = 8; 1 + 1 = 2; 8 – 2 = 6

5 + 6 = 11; 2 + 2 = 4, 11 – 4 = 728. (b) Suppose the present age of Shriram is x years.

\ Present age of Sreedevi 5x years6

=

12 years ago3 5x(x 12) 124 6

- = - 3 5xx 9 124 6

Þ - = -

5x 3x 12 96 4

Þ - = -

10x 9x 312-Þ =

\ x = 3 × 12 = 36

\ Age of Sreedevi =5 36 30 years

6´ =

29. (d) As, 17 + 19 = 3636 + 17 = 5353 + 15 = 6868 + 13 = 81 8381 + 11 = 92

30. (b) P Þ ¸ Q ×Þ

R +Þ S –Þ

18 Q 12 P 4 R 5 S 6 = ?Þ ? = 18 × 12 ̧ 4 + 5 – 6Þ ? = 18 × 3 + 5 – 6Þ ? = 54 + 5 – 6 = 53

31. (b) D E A F

4 + 5 + 1 + 6 = 16

Þ 16 × 2 = 32L E A F

12 + 5 + 1 + 6 = 24 Þ 24 × 2 = 48

32. (d) Here, 25 ¸ 5 = 5; 5 × 3 = 1530 ¸ 6 = 5; 5 × 4 = 2035 ¸ 7 = 5; 5 × 5 = 25

33. (d) C E R T A I N

Z R MPairs of Opposite LettersS E Q U E N C

V M X

E

VTherefore,R E Q U I R E

R I V

D

W34. (d) There is no 'E' letter in the given word.35. (a) As, 3 + 3 + 4 + 5 = 15 Þ 1 + 5 = 6

and, 6 × 5 = 309 + 0 + 2 + 6 = 17 Þ 1 + 7 = 8and, 8 × 5 = 40Similarly,3 + 0 + 4 + 5 = 12 Þ 1 + 2 = 3and, 3 × 5 = 15

36. (b) The sun of numbers of the left verticallyhalf part is equal to the sum of the numbers onthe right.

1 + 7 + 9 + 5 + 1 + 9 + 6 + 4 = 42Therefore,? + 7 + 8 + 4 + 2 + 8 + 8 + 3 = 42Þ ? = 42 – 40 = 2

37. (b) First column(7 + 2) × 4 = 36Second Column(6 + 8) × 3 = 42Third Column(9 + 4) × ? = 26

Þ 13 × ? = 26 26? 213

\ = =

38. (d) B

A C

D

OShamRam

6 km 8 km

8k m

6km

2 2OB (AB) (AO)= + 2 2(8) (6)= +

64 36 100 10km= + = =

yoursmahboob.w

ordpress.com

124 SSC CGL SolvedPaper

39. (c)15 m

15m

E D

CB

A

10m15m

15m

North

West East

South

AF = AE – EF = 25 – 15 = 10 metres

40. (c)

Teachers

Aged

Women

Case I : FalseCase II : True

41. (d)

Skaters goodswimmers

runners

Case I : TrueCase II : True

42. (b) 869 5359683 496 5269737

294 13794173 498

45397615319574 296

658329574 894 5143. (d) As,

L U X O R

90 303

=

Similarly, G U I L D S

72 243

=

44. (c) X N F A P S R W L T M D E X

M G B C X Q J L O P V R C

Q J Z OH S G O D I PT SM RA B E F G N U N E

There is no 'K' letter.45. (a) As,

5 1 3 5 20 8 18

E A C E T H Rand,18 5 8 1 3 5 20

R E H A C E Tand,20 5 8 1 3 5 18

T E H A C E R\ 5 18 5 8 1 3 5

E R E H A C E

20

TTEACHER

46. (d) Option (d) will complete the question figure.47. (c) Option (c) is the hidden/ embedded figure.48. (b) Option (b) is the right image of the given

figure.49. (a) Option (a) is correct answer.50. (d) S Þ 14, 23, 41, 69, 99

T Þ 31, 33, 76, 96A Þ 03, 34, 86, 97R Þ 10, 55, 98

Option

(a)

(b)

(c)

(d)

S T A R

23 76 33 98

17 87 98 97

69 96 03 56

99 31 86 98

51. (c) Methylated spirit contains mostly ethanol.Ethanol is slightly soluble in water (mixing50 ml of ethanol and water will make80 – 90 ml of mixture, not 100ml).

yoursmahboob.w

ordpress.com

125SSC CGL SolvedPaper

52. (c) A sporozoite is the cell form that infectsnew hosts. In plasmodium, for instance, thesporozoites are cells that develop in themosquito’s salivary glands, leave themosquito during a blood meal, and enterliver cells (hepatocytes) where they multiply.

53. (c) After approximately 200 years of Frenchrule, Alsace and the German-speaking partof Lorraine were ceded to Germany in 1871under the Treaty of Frankfurt. In 1919, bothregions were returned to France.

54. (d) James Princep was an English scholar andantiquary. From 1832 to 1838 he was assay-master in the India Government Mint,Kolkata. He is most noted as a philologistfor fully deciphering and translating therock edicts of Asoka from the Brahmi script.

55. (c) Ex-situ conservation is the process ofprotecting an endangered species of plantor animal outside of its natural habitat. Zoosand botanical gardens are the mostconventional methods of ex-situconservation. Endangered plants may alsobe preserved in part through seed banks orgermplasm banks.

56. (b) Diatom ooze (formed from microscopicunicellular algae having cell walls consistingof or resembling silica) is the most widespreaddeposit in the high southern latitudes.

57. (b) Photosynthetic Chromatophores vesiclesfound in some purple bacteria constituteone of the simplest light-harvestingsystems in nature.

58. (a) Convex mirrors reflect light outwards;therefore they are not used to focus light.So, the convex mirror has a wide field ofview and hence is used as rear view mirroras it gives a clear diminished and an erectimage of the traffic that is behind.

59. (b) Mobile Commerce, also known asM-Commerce or mCommerce, is the abilityto conduct commerce using a mobile device,such as a mobile phone, a Personal DigitalAssistant (PDA), a smartphone, or otheremerging mobile equipment.

60. (b) K.T. Shah said that Dr. Rajendra Prasad wonhis first election with 507,400 votes over hisnearest rival K.T. Shah who got 92,827 votes.

61. (b) Most cereals, including wheat, rye, rice,oats, barley, corn, sorghum, millet, greenbristlegrass and pearl millet belong to thebotanical family Gramineae. Orange andlemon are citrus fruits.

62. (c) Tilak propounded the theory of the Arctichome of the Aryans, meaning that theAryans originated in the Arctic region, andlater, on the journey south, divided into twobranches. One branch went to Europe,while the other branch came to India.

63. (b) A cultigen is a plant that has beendeliberately altered or selected by humans;it is the result of artificial selection. These"man-made" or anthropogenic plants are,for the most part, plants of commerce thatare used in horticulture, agriculture andforestry.

64. (c) Article 312 provides that an All India Servicecan be created only if the Council of Statedeclares by a resolution supported by notless than a two-thirds majority that it isnecessary in the national interest to createone or more such All india Services.

65. (b) The principal components of pyroligneousacid are acetic acid, acetone and methanol. Itwas once used as a commercial source foracetic acid.

66. (b) UCIL produced batteries, carbon products,welding equipment, plastics, industrialchemicals, pesticides, and marine products.

67. (b) Bokaro Steel Plant – the fourth integratedplant in the Public Sector – started takingshape in 1965 in collaboration with theSoviet Union. Once in Bihar, it is now inJharkhand.

68. (b) Alara Kalama was a hermit saint and ateacher of yogic meditation who lived nearRajagriha. According to the Pali Canonscriptures, he was one of the teachers ofGautama Buddha.

69. (c) Ram Sharan Sharma was an eminenthistorian of Ancient and early MedievalIndia.

70. (d) The 2011 National Games, also known asthe 34th National Games of India, was heldfrom 12 February 2011 to 26 February 2011in Ranchi, Jharkhand.

71. (b) Retributive justice is a theory of justic thatholds is a proportionals punishment inflicted for its own sake rather than to servean extrinsic social purpose.

72. (d) The concept of a merit good introduced ineconomics by Richard Musgrave (1957,1959) is a commodity which is judged thatan individual or society should have on thebasis of some concept of need, rather thanability and willingness to pay. Examplesinclude the provision of food stamps to

yoursmahboob.w

ordpress.com

126 SSC CGL SolvedPaper

support nutrition, the delivery of healthservices to improve quality of life and reducemorbidity, subsidized housing and arguablyeducation.

73. (c) The Torah is Judaism's most important text.It is composed of the Five Books of Moses.The term "Torah" means instruction andoffers a way of life for those who follow if.

74. (c) Indian Science congress Association(ISCA) is a premier scientific organisationof India with headquarters at Kolkata, WestBengal. The 98th session was held at SRMUniversity, near Chennai in January 2011.

75. (a) Most drying oils owe their drying propertiesto the presence of a large percentage oflinolenic acid (which derives its name from"linseed"), which is highly unsaturated.

76. (c) Conspicuous consumption is the purchaseof goods or services for the specific purposeof displaying one's wealth. It is a means toshow ones social status.

77. (a) Carotenoids are tetraterpenoid organicpigments that are naturally occurring in thechloroplasts and chromoplasts of plantsand some other photosynthetic organismlike algae, some bacteria, and some typesof fungus.

78. (d) The Citibank of New York was the first suchbank.

79. (a) The variations in sunlight affect differentlatitudes differently, as we might expect fromthe different angles at which sunlight fallson the earth's surface at different latitudes.

80. (c) Nagaland had recorded the country’shighest decadal population growth of 64.41per cent in 2001 and 56.08 in 1991respectively.

81. (b) The inclination of the Earth's field is 90°atthe North Magnetic Pole and –90° at theSouth Magnetic Pole.

82. (b) Alfred Marshall was committed to partialequilibrium models over general equilibriumon the grounds that the inherentlydynamical nature of economics made theformer more practically useful.

83. (a) In the original Constitution, Article 338provided for a Special officer, called theCommissioner for SCs and STs, to have theresponsibility of monitoring the effectiveimplementation of various safeguards forSCs/STs in the Constitution as well as otherrelated legislations and to report to thePresident.

84. (a) Polar bears, which pile on fat to survivehibernation and yet do not become diabetic,hold clues for treating Type II diabetes, adisease associated with obesity that afflictsmore than 190 million people worlwide,reaching epidemic proportions in manycountries.

85. (d) All electromagnetic waves regardless oftheir wavelengths, including all colors oflight, have the same identical speed in avacuum.

86. (b) In hydroelectric power plants the potentialenergy of water is utilized to produceelectricity. The height of water in thereservoir decides how much potential energywater possesses.

87. (c) Former Rajasthan Mahila CongressPresident chief Mamta Shrma wasappointed as National Commission forWomen (NCW) chairperson. Theappointment was cleared by the PrimeMinister's Office.

88. (a) These agreements give the right of taxationin respect of the income of the nature ofinterest, dividend, royalty and fees fortechnical services to the country ofresidence. It promotes bilateral trade andinvestment.

89. (d) The High Court of Kerala is headquarteredat Kochi. It is located in Ernakulam.

90. (c) In people with emphysema, the lung tissueinvolved in exchange of gases (oxygen andcarbon dioxide) is impaired or destroyed.Emphysema is included in a group ofdiseases called chronic obstructivepulmonary disease of COPD.

91. (c) Bihar with 1,102 persons per square kilometreis the most densely populated state of Indiaas per the 2011 Census. West Bengal comessecond with 1,029 persons per kilometre asper the option.

92. (c) Brain drain can be described as the processin which a country loses its most educatedand talented workers to other countriesthrough migration. The main causes includeseeking employment or higher paying jobs,political instability and to seek a betterquality of life.

93. (c) The origins of the HDI are found in theannual Human Development Reports of theUnited Nations Development Programme(UNDP). These were devised and launchedby Pakistani economist Mehboob ul Haq in1990.

yoursmahboob.w

ordpress.com

127SSC CGL SolvedPaper

94. (b) Jupiter is the fifth planet from the sun andthe largest planet within the Solar System.

95. (a) Red and Yellow Soil in India is largelyavailable in the Deccan Plateau.

96. (a) As Samhita is the collection of the mantras,so sometimes Samhitas are referred to asMantras. Most of these mantras or hymnsare concerned with nature and deities.

97. (a) Demography is the statistical study ofhuman populations and sub-populations.It encompasses the study of the size,structure, and distr ibution of thesepopulations, and spatial and/or temporalchanges in them in response to birth,migration, aging and death.

98. (c) Potassium Nitrate occurs as a mineral niterand is a natural solid source of nitrogen.Potassium nitrate is one of several nitrogen-containing compounds collectively referredto as saltpeter.

99. (b) Maleeha Lodhi is a journalist, academic anddiplomat from Pakistan. She was the HighCommissioner of Pakistan to the UnitedKingdom and is a former PakistaniAmbassador to the United States.

100. (d) Ganesh Vasudev Mavalankar, popularlyknown as Dadasaheb, was an independenceactivist, the President (from 1946 to 1947)of the Central Legislative Assembly, thenSpeaker of the Constituent Assembly ofIndia, an later the first Speaker of the LokSabha.

101. (a) cot A + cosec A = 3cosec2 A – cot2 A = 1(cosec A – cot A) (cosec A + cot A) = 1

cosec A – cot A =13

cosec A + cot A = 3By Adding

2cosec A = 1 1033 3

+ =

cosec A 106

5 H3 P

=

2 2B 5 3 4= - =

cos A 45

=

102. (d) 4 a 39 8 4

1 3 b 7Q 13b7 is exactly divisible by 11.\ b = 9 \ a = 1\ a + b = 9 + 1 = 10

103. (b) A

B CLet BC = x\ AB = x + 2\ AB2 + BC 2 = AC2

Þ (x + 2)2 + x2 = 2(2 5)Þ x2 + 4x + 4 + x2 = 20Þ 2x2 + 4x – 16 = 0Þ x2 + 2x – 8 = 0Þ x2 + 4x – 2x – 8 =0Þ x (x+ 4) – 2 (x + 4) = 0Þ (x – 2) (x + 4) = 0Þ x = 2 = BC\ AB = 2 + 2 = 4cm

2 22 2

2 2AB BCcos A cos CAC AC

\ - = -

16 4 12 320 20 20 5= - = =

104. (d) D

C A

B

50 3Ö60° 30°

O

metre

AB = CD = 50 3 metreFrom D OAB,

tan 30°ABOA

=

1 50 3OA3

Þ =

Þ OA = 50 3 3 150 metre´ =From DOCD,

tan 60° CDOC

=

50 33 OC 50 metreOC

= Þ =

yoursmahboob.w

ordpress.com

128 SSC CGL SolvedPaper

\ BD = AC = 150 + 50= 200 metre\ Speed of bird

200 1002

= = m/minute

100 60 kmph1000

= ´

= 6 kmph

105. (a) A

B CAB = BC = x

\ 2 2 2 2AC AB BC x x= + = +

2x units=

2x 2x 2p\ + =

x(2 2) 2pÞ + =

2p 2p(2 2)x2 2 (2 2)(2 2)

-Þ = =+ + -

2(2 2)p (2 2)p4 2-= = --

\ Area of triangle = 21 x2

2 21 (2 2) p2

= ´ - 24 2 4 2 p2

+ -=

2(3 2 2)p= - sq.units

106. (b)2

2ABC 64 BCDEF 121 EF

D = =D

8 BC 8 BC11 EF 11 15.4

Þ = Þ =

8 15.4BC 11.2cm11

´Þ = =

107. (c) In D ABCGiven AG = BC

A

B D C

G1 AG2

= 1 BC2

i.e., GD = BD = DC

In D BGDBD = DG \ ÐGBD = ÐDGB …(i)In D CGDGD = DC, \ ÐGCD = DDGC …(ii)ÐGBD + ÐDGB + ÐDGC + ÐDCG = 1802(ÐBGD + ÐCGD) = 180

ÐBGC = 180 90

2= °

108. (b) tan (x + y) tan (x – y) = 1Þ tan (x + y) = cot (x – y)= tan (90° – (x – y))Þ x + y = 90° – ( x – y)Þ 2x = 90° Þ x = 45°\ tan x = tan 45° = 1

109. (d) Ratio of profit sharing1 1 1 1 7: : 16 6 4 4 12

= ´ ´ ´ 1 1 7: :36 16 12

=

1 1 7144 : 144 : 14436 16 12

= ´ ´ ´ = 4 : 9 : 84

Sum of ratios = 97

9B's share 1940097

\ = ´ = ̀ 1800

110. (c) Liquid A = 4x litreLiquid B = x litreIn 10 litres of mixture,

Liquid A 4 10 85

= ´ = litre

Liquid B = 2 litre

4x 8 2x 2 10 3

-\ =- +

Þ 12x – 24 = 2x + 16Þ 10x = 40 Þ x = 4\ Initial quantity of liquid A = 16 litre

111. (c)2 25x 3y 112y 2- =

10x2 – 6y2 = 11xy10x2 – 11xy – 6y2 = 010x2 – 15xy + 4xy – 6y2 = 05x(2x – 3y) + 2y(2x – 3y) = 0(5x + 2y) (2x – 3y)5x ¹ 2y, 2x = 3y

x 3y 2

=

yoursmahboob.w

ordpress.com

129SSC CGL SolvedPaper

112. (a) 2x + 3x + 5x = 180° – 45° = 135°Þ 10x = 135°

135 27x10 2

Þ = =

\ Largest angle275x 15 5 152

æ ö= + ° = ´ ° + °ç ÷è ø

135 30 1652 2+ °= =

180 radian° = pQ

165 1652 180 2° p\ = ´ 11

24p= radian

113. (d) 4 cosec2a + 9 sin2a= 4 cosec2a + 4 sin2a + 5 sin2a= 4 [coseca – sina)2+2] + 5 sin2a= 12 [Q coseca – sina ³1]

114. (d) Required number = HCF of(35 – 19), (59 – 35) and (59 – 19)= HCF 16, 24 and 40 = 8

115. (c) M + T + W = 90° ...(i)T + W + Th = 99° ...(ii)By equation (ii) – (i)Th – M = 9° Þ Th – 32 = 9Þ Th = 9° + 32 = 41°

116. (b)3 2a3 2-=+

3 2 3 23 2 3 2- -= ´+ -

( )23 2 3 2 2 63 2-= = + --

= 5 – 2 6

3 2b 5 2 63 2

+\ = = +-

Þ a + b = 10;

( )( )ab 5 2 6 5 2 6= - + = 25 – 24 = 1

2 2 3 3a b a bb a ab

+\ + =

( ) ( )3a b 3ab a bab

+ - +=

= 103 – 3 × 10 = 1000 – 30 = 970

117. (c) The pattern is :13 – 2 = –123 – 2 = 633 – 2 = 2543 – 2 = 6253 – 2 = 12363 – 2 = 21473 – 2 = 341

118. (d) A

B CD

O

BCBD 12 cm2

= =

OB = 13 cmFrom D OBD,

2 2OD OB BD= = -

2 213 12 169 144= - = - 25 5 cm= =119. (a) ax + by = 6 ...(i)

bx – ay = 2 ...(ii)On squaring and adding,a2x2 + b2y2 + 2abxy + b2x2 + a2y2 – 2abxy= 36 + 4Þ x2 (a2 + b2) + y2 (a2 + b2) = 40Þ (a2 + b2) (x2 + y2) = 40Þ (a2 + b2) × 4 = 40Þ a2 + b2 = 10

120. (b) A

B O C

1OA BC radius2

= =

Area of the largest triangle

1 1BC OA 2x x2 2= ´ ´ = ´ ´ = x2

yoursmahboob.w

ordpress.com

130 SSC CGL SolvedPaper121. (a) A

B C

DE

P

DE | | BC1DE BC2

=

\ D BDE = D DEP 2D BDE = D BEC\ D ADE = D BDE\ D ABC = 4 D ADE

1PED ABC4

\ = D

122. (c)1a 1a

+ =

a2 + 1 = aa2 – a + 1 = 0Multiplying both side by (a + 1)(a + 1) (a2 – a + 1) = 0a3 + 1a3 = –1

123. (c) 10th observation= 24 × 10 + 17 × 10 – 19 × 24= 240 + 170 – 456 = – 46

124. (b) Difference of P% and L% = 30 – (– 10)= 40%40 x 80

100=

x = 200125. (d) Let the length of train be x km and its speed

by kmph.x 9 1

y 2 3600 400\ = =

- ...(i)

x 10 1y 4 3600 360

= =- ...(ii)

By dividing equation (i) by (ii),y 4 360 9y 2 400 10

- = =-

Þ 10y – 40 = 9y – 18Þ y = 40 – 18 = 22From equation (i),

x 122 2 400

=-

1x km20

Þ = 1000 50 metre20

= =

126. (b)P

O O1E

F

20 15

25

Q 15 20 25Þ 3 : 4 : 5 and forming at D

or 1 120 15 25 x2 2

´ = ´

12 = xchord = 2 × 12 = 24 cm

127. (c) C.P. of article = ̀ 100

100 120Marked price90´\ = 400

3= `

400 400 8585% of3 3 100

\ = ´ 3403

= `

340 40 1Gain 100 13 %3 3 3

= - = =

128. (d)5 3 5 32 x5 3 5 3

+ -= -- +

( ) ( )( )( )

2 25 3 5 3

5 3 5 3+ - -=

- +

4. 5. 3 2 155 3

= =-

2 x 2 15 x 15\ = Þ =129. (b) x 2 3= +

1 1 1 2 3x 2 3 2 3 2 3

-= = ´+ + -

2 3 2 34 3-= = --

21 1x x 2xx

æ ö\ + = + +ç ÷è ø

2 3 2 3 2= + + - +

1x 6x

\ + =

yoursmahboob.w

ordpress.com

131SSC CGL SolvedPaper

130. (a) Q 12 men º18 women\ 2 men º 3 women\ 8 men + 16 women= 28 women\ M1D1 = M2D2Þ 18 × 14 = 28 × D2

218 14D 9days

28´Þ = =

131. (d) C.P. of 9 articles100 2596 24

= = `

\ S.P. for a gain of 44 %

25 144 324 100 2

= ´ = `

\ Required number of articles

29 63

= ´ =

132. (a)2sin A 1 cos A sin A1

1 cos A sin A 1 cosA+- + -

+ -

2 2 2(1 cosA)(1 cos A) 1 cos A sin A11 cos A sin A(1 cosA)

- + - -- ++ -

2 2sin A sin A1 1 cosAsin A(1 cos A)

-- + +-

Þ cosA133. (c) (a – b)3 = a3 – b3 – 3ab (a –b)

Þ 8 = 56 – 3ab (2)Þ 6ab = 56 – 8 = 48Þ 2ab = 16 ...(i)\ a2 + b2 = (a – b)2 + 2ab= 4 + 16 = 20

134. (a) Put q = 45°a = tan 45° – cot 45°, b = sin 45° – cot 45°a = 1 – 1 = 0

b = 1 1 02 2

- =

Put in equation(a2 + 4) (b2 –1)2 = (0 + 4) (0 – 1)2 = 4

135. (b) For 3x + 4y = 12,By putting x = 0, y = 3By putting, y = 0, x = 4For 6x + 8y = 60,

By putting x = 0, 15y2

=

By putting y = 0, x = 10

15C 0,2

æ öç ÷è ø

A

Y

Y1

XX1

DBO (10, 0)

(0, 3)

(4, 0)

\ Area of D OCD1 OD OC2

= ´ ´ 1 15 75102 2 2

= ´ ´ =

\ Area of D OAB1 OB OA2

= ´ ´1 4 3 62

= ´ ´ =

\ Area of trapezium 75 62

= -

75 12 632 2-= = = 31.5 sq. units

136. (c) Putting x = 0 in2x – 3y + 6 = 0Þ y = 2Putting y = 0 in 2x + 3y + 6 = 0Þ x = – 3

A

Y

Y1

XX1

BO

(0, 2)

(–3, 0)

Area of D OAB

1 OB OA2

= ´ ´ 1 3 2 32

= ´ ´ = sq. units.

yoursmahboob.w

ordpress.com

132 SSC CGL SolvedPaper

137. (c) (a2 –b2) sin q + 2ab cos q = a2 + b2

2 2

2 2 2 2a b 2absin cos 1a b a b

- q + q =+ +

2 2

2 2 2 2a b 2absin cosa b a b

-q = q =+ +

{sin2q + cos2q = 1}2 2sin a btan

cos 2abq -q = =q

138. (c) Difference of rates = 4%Pr incipal Time Rate

100´ ´\

= 960x 12 4 960

100´ ´Þ =

960 100x 200012 4

´Þ = =´

`

139. (c)

AO B

C

D

P

In D OCD,OC = OD = CD = rD OCD is an equilateral triangle,ÐCOD = 60°

ÐCBD1 COD 302

= Ð = °

ÐACB = 90°ÐBCP = 180° – ÐACB= 180° – 90° = 90°In D BCP,ÐBCP = 90°, ÐCBP = ÐCBD= 30°\ ÐBCP + ÐCBP + Ð CPB= 180°Þ 90° + 30° + ÐCPB = 180°Þ ÐCBP = 60° Þ ÐAPB = 60°

140. (b) Length of common tangent

( )22d R r= - -

141. (b)

A

BC

O

P

OC ̂ ABAC = BC = 3.5 cm OP = 13 cmPC = 9 + 3.5 = 12.5 cm

2 2OC OP PC\ = -

2 213 12 12.75= - =

2 2OA OC CA\ = + 212.75 (3.5)= +

12.75 12.25 25 5 cm= + = =

142. (a)1a 0

a 2+ =

+1a 2 2

a 2Þ + + =

+On cubing,

( )31a 2 8

a 2é ù+ + =ê ú+ë û

( )( )

( )

( )

33

1a 2 3 a 2a 2

1 1a 2 8a 2 a 2

Þ + + + ++

æ ö´ + + =ç ÷+ +è ø

( )( )

33

1a 2 3 2 8a 2

Þ + + + ´ =+

( )( )

33

1a 2 8 6 2a 2

Þ + + = - =+

143. (a) 2 sin a + 15 cos2 a = 7Þ 2 sin a + 15 (1 –sin2 a) = 7Þ 2 sin a + 15 – 15 sin2 a = 7Þ 15 sin2 a – 2 sin2 a – 8 = 0Þ 15 sin2 a – 12 sin a + 10 sin a – 8 = 0Þ 3 sin a (5 sin a – 4) + 2 (5 sin a – 4) = 0Þ (3sin a + 2) ( 5 sin a – 4) = 0Þ 5 sin a – 4 = 0

yoursmahboob.w

ordpress.com

133SSC CGL SolvedPaper

Þ 5 sin a 45

=

\ cosec a 54

=

2cot cos ec 1a = a -

25 9 3116 16 4

= - = =

144. (c) ( )

( )21 876542 876542 2

876542 1

+ +

+

( )

( )

2

21 876542 2 876542

876542 1

+ + ´=+

( )

( )

2

2876542 1 1876542 1

+= =+

145. (c)AB BC CAPQ QR RP

= = AB BC CAPQ QR RP

+ +=+ +

AB 36PQ 24

Þ =

AB 3610 24

Þ =

36 10AB 15 cm24´Þ = =

146. (c) Required ratio = 2 : 5147. (a) It is obvious from the bar diagram.

148. (a) Required average121 30.25 lakhs

4= =

149. (d) Required range = 16 – 3 = 13 lakhs

150. (b) Required percentage 46 100 38121= ´ =

151. (d)152. (b) When we use Along with, together, with,

with, verb agrees according to first subject.Hence, is going to Delhi should be used.

153. (b) Here, in getting (gerund) the job should beused.

154. (c) Here, are located at the Madison Avenueshould be used.

155. (c) Here, which she has given in respect of youreyes should be used.

156. (a) 157. (d) 158. (d) 159. (c) 160. (b)161. (c) The word Annexure (Noun) means :

attachment; appendix.

162. (b) The word Errand (Noun) means : a job thatyou do for somebody; task.

163. (a) The word Bequeath (Verb) means : leave;to say in a will that you want somebody tohave your property etc., give.

164. (b) The word Nonchalant (Adjective)means : careless; indifferent; behaving in acalm and relaxed way; casual.

165. (b) The word Forbearance (Noun) means :patience; the quality of being patient andsympathetic towards other people.

166. (d) The word Amenable (Adjective) means :responsible; responsive; easy to control;willing. Its antonym should be unwilling.

167. (d) Thye word Conspicuous (Adjective) means: eminent, easy to see or notice; obvious.The word Obscure (Adjective) means :difficult to understand; not well known;unknown.

168. (a) The word Reproof (Noun) means :condemnation; blame, rebuke; disapproval.The word Approbation (Noun) means :approval or agreement.

169. (c) The word Niggard (Adjective) means :mean; miserly; unwilling to be generous withmoney.The word Generous (Adiective) means :giving on willing to give freely; lavish.

170. (d) The word Exotic (Adjective) means : fromor in another country.The word Indigenous (Adjective) means :native; belonging to a particular place ratherthan coming to it from somewhere else.

171. (a) Idiom take the wind out of somebody's sailsmeans : to make somebody suddenly lessconfident or angry when you say or dosomething that they do not expect.

172. (c) Idiom pour/throw cold water on somethingmeans : to give reasons for not being infavour of something; to criticize something

173. (a) Idiom let your hair down means : to relaxand enjoy your-self especially in a lively way.

174. (a) Idiom jump out of your skin means : to moveviolently because of a sudden shock.

175. (a) Idiom take somebody for a ride means : tocheat or trick somebody.

176. (b) 177. (d) 178. (c) 179. (b)180. (c) Tuck somebody in/up means : to make

somebody fell comfortable in bed by pullingthe covers up around them.

181. (c) 182. (d) 183. (a) 184. (d) 185. (a)186. (c) 187. (c) 188. (b) 189. (b) 190. (c)191. (b) 192. (d) 193. (a) 194. (c) 195. (c)196. (a) 197. (b) 198. (c) 199. (a) 200. (c)

yoursmahboob.w

ordpress.com

134 SSC CGL Solved Paper

PART-A : GENERAL INTELLIGENCE & REASONING

Directions (1-9): In each of the following questions,select the related letter/word/number from the givenalternatives.1. 9 : 80 : : 100 : ?

(a) 901 (b) 1009(c) 9889 (d) 9999

2. 25 : 125 : : 36 : ?(a) 180 (b) 206(c) 216 (d) 318

3. 335 : 216 : 987 : ?(a) 868 (b) 867(c) 872 (d) 888

4. BEGK : ADFJ ::PSVY?(a) ROUX (b) ORUX(c) LQUT (d) LOQT

5. AZBY : CXDW :: EVFU : ?(a) GTHS (b) GHTS(c) GSTH (d) TGSH

6. BDFH : SUWY :: CEGI : ?(a) QTWZ (b) PTVX(c) JLNP (d) TVXZ

7. food : man :: fuel:?(a) wood (b) fire(c) heat (d) smoke

8. Spiritual : Belief :: Orchestral: ?(a) Theatre (b) Situation(c) Music (d) Direction

9. Latter : Former :: Hazardous : ?(a) Risky (b) Comfortable(c) Safe (d) Harmful

Directions (10-18): In each of the following questions,find the odd word/number/letters/word/number pairfrom the given alternatives.10. (a) VWY (b) QRT

(c) LMO (d) JKL11. (a) BE (b) GJ

(c) NP (d) QR12. (a) 400 (b) 484

(c) 625 (d) 729

13. (a) 1000 (b) 1725(c) 2744 (d) 4096

14. (a) 12–16 (b) 60–80(c) 30–50 (d) 36–48

15. (a) CX (b) DW(c) JQ (d) LR

16. (a) Cyclotron (b) Basic(c) Pascal (d) Fortran

17. (a) Rooster (b) Buck(c) Gander (d) Peahen

18. (a) Booklet (b) Pamphlet(c) Voucher (d) Brochure

19. Which word will appear third in thedictionary?(a) BALLIUM (b) BALLISTICS(c) BALLERINA (d) BALLISTITE

20. Which word will appear last in the dictionary?(a) laugh (b) latch(c) laurels (d) latitude

21. Arrange the given words in a meaningfulorder:1. INFANT2. ADOLESCENT3. CHILD4. OLD5. ADULT(a) 3, 1, 2, 4, 5(b) 1, 3, 2, 5, 4(c) 3, 2, 4, 5, 1(d) 5, 4, 3, 2, 1

Directions (22-23): In each of the followingquestions,which one set of letters whensequentially placed at the gaps in the given letterseries shall complete it?22. c_ab_ca_bc_a

(a) b c a b (b) a b c b(c) b a c b (d) c b a c

23. _ba _ bab_babb_b(a) b a a a (b) a b b b(c) b a b b (c) a b a b

SSC Combined Graduate Level (CGL) Solved PaperMORNING SHIFT 08 JULY, 2012

yoursmahboob.w

ordpress.com

135SSC CGL Solved Paper

Directions (24-27): In each of the followingquestions, select the missing number from the givenresponses.24. 500, 484, 459, __?__, 374

(a) 384 (b) 432(c) 418 (d) 423

25. MNOPWXYZRSTUBCD__?__(a) A (b) E(c) I (d) F

26. 60, 69, 85, __?__, 146(a) 110 (b) 117(c) 109 (d) 120

27. 46, 50, 47, 55, 49, 61, __?__(a) 54 (b) 52(c) 57 (d) 51

28. The average age of 25 suboridinates in an officeis 30 years. If the age of Manager is added, theaverage increases to 31 years. What is the ageof the Manager?(a) 26 (b) 36(c) 46 (d) 56

29. Class A has students twice that of class B. Afteradding 20 students to class A and 30 studentsto class B, the total number of students in boththe classes is 140. What is the number ofstudents in class A in the beginning?(a) 30 (b) 60(c) 80 (d) 140

30. Find the lowest number which when dividedby 8, 12, 15 and 20 leaves the remainder 2.(a) 360 (b) 242(c) 122 (d) 82

31. If B becomes A and P becomes O, what will Kbecome in the English alphabet?(a) L (b) J(c) H (d) N

32. The given word is followed by 4 responses, oneof which cannot be formed by using the lettersof the word given ‘CAMBRIDGE’. Find outthe word.(a) BRIDGE (b) BRIDE(c) CAME (d) CAMP

33. If SEASONAL is written as ESSANOLA, howcan SEPARATE be written in that code?(a) SEAPARET (b) ESPARATE(c) ESPAARTE (d) ESAPARET

34. In certain code, RAGHAVAN is written asGARVAHNA. In that code which word will bewritten as MATHAVAN?(a) TAMVAHNA (b) TAMVAHAN(c) TAMHAVNA (d) MATVAHNA

35. If ‘FRIEND’ is coded as ‘IULHQG’, how willyou code ‘ENEMY’?(a) HQHPB (b) HQHPA(c) HQEMY (d) HQHPG

36. In a coded language, if the word EQUATIONis coded as GSWCVKQP, then how is the wordDONKEY coded?(a) FQPMGA (b) YEKNOD(c) GWCVKJ (d) PQKUCW

37. If 38 + 15 = 66 and 29 + 36 = 99, then 82 + 44 = ?(a) 77 (b) 88(c) 80 (d) 94

38. If + means ¸, means ×, × means +, ¸ means –,give the value for 45 + 9 – 3 × 15 ¸ 2(a) 40 (b) 36(c) 56 (d) 28

Directions (39-40): In each of the following questions,select the missing number from the given responses.39. 7 8 6

4 9 53 2 ?25 70 29(a) 9 (b) 8(c) 1 (d) 5

40. 3 4 56 7 89 1 2

57 11 ?(a) 42 (b) 21(c) 11 (d) 18

41. From the given details, estimate the number ofpeople affected by Tuberculosis in particularlocality in the year 1994.1994 1995 1996 1997 1998 ? 92 113 141 176(a) 99 (b) 85(c) 71 (d) 78

42. A man started from a place walked towardsNorth for 5 km then turned 90° to his right andwalked another 5 km. then he turned 45° to hisright and walked 2 km and turned 45° to hisleft. What is his direction now?(a) South (b) South East(c) East (d) South East

43. A boy running towards South, turns to his rightand runs. Then he turns to his right and finallyturns to his left. Towards which direction is herunning now?(a) East (b) West(c) South (d) North

yoursmahboob.w

ordpress.com

136 SSC CGL Solved Paper

Directions (44-45): In the following questions, onestatement is given, followed by two conclusions Iand II. You have to consider the statements to betrue even if they seem to be at variance fromcommonly known facts. You have to decide which ofthe given conclusions, if any, follow from the givenstatements.44. Statement

To keep myself updated, I always listen to9 O’clock news on radio.Conclusions:I. The persons does not read newspaper.II. Recent news is available only on radio.(a) Only conclusion I is implicit(b) Only conclusion II is implicit(c) Neither conclusion I nor II is implicit(d) Both conclusions I and II are implicit

45. StatementAll wealthy people are happy in life.Conclusions:I. Wealth and happiness are directly related.II. Wealth leads to happiness in life.(a) Only conclusion I follows(b) Only conclusion II follows(c) Both conclusions I and II follow(d) Neither conclusion I nor II follows

Directions: In the following questions, which answerfigure will complete the question figure?46. Question Figure

Answer figure

(1) (4)(2) (3)47. From the given answer figures, select the one

in which the question figure is hidden/embedded.Question Figure

Answer figures

(a) (b) (c) (d)

Directions: In the following question, a piece ofpaper is folded and cut as shown below in thequestion figures. From the given answer figures,indicate how it will appear when opened.48. Question Figures:

Answer figures:

(a) (b) (c) (d)49. Select the correct option that will be the mirror

reflection of the problem figure.Question Figure:

fAnswer figures:

(a) (b) (c) (d)

f

f t

tDirections: In the following question, a word isrepresented by only one set of numbers as given inany one of the alternatives. The sets of numbers givenin the alternatives are represented by two classes ofalphabets as in two matrices given below. Thecolumns and rows of Matrix I are numbered from 0to 4 and that of Matrix II are numbered from 5 to 9.A letter from these matrices can be represented firstby its row and next by its column, e.g., ‘P’ can berepresented by 11, 32, etc. and ‘K’ can be representedby 65, 78, etc. Similarly, you have to identify the setfor the word SALM.

yoursmahboob.w

ordpress.com

137SSC CGL Solved Paper

50. MATRIX-I0 1 2 3 4

O P A I V R1 1 P R A V2 A R V P I3 V I P R A4 R V A I P

MATRIX-II5 6 7 8 9

5 S L K M E6 K M S E L7 M E L K S8 L K E S M9 E S M L K

(a) 55, 20, 56, 59 (b) 79, 13, 69, 75(c) 96, 34, 76, 89 (d) 67, 21, 85, 97

PART-B : GENERAL AWARENESS51. Non-violence as taught and practised by

Mahatma Gandhi is rooted in the IndianDoctrine of(a) Swaraj (b) Swadeshi(c) Satyagraha (d) Ahimsa

52. The Social Contract theory deals with(a) The Purpose of State(b) The Origin of the State(c) The Nature of State(d) The Functions of State

53. Who was the founder of the Aligarh Movement ?(a) Syed Amir Ali(b) Maulvi Chiragh Ali(c) Sir Syed Ahmed Khan(d) Abdul Halim Sharar

54. ‘Prince of Pilgrims’ was the name attributed to(a) Plutarch (b) Hiuen Tsang(c) Fa-Hien (d) I-Tsing

55. Where did Buddha deliver his first sermon ?(a) Sarnath (b) Rajagriha(c) Kapilavastu . (d) Bodh-Gaya

56. Name the Mughal Prince, who translatedBhagavat Gita into Persian ?(a) Dara Shukoh (b) Sulaiman Shukoh(c) Khusru (d) Murad

57. The surgery that was practised in ancient Indiais known from the works of which of thefollowing scholars ?(a) Atreya (b) Sushruta(c) Charaka (d) Vagbhata

58. ‘Eutrophication’ is associated with(a) Nitrates and Phosphates(b) Sewage(c) Silt load(d) Vegetation

59. The BSE Sensex is based on how many stocks ?(a) 80 (b) 100(c) 30 (d) 50

60. The funds raised by the Government within thecountry is known as(a) Domestic budgetary source(b) Domestic banking source(c) Domestic saving source(d) Domestic monetary source

61. Sick units are referred to(a) SIA (b) DGTD(c) FIPB (d) BIFR

62. The concept ‘Duopoly’ was introduced by(a) Sweezy (b) Malthus(c) Ricardo (d) Cournot .

63. Which of the following is the feature ofmonopolistic competition ?(a) Single firm(b) Large number of firms(c) Group of firms(d) None of the above

64. The Supreme Court of India offers advice tothe President on matters of Legal, Public orConstitutional importance based on(a) Article-148 (b) Article-129(c) Article-147 (d) Article-143

65. Which is the most effective means of executivecontrol of administration ?(a) Financial administration(b) Political direction(c) Appointment and removal of top officials(d) Subordinate legislation

66. Violation of‘Rule of Law’ arises mostly from(a) Lack of Checks and Balances(b) Executive’s Quasi-Judicial Powers(c) Limited Franchise(d) Delegated Legislation

67. The best way to maintain a natural equilibriumbetween the pest and predator is by using(a) insecticides (b) biological control(c) pesticides (d) herbicides

68. ‘Dry ice’ is the condensed form of(a) sulphur tri-oxide(b) carbon dioxide(c) highly cooled water(d) oxygen

yoursmahboob.w

ordpress.com

138 SSC CGL Solved Paper

69. The materials which are strongly attracted bymagnet are called(a) ferro-magnetic substances(b) universal substances(c) para-magnetic substances(d) dia-magnetic substances

70. In a pin-hole camera, we usually get(a) erect impression(b) inverted impression(c) erect image(d) inverted image

71. What happens to the kinetic energy of gasmolecules with rise of temperature ?(a) Remains same (b) Fluctuates(c) Increases (d) Decreases

72. Who invented the modem ?(a) Apple Computers Inc.(b) Digital Equipment Corporation(c) Wang Laboratories Ltd.(d) AT&T Information Systems

73. What is the name given to the smallest unit ofdata space available in a spreadsheet softwareapplication ?(a) Row (b) Block(c) Cell (d) Pixel

74. The land between two rivers is called(a) Natural Levees (b) Alluvial Cones(c) Braided Stream (d) Doab

75. Which year is called as the ‘Demographicdivide’ ?(a) 1941 (b) 1921(c) 1901 (d) 1931

76. Which country has a high density of population?(a) India (b) Canada(c) Sweden (d) Greenland

77. The iron and steel plant in Chhattisgarh is at(a) Burnpur (b) Salem(c) Bhilai (d) Bokaro

78. The sex of a child is determined(a) six to seven weeks after conception(b) in the third month of pregnancy(c) at the time of sperm’s entry(d) at the time of fertilisation of ovum

79. The food chain of the ecosystem is maintainedby the activities of(a) Decomposers (b) Predators(c) Producers (d) Consumers

80. Which one of these primates is closest to themodern man ?(a) Orang-utan (b) Chimpanzee(c) Gorilla (d) Gibbon

81. A gram of fertile agricultural soil may containbacteria upto(a) five million(b) one billion and above(c) fifty thousand(d) five hundred thousand

82. The leading sesame producing country in theworld is(a) Mexico (b) U.S.A.(c) China (d) India

83. What is the name of the currency of Bangladesh ?(a) Taka (b) Lek(c) Dinar (d) Peso

84. The famous car (chariot) festival is held at(a) Puri (b) Gwalior(c) Bhopal (d) Konark

85. The most literate Union Territory in India is(a) Chandigarh (b) Lakshadweep(c) Delhi (d) Pondicherry

86. The first Secretary-General of the UnitedNations was(a) U. Thant(b) Dr. Kurt Waldheim(c) Dag Hammarskjold(d) Trygve Lie

87. Which of the following has been ranked firstin the list of top global cities index releasedin 2011 ?(a) Tokyo (b) New York(c) London (d) Washington

88. The book “Tinderbox : The Past and Future ofPakistan” is written by(a) Jamil Ahmad (b) Khatija Akbar(c) Khurram Iqbal (d) M.J. Akbar

89. The 37th G-8 Summit was held in 2011 at(a) Paris (b) Moscow(c) Montreal (d) Deauville

90. Bina Refinery, which was dedicated to thenation in 2011, is located in(a) Madhya Pradesh (b) Gujarat(c) Orissa (d) Manipur

91. Golden Player Award in the IPL-IVChampionship (2011) was won by(a) Lasith Malinga (b) Virat Kohli(c) Chris Gayle (d) Kieron Pollard

92. A reduction reaction involves(a) addition of oxygen(b) addition of nitrogen(c) addition of hydrogen(d) None of the above

yoursmahboob.w

ordpress.com

139SSC CGL Solved Paper

93. An antiknock for petrol is(a) Sodium hydroxide(b) Ethanol(c) Sodium benzoate(d) Lead tetraethyl

94. Which one of the following pairs is not correctlymatched ?(a) Vitamin C - Scurvy(b) Vitamin K - Clotting of blood(c) Vitamin A - Night blindness(d) Vitamin E - Rickets

95. The fibre that will yield ammonia whendestructively distilled is(a) Wool (b) Cellulose acetate(c) Cotton (d) Silk

96. During respiration, energy is derived from(a) ATP (b) Chlorophyll(c) RNA (d) DNA

97. When was a global network of daily temperaturerecords created ?(a) Around 1890 (b) Around 1920(c) Around 1800 (d) Around 1850

98. Sex hormones are(a) Vitamins (b) Alkanes(c) Carbohydrates (d) Steroids

99. Which of the following cities in India isconsidered the greenest ?(a) Delhi(b) Thiruvananthapuram(c) Bangalore(d) Chandigarh

100. Synagogue is the place of worship of(a) Zoroastrianism (b) Taoism(c) Judaism (d) Shintoism

PART-C : QUANTITATIVE APTITUDE

101. The next term of the series 1, 5 12, 24, 43 is(a) 51 (b) 62(c) 71 (d) 78

102. The least multiple of 13 which when dividedby 4, 5, 6, 7 leaves remainder 3 in each case is(a) 3780 (b) 3783(c) 2520 (d) 2522

103. The simplest value of

1 12 3 3 4

+ ++ +

1 14 5 5 6

++ +

is

(a) ( )3 2 1- (b) 2 3 1)-

(d) 3 1- (d) 2 1-

104. A and B are partners in a business. A contributes

14

of the capital for 15 months and B received

23 of the profit. Find for how long B’s money

was used?(a) 6 months (b) 8 months(c) 10 months (d) 12 months

105. If 21 is added to a number, it becomes 7 lessthan thrice of the number. Then the number is(a) 14 (b) 16(c) 18 (d) 19

106. Two men A and B started a job in which A wasthrice as good as B and therefore took 60 daysless than B to finish the job. How many dayswill they take to finish the job, if they startworking together?(a) 15 days (b) 20 days

(c)122 days2

(d) 25 days

107. A rectangular garden is 100 m × 80 m. Thereis a path along the garden and just outside it.Width of path is 10 m. The area of the path is(a) 1900 sq m (b) 2400 sq m(c) 3660 sq m (d) 4000 sq m

108. A dealer offered a machine for sale for `27,500but even if he had charged 10% less, he wouldhave made a profit of 10% . The actual cost ofthe machine is(a) `22,000 (b) `24,250(c) `22,500 (d) `22,275

109. An employer reduces the number of employeesin the ratio 8: 5 and increases their wages inthe ratio 7:9. As a result, the overall wages bill is(a) Increased in the ratio 56 : 69(b) Decreased in the ratio 56 : 45(c) Increased in the ratio 13 : 17(d) Decreased in the ratio 17 : 13

110. The average age of a jury of 5 is 40. If a memberaged 35 resigns and a man aged 25 becomes amember, then the average age of the new jury is(a) 30 (b) 38(c) 40 (d) 42

111. With average speed of 40 km/hour, a trainreaches its’ destination in time. If it goes withan average speed of 35 km/hour, it is late by15 minutes. The total journey is(a) 30 km (b) 40 km(c) 70 km (d) 80 km

yoursmahboob.w

ordpress.com

140 SSC CGL Solved Paper

112. A man makes a profit of 20% on the sale byselling 20 articles for ̀ 1. The number of articleshe bought by ̀ 1cs(a) 20 (b) 24(c) 25 (d) 30

113. The number of seats in an auditorium isincreased by 25%. The price of a ticket is alsoincreased by 12%. Then the increase in revenuecollection will be(a) 40% (b) 35%(c) 45% (d) 48%

114. A ship is moving at a speed of 30 km/hr. Toknow the depth of the ocean beneath it, it sendsa radiowave which travels at a speed 200 m/s.The ship receives the signal after it has moved500 m. The depth of the ocean is

(a)143 km2

(b) 12 km

(c) 6 m (d) 8 km115. A person takes a loan of `10,000 partly from a

bank at 8% p.a. and remaining from anotherbank at 10% p.a. He pays a total interest of ̀ 950per annum. Amount of loan taken from the firstbank (in `) is(a) 2500 (b) 5200(c) 2050 (d) 5020

116. If ( )221a 98 a 0 ,

a+ = > then the value of

33

1aa

+ will be

(a) 535 (b) 1030(c) 790 (d) 970

117. If x = 1 + 2 3,+ then the value of(2x4 – 8x3 – 5x2 + 26x – 28) is(a) 6 6 (b) 0

(c) 3 6 (d) 2 6118. If the distance between two points (0,–5) and

(x,0) is 13 unit, then x =(a) 10 (b) 10±(c) 12 (d) 12±

119. If 4x = 18y, then the value of x 1yæ ö-ç ÷

è øis

(a)13 (b)

72

(c)23 (d)

32

120. If 1x 15,x

+ = t hen the value of 4

2

2

1xx

x 3x 1

+

- +is

(a) 70 (b) 50(c) 110 (d) 55

121. If x = 2 3, y 2 3,+ = - then the value of

2 2

3 3x yx y

++

is

(a)738 (b)

740

(c)7

19 (d)726

122. If a2 + b2 + c2 = 2(a – b – c) – 3 then the value of2a – 3b + 4c is(a) 3 (b) 1(c) 2 (d) 4

123. If 2x – 1 6,

2x= then the value of

22

1x16x

+ is

(a)192

(b)172

(c)183 (d)

152

124. If 15a 5,3a

+ = the value of 2

219a

25a+ is

(a)345 (b)

395

(c)425 (d)

525

125. The area of the triangle formed by the lines5x + 7y = 35, 4x + 3y = 12 and x-axis is

(a)160 sq. unit13 (b)

150 sq. unit13

(c)140 sq. unit13 (d) 10 sq. unit

126. In an obtuse-angled triangle ABC, ÐA is theobtuse angle and O is the orthocenter. If ÐBOC= 54°, then ÐBAC is(a) 108° (b) 126°(c) 136° (d) 116°

yoursmahboob.w

ordpress.com

141SSC CGL Solved Paper

127. If the ratio of areas of two similar triangles is9 : 16, then the ratio of their corresponding sidesis(a) 3 : 5 (b) 3 : 4(c) 4 : 5 (d) 4 : 3

128. Let BE and CF the two medians of a DABCand G be their intersection. Also let EF cut AGat O. Then AO: OG is(a) 1 : 1 (b) 1 : 2(c) 2 : 1 (d) 3 : 1

129. If S is the circumcentre of DABC and ÐA =50°, then the value of ÐBCS is(a) 20° (b) 40°(c) 60° (d) 80°

130. AC and BC are two equal chords of a circle.BA is produced to any point P and CP, whenjoined cuts the circle at T. Then(a) CT : TP = AB : CA(b) CT : TP = CA : AB(c) CT : CB = CA : CP(d) CT : CB = CP : CA

131. PQ is a direct common tangent of two circlesof radii r1 and r2 touching each other externallyat A. Then the value of PQ2 is(a) r1r2 (b) 2r1r2(c) 3r1r2 (d) 4r1r2

132. A

B C

O

BC is the chord of a circle with centre O. A is apoint on major arc BC as shown in the abovefigure. What is the value of ÐBAC + ÐOBC ?(a) 120° (b) 60°(c) 90° (d) 180°

133. Two circles with radii 5 cm and 8 cm toucheach other externally at a point A. If a straightline through the point A cuts the circles at pointsP and Q respectively, then AP : AQ is(a) 8 : 5 (b) 5 : 8(c) 3 : 4 (d) 4 : 5

134. If I is the In-centre of DABC and ÐA = 60°,then the value of ÐBIC is(a) 100° (b) 120°(c) 150° (d) 110°

135. The external bisectors of ÐB and ÐC of DABCmeet at point P. If ÐBAC = 80°, then ÐBPC is(a) 50° (b) 40°(c) 80° (d) 100°

136. When a pendulum of length 50 cm oscillates,it produces an arc of 16 cm. The angle so formedin degree measure is (approx)(a) 18°25¢ (b) 18°35¢(c) 18°20¢ (d) 18°08¢

137. If x, y are positive acute angles, x + y < 90° andsin (2x – 20°) = cos (2y + 20°), then the valueof sec (x + y) is

(a) 2 (b)12

(c) 1 (d) 0138. If 5 tanq = 4, then the value of

5sin 3cos5sin 3cosq qq q-æ ö

ç ÷è ø- is

(a)17 (b)

27

(c)57 (d)

25

139. The least value of (4sec2q + 9 cosec2q) is(a) 1 (b) 19(c) 25 (d) 7

140. If tan (x + y) tan (x – y) = 1, then the value of

tan 2x3æ öç ÷è ø

is

(a)13

(b)23

(c) 3 (d) 1141. If x = cosecq – sinq and y = secq – cosq, then

the value ofx2y2 (x2 + y2 + 3) is(a) 0 (b) 1(c) 2 (d) 3

142. If 0 ,2y cos x2p£ q £ q = sinq and 2x secq – y

cosecq = 3, then the value of x2 + 4y2 is(a) 1 (b) 2(c) 3 (d) 4

yoursmahboob.w

ordpress.com

142 SSC CGL Solved Paper

143. When the angle of elevation of the sumincreases from 30° to 60°, the shadow of a postis diminished by 5 metres. then the height ofthe post is

(a)5 3 m

2(b)

2 3 m5

(c)2 m

5 3(d)

4 m5 3

144. A rail road curve is to be laid out on a circle.What radius should be used if the track is tochange direction by 25° in a distance of40 metres?(a) 91.64 metres (b) 90.46 metres(c) 89.64 metres (d) 93.64 metres

145. If sinq + sin2q = 1, then the value of cos12q +3cos10q + 3cos8q + cos6q – 1 is(a) 0 (b) 1(c) –1 (d) 2

Directions (146-150): Population of five adjacentareas of a town, in the year of 2010, are representedin the following Pie-chart. the ratio of the numbersof males to that of females in these areas are statedin the table below. The total of the population in allthe five areas is 72 lakh. Study the Pie-chart andthe table and then answer the questions.

45°60°

105°135°

Area S1

Area S2

Area S3

Area S4

Area S5

Ratio of numbers of males (M) to females (F)

1 2 3 4 5Areas S S S S SRatio

3 : 2 4 :1 7 : 3 2 : 3 13 : 7M : F

146. 12 lakh is the population of the area(a) S1 (b) S3(c) S5 (d) S4

147. The number of males in the areas S1 and S4together is(a) 13.8 lakh (b) 8.2 lakh(c) 16.2 lakh (d) 15.8 lakh

148. The ratio of number of females in the area S2 tothat in the area S5 is(a) 7 : 9 (b) 36 : 13(c) 9 : 7 (d) 13 : 36

149. If, in the year 2010, there was an increase of5% population in the area S1 and 8% increasein population of the area S3 compared to theprevious year, then the ratio of population inthe areas S1 and S3, in the year 2009 was(a) 3 : 10 (b) 27 : 10(c) 27 : 70 (d) 10 : 3

150. The average of female population in all the fiveareas is lower than the female population ineach of the areas(a) S1 and S2 (b) S2 and S5(c) S2 and S4 (d) S4 and S5

PART-D : ENGLISH COMPREHENSION

Directions (151-155) : In the following questions,some parts of the sentences have errors and somehave none. Find out which part of a sentence has anerror. The number of that part is the answer. If thereis no error, then your answer is (d) i.e. No error.151. Instead of being (1)/ helpful he was (2)/ being

hindrance. (3) / No error (4)152. Where (1)/ have I (2)/ to deposit fees ? (3)/ No

error (4)153. By the time she had finished her work (1)/ I

had nearly given up (2)/ all hope of arriving atthe party in time. (3)/ No error (4)

154. Some categorically suspected (1) / having seenthe (2)/ guard and thief together. (3) No error (4)

155. He was (1)/ not in a position to state (2)/ thespeed the ship travelled. (3)/ No error (4)

Directions (156-160): In the following questions,sentences are given with blanks to be filled in withan appropriate word(s). Four alternatives aresuggested for each question. Choose the correctalternative out of the four as your answer.156. I _________ lunch one hour ago.

(a) have had (b) had had(c) have (d) had

157. He thought that my car _________ for sale.(a) is (b) was(c) has been (d) will be

158. He tried to prevent me_______doing my duty.(a) against (b) at(c) with (d) from

yoursmahboob.w

ordpress.com

143SSC CGL Solved Paper

159. To the dismay of the student body, the classpresident was ____________ berated by theprincipal at the school assembly,(a) ignominiously (b) privately(c) magnanimously (d) fortuitously

160. I would apologize if I ________ you.(a) am (b) was(c) have been (d) were

Directions (161-165): In the following questions,out of the four alternatives, choose the one whichbest expresses the meaning of the given word as youranswer.161. Gaol

(a) destination (b) garden(c) jail (d) bird

162. Loathing(a) warmth (b) affectation(c) hatred (d) affection

163. Pragmatic(a) intelligent (b) wise(c) religious (d) practical

164. Notion(a) thought (b) fact(c) truth (d) hypothesis

165. Vivacious(a) poisonous (b) energetic(c) tricky (d) slow

Directions (166-170): In the following questions,choose the word opposite in meaning to the givenword as your answer.166. Hasten

(a) Dash (b) Dawdle(c) Hurry (d) Scurry

167. Spirited(a) Animated (b) Excited(c) Lively (d) Dull

168. Forthright(a) Blunt (b) Tricky(c) Candid (d) Plainspoken

169. Antagonism(a) Cordiality (b) Animosity(c) Hostility (d) Enmity

170. Vanity(a) Humanity (b) Humility(c) Pretension (d) Arrogance

Directions (171-175): In the following questions,four alternatives are given for the idiom/phraseprinted in bold in the sentence. Choose thealternative which best expresses the meaning of theidiom/phrase.

171. The project is carried over to this year, and weneed to keep the ball rolling.(a) to continue the work(b) more information(c) to do better(d) new strategies

172. The host team bore the palm in the leaguematches.(a) played quite well(b) was victorious(c) was defeated(d) played a very boring match

173. Just keep your wig on. Everything will bealright.(a) Hold on to your wig, so it won’t fall off(b) Get another hair cut(c) Calm down(d) Take off your wig

174. Parents pay through their nose for theirchildren’s education.(a) by taking loans(b) an extremely high price(c) grudgingly(d) willingly

175. Monica’s habit of picking holes in everyrelationship is very irksome.(a) admiring people(b) finding fault(c) criticizing people(d) arguing with people

Directions (176-180): In the following questions, apart of the sentence is printed in bold. Below aregiven alternatives to the bold part at (a), (b), (c)which may improve the sentence. Choose the correctalternative. In case no improvement is needed youranswer is (d).176. His life is hanging with a thread.

(a) from (b) by(c) to (d) No improvement

177. After twenty years of exile, the prisoner was inthe end of his tether.(a) on (b) though(c) at (d) No improvement

178. Grandfather is often so tired that he drops —in his armchair.(a) slides away (b) falls out(c) slips in (d) No improvement

179. The court was forced to respect the profundityof the learned judge’s knowledge.(a) probity (b) proximity(c) prodigality (d) No improvement

yoursmahboob.w

ordpress.com

144 SSC CGL Solved Paper

180. He has lost his nearly all many pets.(a) all his nearly many pets(b) his many pets nearly all of them(c) nearly all his many pets(d) No improvement

Directions (181-185): In the following questions,out of the four alternatives choose the one whichcan be substituted for the given words/sentence.181. A group of three books, films etc. that have the

same subject or characters(a) trinity (b) trilogy(c) trio (d) tripod

182. A study of the human race(a) anthropology (b) archaeology(c) ethnology (d) etymology

183. An expert in an area of the fine or other arts(a) neophyte (b) amateur(c) connoisseur (d) enthusiast

184. The art of preserving skin of animals, birds,fishes(a) Topology (b) Taxonomy(c) Seismology (d) Taxidermy

185. Chanting of magic spells(a) Narration (b) Recitation(c) Incartation (d) Utterance

Directions (186-190): In the following questions,four words are given in each question, out of whichonly one word is correctly spelt. Find the correctlyspelt word.186. (a) abracadebra (b) abracadebri

(c) abracadabra (d) abrakadabra187. (a) apparrel (b) aparell

(c) aparel (d) apparel188. (a) refferee (b) referre

(c) referre (d) referee189. (a) commissioner (b) comissioner

(c) commissionar (d) comissionor190. (a) etiquete (b) etiquette

(c) ettiquette (d) ettiquete

Directions (191-200): In the following questions,you have two brief passages with 5 questions in eachpassage. Read the passages carefully and choosethe best answer to each question out of the fouralternatives.

PASSAGE-I(Question Nos. 191 to 195)

“Nobody knows my name” is the title of one of JamesBaldwin’s celebrated books. Who knows the name

of the old man sitting amidst ruins pondering overhis hubble-bubble? We do not. It does not matter.He is there like the North Pole, the Everest and theAlps but with one difference. The North Pole, theEverest and the Alps will be there when he is notthere any more. Can we really say this? “Dust thouact to dust returneth” was not spoken of the soul.We do not know whether the old man’s soul will gomarching on like John Brown’s. While his body liesmouldering in the grave or becomes ash driven bythe wind or is immersed in water, such speculationis hazardous. A soul’s trip can take one to thetreacherous shoals of metaphysics where there is no“yes” or “no”. “Who am I?” asked Tagore of therising sun in the first dawn of his life, he receivedno answer. “Who am I?” he asked the setting sun inthe last twilight of his life. He received no answer.

We are no more on solid ground with dust whichwe can feel in our hands, scatter to the wind and wetwith water to turn it into mud. For this much is sure,that in the end, when life’s ceaseless labour grindsto a halt and man meets death, the brother of sleep,his body buried or burnt, becomes dust. In the formof dust he lives, inanimate yet in contact with theanimate. He settles on files in endless governmentalmirahs, on manuscripts written and not publishedon all shelves, on faces and hands. He becomesubiquitous all pervasive, sometimes sneaking eveninto hermetically sealed chambers.191. What is the difference between the old man and

the North Pole, the Everest and the Alps?(a) he ponders over his hubble-bubble while

they don’t(b) they are known to all while he is known to

none(c) they remain while he will soon become dust(d) they are not as old as he

192. What, according to the passage, happens to aperson’s soul after death?(a) the soul also dies with the body(b) the soul continues to live after the body is

dead(c) the soul certainly becomes dust after death(d) it is dangerous to guess

193. Which of the following statement is true?(a) The rising sun told Tagore who he was(b) The rising sun did not tell Tagore who he was(c) The rising sun advised Tagore to ask no

questions(d) The rising sun told Tagore that he would

become dust

yoursmahboob.w

ordpress.com

145SSC CGL Solved Paper

194. What happens to man after he becomes dust?(a) he disappears from the world for ever(b) he appears in the form of man again(c) he becomes all pervasive as dust(d) he often sneaks into hermetically sealed

chambers195. What figure of speech is used in the expression

‘the brother of sleep’?(a) Simile (b) Metaphor(c) Oxymoron (d) Irony

PASSAGE-II(Question Nos. 196 to 200)

To write well you have to be able to write clearlyand logically, and you cannot do this unless you canthink clearly and logically. If you cannot do this yetyou should train yourself to do it by taking particularproblems and following them through, point bypoint, to a solution, without leaving anything outand without avoiding any difficulties that you meet.

At first you find clear, step-by- step thought verydifficult. You may find that your mind is not able toconcentrate. Several unconnected ideas may occurtogether. But practice will improve your ability toconcentrate on a single idea and think about it clearlyand logically. In order to increase your vocabularyand to improve your style, you should read widelyand use a good dictionary to help you find the exactmeanings and correct usages of words.

Always remember that regular and frequentpractice is necessary if you want to learn to write

well. It is no good waiting until you have aninspiration before you write. Even with the mostfamous writers, inspiration is rare. Someone saidthat writing is ninety-nine percent hard work andone percent inspiration, so the sooner you get intothe habit of disciplining your-self to write, the better.196. To write well, a person must train himself in

(a) dealing with a difficult problem(b) not leaving anything out(c) thinking clearly and logically(d) following a step-by-step approach

197. Initially it is difficult to write because(a) a good dictionary is not used(b) ideas occur without any sequence(c) aids to correct writing are not known(d) exact usages of words are not known

198. According to the passage, writing style can beimproved by(a) thinking logically(b) writing clearly(c) undergoing training(d) reading widely

199. Famous writers have achieved success by(a) using their linguistic resources properly(b) disciplining their skill(c) following only one idea(d) waiting for inspiration

200. All the following words mean ‘exact’ except(a) precise (b) accurate(c) very (d) erect

yoursmahboob.w

ordpress.com

146 SSC CGL Solved Paper

1. (d) The relationship between the numbers is :x : (x2 – 1)(9)2 – 1 = 81 – 1 = 80(100)2 – 1 = 10000 – 1 = 9999

2. (c) The relationship between the numbers is :x2 : x3

(5)2 = 25; (5)3 = 125(6)2 = 36; (6)3 = 216

3. (a) The relationship between the numbers is :x : x – 119335 – 119 = 216987 – 119 = 868

4. (b) B E G K A D F J

–1–1

–1

–1

Similarly,

–1

–1

–1

–1

P S V Y O R U X

5. (a) Pairs of consecutive opposite letters aregiven :AZ, BY; CX, DW; EV, FU; GT, HS

6. (d) 2 2 2

2 2 2

B D F H

S U W Y

+ + +

+ + +

¾¾® ¾¾® ¾¾®

¾¾® ¾¾® ¾¾®

1

2 2 2

1

2 2 2

B C

C E G I

S T

T V X Z

+

+ + +

+

+ + +

¾¾®

¾¾® ¾¾® ¾¾®

¾¾®

¾¾® ¾¾® ¾¾®7. (b) Food is necessary for man. Similarly, fuel

is necessary for fire.8. (c) Spiritual and Belief are inter – related

terms. Similarly, Orchestral and music arerelated.

9. (c) Latter and Former are antonymous to eachother. Similarly, Hazardous and Safe areantonymous to each other.

10. (d) 1 2

1 2

1 2

V W Y

Q R T

L M O

+ +

+ +

+ +

¾¾® ¾¾®

¾¾® ¾¾®

¾¾® ¾¾®But,

1 1J K L+ +¾¾® ¾¾®11. (a) Only BE has a vowel12. (d) Except the number 729, all others are

perfect squares.13. (b) Except the number 1725, all other numbers

are completely divisible by 4. The number1725 is completely divisible by 5.1000 2744250; 686 ;

4 44096 1024

41725But, 345

5

= =

=

=

14. (c) Except the number pair 30 – 50, all other

numbers pairs has ratio = 34

15. (d) Except LR, all others are pairs of oppositeletters.C X; D W;J QL O(Not R)

« «««

16. (a) Except Cyclotron, all others are languagesof computer. Cyclotron is an apparatus inwhich charged atomic and subatomicparticles are accelerated by an alternatingelectric field while following an outwardspiral or circular path in a magnetic field.

17. (b) Rooster, Gander and Peahen are birds.Buck is an animal.

18. (c) Except Voucher, all others are small bookcontaining information on a specificsubject. Voucher is a small printed pieceof paper which can be exchanged forcertain goods and services.

HINTS & SOLUTIONS

yoursmahboob.w

ordpress.com

147SSC CGL Solved Paper

19. (d) Arrangement of words as per dictionary:3. BALLERINA

¯2. BALLISTICS

¯4. BALLISTITE

¯1. BALLIUM

20. (c) Arrangement of words as per dictionary:2. Latch

¯4. Latitude

¯1. Laugh

¯3. Laurels

21. (b) Meaningful order of words :1. INFANT

¯3. CHILD

¯2. ADOLESCENT

¯5. ADULT

¯4. OLD

22. (c) c b a / b a c / a c b / c b a

23. (b) a b / a b b / ab b b / abb b b24. (d) 500 – (4)2 = 484

484 – (5)2 = 459

459 – (6)2 = 423423 – (7)2 = 374

25. (b) 2

2

MNO P R STU

WXY Z B CD E

+

+

¾¾®

¾¾®26. (a) 60 + (3)2 = 69

69 + (4)2 = 8585 + (5)2 = 110110 + (6)2 = 146

27. (b)

46 50 47 55 49 61 52

+1 +2 +3

+5 +6

28. (d) Age of Manager = New Average Age +(No. of Subordinates × Change in Average)= 31 + (25 × 1) = 56 years

29. (b) Suppose, in the beginning the number ofstudents in Class B = xTherefore, the number of Students inClass A = 2xNow,2x + 20 + x + 30 = 140Þ 3x = 140 – 50

90x 303

\ = =

Number of Students in Class A= 2x = 2 × 30 = 60

30. (c) Take LCM of 8, 12, 15 and 202 8, 12, 15, 202 4, 6, 15, 103 2, 3, 15, 55 2, 1, 5, 5

2, 1, 1, 1\ LCM = 2 × 2 × 3 × 5 × 2 = 120Since the remainder to be left is 2, thenumber can be given by 120K + 2, wherek is a positive integer120 × 1 + 2 = 122(K = 1)

31. (a) 1

1

B A

P O

K J

-

-

-

¾¾®

¾¾®

¾¾®AB BA�

OP PO�

KL LK\ �

32. (d) There is no ‘P’ letter in the given word.33. (d) S

E

A O N

LS S

SE A

AO

L

A NTherefore,S

E

A

S

E

A

P R T

P RA

A

TE

E

34. (a)R

G

A

HV NR

G

A A

A A

A

NH V

Similarly,M AT H

V NMAT H

V

A

A

A

NA

yoursmahboob.w

ordpress.com

148 SSC CGL Solved Paper

36. (a) EQU AT I ON G SWCVKQ P+2+2+2+2+2+2+2+2

Similarly,

+2+2+2+2+2+2

DON KE Y F Q PMGA

37. (c) (3 + 8) × (1 + 5)Þ 11 × 6 = 66(2 + 9) × (3 + 6)Þ 11 × 9 = 99Similarly,(8 + 2) × (4 + 4)

Þ 10 × 8 = 80

38. (d) + Þ ¸

×

– Þ

ÞÞ + ¸

×

–45 + 9 – 3 × 15 ¸ 2Þ ? = 45 ¸ 9 × 3 + 15 – 2Þ ? = 5 × 3 + 15 – 2

Þ ? = 30 – 2 = 2839. (c) First Column

7 × 4 – 3 = 28 – 3 = 25Second Column8 × 9 – 2 = 72 – 2 = 70Third column6 × 5 – ? = 29

Þ ? = 30 – 29 = 1 .40. First Column

3 + 6 × 9 = 3 + 54 = 57Second Column4 + 7 × 1 = 4 + 7 = 11Third Column

5 + 8 × 2 Þ 5 + 16 = 2141. (d) 78 + 14 = 92

92 + 21 = 113113 + 28 = 141141 + 35 = 176

42. (c) 5 km2 km45°

Starting Point

5 km

North

West East

South43. (b) Starting

Point

44. (c) Neither Conclusion I nor Conclusion IIfollows. The person always listens to 9O’clock news on radio to keep himselfupdated. This does not imply that he doesnot read newspaper. The use of term ‘only’in the Conclusion II makes it unsuitable.

45. (c) Clearly both the conclusions follow. If allwealthy people are happy in life, then itcan be said that wealth and happiness aredirectly related.

46. (c) 47. (c) 48. (d) 49. (b)50. (b) S Þ 55, 67, 79, 88, 96

A Þ 01, 13, 20, 34, 42L Þ 56, 69, 77, 85, 98M Þ 58, 66, 75, 89, 97Option S A L M

(a) 55 20 56 59(b) 79 13 69 75(c) 96 34 76 89(d) 67 21 85 97

51. (d) 52. (b) 53. (c) 54. (b) 55. (a)56. (a) 57. (b) 58. (a) 59. (c) 60. (a)61. (d) 62. (d) 63. (c) 64. (d) 65. (c)66. (a) 67. (b) 68. (b) 69. (a) 70. (b)71. (c) 72. (d) 73. (c) 74. (d) 75. (b)76. (a) 77. (c) 78. (d) 79. (a) 80. (b)81. (b) 82. (d) 83. (a) 84. (a) 85. (b)86. (d) 87. (b) 88. (d) 89. (d) 90. (a)91. (c) 92. (c) 93. (d) 94. (d) 95. (a)96. (a) 97. (b) 98. (d) 99. (d) 100. (c)101. (c) The pattern is :

1 + 4 = 55 + 7 (= 4 + 3) = 1212 + 12 (= 7 + 5) = 2424 + 19 (= 12 + 7) = 4343 + 28 (= 19 + 9) = 71

yoursmahboob.w

ordpress.com

149SSC CGL Solved Paper

102. (b) LCM of 4, 5, 6 and 7 = 420\ Required number= 420k + 3 which is exactly divisible by 13.= 32 × 13k + 4k + 3Hence, 4k + 3 should be divisible by 13for some value of k.For k = 9, 4k + 3 = 39 which is divisible by 13.\ Required number= 420 × 9 + 3 = 3783

103. (b)1

2 3+

1 3 2 3 23 23 2 3 2

3 21 4 3;

4 31 5 4;

4 51 6 5

5 6

- -= ´ =-+ -

= -

\ = -+

= -+

= -+

\ Expression

( )3 2 4 3 5 4 6 5

6 2 2 3 1

= - + - + - + -

= - = -104. (c) A’s profit : B’s profit

1 2: 1: 23 3

= =

A 'sequivalent capital 1B's equivalent capital 2

\ =

x 15 1 15 143x 2 3n 2n4

´Þ = Þ =

´

Þ n = 10 months105. (a) If the number be x, then

x + 21 = 3x – 7Þ 3x – x = 21 + 7Þ 2x = 28Þ x = 14

106. (c) If time taken by A be x days,then time taken by B = 3x days\ 3x – x = 60Þ 2x – 60

Þ x = 30Time taken by B = 90days\ (A + B)’s 1 day’s work

1 1 3 1 4 230 90 90 90 45

+= + = = =

\The work will be completed in

45 1i.e. 22 days2 2

107. (d) F

GH

EA B

CD

Area of the shaded region= (100 + 2 × 10) (80 + 2 × 10) – 100 × 80= 120 × 100 – 8000= 4000 sq. metre

108. (c) If the C.P.of machine by `x, then110 27500 90x100 100

´´ =

11x 275 9010

Þ = ´

275 900x11´Þ = =

`22500

109. (b) Required ratio = 8 × 7 : 5 × 9 = 56 : 45110. (b) Required average

40 5 35 255

´ - +=

190 38 years5

= =

111. (c) If the total length of journey be x km, then

x x 1535 40 60- =

8x 7x 1280 4-Þ =

x 1280 4

Þ =

1x 280 70 km4

Þ = ´ =

yoursmahboob.w

ordpress.com

150 SSC CGL Solved Paper

112. (b) C.P. of 20 articles

100 5Re.120 6

= =

\ Number of articles bought for Re.1.

6 20 245= ´ =

113. (a) Required increase

25 1225 12 %100´æ ö= + +ç ÷è ø

= 40%114. (a) Speed of ship = 30kmph

30 5 25m / sec. m / sec.18 3´= =

Time taken in covering 500 metre

500 3 60 seconds25´= =

Speed of radio waves

200 1km / sec. km / sec.1000 5

= =

x = depth of ocean

500 metre

A

BC

2 1x4

+

2 1x x4 601 15 5

+\ + =

2 1 1x x 60 124 5

Þ + + = ´ =

( )2 2 112 x x4

\ - = +

2 2 1144 x 24x x4

Þ + - = +

1 57524x 1444 4

Þ = - =

575x 6 km4 24

Þ = =´

115. (a) If the amount of loan taken from the firstbank be `x, then

( )x 8 1 10000 x 10 950100 100´ ´ - ´+ =

Þ 8x + 100000 – 10x = 95000Þ 2x = 100000 – 95000 = 5000Þ x = `2500

116. (d)2

21a 98

a+ =

21a 2 98a

æ öÞ + - =ç ÷è ø21a 100

aæ öÞ + =ç ÷è ø

1a 10a

Þ + =

On cubing both sides,31a 1000

aæ ö+ =ç ÷è ø

331 1a 3 a 1000

aaæ öÞ + + + =ç ÷è ø

33

1a 1000 30 970a

Þ + = - =

117. (a) x – 1 = 2 3+On squaring,x2 – 2x + 1 = 2 + 3 + 2 6

2x 2x 4 2 6Þ - - =On squaring again,x4 + 4x2 + 16 – 4x3 – 8x2 + 16x = 24Þ x4 – 4x3 – 4x2 + 16x – 8 = 0Þ 2x4 – 8x3 – 8x2 + 32x – 16 = 0Þ 2x4 – 8x3 – 5x2 + 26x – 28 – 3x2

+ 6x + 12 = 0Þ 2x4 – 8x3 – 5x2 + 26x – 28= 3x2 – 6x – 12= 3 (x2 – 2x – 4)

3 2 6 6 6= ´ =

118. (c) ( ) ( )2 2x 0 0 5 13- + + =Þ x2 + 25 = 169Þ x2 = 169 – 25 = 144\ x = 144 12=

yoursmahboob.w

ordpress.com

151SSC CGL Solved Paper

119. (b) 4x = 18y

x 18 9y 4 2

Þ = =

x 9 71 14 2 2æ ö\ - = - =ç ÷è ø

120. (d)1x 5x

+ =

Þ x2 – 5x + 1 = 0Þ x2 – 3x + 1 = 2x

4 42 2

2

1 1x x1x x2 xx 3x 1

æ ö+ +ç ÷ç ÷\ =è ø- +

33

1 1x2 xæ ö= +ç ÷è ø

31 1 1x 3 x2 x x

é ùæ ö æ ö= ê + - + úç ÷ ç ÷è ø è øë û

( )1 125 3 52= - ´

1 110 552= ´ =

121. (d) x 2 3, y 2 3= + = -x + y = 4; xy = 4 – 3 = 1

( )( ) ( )

22 2

3 3 3x y 2xyx y

x y x y 3xy x y

+ -+\ =+ + - +

16 2 14 764 3 4 52 26-= = =- ´

122. (b) a2 + b2 + c2 = 2(a – b –c) –3Þ a2 + b2 + c2 = 2a + 2b + 2c + 3 = 0Þ a2 – 2a + 1 + b2 + 2b + 1 + c2 + 2c + 1 = 0Þ (a –1)2 + (b + 1)2 + (c + 1)2 = 0[If x2 + y2 + z2 = 0 Þ x = 0; y = 0; z =0]\ a – 1 = 0 Þ a = 1b + 1 = 0 Þ b = – 1c + 1 = 0 Þ c = – 1\ 2a – 3b + 4c = 2 + 3 – 4 = 1

123. (a)12x 6

2x- =

1x 34x

Þ - = [on dividing by 2]

42

1 1x 2 x 94x16x

Þ + - ´ ´ =

[On Squaring]

42

1 1 19x 92 216x

Þ + = + =

124. (b)15a 53a

+ =

On multiplying by 35 ,

1 33a 5 35a 5

+ = ´ =

On squaring,

22

1 19a 2 3a 95a25a

+ + ´ ´ =

22

19a25a

Þ +

6 45 6 3995 5 5

-= - = =

125. (a) 5x + 7y = 35 ...(i)4x + 3y = 12 ...(ii)By equation (i) × 4 – (ii) × 520x 28y 14020x 15y 60

13y 80

+ =+ =

- - -=

80y13

Þ = = Height of triangle

Point of intersection on x-axis of equation5x + 7y = 35Þ 5x + 7 × 0 = 35Þ 5x = 35Þ x = 7\ (7, 0)Similarly, point of intersection of4x + 3y = 12 = (3, 0)\ Base = 7 – 3 = 4

\ Area =1 8042 13´ ´ 160 sq. unit

13=

126. (b) ÐBAC = 180° – ÐBOC = 180° – 54° = 126°127. (b) Ratio of corresponding sides

9 316 4

= =

yoursmahboob.w

ordpress.com

152 SSC CGL Solved Paper

128. (c) AF = FBAE = EC

\ FE PBC1 BC2

=

\ AO : OG = 2 : 1

129. (b) A

B C

.S

ÐBAC = 50°\ ÐBSC = 100°BS = SC = radius

\ ÐBCS ( )1 180 100 402= - = °

130. (c) It is based on fundamental concept.131. (d) PQ2 = (r1 + r2)

2 – (r1 – r2)2 =4r1r2

132. (c) A

B C

O.

ÐBOC = 2ÐBACOB = OC\ ÐOBC = ÐOCB

BOCOBC 902

Ð\Ð = °-

= 90° – ÐBAC\ ÐBAC + ÐOBC= 90° – ÐBAC + ÐBAC= 90°

133. (b) P

A

Q

\ AP : AQ = 5 : 8

134. (b) ÐBIC = 90° +A2

= 90° + 30° = 120°

135. (a) ÐBPC = 90° –A2

= 90° – 40° = 50°136. (c) s = 16 cm

r = 50 cm

s 16 8 radianr 50 25

\q = = =

8 18025= ´

p

8 180 1008725 22 55= ´ ´ =

181855

°=

1818 60 18 2055æ ö= ° ´ » °ç ÷è ø

137. (a) sin (2x – 20°) = cos (2y + 20°)Þ sin (2x – 20°)= sin (90° – 2y – 20°)= sin (70° – 2y)Þ 2x – 20° = 70° – 2yÞ 2 (x + y) = 90°Þ x + y = 45°\ sec (x + y) =sec 45° = 2

138. (a) 5 tanq = 4

Þ tanq 45

=

5 sin 3cos5 sin 3cos

q- q\q+ q

5 sin 3coscos

5 sin 3coscos

q- qq=

q+ qq

45 35 tin 3 545 tan 3 5 35

´ -q-= =+ ´ +

4 3 14 3 7-= =+

yoursmahboob.w

ordpress.com

153SSC CGL Solved Paper

139. (c) 4 sec2q + 9coses2q= 4 (1 + tan2q) + 9 (1 + cot2q)= 4 + 4 tan2q + 9 + 9cot2q= 4 tan2q + 9cot2q + = 12 + 12 + 13= (2tan2q – 3Cot2q)2 + 25(2 + tan2q – 3Cot2q) = 0the minimum value is 25.

140. (a) tan (x + y). tan (x – y) = 1Þ tan (x + y) = cot (x – y)= tan (90°– x + y)Þ x + y = 90° – x + yÞ 2x = 90°

\ tan 2x3 = tan 30° =

13

141. (b) x2y2 (x2 + y2 + 3)= (cosecq – sinq)2 (secq – cosq)2

{(cosecq – sinq)2 + (secq – cosq)2 + 3}2 21 1sin cos

sin cosæ ö æ ö= - q - qç ÷ ç ÷q qè ø è ø

2 21 1sin cos 3sin cos

ì üï ïæ ö æ ö- q - q +í ýç ÷ ç ÷q qè ø è øï ïî þ2 22 21 sin 1 cos

sin cosæ ö æ ö- q - q= +ç ÷ ç ÷

q qè ø è ø2 22 21 sin 1 cos 3

sin cos

ì üæ ö æ ö- q - qï ï+ +ç ÷ ç ÷í ýq qè ø è øï ïî þ2 22 2cos sin

sin cosæ ö æ öq q= ç ÷ ç ÷

q qè ø è ø2 22 2cos sin 3

sin cos

ì üæ ö æ öq qï ï= + +ç ÷ ç ÷í ýq qè ø è øï ïî þ= cos2q × sin2q

6 6 2 2

2 2cos sin 3cos .sin

cos .sin

æ öq + q+ q qç ÷

q qè ø= cos6q + sin6q + 3cos2q sin2q

( ) ( )3 32 2cos sinì ü= q + qí ýî þ

+ 3cos2q.sin2q

= (cos2q + sin2q)3 – 3cos2q.sin2q(cos2q + sin2q) + 3cos2q.sin2q= 1 – 3cos2q.sin2q + 3cos2q.sin2q= 1

142. (d) 2y cosq = xsinqÞ x secq = 2y cosecq\ 2x secq – y cosecq = 3Þ 4y cosecq – y cosecq = 3Þ 3y cosecq = 3Þ y cosecq = 1Þ y = sinq\ x secq = 2y cosecq= 2sinq.cosecq = 2Þ x = 2cosq\ x2 + 4y2 = 4cos2q + 4sin2q = 4

143. (a) A

B CD

h

60° 30°5 metrex

AB = Pole = h metreBD = x metreFrom DABC,

htan 30x 5

° =+

1 hx 53

Þ =+

x 5 3hÞ + = ...(i)From DABD,

htan 60x

° =

h3x

Þ =

hx3

Þ =

x 5 3h\ + =

h 5 3h3

Þ + =

h 5 3 3hÞ + =

2h 5 3Þ +

5 3h metre2

Þ =

yoursmahboob.w

ordpress.com

154 SSC CGL Solved Paper

144. (a) q = 25° 25 radians180´p=

5 radians36p=

sr

q =

s 40 40 36r5 536

´Þ = = =pq p

40 36 7 metre5 22´ ´=´

= 91.64 metre145. (a) sinq + sin2q = 1

Þ sinq = 1 – Sin2qÞ sinq = cos2q\ cos12q + 3cos10q + 3cos8q + cos6q – 1= (cos4q + cos2q)3 – 1= (sin2q + cos2q)3 – 1 = 1 – 1 = 0

146. (c) 72 lakhs 360º °Q

\ 12 lakhs 360 1272

º ´

60° = S5147. (a) Population of region S1

45 72 9 lakhs360= ´ =

Males 3 9 5.4 lakhs5= ´ =

Population of region S4

105 72 21 laksh360= ´ =

Males 2 21 8.4 lakhs5= ´ =

Sum = 5.4 + 8.4 = 13.8 lakhs148. (c) Population of region S2

135 72 27 lakhs360= ´ =

Females 1 27 5.4 lakhs5= ´ =

Population of region S5

60 72 12 lakhs360= ´ =

Famales 7 12 4.2 lakhs20= ´ =

Required ration = 5.4 : 4.2 = 9.7149. (*) Population in 2009

1100Region S 9105

= ´

60 lakhs7

=

3100 15Region S 72108 360= ´ ´

25 lakhs9

=

\ Required ratio 60 25:7 9

=

12 5: 108 : 357 9

= =

150. (c) Number of females :Region S1 Þ 9 – 5.4 = 3.6 lakhsRegion S2 Þ 5.4 lakhs

33Region S 3 0.9 lakhs

10Þ ´ =

Region S4 Þ 21 – 8.4 = 12.6 lakhs

5Region S 4.2 lakhsÞAverage = 5.34 lakhs

151. (c) Here, a hindrance …………… should beused.Look at the sentences :To be honest, she was more of a hindrancethan a help.The high price is a major hindrance topotent buyers.

152. (b) Here, I have ……… should be used.153. (a) Here, By the time she finished her work

(Simple Past) ……… should be used.154. (a) Here, Some were surprised/ Some

categorically denied ……… should beused.

155. (c) Here, the speed the ship sailed/travelledat ……… should be used.

156. (d) 157. (b) 158. (d) 159. (a) 160. (d)161. (c) The word Gaol (Noun) means : jail.162. (c) The word Loathing (Noun)

means : a strong feeling of hatred.

yoursmahboob.w

ordpress.com

155SSC CGL Solved Paper

163. (d) The word Pragmatic (Adjective)means : practical, busy; realistic; solvingproblems in a practical and sensible wayrather than by having fixed ideas ortheories.

164. (a) The word Notion (Noun) means : belief;desire; intention; thought.

165. (b) The word Vivacious (Adjective) means :having a lively, attractive personality;energetic.

166. (b) The word Hasten (Verb) means : hurry;to say or do something without delay. Theword Dawdle (Verb) means : to take along time to do something or goSomewhere.

167. (d) The word Spirited (Adjective) means :lively; gay; vigorous; enthusiastic.The word Dull (Adjective) means :sluggish; depressed; tedious.

168. (b) The word Forthright (Adjective) means: frank; direct and honest in manner andspeech; candid.The word Tricky (Adjective) means :Clever but likely to trick you, deceive you,deceitful.

169. (a) The word Antagonism (Noun) means :hostility; feelings of hatred and opposition;animosity; enmity.The word Cordiality (Noun) means : in apleasant and friendly manner.Look at the sentence :The antagonism he felt towards his oldenemy was still very strong.You are cordially invited to the celebration.

170. (b) The word Vanity (Noun) means : toomuch pride in your own appearance,abilities or achievements; arrogance. Theword Humility (Noun) means : the qualityof being humble.

171. (a) Idiom get/set/start/keep the ball rollingmeans : to make something start happening;to make sure that something continues tohappen.

172. (b) 173. (c)174. (b) Idiom pay through the nose means : to

pay too much money for something.175. (b) Idiom pick holes in something means :

to find the weak points in something suchas a plan, suggestion etc.

176. (b) Hang by a thread/hair means : to be ingreat danger.

177. (c) Be at the end of your tether means : tofeel that you cannot deal with a difficultsituation any more because you are tootired, worried etc.

178. (d) 179. (d) 180. (c) 181. (b)182. (a) 183. (c) 184. (d)185. (c) The word Incantation means ; special

words that are spoken or sung to have amagic effect.

186. (c) 187. (d) 188. (d) 189. (a)190. (b) 191. (c) 192. (d) 193. (b)194. (c) 195. (b) 196. (c) 197. (b)198. (d) 199. (b) 200. (d)

yoursmahboob.w

ordpress.com

156 SSC CGL SolvedPaperc

PART-A : GENERAL INTELLIGENCE & REASONING

Directions (Qs. 1 - 9) : Select the related letter / word/ number from the given alternatives.1. Length : Metre :: Power : ?

(a) Calories (b) Degree(c) Watt (d) Kilogram

2. Square : Cube :: Circle : ?(a) Ellipse (b) Parabola(c) Cone (d) Sphere

3. Paper : Tree :: Glass : ?(a) Window (b) Sand(c) Stone (d) Mirror

4. ACFJ : ZXUQ :: EGIN : ?(a) VUSQ (b) VRPM(c) UTRP (d) VTRM

5. ACEG : DFHJ :: QSUW : ?(a) TVXZ (b) TQST(c) MNPR (d) EGIJ

6. EGIK : FILO :: FHJL : ?(a) JGMP (b) JGPM(c) GJPM (d) GJMP

7. 10 : 91 :: 9 : ?(a) 69 (b) 72(c) 89 (d) 97

8. 7 : 56 :: 9 : ?(a) 63 (b) 81(c) 90 (d) 99

9. 20 : 50 :: 100 : ?(a) 150 (b) 250(c) 200 (d) 156

Directions (Qs. 10 - 18) : Find the odd number /letters / number pair from the given alternatives.10. (a) Pathology (b) Geology

(c) Cardiology (d) Radiology11. (a) Rivulet (b) Stream

(c) River (d) Pond12. (a) Konark (b) Madurai

(c) Dilwara (d) Ellora13. (a) RTW (b) QOM

(c) IKG (d) IKM14. (a) EFH (b) OPQ

(c) BCE (d) IJL

15. (a) DH (b) FJ(c) HK (d) PR

16. (a) 24 (b) 49(c) 80 (d) 15

17. (a) 121 (b) 324(c) 523 (d) 729

18. (a) 704, 11 (b) 256, 4(c) 832, 13 (d) 310, 5

19. Arrange the following words in their ascendingorder, as in a dictionary:1. Pick 2. Pith3. Pile 4. Perk5. Pour(a) 4, 1, 2, 3, 5 (b) 4, 1, 3, 2, 5(c) 4, 3, 2, 1, 5 (d) 5, 4, 3, 2, 1

20. Arrange the following words in their ascendingorder:1. Millenium2. Diamond Jubilee3. Silver Jubilee4. Centenary5. Golden Jubilee(a) 2, 3, 5, 4, 1 (b) 2, 5, 3, 1, 4(c) 3, 5, 2, 4, 1 (d) 2, 3, 5, 1, 4

21. Arrange the following words in their descendingorder:1. Weekly 2. Bi-annual3. Fortnightly 4. Monthly5. Annual(a) 1, 3, 4, 2, 5 (b) 2, 5, 4, 1, 3(c) 4, 1, 2, 3, 5 (d) 5, 2, 4, 3, 1

22. Which one set of letters when sequentiallyplaced at the gaps in the given letter series shallcomplete it?a _ cbc _ ca _ ab _ bca _ ab(a) b a b c c (b) b c a b b(c) a b c b c (d) b c a b c

Directions (Qs. 23 -27) : In the following questions aseries is given, with one term missing. Choose thecorrect alternative from the given ones that willcomplete the series.23. BCFG, JKNO, RSVW, ?

(a) ZADE (b) HIKL(c) STUX (d) MNPQ

SSC Combined Graduate Level (CGL) Solved PaperEVENING SHIFT 8 JULY 2012

yoursmahboob.w

ordpress.com

157SSC CGL SolvedPaper

24. CIM, HNR, MSW, ?(a) SXA (b) UYB(c) RXB (d) ZEH

25. 2, 3, 6, 7, 14, 15, ?(a) 16 (b) 30(c) 31 (d) 32

26. 3120, ? , 122, 23, 4(a) 488 (b) 621(c) 610 (d) 732

27. 0, 5, 60, 615,?(a) 6030 (b) 6170(c) 6130 (d) 6000

28. Nithya is Sam’s Sister. Mogan is Sam’s Father.Selvan is Rajan’s Son. Rajan is Mogan’s Brother.How is Nithya related to Selvan?(a) Daughter (b) Sister(c) Cousin (d) Wife

29. A boy’s age is one fourth of his father’s age.The sum of the boy’s age and his father’s age is35. What will be father’s age after 8 years?(a) 15 (b) 28(c) 35 (d) 36

30. A man said to a lady “Your mother’s husband’ssister is my aunt”. How is the lady related toman?(a) Sister (b) Mother(c) Daughter (d) Granddaughter

31. If South East becomes North, then what willSouth West become?(a) North (b) West(c) East (d) North West

Direction (Q. 32) : From the given alternatives selectthe word which cannot be formed using the letters ofthe given word.32. CONTENTION

(a) TONIC (b) NOTE(c) NATION (d) NOTION

33. In a certain code language, GRAPE is written as27354 and FOUR is written as 1687. How isGROUP written in that code?(a) 27384 (b) 27684(c) 27685 (d) 27658

34. WAYIN is written as TXVFX. How LBUK canbe written in that code?(a) IYRH (b) KATJ(c) JZSI (d) NDWM

35. In a certain code language, if the word PARTNERis coded as OZQSMDQ, then what is the codefor the word SEGMENT?

(a) TFHNFOU (b) RDFLDMS(c) RDELDMS (d) RDFEDNS

36. If DOCTOR is written as FQEVQT; howPATIENT can be written in that code?(a) RVKGPV (b) RCKPGVV(c) RCVKGPV (d) RVCKGVP

Directions (Qs. 37 & 38) : Find the missing number.

37. 21 24 3611 14 123 ? 4

77 112 108

(a) 2 (b) 4(c) 3 (d) 5

38. 12 16 1816 16 205 7 ?

197 263 356

(a) 9 (b) – 4(c) 4 (d) – 8

39. If + means ¸ , – means ×, × means +, ¸ means –, then90 + 18 – 6 × 30 ¸ 4 = ?(a) 64 (b) 65(c) 56 (d) 48

40. If 73 + 46 = 42 and 95 + 87 = 57, then 62 + 80 = ?(a) 32 (b) 48(c) 64 (d) 36

41. Based on the given data, estimate the number of‘Television-buyers’ for the year 1990.1982 1984 1986 1988 1990 447 458 489 540 ?(a) 611 (b) 591(c) 571 (d) 601

42. A man coming out of the backdoor of his housewhich is facing East, walked for one kilometre,turned to his right and walked for anotherkilometre. Then he turned to his right and walkeda kilometre again. Where was he from his houseat the end?(a) 1 km away in north(b) 1 km away in south(c) 1 km away in east(d) 1 km away in west

yoursmahboob.w

ordpress.com

158 SSC CGL SolvedPaperc

43. Two squads of soldiers A and B, facing East andWest respectively received the followingcommands - Left Turn, About Turn, Right Turn,Left Turn. Which directions would the squadsA and B face at the end?(a) East, West (b) West, East(c) North, South (d) South, North

Directions (Qs. 44 & 45) : In the following questionstwo statements are given, followed by two conclusionsI and II. You have to consider the statements to betrue even if they seem to be at variance from commonlyknown facts. You have to decide which of the givenconclusions, if any, follow from the given statements.

44. Statements :(1) Due to contamination of water, large

number of people were admitted to hospital.(2) The symptoms were of Typhoid.Conclusions :I. Contamination of water may lead to

Typhoid.II. Typhoid is a contagious disease.(a) Only conclusion I is true(b) Only conclusion II is true(c) Both conclusions I and II are true(d) Both conclusions I and II are false

45. Statements :(1) 60% of the government employees went

on strike.(2) Mr. Gopal is a government employee.Conclusions :I. Mr. Gopal went on strike.II. Mr. Gopal did not participate in the strike.(a) Only conclusion I follows(b) Only conclusion II follows(c) Both conclusions I and II follow(d) Either conclusion I and II follows

Direction (Q. 46) : In the following question whichanswer figure will complete the question figure?46. Question figure

Answer Figure

(a) (b) (c) (d)

47. From the given answer figures, select the one inwhich the question figure is hidden / embedded.Question figure

Answer figures

(a) (b) (c) (d)

Direction (Q.48) : In the following question a pieceof paper is folded and punched as shown below inthe question figure. From the given answer figures,indicate how it will appear when opened?48. Question figure

Answer figures

(a) (b) (c) (d)

Direction (Q. 49) : In the following question if a mirroris placed on the line MN, then which of the answerfigures is the right image of the given figure?49. Question figure

M

NAnswer figures

(a) (b) (c) (d)

yoursmahboob.w

ordpress.com

159SSC CGL SolvedPaper

Direction (Q. 50) : In the following question a wordis represented by only one set of numbers as given inany one of the alternatives. The sets of numbers givenin the alternatives are represented by two classes ofalphabets as in two matrices given below. Thecolumns and rows of Matrix I are numbered from 0 to4 and that of Matrix II are numbered from 5 to 9. Aletter from these matrices can be represented first byits row and next by its column, e.g., ‘R’ can berepresented by 13, 22, etc. and ‘P’ can be representedby 67, 76 etc. Similarly, you have to identify the setfor the word SHRI.50. Matrix I Matrix II

0 1 2 3 40 R H E L I1 I E L R H2 H L R I E3 E R I H L4 L I H E R

5 6 7 8 95 B S N A D6 D N B S A7 A B D N S8 S D A B N9 N A S D B

(a) 58, 02, 13, 01 (b) 85, 42, 31, 14(c) 68, 20, 13, 32 (d) 85, 02, 44, 30

PART-B : GENERAL AWARENESS

51. Which one of the following is an example for anon-economic good?(a) Doctor’s service (b) Teacher’s service(c) Mother’s service (d) Banker’s service

52. Which one of the following committee isassociated with banking sector reforms in India?(a) L. C. Gupta (b) Narsimhan(c) Chakravarty (d) Kelkar

53. Which one of the following is not a qualitativecredit control measure of the RBI?(a) Fixing margin requirements(b) Variable interest rates(c) Open market operations(d) Credit rationing

54. The 13th Five Year Plan will be operative for theperiod.(a) 2010 - 2015 (b) 2011- 2016(c) 2012 - 2017 (d) 2013 - 2018

55. The national income of a nation is the(a) Government’s annual revenue(b) Sum total of factor incomes(c) Surplus of public sector enterprises(d) Exports minus imports

56. The Constitutional Amendment Act that hasintroduced safeguards against the misuse ofproclamation of national emergency is the(a) 42nd Amendment Act(b) 43rd Amendment Act

(c) 44th Amendment Act(d) 45th Amendment Act

57. The Fundamental Rights can be suspended by the(a) Governor (b) President(c) Law Minister (d) Prime Minister

58. The main reason for the growth of communalismin India is(a) Educational and economic backwardness of

minority groups(b) Political consciousness(c) Social inequalities(d) Imposing ban on communal organisations

59. A Retired Judge of a High Court is not permittedto practice as a lawyer in(a) Supreme Court(b) Any Court in India(c) High Courts(d) Except the High Court where he retired

60. Which one of the following does not match?(a) Hindu Marriage Act : 1955(b) Medical Termination of Pregnancy Act : 1971(c) Domestic Violence on women Act : 1990(d) Cruelty against Women : 1995

61. Who among the following was the First Viceroyof India?(a) Lord Ripon (b) Lord Curzon(c) Lord Mountbatten (d) Lord Canning

62. Prithvi Raj Chauhan was defeated in the SecondBattle of Tarain by(a) Mahmud Ghazni (b) Muhammad Ghori(c) Qutbuddin Aibak (d) Yalduz

63. The original name of Nana Phadnavis was(a) Mahadaji Sindhia(b) Tukoji Holkar(c) Narayan Rao(d) Balaji Janardan Bhanu

64. Who among the following first propounded theidea of Basic Education?(a) Jawahar Lal Nehru(b) Raja Ram Mohan Roy(c) Mahatma Gandhi(d) Dayanand Saraswati

65. Arrange the following in chronological order:I. Dandi MarchII. Simon CommissionIII. Poona PactIV. Gandhi Irwin Pact(a) II, I, III, IV (b) II, I, IV, III(c) IV, III, I, II (d) IV, III, II, I

yoursmahboob.w

ordpress.com

160 SSC CGL SolvedPaperc

66. Which one of the following is not correctlymatched?(a) Darjeeling - West Bengal(b) Mount Abu - Rajasthan(c) Kodaikanal - Tamil Nadu(d) Simla - Uttar Pradesh

67. The earth is at its maximum distance from the Sunon(a) January 30th (b) December 22nd(c) September 22nd (d) July 4th

68. Consider the following pairs:Tributary Main River

1. Chambal : Yamuna2. Sone : Narmada3. Manas : BrahmputraWhich one of the pairs given above is/arecorrectly matched?(a) 1, 2 and 3 (b) 1 and 2(c) 2 and 3 (d) 3 only

69. A form of condensation that reduces visibilityand causes breathing problems is(a) Dew (b) Frost(c) Smog (d) Mist

70. The total population divided by available arableland area is referred to as(a) Population density(b) Nutritional density(c) Agricultural density(d) Industrial density

71. Green glands are associated with(a) Reproduction (b) Excretion(c) Respiration (d) Digestion

72. During respiration, the gases enter into the bloodand leave the same by the process of(a) Active transport(b) Diffusion(c) Diffusion and active transport(d) Osmosis

73. Heart is devoid of(a) Cardiac muscle (b) Involuntary muscle(c) Voluntary muscle (d) Smooth muscle

74. The soil salinity is measured by(a) Conductivity meter (b) Hygrometer(c) Psychrometer (d) Auxanometer

75. Which of the following is a fungal disease?(a) Leucoderma (b) Eczema(c) Ringworm (d) Elephantiasis

76. Chickenpox is caused by(a) DNA virus (b) Variola virus(c) Streptococcus (d) Vibrio cholerae

77. Instruments can be shielded from outsidemagnetic effects by surrounding them with(a) Iron shield (b) Rubber shield(c) Brass shield (d) Glass shield

78. Find the odd one.(a) Marble (b) Chalk(c) Limestone (d) Slaked lime

79. The following is a pseudo-force:(a) Centrepetal force(b) Centrifugal reaction force(c) Centrifugal force(d) Strong nuclear force

80. The hydraulic brake used in automobiles is a directapplication of(a) Archimedes’ principle(b) Torricellean law(c) Bernoulli’s Theorem(d) Pascal’s law

81. Of the following, which is the fastest?(a) CD-ROM (b) RAM(c) Registers (d) Cache

82. An alternate name for the completelyinterconnected network topology is(a) Mesh (b) Star(c) Tree (d) Ring

83. Amides can be converted to amines by thereaction named(a) Perkin (b) Claisen(c) Hoffman (d) Clemmesen

84. The base used as an antacid is(a) Calcium hydroxide(b) Barium hydroxide(c) Magnesium hydroxide(d) Silver hydroxide

85. A process which is not helpful in the preventionof rusting of iron is(a) annealing (b) applying grease(c) galvanising (d) painting

86. Denatured alcohol(a) is a form of alcohol(b) is unfit for drinking as it contains poisonous

substances(c) contains coloured impurities(d) is sweet to taste

87. Phenolics as pollutants can be removed fromwaste water by use of(a) Ion exchange resin technique(b) Electrolyte decomposition technique(c) Reverse osmosis method(d) Polymeric adsorbents

88. The stability of a pond ecosystem depends on(a) micro-organisms and fishes(b) micro-organisms and zoo planktons(c) fishes and reptiles(d) producers and consumers

yoursmahboob.w

ordpress.com

161SSC CGL SolvedPaper

89. Supersonic air planes create a shock wave called(a) Transition wave (b) Ultrasound(c) Transverse wave (d) Sonic boom

90. The main factor which determines the balance ofnature is(a) human activities(b) Rabit and habitat(c) environmental conditions(d) availability of food

91. The danger signals are red while the eye is moresensitive to yellow because(a) absorption in red is less than yellow and

hence red is visible from a distance(b) scattering in yellow light is less than red(c) the wavelength of red light is more than

yellow light(d) none of the above reasons

92. One of the following Chief Ministers receivedthe Highest Civilian Award “Bharat Ratna”(a) Lalu Prasad Yadav of Bihar(b) Late M. G. Ramachandran of Tamilnadu(c) Jyothi Basu of West Bengal(d) Late N. T. Rama Rao of Andhra Pradesh

93. The first ‘disabled athelete’ to qualify for an ablebodied event in the 2012 London Olympics in400 metres race is(a) Oscar Pistorius (b) Walter Rallis(c) Raymond Irchovich (d) Gerald Hangovin

94. Which one of the following film-actors has beenconferred the Honorary Doctorate in Arts andCulture by the Bedford University, London?(a) Shahrukh Khan (b) Amir Khan(c) Saif Ali Khan (d) Anil Kapoor

95. The city of Prayag was named Allahabad - thecity of Allah by(a) Aurangzeb (b) Akbar(c) Shahjahan (d) Bahadur Shah Zafar

96. Spot the odd one from the following:(a) Tsunami (b) Earthquakes(c) Windmills (d) Cyclones

97. Who among the following has won the Pantaloon’sFemina Miss India World, 2012 Award?(a) Prachi Mishra (b) Vanya Mishra(c) Rochella Maria (d) Sandhya Agarwal

98. Which one of the following statements is notcorrect about India’s population as per the 2011census?(a) Literacy rate has gone up to 74% from 65%(b) Kerala has the highest literacy rate(c) Bihar has the lowest literacy rate(d) Men outnumber women in growth of

literates

99. Who advocated the adoption of ‘PURA’ modelto eradicate rural poverty?(a) Dr. A. P. J. Abdul Kalam(b) Sri Abhijit Sen(c) Maulana Abdul Kalam Azad(d) Prof. A. M. Patha

100. Which one of the following though called agarden is infact, not a garden?(a) Vrindavan Garden of Mysore(b) Hanging Garden of Mumbai(c) Eden Garden of Kolkata(d) Shalimar Garden of Kashmir

PART-C : QUANTITATIVE APTITUDE

101. What would be the sum of1 + 3 + 5 + 7 + 9 + 11 + 13 + 15 + ....... up to 15thterm?(a) 250 (b) 240(c) 225 (d) 265

102. The least number which when divided by 48, 64,90, 120 will leave the remainders 38, 54, 80, 110respectively, is(a) 2870 (b) 2860(c) 2890 (d) 2880

103. A, B, C subscribe together ` 50,000 for a business.A subscribes ̀ 4,000 more than B and B ` 5,000more than C. Out of a total profit of ` 35,000, Areceives(a) ` 8,500 (b) ` 11,998(c) ` 12,600 (d) ` 14,700

104. If 13 + 23 + ....... + 93 = 2025, then the approx. valueof(0.11)3 + (0.22)3 + ....... + (0.99)3 is(a) 0.2695 (b) 0.3695(c) 2.695 (d) 3.695

105. With a two digit prime number, if 18 is added, weget another prime number with digits reversed.How many such numbers are possible?(a) 2 (b) 3(c) 0 (d) 1

106. X and Y can do a piece of work in 30 days. Theywork together for 6 days and then X quits and Yfinishes the work in 32 more days. In how manydays can Y do the piece of work alone?(a) 30 days (b) 32 days(c) 34 days (d) 40 days

107. A metal pipe of negligible thickness has radius21 cm and length 90 cm. The outer curved surfacearea of the pipe in square cm is(a) 11880 (b) 11680(c) 11480 (d) 10080

yoursmahboob.w

ordpress.com

162 SSC CGL SolvedPaperc

108. A businessman allows a discount of 10% on thewritten price. How much above the cost pricemust he mark his goods to make a profit of 17%?(a) 30% (b) 20%(c) 27% (d) 18%

109. 465 coins consists of 1 rupee, 50 paise and 25paise coins. Their values are in the ratio 5: 3 : 1.The number of each type of coins respectively is(a) 155, 186, 124 (b) 154, 187, 124(c) 154, 185, 126 (d) 150, 140, 175

110. A batsman makes a score of 58 runs in the 15thinnings and thus increases his average by 3 runs.What is the average after 15th inning?(a) 12 (b) 14(c) 16 (d) 18

111. The average of 5 consecutive numbers is n. Ifthe next two numbers are also included, theaverage of the 7 numbers will(a) increase by 2 (b) increase by 1(c) remain the same (d) increase by 1.4

112. A man sold an article at a loss of 20%. If he sellsthe article for ` 12 more, he would have gained10%. The cost price of the article is(a) ` 60 (b) ` 40(c) ` 30 (d) ` 22

113. A trader has a weighing balance that shows 1,200gm for a kilogram. He further marks up his costprice by 10%. Then the net profit percentage is(a) 32% (b) 23%(c) 31.75% (d) 23.5%

114. A car covers four successive 6 km stretches atspeeds of 25 kmph, 50 kmph, 75 kmph and 150kmph respectively. Its average speed over thisdistance is(a) 25 kmph (b) 50 kmph(c) 75 kmph (d) 150 kmph

115. The difference between C. I. (CompoundInterest) and S.I. (Simple Interest) on a sum of ̀4,000 for 2 years at 5% p.a. payable yearly is(a) ` 20 (b) ` 10(c) ` 50 (d) ` 60

116. If 4=

+abx

a b , then the value of

2 22 2

+ ++- -

x a x bx a x b is

(a) a (b) b(c) 0 (d) 2

117. If x = 997, y = 998, z = 999, then the value of x2 +y2 + z2

– xy – yz – zx will be(a) 3 (b) 9(c) 16 (d) 4

118. The area (in sq. unit) of the triangle formed bythe three graphs of the equations x = 4, y = 3,and 3x + 4y = 12, is(a) 12 (b) 10(c) 6 (d) 8

119. If a + b + c = 8, then the value of(a – 4)3 + (b – 3)3 + (c – 1)3 – 3 (a – 4) (b – 3) (c –1) is(a) 2 (b) 4(c) 1 (d) 0

120. If 1= +x aa

, 1= -y aa

, then the value

of x4 + y4 – 2x2y2 is(a) 16 (b) 20(c) 10 (d) 5

121. If 15 53

+ =aa

, then the value of 22

1925

+aa

is

(a)515 (b)

295

(c)525 (d)

395

122. If 3 2 2= +x , then the value of 1-xx

is

(a) 2 2± (b) 2±

(c) 2± (d)12

±

123. If 0+ + =a b c , the value of

2 2 2æ öç ÷+ +ç ÷è ø

a b cbc ca ab is

(a) 2 (b) 3(c) 4 (d) 5

124. If a, b, c are real and a3 + b3 + c3 = 3abc and0+ + ¹a b c , then the relation between a, b, c

will be(a) a + b = c (b) a + c = b(c) a = b = c (d) b + c = a

125. The radius of the circumcircle of the triangle madeby x-axis, y-axis and 4x + 3y = 12 is(a) 2 unit (b) 2.5 unit(c) 3 unit (d) 4 unit

126. The length of the circum-radius of a trianglehaving sides of lengths 12 cm, 16 cm and 20 cmis(a) 15 cm (b) 10 cm(c) 18 cm (d) 16 cm

yoursmahboob.w

ordpress.com

163SSC CGL SolvedPaper

127. If D is the mid-point of the side BC of D ABC andthe area of D ABD is 16 cm2, then the area of DABC is(a) 16 cm2 (b) 24 cm2

(c) 32 cm2 (d) 48 cm2

128. ABC is a triangle. The medians CD and BEintersect each other at O. Then D ODE : D ABCis(a) 1 : 3 (b) 1 : 4(c) 1 : 6 (d) 1 : 12

129. If P, R, T are the area of a parallelogram, a rhombusand a triangle standing on the same base andbetween the same parallels, which of thefollowing is true?(a) R < P < T (b) P > R > T(c) R = P = T (d) R = P = 2T

130. AB is a diameter of the circumcircle of D APB; Nis the foot of the perpendicular drawn from thepoint P on AB. If AP = 8 cm and BP = 6 cm, thenthe length of BN is(a) 3.6 cm (b) 3 cm(c) 3.4 cm (d) 3.5 cm

131. Two circles with same radius r intersect eachother and one passes through the centre of theother. Then the length of the common chord is(a) r (b) 3r

(c)3

2r (d) 5r

132. The bisector of ÐA of DABC cuts BC at D andthe circumcircle of the triangle at E. Then(a) AB : AC = BD : DC(b) AD : AC = AE : AB(c) AB : AD = AC : AE(d) AB : AD = AE : AC

133. Two circles intersect each other at P and Q. PAand PB are two diameters. Then ÐAQB is(a) 120° (b) 135°(c) 160° (d) 180°

134. O is the centre of the circle passing through thepoints A, B and C such that ÐBAO = 30°, ÐBCO= 40° and ÐAOC = x°. What is the value of x ?(a) 70° (b) 140°(c) 210° (d) 280°

135. A and B are centres of the two circles whoseradii are 5 cm and 2 cm respectively. The directcommon tangents to the circles meet ABextended at P. Then P divides AB.(a) externally in the ratio 5 : 2(b) internally in the ratio 2 : 5(c) internally in the ratio 5 : 2(d) externally in the ratio 7 : 2

136. A wheel rotates 3.5 times in one second. Whattime (in seconds) does the wheel take to rotate55 radian of angle?(a) 1.5 (b) 2.5(c) 3.5 (d) 4.5

137. If 2y cosq = x sinq and 2x secq – y cosecq = 3,then the relation between x and y is(a) 2x2 + y2 = 2 (b) x2 + 4y2 = 4(c) x2 + 4y2 = 1 (d) 4x2 + y2 = 4

138. If sec tan 3q + q = , then the positive value ofsinq is

(a) 0 (b)12

(c)3

2(d) 1

139. The radian measure of 63 14 51¢ ¢¢° is

(a)28118000

pæ öç ÷è ø

c(b)

38118000

pæ öç ÷è ø

c

(c)48118000

pæ öç ÷è ø

c(d)

58118000

pæ öç ÷è ø

c

140. In a triangle ABC, AB = AC, BA is produced to Din such a manner that AC = AD. The circularmeasure of ÐBCD is

(a) 6p

(b) 3p

(c)23p

(d)2p

141. If 4 4

2 2cos sin 1cos sin

a a+ =b b

, then the value of

4 4

2 2cos sin

cos sin

b b+

a a is

(a) 4 (b) 0

(c)18 (d) 1

142.sin cos 1sin cos 1

q - q +q + q -

where2pæ öq ¹ç ÷è ø

is equal to

(a)1 sin

cos+ q

q (b)1 sin

cos- q

q

(c)1 cos

sin- q

q (d)1 cos

sin+ q

q

yoursmahboob.w

ordpress.com

164 SSC CGL SolvedPaperc

143. The angles of elevation of the top of a towerstanding on a horizontal plane from two pointson a line passing through the foot of the towerat a distance 9 ft and 16 ft respectively arecomplementary angles. Then the height of thetower is(a) 9 ft (b) 12 ft(c) 16 ft (d) 144 ft

144. If sin2a = cos3a, then the value of (cot6a – cot2a)is(a) 1 (b) 0(c) – 1 (d) 2

145. The simplified value of(1 + tanq + secq) (1 + cotq – cosecq) is(a) – 2 (b) 2(c) 1 (d) – 1

Directions (Qs. 146-150) : The following pie-chartrepresents the profits earned by a certain companyin seven consecutive years. Study the pie-chartcarefully and answer the question.

Years19911992

1993

1994

199519961997

42°

24°

60°

54° 72°

66°

146. If the expenditure in the year 1993 was 30% morethan the expenditure in the year 1991, then theincome in the year 1993 exceeds the income inthe year 1991 by 30% of(a) the income in the year 1991(b) the expenditure in the year 1993(c) the income in the year 1993(d) the expenditure in the year 1991

147. If x% of the total of profits earned in all the givenyears is same as the profit earned in the year1994, then x is

(a)2163 (b)

1333

(c)1122

(d)2113

148. The ratios of expenditures and incomes in theyears 1992, 1994 and 1996 are given to be 6 : 5 :

8 and 2 : 3 : 4 respectively. The ratio of the incomein the year 1996 to the total expenditure in theyears 1992 and 1994 is(a) 40 : 11 (b) 10 : 7(c) 20 : 11 (d) 20 : 13

149. The year in which the profit is nearest to theaverage of the profits earned in all the givenyears is(a) 1991 (b) 1995(c) 1993 (d) 1994

150. If the income in the year 1997 was 5 times theexpenditure made in the same year, then the ratioof the profit earned in the year 1991 to theexpenditure in the year 1997 was(a) 11 : 28 (b) 44 : 7(c) 28 : 11 (d) 7 : 44

PART-D : ENGLISH COMPREHENSION

Directions (Qs. 151 - 155) : In the following questionssome parts of the sentences have errors and somehave none. Find out which part of a sentence has anerror.

151. Air pollution, together with littering ,a

are causing many problemsb

in our cities.c

No error.d

152.The accused refused

ato answer to the policeman

b

on duty.c

No error.d

153.What is

athe use of me

b attending the session?

cNo error.

d

154.We met our prospective employer,

afor a briefing session

bin the Taj Hotel.

cNo error.

d

yoursmahboob.w

ordpress.com

165SSC CGL SolvedPaper

155. Because of the severe snow storm and the road blocks,

athe air force dropped food and

bmedical supplies close to the city.

cNo error.

d

Directions (Qs.156 - 160) : In the following questionssentences are given with blanks to be filled in withan appropriate word(s). Four alternatives aresuggested for each question. Choose the correctalternative out of the four and indicate it byblackening the appropriate rectangle [ ] in theAnswer-Sheet.156. ______________ pollution control measures

are expensive, many industries hesitate to adoptthem.(a) Although (b) However(c) Because (d) Despite

157. It is not ______________ for a man to beconfined to the pursuit of wealth.(a) healthy (b) easy(c) possible (d) common

158. ______________ his being innocent of thecrime, the judge sentenced him to one yearimprisonment.(a) Inspite of (b) In case of(c) On account of (d) In the event of

159. It is a story of two men and a batch of _____armoured cars.(a) deceased (b) diseased(c) decrepit (d) defeated

160. Although there is _________ gunfire, there isno stiff resistance to the revolutionary army.(a) bitter (b) meagre(c) continuous (d) sporadic

Directions (Qs. 161 - 165) : In the following questionsout of the four alternatives, choose the one whichbest expresses the meaning of the given word.161. Vociferous

(a) violent (b) loud(c) secret (d) true

162. Fictional(a) genuine (b) authentic(c) fanciful (d) real

163. Trivial(a) crucial (b) significant(c) vital (d) ordinary

164. Impudent(a) Vigilant (b) Astute(c) Insolent (d) Arrogant

165. Pompous(a) Pretentious (b) Supportive(c) Demanding (d) Flashy

Directions (Qs. 166 - 170) : In the following questionschoose the word opposite in meaning to the givenword and mark it in the Answer-Sheet.166. Cultivated

(a) Crude (b) Genteel(c) Suave (d) Refined

167. Impertinent(a) Insolent (b) Impudent(c) Cheeky (d) Courteous

168. Divulge(a) Disseminate (b) Dissemble(c) Publicize (d) Transmit

169. Appreciation(a) Aspersion (b) Admiration(c) Commendation (d) Compliment

170. Supple(a) Pliant (b) Pliable(c) Rigid (d) Flexible

Directions (Qs. 171 - 175) : In the following questionsfour alternatives are given for the idiom / phrase andbold italicised in the sentence. Choose the alternativewhich best expresses the meaning of the idiom / phrase.171. Once the case reached the court, the police

washed their hands off it.(a) waited for a response to(b) claimed credit for(c) disassociated themselves from(d) seemed eager to continue

172. She wanted to go hitch-hiking but her motherput her foot down and now she’s going by bus.(a) took a firm stand(b) expressed her displeasure(c) scolded her badly(d) got irritated

173. Adolescence is a period of halcyon days.(a) hard days(b) of mental pressure(c) happy days(d) days of preparation

174. My sincere advice to my maidservant fell onstony ground.(a) was counter productive(b) had a strong impact(c) made on stubborn(d) had little success

yoursmahboob.w

ordpress.com

166 SSC CGL SolvedPaperc

175. He has all his ducks in a row; he is complacent.(a) has everything ready(b) is well organised(c) always scores a zero(d) never gets confused

Directions (Qs. 176 - 180) : In the following questionsa part of the sentence is underlined. Below are givenalternatives to the underlined part at (a), (b) and (c)which may improve the sentence. Choose the correctalternative. In case no improvement is needed youranswer is (d).176. You shall have attended if the court had

instructed you to do so.(a) would have had to attend(b) would attend(c) would have to(d) No improvement

177. The relics of Greece over which such a greatdeal of evidence has been collected should bepreserved.(a) from which (b) on which(c) ascent which (d) N oimprovement

178. When the beverage was ready, they drankpossibly as much as they could.(a) as much as they possibly could(b) as much as possibly they could(c) as much as they could possibly(d) No improvement

179. A citizen is expected to give allegiance to hiscountry of origin.(a) homage(b) loyalty(c) obedience(d) No improvement

180. We were with daggers drawn despite attemptsto understand each other.(a) in(b) on(c) at(d) No improvement

Directions (Qs. 181 - 185) : In the following questionsout of the four alternatives, choose the one whichcan be substituted for the given words / sentence.181. A round or cylindrical container used for storing

things such as food, chemicals or rolls of film(a) tankard (b) canister(c) vessel (d) casket

182. A place of permanent residence(a) abode (b) dormitory(c) domicile (d) apartment

183. That cannot be altered or withdrawn(a) irrevocable (b) irretrievable(c) irrefutable (d) irresistible

184. Money paid to employees on retirement(a) gratuity (b) gift(c) pension (d) arrears

185. A place where clothes are kept(a) closet (b) drawer(c) wardrobe (d) cupboard

Directions (Qs. 186 - 190) : In the following questionsfour words are given in each question, out of whichonly one word is correctly spelt. Find the correctlyspelt word .186. (a) garulous (b) garrulous

(c) garullous (d) garrullous187. (a) marquee (b) markue

(c) marquei (d) marquie188. (a) puissant (b) puiscant

(c) puiscent (d) puissent189. (a) disconncerting (b) disconserting

(c) discuncerting (d) disconcerting190. (a) exilarate (b) exhilerate

(c) exsilarate (d) exhilarate

Directions (Qs. 191 - 200) : In the following questionsyou have two brief passages with 5 question in eachpassage. Read the passages carefully and choose thebest answer to each question out of the fouralternatives.

PASSAGE - IStuck with the development dilemma? Stay away frommanagement courses. Seriously, one of the biggestcomplaints that organisations have about managementcourses is that they fail to impact the participants’ on-the-job behaviour. Some management trainers stressthe need for follow-up and reinforcement on the job.Some go so far as briefing the participants’ managerson what behaviour they should be reinforcing backon the job. Other include a follow-up training day toreview the progress of the participants. None of this isreally going far enough.

The real problem is that course promoters viewdevelopment as something which primarily, takes placein a classroom. A course is an event and events are, bydefinition limited in time. When you talk about follow-up after a course, it is seen as a nice idea, but not as anessential part of the participants’ developmentprogramme. Any rational, empowered individual shouldbe able to take what has been learnt in a course andtransfer it to the work place - or so the argument goes.Another negative aspect of the course mindset is that,primarily, development is thought to be about skill-acquisition.

yoursmahboob.w

ordpress.com

167SSC CGL SolvedPaper

So, it is felt that the distinction between takingthe course and behaving differently in the work placeparallels the distinction between skill-acquisition andskill-application. But can such a sharp distinction bemaintained? Skills are really acquired only in thecontext of applying them on the job, finding themeffective and, therefore, reinforcing them.

The problem with courses is that they are events,while development is an on-going process which,involves, within a complex environment, continualinteraction, regular feedback and adjustment. As wetend to equate development with a one-off event, it isdifficult to get seriously motivated about the follow-up. Anyone paying for a course tends to look at follow-up as an unnecessary and rather costly frill.191. What is the passage about?

(a) personal management(b) development dilemma(c) management courses(d) course promotors’ attitude

192. Which of the following statements is false?(a) Some management trainers stress the need

for follow-up and reinforcement on the job(b) Some suggest a follow-up training day to

review the progress of the participants(c) Some go to the extent of briefing the

participants’ managers on what behaviourthey should be reinforcing back on the job

(d) The real problem is that course promotersview development as something whichdoes not take place during a course.

193. The writer’s attitude, as reflected in the passage,is(a) critical (b) ironic(c) sympathetic (d) philosophical

194. The course promoters’ attitude is(a) self-righteous (b) indifferent(c) easy-going (d) unprogressive

195. The word ‘mindset’ here means(a) a determined mind(b) a (fixed) attitude of mind(c) an open mind(d) mindful

PASSAGE -IIOne may look at life, events, society, history, in anotherway. A way which might, at a stretch, be described asthe Gandhian way, though it may be from times beforeMahatma Gandhi came on the scene. The Gandhianreaction to all grim poverty, squalor and degradation ofthe human being would approximate to effort at self-change and self-improvement, to a regime of livingregulated by discipline from within. To change society,the individual must first change himself. In this way of

looking at life and society, words too begin to meandifferently. Revolution, for instance, is a term frequentlyused, but not always in the sense it has been in thelexicon of the militant. So also with words like peaceand struggle. Even society may mean differently, beingsome kind of organic entity for the militant, and more orless a sum of individuals for the Gandhian. There is yetanother way, which might, for want of a betterdescription, be called the mystic. The mystic’sperspective measures these concerns that transcendpolitical ambition and the dynamism of the reformer,whether he be militant or Gandhian. The mystic measuresthe terror of not knowing the remorseless march of time;he seeks to know what was before birth, what comesafter death? The continuous presence of death, of theconsciousness of death, sets his priorities and values:militants and Gandhians, kings and prophets, must leaveall that they have built; all that they have unbuilt anddepart when messengers of the buffalo-riding Yamacome out of the shadows. Water will to water, dust todust. Think of impermanence. Everything passes.196. The Gandhian reaction of poverty is

(a) a total war on poverty(b) self-discipline(c) self-abnegation(d) a regulated distribution of wealth

197. According to Gandianism, the individual whowants to change society(a) should destroy the existing society(b) must re-form society(c) must change himself(d) may change society without changing

himself198. Who, according to the passage, finds new meaning

for words like revolutions, peace and struggle?(a) A Gandhian who believes in non-violent

revolution(b) A militant(c) A mystic(d) A Gandhian who disciplines himself from

within199. The expression ‘water will to water, dust to dust’

means(a) water and dust can mix well(b) man will become water after death(c) man will one day die and become dust(d) man will become dust and water after death

200. What does society mean to a Gandhian?(a) a sum of individuals(b) an organic entity(c) a regime of living regulated by discipline

from within(d) a disciplined social community

yoursmahboob.w

ordpress.com

168 SSC CGL SolvedPaperc

1. (c) Metre is a unit of length likewise watt is aunit of power.

2. (d) As cube is 3-D of square. Similarly sphereis 3-D of circle.

3. (b) As paper is product of Tree. Similarly glassis a product of sand.

4. (d) As, A C F J

Z X U Q¯ ¯ ¯ ¯

Similarly, E G I N

V T R M¯ ¯ ¯ ¯

5. (a) As, Similarly3A D+¾¾® 3Q T+¾¾®

3C F+¾¾® 3S V+¾¾®3E H+¾¾® 3U X+¾¾®3G J+¾¾® 3W Z+¾¾®

6. (d) As, Similarly1E F+¾¾® 1F G+¾¾®2G I+¾¾® 2H J+¾¾®

3I L+¾¾® 3J M+¾¾®4K O+¾¾® 4L P+¾¾®

7. (b) The relationship is x : (x2 – 9).8. (c) The relationship is x : x (x + 1)10. (b) As all terms given in question are medical

terms except geology.12. (d) All except Ellora are famous for temples,

while Ellora is famous for caves.13. (a) Consonants and vowel are used in option

(b), (c) and (d). Only consonants are usedin option (a).

14. (b) E F H+1 +2

O P Q+1 +1

B C E+1 +2

I J L+1 +2

15. (c) D H

4 8¯ ¯

F J

6 10¯ ¯

H K

8 11¯

¯ ¯

E555F

P R

16 18¯ ¯

Pair of odd and even number16. (b) 24 = 52 – 1 80 = 92 – 1

49 = 72 – 0 15 = 42 – 117. (c) All are squares except (c).18. (d) 704 ̧ 11 = 64 256 ̧ 4 = 64

832 ̧ 13 = 64 310 ̧ 5 = 6219. (b) Perk > Pick > Pile > Pith > Pour20. (c) Silver jublee - 25 yr.

Golden jublee - 50 yr.Diamond jublee - 75 yr.Centenary - 100 yrMillennium - 1000 yr.

21. (d) Annual - 1 year Monthly - 30 daysWeekly - 7 days Biannual - 6 monthFortnightly - 15 days.

22. (a) a b c b c a ca b / a b c bc a c ab.23. (a) B C F G ¾¾® 2, 3, 6, 7

J K N O ¾¾® 10, 11, 14, 15

R S V W ¾¾® 18, 19, 22, 23Next sequence = 26, 27, 30, 31 = 26, 1, 4, 5 = Z A D E (subtract the excess value by 26)

24. (c) C I M, H N R, M S W, R X B

+5+5+5

+5+5+5

+5+5+5

25. (b)

2 3 6 7 14 15 30

+1 ×2 +1 ×2 +1 ×226. (b) 4th term 23 = 5 × 4 + 3 = 23

3rd term 122 = 5 × 23 + 7 = 1222nd term ? = 5 × 122 + 11 = 6211st term 3120 = 5 × 621 + 15 =3120

HINTS & SOLUTIONS

yoursmahboob.w

ordpress.com

169SSC CGL SolvedPaper

27. (b)

0 5 60 615

+5 +55 +555 +5555

6170

28. (c) Nithya is Sam’s Sister and Mogan is Sam’sFather ÞNithya is Mogan’s Daughter.Selvan is Rajan’s Son and Rajan is Mogan’sBrother ÞSelvan is Mogan’s Nephew.So, Nithya is Selvan’s Cousin.

29. (d) Let father’s age is x yr.

Son’s age is x4

yr..

xx 354

+ = Þ x = 28 yr..Father’s age after 8 year is 36 years.

30. (a) Mother’s husband Þ FatherFather’s sister Þ AuntSo, man’s and lady’s aunt is same positionÞ both are brother and sister.

31. (c) If South East becomes North then south westbecomes east as shown in direction chart.

SW

W

NW

NE

ESE

S

NW

SEN

SSWE

WNEN

32. (c) NATION as a letter is not there in wordCONTENTION.

33. (c) GRAPE = 2 7 3 5 4FOUR = 1 6 8 7So, G = 2, R = 7, A = 3, P = 5, E = 4, F = 1, O = 6,U = 8,R = 7GROUP = 2 7 6 8 5

34. (a) As W A Y I N

3 3 3 3 3T X V F K

- ¯ - ¯ - ¯ - ¯ - ¯

Similarly, L B U K

3 3 3 3I Y R H

- ¯ - ¯ - ¯ - ¯

35. (b) As,P A R T N E R

1 1 1 1 1 1 1O Z Q S M D Q

- ¯ - ¯ - ¯ - ¯ - ¯ - ¯ - ¯

Similarly,S E G M E N T

1 1 1 1 1 1 1R D F L D M S

- ¯ - ¯ - ¯ - ¯ - ¯ - ¯ - ¯

36. (c) As,D O C T O R

2 2 2 2 2 2F Q E V Q T

+ ¯ + ¯ + ¯ + ¯ + ¯ + ¯

Similarly,P A T I E N T

2 2 2 2 2 2 2R C V K G P V

+ ¯ + ¯ + ¯ + ¯ + ¯ + ¯ + ¯

37. (c) As, 3 × 7 = 21, 11 × 7 = 774 × 9 = 36, 12 × 9 = 108Therefore,14 × 8 = 112? × 8 = 24

? 3=39. (c) 90 ¸ 18 × 6 + 30 – 4 = 5640. (d) As, 73 + 46 = 42

7 – 3 = 4, 4 + 6 = 10Add 4 + 10 = 1414 × 3 = 42Similarly, 6 – 2 = 4, 8 + 0 = 84 + 8 = 1212 × 3 = 36

41. (a) Series

447 458 489 540

+11 +31 +51 +71

611

42. (a)

Startingpoint

North

1 km·

1 km

1 km

yoursmahboob.w

ordpress.com

170 SSC CGL SolvedPaperc

W

N

S

E

43. (d)

LeftLeft

Right Right

South (A)

About turn

Starting point of B.

North (B)

Left Left

Starting point of A

About turn

About turn-turning in reverse direction.46. (d)

47. (b)

50. (c) S ¾¾® 56, 68, 79, 85, 97H ¾¾® 01, 14, 20, 33, 42R ¾¾® 00, 13, 22, 31, 44I ¾¾® 04, 10, 23, 32, 41

52. (b) In order to raise the standards of the banksinternationally, a number of committeeswere appointed by RBI. Among themNarasimham committee I (1991), NarasimhamCommittee II (1998) and Verma Committee(1999) were influential in improvinginternational standards, and led to bankingsector reforms, globally flexible to itsderegulation, norms and conditions etc.The above said committees have basicallyidentified the causes for the weak banksand guidelines have been given to improvetheir efficiency.

54. (c) Here, statement of the question is wrong.The tenure from 2012 to 2017 is designatedas12th Five Year Plan not 13th Five YearPlan. The tenure of 13th Five-Year Planwould be 2018 - 2022.

57. (b) The Fundamental Rights can be suspendedduring the Emergency under Article 359 ofthe Constitution by the President of India.

60. (c) Domestic Violence on Women Act 2005 isthe first significant attempt in India torecognise domestic abuse as a punishableoffence, to extend its provisions to thosein live-in relationships, and to provide foremergency relief for the victims, in additionto legal recourse.

61. (d) Lord Canning was the Governor General ofIndia from 1856 - 1862 and the first Viceroyin India from 1 November 1858. LordMountbatten was the First GovernorGeneral of Independent India.

62. (b) 1191 - First Battle of Tarain in whichPrithviraj Chauhan defeated Mohd.Ghori.1192 - Second Battle of Tarain inwhich Mohd.Ghori defeated PrithvirajChauhan.

64. (c) The first major attempt in curriculumreconstruction in India was made in 1937when Gandhiji propounded the idea of BasicEducation.

65. (b) Simon Commission (1927) > Dandi March(1930) > Gandhi Irwin Pact (1931) > PoonaPact (1932)

69. (c) Two pollutants emitted by motor vehiclesreact to form ground-level ozone or smogwhich can cause respiratory problems andreduce visibility.

73. (c) Cardiac muscle is an involuntary striatedmuscle tissue found only in the organ heart.Involuntary muscles are smooth musclesthat are not directly controllable at will. Forexample You don't have to remind yourselfto make your heart beat, so it is involuntary.Voluntary muscles are controllable likethose found in your arms, legs, hands, etc.

75. (c) Ringworm is common disease, especiallyamong children. It is caused by a fungus,not a worm like the name suggests. It is acommon and highly infectious skin infectionthat causes a ring-like red rash on the skin.

yoursmahboob.w

ordpress.com

171SSC CGL SolvedPaper

79. (c) Centrifugal force is an example of a pseudo-force, that is, an apparent force to someonewhose frame of reference is not at rest ormoving with a constant velocity. In the caseof centrifugal force, the frame of referenceis rotating.

81. (c) Register memory is faster . Register is anintegral part of the CPU chip .Therebyfetching data is faster. Cache on the otherhand is a seperate memory unit

84. (c) An antacid is a substance which neutralizesstomach acidity.

85. (a) Annealing is the process by which bothmetal and glass are treated with heat in orderto change their properties.

91. (c) This is because the scattering in red light isless than that of yellow colour. The longestvisible wavelength is red and the shortestis violet. The wavelength of red light is morethan yellow light.

95. (b) Emperor Akbar named Prayag as Allahabad- City of God- also called Allahabad in 1575AD. The city of Allahabad is situated atthe confluence of three rivers - Ganga,Yamuna and the invisible Saraswati. Every12th year when the waters are felt to beespecially purifying, Allahabad holds amuch greater festival called Kumbh Mela.Built by Emperor Akbar in 1583 AD, theAllahbad fort stands on the banks of theriver Yamuna near the confluence site i.eSANGAM.

101. (c) The sum forms A.P.First term (a) = 1Common difference (d) = 2

Sum of 15 term = (2 ( 1) )2n a n d+ -

Sum = 15 (2 1 (15 1)2)2´ + -

= 15 30 2252´ =

102. (a) Here, (48 – 38) = 10, (64 – 54) = 10, (90 – 80)= 10 and (120 – 110) = 10.\ Required number = (L.C.M of 48, 64, 90and 120) – 10 = 2870

103. (d) A = B + 4000B = C + 5000A + B + C = 50000

A + A – 4000 + A – 9000 = 50000So, A = 21000B = 17000C = 12000\ A : B : C = 21000 : 17000 : 12000 = 21 : 17 : 12

A’s Profit =21 3500050´ = ̀ 14700

105. (a) Let the number be 10x + y.According to condition10x + y + 18 = 10y + xy – x = 2So those numbers are 02, 13, 24, 35, 46, 57,68, 79, 80But 13 and 79 are prime numbers.

106. (d) (x + y) ‘s 6 days’ work = 1 16

30 5æ ö´ =ç ÷è ø

.

Remaining work = 1 415 5

æ ö- =ç ÷è ø

Now, 45

work is done by y in 32 days.

Whole work will be done by y in 5324

æ ö´ç ÷è ø=

40 days.107. (a) Curved surface area of cylinder = 2prh

222 21 90 11880 sq.cm7

= ´ ´ ´ =

108. (a) Let CP = ̀ 100Then, S.P = ̀ 117Let marked price be Rs x.Then, 90% of x = 117

117 100 13090

x ´æ öÞ = =ç ÷è ø\ Marked price = 30% above C.P.

109. (a) The ratio of number of coins = 5 : 6 : 4\ The number of one rupee coins

465 5 1555 6 4= ´ =

+ +The number of 50 paise coins

465 6 1865 6 4= ´ =

+ +The number of 25 paise coins

465 4 1245 6 4= ´ =

+ +

yoursmahboob.w

ordpress.com

172 SSC CGL SolvedPaperc

110. (c) Let average for 14 innings be x. Then,

14 58 3 15 45 14 58 1315x x x x x+ = + Þ + = + Þ =

\ New average = (x + 3) = 13 + 3 = 16 runs111. (b) Let the numbers be n – 2, n – 1, n, n + 1 and

n + 2. Their average = n.Next two consecutive numbers are n + 3and n + 4.Therefore the average of 7 consecutivenumbers

( 2) ( 1) ( 1) ( 2)( 3) ( 4)

7

n n n n nn n

- + - + + + + + ++ + +=

5 2 7 17

n n n+ += = +

112. (b) S.P = C.P 80

100æ öç ÷è ø

Þ 4S.P C.P5

= ...(1)

110 11S.P 12 C.P S.P C.P 12100 10

æ ö+ = Þ = -ç ÷è ø...(2)

From eqn. (1) and (2)

4 11C.P C.P 125 10

= -

11 4C.P C.P 12 C.P 4010 5

Þ - = Þ = `

113. (a) The trader professes to sell 1200 kg but sellsonly 1000 kg.So profit = 20%Markup = 10%

Total profit 10 2010 20 32%100´= + + =

114. (b)Total Distance CoveredAverage Speed =

Total Time Taken

6

6+6+6+6 24= 6 6 6 6 1 1 1 125 50 75 150 25 50 75 150

Þé ù+ + + + + +ê úë û

24 300 50 km/hr6 24

´= Þ´

115. (b) 2Required difference = (100)

2PR

4000 5 5 10100 100

´ ´Þ´

=`

116. (d)2 22 2

x a x bx a x b

+ ++- -

Applying compodendo and Dividendo2 2 2 22 2 2 2

x a x a x b x bx a x a x b x b

+ + - + + -Þ ++ - + + - +

2 2 4 44 4 2 2 ( )2 ( )2

x x x x ab aba b a b a b a a b b

Þ + Þ + Þ ++ +

2 2 2b aa b a b

Þ + Þ+ +

117. (a) x2 + y2 + z2 – xy – yz – zx2 2 22 ( )

2x y z xy yz zx= + + - - -

2 2 21 (2 2 2 2 2 2 )2

x y z xy yz zx= + + - - -

2 2 2 2 2 21 ( 2 2 2 )2

x y xy y z yz x z zx= + - + + - + + -

2 2 21[( ) ( ) ( ) ]2

x y y z z x= - + - + -

2 2 21 [(997 998) (998 999) (999 997) ]2

= - + - + -

2 2 21 1[1 1 2 ] 6 32 2

= + + = ´ =

118. (c) x = 4 ...(1)y = 3 ...(2)3x + 4y = 12 ...(3)Putting x = 0 in 3rd equation we get y = 4Putting y = 0 in 3rd equation we get x = 3The triangle will be formed by joining thepoints (3, 0) and (0, 4).So, base = 3 and altitude = 4

Area = 1 1 3 4 62 2

b h´ ´ Þ ´ ´ =

119. (d) We have x3 + y3 + z3 – 3xyz = (x + y + z)(x2 + y2 + z2–xy – yz - zx)

Here x = a – 4, y = b – 3, z = c –1So, given expression is (x + y + z)

(x2 + y2 + z2 – xy – yz – zx)= (a – 4 + b – 3 + c – 1) (x2 + y2 + z2 – xy – yz– zx)= (a + b + c – 8) (x2 + y2 + z2 – xy – yz – zx)= (8 – 8) (x2 + y2 + z2 – xy – yz – zx)= 0

yoursmahboob.w

ordpress.com

173SSC CGL SolvedPaper

120. (a) x4 + y4 – 2x2y2

Þ (x2 – y2)2 Þ [(x + y) (x – y)]2

21 1 1 1a a a aa a a a

é ùæ öæ öÞ + + - + - +ê úç ÷ç ÷è øè øë û

222 16aa

æ öÞ ´ Þç ÷è ø

121. (d)15 5

3a

a+ =

Multiply by 35 on both sides

3 1 35 55 3 5

aa

æ ö+ = ´ç ÷è ø

13 35

aa

+ =

Squaring on both sides

22

1 19 2 3 9525

a aaa

+ + ´ ´ =

22

1925

aa

Þ + 6 3995 5

= - =

122. (b) 3 2 2x = +

1 1 3 2 23 2 2 3 2 2x

-= ´+ -

1 3 2 2 3 2 29 8x-= = -

-21 1 2x x

xxæ ö- = + -ç ÷è ø

21xx

æ ö-ç ÷è ø

3 2 2 3 2 2 2= + + - - = 4

1 4 2æ ö- = = ±ç ÷è øx

x123. (b) If a + b + c = 0

then a3 + b3 + c3 = 3abcDividing both sides by abc

3 3 3 3a b c abcabc abc abc abc

+ + =

2 2 23a b c

bc ac ab+ + =

125. (b) Putting x = 0 in 4x + 3y = 12 we get y = 4Putting y = 0 in 4x + 3y = 12 we get x = 3The triangle so formed is right angle trianglewith points (0, 0) (4, 0) (0, 3)So diameter is the hypotenus of triangle

16 9= + = 5 unitRadius = 2.5 unit

126. (b) Circum Radius (R)=4 Area of triangle´

abc

[where a, b and c are sides of triangle]

Area of Triangle = s(s )(s )(s )a b c- - -

242

a b cs + +é ù\ = =ê úë û

Area of Triangle 24 12 8 4= ´ ´ ´ = 8 × 3× 4 cm2

12 16 20R 10 cm4 8 3 4

´ ´= =´ ´ ´

127. (c) Area of DABD = 16 cm2

Area of DABC = 2 × Area of DABD [Q Intriangle, the midpoint of the opposite side,divides it into two congruent triangles. Sotheir areas are equal and each is half thearea of the original triangle]Þ 32 cm2

128. (d) Area of 1ODE OK DE2

D = ´

1 1 BC OK2 2

æ ö= ´ç ÷è øA

EK

O

F CB

D

1[BC (AO AK)]4

= ´ -

1 2 1BC AF AF4 3 2

é ùæ ö= ´ -ç ÷ê úè øë û

1 1 14 3 2

AF BCé ù= ´ ´ê úë û =

112

area of DABC

= 1 : 12

yoursmahboob.w

ordpress.com

174 SSC CGL SolvedPaperc

129. (d) Parallelogram Area = l × bRhombus Area = l × b

Triangle Area2

l b´=

Therefore R = P = 2T.130. (a) Since AB is a diameter. Then ÐAPB = 90°

(angle in the semicircle)DBPN ~ DAPBSo, BN = BP2 / AB

BN = 6 6 3.6cm10´ =

131. (b) A

O

r

M O'

B

xr

In DAOMr2 = AM2 + x2

AM2 = r2 – x2 ...(1)In DAMO'r2 = (r – x)2 + AM2

AM2 = r2 – (r – x)2 ...(2)From eqn. (1) & (2)r2 – x2 = r2 – (r – x)2

Þ 2rx = r2

2rxÞ =

From eq. (1)2

2 2 23AM2 4rr ræ ö= - =ç ÷è ø

AM = 32

r

Length of chord AB = 2AM

= 32 32

r r´ =

133. (d)O O¢

P

QBA

ÐAQP = 2p

(Angle in the semicircle is 90°)

ÐBQP = 2p

(Angle in the semicircle is 90°)

ÐAQB = ÐAQP + ÐBQP = 2p

+ 2p

Þ p

or 180°134. (b) In DAOB

AO = BO (radii of circles)\ ÐABO = ÐBAO = 30ºIn DBOC

30º

40º

A

B C

O

BO = CO (radii of circles)\ ÐBCO = ÐOBC = 40ºÐABC = ÐABO + ÐOBCÐABC = 30° + 40° = 70°2 × ÐABC = ÐAOC Þ x° = 140

135. (a) Externally in the ratio of 5 : 2136. (b) Radian covered in one second

222 3.57

= ´ ´

Time required to covered 55 radian55 2.5222 3.57

= =´ ´

137. (b) 2y cos q = x sin q2sin cosyx

Þ q = q

And 2x sec q – y cosec q = 3

2 sec 3sin

yxÞ q - =q

2 3cos 2 cos

x yxy

Þ - =q q

33cos cos2 2

xxÞ q = Þ q =

Now sin2 q + cos2 q = 12

2 14xyÞ + =

2 24 4y xÞ + =

yoursmahboob.w

ordpress.com

175SSC CGL SolvedPaper

138. (b) sec2 q – tan2 q = 1(sec q + tan q) (sec q – tan q) = 1

13(sec tan ) 1 sec tan3

q - q = Þ q - q =

...(1)

sec tan 3q + q = (Given) ...(2)Adding eqn. (1) and (2)

1 4 22sec 3 2sec sec3 3 3

q = + Þ q = Þ q =

3 1cos sec2 cos

é ù\ q = q =ê úqë ûQ

Therefore, 2sin 1 cosq = - q

3 114 2

Þ - =

139. (a)'5163 14'

60æ ö° ç ÷è ø

[1 minute = 60 seconds]

' '17 297 29763 14 63 63º20 20 20 60

é ù é ùÞ ° + Þ ° Þ +ê ú ê ú ´ë û ë û[1 degree = 60 minutes]

75897 75897 radian1200 1200 180

° pæ öÞ Þ ´ç ÷è ø

28118000

cæ öÞ pç ÷è ø140. (d) AB = AC

\ ÐABC = ÐACB ...(1)

A

B C

D

[opposite angle of equal sides are equal]AC = AD\ ÐACD = ÐADC ...(2)In a triangle,ÐABC + ÐADC + ÐDCB = 180°ÐABC + ÐADC + ÐACB + ÐACD = 180°

2ÐACB + 2ÐAOC = 180°[From eqn. (1) & (2)]

\ÐBCD = 90° or p / 2

141. (d)4 4

2 2cos sin 1cos sin

a a+ =b b

Þ cos4 a sin2 b + sin4 a cos2 b = cos2 b sin2 bÞ cos4 a (1 – cos2 b) + cos2 b (1 – cos2 a)2

= cos2 b (1 – cos2 b)Þ cos4 a – cos4 a cos2 b + cos2 b – 2 cos2 a

cos2 b + cos4 a cos2 b = cos2 b – cos4 bÞ cos4 a – 2 cos2 a cos2 b + cos4 b = 0Þ (cos2 a – cos2 b)2 = 0Þ cos2 a = cos2 bÞ sin2 a = sin2 b

Then, 4 4

2 2cos sincos sin

b b+a a

2 2 2 2

2 2cos cos sin sin

cos sinb a b aÞ +

a aÞ cos2 b + sin2 b = 1

142. (a) sin cos 1sin cos 1

q - q +q + q -

Dividing Numerator and Denominator bycos q

sin cos 1tan 1 seccos cos cos

sin cos 1 tan 1 seccos cos cos

q q- + q - + qq q qÞ Þq q q + - q+ -q q q

2 2(tan sec ) – (sec tan )tan sec 1

q + q q - qÞq - q +

(tan sec )[1 sec tan ]tan sec 1

q + q - q + qÞq - q +

tan secÞ q + q

sin 1cos cos

qÞ +q q

1 sincos+ qÞ

q143. (b) In DABC

tan9ha = ...(1)

In DABD

tan16hb =

yoursmahboob.w

ordpress.com

176 SSC CGL SolvedPaperc

A

h

ba9B C

16

a + b = 90° (given)b = 90 – a

since tan16hb =

16tan(90 ) cot or tan16 16h h

h-a = Þ a = a =

...(2)From eqn. (1) and (2)

216 16 9 12feet.9h h h

h= Þ = ´ Þ =

144. (a) If 2 3sin cosa = a2tan cosa = a ...(1)

Now consider, cot6 a – cot2 a

6 21 1

tan tan= -

a aSince 1cot

tana =

aSubstituting for tan2a with cos a from (1)above equation will be

2 2 2

3 3 31 1 1 cos sin tan 1

cos coscos cos cos- a a a= - = = = =

a aa a a145. (b) (1 + tan q + sec q) (1 + cot q – cosec q)

sin 1 cos 11 1cos cos sin sin

q qæ öæ öÞ + + + -ç ÷ç ÷q q q qè øè ø

sin cos 1 sin cos 1cos sin

q + q + q + q -æ öæ öÞ ç ÷ç ÷q qè øè ø2(sin cos ) 1

sin cosq + q -=

q q

2 2sin cos 2sin cos 1sin cos

q + q + q q -=q q

2sin cos 2sin cos

q q= =q q

146-150. Profit percentage in given years.421991 100 11.67%

360Þ ´ =

241992 100 6.67%360

Þ ´ =

421993 100 11.67%360

Þ ´ =

601994 100 16.67%360

Þ ´ =

541995 100 15%360

Þ ´ =

721996 100 20%360

= ´ =

661997 100 18.33%360

= ´ =

147. (a) x % of 100.01 = 16.67%216 %3

xÞ =

149. (b) Average =11.67 6.67 16.67 15 20 18.33

7+ + + + +

= 14.28% » 15%151. (b) ‘are’ is replaced by ‘is’152. (b) Remove ‘to’ before the policeman153. (b) ‘me’ is replaced by ‘myself’154. (c) ‘in’ is replaced by ‘at’155. (d) No error156. (c) Because157. (a) Healthy158. (a) Inspite of159. (c) Decrepit160. (d) Sporadic161. (b) The meaning of word Vociferous

(Adjective) is :outspoken, blunt.Its synonym should be : Loud.

162. (c) The meaning of word Fictional (Adjective)is :Imaginary, unreal, fabricated, mythicalIts synonym should be : fanciful

163. (d) The meaning of word Trivial (Adjective) is: not important.Its synonym should be : ordinary.

yoursmahboob.w

ordpress.com

177SSC CGL SolvedPaper

164. (c) The meaning of word Impudent (Adjective)is :Insolence.It’s synonyms should be : Insolent.

165. (a) The meaning of word pompous (Adjective)is : self Important.It’s synonym should be : Pretentious.

166. (a) The meaning of word cultivated (Adjective)is : Eductated.It’s antonym should be : Crude.

167. (d) The meaning of word Impertinent(Adjective) is : Ill mannered, disrespectful.It’s antonym should be : courteous.

168. (b) The meaning of word Divulge (verb) is :Reveal, make known.Its antonym should be : Dissemble.

169. (a) The meaning of word Appreciation (Noun)is : Thankfullness.It’s antonym should be : Aspersian.

170. (c) The meaning of word Supple (Adjective) is: Flexible.It’s Antonym should be : Rigid.

171. (c) Idiom washed their hands off means : toabsolve oneself of responsibility or futureblame.

172. (a) Idiom put her foot down means : to tellsomeone in a strong way that they must dosomething or that they must stop doingsomething.

173. (c) Idiom Halcyon days means : a very happyor successful period in the past.

174. (d) Idiom fell on stony ground means : It arequest, a warning, or advice falls on stonyground, people ignore it.

175. (b) Idiom has all his ducks in a row means : toorganize things well.

176. (a) Would have had to attend177. (c) ascent which178. (a) as much as they possibly could179. (b) Alternative should be ‘Loyalty’180. (c) at181. (b) canister182. (c) Domicile183. (a) Irrevocable184. (a) Gratuity185. (c) Wardrobe186. (b) Garrlilous187. (a) Marquee188. (a) Puissant189. (d) Disconcerting190. (d) Exhilarate191. (c) The passage is about the management

courses192. (d) Look at the sentence : The real problem is

that course promoters view developmentas something which primarily, takes placein a class room.

193. (a) Critical194. (d) Unprogressive195. (b) a (fixed) attitude of mind196. (b) self-discipline197. (c) must change himself199. (c) Man well one day die and become dust200. (a) A sum of Individuals

yoursmahboob.w

ordpress.com

178 SSC CGL SolvedPaper

PART-A : GENERAL INTELLIGENCE & REASONING

Directions : In the following, select the related letter/word from the given alternatives.

1. Horse : Neigh : : Bells : ?(a) Beat (b) Chime(c) Rustle (d) Roar

2. A - E : R - V : : B - F : F - J(a) CTDH (b) BSCG(c) FUGK (d) CSCG

3. Which one set of letters when sequentiallyplaced at the gaps in the given letter series shallcomplete it ?a_bba_bba_bb(a) bba (b) aab(c) abb (d) bbb

Directions : In question nos 4 to 12, a series is given,with one term missing. Choose the correct alternativefrom the given ones that will complete the series.

4. EFA, GHC, IJE, ___?___.(a) KDA (b) JKG(c) KLG (d) HIF

5. 13, 10, ___?___, 100, 1003, 1000, 10003.(a) 130 (b) 1030(c) 1130 (d) 103

6. 61, 52, 63, 94, 46, ___?___.(a) 19 (b) 18(c) 17 (d) None

7. 5, 13, 29, 61, 125, ___?___.(a) 145 (b) 253(c) 196 (d) 245

8. YZ, VYZ, SYZ, PYZ, ___?___.(a) XYZ (b) TYZ(c) RYZ (d) MYZ

9. P 3 C, R 5 F, T 8 I, V12 L ___?___.(a) X16O (b) X17M(c) X17O (d) Y17O

10. NP MK RT IG ____?____.(a) VX (b) UW(c) FD (d) EC

11. AZ, CX, EV, GT ___?___, KP, ___?___.(a) IR and MN (b) IR and NM(c) RI and MN (d) RI and NM

12. T Q N K H E B Y V S P ???(a) MJH (b) NKG(c) NKI (d) MJG

SSC Combined Graduate Level (CGL) Solved PaperMORNING SHIFT 21 APRIL 2013

13. Certain numbers have symbols as given below.1 2 3 4 5 6 7 8 9 0

( [ ] ) What is the number indicated by these symbols ?[ ] (a) 4 7 0 9 5 (b) 5 6 9 0 7(c) 4 5 0 9 6 (d) 4 5 9 0 6

Directions: In question nos. 14 to 17, find the oddword/ number from the given alternatives.

14. (a) Franc (b) Pound (sterling)(c) Deutsche Mark (d) Yen

15. (a) Yellow Green (b) Yellow Orange(c) Red Yellow (d) Red Orange

16. (a) fastidious (b) firm(c) grave (d) agreeable

17. 27, 125, 216, 343(a) 27 (b) 125(c) 216 (d) 343

18. What is the least number to be subtracted from2486 to make it a perfect square ?(a) 80 (b) 85(c) 90 (d) 95

19. Shiela and Belah start from their office and walkin opposite direction each travelling 10 kms. Shielathen turns left and walks 10 kms. Belah turns rightand walks 10 km. How far are they now from eachother ?(a) 20 km (b) 10 km(c) 5 km (d) 8 km

20. Roshan is taller than Hardik who is shorter thanSusheel. Niza is taller than Harry but shorter thanHardik. Susheel is shorter than Roshan. Who isthe tallest ?(a) Roshan (b) Susheel(c) Hardik (d) Harry

Directions: In questions no. 21 to 23, from the givenalternatives select the word which cannot be formedusing the letters of the given word.

21. DISAPPOINTMENT(a) POINTER (b) OINTMENT(c) TENEMENT (d) POSITION

22. DECOMPOSITION(a) COMPOSE (b) ECONOMIST(c) POSITION (d) DOCTOR

yoursmahboob.w

ordpress.com

179SSC CGL SolvedPaper

23. INTELLIGENCE(a) NEGLECT (b) GENTLE(c) INCITE (d) CANCEL

24. In the following question, the number of lettersskipped in between adjacent letters in the seriesis successive even numbers. Which of thefollowing series observes this rule ?(a) FINUZ (b) ADGJM(c) BEJQZ (d) BDGKQ

25. If MOBILE is written as ZAMSUM, how TUMORcan be written in that code ?(a) BRAIN (b) HGYAD(c) GGXYA (d) IHZBE

26. In a question paper, there are 12 questions in allout of which only six are to be answered. Sixquestions have an alternative each. Eachquestion has four parts. How many questionsincluding parts are there in the question paper?(a) 24 (b) 48(c) 72 (d) 96

27. If × stands for addition, < for subtraction, + standsfor division, > stands for multiplication, – standsfor equation, ¸ stands for greater than, and = standsfor less than, state which of the following is true ?(a) 3 × 2 < 4 ¸ 16 > 2 + 4(b) 5 > 8 + 4 = 10 < 4 × 8(c) 3 × 4 > 2 – 9 + 3 < 3(d) 5 × 3 < 3 ̧ 8 + 4 × 1

28. If 55 + 66 = 33 and 22 + 99 = 33, what is 44 + 88 ?(a) 33 (b) 36(c) 38 (d) 40

Directions: In questions no. 29 to 34, select themissing number from the given responses.

29. 3 4 65 7 31 2 735 69 ?(a) 82 (b) 94(c) 84 (d) 42

30. 16 49 6425 36 819 13 ?(a) 21 (b) 22(c) 17 (d) 14

31. 19 18 34 32 44 41 2 4 ?(a) 3 (b) 6(c) 9 (d) 4

32. 4 3 236 2 100 7 ? 5(a) 71 (b) 49(c) 64 (d) 81

33. 5, 11, 24, 51, 106, ___?___.(a) 115 (b) 122(c) 217 (d) 221

34. 3917, 3526, ___?___, 2857(a) 3082 (b) 3174(c) 3389 (d) 2682

35. A and B both are walking away from point 'X'. Awalked 3 m and B walked 4 m from it, then Awalked 4 m north of 'X' and B walked 5 m south ofA. What is the distance between them now?(a) 11.40 m (b) 9.5 m(c) 9 m (d) 16 m

36. Pipe A can fill a tank completely in 5 hours.However, on account of a leak at the tank, it takes3 more hours to fill the tank. How long will theleak take to empty the full tank when pipe A isclosed/shut ?(a) 13 hours 20 minutes (b) 7.5 hours(c) 14 hours 40 minutes (d) 12 hours 20 minutes

Directions: In questions no. 37 and 38, one/twostatements are given followed by two/threeconclusions I, II and III. You have to consider thestatements to be true even if they seem to be atvariance from commonly known facts. You have todecide which of the given conclusions, if any, followfrom the given statements.

37. Statement : Sun is the source of light.Conclusions : (I) Moon is not the source of light.

(II) Light has only one source.(a) Only conclusion (I) follows(b) Only conclusion (II) follows(c) Both conclusions (I) and (II) follow(d) Neither conclusions (I) nor (II) follows

38. Statement : (I) All cities are towns.(II) Some cities are villages.

Conclusions : (I) All villages are towns(II) No village is a town(III) Some villages are towns.

(a) Only conclusion (III) follows(b) Only conclusion (I) follows(c) Only conclusion (II) follows(d) None of these

Directions : In questions no. 39 to 41, which answerfigure will complete the question figure ?

39. Question Figure :

yoursmahboob.w

ordpress.com

180 SSC CGL SolvedPaper

Answer Figures :

(a) (b) (c) (d)40. Question Figure :

Answer Figures :

(a) (b) (c) (d)41. Question Figure :

Answer Figures :

(a) (b) (c) (d)42. Which one of the following box can be created

by folding the given key design ?Question Figure :

Answer Figures :

(a) (b) (c) (d)

43. Which figure represents the relationship amongSun, Moon, Molecule ?

(a)

(b)

(c)

(d)

44. Choose from the following diagrams (a), (b), (c)and (d) the one that illustrates the relationshipamong three given classes :North America, United States of America, New YorkAnswer Figures :

(a) (b) (c) (d)45. How many circles are there in the following figure ?

(a) 11 (b) 12(c) 13 (d) 14

46. Three views of the same cube are given. All thefaces of the cube are numbered from 1 to 6. Selectone figure which will result when the cube isunfolded.Question Figure :

?23

5 513 4

3 2

Answer Figures :

41 6523

64 5312

45 6123

53 6412

(a) (b) (c) (d)

yoursmahboob.w

ordpress.com

181SSC CGL SolvedPaper

47. Which answer figure includes all the componentsgiven in the question figure ?Question Figure :

Answer Figures :

(a) (b) (c) (d)48. A piece of paper is folded and cut as shown below

in the question figures. From the given answerfigures, indicate how it will appear when opened.Question Figure :

Answer Figures :

(a) (b) (c) (d)49. If a mirror is placed on the MN, then which of the

answer figures is the right image of the givenfigure ?Question Figure :

M

NAnswer Figures :

(a) (b) (c) (d)

50. A word is represented by only one set of numbersas given in any one of the alternatives. The sets ofnumbers given in the alternatives are representedby two classes of alphabets as in the matrix givenbelow. The columns and rows of Matrix arenumbered from 0 to 6. A letter from the matrix canbe represented first by its row and next by itscolumn, e.g.'A' can be represented by 42, 62, etc.and 'P' can be represented by 15, 43, etc. Similarly,you have to identify the set for the word 'CALM'.

Matrix0 1 2 3 4 5 61 H R E I P S2 S G N D Z I3 B U F T K L4 V A P C Y A5 M W C O X N6 B A E J L O

(a) 44, 62, 65, 51 (b) 53, 42, 65, 36(c) 53, 54, 51, 31 (d) 44, 54, 65, 24

PART-B : GENERAL AWARENESS51. Externality theory is the basic theory of the

following branch of Economics :(a) Macro Economics(b) Environomics(c) Fiscal Economics(d) International Economics

52. Diamonds are priced higher than water because :(a) consumers do not buy them at lower prices.(b) they are sold by selected firms with

monopolistic powers.(c) their marginal utility to buyers is higher than

that of water(d) their total utility to buyers is higher than

that of water53. Transboundary pollution (or) Acid rain is caused

by :(a) Nitrogen oxide and sulphur dioxide(b) Carbon monoxide(c) Carbon dioxide(d) Hydrocarbon

54. "Functional Finance" is associated with :(a) Abba 'P' Lerner (b) Adolph Wogner(c) Adam Smith (d) Adams

55. Of the following land uses, which is restricted toSpecial Economic Zones ?(a) Information Technology Companies(b) Educational Institutions(c) Free trade Centres(d) Marketing Centres

yoursmahboob.w

ordpress.com

182 SSC CGL SolvedPaper

56. Who of the following enjoys the rank of CabinetMinister in Union Cabinet?(a) Deputy Chairman of Planning Commission(b) Judge of Supreme Court(c) Secretary of Government of India(d) Political Advisor to PM

57. Who gave the title of "Mahamana" to MadanMohan Malviya ?(a) Bal Gandadhar Tilak(b) Dada Bhai Naurozi(c) Gopal Krishna Gokhale(d) Mahatma Gandhi

58. The vacancy of the office of the President mustbe filled within :(a) 3 months (b) 6 months(c) 12 months (d) 1 month

59. In which part of the Indian Constitution, thefundamental duties are enshrined ?(a) IV (b) IV A(c) IV B (d) V

60. Who described the Government of India Act, 1935as a new charter of bondage ?(a) B.R. Ambedkar (b) Mahatma Gandhi(c) Rajendra Prasad (d) Pt. Jawaharlal Nehru

61. Who was the Viceroy of the time of Quit IndiaMovement ?(a) Lord Irwin(b) Lord Mountbatten(c) Lord Wavell(d) Lord Lin Lithgow

62. Who was the other Congress leader who joinedwith Motilal Nehru to start the Swaraj Party in1923?(a) G.K. Gokhale (b) B.G. Tilak(c) Chittaranjan Das (d) M.K. Gandhi

63. What is the first sermon of Buddha called as ?(a) Mahaparinivansutta(b) Brahmajalasutta(c) Dhammachakkapabattanasutta(d) Kachchayanagottasutta

64. From which of the following banks did MadanMohan Malaviya take loans for financing "TheHindustan Times"?(a) State Bank of India(b) Punjab National Bank(c) Bank of Maharashtra(d) Bank of Baroda

65. Who wrote the book "Why Socialism"?(a) Jayaprakash Narayan(b) Mahatma Gandhi(c) Acharya Narendra Dev(d) M.N. Roy

66. Multi purpose river valley projects are the "Newtemples of modern India ".(a) Jawaharlal Nehru (b) Motilal Nehru(c) Mahatma Gandhi (d) Rajiv Gandhi

67. Where are the Todas found ?(a) Madhya Pradesh(b) Tamil nadu(c) Rajasthan(d) Aruncachal Pradesh

68. The eastward continuation of the Brazil currentis called :(a) North Atlantic drift(b) South Atlantic drift(c) Counter Equatorial drift(d) West Atlantic drift

69. Which of the following is an endemic species ?(a) Nicobar pigeon (b) Horn bill(c) Indian Rhino (d) Pink head duck

70. The natural disaster in which carbon-di-oxidesuddenly erupts from a deep lake water is knownas __________.(a) Liminic (b) Lacaustrine(c) Fluvial (d) Glacial

71. Blood group was discovered by :(a) Alexander Fleming (b) William Harvey(c) Landsteiner (d) Pavlov

72. Pick out the correct match :(a) Egg yolk Protein and Fat(b) Fleshy foods Calcium and Protein(c) Fish Starch and Vitamin(d) Milk Fibre and Minerals

73. Blood is a :(a) reproductive tissue (b) connective tissue(c) epithelial tissue (d) muscular tissue

74. DPT vaccine is administered to prevent diseaseslike :(a) Diphtheria, Pertussis and Typhoid(b) Diphtheria, Pertussis and Tetanus(c) Dengue, Pertussis and Typhoid(d) Dengue, Polio and Tetanus

75. The disease that has been eradicated from theworld, is :(a) Small pox (b) Leprosy(c) Poliomyelitis (d) Chicken pox

76. In dicots the pollen grains possess :(a) one germ pore (b) two germ pores(c) three germ pores (d) four germ pores

77. 'Farad' is the unit of :(a) Conductnce (b) Capacitance(c) Inductance (d) Resistance

78. Steel is more elasitc than rubber because it :(a) is harder than rubber(b) requires larger deforming force(c) is never deformed(d) is deformed very easily

yoursmahboob.w

ordpress.com

183SSC CGL SolvedPaper

79. Communication satellites are used to :(a) transmit communication signal only(b) receive communication signal only(c) receive and redirect communication signal(d) provide information of natural resources

only80. What type of information system would be

recognised by digital circuits ?(a) Hexadecimal system(b) Binary system(c) Both hexadecimal and binary system(d) Only Roman system

81. Which of the following is an impact printe ?(a) Daisy wheel printer(b) Ink jet printer(c) Bubble jet printer(d) Laser printer

82. Identify the old term amongst the followinggroup :(a) Coaxial cable (b) Optical fibre(c) Twisted pair wire (d) Microwaves

83. Stains of rust on clothes can be removed by :(a) H2O2 (b) Oxalic acid(c) Petrol (d) Alcohol

84. The percentage of nitrogen present in ammoniumsulphate is :(a) 18% (b) 21%(c) 25% (d) 30.5%

85. Ethanol containing 5% water is known as :(a) Absolute alcohol (b) Dilute alcohol(c) Power alcohol (d) Rectified spirit

86. The hormone used as an oral contraceptive is :(a) Aldesterone (b) Cortisone(c) Progesterone (d) Testosterone

87. Expand the term IPCC:(a) International Pollution Control Council(b) International Panel of Climate Control(c) Interim Panel of Climate Change(d) Intergovernmental Panel on Climate Change

88. The 'One Straw Revolution" was written by:(a) Masanobu Fukuoka(b) Richael Carlson(c) M.S. Swaminathan(d) Norman Borlaug

89. Acceptable "Noise Pollution Level" in India rangebetween:(a) 10-15 dec (b) 16-35 dec(c) 40-45 dec (d) 70-100 dec

90. Endosulfan spray on cashew crop resulted in thepollution to the tune of tragedy in :(a) Tamil Nadu (b) Kerala(c) Andhra Pradesh (d) Karnataka

91. Aung San Suu Kyi is a native to :(a) Tibet(b) China(c) Myanmar(d) Arunachal Pradesh

92. India and Japan inked two agreements duringNovember 2012 to enable Tokyo to import fromIndia :(a) Leather goods(b) Rare earth minerals(c) Basmati type rice(d) Rubber based products

93. Non Residential Indians (NRI) Day is marked on:(a) January 7 (b) January 9(c) January 17 (d) January 19

94. "Whether I earned your vote or not, I have listenedto you, I have learned from you. You have mademe a better President", was said by :(a) Pranab Mukherjee (b) Barack Obama(c) George Bush (d) APJ Abdul Kalam

95. First Hindu American to enter the US House ofRepresentative as senator is :(a) Tulsi Gabbard (b) Sunita Williams(c) Jyoti Sengupta (d) Ami bera

96. Who was not a politician ?(a) H.N. Bahuguna (b) I.K. Gujral(c) S.L. Bahuguna (d) J.Jayalalitha

97. The WISE Prize 2012, also known as the "NobelPrize for Education" was conferred by the WISESummit is Doha on :(a) Dr. APJ Abdul Kalam(b) Dr. Madhav Chavan(c) Dr. Yash Pal(d) Dr. M.S. Swaiminathan

98. The most powerful woman in business in Indiaas rated by the "Fortune" for the year 2012 is :(a) Chanda Kochhar(b) Prabha Parameswaran(c) Debjani Ghosh(d) Anjali Bansal

99. Indira Gandhi Prize for Peace, Disarmament andDevelopment 2012 was awarded to :(a) Tereza Fajksova(b) Sunita Williams(c) Chanda Kochhar(d) Ellen Johnson Sirleaf

100. Hypothensmia occurs due to loss of excessiveheat from body due to sudden low bodytemperature in :(a) Snakes (b) Frogs(c) Human beings (d) Lizards

yoursmahboob.w

ordpress.com

184 SSC CGL SolvedPaper

PART- C : QUANTITATIVE APTITUDE

101. If 3 7 2x3 4 3

4 3 4

-æ ö æ ö æ ö=ç ÷ ç ÷ ç ÷è ø è ø è ø , then x is :

(a) 2½ (b) –2(c) 2 (d) 5

102. Number of digits in the square root of 62478078is :(a) 3 (b) 4(c) 5 (d) 6

103. If 10 men or 20 women or 40 children can do apiece of work in 7 months, then 5 men, 5 womenand 5 children together can do half of the work in:(a) 8 months (b) 6 months(c) 4 months (d) 5 months

104. A man undertakes to do a certain work in 150days. He employs 200 men. He finds that only aquarter of the work is done in 50 days. The numberof additional men that should be appointed sothat the whole work will be finished in time is :(a) 50 (b) 75(c) 100 (d) 125

105. A can finish a work in 18 days and B can do thesame work in 15 days. B worked for 10 days andleft the job. In how many days. A alone can finishthe remaining work ?(a) 8 (b) 6(c) 5½ (d) 5

106. The ratio of inradius and circumradius of a squareis :(a) 1 : 2 (b) 1 : 2(c) 2 : 3 (d) 1 : 3

107. The perimeter of the base of a right circular coneis 8 cm. If the height of the cone is 21 cm, then itsvolume is :

(a) 3108 cmp

(b) 108 p cm3

(c) 3112cm

p(d) 112 p cm3

108. A circular road runs around a circular ground. Ifthe difference between the circumferences of theouter circle and the inner circle is 66 metres, the

width of the road is : 22

Take7

æ öp =ç ÷è ø(a) 21 metres (b) 10.5 metres(c) 7 metres (d) 5.25 metres

109. A shopkeeper marks the price of an article at ̀ 80.What will be the selling price, if he allows twosuccessive discounts at 5% each ?(a) ` 7.2 (b) ` 72.2(c) ` 72 (d) ` 85

110. The marked price of a mixie is ` 1600. Theshopkeeper gives successive discount of 10%and x% to the customer. If the customer pays`1224 for the mixie, find the value of x :(a) 8% (b) 10%(c) 12% (d) 15%

111. Which of the following successive discounts isbetter to a customer?(A) 20%, 15%, 10% or(B) 25%, 12%, 8%(a) (A) is better(b) (B) is better(c) (A) or (B) (both are same)(d) None of these

112. Given A is 50% larger than C and B is 25% largethan C, then A is what percent larger than B?(a) 20% (b) 25%(c) 50% (d) 75%

113. Three numbers are in the ratio 1 : 2 : 3. By adding5 to each of them, the new numbers are in theratio 2 : 3 : 4. The numbers are :(a) 5, 10, 15 (b) 10, 20, 30(c) 15, 30, 45 (d) 1, 2, 3

114. Out of 10 teachers of a school, one teacher retiresand in his place, a new teacher of age 25 yearsjoins. As a result, average age of teachers isreduced by 3 years. The age (in years) of theretired teacher is :(a) 50 (b) 58(c) 60 (d) 55

115. The average of 50 numbers is 38. If two numbersnamely 45 and 55 are discarded, the average ofthe remaining numbers is :(a) 36 (b) 35(c) 32.5 (d) 37.5

116. On selling an article for ̀ 170, a shopkeeper loses15%. In order to gain 20%, he must sell that articleat rupees :(a) 210 (b) 215.50(c) 212.50 (d) 240

117. The value of a machine depreciates every yearby 10%. if its present value is `50,000 then thevalue of the machine after 2 years is __________.(a) `40,500 (b) `40,050(c) `45,000 (d) `40,005

yoursmahboob.w

ordpress.com

185SSC CGL SolvedPaper

118. A man can swim 3 km/hr. in still water. If thevelocity of the stream is 2 km/hr. the time takenby him to swim to a place 10 km upstream andback is :

(a)1

8 hr.3

(b)1

9 hr.3

(c) 10 hr. (d) 12 hr.119. A train moving at a rate of 36 km/hr. crosses a

standing man in 10 seconds. It will cross aplatform 55 metres long, in:

(a)1

52

seconds (b) 6 seconds

(c) 7 seconds (d)1152

seconds

120. The time in which ̀ 80,000 amounts to ̀ 92,610 at10% p.a. at compound interest, interest beingcompounded semi annually is :

(a) 3 years (b)1

12

years

(c) 2 years (d)122

years

121. A man buys 3 cows and 18 goats in ` 47,200.Instead if he would have bought 8 cows and 3goats, he had to pay ̀ 53,000 mroe. Cost of onecow is :(a) ` 10,000 (b) ` 11,000(c) ` 12,000 (d) ` 13,000

122. If p – 2q = 4, then the value of p3 – 8q3 – 24pq– 64 is :(a) –1 (b) 2(c) 0 (d) 3

123. If 2x 1

3x 2x 1=

- +, then the value of 3

31

xx

+ is:

(a) 27 (b) 81(c) 110 (d) 125

124. If a2 + b2 + c2 + 3 = (2 (a – b – c), then the value of2a – b + c is :(a) 2 (b) 3(c) 4 (d) 0

125. If x 1 1a a x= - , then the value of x – x2 is :

(a) a (b) – a

(c)1a

(d) –1a

126. If r 1n t n4

æ ö- +ç ÷è ø be a perfect square, then the

values of t are :(a) ±1 (b) ±2(c) 1, 2 (d) 2, 3

127. If 1xx

æ ö+ç ÷è ø = 4, then the value of 4

41

xx

+ is :

(a) 124 (b) 64(c) 194 (d) 81

128. Equation of the straight line parallel to x-axis andalso 3 units below x-axis is :(a) x = 3 (b) x = –3(c) y = 3 (d) y = –3

129. If DABC is similar to DDEF such that BC = 3 cm,EF = 4 cm and area of DABC = 54 cm2, then thearea of DDEF is :(a) 54 cm2 (b) 66 cm2

(c) 78 cm2 (d) 96 cm2

130. A chord AB of a circle C1 of radius ( )3 1+ cmtouches a circle C2 which is concentric to C1. If the

radius of C2 is ( )3 1- cm, the length of AB is:

(a) 4 3 cm (b) 42 3 cm

(c) 8 3 cm (d) 44 3 cm131. In a triangle ABC, AB = AC, ÐBAC = 40°. Then

the external angle at B is :(a) 80° (b) 90°(c) 70° (d) 110°

132. A chord of length 30 cm is at a distance of 8 cm fromthe centre of a circle. The radius of the circle is :(a) 19 (b) 17(c) 23 (d) 21

133. If ABCD be a rectangle and P, Q, R, S be the midpoints of AB,BC,CD and DA respectively,,then the area of the quadrilateral PQRS is equal to:

(a)12

area (ABCD) (b) area (ABCD)

(c)13

area (ABCD) (d)34

area (ABCD)

134. P and Q are two points on a circle with centre atO. R is a point on the minor arc of the circle,between the points P and Q. The tangents to thecircle at the points P and Q meet eahc other at thepoint S. If ÐPSQ = 20°, ÐPRQ = ?(a) 100° (b) 80°(c) 200° (d) 160°

yoursmahboob.w

ordpress.com

186 SSC CGL SolvedPaper

135. AB and CD are two parallel chords of a circlesuch that AB = 10 cm and CD = 24 cm. If thechords are on the opposite sides of the centreand distance between them is 17 cm, then theradius of the circle is:(a) 10 cm (b) 11 cm(c) 12 cm (d) 13 cm

136. ABC is an isosceles triangle such that AB = ACand ÐB = 35°. AD is the median to the base BC.Then ÐBAD is:(a) 55° (b) 70°(c) 35° (d) 110°

137. The angle of elevation of a tower from a distance100m from its foot is 30°. Height of the tower is :

(a) 100 3m (b)100

m3

(c) 50 3m (d)200

m3

138. The value ofcos 1° cos 2° cos 3° ............. cos 177° cos 178°cos 179° is:

(a)12 (b) 0

(c)12

(d) 1

139. The value of (sin2 25° + sin2 65°) is :

(a)23 (b)

32

(c) 1 (d) 0

140. The degree measure of 1 radian (taking p = 227

) is(a) 57°22' 16° (approx.)(b) 57° 61' 22'' (approx.)(c) 57° 16' 22'' (approx.)(d) 57° 22' 16'' (approx.)

141. If sinq + cosec q = 2, then the value of sin9q +cosec9q is :(a) 1 (b) 3(c) 2 (d) 4

142. If sec q + tan q = 2 + 5 , then the value of sin q+ cos q is:

(a)15 (b)

35

(c) 5 (d)75

143. Evaluate : tan 1° tan 2° tan 3° ........... tan 89°.(a) 0 (b) 1(c) –1 (d) 2

Directions: Question no. 144 to 146, the followingpie-chart shows the number of students admitted indifferent faculties of a college. Study the chart andanswer the question.

120°

100°65°

45°

30°

Education

Law

Commerce

Science

Arts

144. If 1000 students are admitted in science, what isthe total number of students ?(a) 360 (b) 180(c) 1800 (d) 3600

145. If 1000 students are admitted in science, what isthe ratio of students in science and arts ?(a) 5 : 6 (b) 6 : 5(c) 7 : 5 (d) 7 : 6

146. How many students are more in commerce thanin law if 1000 students are in science ?(a) 20 (b) 200(c) 2000 (d) 500

Directions : Questions no. 147 to 150, Study the twopie-charts and answer the questions.

21%E

A-EducationB- SavingsC - GroceryD- Electricity and Phone BillsE -Miscellaneous

D 14%

B 18%

April month’s salary : 24000/-`

C

4%

A47%

May month’s salary : 25000/- `

E

2%

B 25%C 14%

D9%

A50%

yoursmahboob.w

ordpress.com

187SSC CGL SolvedPaper

147. The average amount spent on Education, Groceryand Savings from April month's salary is :(a) ` 5520 (b) ` 5800(c) ` 6000 (d) ` 6325

148. From the salary of May, the amount spent onGrocery and Electricity are :(a) ` 2160, ̀ 480 (b) ` 6250, ̀ 3360(c) ` 960, ̀ 5040 (d) ` 3500, ̀ 2250

149. The ratio of amount spent for savings in Aprilmonth's salary and miscellaneous in May month'ssalary is :(a) 235 : 50 (b) 216 : 25(c) 217 : 26 (d) 205 : 13

150. What is the percent increase in Education in Maymonth than April month ?(a) 10.82% (b) 9.56%(c) 12.35% (d) 20%

PART-D : ENGLISH COMPREHENSION

Directions: In question numbers 151 to 155, someparts of the sentences have errors and some arecorrect. Find out which part of a sentence has anerror. If a sentece is free form error, blacken the ovalcorresponding to (d) in the Answer Sheet.

151.You can eat as much as you like/

/(a) (b)

at the newly lunch bar No Error/

(c) (d)

152.The teahcer, as well as the students

/(a)

have gone on an excursion/

(b)

to Ooty during their summer vacation NoError./(c) (d)

153.TheUS don 't want Indian in theSecurityCouncil/ / /

(a) (b) (c)

No Error(d)

154.The cruel lady made herstep daughter todo

/ /(a) (b)

-

all the household chores. No Error/

(c) (d)

155.My sister asked me that how long

/ /(a) (b)

I would stay there. No Error/

(c) (d)

Directions : In question numbers 156 to 160,sentences are given with blanks to be filled with anappropriate word(s).

156. He was assured by his friends _________ everytype of help in an emergency.(a) about (b) by(c) of (d) with

157. He was not a man _________ intelligence.(a) empty of (b) lacking of(c) devoid of (d) absent of

158. The work _________, he left his office.(a) having been over(b) having completed(c) having been completed(d) on being finished

159. Our monthly expenditure _________ by fivehundred rupees when we decided to buy milkfrom the milkman.(a) grew up (b) shot up(c) got up (d) lifted up

160. _________ to people looked well enough, butwhen one looked more closely one saw that theirfaces were filled with despair.(a) At first sight(b) At first looking(c) At first observation(d) On first sight

Directions: In question numbers 161 to 163, out ofthe four alternatives, choose the one which bestexpresses the meaning of the given.

161. Advocate:(a) predict (b) pronounce(c) support (d) determine

162. Preamble :(a) definition (b) mediation(c) conclusion (d) introduction

163. Students are asked to collate for an importantprogramme by the principal.(a) assemble (b) describe(c) narrate (d) prescribe

yoursmahboob.w

ordpress.com

188 SSC CGL SolvedPaper

Directions: In question numbers 164 to 166, choosethe work opposite in meaning to the given .

164. Ambiguous :(a) easy (b) plain(c) clear (d) simple

165. Accolade :(a) permeate (b) blame(c) reticent (d) decorate

166. Wary :(a) rash (b) conscientious(c) daring (d) thrifty

Directions: In question numbers 167 to 171, fouralternatives are given for the Idiom/phraseunderlined. Choose the alternative which bestexpresses the meaning of the Idiom/Phrase and markit in the Answer Sheet.

167. Villagers always call a spade a spade.(a) to speak about spades(b) to speak in a straightforward manner(c) to call someone a spade(d) to speak ill about someone

168. Marty broke a dining-room window and had toface the music when her father got home.(a) accept the punishment(b) listen carefully(c) ask a lot of questions(d) listen to music

169. To play second fiddle(a) to be happy, cheerful and healthy(b) to reduce the importance of one's senior(c) take a subordinate role(d) to do back seat driving

170. Why are you jumping down my throat ? I wasn'teven in the house when it happened.(a) running away(b) making a joke(c) scolding me(d) forcing me to eat

171. I am out of my wits and therefore cannot find away to solve the problem immediately.(a) not intelligent enough(b) greatly confused(c) helpless without power(d) totally ignorant

Directions: In question numbers 172 to 181, asentence/ part of the sentence is underlined. Beloware given alternatives to the underlined sentence/part of the sentence at (a), (b) and (c) which mayimprove the sentence. Choose the correct alternative.In case no improvement is needed your answer is (d).

172. If he had time he will call you(a) would have (b) would have had(c) has (d) No improvement

173. The workers are hell bent at getting what is dueto them.(a) hell bent on getting(b) hell bent for getting(c) hell bent upon getting(d) No improvement

174. We are looking forward to see you tomorrow:(a) looking forward towards seeing(b) looking forward for seeing(c) looking forward to seeing(d) No improvement

175. they could not tell me why did he not eat hislunch.(a) why not had he eaten(b) why he did not eat(c) why had he not eaten(d) No improvement

176. He who will bid the highest will get the product.(a) who bids the highest(b) who the highest bids(c) who would bid the highest(d) No improvement

177. John had told me that he hasn't done it yet.(a) told (b) tells(c) was telling (d) No improvement

178. The clients are waiting outside since morning andwill continue to wait until you meet them.(a) have waited(b) have been waiting(c) were waiting(d) No improvement

179. He may have grown taller when i last saw him(a) from when I last saw him(b) since I last saw him(c) before I last saw him(d) No improvement

180. While crossing the highway, a five year old childwas knocked out by a passing car.(a) away (b) up(c) down (d) No improvement

181. During his long discourse, he did not touch thatpoint.(a) touch upon (b) touch in(c) touch of (d) No improvement

yoursmahboob.w

ordpress.com

189SSC CGL SolvedPaper

Directions : In question numbers 182 to 188, out ofthe four alternatives, choose the one which can besubstituted for the given words/sentence.

182. Detailed plan of a journey :(a) Travelogue (b) tavel kit(c) Schedule (d) itinerary

183. One who cannot be corrected :(a) Incurable (b) Incorrigible(c) Hardened (d) Invulnerable

184. A general pardon granted by the Government topolitical offenders.(a) Pardon (b) Excuse(c) Honesty (d) Amnesty

185. One who hates women:(a) Misogynist (b) Misogamist(c) Ambivert (d) Misanthrope.

186. A person who consumes human flesh :(a) Cannibal (b) Javage(c) Captor (d) Carnivore

187. A school boy who cuts classes frequently is a :(a) Defeatist (b) Sycophant(c) Truant (d) Martlinet

188. Stealing of ideas or writings of someone else.(a) autism (b) scepticism(c) mesmerism (d) plagiarism

Directions: In question numbers 189 and 190, fourwords are given in each question, out of which onlyone word is correctly spelt. Find the correctly speltword and mark your answer in the Answer Sheet.

189. The laboratories are _________ with high techelectronic gadgets.(a) equipped (b) equpped(c) eqiuped (d) equepped

190. Software companies have ushered in _________culture along with advanced technology.(a) foreign (b) foreegn(c) forigen (d) foreign

Directions: In question number 191 to 200, you havetwo passages with 5 questions in each passage. Readthe passages carefully and choose the best answer toeach question out of the four alternatives.

PASSAGE - IThe World health Organisation is briefly called W.H.O.It is a specialesed agency of the United Nations andwas established in 1948.

International helath workers can be seen workingin all kinds of surroundings in deserts, jungles,mountains, coconut groves, and rice fields. They helpthe sick to attain health and the healthy to maintaintheir health.

This global health team assists the local healthworkers in stopping the spread of what are calledcommunicable diseases, like cholera. These diseasescan spread from one country to another and so can bea threat to world health.

W.H.O. assists different national healthauthorities not only in controlling diseases but also inpreventing them altogether. Total prevention ofdiseases is possible in a number so ways. Everyoneknows how people, particularly children, are vaccinatedagainst one disease or another. Similarly, most peopleare familiar with the spraying of houses with poisonoussubstances which kill disease-carrying insects.191. "It is a specialised agency of the United Nations

and was established in 1948". Here specialisedmeans :(a) made suitable for a particular purpose(b) expert(c) extraordinary(d) uncommon

192. "International health workers can be seenworking in all kinds of surroundings: in deserts,jugnles, mountains, coconout groves, and ricefields". Here International means:(a) belonging to the whole world(b) drawn from all countries of the world(c) believing in cooperation among nations(d) belonging to an organisation which has

something to do with different nations.193. They help the sick to attain health and the healthy

to maintain their health. here they stands for:(a) deserts(b) rice fields(c) international health workers(d) jungles

194. "WHO assists different national health aurhotiriesnot only in controlling diseases but also inpreventing them altogether". The above sentenceimplies that :(a) W.H.O. assist many others in addition to

the national health authorities(b) W.H.O. assists more in preventing diseases

than in controlling them.(c) W.H.O. assists in controlling diseases only

if they have not been prevented.(d) W.H.O. assists both in controlling diseases

and in preventing them.195. "Total prevention of diseases is possible in a

number of ways". The author has givenillustrations of :(a) only two such ways(b) only one such way(c) more than two such ways(d) none of these

yoursmahboob.w

ordpress.com

190 SSC CGL SolvedPaper

PASSAGE - IIWho don't I have a telephone? No because I pretendto be wise or pose as unusual. There are two chiefreasons: because I don't really like the telephne, andbecause I find I can still work and play, eat, breathe,and sleep without it. Why don't I like the telephone?because I think it is a pest and time waster. It maycreate unnecessary suspense and anxiety, as whenyou wait for an expected call, that doesn't come; orirritating delay, as when you keep ringing a numberthat is always engaged. As for speaking in a publictelephone booth, it seems to me really horrible. youwould not use it unless you were in a hurry, andbecause you are in a hurry, you will find other peoplewaiting before you. When you do get into the booth,you are half suffocated by the stale, unventilated air,flavored with cheap face powder and chain smoking;and by the time you have began your conversationyour back is chilled by the cold looks of somebodywho is moving about restlessly to make your place.

If you have a telephone in your house, you willadmit that it tends to ring when you least want it toring; when you are asleep, or in the middle of a meal ora conversation, or when you are just going out, orwhen you are in your bath. Are you strong mindedenough to ignore it, to say to yourself." Ah well, it willbe all the same in hundred years time". You are not.You think there may be some important news ormessage for you. Have you never rushed droppingfrom the bath, of chewing from the table, or dazed frombed, only to be told that you are a wrong number? Youwere told the truth. In my opinion all telephonenumbers are wrong numebrs. If, of course, yourtelephone rings and you decide not to answer it, thenyou will have to listen to an idiotic bell ringing andringing in what is supposed to be the privacy of yourown home. You might as well buy a bicycle bell andring it yourself.

196. The author does not have a telephone because :(a) he pretends to be wise(b) he poses as unusual(c) he would prefer to do something else(d) he thinks that it can create unnecessary

suspense and anxiety.197. He hates speaking in a public telephone booth

because :(a) it is costlier(b) he is suffocated by the stale, unventilated

air, flavoured with cheap face power andchain-smoking

(c) others look at him angrily(d) the other side may not know your number

198. ..............your back is chilled by the cold look ofsomebody means:(a) other look at you angrily(b) you feel cold at the back(c) you feel uneasy because the person next in

the queue looks at you restlessly(d) people are very cold.

199. 'Ah well, it will be all the same in hundred yearstime'. This sentence means:(a) Nothing is going to change even if you don't

answer the telephone bell.(b) Things have not changed for the past

100 years.(c) Things will remain the same for 100 years to

come.(d) One should be strong minded.

200. 'All telephone numbers are wrong numbers',because :(a) the author always gets wrong calls(b) whenever he tries it always goes wrong.(c) he doesn't give much importance to

telephone and telepone numbers(d) none of the statements given above.

yoursmahboob.w

ordpress.com

191SSC CGL SolvedPaper

1. (b) The sound made by horse is called Neigh.Similarly, the sound made by a bell is calledchime.

2. (a) As,2

2

2

2

A

R

B

F

+

+

+

+

¾¾®

¾¾®

¾¾®

¾¾®

CTDH

2

2

2

2

E

V

F

J

+

+

+

+

¾¾®

¾¾®

¾¾®

¾¾®3. (d) a b bb / a b bb / a b bb4. (c) The pattern is as follows :

2 2 2

2 2 2

2 2 2

E G I

F H J

A C E

+ + +

+ + +

+ + +

¾¾® ¾¾® ¾¾®

¾¾® ¾¾® ¾¾®

¾¾® ¾¾® ¾¾®

KLG

5. (d) 13 ® 103 has been replaced with zero in the nextterm.103 ® 1001003 ® 100010003.....

6. (b) Reversing number is giving a squarenumber16, 25, 36, 49, 64, 81 and 18

7. (b) The patterns is as follows :5 × 2 + 3 = 1313 × 2 + 3 = 2929 × 2 + 3 = 6161 × 2 + 3 = 125

125 × 2 + 3 = 2538. (d) The patterns is as follows :

YZ VYZ SYZ PYZ MYZ

+3 +3 +39. (c) The pattern is as follows :

2 2 2 2

2 3 4 5

3 3 3 2

P R T V

3 5 8 12

C F I L

+ + + +

+ + + +

+ + + +

¾¾® ¾¾® ¾¾® ¾¾®

¾¾® ¾¾® ¾¾® ¾¾®

¾¾® ¾¾® ¾¾® ¾¾®

X17O

HINTS & SOLUTIONS

10. (a) The pattern is as follows :1 5 9 13

5 9 13 17

N M R I

P K T G

- + - +

- + - -

¾¾® ¾¾® ¾¾® ¾¾¾®

¾¾® ¾¾® ¾¾¾® ¾¾¾®

VX

11. (a) The patterns is as follows :2 2 2 2

2 2 2 2

A C E G

Z X V T

+ + + +

- - - -

¾¾® ¾¾® ¾¾® ¾¾®

¾¾® ¾¾® ¾¾® ¾¾®

IR

2 2

2 2

K

P

+ +

- -

¾¾® ¾¾®

¾¾® ¾¾®

MN

12. (d) 3 3 3 3T Q N K- - - -¾¾® ¾¾® ¾¾® ¾¾®3 3H E B- -¾¾® ¾¾®3 3 3 3B Y V S- - - -¾¾® ¾¾® ¾¾® ¾¾®3 3 3P M J G- - -¾¾® ¾¾® ¾¾®

13. (d) Here,

4 5 9 0 6

[ [

14. (d) Yen is the currency of Japan, an Asiancounty. Pound (Sterling), Deutsche Markand Franc are currencies of Europeancountries.

15. (a) Red, Green and Blue are called primarycolours or basics colours.Red + Green Þ Yellow colour Except inoption (b), in all others there is one primarycolour.

16. (d) Agreeable is different from the other threewords.Agreeable (Adjective) means ‘pleasant’,‘giving pleasure’, ready to agree’.Firm (Adjective) means ‘not yielding whenpressed’, ‘fairly ‘hard’, ‘definite’, ‘not likelyto change’ etc.Grave (Adjective) means ‘serious andimportant’, ‘giving cause for worry’.Fastidious (Adjective) means ‘selectingcarefully’, ‘hard to please’, ‘easilydisgusted’ etc.

yoursmahboob.w

ordpress.com

192 SSC CGL SolvedPaper

17. (c) 27 = 3 × 3 × 3125 = 5 × 5 × 5216 = 6 × 6 × 6343 = 7 × 7 × 7The number 216 is a perfect cube of aneven number.

18. (b) The nearest perfect square less than 2486is 2401.2486 – 85 = 2401 = 49 × 49

19. (d) 20 km

Belah Shiela

10 km

10 km 10 km

10 km

Required distance= 10 + 10 = 20 km

20. (a) Roshan, Susheel > HardikHardik > Niza > HarryRoshan > SusheelRohan > Susheel > Hardik > Niza > HarryTherefore, Roshan is the tallest.

21. (a) There is no ‘R’ letter in the given word.Therefore, the word POINTER cannot beformed.

22. (d) There is no ‘R’ letter in the given word.Therefore, the word DOCTOR cannot beformed.

23. (d) There is no ‘A’ letter in the given word.Therefore, the word CANCEL cannot beformed.

24. (b) Skipped letter are

BC EF HI KLA D G J M

2 2 2 2

¾¾® ¾¾® ¾¾® ¾¾®

CD FGHI KLMNOP RSTUVWXYB E J Q Z

2 4 6 8

¾¾® ¾¾® ¾¾® ¾¾®

C EF HIJ LMNOPB D G K Z

1 2 3 5

¾¾® ¾¾® ¾¾® ¾¾®

GH JKLM OPQRST VWXYF I N U Z

2 4 5 4

¾¾® ¾¾® ¾¾® ¾¾®

25. (c) As,M O B I L E Z A M S U M

+13+12+11+10+9

+8Similarly,T U M O R G G X Y A

+13+12+11+10+9

26. (c) (6 + 12) × 4 Þ 18 × 4 = 7227. (*) xÞ + <Þ - +Þ ¸ >Þ ´

-Þ= ¸Þ> =Þ<

Option (a)3 × 2 < 4 ÷ 16 > 2 + 4Þ 3 + 2 – 4 > 4 > 16 × 2 ÷ 4

Þ 5 – 4 > 16 2

Þ 1 × 8

Option (b)5 > 8 + 4 = 10 < 4 × 8Þ 5 × 8 ÷ 4 < 10 – 4 + 8Þ 5 × 2 < 18 – 4 Þ 10 < 14Option (c)3 × 4 > 2 – 9 + 3 < 3Þ 3 + 4 × 2 = 9 ÷ 3 – 3Þ 3 + 8 ¹ 3 – 3Option (d)5 × 3 < 3 ̧ 8 + 4 × 1Þ 5 + 3 – 3 > 8 ̧ 4 + 1Þ 8 – 3 > 2 +1Þ 5 > 3Both options (b) and (d) are correct.

28. (b) 55 + 65 Þ 5 + 6 = 1111 × 3 = 3322 + 99 Þ 2 + 9 = 1111 × 33 Þ 33Similarly,44 + 88 Þ 4 + 8 = 1212 × 3 = 36

yoursmahboob.w

ordpress.com

193SSC CGL SolvedPaper

29. (b) First Column(3)2 + (5)2 + (1)2

Þ 9 + 25 + 1 = 35Second Column(4)2 + (7)2 + (2)2

Þ 16 + 49 + 4 = 69Third column(6)2 + (3)2 + (7)2

Þ 36 + 9 + 49 = 9430. (c) First Column

16 25+Þ 4 + 5 = 9Second Column

49 36+Þ 7 + 6 = 13Third column

64 81+

8 9 17Þ + =31. (b) 19 – 18 = 1; 1 × 2 = 2

34 – 32 = 2; 2 × 2 = 4

44 – 41 = 3; 3 × 2 = 6

32. (b)4

2(4 + 2)2 = (6)2 = 363

7(3 + 7)2 = (10)2 = 1002

5

(2 + 5)2 = (7)2 = 4933. (c) 5 × 2 + 1 = 11

11 × 2 + 2 = 2424 × 2 + 3 = 5151 × 2 + 4 = 106

106 × 2 + 5 = 217

34. (b) 3917 3526 3174 2857

–391 –352 –317

+39 +35Subtract first 3 digit from no. to get next no.

35. (d)

×

4m

3mA

B 4m

5m

Distance betweenA and B = 4 + 3 + 4 + 5 = 16 m

36. (a) Pipe A can fill a tank completely in 5 hours.On account of a leak at the tank, it takes5 + 3 = 8 hours to fill the tank. Time takenby the leak to empty the full tank

= 5 8 408 5 3´

=-

= 13 hours 20 minutes37. (d) Neither Conclusion I nor II follows. Sun is

the source of light . It does not imply thatlight has only one source.

38. (a)

Cities

Towns

Village

Conclusion III follows.

39. (b)

40. (c) 41. (c)42. (d) The shaded parts are narow. So Answer

Figure (b) is invalid.The white part is larger. So Answer Figure(c) is invalid.If dot is on the top surface, then the visiblesurface can not be white. So answer figure(d) is invalid.

yoursmahboob.w

ordpress.com

194 SSC CGL SolvedPaper

43. (d) Every thing is composed of molecules. Sunis different from Moon.

Molecules

Sun Moon

44. (b) New York is a city of North America. NorthAmerica comes under United States ofAmerica.

USANA

NY

45. (c) There are altogether 13 circles.

46. (a) If we fold the option (b) the number 2 willlie opposite 5.If we fold the option (c) the number 1 willlie opposite 3.If we fold the option (d) the number 2 willlie opposite 5.Therefore, Answer Figure (4) is correct.

48. (c)

47. (a)

49. (c)

50. (a) By matching code42, 62, 85, 51 Letters Resembles to CALMin the MATRI X.

51. (d) In economics, an externality is the cost orbenefit that affects a party who did notchoose to incur that cost or benefit.Economists often urge governments toadopt policies that "internalize" anexternality, so that costs and benefits willaffect mainly parties who choose to incurthem.

52. (c)53. (a) Transboundary pollution is the pollution

that originates in one country but is able tocause damage in another country'senvironment, by crossing borders throughpathways like water or air. Acid rain is aclassic example of a transboundary pollutionbecause it can be blown anywhere by thewind. Sulphur dioxide and nitrous oxides arethe two main chemicals that react with waterto make acid rain. The chemicals arecommonly released from power stations,factories and transport.

54. (a) Functional finance is an economic theoryproposed by Abba P. Lerner, based oneffective demand principles and chartalism.It states that government should financeitself to meet explicit goals, such as tamingthe business cycle, achieving fullemployment, ensuring growth, and lowinflation.

55. (c) 56. (a)57. (d) Madan Mohan Malaviya was an Indian

educationist and politician notable for hisrole in the Indian independence movementand as the two time president of IndianNational Congress. He was respectfullyaddressed as Pandit Madan Mohan Malaviyaand also addressed as 'Mahamana' byMahatama Gandhi.

58. (b) 59. (b) 60. (d)61. (d) Lord Linlithgow was Viceroy of India from

1936 to 1944 and this eight years period waslongest reign as Viceroy of India.

62. (c)63. (c) The Dhammacakkappavattana Sutta is

considered to be a record of the first teachinggiven by Gautama Buddha after he attainedenlightenment. The main topic of this sutrais the Four Noble Truths, which are the

yoursmahboob.w

ordpress.com

195SSC CGL SolvedPaper

central teachings of Buddhism that providea unifying theme, or conceptual framework,for all of Buddhist thought.

64. (b) 65. (a) 66. (a) 67. (b)68. (b) South Atlantic Current is an eastward

ocean current, fed by the Brazil Current. Thatfraction of it which reaches theAfrican coastfeeds the Benguela Current. It is continuouswith the northern edge of the AntarcticCircumpolar Current.

69. (a) The Nicobar pigeon is a pigeon found onsmall islands and in coastal regions from theNicobar Islands, India, east through theMalay Archipelago, to the Solomons andPalau. It is the only living member of thegenus Caloenas and the closest livingrelative of the extinct dodo..

70. (a) A limnic eruption, also referred to as a lakeoverturn, is a rare type of natural disaster inwhich dissolved carbon dioxide (CO2)suddenly erupts from deep lake waters,forming a gas cloud that can suffocatewildlife, livestock and humans.

71. (c) The inventor of blood group is KarlLandsteiner, was born in Vienna, June 14,1868.He is a scientist in the field of bio-treatment.He was awarded Nobel Prize for Medicineand Fisiologi field in 1930. This is becausethe classification of all types of blood intofour namely; A, B, AB, and O in the year1909.

72. (a) 73. (b)74. (b) DPT refers to a class of combination

vaccines against three infectious diseasesin humans: diphtheria, pertussis (whoopingcough), and tetanus. The vaccinecomponents include diphtheria and tetanustoxoidsand kills whole cells of the organismthat cause pertussis (wP).

75. (a) 76. (c) 77. (b)78. (b) 79. (c) 80. (b)81. (a) Daisy wheel printing is an impact printing

technology invented in 1969 by DavidS. Lee at Diablo Data Systems. It usesinterchangeable pre-formed type elements,each with typically 96 glyphs, to generatehigh-quality output comparable to premiumtypewriters such as the IBM Selectric, buttwo to three times faster.

82. (d)83. (b) The oxalic acid is an ideal chemical for

cleaning purposes. Its bleach-like qualitiesmake it perfect for sterilizing householditems. It is also efficient in removing rust onvarious different surfaces. Stains oncounters, bathtubs and kitchen sinks canbe removed through careful application ofthis chemical.

84. (b)85. (d) Rectified spirit, also known as neutral

spirits, rectified alcohol, or ethyl alcohol ofagricultural origin is highly concentratedethanol which has been purified by meansof repeated distillation, a process that iscalled rectification. It is 95.5% alcohol and4.5% water. It is treated with CaO to formlime of alcohol, which undergoes fractionaldistillation to give ethanol.

86. (c)87. (d) The Intergovernmental Panel on Climate

Change (IPCC) is a scientific andintergovernmental body under the auspicesof the United Nations, set up at the requestof member governments, dedicated to thetask of providing the world with an objective,scientific view of climate change and itspolitical and economic impacts.

88. (a) 89. (c) 90. (b)91. (c) Aung San Suu Kyi, also called Daw Aung

San Suu Kyi (born June 19, 1945, Rangoon,Burma [now Yangon, Myanmar]) politicianand opposition leader of Myanmar, daughterof Aung San (a martyred national hero ofindependent Burma) and Khin Kyi (aprominent Burmese diplomat), and winnerof the Nobel Prize for Peace in 1991. She hasheld multiple governmental posts since 2016.

92. (b) 93. (b)94. (b) Barack Hussein Obama II who is the 44th

and current President of the United States.He is the first African American to hold theoffice and the first president born outsidethe continental United States.

95. (a)96. (c) Sunderlal Bahuguna (born 9 January 1927)

is a noted Garhwali environmentalist, Chipkomovement leader and a follower of MahatmaGandhi's philosophy of Non-violence andSatyagraha.He was awarded the Padma Vibhushan,India's second highest civilian honour, on26 January 2009.

yoursmahboob.w

ordpress.com

196 SSC CGL SolvedPaper

97. (b) 98. (a)99. (d) Ellen Johnson Sirleaf, née Ellen Johnson

(born October 29, 1938,Monrovia, Liberia)Liberian politician and economist, who waspresident of Liberia from 2006. She was thefirst woman to be elected head of state of anAfrican country. Johnson Sirleaf was one ofthree recipients, along with Leymah Gboweeand Tawakkul Karman, of the 2011 NobelPrize for Peace for their efforts to furtherwomen's rights.

100. (c) Hypothermia is a medical emergency thatoccurs when your body loses heat fasterthan i t can produce heat, causing adangerously low body temperature. Normalbody temperature is around 98.6 F (37 C).Hypothermia occurs as your bodytemperature passes below 95 F (35 C).

101. (d)3 7 2x3 4 3

4 3 4

-æ ö æ ö æ ö´ =ç ÷ ç ÷ ç ÷è ø è ø è ø

Þ 3 7 2x3 3 3

4 4 4æ ö æ ö æ ö´ =ç ÷ ç ÷ ç ÷è ø è ø è ø

Þ 10 2x3 3

4 4æ ö æ ö=ç ÷ ç ÷è ø è ø

Þ 2x = 10 Þ x = 5102. (b) When no. of digit in a no. is 7 or 8 then in

square root will be 4103. (c) 10 men = 20 women = 40 children

i.e. 1m = 2w = 4 m\ Sm + Sw + Sch

Þ 5 × 4 + 5 × 2 + 5 = 35

2 81 1

1 2

M DM DW W

=

235 D40 7112

´´=

40 735 2´´

= D2

D2 = 4 months.

104. (c) 200 men do 14

work in 50 days.

\ 1 1 2 2

1 2

M D M DW W

=

Þ 2M 100200 50

1 34 4

´´=

Þ M2 × 100= 200 × 50 × 3Þ M2 = 300\ Additional men = 100

105. (b) Work done by B in 10 days = 10 215 3

=

Remaining work = 1 – 2 13 3

=

\ Time taken by A

= 1 183´ = 6 days.

106. (b)

Radius of circum-circle

= Diagonal 2 Side Side

2 2 2´= =

Radius of in-circle = Side

2

\ Rate = Side

2:

Side2 = 1: 2

107. (c) 2pr = 8 Þ pr = 4

Þ r = 4p

\ V = 21 1 4 4r h 213 3

´p = p ´ ´p ´ p

= 112p

cu.cm.

yoursmahboob.w

ordpress.com

197SSC CGL SolvedPaper

108.B

A

O

r2

r1

Breadth of road = r2 – r1C2 – C1 = 66\ 2pr2 = 2pr1 = 66Þ 2p (r2 – r1) = 66

Þ r2 – r1 = 66 66 72 2 22

´=p ´

= 10.5 metre

109. (b) Net discount = 25

5 5 %100

æ ö+ -ç ÷è ø

= 3 39

9 %4 4

=

\ S.P. = 80 × 361400

= ̀ 72.2

110. (d) First discount

= 1600 10

100´

= ̀ 160

Price after it = 1600 – 160= ̀ 1440

\ 1440 x

100´

= 1440 – 1224 = 216

\ x = 216 100

1440´

= 15%

111. (b) (a) Net discount for 20% and 15%

= 20 15

20 15 %100´æ ö+ -ç ÷è ø = 32%

Net discount for 32% and 10%

= 32 10

32 10100´æ ö+ -ç ÷è ø = 38.8%

(b) Net discount for 25% and 12%

= 25 12

25 12100´æ ö+ -ç ÷è ø = 34%

Net discount for 34% and 8%

= 34 8

34 8 %100´æ ö+ -ç ÷è ø

= 42 – 272 = 39.28%112. (a) C = 100

A = 150B = 125A is larger than C by

= 150 125 100

125- ´ = 20%

113. (a) Number = x,2x and 3x

\ x 5 2

2x 5 3+ =+

Þ 4x +10Þ 3x +15Þ x =5Þ Number = 5, 10 and 15,

114. (c) Age of retired teacher = 25 + 3 × 10 = 55 years115. (d) New average

= 38 50 45 55

48´ - -

= 1800

48 = 37.5

116. (d) C.P. of article = 170 100

85´

= ̀ 200

\ Required S.P. = 200 120

100´

= ̀ 240

117. (a) Depriciated value

= 50000 2101

100æ ö-ç ÷è ø

= 50000 × 9 9100´

= ̀ 40500

118. (d) Downstream speed = 5 kmphUpstream speed = 1 kmph\ Required time

= 10 105 1

+ = 12 hours

119. (d) Speeds of train = 36 kmph

= 36 × 5

18 = 10 m/sec

yoursmahboob.w

ordpress.com

198 SSC CGL SolvedPaper

Length of train = 10 × 10 = 100 metre

\ Required time = 100 55

10+

= 15.5 seconds Þ 1152

seconds.

120. (b) Time = t half year= R = 5% per half year

\ A = P TR1

100æ ö+ç ÷è ø

Þ T92610 51

80000 100æ ö= +ç ÷è ø

Þ T9261 21

8000 20æ ö= ç ÷è ø

Þ T = 3 half years

Þ 3 T21 21

20 20æ ö æ ö=ç ÷ ç ÷è ø è ø

\ T = 3 or 112

year

121. (c) C.P of 1 cow = ` xC.P of a goat = ` y3x + 8y = 47200 ...(i)Þ 8x + 3y = 100200 ...(ii)By equation (i) × 3 – (ii) × 8, 9x + 24y – 64x– 24y= 141600 – 801600Þ 55x = 660000

x = 660000

55 = ̀ 12000

122. (c) p – 21 = 4cubing both sides,( p – 2q)3 = 64Þ p3 – 8q3 + 3p. 4q2 – 3p2 . 2q = 64Þ p3 – 8q3 + 12pq2 – 6p2q = 64Þ p3 – 8q3 – 6pq (p – 2q) = 64Þ p3 – 8q3 – 6pq × 4 = 64Þ p3 – 8q3 – 24pq – 64 = 0

123. (c) 2x 1

3x 2x 1=

- +

Þ 2x 2x 1

x- + = 3

Þ 1x 2x

- + = 3

Þ 1

xx

+ = 5

On cubing both sides3

31 1

x 3 xxx

æ ö+ + +ç ÷è ø = 125

Þ 3

31

xx

+ = 125 – 3 × 5 = 110

124. (a) a2 + b2 + c2 + 3= 2a – 2b – 2cÞ a2 – 2a + 1 + b2 + 2b + 1 + c2 * + 2c + 1 = 0Þ (a – 1)2 + (b + 1)2 + (c + 1)2 = 0\ a – 1 = 0 Þ a = 1b + 1 = 0 Þ b = –1c + 1 = 0 Þ c = –1\ 2a – b + c = 2 + 1 – 1 = 2

125. (a)x 1 1a a x= -

Þ x x aa ax

-=

Þ x2 = x – a Þ x – x2 = a

126. (a) For nr – tn + 14

to be a perfect square,

r = 2 and t = ± 1Look :

n2 – n + 14

= n2 – 2.n. 1 12 4

+ = 21n

2æ ö-ç ÷è ø

2 21 1 1n n n 2.n.

4 2 4+ + = + +

= 21

n2

æ ö+ç ÷è ø

127. (c)1

xx

æ ö+ç ÷è ø = 4

On squaring both sides

22

1x 2

x+ + = 16

Þ x2 + 21

x = 14

On squaring again4

41

x 2x

+ + = 196

Þ 4

41

xx

+ = 194

yoursmahboob.w

ordpress.com

199SSC CGL SolvedPaper

128. (d)Y

X

Q

X¢ O

P

\ Equation is : y = –3129. (d) A

B C E F

D

DABC ~ DDEF

\ 2

2ABC 3 54 9DEF DEF 164D = Þ =D D

Þ DDEF = 16 54

= 96 sq. cm.

130. (d)A

C

B

O

OC = 3 1-OA = 3 1+

AC = ( ) ( )2 23 1 3 1+ - -

= 44 3 2 3=

\ AB = 44. 3 cm

131. (d) A

D B C

ÐABC = ÐACBÐBAC = 40°\ ÐABC + ÐACB = 140°\ ÐABC = 70°\ ÐABD = 180° – 70° = 110°

132. (b)

O

A D B

AD = 15 cmOD = 8 cm

OA = 2 215 8+

= 225 64 289+ == 17 cm

133. (a)A S D

B Q C

P R

ar D PSR = 12

APRD ....(i)

as D PQR = 12

PBCR ....(ii)

Adding Both eq.

ar D PSRQ = 12

ABCD

yoursmahboob.w

ordpress.com

200 SSC CGL SolvedPaper

134. (a)

T OR S

P

Q

ÐOPS = ÐOQS = 90°ÐPSQ = 20°;\ ÐPOQ = 160°\ ÐPTQ = 80°PRQT is a concyclic quadrilateral.\ ÐPRQ = 180° – 80° = 100°

135. (d) A D

E O F

B C

AB = 10 cm, AE = 5 cmOE = xCD = 24 cm, DF = 12 cmOF = 17 – xOA = ODÞ 52 + x2 = 122 + (17 – x)2

Þ 25 + x2 = 144 + 289 – 34x + x2

Þ 34x = 408

Þ x = 40834

= 12

\ OA = 2 25 12+ = 13 cm

136. (a) A

B D C

35°

AB = AC\ ÐABC = ÐACB = 35°\ ÐADB = 90°\ ÐBAD = 55°

137. (b)A

B C30°

100 metreAB = h metreÐACB = 30°;BC = 100 metre

\ tan 30° = ABBC

Þ 1 h

1003=

Þ h = 100

3 metre

138. (b) cos 90° = 0\ cos 1°, cos 2° .... cos 179° = 0

139. (c) sin2 25° + sin2 65°= sin2 25° + sin2 (90° – 25°)= sin2 25° + cos2 25° = 1

140. (c) p = radian = 180°

\ 1 radian = 180°p

= 180 7 630 357

22 11 11´ ° = = °

= 3 180 '

57 60 ' 5711 11° ´ = °

= 4

57 16' 60 ' 57 16 '22 ''11

° ´ = °

141. (c) sinq + cosecq = 2

if x + 1x

= 2 then xn + n1

x = 2

\ sinq + 1

sinq = 2

\ sin9q + 91

sin q = 2

yoursmahboob.w

ordpress.com

201SSC CGL SolvedPaper

142. (b) sec q + tan q = 2 + 5

\ secq – tanq = 1

5 2+

= 5 2 5 2

( 5 2)( 5 2)- = -

+ -On adding,2 secq = 2 + 5 + 5 – 2 = 2 5

Þ secq = 5 Þ cos q = 15

On subtracting,2 tan q = 2 + 5 – 5 + 2 =– 4Þ tan q = 2

\ tan 2

sinsec 5q= q =q

\ 2 1

sin cos5 5

q + q = +

= 35

143. (b) tan 89° = tan (90° – 1°) = cot 1°tan 88° = tan (90° – 2°)= cot 2°\ Expression = tan 1°, cot 1°, tan 2°,cot2°.... tan 45° = 1

[ tan .cot 1]q q =Q

144. (d) Q 100° º 1000

\ 360° º 1000

360100

´ = 3600

145. (a) Required ratio= 100 : 120 = 5 : 6

146. (b) Difference between the angles of studentsof commerce and scienceQ 100° = 1000\ 1° = 10\ 20° = 200

147. (a) Required average

= 1 24000 (47 4 18)%3´ ´ + +

= 1 24000 693 100

´´ = ̀ 5520

148. (d) Expenditure on grocery

= 25000 14

100´

= ̀ 3500

Expenditure on electricty = 25000 9

100´

= ̀ 2250149. (b) Required ratio

= 24000 18 25000 2

:100 100´ ´

= 24 × 18 : 25 × 2 = 216 : 25150. (a) Expenditure on education in April

= 24000 × 47

100= ̀ 11280Expenditure on education in May

= 25000 50

100´

= ̀ 12500

Percentage increase

= 12500 11280 100

11280- ´

= 10.82%151. (d) 152. (b) 153. (b) 154. (b)155. (b) 156. (c) 157. (c) 158. (b)159. (a) 160. (a) 161. (c) 162. (d)163. (a) 164. (c) 165. (b) 166. (a)167. (b) 168. (a) 169. (c) 170 (c)171. (b) 172. (a) 173. (c) 174. (c)175. (b) 176. (a) 177. (b) 178. (b)179. (b) 180. (c) 181. (c) 182. (c)183. (a) 184. (a) 185. (d) 186. (a)187. (c) 188. (d) 189. (a) 190. (d)191. (a) 192. (a) 193. (c) 194. (d)195. (a) 196. (d) 197. (b) 198. (c)199. (d) 200. (d)

yoursmahboob.w

ordpress.com

202 SSC CGL SolvedPaper

PART-A : GENERAL INTELLIGENCE & REASONING

Directions (Qs. 1-3) : In questions nos. 1 and 2, whichof the following interchange of signs would make thegiven equation correct?1. 10 + 10 ̧ 10 – 10 × 10 = 10

(a) + and – (b) + and ¸(c) + and × (d) ¸ and +

2. (8 – 8) + 8 × 32 = 64(a) ×, +, – (b) –, ̧ , +(c) +, ̧ , + (d) +, ̧ ×

3. If ‘R’ stands for ‘–’, ‘A’ stands for ‘+’, ‘B’ standsfor ‘¸’ and ‘C’ stands for ‘×’, then what is the valueof the given equation? (BODMAS rule will not beapplicable)25 A 37 C 2 B 4 R 1 = ?(a) 32 (b) 35(c) 30 (d) 27

Directions (Qs. 4-9) : In each of the following questions,select the related letters/word/number from the givenalternatives.4. Chisel : Sculptor :: Harrow?

(a) Gardener (b) Mason(c) Blacksmith (d) Guard

5. Moon : Satellite :: Earth?(a) Sun (b) Planet(c) Solar system (d) Round

6. BJCI : JBIC :: CXDW : ?(a) JCDU (b) BCJU(c) EVFU (d) XCWD

7. AB : NO :: LM : ?(a) OL (b) KP(c) PK (d) YZ

8. AG : IO :: EK : ?(a) LR (b) MS(c) PV (d) SY

9. 25 : 175 :: 32 : ?(a) 150 (b) 170(c) 162 (d) 160

Directions (Qs. 10-15) : In question nos. 10 to 15,select the one which is different from the other threeresponses.10. (a) Shimla (b) Darjeeling

(c) Ooty (d) Agra

11. (a) Foal (b) Hen(c) Lamb (d) Leveret

12. (a) BADC (b) XWZY(c) VUST (d) NMPO

13. (a) DCFG (b) FEHI(c) JILM (d) HGJL

14. (a) (1, 0) (b) (2, 3)(c) (3, 8) (d) (4, 27)

15. (a) (96, 24) (b) (39, 18)(c) (81, 54) (d) (82, 64)

16. Which one of the given responses would be ameaningful order of the following words inascending order?1. Accommodation 2. Perception3. Scheme formation 4. Assimilation5. Sensation(a) 1, 2, 3, 5, 4 (b) 5, 2, 3, 4, 1(c) 5, 1, 4, 2, 3 (d) 5, 2, 4, 3, 1

17. Which of the given responses would be ameaningful order of the following starting fromthe inner layer?1. Hydrosphere 2. Atmosphere3. Biosphere 4. Lithosphere(a) 4, 1, 3, 2 (b) 2, 4, 3, 1(c) 3, 1, 2, 4 (d) 1, 2, 3, 4

Directions (Qs. 21-24) : In each of the followingquestions, a series is given with one term missing.Choose the correct alternative from the given onesthat will complete the series.

18. EJO, FKP, GLQ, HMR, ?(a) ABC (b) DEF(c) MNO (d) INS

19. B O C N D M E ? ?(a) L F (b) O P(c) K L (d) E F

20. 7, 2 = 59; 5, 3 = 28; 9, 1 = 810; 2, 1 = 13; 5, 4 = ?(a) 19 (b) 9(c) 20 (d) 239

21. 120, 440, 960, 1680, ?(a) 2600 (b) 3240(c) 3040 (d) 2400

SSC Combined Graduate Level (CGL) Solved PaperEVENING SHIFT 21 APRIL 2013

yoursmahboob.w

ordpress.com

203SSC CGL SolvedPaper

Directions (Qs. 9-14) : In each of the followingquestions, select the missing number from the givenresponses.

22. 81 64 164 9 49

36 16 25108 96 ?(a) 230 (b) 140(c) 120 (d) 410

23. 25 5 530 5 635 ? 5(a) 5 (b) 4 (c) 6 (d) 7

24. 24 51 672 4 65 7 553 211 ?(a) 135 (b) 235(c) 347 (d) 407

25. G is fatter than H but not as fat as M. Q is alsonot as fat as M. Who is the most lean person inthe group?(a) Q (b) H (c) G (d) M

26. A man walks 15 metres south. Then turning tohis right he walks 15 metres. Then turning to hisleft, he walks 10 meters. Again turns to his leftand walks 15 metres. How far is he from his initialposition?(a) 10 m (b) 25 m(c) 15 m (d) 60 m

27. There are five buses M, N, O, P, Q in a row on aroad. Bus M is standing at the front and Q isstanding at the back end. Bus N stands betweenM and O. Bus P stands between O and Q. Whichbus is in the middle of the five?(a) M (b) P(c) N (d) Q

Directions: In question nos. 28 to 30, from the givenalternative words, select the word which cannot beformed using the letters of the given words.28. DISTANCE

(a) DANCE (b) STAND(c) SANE (d) TEASE

29. RESURRECTIONIST(a) TOURIST (b) NOISE(c) SORCERER (d) TENDER

30. COMMERCIALISM(a) CROME (b) LANCER(c) MISER (d) OSCAR

31. If DIVINE is coded as AFSFKB, thenPOWERFUL is coded as(a) XLHOJVIM (b) MLTBDCRI(c) MLWBOCRI (d) HLTBNCRI

32. If NOTE is written as PQVG, then TIME is written as(a) VQOG (b) VKOG(c) VOKG (c) VGKO

33. If SMART is coded as UKCPV, then WONDERis coded as(a) YMPPRT (b) YMPBGP(c) YMPBFP (d) YMBPPG

34. If ‘+’ stands for ‘multiplication’, ‘<’ stands for‘division’, ‘¸’ stands for ‘subtraction’, ‘–’ standsfor ‘addition’ and ‘×’ stands for ‘greater than’,identify which expression is correct.(a) 20 – 4 ̧ 4 + 8 < 2 × 26(b) 20 × 8 + 15 < 5 ̧ 9 – 8(c) 20 < 2 + 10 ̧ 4 – 6 × 100(d) 20 < 5 + 25 ̧ 10 – 2 × 96

35. One day, Nita left home and cycled 10 kmsouthwards, turned right and cycled 5 km andturned right and cycled 10 km and turned leftand cycled 10 km. How many kilometers will shehave to cycle to reach her home straight?(a) 10 km (b) 15 km(c) 20 km (d) 25 km

36. Rajat moves from his office to the canteenstraight at a distance of 12 meters. Then he turnedleft and walked for 2 metres. Then he turns leftagain and walks straight for 12 metres. How faris he from his office?(a) 10 metres (b) 12 metres(c) 8 metres (d) 2 metres

Directions: In question nos. 37 and 38, two/fourstatements are given followed by two/fourconclusions I, II, III and IV. You have to consider thestatements to be true even it they seem to be atvariance from commonly known facts. You have todecide which of the given conclusions, if any, followfrom the given statements.37. Statements:

I. All apples are bananas.II. All bananas are sweet.Conclusions:I. Some apples are sweet.II. Some bananas are apples.(a) Conclusion I follows.(b) Conclusion II follows.(c) Either conclusion I or II follows.(d) Both conclusions I and II follow.

yoursmahboob.w

ordpress.com

204 SSC CGL SolvedPaper

38. Statements:1. All metals are silver.2. All silver are diamond.3. Some diamonds are gold.4. Some gold are marbles.Conclusions:I. Some gold are metals.II. All metals are diamond.III. Some silver are marble.IV. Some gold are silver(a) Only conclusion I follows.(b) Only conclusion II follows.(c) Only conclusion III follows.(d) Only conclusions IV follows.

39. Which of the following cubes can be created byfolding the given figure?Question Figure.

ABC D

EF

Answer Figures.

B E

FE D

FB CF

ED

A

(a) (b) (c) (d)

40. Two positions of a dice are given. Which numberwould be at the top when bottom is 2?

2

3

5 6 13

(a) 4 (b) 1(c) 5 (d) 6

41. Identify the response figure from which thequestion figure’s pieces have been cut.Question figure

Answer Figures.

(a) (b) (c) (d)

42.

12 3

45

6

78 9

1011

12

In the above figure, the circle stands foremployed, the square stands for social worker,the triangle stands for illiterate and the rectanglestands for truthful. Employed, truthful andilliterate social workers are indicated by whichregion?(a) 5 (b) 4(c) 2 (d) 1

43. Find the missing number.

4

2

527

3

37

3

25

6

5 2

9

?

2

(a) 37 (b) 45(c) 47 (d) 57

44.

34

568

10

13

In the above diagram, parallelogram representswomen, triangle represent the sub-inspectors ofpolice and circle represents the graduates. Whichnumbered area represents women graduate sub-inspectors of police?(a) 5 (b) 3(c) 8 (d) 13

yoursmahboob.w

ordpress.com

205SSC CGL SolvedPaper

Directions: In question nos. 45 and 46, which answerfigure will complete the pattern in the question figure?45. Question figure

Answer figures

(a) (b) (c) (d)46. Question figure

Answer figures

(a) (b) (c) (d)47. From the given answer figures, select the one in

which the question figure is hidden/embedded.Question figure

Answer Figures

(a) (b) (c) (d)48. A piece of paper is folded and punched as shown

below in the question figures. From the givenanswer figures, indicate how it will appear whenopened.Question figure

Answer Figures

(a) (b) (c) (d)

49. Which of the answer figures is exactly the mirrorimage of the given pattern of numbers when themirror is held at MN?Question figure

2 4 63 5

7

M

NAnswer Figures

(a) (b) (c) (d)

6 4

5

2

377

750. A word is represented by only one set of numbers

as given in any one of the alternatives. The setsof numbers given in the alternatives arerepresented by two classes of alphabets as intwo matrices given below. Two columns and rowsof Matrix I are numbered-from 0 to 4 and that ofMatrix II are numbered from 5 to 9. A letter fromthese matrices can be represented first by itsrow and next by its column, e.g., ‘B’ can berepresented by 67, 75 etc. Similarly, you have toidentify the set for the word ‘CARD’.

MATRIX I MATRIX II

0 1 2 3 40 A B C D E1 D C B A E2 B A D C E3 D B C A E4 C D A E B

5 6 7 8 95 P Q R S T6 Q S P R T7 P T R S Q8 Q S P R T9 T P S Q R

(a) 32, 00, 56, 10 (b) 40, 21, 68, 44(c) 11, 33, 57, 22 (d) 02, 42, 77, 20

yoursmahboob.w

ordpress.com

206 SSC CGL SolvedPaper

PART-B : GENERAL AWARENESS

51. Solids which conduct electricity at highertemperature but not at lower temperature arecalled(a) super-conductor(d) metallic-conductor(c) semi-conductor(d) insulator

52. Which one of the following has greatest mass?(a) electron(d) proton(c) neutron(d) hydrogen nucleus

53. A television channel is characterised by(a) frequency of transmitted signal(b) velocity of transmitted signal(c) physical dimension of television screen(d) size of picture tube

54. The first computer mouse was built by(a) Douglas Engelbart (d) William English(c) Oaniel Coogher (d) Robert Zawacki

55. An organization’s profitability depends on(a) Quality of data processed(b) Quantity of data processed(c) Speed of processing the data(d) Both (a) and (c)

56. The density of water is 1 g/cc. This is strictlyvalid at(a) 0°C (b) 4°C(c) 25°C (d) 100°C

57. The process of photosynthesis involvesconversion of(a) chemical energy into radiant energy(b) chemical energy into mechanical energy(c) solar energy into chemical energy(d) mechanical energy into solar energy

58. A colloidal system in which a liquid is dispersedin a liquid is called.(a) gel (b) emulsion(c) sol (d) precipitate

59. The antiseptic compound present in dettol is(a) Iodine (b) Enloroxylenol(c) Biothional (d) Cresol

60. As per the TRIPS Agreement-1994, a goodoriginating from a region with specific character/quality/reputation is covered/to a protectedunder the IPR as(a) Patent(b) Trademark(c) Trade secret(d) GI (Geographical Indicator)

61. Which of the following crop cultivation isbanned by the Hon’ble Supreme Court of India?(a) Lathyrus (Khesari)(b) Genetically modified brinjal(c) Bt cotton for export(d) Bt cotton for local use

62. Ice glacier’s melting is a common phenomenonlinked to the rise in a seawater level. The glaciersare predominantly present in(a) Greenland (b) Antarctica(c) Himalayas (d) Arctic

63. Who is known for establishing-the “AnandVan”?(a) Jubilant Buddha (b) H. N. Bahuguna(c) Baba Amte (d) Motilal Nehru

64. The civilian Airport of highest altitude is in(a) Tibet (d) Nepal(c) India (d) China

65. Genomic (DNA) studies in camel have beencompleted recently by the scientists of(a) South Africa (b) India(c) China (d) Pakistan

66. International Simon Bolivar Prize was recentlyawarded to Aung San Suu Kyi by the government of(a) Mauritius (b) China(c) Venezuela (d) Cuba

67. BCCI conferred “Col. C. K. Naydu LifetimeAchievement Award” during 2012 to(a) Sachin Tendulkar (b) M. S. Dhoni(c) VVS Laxman (b) Sunil Gavaskar

68. Air quality depicting PM 2.5 is more hazardous to(a) Archaeological Monuments(b) National Parks(c) Botanical Gardens(d) Old Men and Women

69. Which of the following is not a fundamental rightas per the Indian Constitution?(a) Right to Education(b) Right to Information(c) Right to Speech(d) Right to Life

70. Who is custodian of the Indian Constitution?(a) President of India(b) Chief Justice of India(c) Prime Minister of India(d) Chairman of Rajya Sabha

71. Piped Natural Gas (PNG) is used for(a) Mining (b) Welding(c) Anaesthesia (d) Cooking

72. Greenpark Stadium is in(a) Bengaluru (b) Dehradun(c) Chandigarh (d) Kanpur

yoursmahboob.w

ordpress.com

207SSC CGL SolvedPaper

73. Rowlatt Act 1919 was enacted during the period of(a) Lord Chelmford (b) Lord William(c) Lord Minto (d) Lord Bentinck

74. Panchayati Raj System was implemented first inthe pair of states(a) Andhra Pradesh and Rajasthan(b) Assam and Bihar(c) Arunachal Pradesh and Uttar Pradesh(d) Punjab and Chandigarh

75. Human Environment Conference-1972 was held at(a) Stockholm (b) Paris(c) Geneva (d) Australia

76. ‘Gold’ is mainly related to(a) Local market(b) National market(c) International market(d) Regional market

77. Bilateral monopoly refers to the market situation of(a) two sellers, two buyers(b) one seller and two buyers(c) two sellers and one buyer(d) one seller and one buyer

78. The economist who believed that unemploymentis impossible and that market mechanism has abuilt in regulatory system to meet any ups anddowns(a) J. M. Keynes (b) Ohlin(c) J. B. Say (d) Galbraith

79. Constituent Assembly of India was formulatedon the recommendation of(a) Wavel Plan (b) Cripps Mission(c) August Offer (d) Cabinet Mission

80. Which of the following is an essential elementof the state?(a) Sovereignty (b) Government(c) Territory (d) All these

81. Which has become a legal right under 44thAmendment?(a) Right to Education(b) Right to Property(c) Right to Judicial Remedies(d) Right to work

82. By which Constitution Amendment Act, Rightto Property ceased to remain a fundamental right?(a) 44th (d) 42nd(c) 43rd (d) 45th

83. Who said “Truth is the ultimate reality and it isGod”?(a) Swamy Vivekananda(b) Rabindra Nath Tagore(c) M. K. Gandhi(d) Radhakrishnan

84. Which of the following tribes is associated withthe “Tana Bhagat” movement?(a) Uraon (b) Munda(c) Santhal (d) Kondadora

85. Who founded the Naujawan Bharat Sabha?(a) B. C. Pal(b) G. Subramania Iyer(c) Sardar Bhagat Singh(d) Rukmani Lakshmipathi

86. The Narendra Mandal or Chamber of Princes wasinaugurated in 1921 by(a) Lord Curzon(b) Lord Wellesley(c) Duke of Cannaught(d) Duke of Wellingdon

87. Buddha, Dhamma and Sangha together are known as(a) Triratna (b) Trivarga(c) Trisarga (d) Trimurti

88. Who was called Lichchavi Dauhitra?(a) Chandragupta I (b) Skandagupta(c) Kumaragupta (d) Samudragupta

89. Which hill station is called as the ‘Queen of theSatpuras’(a) Pachmarhi (b) Nilgiri(c) Mahenderagiri (d) Cardamom

90. Which national highway connects Delhi andKolkata via Mathura and Varanasi?(a) NH 4 (b) NH 2(c) NH 10 (d) NH 6

91. The country where drip irrigation is moreefficiently used is(a) India (b) Israel(c) Sri Lanka (d) England

92. Which of the following is an endangeredspecies?(a) Black buck (b) Blue sheep(c) Gangetic dolphin (d) Mithun

93. Of the following man-made disasters, which issocially induced?(a) Debris Avalanche (b) Salt Water Intrusion(c) Arson (d) Ozone depletion

94. Which one of the following endocrine gland issituated in the neck?(a) Pancreas (b) Thyroid(c) Pituitary (d) Adrenals

95. The seat of intelligence is situated in the(a) cerebrum (b) cerebellum(c) medulla (d) thalamus

96. What is the Normal Blood Volume in human adult?(a) One litre (b) Three litres(c) Five litres (d) Seven litres

yoursmahboob.w

ordpress.com

208 SSC CGL SolvedPaper

97. The fasting blood glucose level in adults in mg/100 ml is(a) 200 (b) 160(c) 100 (d) 60

98. Entomology is the study of(a) Birds (b) Insects(c) Fossils (d) Fungi

99. Exobiology is a science that deals with(a) extinct forms(b) life in other planets(c) life in the outer space(d) life in marine habitat

100. In radio-communication, the signals emitted bytransmitting antenna are reflected on(a) stratosphere (b) ozonosphere(c) ionosphere (d) troposphere

PART-C : NUMERICAL APTITUDE

101. ABCD is a cyclic trapezium with AB || DC andAB = and diameter of the circle. If ÐCAB = 30°then ÐADC is(a) 60° (b) 120°(c) 150° (d) 30°

102. ABC is a triangle. The bisectors of the internalangle ÐB and external angle ÐC intersect at D. IfÐBDC = 50°, then ÐA is(a) 100° (b) 90°(c) 120° (d) 60°

103. AB is the chord of a circle with centre O and DOCis a line segment originating from a point D on thecircle and intersecting, AB produced at C suchthat BC = OD. If ÐBCD = 20°, then ÐAOD = ?(a) 20° (b) 30°(c) 40° (d) 60°

104. In a circle of radius 17 cm, two parallel chords oflengths 30 cm and 16 cm are drawn. If both thechords are on the same side of the centre, thenthe distance between the chords is(a) 9 cm (b) 7 cm(c) 23 cm (d) 11 cm

105. If sin (A – B) = 12

and cos (A + B) = 12

where

A > B > 0 and A + B is an acute angle, then thevalue B is

(a) 6p

(b)12p

(c)4p

(d)2p

106. The fifth term of the sequence for which t1 = 1, t2= 2 and tn + 2 = tn + tn + 1, is(a) 5 (b) 10(c) 6 (d) 8

107. If (x + 7954 × 7956) be a square number, then thevalue of ‘x’ is(a) 1 (b) 16(c) 9 (d) 4

108. A can do a piece of work in 12 days while B alonecan do it in 15 days. With the help of C they canfinish it in 5 days. If they are paid ` 960 for thewhole work how much money A gets?(a) ` 480 (b) ` 240(c) ` 320 (d) ` 400

109. Ronald and Elan are working on an Assignment.Ronald takes 6 hours to type 32 pages on acomputer, While Elan takes 5 hours to type 40pages. How much time will they take workingtogether on two different computers to type anassignment of 110 pages?(a) 7 hrs. 30 min. (b) 8 hrs.(c) 8 hrs. 15 min. (d) 8 hrs. 25 min.

110. One man, 3 women and 4 boys can do a piece ofwork in 96 hours, 2 men and 8 boys can do it in80 hours, 2 men and 3 women can do it in 120hours. 5 men and 12 boys can do it in

(a)139 hours11

(b)742 hours

11

(c)743 hours11

(d) 44 hours

111. ABC is a right angled triangle, B being the rightangle. Mid-points of BC and AC are respectivelyB¢ and A¢. The ratio of the area of the quadrilateralAA¢ B¢B to the area of the triangle ABC is(a) 1 : 2 (b) 2 : 3(c) 3 : 4 (d) None of the above

112. A square ABCD is inscribed in a circle of unitradius. Semicircles are described on each side asa diameter. The area of the region bounded bythe four semicircles and the circle is(a) 1 sq. unit (b) 2 sq. unit(c) 1.5 sq. unit (d) 2.5 sq. unit

113. If the perimeters of a rectangle and a square areequal and the ratio of two adjacent sides of therectangle is 1 : 2 then the ratio of area of therectangle and that of the square is(a) 1 : 1 (b) 1 : 2(c) 2 : 3 (d) 8 : 9

yoursmahboob.w

ordpress.com

209SSC CGL SolvedPaper

114. The interest on a certain sum of money is ̀ 22 andthe true discount on the same sum for the sametime and at the same rate is ` 20, find the sum.(a) ̀ 220 (b) ` 200(c) ` 210 (d) ` 212

115. A retailer purchased radiosets at the rate of `400 each from a wholesaler. He raised the priceby 30% and then allowed a discount of 8% oneach set. His profit will be(a) 19% (b) 78.4%(c) 22% (d) 19.6%

116. A reduction in the price of apples enables aperson to purchase 3 apples for ` 1 instead of `1.25. What is the % of reduction in price(approximately)?(a) 20 (b) 25

(c) 30 (d) 1333

117. ` 700 is divided among A, B, C in such a way thatthe ratio of the amount of A and B is 2 : 3 and thatof B and C is 4 : 5. Find the amounts in ` eachreceived, in the order A, B, C.(a) 150, 250, 300 (b) 160, 240, 300(c) 150, 250, 290 (d) 150, 240, 310

118. The ratio of monthly incomes of A, B is 6 : 5 andtheir monthly expenditures are in the ratio 4 : 3. Ifeach of them saves ` 400 per month, find thesum of their monthly incomes.(a) 2300 (b) 2400(c) 2200 (d) 2500

119. A and B have together three times what B and Chave, while A, B, C together have thirty rupeesmore than that of A. If B has 5 times that of C,then A has(a) ` 60 (b) ` 65(c) ` 75 (d) ` 45

120. A cricket player after playing 10 tests scored 100runs in the 11th test. As a result, the average ofhis runs is increased by 5. The present averageof runs is(a) 45 (b) 40(c) 50 (d) 55

121. A fruit seller buys some oranges at the rate of 4for ` 10 and an equal number more at 5 for ` 10.He sells the whole lot at 9 for ` 20. What is hisloss or gain percent?

(a) Loss percent 191 %81

(b) Gain percent 191 %81

(c) No loss or no profit(d) Loss percent 2%

122. 15 litres of a mixture contains alcohol and waterin the ratio 1 : 4. If 3 litres of water is mixed in it,the percentage of alcohol in the new mixturewill be

(a) 15 (b)2163

(c) 17 (d)1182

123. A man rides at the rate of 18 km/hr, but stops for6 mins. to change horses at the end of every 7th km.The time that he will take to cover a distance of90 km is(a) 6 hrs. (b) 6 hrs. 12 min.(c) 6 hrs. 18 min. (d) 6 hrs 24 min.

124. A man rows down a river 15 km in 3 hrs. with the

stream and returns in 172

hrs. The rate at whichhe rows in still water is(a) 2.5 km/hr (b) 1.5 km/hr(c) 3.5 km/hr (d) 4.5 km/hr

125. There is 100% increase to an amount in 8 years,at simple interest. Find the compound interest of` 8000 after 2 years at the same rate of interest.(a) ` 2500 (b) ` 2000(c) ` 2250 (d) ` 2125

126. If the number p is 5 more than q and the sum ofthe squares of p and q is 55, then the product ofp and q is(a) 10 (b) – 10(c) 15 (d) – 15

127. If 1a 4

a 2+ =

- , then the value of

( )2

2 1a 2a 2

æ ö- + ç ÷-è ø is

(a) 0 (b) 2(c) – 2 (d) 4

128. If a + b + c = 2s, then

( ) ( ) ( )2 2 2 2

2 2 2s a s b s c s

a b c

- + - + - +

+ +is equal to

(a) a2 + b2 + c2 (b) 0(c) 1 (d) 2

129. If xy (x + y) = 1 then, the value of

3 33 31 x y

x y- - is

(a) 3 (b) – 3(c) 1 (d) – 1

yoursmahboob.w

ordpress.com

210 SSC CGL SolvedPaper

130. If a3 – b3 – c3 = 0 then the value of a9 – b9 – c9 –3a3b3c3 is(a) 1 (b) 2(c) 0 (d) – 1

131. The minimum value of (x – 2) (x – 9) is

(a)114

- (b)494

(c) 0 (d)494

-

132. If x + y + z = 6 and x2 + y2 + z2 = 20 then the valueof x3 + y3 + z3 – 3xyz is(a) 64 (b) 70(c) 72 (d) 76

133. The third proportional tox yy x

æ ö+ç ÷

è ø and 2 2x y+ is

(a) xy (b) xy

(c) 3 xy (d) 4 xy134. In a triangle ABC, the side BC is extended up to

D. Such that CD = AC, if ÐBAD = 109° and ÐACB= 72° then the value of ÐABC is(a) 35° (b) 60°(c) 40° (d) 45°

135. Two circles touch each other internally. Theirradii are 2 cm and 3 cm. The biggest chord of thegreater circle which is outside the inner circle oflength.(a) 2 2cm (b) 3 2cm(c) 2 3cm (d) 4 2cm

136. ABCD is a cyclic quadrilateral AB and DC areproduced to meet at P. If ÐADC = 70° and ÐDAB= 60°, then the ÐPBC + ÐPCB is(a) 130° (b) 150°(c) 155° (d) 180°

137. From a point P which is at a distance of 13 cm fromcenter O of a circle of radius 5 cm, in the sameplane, a pair of tangents PQ and PR are drawn tothe circle. Area of quadrilateral PQOR is(a) 65 cm2 (b) 60 cm2

(c) 30 cm2 (d) 90 cm2

138. A horse is tied to a post by a rope. If the horsemoves along a circular path always keeping therope stretched and describes 88 metres when ithas traced out 72° at the centre the length of the

rope is 22Take7

æ öp =ç ÷è ø

(a) 70 m (b) 75 m(c) 80 m (d) 65 m

139. Maximum value of (2 sinq + 3 cosq) is(a) 2 (b) 13(c) 15 (d) 1

140. The value of152 (sin 30° + 2 cos2 45° + 3 sin 30° + 4 cos2 45° +.... + 17 sin 30° + 18 cos2 45°) is(a) an integer but not a perfect square(b) a rational number but not an integer(c) a perfect square of an integer(d) irrational

141. If (1 + sina) (1 + sinb) (1 + sing) = (1 – sina) (1 –sinb)(1 – sing), then each side is equal to(a) cos cos cos± a b g (b) sin sin sin± a b g

(c) sin cos cos± a b g (d) sin sin cos± a b g142. One of the four angles of a rhombus is 60°. If the

length of each side of the rhombus is, 8 cm, thenthe length of the longer diagonal is

(a) 8 3 cm (b) 8 cm

(c) 4 3 cm (d)8 cm3

143. If the arcs of square length in two circles subtendangles of 60° and 75° at their centres, the ratio oftheir radii is(a) 3 : 4 (b) 4 : 5(c) 5 : 4 (d) 3 : 5

144. Study the above bar graph showing theproduction of food grains (in million tons).What is the ratio between the maximumproduction and the minimum production duringthe given period?

120100806040

200Pr

oduc

tion

of fo

od g

rai

(a) 1 : 2 (b) 2 : 3(c) 3 : 4 (d) 5 : 2

yoursmahboob.w

ordpress.com

211SSC CGL SolvedPaper

Directions: Study the following Histogram andanswer the following question nos. 145 to 149.

0123456789

10

10 20 30 40 50Marks

No.

of S

tude

nts

145. The total number of students involved in thedata is(a) 33 (b) 32(c) 43 (d) 42

146. The maximum number of students got the marksin the interval of(a) 10 – 20 (b) 20 – 30(c) 30 – 40 (d) 40 – 50

147. The least number of students got the marks inthe interval(a) 40 – 50 (b) 20 – 30(c) 10 – 20 (d) 0 – 10

148. The ratio of the students obtaining marks in thefirst and the last interval is(a) 5 : 4 (b) 6 : 5(c) 4 : 5 (d) 3 : 4

149. The difference in the amount estimated by thefamily on interior decoration and architect’s fees is(a) ` 10000 (b) ` 9500(c) ` 7200 (d) ` 9600

150. In a certain country, allocations to various sectorsof the yearly budget per ` 1000 crores arerepresented by this pie-diagram. The expenditure(in `) on Agriculture is

Agricu

lture

I ndu

s try

Education

DefenceTechnology

Miscellan eous

72°90°

72°54°54°

18°

(a) 250 crores (b) 150 crores(c) 300 crores (d) 200 crores

PART-D : ENGLISH COMPREHENSION

Directions: In question nos. 151 and 152, there arefour different words out of which only one word iscorrectly spelt. Find the correctly spelt word andindicate it by blackening the appropriate oval [ ]in the Answer Sheet.151. (a) humorous (b) humourous

(c) humorus (d) humourus152. (a) narcisism (b) narcissism

(c) narcisim (d) narsisism

Directions: In question nos. 153 to 162, you havetwo brief passages with 5 questions in each passage.Read the passages carefully and choose the bestanswer to each question out of the four alternativesand mark it by blackening the appropriate oval[ ] in the Answer Sheet.

PASSAGE I(Question Nos. 153 to 157)

Pidgins are languages that are not, acquired as mothertongues and that are used for a restricted set ofcommunicative functions. They are formed from amixture of languages and have a limited vocabularyand a simplified grammar. Pidgins serve as a means ofcommunication between speakers of mutuallyunintelligible languages and may become essential,in multilingual areas. A creole develops from a pidginwhen the pidgin becomes the mother tongue of thecommunity. To cope with the consequent expansionof communicative functions the vocabulary isincreased and the grammar becomes more complex.Where a creole and the standard variety of Englishcoexist, as in the Carribbean, there is a continuumfrom the most extreme form of creole to the form thatis closest to the standard language. Linguists markoff the relative positions on the creole continuum asthe ‘basilect’ (the furthest from the standardlanguage), the ‘mesolect’ and the ‘acrolet’. In suchsituations, most creole speakers can vary their speechalong the continuum and many are also competent inthe standard English of their country.153. A pidgin develops in a situation when

(a) Different and mutually unintelligiblelanguages exist side by side

(b) A creole becomes the mother tongue of alinguistic community

(c) A language with restricted vocabularyundergoes an expansion in grammar andvocabulary

(d) Two similar languages are mixed to create anew language.

yoursmahboob.w

ordpress.com

212 SSC CGL SolvedPaper

154. According to the given passage a pidgin becomesa creole when(a) It ceases to be a means of communication(b) It becomes the mother tongue for a new

generation of speakers(c) Its vocabulary undergoes some kind of

change(d) Two or more languages are mixed with an

existing pidgin155. According to the passage, a creole continuum is

(a) A linguistic term for the mixture of more thantwo languages

(b) A scale which measures the linguisticcompetence of the speaker.

(c) A scale in which the proximity of the creoleto the standard language is measured

(d) A record of the continuous history of acreole

156. According to the passage ‘basilect’ means(a) An impure form of a creole(b) A form of creole which is furthest from the

standard language(c) A form of creole which has an extended

vocabulary(d) A form of creole which is very close to the

standard language157. Find out a word in the passage which is opposite

in meaning to the word - ‘Simplified’(a) Complex (b) Expansion(c) Restricted (d) Consequent

PASSAGE II(Question Nos. 158 to 162)

There were four of us - George and William SamuelHarris, and myself, and Montmorency. We were sittingin my room, smoking and talking about “how badwere - bad from a medical point of view I mean, ofcourse.We were all feeling seedy and we were getting quitenervous about it. Harris said he felt such extraordinaryfits of giddiness come over him at times, that he hardlyknew what he was doing and then George said that hehad fits of giddiness too, and hardly knew what hewas doing. With me, it was my liver that was out oforder. I knew it was my liver that was out of order,because I had just been reading a patent liver-pillcircular, in which were detailed the various symptomsby which a man could tell when his liver was out oforder. I had them all.It is a most extraordinary thing, but I never read apatent medicine advertisement without being impelled

to the conclusion that I am suffering from the particulardisease therein dealth with in its most virulent form.The diagnosis seems in every case to correspondexactly with all the sensations that I have ever felt.158. The four felt down and out because

(a) the room was too smoky(b) they could never read a patent medicine

advertisement(c) they thought they were ill(d) they had experienced a most extraordinary

thing159. Whenever the speaker read a liver pill circular

(a) he suffered from an extraordinary surge ofgiddiness

(b) he felt sure that he had a liver disorder(c) he felt the urge to smoke(d) All of the above

160. The author of the above passage seems to besuffering from(a) fits of morbid depression without real cause(b) an abnormal anxiety about his health(c) melancholia(d) an unnecessarily dark, gloomy and

pessimistic attitude to life161. Harris was troubled by

(a) a symptom of vertigo(b) garrulity(c) tribulation(d) frailty

162. The word which is closest in meaning to virulentis(a) fantastic (b) vital(c) viral (d) hostile

Directions: In question nos. 163 to 167, some partsof the sentences have errors and some have none.Find out which part of a sentence has an error andblacken the oval [ ] corresponding to theappropriate letter (a, b, c). If a sentence is free fromerror, blacken the oval corresponding to (d) in theAnswer Sheet.

163. I whistled thrice with full might and raise my arms (a) (b)towards the sky. No error. (c) (d)

164. Science and religion (a)are both necessary for man and for their

(b)outer and inner self respectively. No error. (c) (d)

yoursmahboob.w

ordpress.com

213SSC CGL SolvedPaper

165. At certain seasons, some areas on Mars (a) (b)is subject to strong winds. No error. (c) (d)

166. As an artist Raju is as good (a) (b)if not better than Ramesh. No error. (c) (d)

167. The scientists could not hardly (a) (b)complete all the experiments. No error. (c) (d)

Directions: In question nos. 168 to 172, sentencesare given with blanks to be filled in with anappropriate word(s). Four alternatives aresuggested for each question. Choose the correctalternative out of the four and indicate it byblackening the appropriate oval [ ] in the AnswerSheet.168. Google is one of the most popular search engines,

it is ________ by the internet users.(a) utilized (b) effected(c) examined (d) flabbergasted

169. Raj was tired of Puja’s ________ approach, sohe asked her to make her final decision by thatevening.(a) silly-willy (b) dilly-dally(c) wasting (d) dilly-nally

170. Ria is ________ at speaking languages. It isdifficult to _______ only one puppy for animalshelter.(a) adept, adapt (b) adapt, adapt(c) adept, adopt (d) adapt, adopt

171. School days are considered to be the best yearsof your life. When my ________ year in schoolbegan. I began ,to think of those past enjoyabledays and of my future also.(a) penultimate (b) absolute(c) integral (d) termination

172. Being ________, the judge gave a favourableverdiot.(a) sagacious (b) pugnacious(c) malicious (d) tenacious

Directions: In question nos. 173 to 175, out of thefour alternatives, choose the one which best expressesthe meaning of the given word and mark it in theAnswer Sheet.173. Garrulous

(a) talkative (b) sedative(c) vocative (d) positive

174. Tinsel(a) tinkle (b) decoration(c) tin (d) colourful

175. Labyrinth(a) meandering (b) rotating(c) pacing (d) wriggling

Directions: In question nos. 176 to 178, choose theword opposite in meaning to the given word andmark it in the Answer Sheet.176. Cordial

(a) fast (b) heartfelt(c) friendly (d) hostile

177. Instinctive(a) innate (b) rational(c) inherent (d) inborn

178. Venial(a) minor (b) pardonable(c) unpardonable (d) clean

Directions: In question nos. 179 to 183, fouralternatives are given for the Idiom/Phrase printedin bold the sentence. Choose the alternative whichbest expresses the meaning of the Idiom/Phrase andmark it in the Answer Sheet.179. Hard work pays in the long run.

(a) always(b) over a period of time(c) indefinitely(d) never

180. I felt a fish out of water among the lawyers.(a) special (b) happy(c) uncomfortable (d) proud

181. The Cauvery water issue led to apple of discordbetween the two Governments.(a) cause of anger(b) cause of hatred(c) cause of quarrel(d) cause of animosity

182. The construction remains unfinished and theworkers have let the grass grow under their feet.(a) grown grass all over the lawn(b) gone on a luxury tour(c) delayed doing the work(d) demanded more benefits

183. The police smelt the rat behind the death of thegirl.(a) got very much confused(b) identified the cause of death(c) suspected that something is fishy(d) jumped to the conclusion

yoursmahboob.w

ordpress.com

214 SSC CGL SolvedPaper

Directions: In question nos. 184 to 193, a sentence/part of the sentence is printed in bold. Below aregiven alternatives to the sentence/part of thesentence at (a), (b) and (c) which may improve thesentence. Choose the correct alternative. In case noimprovement is needed, your answer is (d).184. The disparity of the GDP between the rich and

the poor has broadened in the last some decades.(a) have widened in the last some decades(b) have widened in the last few decades.(c) have broadened in the last few decades.(d) No improvement

185. How, is beyond my understanding, the boy couldfall into the ditch.(a) How the boy could fall into the ditch is

beyond my understanding(b) Beyond my understanding is how the boy

could fall into the ditch(c) How could the boy fall into the ditch is

beyond my understanding.(d) No improvement

186. The pioneer spacecraft went beyond Pluto.(a) made its way past. (b) went across(c) went after (d) No improvement

187. The firm buys frozen seafood is bulk, packs itinto smaller pouches and then they sell them tothe local grocery stores.(a) it sells them(b) sell them(c) they sell the pouches(d) No improvement

188. The learners are intended to read the sources athome.(a) are meant (b) are suggested(c) are expected (d) No improvement

189. Fuji’s invention of supercomputer will be enableto make Japan supercede America in computertechnology.(a) will make Japan (b) will enable Japan(c) can make Japan (d) No improvement

190. I could never repay the debt I owe to my place ofstudy.(a) Alma Master (b) Motherland(c) Place of worship (d) No improvement

191. She cries all the time.(a) mostly everytime (b) Day in and day out(c) pretty frequently (d) No improvement

192. For a week last month, the team’s 20 players werestraded because the Government-issuedpassport is not up to international standards.

(a) Government-issued passports were not upto international standards.

(b) Government-issued passports are not up tointernational standards

(c) The passports issued by the governmentwere not up to international standards

(d) No improvement193. Since July 2008, our customers will be able to

use the ATM network of BBY Bank, the bankthat was acquired by us during that year.(a) have been able to use(b) were using(c) will have been able to use.(d) No improvement

Directions: In question nos. 194 to 200, one of thefour alternatives, choose the one which can besubstituted for the given words/sentence and indicateit by blackening the appropriate oval [ ] in theAnswer Sheet.

194. One who is unaffected or indifferent to joy, pain,pleasure or grief(a) Tolerant (b) Resigned(c) Passive (d) Stoic

195. A person who is greatly respected because ofwisdom(a) veracious (b) vulnerable(c) venerable (d) verger

196. An excessively morbid desire to steal.(a) stealomania (b) kleptomania(c) cleftomania (d) keptomania

197. Prohibited by law or treaty from being importedor exported(a) contraband (b) smuggled(c) counterfeit (d) forged

198. Intentional destruction of racial groups.(a) regicide (b) genocide(c) homicide (d) fratricide

199. A person in a vehicle or on horseback escortinganother vehicle(a) Navigator (b) Escort(c) Outrider (d) Security

200. A person specially interested in the study ofcoins and medals.(a) medallist (b) coinist(c) numismatist (d) numerist

yoursmahboob.w

ordpress.com

215SSC CGL SolvedPaper

HINTS & SOLUTIONS1. (c) 10 + 10 ̧ 10 – 10 × 10 = 10

Þ 10 × 10 ̧ 10 – 10 + 10 = 10Þ 10 – 10 + 10 = 10

2. (d) (8 – 8) + 8 × 32 = 64Þ (8 + 8) ̧ 8 × 32 = 64Þ 16 ̧ 8 × 32 = 64Þ 2 × 32 = 64

3. (a) R AB C

Þ - Þ +Þ + Þ ´

25 A 37 C 2 B 4 R 1 = ?Þ ? = 25 + 37 × 2 ¸ 4 – 1? = 62 × 2 ̧ 4 + 1? = 124 ̧ 4 + 1

? = 31 + 1 = 324. (a) Here, Tool–Worker relationship has been

shown. Chisel is the tool of Sculptor.Similarly, Harrow is the tool of Gardener.

5. (b) Moon is a Satellite. Earth is a Planet.6. (d) As

J B I C

B J C I

SimilarlyC X D W

X C W D7. (d) As,

A B N O+13+13

Similarly

L M Y Z+13+13

8. (b) As,A G I O

+8+8

SimilarlyE K M S

+8+8

9. (d) As,25 Þ 25 × (2 + 5) = 175Similarly,32 Þ 32 × (3 + 2) = 160

10. (d) Except Agra, all others are hill stations.11. (b) Foal is the young one of a horse or of a related

animal. Lamb is the youngone of the sheep.Leveret is the young one of a hare.Hen is a female bird especially of a fowl.

12. (c) Here,1 3 1

1 3 1

1 2 1

1 3 1

B A D C

X W Z Y

V U S T

N M P O

- + -

- + -

- - +

- + -

¾¾® ¾¾® ¾¾®

¾¾® ¾¾® ¾¾®

¾¾® ¾¾® ¾¾®

¾¾® ¾¾® ¾¾®13. (d) Here,

1 3 1

1 3 1

1 3 1

1 3 2

D C F G

F E H I

J I L M

H G J L

- + -

- + +

- + +

- + +

¾¾® ¾¾® ¾¾®

¾¾® ¾¾® ¾¾®

¾¾® ¾¾® ¾¾®

¾¾® ¾¾® ¾¾®14. (b) In the number pair (2, 3) both the numbers

are prime numbers.15. (d) Except the number pair (82, 64), in all other

number pairs both the numbers aremultiples of 3.96 = 32 × 3; 24 = 8 × 339 = 13 × 3; 18 = 6 × 381 = 27 × 3; 54 = 18 × 3

16. (d) Meaningful order of the words inascending order :

5. Sensation ̄

2. Perception ̄

4. Assimilation ̄

3. Scheme Formation ̄

1. Accommodation

yoursmahboob.w

ordpress.com

216 SSC CGL SolvedPaper

17. (a) Meaningful order of the words inascending order :

4. Lithosphere ̄

1. Hydrosphere ̄

3. Biosphere ̄

2. Atmosphere18. (d) The pattern of the series is as follows :

1 1 1 1

1 1 1 1

1 1 1 1

E F G H

J K L M

O P Q R

+ + + +

+ + + +

+ + + +

¾¾® ¾¾® ¾¾® ¾¾®

¾¾® ¾¾® ¾¾® ¾¾®

¾¾® ¾¾® ¾¾® ¾¾®

INS

19. (a) The patter is as follows :1 1 1 1

1 1 1

B C D E

O N M

+ + + +

- - -

¾¾® ¾¾® ¾¾® ¾¾®

¾¾® ¾¾® ¾¾®

FL

? = LF20. (a) 7 – 2 = 5; 7 + 2 = 9

Þ 7, 2 = 595 – 3 = 2; 5 + 3 = 8Þ 5, 3 = 289 – 1 = 8; 9 + 1 = 10Þ 9, 1 = 8102 – 1 = 1; 2 + 1 = 3Þ 2, 1 = 13Therefore,5 – 3 = 1; 5 + 4 = 9Þ 5, 4 = 19

21. (a) The pattern is as follows :120 440 960 1680 2600

+320 +520 +720 +920

+200 +200 +20022. (b) First Column

81 4 36´ ´Þ 9 × 2 × 6 = 108Second Column

64 9 16´ ´Þ 8 × 3 × 94Third column

16 49 25´ ´Þ 4 × 7 × 5 = 140

23. (d) First number in each row is the product ofthe second and third numbers.

First Row25 = 5 × 5Second Row30 = 5 × 6Third Row35 = ? × 5

\ ? = 35 75

=

24. (d) First Column24 × 2 + 5 = 48 + 5 = 53Second Column51 × 4 + 7 = 204 + 7 = 211Third Column67 × 6 + 5 = 402 + 5 = 407

25. (b) M > G > HM > QH is the most lean person in the group.

26. (b) North

Wes

t East

South15 m

15 m

10 m

15 m\ Required distance = (15 + 10) m = 25 m

27. (*) M N O P QBus O is in the middle of the five.

28. (d) There is only one ‘E’ in the given word butthere are two Es in the word TEASE.

29. (d) There is no ‘D’ letter in the given word.Therefore, the word TENDER cannot beformed.

30. (b) There is no ‘N’ letter in given word.Therefore, the word LANCER cannot beformed.

31. (*) As,D I V I N E A F S F K B

–3–3–3–3–3–3

yoursmahboob.w

ordpress.com

217SSC CGL SolvedPaper

Similarly

–3–3–3–3–3–3–3–3

P O W E R F U L M L T B O C R I

32. (b) As,N O T E P Q V G

+2+2+2+2

SimilarlyT I M E V K O G

+2+2+2+2

33. (b) As,S M A R T U K C P V

+2–2+2–2+2

Similarly

W O N D E R Y M P B G P+2–2+2–2+2–2

34. (c) + Þ ´ <Þ + + Þ -- Þ + ´ Þ>

Option (a)20 – 4 ̧ 4 + 8 < 2 × 26Þ 20 + 4 – 4 × 8 ¸ 2 > 26Þ 20 + 4 – 4 × 4 > 26Þ 24 – 16 × 26Option (b)20 × 8 + 15 < 5 ̧ 9 – 8Þ 20 > 8 × 15 ̧ 5 – 9 + 8Þ 20 > 8 × 3 – 9 + 8Þ 20 > 24 – 9 + 8 Þ 20> 23Option (c)20 < 2 + 10 ̧ 4 – 6 × 100Þ 20 ̧ 2 × 10 – 4 + 6 > 100

Þ 10 × 10 – 4 + 6 > 100Þ 100 – 4 + 6 > 100Þ 100 – 4 > 100Option (d)20 < 5 + 25 ̧ 10 – 2 × 96Þ 20 + 5 × 25 – 10 + 2 > 96Þ 4 × 25 – 10 + 2 > 96Þ 100 – 10 + 2 > 96Þ 102 – 10 > 96

35. (b) 10 km

10 km

10 km

5 km\ Required distance = (10 + 5) km = 15 km

36. (d) 12 m

12 mOffice Canteen

2 m

\ Required distance = 2 metres37. (d)

Apples Bananas Sweets

Conclusions :I. Some apples are sweet. (True)II. Some bananas are apples.(True)So, both I and II follow.

38. (b)

Metals Silver Diamond

Gold Marbles

Conclusions :I. Some gold are metals. (False)II. All metals are diamonds. (True)III. Some silver are marble. (False)IV. Some gold are silver. (False)So, only II follows.

yoursmahboob.w

ordpress.com

218 SSC CGL SolvedPaper39. (b) When folded in the form of a cube, then 'F'

appears opposite 'B', 'E' appears opposite'C' and 'A' appears opposite ‘D’.In option (a) 'F' is adjacent to 'B'In option (c) 'E' is adjacent to ‘C’.In option (d) is adjacent to ‘A’.

40. (d) The numbers 1, 2, 5 and 6 are on theadjacent faces of number 3. Therefore. thenumber 4 lies opposite 3.The numbers 3, 4 and 6 can not be on thefaces opposite to 1. Therefore, 5 liesopposite 1.Now, 2 lies opposite 6.

41. (d) All the components of question figure arepresent in the Answer Figure (d).

42. (d) Employed, truthful and iliterate socialworkers would be indicated by the regioncommon to all the four geometrical figures.Such region is marked '1’.

43. (c) First Figure3 + 5 + 2 + 4 = 14Þ 14 + 13 = 27Second Figure6 + 2 + 3 + 5 = 16Þ 16 + 21 = 37Third Figure2 + 2 + 9 + 5 = 18Þ 18 + 29 = 47

44. (b) Women graduate sub–inspectors of policecan be represented by the region commonto all the three geometrical figures. Suchregion is marked '3'.

45. (c)46. (b)

47. (c)

48. (d)

49. (c)50. (c) C Þ 02, 11, 23, 32, 40

A Þ 00, 13, 21, 33, 42R Þ 57, 68, 77, 88, 99D Þ 03, 10, 22, 30, 41

Option C A R D

(a) 32 00 56 10

(d) 02 42 77 20

(c) 11 33 57 22

(b) 40 21 68 44

51. (c) Semiconductors are insulators at lowtemperatures and reasonably goodconductors at higher temperatures. Astemperature increases, the semi–conductormaterial becomes a better and betterconductor.

52. (c) Mass of proton is 1.672621777 (74) × 10–27

kg. An electron has a mass (9.1093829140)× 10–31 kg) that is approximately 1/1836 thatof the proton. The mass of neutron isslightly larger than that of a proton. The massof the hydrogen nucleus is 1.7 × 10–27 kg.The heaviest of these particles is theneutron.

53. (a) A television channel is a physical or virtualchannel over which a television station ortelevision network is distributed. Channelnumbers represent actual frequencies usedto broadcast the television signal. Forexample, in North America. "Channel 2"refers to the broadcast or cable band of 54to 60 MHz, with carrier frequencies of 55.25MHz for NTSC analog video (VSB) and59.75 MHz for analog audio (FM), or 55.31MHz for digital ATSC (8VSB).

54. (a) The computer mouse as we know it todaywas invented and developed by DouglasEnglebart during the 1960's. While creatingthe mouse, ' Douglas was working at theStanford Research Institute and originallyreferred to the mouse as a "X–Y PositionIndicator for a Display System."

55. (d) Profitability, as a fundamental goal of eachprofit oriented organization, depends on anelaborated strategy to provide a real valuefor customers. One of the determinants ofthe profitability of an organization is thequality of' data processed. However, thespeed of processing the data is alsoimportant as speedy delivery is also a keydeterminant. For example, outsourcingincreases profitability and it also helpscompanies in saving for the cost of labor.

yoursmahboob.w

ordpress.com

219SSC CGL SolvedPaper

56. (b) Water never has an absolute densitybecause its density varies with temperature.Water has its maximum density of lg/cm3 at4 degrees Celsius. When the temperaturechanges from either greater or less than 4degrees, the density will become less than1 g/cm3.

57. (c) Photosynthesis is the process by whichlight energy is converted into chemicalenergy by organisms. Carbon dioxide andwater are the raw materials of this process.The light energy comes from the sun andits end products are oxygen and glucose.

58. (b) An emulsion is a mixture of two or moreliquids that are normally immiscible(nonmixable or unblendable). Emulsions arepart of a more general class of two–phasesystems of matter called colloids. In anemulsion, one liquid (the dispersed phase)is dispersed in the other (the continuous.

59. (b) The active ingredient in Dettol that confersits antiseptic property is chloroxylenol(C8H9CIO), an aromatic chemicalcompound. Chloroxylenol comprises 4.8%of Dettol's total mixture, with the restcomposed of pine oil, isopropanol, castoroil soap caramel and water.

60. (d) Geographical Indication (GI) means thename of a region or a locality, a specificplace, or, in exceptional cases, a country,used to describe a product originating inthat region, locality, specific place orcountry, which possesses a specific quality,reputation or other characteristicsattributable to that geographical origin, andthe production and/or processing and orpreparation of which take place in thedefined geographical area.

61. (a) Grass pea also known as khesari dal wasbanned for consumption in 1961 as it wasbelieved that Beta–N–Oxalylaminoalanine,a neuro–toxic amino–acid in the legume,caused neurolathyrim or a paralysis of thelower limbs,

62. (b) It is mainly cause crbyo phenomena: globalwarming and ozone depletion. Polar ice capsin Antarctica and other places are–meltingfast. This has led to increase in sea levelover the years.

63. (c) Founded in 1948 by noted social activist,Baba Amte, Anandwan, located near Arorain Chandrapur district in the state ofMaharashtra. India, is an ashram, and a

community rehabilitation centre for leprosypatients and the disabled fromdowntrodden sections of society.

64. (d) Daocheng Yading Airport in A southwestChina's Sichuan Province is the world'shighest–altitude civilian airport. It is locatedat a height of 4,411 meters above sea level

65. (c) Researchers in Mangolia and Chinacompleted studies to unravel the genomicPeculiaritces behind the physiological tricksthat camels use to survive in the harshestof conditions. They described the genomesof wild and domesticated Bactrian camels.

66. (c) International Simon Bolivar Prize winnersare selected by the unanimous decision ajury of seven "eminent persons" – fiverepresenting the regions of the world, onechosen by the Venezuelan authorities, anda representative of the director–general –from a list of candidates submitted byUNESCO member states and associatemembers.

67. (d) The BCCI conferred the C.K. NayuduLifetime Achievement Award on formerIndia captain Sunil Gavaskar. The awardcomprises a trophy, citation and cheque for` 25 lakh.

68. (d) Fine particles known as PM10 and PM25canpenetrate deep into the lungs, creatinghealth problems. People with heart or lungdiseases, older adults, and children aremost likely to have problems because ofcontact with particle pollution. Short–termexposure to PM among pregnant womenhas been associated with prematurity andgrowth retardation.

69. (*) Right to Information is a part of fundamentalr ights under Article 19 (1) of theConstitution which states that every citizenhas freedom of speech and expression. The86th Constitutional amendment makingeducation a fundamental right was passedby Parliament in 2002. Right to speechcomes under freedom of speech andexpression which is a fundamental right.The constitution guarantees the right to lifeand personal liberty as a fundamental rightunder article 21.

70. (b) The Constitution has made the SupremeCourt as the custodian and protector of theConstitution. The Supreme Court decidesdisputes between the Centre and the Unitsas well as protects the Fundamental Rightsof the citizens of India.

yoursmahboob.w

ordpress.com

220 SSC CGL SolvedPaper71. (d) PNG is a mixture consisting mainly methane

CH4 with a small percentage of other higherhydrocarbons. The ratio of carbon tohydrogen is least in methane and hence itburns almost completely making it thecleanest fuel. Domestic PNG customers,also known as Residential customers, usegas for cooking purpose and also forheating water through gas geysers.

72. (d) Green Park Stadium is a 60,000 capacityfloodlit multi–purpose stadium located inKanpur, India, and the home of the UttarPradesh cricket team.

73. (a) The Rowlatt Act was passed by the ImperialLegislative Council in London on 18 March1919, indefinitely extending "emergencymeasures" enacted during the First WorldWar in order to control public unrest androot out conspiracy. Lord Chelmsford (1916to 1921) was the Viceroy of India then.

74. (a) Jawaharlal Nehru inaugurated the firstgeneration of panchayat raj at Nagaur inRajasthan on 2 October 1959. It was alsoimplemented in Andhra Pradesh in the sameyear as per the recommendations of theBalwant Rai Mehta Committee.

75. (a) The conference was held in Stockholm. thecapital of Sweden. in 1972. The conferenceagreed upon a declaration containing 26principles concerning the environment anddevelopment.

76. (c) Gold is mainly related to the internationalmarket as of all the precious metals, it is themost popular as an investment. Gold hasbeen used throughout history as moneyand has been a relative standard forcurrency equivalents specifc to economicregions or countries, until recent times. Goldprice has shown a long term correlation withthe price of crude oil.

77. (d) Bilateral monopoly is a market structureboth a monopoly (a single seller) andmonopsony (a single buyer) in the samemarket. The one supplier tends to act as amonopoly power, and looks to charge highprices to the one buyer. The lone buyerlooks towards paying a price that is as lowas possible.

78. (c) Say's law of markets is the core of theclassical theory of employment. J.B. Say,an early 19th century French Economist,enunciated the proposition that "supplycreates its own demand." Therefore, therecannot be general over production and theproblem of unemployment in the economy.

79. (d) May preceded by the Shimla Conferenceof 1945.

80. (d) The state has four essential elements:population, territory, government andsovereignty. Absence of any of theseelements denies to it the status of statehood.

81. (b) The 44th amendment eliminated the rightto acquire, hold and dispose of property asa fundamental right. However, in anotherpart of the Constitution. Article 300 (A) wasinserted to affirm that no person shall bedeprived of his property save by authorityof law.

82. (a) The 44th amendment of 1978 eliminated theright to acquire, hold and dispose ofproperty as a fundamental right. TheConstitution originally provided for theright to property under Articles 19 and 31.

83. (c) Gandhi was known to have considered truthsomething similar to that. Baruch Spinozaconsidered ultimate truth as the ultimatereality of a rationally ordered system that isGod.

84. (a) Tana Bhagat movement was apparentlyinitiated in 1914 by a young Oraontribesman known as Jatra Oraon.

85. (c) The Naujawan Bharat Sabha was foundedby Bhagat Singh in March 1926 and wasdeclared illegal under the Criminal LawAmendment Act of 1908 in September 1934.Its purpose was to forment revolutionagainst the British Raj by gatheringtogether worker and peasant youths.

86. (c) The chamber of Princes was instituted on 8February 1921 . The inauguration ceremonywas performed by His Royal Highness theDuke of Connaught in the Diwan–i–am ofRed Fort on be; half of His Majesty theKing Emperor.

87. (a) The Three Jewels (triratna) are the threethings that Buddhists take refuge in, andlook toward for guidance, in the processknow as taking refuge. The Three Jewelsare: Buddha, Dhamma and Sangha.

88 (a) Chandragupta I was known as'Lichchavidauhitra.' His marriage with theLichchavi Princess Kumaradevi was one ofthe significant events in the Gupta rule. Theimportance of this marriage can be knownfurther from Samudragupta's Allahabadinscription in which he has describedhimself as "Lichchhavis–dauhitra ordaughter's son of the Lichchavis.

yoursmahboob.w

ordpress.com

221SSC CGL SolvedPaper

89. (a) Panchmarhi a beautiful hill station in MadhyaPradesh is known as Queen of Satpurabecause of its mystic beauty, it is a knowndestination for Bollywood film shootings.

90. (b) National Highway 2, commonly referred asDelhi–Kolkata Road, is a busy IndianNational Highway that runs through thestates of Delhi, Haryana, Uttar Pradesh,Bihar, Jharkhand, and West Bengal. Itconstitutes a major portion of the historicalGrand Trunk. Road along with NH 91 andNH 1 in India.

91. (b) Drip irrigation is a form of irrigation thatsaves water and fertilizer by allowing waterto drip slowly to the roots of many differentplants, either onto the soil surface or directlyonto the root zone, through a network ofvalves, pipes, tubing, and emitters.Drip irrigation was developed in Israel bySimcha Blass and his son Yeshayahu.

92. (c) According to the Red list of 2012. GangesRiver Dolphin is one of criticallyendangered species in lndia Endangeredspecies in India comprise large varieties ofrare species of wild animals, aquatic animalsand insects.

93. (c) Arson is the crime of intentionally andmaliciously setting fire to buildings, wildland areas, vehicles or other property withthe intent to cause damage, Distinct fromspontaneous combustion and naturalwildfires, it is considered to be a man–madedisaster which is socially induced.

94. (b) The thyroid gland, is one of the largestendocrine gland in the throat, and consistsof two connected lobes. It is found at thefront of the neck, below the Adam’s apple.The thyroid gland secretes thyroidhormones, which influence the metabolicrate, protein synthesis, and have a widerange of other effects, including ondevelopment.

95. (b) The cerebrum is the seat of intelligence andprovides us with the ability to read, writeand speak: make calculations and composemusic; remember the past and plan for thefuture; and create works.

96. (c) The average adult has a blood volume ofroughly 5 liters, which is composed ofplasma and several kinds of cells. By volume,the red blood cells constitute about 45% ofwhole blood, the plasma about 54.3%, andwhite cells about 0.7%.

97. (c) The normal range of, conentration offasting blood sugar is 80 to 120 mg/ml; inthe testing of true blood sugar, the normalrange of concentration is 70 to 100 mg/ml.so normal glucose levels fall between 70and 150 mg. Higher levels may indicatediseases such as diabetes mellitus.

98. (b) Entomology is the scientific study ofinsects, a branch of arthropodology, whichin turn is a branch of biology. It is derivedfrom the Greek word 'entomos' which means"that which is cut in pieces or engraved/segmented", hence "insect".

99. (a)100. (c) The ionosphere is a region of Earth's upper

atmosphere, from about 60 km (37 mi) to1,000 km (620 mi) altitude and includes thethermosphere and parts of the mesosphereand exosphere, it is ionized by solarradiation, plays an important part inatmospheric electricity and forms the inneredge of the magnetosphere. It has practicalimportance because, among otherfunctions, it influences radio propagationto distant places on the Earth.

101. (b) D

AO

B

C

ÐACB = 90° (Angle in semi–circle)ÐCAB = 30°\ ÐCBA = 180° – 90° – 30° = 60°ÐADC + ÐABC = 180°\ ÐADC = 180° – 60° = 120°

102. (a) A

B C E

D

ÐACE = ÐBAC + ÐABCÐDCE = ÐDBC + ÐBDC= ÐDBC + 50°= ÐACE = ÐB + 100\ ÐBAC = 100°

yoursmahboob.w

ordpress.com

222 SSC CGL SolvedPaper

103. (d)

D

A

O

B C

BC = DO = OAÐDAB = 90°ÐDOA = 2 × DCA = 40°

104. (b)A C

O FE

DBAE = 15 cmOA = 17 cm

\ OE = 2 217 15-= (17 15)(17 15)+ - = 32 2´ = 8 cmCF = 8 cmOC = 17 cm

\ OF = 2 217 8-= (17 8)(17 8)+ - = 25 9´= 15 cm= 15 –8 = 7 cm

105. (b) sin (A – B) = 12

= sin 30°

Þ A – B= 3°

cos (A + B) = 12

= cos 60°Þ A + B = 60°\ A + B + A – B = 30° + 60° = 90°Þ 2A = 90°Þ A = 45°\ A – B = 30°Þ B = A – 30° = 45° – 30° = 15°

= 15

180 12´ p p= radian

106. (d) tn + 2 = tn + tn + 1t2 = t2 + t2 = 3t4 = t3 + t2 = 3 + 2 = 5t5 = t4 + t3 = 3 + 5 = 8

107. (a) x + 7954 × 7956x + 7954 (7954 + 2)x + 79542 + 2 × 7954 × 1Putting x = 1(x + 7954)2 or (1 + 7954)2

108. (d) Work done by A and B in 5 days

= 1 1 5 4

5 512 15 60

+æ ö æ ö+ =ç ÷ ç ÷è ø è ø

= 9 3

12 4=

Time taken by C in doing 14

work = 5 days\ C will complete in 20 days.

\ Ratio of wages = 1 1 1: :

12 15 20= 5 : 4 : 3\ Amount received by A

= 5

96012

´ = ̀ 400

109. (c) Ronald’s 1 hour’s work

= 32 166 3

= pages

Elan’s 1 hour’s work = 8 pages1 hour’s work of the both

= 16 40

83 3

+ = pages

\ Requried time

= 110 3 33

40 4´ = hours

= 8 hours 15 minutes

110. (c) 1 hr’s work of 1 man and 4 boys = 1

160

1 hr’s work of 1 man and 3 women = 196

1 hr work of 3 women

= 1 1 10 6 196 160 960 240

-- = =

1 hr work of 2 men

= 1 1 1

120 240 240- =

yoursmahboob.w

ordpress.com

223SSC CGL SolvedPaper

1 hr work of 4 boys

= 1 1

160 480-

= 3 1 1480 240-

=

\ 2 men = 3 women = 4 boys\ 2 men + 8 boys = 12 boys5 men + 12 boys = 22 boys\ By M1 D1 = M2D2Þ 12 × 80 = 22 × D2

Þ D2 = 12 80

22´

= 480 74311 11

= hours

111. (c) A

B B¢

CIn DABCA¢ and B¢ are mid point of AC and BC

A¢B¢ = 12

AB

DA¢B¢C ~ DABC

\ ABC 4

A B C 1D

=D ¢ ¢

Þ A B C 1ABC 4D ¢ ¢ =D

Þ 1 – A B C 11ABC 4D ¢ ¢ = -D

Þ ar AA B B 3

ar ABC 4D ¢ ¢ =D

112. (b)

A

B C

DBD = 2 unitsAB = 2 units

Area of square = 2 square unitsArea of four semicircles

= 4 × 2r

2p =

142

2

´ p ´ = p sq. units

\ Required area= 2 + p – p = 2 sq. units.

113. (d) Sides of rectangle are 2x and x units.Side of square = y units\ 4y = 6x

Þ x 4 2y 6 3

= =

\ 2

2 22x x 2x 2 4

9y y´ ´= =

= 8 : 9

114. (a) Sum = S.I. True discountS.I. – True discount´

= 22 2022 20´-

= ̀ 220

115. (d) Marked price of a ratio set

= 400 130

100´

= ̀ 520

S.P. = 520 92

100´

= ̀ 478.4

\ Gain percent = 78.4 100100´ = 19.6%

116. (a) Percentage decrease

= 25

125 × 100 = 20%

117. (b) A : B = 2 : 3 = 8 : 12B : C = 4 : 5 = 12 : 15\ A : B : C = 8 : 12 : 15Sum of ratio = 35

\ A’s share = 8 70035´

= ̀ 160

B’s share = 1235

× 700 = ̀ 240

C’s share = 1535

× 700 = ̀ 300

118. (c) Incomes of A and B= ` 6x and 5xExpenses of A and B= ` 4y and 3y

yoursmahboob.w

ordpress.com

224 SSC CGL SolvedPaper

\ 6x – 4y = 400 ...(i)5x – 3y = 400 ...(ii)By equation (i) × 3 – (ii) × 4Þ 18x – 12y – 20x + 12y= 1200 – 1600Þ 2x = 400 Þ x = 200\ Total income= 6x + 5x = 11x = ̀ 2200

119. (b) A + B = 3 (B + C)A + B + C = A + 30B = 5C\ A + B = 3 (B + C)Þ A + 5C = 18C Þ A = 13C\ A + B + C = A + 30Þ A + 5C + C = A + 30

Þ A + 6A13

= A + 30

Þ 6A = 30 × 13Þ A = ̀ 65

120. (c) If the average in 10 tests be x, then,x 10 100

11´ +

= x + 5

x × 10 + 100 = (x + 5) × 11Þ 11x – 10x = 100 – 55Þ x = 45\ Required average = 50

121. (a) Let 20 apples of each type be bought.C.P. of 40 apples

= 10 10

20 204 5

æ ö´ + ´ç ÷è ø = ` 90

Total S.P. = 40 20

= ̀ 800

9

Loss = 90 – 800 10

9 9=

\ Loss per cent = 109 100

90´

= 100 191 %81 81

=

122. (b) Alcohol = 3 litresWater = 12 litres\ Required percentage

= 3 100

15 3´

+

= 50 2163 3

=

123. (b) Number of stoppages = 90 927

@

\ Total time = 9018

æ öç ÷è ø hours +

12 6100

´ minutes

= 6 hours 12 minutes124. (c) Speed of person in still water = x kmph and

speed of current = y kmph

\ x + y = 153

= 5 kmph

x – y = 15152

= 2 kmph

On adding,

2x = 7 Þ x = 72

= 3.5 kmph

125. (d) S.I. = ̀ 100%Principal = ̀ 8000SI = 8000

\ Rate = S.I 100

Principal 100´

´

= 8000 100 25 %8000 8 2

´ =´

\ C.I. = TrP 1 1

100

é ùæ ö+ -ê úç ÷è øê úë û

= 8000 2251 1

200

é ùæ ö+ -ê úç ÷è øê úë û

= 81 8000 17

8000 164 64

´æ ö- =ç ÷è ø = ̀ 2125

126. (c) p = q + 5Þ p – q = 5p2 + q2 = 55\ (p – q)2 + 2pq = 55Þ 25 + 2 pq = 55Þ 2 pq = 30Þ pq = 15

127. (b) a + 1

a 2- = 4

Þ (a – 2) + 1

(a 2)- = 4 – 2 = 2

On squaring,

Þ (a – 2)2 + 21 2

(a 2)+

- = 4

Þ (a – 2)2 + (a – 2)2 = 2

yoursmahboob.w

ordpress.com

225SSC CGL SolvedPaper

128. (c) Expression

= 2 2 2 2

2 2 2(s a) (s b) (s c) s

a b c- + - + - +

+ +

= 2 2 2 2 2 2 2

2 2 2s – 2sa a s b 2sb s 2sc c s

a b c+ + + - + - + +

+ +

= 2 2 2 2

2 2 24s a b c 2s(a b c)

a b c+ + + - + +

+ +

= 2 2 2 2 2

2 2 24s a b c 4s

a b c+ + + -

+ + = 1

129. (a) xy (x + y) = 1

Þ x + y = 1xy

Cubing both sides,

x3 + y3 + 3xy (x + y) = 3 31

x y

Þ x3 + y3 + 3 × 1 = 3 31

x y

Þ 3 31

x y – x3 + y3 = 3

130. (c) If a + b + c = 0, thena3 + b3 + c3 – 3abc = 0

131. (d) Expression = (x – 2) (x – 9)= x2 – 11x + 18 = ax2 + bx + c

Minimum value = 24ac b

4a-

= 4 1 18 121 49

4 4´ ´ - -=

132. (c) x + y + z = 6On squaring,x2 + y2 + z2 + 2xy + 2zy + 2zx = 36Þ 20 + 2 (xy + yz + zx) = 36Þ xy + yz + zx = 8\ x3 + y3 + z3 – 3xyz= (x + y + z) (x2 + y2 + z2 – xy – yz – zx)= 6 (20 – 8) = 72

133. (a) Third proportional of a and ba : b : ca bb c

=

c = 2b

a

=

( )22 2

2 2

2 2

x y x yx y x yy x xy

+ +=++ = xy

134. (a) A

B C DÐACD = 180° – ÐACB= 180° – 72° = 108°

ÐCAD = ÐADC = 722

= 36°

\ ÐABC = 180° – 109° – 36° = 35°

135. (d) D

C B OA

E

O¢A = 3 cmOA = 2 cmCA = 6 cmO¢D = 3 cmO¢B = 1 cmBD = 23 1 2 2- =

DE = 4 2 cm136. (a)

D

A B

C

P

ÐADC = 70°ÐABC = 180° – 70° = 110°Þ ÐPBC = 70°ÐBCD = 180° – 60° = 120°Þ ÐPCB = 60°\ ÐPBC + ÐPCB = 70° + 60° = 130°

yoursmahboob.w

ordpress.com

226 SSC CGL SolvedPaper

137. (b)

R

PO

Q

ÐOQP = ÐORP = 90°

PQ = 2 2OP OQ-

= 2 213 5- = 12\ PQOR =2 × DOPQ

= 1

2 5 122

´ ´ ´

= 60 sq. cm

138. (a) B

AO

72°

q = 72°Q 180° = p radius

\ 72° = 72180

p ´

= 25p

radians

Arc AB = s = 88 metre

\ q = sr

Þ 2 885 rp =

Þ 2pr = 88 × 5

Þ r = 88 5

2´p

= 88 5 7

2 22´ ´´

= 70 metre

139. (b) Maximum value of a sin q + b cosq

= 2 2a b+\ Maximum value of 2 sin q + 3 cos q= 2 22 3 13+ =

140. (c) 152 (sin 30° + 2 cos2 45° + 3 sin 30° + 4 cos2

45° + .... + 17 sin 30° + 18 cos2 45°)

= 152 = 1 1 1 1 1 1

2 3 4 .... 17 182 2 2 2 2 2

æ ö+ ´ + ´ + ´ + + ´ + ´ç ÷è ø

= 1 3 17152 1 2 .... 92 2 2

æ ö+ + + + + +ç ÷è ø

It is an A.P. whose a = 12

, d = 12

, n = 18

= 18 1 1152 2 (18 1)2 2 2

é ùæ ö´ + -ç ÷ê úè øë û

= 17152 9 13

é ùæ ö+ç ÷ê úè øë û

= 152 9 19

2´ ´

= 12996 and 12996 = 114

141. (a) (1 + sin a) (1 + sin b) (1 + sin g) = (1 – sin a)(1 – sin b) (1 – sin g) = x\ x.x = (1 + sin a) (1 – sin a) (1 + sin b)(1 – sin b) (1 + sin g) (1 + sin g)= (1 – sin2 a) (1 – sin2 b) (1 – sin2 g)= cos2 a. cos2b.cos2 g= x = ± cosa . cos b . cosg

142. (a) A

B D

c

o

ÐBAD = 60°\ÐBAO = 30°ÐABO = 60°

\ sin 60° = OAAB

Þ 3

82

´ = OA

\ OA = 4 3\ AC = 8 3 metre

143. (c)sr

q = [When q = 2p]

Þ s = rqÞ s = r1q1 = r2q2

Þ 1 2

2 1

r 75 5r 60 4

q= = =

q

yoursmahboob.w

ordpress.com

227SSC CGL SolvedPaper

144. (d) Required ratio = 100 : 40 = 5 : 2145. (a) Required number of students

= 4 + 6 + 10 + 8 + 5 = 33146. (b) Students in class interval

20 – 30 = 10147. (d) Required class interval

= 0 – 10148. (c) Required ratio = 4 : 5149. (*) The question is incomplete.150. (d) Expenditure on agriculture

Sector = 72

1000360

´

` 200 crore151. (a)152. (b) Narcissism means : the habit of admiring

yourself too much.153. (a)154. (b) 155. (c) 156. (b) 157. (a)158. (c) 159. (b) 160. (d) 161. (a)162. (b)163. (b) The sentence is in past simple. Hence, with

full might and raised my arms .... should beused.

164. (b) Here, are both necessary for men and fortheir ....... should be used.Plural sense is evident.

165. (c) Here, subject (some areas) is in pluralnumber. Hence, plural verb i.e. are subjectto strong winds ......... should be used.

166. (b) Here, Raju is (so/very) good should beused. As is super-fluous here.

167. (b) Hardly means almost no: almost not. Hence,could hardly should be used. Hardlyconveys negative sense.

168. (a) Utilize means to use something, especiallyfor a practical purpose

169. (b) Dilly–dally means too take too long to dosomething, go somewhere or make adecision.

170. (b) Adept means good at doing something:skilful.Adopt means to take some-body's child intoyour family.

171. (d) Penultimate means before the last one: lastbut one Termination means the act ofending something

172. (a) Pugnacious = having a strong desire to fightwith other peopleMalicious = having or showing hatred anda desire to harm somebodySagacious = showing good judgementTenacious = persistent; continuing to exist

173. (a) The word Garrulous (Adjective) means :talkative; talking a lot.

174. (b) The word Tinsel (Noun/Adjective) means :strips of shiny material like metal used asdecorations.

175. (a) The word Labyrinth (Noun) means : acomplicated series of paths which it isdifficult to find your way through,meandering.

176. (d) The word Cordial (Adjective) means :pleasant and friendly.The word Hostile (Adjective) means : veryunfriendly or aggressive.

177. (b) The word Instinctive (Adjective) means :natural : based on instinct not thought ortraining.The word Rational (Adjective) means :reasonable.

178. (c) The word Venial (Adjective) : meanspardonable; minor; not very serious andhence to be forgiven.

179. (b) In the long run = concerning a longer periodin the future.

180. (c) A fish out of water = a person who feelsuncomfortable or awkward

181. (d) Apple of discord = cause of animosity.182. (c) Let the grass grow under your feet = to

delay in getting things done.183. (c) Smell the rat = to suspect that somebody is

wrong about a situation.184. (b) Here, has widened ..... should be used.

Widen = to become wider; larger in degree.185. (a) Arrangement of words186. (a)187. (a) For singular subject (the firm), it should be

used.188. (c)189. (b) Enable is a verb.190. (a) Motherland = the country that you are born

in and have a strong emotional connection.Alma mater = the school, college thatsomebody went to

191. (d) All the time = very often; frequently.192. (a) Past tense should follow as the first clause

is in past tense.193. (a) Present Perfect Continuous should be used

as the work continued in the past and stillcontinues.

194. (d) 195. (c) 196. (b) 197. (a)198. (b) 199. (b) 200. (c)

yoursmahboob.w

ordpress.com

228 SSC CGL SolvedPaper

PART-A : GENERAL INTELLIGENCE & REASONING

Directions: In questions nos. 1 and 4, select therelated letters/word/number from the givenalternatives.

1. F : 216 :: L : ?(a) 1700 (b) 1600(c) 1723 (d) 1728

2. MOUSE : KPSTC :: LIGHT : ?(a) JGEFR (b) JJEIR(c) MJHIU (d) MGHFUL

3. Tanning : Leather :: Pyrotechnics : ?(a) Fireworks (b) Wool(c) Machinery (d) Bombs

4. King : Throne :: Rider ?(a) Horse (b) Seat(c) Saddle (d) Chair

5. If 264 * 2 = 6, 870 * 3 = 11, then what should 735* 5 be?(a) 16 (b) 03(c) 05 (d) 12

6. Which one of the given responses would be ameaningful order of the following?1. Implementation2. Conceptual Modelling3. Requirements Analysis4. Logical Modelling5. Physical Model6. Schema Refinement(a) 1, 3, 2, 6, 5, 4 (b) 3, 2, 5, 4, 6, 1(c) 3, 2, 1, 4, 6, 5 (d) 3, 2, 4, 6, 5, 1

Directions: In question nos. 7 and 8, which one setof letters when sequentially placed at the gaps in thegiven letter series shall complete it?

7. b_ab_b_aab_b(a) abba (b) baaa(c) aabb (d) abbb

8. _bcab_cabc_abca_b(a) aabc (b) bbca(c) abac (d) abca

Directions: In question nos. 9 and 13, find the oddword/number/number pair from the givenalternatives.

9. (a) Rickets (b) Night-blindness(c) Influenza (d) Scurvy

10. (a) Shower (b) Sleet(c) Raisin (d) Rain

11. (a) 71 – 55 (b) 517 – 523(c) 248 – 231 (d) 547 – 563

12. 10, 13, 234, 681, 997(a) 234 (b) 13(c) 681 (d) 10

13. (a) Heed (b) Led(c) Heap (d) Head

Directions : In question nos. 14 to 18, a series isgiven, with one term missing. Choose the correctalternative from the given ones that will completethe series.

14. C – 3, E – 5, G – 7, I – 9, ?, ?(a) K – 11, M – 13 (b) O – 15, X – 24(c) M – 18, K – 14 (d) X – 24, M – 21

15. 5, 16, 51, 158, ?(a) 483 (b) 481(c) 1454 (d) 1452

16. 9 * 2 : 9 * 9 :: 9 × 5 : ?(a) 9 × 6 (b) 9 × 7(c) 9 × 8 (d) 9 × 4

17. AFI, JOR, MRU, ?(a) HMP (b) PMO(c) RJL (d) GJN

18. bc, cde, de, efg, fg ?(a) fgh (b) hij(c) ijk (d) ghi

19. A man showed a boy next to him and said - “Heis the son of my wife’s sister-in-law, but I am theonly child of my parents.” How is my son relatedto him?(a) Brother (b) Uncle(c) Nephew (d) Cousin

SSC Combined Graduation Level (CGL) Solved PaperMORNING SHIFT 19 MAY 2013

yoursmahboob.w

ordpress.com

229SSC CGL SolvedPaper

20. Find the correct group of signs to solve theequation.24 * 16 * 8 * 32(a) – + = (b) × ̧ =(c) + – = (d) ¸ – =

21. Sunil is the son of Kesav. Simran, Kesav’s sister,has a son Maruti and daughter Sita. Prem is thematernal uncle of Maruti. How is Sunil related toMaruti?(a) Uncle (b) Brother(c) Nephew (d) Cousin

22. Priti scored more than Rahul. Yamuna scored asmuch as Divya. Lokita scored less than Manju.Rahul scored more than Yamuna. Manju scoredless than Divya. Who scored the lowest?(a) Yamuna (b) Lokita(c) Rahul (d) Manju

Directions: In questions no. 23 to 25, from the givenalternatives select the word which cannot be formedusing the letters of the given word.

23. CONSTITUTIONAL(a) TALENT (b) LOCATION(c) CONSULT (d) TUITION

24. MANUSCRIPT(a) SMART (b) RUSTIC(c) MASTER (d) PRIMUS

25. SEGREGATION(a) GREAT (d) GREETINGS(c) SEATING (d) NATION

26. If the first and second letters in the word‘COMMUNICATIONS’ were interchanged, alsothe third and the fourth letters, the 5th and 6thletters and so on, which letter would be the tenthletter counting from your right?(a) A (b) T(c) N (d) U

27. Which of the following interchanges of numberswould make the given equation correct?8 × 20 ̧ 3 + 9 – 5 = 38(a) 8, 9 (b) 3, 5(c) 3, 9 (d) 3, 8

28. Put the correct mathematical signs in thefollowing equation from the given alternatives.33 ? 11 ? 3 ? 6 = 115(a) ×, ̧ , – (b) ¸, ×, ×(c) –, ×, + (d) +, –, ×

29. Select the correct combination of mathematicalsigns to replace * signs and to balance the givenequation.15 * 24 * 3 * 6 * 17(a) + × = ¸ (b) – × = +(c) – ¸ + = (d) + ¸ – =

30. If each of the letters in the English alphabet isassigned an even numerical value by giving A =2, B = 4 and so on, what would be the total valueof the letters for the word LADY when similarlycoded?(a) 72 (b) 84(c) 82 (d) 74

31. If the word LEADER is coded as 20-13-9-12-13-26, how would you write LIGHT?(a) 20-15-16-18-23 (b) 20-17-15-16-28(c) 20-16-15-17-22 (d) 20-16-17-15-27

Directions : In question nos. 32 to 34, select themissing number from the given responses?

32. 345

58

562

842

62

763?58

(a) 122 (b) 128(c) 124 (d) 126

33. 4 3 26 9 109 27 ?

(a) 20 (b) 50(c) 54 (d) 30

34. 84, 81 88 14 12 18 9 ? 11

(a) 12 (b) 14(c) 16 (d) 10

35. Raj is standing in the middle of a square field. Hestarts walking diagonally to North-East. Thenhe turns right and reaches the far end of thefield. Then he turns right and starts walking. Inthe midway he agains turns right and startswalking. In halfway, he turns to his left andreaches a new far end. In what direction is Rajnow?(a) South (b) North-West(c) South-West (d) North

yoursmahboob.w

ordpress.com

230 SSC CGL SolvedPaper

36. A boy starts from home in early morning andwalks straight for 8 km facing the Sun. Then hetakes a right turn and walks for 3 km. Then heturns right again and walks for 2 km and thenturns left and walks for 1 km. Then he turns right,travels 1 km and then turns right and travels for4 km straight. How far is he from the starting point?(a) 6 km (b) 2 km(c) 4 km (d) 5 km

Directions: In question nos. 37 to 38, one/twostatements are given, followed by two/threeconclusions I, II and III. You have to consider thestatements to be true even if they seem to be atvariance from commonly known facts.You have todecide which of the given conclusions, if any, followfrom the given statements.37. Statement:

Sick people need medicineConclusions:I. Healthy people do not need medicineII. People keep medicine in their home.(a) Only conclusion II follows(b) Both conclusions I and II follow(c) Neither conclusion I nor II follows(d) Only conclusion I follows

38. Statement:1. Some years are decades.2. All centuries are decades.Conclusions:I. Some centuries are years.II. Some decades are years.III. No century is a year.(a) Only conclusion I and III follow(b) Only conclusion I follows(c) Only conclusion II follows(d) Only conclusion I and II follow

39. How many cubes are there in the group?

(a) 16 (b) 18(c) 20 (d) 10

40. Which one of the following diagrams bestdepicts the relationship among Earth, Sea, Sun?

(a) (b)

(c) (d)

41. There is a ball and a rectangular jar. Fourpositions are shown below to keep thembalanced. Which of the following will not getbalanced easily?Question figures:

Answer figures:

(a) (b) (c) (d)

42. Which one of the four boxes given below iscreated by folding the given key design in thequestion figure?Question figures:

Answer figures:

(a) (b) (c) (d)

yoursmahboob.w

ordpress.com

231SSC CGL SolvedPaper

43. In the given diagram, circle representsprofessionals, square represents dancers,triangle represents musicians and rectanglerepresents Europeans. Different regions in thediagram are numbered 1 to 11. Who among thefollowing is neither a dancer nor a musician butis professional and not a European?

1

2 3

4 5

6 78 9

1011

(a) 11 (b) 1(c) 10 (d) 8

44. Read the figure and find the region representingpersons who are educated and employed butnot confirmed.

Employed

Confirmed in job

Educated

a

bc

dcf

(a) abc (b) bd(c) adc (d) ac

Directions: In question no. 45 to 46, which answerfigure will complete the pattern in the questionfigure?

45. Question figure:

Answer figure:

(a) (b) (c) (d)

46. Question figure:

«««

«««

Answer figure:

(a) (b) (c) (d)

47. A piece of paper is folded and punched as shownbelow in the question figures. From the givenanswer figures, indicate how it will appear whenopened.Question figure:

Answer figure:

(a) (b) (c) (d)

48. Which of the answer figures include the separatecomponents found in the question figure?Question figure:

Answer figure:

(a) (b) (c) (d)

yoursmahboob.w

ordpress.com

232 SSC CGL SolvedPaper

49. A word is represented by only one set of numbersas given in any one of the alternatives. The setsof numbers given in the alternatives arerepresented by two classes of alphabets as intwo matrices given below. The columns and rowsof Matrix I are numbered from 0 to 4 and that ofMatrix II are numbered from 5 to 9. A letter fromthese matrices can be represented first by itsrow and next by its column, e.g. ‘D’ can berepresented by 02, 14, etc., and ‘R’ can berepresented by 57, 76 etc. Similarly, you have toidentify the set for the word “BEST”. MATRIX- I MATRIX -II

0 1 2 3 40 B C D E F1 E F B C D2 C D E F B3 F B C D E4 D E F B C

5 6 7 8 95 P Q R S T6 S T P Q R7 Q R S T P8 T P Q R S9 R S T P Q

(a) 24, 21, 77, 97 (b) 24, 22, 77, 97(c) 24, 22, 77, 96 (d) 24, 22, 76, 97

50. Choose the right water-image of the questionfigure from the given answer figures.Question figure:

Answer figure:

(a) (b) (c) (d)

PART-B : GENERAL AWARENESS

51. The demand for labour is called(a) Derived demand (b) Factory demand(c) Market demand (d) Direct demand

52. Which of the following is not an investmentexpenditure in goods and services?(a) Purchase of machinery(b) An increase in business inventories(c) Expansion of the main plant of a company(d) Purchase of a house

53. Which one of the following represents theSavings of the Private Corporate Sector?(a) Undistributed profits(b) Excess of income over expenditure(c) Dividends paid to shareholders(d) Total profits of a company

54. Who presides over the Joint Session of IndianParliament?(a) Chairperson of Rajya Sabha(b) Seniormost Member of Parliament(c) Speaker of Lok Sabha(d) President of India

55. Social accounting system in India is classifiedinto(a) Assets, liabilities and debt position(b) Public sector, Private sector and Joint sector(c) Income, product and expenditure(d) Enterprise, households and government

56. Forced Savings refer to(a) Compulsory deposits imposed on income

tax payers(b) Provident fund contribution of private

sector employees(c) Reduction of consumption consequent to

a rise in prices(d) Taxes on individual income and wealth

57. Under which Article of the Constitution of India,can the fundamental rights of the members ofthe Armed Forces be specifically restricted?(a) Article 21 (b) Article 25(c) Article 33 (d) Article 19

58. Pulakesin II was the greatest ruler of the(a) Chalukyas of Kalyani(b) Pallavas of Kanchi(c) Cholas of Tamil Nadu(d) Chalukyas of Badami

59. The Uttaramerur inscription providesinformation on the administration of the(a) Pallavas (b) Cholas(c) Chalukyas (d) Satavahanas

60. Pitts India Act of 1784 was a/an(a) Ordinance (b) Resolution(c) White paper (d) Regulation Act

61. Which one of the following statements is nottrue in respect of A.O. Hume?(a) He founded the Indian National Congress.(b) He presided over the Congress Annual

Sessions twice.(c) He was an ornithologist.(d) He was a member of the Indian Civil Service.

yoursmahboob.w

ordpress.com

233SSC CGL SolvedPaper

62. Division of Powers and Independent Judiciaryare the two important features of(a) Democratic form of Government(b) Federal form of Government(c) Socialist form of Government(d) Unitary form of Government

63. Which Article of the Indian Constitution did Dr.B. R. Ambedkar term as the “Heart and Soul ofthe Indian Constitution”?(a) Article 356 (b) Article 32(c) Article 14 (d) Article 19

64. Who was the first to use the term ‘State’?(a) Aristotle (b) Machiavelli(c) Hobbes (d) Plato

65. Which river in India flows in a rift-valley?(a) Narmada (b) Krishna(c) Cauvery (d) Tapti

66. A narrow strip of land that connects two largerland masses is called(a) Cape (b) Isthmus(c) Strait (d) Peninsula

67. Tiny marine animals which constitute limestoneskeletons are called(a) Coral reefs (b) Diatoms(c) Clamitomonous (d) Foraminifera

68. Apatanis are the major tribal group of(a) Sikkim (b) Arunachal Pradesh(c) Jharkhand (d) Nagaland

69. Which of the following plant pigments absorbsin red and far-red region of light?(a) Phytochrome (b) Cryptochrome(c) Carotenoide (d) Chlorophyll

70. The process through which excess of lightenergy is dissipated in photosynthesis is known as(a) Photolysis(b) Photophosphorylation(c) Quenching(d) Scavenging

71. AIDS virus destroys(a) Neutrophils (b) Basophils(c) Lymphocytes (d) Monocytes

72. Which of the following movements saw thebiggest peasant guerilla war on the eve ofindependence?(a) Punnapra Vayalar Movement(b) Telangana Movement(c) Noakhali Movement(d) Tebhaga Movement

73. Maps on large scale, representing both naturaland man-made features are called(a) Thematic maps(b) Atlas maps(c) Wall maps(d) Topographic maps

74. The device used for measuring the wavelengthof X-rays is(a) Bragg Spectrometer(b) Mass Spectrometer(c) G. M. Counter(d) Cyclotron

75. Alpha particle is the nucleus of an atom of(a) Helium (b) Oxygen(c) Lithium (d) Hydrogen

76. Who is the founder of “Facebook” which iscurrently the No. 1 social networking website inIndia?(a) Orkut Buyukkokten (b) Mark Zuckerberg(c) Bill Gates (d) Martin Cooper

77. In programming, repeating some statements isusually called(a) Looping (b) Control structure(c) Compiling (d) Structure

78. Teeth and Bones acquire strength and rigidityfrom(a) Chlorine (b) Sodium(c) Calcium (d) Fluorine

79. The type of tail found in Shark is(a) Heterocercal (b) Diphycercal(c) Protocercal (d) Homocercal

80. The Sigmoid Colon is part of(a) Large Intestine (b) Ileum(c) Small Intestine (d) Anal Canal

81. A good conductor while carrying current is(a) positively charged(b) electrically neutral(c) alternately charged positive and negative(d) negatively charged

82. The angle between the magnetic meridian andthe geographical meridian at a place is(a) Declination (b) Latitude(c) Azimuth (d) Dip

83. Coating of solid waste with imperviouos materialis known as(a) Landfill (b) Capping(c) Encapsulation (d) Chemical fixation

yoursmahboob.w

ordpress.com

234 SSC CGL SolvedPaper

84. Ultraviolet rays can be used in water treatmentas(a) Hydrolyser (b) Disinfectant(c) Flocculator (d) Precipitator

85. Thiamidine dimer formation in DNA is causedby(a) b and g-rays (b) UV-rays(c) IR-rays (d) X-rays

86. Winner of the Australian Open Men’s SinglesTitle in 2013 is(a) Roger Federer (b) Novak Djokovic(c) David Ferrer (d) Andy Murray

87. Silicone is a polymer of(a) Dialkyl dichloro silane(b) Silane(c) Tetraalkyl silane(d) Silicon tetrachloride

88. Which is a natural colloid?(a) Cane-sugar (b) Blood(c) Sodium chloride (d) Urea

89. Which one of the following does not containSilver?(a) German Silver (b) Horn Silver(c) Ruby Silver (d) Lunar Caustic

90. The presence of Cobalt in Vitamin B12 wasestablished for the first time by(a) Borax-Bead test(b) Sodium Nitroprusside test(c) Hydrolysis test(d) Spectroscopy

91. Which bacterial strain developed from naturalisolates by genetic manipulations can be usedfor treating oil spills?(a) Clostridium (b) Nitrosomonas(c) Pseudomonas (d) Agrobacterium

92. Who is the author of the book “No Full Stops inIndia”?(a) Nirad C. Choudhuri (b) Mark Tully(c) R. K. Narayan (d) Ved Mehta

93. Which one of the following pairs is wronglymatched?

Place Location(a) Tiananmen Square - Beijing(b) Tahrir Square - Abu Dhabi(c) Trafalgar Square - London(d) Red Square - Moscow

94. The term of a non-permanent member of the U.N.Security Council is(a) 1 year (b) 2 years(c) 3 years (d) 6 months

95. Who is the Chairman of the 20th LawCommission?(a) Justice K.G. Balakrishnan(b) Justice D. K. Jain(c) Justice Usha Mehra(d) Justice J. S. Verma

96. Julia Gillard is the Prime Minister of(a) Canada (b) Australia(c) New Zealand (d) Belgium

97. Which is the first state in India to pass the FoodSecurity Law?(a) Chhattisgarh (b) Gujarat(c) Punjab (d) Kerala

98. Who among the following Presidents of MCCwas a non-professional cricketer?(a) Peter May(b) Christopher Martin Jenkins(c) Ted Dexter(d) Colin Cowdrey

99. Who won the Nobel Prize for Peace in 2012 ?(a) International Atomic Energy Agency(b) Liu Xiaobo(c) European Union(d) U. N. Intergovernmental Panel on ClimateChange

100. Who won the Jnanpith Award for 2011?(a) Sachidananda Routray(b) Pratibha Ray(c) Gopinath Mohanty(d) Sitakant Mahapatra

PART-C : NUMERICAL APTITUDE

101. Product of two co-prime numbers is 117. Thentheir L.C.M. is(a) 13 (b) 39(c) 117 (d) 9

102. The diameters of two circles are the side of asquare and the diagonal of the square. The ratioof the areas of the smaller circle and the largercircle is(a) 2 : 3 (b) 1: 2(c) 1 : 2 (d) 1 : 4

103. The total surface area of a sphere is 8 p squareunit. The volume of the sphere is

(a) 8 3 cubic unitp (b)8 3

5p cubic unit

(c)8 2 cubic unit

3p (d)

8 cubic unit3p

yoursmahboob.w

ordpress.com

235SSC CGL SolvedPaper

104. A conical flask is full of water. The flask has baseradius r and height h. This water is poured into acylindrical flask of base radius mr. The height ofwater in the cylindrical flask is

(a)2hm

(b) 2h

3m

(c)m2h

(d) 2h m2

105. A and B together can complete a piece of work in12 days, B and C can do it in 20 days and C andA can do it in 15 days. A, B and C together cancomplete it in(a) 8 days (b) 10 days(c) 12 days (d) 6 days

106. A and B together can complete a work in 3 days.They start together. But, after 2 days, B left thework. If the work is completed after 2 more days,B alone could do the work in(a) 6 days (b) 8 days(c) 10 days (d) 4 days

107. A does 20% less work than B. If A can complete

a piece of work in 17 hours2

, then B can do it in

(a) 6 hours (b) 8 hours(c) 10 hours (d) 4 hours

108. A rational number between 34

and 38 is

(a)169 (b)

916

(c)127 (d)

73

109. The prices of a school bag and a shoe are in theratio 7 : 5. The price of a school bag is ` 200 morethan the price of a shoe. Then the price of a shoe is(a) ` 500 (b) ` 1,200(c) ` 200 (d) ` 700

110. A sum of ` 300 is divided among P, Q and R insuch a way that Q gets ` 30 more than P and Rgets ` 60 more than Q. The ratio of their share is(a) 3 : 2 : 5 (b) 2 : 5 : 3(c) 5 : 3 : 2 (d) 2 : 3 : 5

111. The average of nine numbers is 50. The averageof the first five numbers is 54 and that of the lastthree numbers is 52. Then the sixth number is(a) 24 (b) 44(c) 30 (d) 34

112. The average of the first nine integral multiples of3 is(a) 15 (b) 18(c) 21 (d) 12

113. An article is sold for ` 300 at a profit of 20%. Hadit been sold ` 235, the loss percentage wouldhave been(a) 5 (b) 6(c) 16 (d) 3

114. A box has 100 blue balls, 50 red balls, 50 black ball.25% of blue balls and 50% of red balls are takenaway. percentage of black balls at present is

(a)133 %3 (b) 40%

(c) 50% (d) 25%115. A dozen pairs of socks quoted at ` 180 are

available at discount of 20%. How many pairs ofsocks can be bought for ` 48?(a) 2 pairs (b) 5 pairs(c) 3 pairs (d) 4 pairs

116. The marked price of a table is ̀ 12,000. If it wassold for `10,500 after allowing a certain discount,then the rate of discount is(a) 12.5% (b) 15%(c) 17.5% (d) 10%

117. The marked price of a radio set is `480. Theshopkeeper allows a discount of 10% and gains8%. If no discount is allowed, his gain percentwould be(a) 18.5% (b) 20%(c) 25% (d) 18%

118. N is the foot of the perpendicular from a point Pof a circle with radius 7 cm, on a diameter AB ofthe circle. If the length of the chord PB is 12 cm,the distance of the point N from the point B is

(a)53 cm7 (b)

210 cm7

(c)56 cm7 (d)

212 cm7

119. Two cars are moving with speeds v1, v2 towardsa crossing along two roads. If their distancesfrom the crossing be 40 metres and 50 metres atan instant of time then they do not collide if theirspeeds are such that(a) v1 : v2 ¹ 5 : 4 (b) v1 : v2 = 25 : 16(c) v1 : v2 = 16 : 25 (d) v1 : v2 ¹ 4 : 5

yoursmahboob.w

ordpress.com

236 SSC CGL SolvedPaper

120. A certain distance is covered at a certain speed.If half of this distance is covered in double thetime, the ratio of the two speeds is(a) 1 : 4 (b) 2 : 1(c) 1 : 2 (d) 4 : 1

121. The simple interest an ` 4,000 in 3 years at therate of x% per annum equals the simple intereston ` 5,000 at the rate of 12% per annum in 2years. The value of x is(a) 8% (b) 9%(c) 10% (d) 6%

122. If x2 – 3x + 1 = 0, then the value of

22

1 1x xx x

+ + + is

(a) 6 (b) 8(c) 10 (d) 2

123. If 4x 3 4y 3 4z 3 0

x y z- - -+ + = , then the value of

1 1 1x y z

+ + is

(a) 4 (b) 6(c) 9 (d) 3

124. Find the simplest value of 2 50 18 72+ -

(given 2 1.414= ).(a) 10.312 (b) 8.484(c) 4.242 (d) 9.898

125. If a2 + b2 + 4c2 = 2(a + b – 2c) – 3 and a, b, c arereal, then the value of (a2 + b2 + c2) is

(a) 2 (b)124

(c) 3 (d)134

126. A number x when divided by 289 leaves 18 as theremainder. The same number when divided by17 leaves y as a remainder. The value of y is(a) 3 (b) 1(c) 5 (d) 2

127. An equation of the form ax + by + c = 0 where a¹ 0, b ¹ 0, c = 0 represents a straight line whichpasses through(a) (0, 0) (b) (3, 2)(c) (2, 4) (d) None of these

128. The numerator of a fraction is 4 less than itsdenominator. If the numerator is decreased by 2and the denominator is increased by 1, then thedenominator becomes eight times the numerator.Find the fraction.

(a)48 (b)

27

(c)38 (d)

37

129. If x2 = y + z, y2 = z + x and z2 = x + y, then the value

of 1 1 1

1 x 1 y 1 z+ +

+ + + is

(a) 2 (b) 0(c) – 1 (d) 1

130. In a triangle ABC, ÐA = 90°, ÐC = 55°,

AD BC^ . What is the value of ÐBAD?(a) 45° (b) 55°(c) 35° (d) 60°

131. If G is the centroid of DABC and DABC =48cm2,then the area of DBGC is(a) 16 cm2 (b) 24 cm2

(c) 32 cm2 (d) 8 cm2

132. The diagonals AC and BD of a cyclicquadrilateral ABCD intersect each other at thepoint P. Then, it is always true that(a) AP . BP = CP . DP (b) AP . CD = AB . CP(c) BP. AB = CD. CP (d) AP . CP = BP . DP

133. If tanq = 34

and q is acute, then cosecq

(a)54

(b)43

(c)45 (d)

53

134. The value of ( ) ( )2 21 1

1 tan 1 cot+

+ q + q is

(a) 2 (b)12

(c)14

(d) 1

135. If O be the circumcentre of a triangle PQR andÐQOR = 110°, ÐOPR = 25°, then the measure ofÐPRQ is(a) 55° (b) 60°(c) 65° (d) 50°

yoursmahboob.w

ordpress.com

237SSC CGL SolvedPaper

136. A vertical stick 12 cm long casts a shadow 8 cmlong on the ground. At the same time, a towercasts a shadow 40 m long on the ground. Theheight of the tower is(a) 65 m (b) 70 m(c) 72 m (d) 60 m

137. A, B, C, D are four points on a circle. AC and BDintersect at a point E such that ÐBEC = 130° andÐECD = 20°. ÐBAC = 130° and ÐECD = 20°.ÐBAC is(a) 100° (b) 110°(c) 120° (d) 90°

138. In a triangle, if three altitudes are equal, then thetriangle is(a) Right (b) Isoceles(c) Obtuse (d) Equilateral

139. The tops of two poles of height 24 m and 36 mare connected by a wire. If the wire makes anangle of 60° with the horizontal, then the lengthof the wire is(a) 8 m (b) 6 3m(c) 6 m (d) 8 3m

140. If tan a = n tan b, and sin a = m sin b, then cos2

a is

(a)2

2m 1n 1--

(b)2

2m 1n 1

++

(c)2

2m

n 1+(d)

2

2mn

141. The value of 1 1

cosec cot sin-

q- q q is

(a) cosec q (b) tan q(c) 1 (d) cot q

142. If cosq + sinq = 2 cosq , then cosq – sinq is

(a) 2 sin- q (b) 2 sinq(c) 2 tanq (d) 2 cos- q

143. If cos4q – sin4q =23 , then the value of 1 – 2 sin2q is

(a)23 (b)

13

(c)43 (d) 0

Directions: Shown below is the multiple bar diagramdepicting the changes in the roll strength of a collegein four faculties from 2001 – 02 to 2003 – 04.

Arts

Science

Commerce

Law

2001-02 2002-03 2003-04

150200250

400

500550600

Numb

erof

Stud

ents

144. The percentage of students in Science faculty in2001 – 2002 is(a) 27.8% (b) 29.6%(c) 30.2% (d) 26.9%

145. The percentage of students in Law faculty in2003 – 04 is(a) 15.6% (b) 16.7%(c) 14.8% (d) 18.5%

146. Percentage of increase in Science students in2003 – 04 over 2001 – 2002 is

(a) 150% (b)266 %3

(c) 75% (d) 50%

Directions: The following pie-chart shows the marksscored by a student in different subject - viz. Physics(Ph), Chemistry (Ch), Mathematical (M), SocialScience (SS) and English (E) in an examination.Assuming that total marks obtained for theexamination in 810, answer the questions givenbelow.

SS

Ch

Ph

E

M

90°60°

85°70°55°

yoursmahboob.w

ordpress.com

238 SSC CGL SolvedPaper

147. The subject in which the student obtained 135marks is(a) Chemistry (b) Mathematics(c) English (d) Physics

148. The marks obtained in English, Physics andSocial Science exceed the marks obtained inMathematics and Chemistry by

(a)110 %9 (b) 11%

(c)111 %9 (d) 10%

149. The difference of marks between Physics andChemistry is same as that between(a) Mathematics and English(b) English and Social Science(c) Chemistry and Social Science(d) Physics and English

150. The marks obtained in Mathematics andChemistry exceed the marks obtained in Physicsand Social Science by(a) 40 (b) 45(c) 50 (d) 30

PART-D : ENGLISH COMPREHENSION

Directions: (Q. Nos. 151-155) In these questionssome parts of the sentences have errors and some arecorrect. Find out which part of a sentence has an error.If a sentence is free from error, your answer is (d).

151. If I was he (a)/ I wouldn't accept (b)/ this project.(c)/ No error (d)

152. The teacher advised to (a)/ the student to borrow(b)/ a book from the library within three days.(c)/ No error (d)

153. I insisted (a)/ on his going (b)/ there immediately.(c)/ No error (d)

154. Neither of the teams (a)/ are sensible enough(b)/ to do this task. (c)/ No error (d)

155. We have been knowing (a)/ each other (b)/ sincewe were children. (c)/ No error (d)

Directions: (Q. Nos. 156-160) In these questionssentences are given with blanks to be filled with anappropriate word(s). Four alternatives aresuggested for each question. Choose the correctalternative out of the four.

156. All is not well _______ the automobile sector.(a) of (b) down(c) in (d) to

157. Many premier educational institutions comeforward to have a _______ with flourshingindustries.(a) tie-down (b) tie-up(c) tie-in (d) tie-on

158. He slipped _______ his old ways and starteddrinking again.(a) into (b) off(c) by (d) in

159. They reached the railway station before the train__________ .(a) had left (b) had been left(c) left (d) was leaving

160. The Information and CommunicationTechnology has _____ age and employes veryhighly paid technocrats.(a) come of (b) come upon(c) come out of (d) come through

Directions: (Q. Nos. 161-163) In these questionsout of the four alternatives, choose the one whichbest expresses the meaning of the given word.

161. Citadel(a) Metropolis (b) Mansion(c) Fortress (d) Palace

162. Stern(a) Lenient (b) Young(c) Stem (d) Strict

163. Aberration(a) Justification (b) Intensification(c) Deviation (d) Rationality

Directions: (Q. Nos. 164-166) In these questionschoose the word opposite in meaning to the givenword and mark it.164. Dwindle

(a) Decrease (b) Diminish(c) Shrink (d) Increase

165. Dormant(a) Active (b) Inactive(c) Dorsal (d) Domestic

166. Tranquility(a) Disturbance (b) Quiet(c) Serenity (d) Peace

Directions: (Q. Nos. 167-171) In these questionsfour alternatives are given for the Idiom/ Phrase boldin the sentence. Choose the alternative which bestexpresses the meaning of the Idiom/Phrase.167. Let sleeping dogs lie.

(a) Do not bring up an old controversial issue(b) Dogs can raise tempers(c) Do not allow dogs to stand(d) Prevent dog mobility

yoursmahboob.w

ordpress.com

239SSC CGL SolvedPaper

168. To get admission in present day educationalinstitutions, all children should be born with asilver spoon in the mouth.(a) be born to silver spoon manufacturer(b) always hold a silver spoon(c) be born with silver spoon(d) be born in a rich family

169. A man of straw means(a) a worthy fellow(b) an unreasonable person(c) a man of no substance(d) a very active person

170. Children complain about their parents' gifts. Theyshould learn not to look gift horse in the mouth.(a) not to look at a horse's mouth(b) not to find fault with the gifts received(c) not to ask for more gift(d) not to find goodness in the gifts

171. Acquiring a job a cakewalk for a student whohas good academic performance coupled withthe good attitude.(a) walk away with a cake(b) a difficult achievement(c) a walkway made with cakes(d) an easy achievement

Directions: (Q. Nos. 172-181) In these questions asentence/ part of the sentence is bold. Below aregiven alternatives to the bold sentence/ part of thesentence at (a), (b) and (c) which may improve thesentence. Choose the correct alternatives. In caseno improvement is needed, your answer is (d).

172. I wish I knew what is wrong with my car.(a) I wish I had known what is wrong with my car(b) I wish I know what is wrong with my car(c) I wish I knew what was wrong with my car(d) No improvement

173. Just before he died, Amar, who is a poet, wrotethis poem.(a) Just before he died, Amar, who was a poet,

wrote this poem(b) Amar, who is a poet, wrote this poem just

before he died(c) Amar wrote this poem, who is poet, just

before he died(d) No improvement

174. The flag will be risen on the 15th of August.(a) The flag will be roused on the 15th of

August(b) The flag will be rising on the 15th August(c) The flag will be raised on the 15th of August(d) No improvement

175. If I had the money I would have bought the house.(a) If I had had the money I would have bought

the house(b) If I have the money I would have bought

the house(c) If I have had the money I would have

bought the house(d) No improvement

176. Don't sit in the grass. It's wet.(a) by the side of (b) on(c) beside (d) No improvement

177. The game is more important than the winningof the prize.(a) The gaming is more important than the

winning of the prize(b) The game is more important than winning

of the prize(c) Gaming is more important than winning of

the prize(d) No imporovement

178. The actor is out of jail, and not exactly a freeman, since he will be under house arrest for anadditional 90 days.(a) but not exactly a free man(b) though exactly a free man(c) if not exactly a free man(d) No improvement

179. Some players on the team suffer from chronicknee problems and will not play in the nextplayoff.(a) Some player on the team suffered(b) Some player on the team suffer(c) Some players on the team suffers(d) No improvement

180. Sushma has wisdom, charm and she has a goodsense of humour.(a) Sushma had a good sense of humour(b) a good sense of humour(c) has a good sense of humour(d) No improvement

181. They knocked down ten houses when they builtthe new road.(a) ruptured (b) removed(c) pulled down (d) No improvement

Directions: (Q. Nos. 182-188) In these questionsout of the four alternatives, choose the one whichcan be substituted for the given words/sentence.

182. A study of sounds is known as(a) semantics (b) sylistics(c) linguistics (d) phonetics

yoursmahboob.w

ordpress.com

240 SSC CGL SolvedPaper

183. To reduce to nothing(a) Cull (b) Lull(c) Null (d) Annul

184. An obviously true or hackneyed statement(a) Truism (b) Syllogism(c) Iconic (d) Imagism

185. Words inscribed on a tomb(a) Epilogue (b) Epitaph(c) Epitome (d) Epistle

186. The act of producing beautiful handwriting usinga brush or a special pen(a) Hieroglyphics (b) Calligraphy(c) Stencilling (d) Graphics

187. A word composed of the first letters of the wordsin a phrase.(a) Acronym (b) Abridgement(c) Almanac (d) Anachronism

188. A person of obscure position who has gainedwealth.(a) Extrabagant (b) Promiscuous(c) Parvenu (d) Sumptuary

Directions: (Q. Nos. 189-190) In these questions,four words are given in each question, out of whichonly one word is correctly spelt. Find the correctlyspelt word.

189.(a) Busyness (b) Bussyness(c) Business (d) Bussiness

190.(a) Questionnaire (b) Questionnair(c) Questionaire (d) Questionnare

Directions: (Q. Nos. 191-200) IIn these questionsyou have a passage with 10 questions. Read thepassage carefully and choose the best answer to eachquestion out of the four.The postmaster first took up his duties in the villageof Ulapur. Though the village was a small one, therewas an indigo factory nearby and the propritor, anEnglishman, had managed to get a post officeestablished.

Our postmaster belonged to Calcutta. He felt likea fish out of water in this remove village. His officeand living-room were in a dark thatched shed, not farfrom a green, silmy pond, surrounded on all sides bya dense growth.

The men employed in the indigo factory had noleisure, moreover they were hardly desirablecompanions for decent folk. Nor is a Calcutta boy anadept in the art of associating with others. Among

strangers he appears either proud or ill at ease. Atany rate the postmaster had but little company, norhad he much to do.

At times he tried his hand at writing a verse ortwo. That the movement of the leaves and clouds ofthe sky were enough to fill life with joy - such werethe sentiments to which he sought to give expression.But God knows that the poor fellow would have felt itas the gift of a new life, if some genie of the ArabianNights had in one night swept away the trees, leavesand all, and replaced them with a macadamised road,hiding the clouds from view with rows of tall houses.191. The adjective used to describe the postmaster's

living-room is(a) bright (b) dark(c) light (d) deep

192. What does the phrase 'ill at ease' in the passagemean?(a) Forward (b) Disease(c) Comfortable (d) Uneasy

193. What does the phrase 'little company' in thepassage mean?(a) Bad friendship (b) Hardly any friends(c) Small business (d) Business-like

194. At times, the postmaster wrote(a) poems (b) novels(c) short stories (d) dramas

195. The postmaster wrote on the(a) beauty of nature(b) beauty of himself(c) beauty of the weather(d) beauty of the village

196. The word 'genie' means(a) monster (b) spirit(c) ghost (d) soul

197. Which factory was situated near the villageUlapur?(a) Chemical (b) Rubber(c) Clothes (d) Dyes

198. What does the idiom 'fish out of water' suggest?(a) In unfamiliar surroundings(b) can die any moment(c) grasping for breath(d) amphibious creature

199. Find a word in the passage which is the oppositeof 'near'.(a) Convenient (b) Unknown(c) Close (d) Remote

200. Find a word in the passage which means 'theowner of a business'.(a) Constructor (b) Businessman(c) Entrepreneur (d) Proprietor

yoursmahboob.w

ordpress.com

241SSC CGL SolvedPaper

1. (a) F Þ 6 ; (6)3 = 216Similarly,L Þ 12 ; (12)3 = 1728

2. (b) As,2

1

2

1

2

M K

O P

U S

S T

E C

-

+

-

+

-

¾¾®

¾¾®

¾¾®

¾¾®

¾¾®Similarly,

2

1

2

1

2

L J

I J

G E

H I

T R

-

+

-

+

-

¾¾®

¾¾®

¾¾®

¾¾®

¾¾®3. (a) Tanning is the art of manufacturing leather.

Similarly, Pyrotechnics is the art of manufacturingfireworks.

4. (c) A king sits in a throne. Similarly, a rider sitson a saddle on the back of a horse.

5. (d)264 132

2=

1 + 3 + 2 = 6

870 2903

=

2 + 9 + 0 = 11Similarly,

735 1475

=

1 + 4 + 7 = 126. (d) Meaningful order :

3. Requirements Analysis ¯2. Conceptual Modelling ¯4. Logical Modelling ¯

6. Schema Refinement ¯5. Physical Model ¯1. Implementation

7. (d) b a a / b b b / b a a / b b b

8. (d) a b c a b / b c a b c / c a c a / a b9. (c) Except Influenza, all others are diseases

caused by deficiency of vitamins while, influenzais caused by virus.

10. (c) Except Raisin, all others are different formsof precipitation.

11. (c) 248 – 231 = 17563 – 547 = 1671 – 55 = 16523 – 517 = 06

Except in the option (c), in all other number pairs,the difference between the two numbers is aneven number.

12. (c) 10 Þ 1 + 0 = 113 Þ 1 + 3 = 4234 Þ 2 + 3 + 4 = 9

681 Þ 6 + 8 + 1 = 15997 Þ 9 + 9 + 7 = 25Consecutive square numbers are given.

13. (d) Except in the word Led, in all other wordsthere are two vowels.

14. (a) C E G I K M3 5 7 9 11 13

+2 +2 +2 +2 +2

+2 +2 +2 +2 +2

15. (b) 5 × 3 + 1 = 1616 × 3 + 3 = 5151 × 3 + 5 = 158158 × 3 + 7 = 481

16. (a) As,9 × 2 : 9 × 9=18 : 81

Similarly,9 × 5 : 9 × 645 : 54

The digits of resultant have interchanged positions.

HINTS & SOLUTIONS

yoursmahboob.w

ordpress.com

242 SSC CGL SolvedPaper

17. (a) 5 3

5 3

5 3

A F I

J O R

M R U

+ +

+ +

+ +

¾¾® ¾¾®

¾¾® ¾¾®

¾¾® ¾¾®Similarly,

5 3H M P+ +¾¾® ¾¾®18. (d) bc ® cde ® de ® efg ® fg ® ghi

19. (d)

Cousin

Man's sister Man's wifeMan

Boy Son

20. (c) 24 * 16 * 8 * 32Þ 24 + 16 – 8 = 32Þ 40 – 8 = 32

21. (d) Prem Kesav Simarn

Sunil maruti SitaHence, sunil is the cousin of Maruti.

22. (b) Priti > RahulRahul > Yamuna = DivyaManju > LokitaDivya > ManjuNow, Priti > Yamuna = Divya > Manju > LokitaTherefore, Lokita scored the lowest.

23. (a) ‘TALENT’ can not be formed from the givenword CONSTITUTIONAL as there is no ‘E’ letterin the given word.

24. (c) ‘MASTER’ can not be formed as there isno ‘E’ letter in the given word.

25. (d) ‘NATION’ can not be formed as there is onlyone ‘N’ in the given word SEGREGATION.

26. (c) 1 2 3 4 5 6 7 8 9 10 11 12 13 14C O M M U N I C A T I O N SAfter rearrangementO C M M N U C I T A O I S N

10th from right27. (b) 8 × 20 ̧ 3 + 9 – 5 = 38

Þ 8 × 20 ¸ 5 + 9 – 3 = 38Þ 8 × 4 + 9 – 3 = 38Þ 32 + 9 – 3 = 38

28. (a) 33 × 11 ̧ 3 – 6 = 115363 6 1153

æ öÞ - =ç ÷è øÞ 121 – 6 = 115

29. (d) 15 * 24 * 3 * 6 * 17Þ 15 + 24 ̧ 3 – 6 = 17Þ 15 + 8 – 6 = 17

30. (b) L Þ 12 × 2 = 24A Þ 01 × 2 = 02D Þ 04 × 2 = 08Y Þ 25 × 2 = 50Total = 84

31. (b) L Þ 12 + 8 = 20E Þ 5 + 8 = 13A Þ 1 + 8 = 9D Þ 4 + 8 = 12E Þ 5 + 8 = 13R Þ 18 + 8 = 26Therefore,L Þ 12 + 8 = 20I Þ 9 + 8 = 17G Þ 7 + 8 = 15H Þ 8 + 8 = 16T Þ 20 + 8 = 28

32. (c) First Column (3 × 4 × 5) – 2 = 58Second Column (5 × 6 × 2) – 2 = 58Third Column (8 × 4 × 2) – 2 = 62

Forth Column (7 × 6 × 3) – 2 = 124

33. (b) First number Third number´= Second number in each column.First Column

4 9 36 6´ = =Second Column

3 27 81 9´ = =Third column

2 ? 10´ =Þ 2 × ? = 100

100? 502

\ = =

34. (c) 84 ¸ 12 = 7 and 7 × 2 = 1481 ¸ 9 = 9 and 9 × 2 = 1888 ¸ 11 = 8 and 8 × 2 = 16

35. (b)

A

B

CD

E Raj'sstarting point

F

It is clear from the diagram that Raj is in North-West direction.

yoursmahboob.w

ordpress.com

243SSC CGL SolvedPaper

36. (d) Boy’s starting point

A B

CDEF

G

3 km

2 km1 km

4 km

8 km

North

West East

South

NW

SW SE

NE

He is 5 km away from the starting point.37. (d) It is mentioned that medicines are needed

for sick people. So, healthy people do not needmedicines.

38. (c) First Premise is Particular Affirmative(I-type).

CenturiesYears

Decades

CenturiesYears

Decades

Or

Conclusions I - FalseConclusions II - TrueConclusions III - FalseConclusions I and III form Complementary Pair.Therefore, either I or III follows.

39. (c) 10 cubes are visible and 10 cubes are hidden.Clearly, there is one column having four cubes.There are two columns each having three cubes.There are three columns, each having two cubes.There are four columns, each having only one cube.Thus, total number of cubes= 4 + 6 + 6 + 4 = 20 cubes

40. (c) Sea is a part of Earth. Sun is the star.

Earth

Sea Sun

41. (c) It is difficult to balance the ball and the jarin the position as shown in Answer figure (c)

42. (c)

43. (a) The required region should be present onlyin circle. Such region is marked ‘10’.

44. (b) The letters ‘b’ and ‘d’ are present in boththe circles.

45. (c)

46. (d)

««««

« «

««

47. (d)

48. (c) All the components of Question Figure arepresent in Answer Figure (c)

49. (b) B Þ 00, 12, 24, 31, 43E Þ 03, 10, 22, 34, 41S Þ 58, 65, 77, 89, 96T Þ 59, 66, 78, 85, 97

yoursmahboob.w

ordpress.com

244 SSC CGL SolvedPaper

50. (a)

××××

××××

××××

××××

––––

––––

––––

––––

51. (a) The demand for labour is “derived” fromthe production and demand for the productbeing demanded. If the demand for the productincreases, either the price will increase or thedemand for production labour will increase untilthe equilibrium price and production numbersare met.

52. (d) Investment expenditure refers to theexpenditure incurred either by an individual or afirm or the government for the creation of newcapital assets like machinery, building etc. Thepurchase of house cannot be considered asinvestment exienditure as it may be for personal use.

53. (a) For private corporate sector, retainedprofits adjusted for non operating surplus/ deficitis considered as its Net Saving. Retained profitsare those which are ploughed back into businessafter making commitments to depreciationprovision for various fixed assets, debts,government and to share-holders.

54. (c) The Speaker of Lok Sabha presides overthe joint sittings of both Houses of Parliament.The joint session of parliament is convened bythe President of India.

55. (c) Social accounting is a method by which afirm seeks to place a value on the impact onsociety of its operations. One social accountingsystem primarily attempts to measure NationalIncome, final product, consumption andaccumulation of capital.

56. (c) Forced saving is an economic situation inwhich consumers spend less than theirdisposable income, not because they want tosave but because the goods they seek are notavailable or because goods are too expensive.

57. (c) Parliament may restrict the application ofthe Fundamental Rights to members of the IndianArmed Forces and the Police, in order to ensurepaper discharge of their duties and the maintenanceof discipline, by a law made under Article 33.

58. (b) Pulakesin II (610 – 642 AD) was the mostfamous ruler of the Chalukya dynasty. In his reignthe Chalukyas of Badami saw their kingdomextend over most of the Deccan.

59. (b) An inscription of the 8th century AD atUttaramerur temple describes the constitution ofthe local council, eligibility and disqualificationsfor the candidates, the method selection, theirduties and delimits their power in Chola dynasty.

60. (b) The Pitt’s India Act of 1784 was in thenature of a regulating act as it was intended toaddress the shortcomings of the Regulating Actof 1773 by bringing the East India Company’srule in India under the control of the BritishGovernment.

61. (b) Allan Octavian Hume was a civil servant,political reformer and amateur ornithologist andhorticulturalist in British India. He was one ofthe founders of the Indian National Congress.Thought it was he who emerged as the founderof the Congress, it had somehow neverhappened to Hume to preside over a plenarysession even once.

62. (a) The principle of separation of powers isthe foundation for a democratic state based onthe rule of law. The judicial power dispensesjustice in disputes between cit izens andgovernment and its agencies. The presence ofan independent judiciary in a democraticgovernment distinguishes that system from atotalitarian one.

63. (b) Dr. B R Ambedkar, the chairman of theDrafting committee called the fundamental rightto constitutional remedies as the heart and soulof the Indian constitution. According to this right,a person can move the Supreme Court in case ofviolation of their fundamental rights. In theConstitution, this right is enshrined in Article 32.

64. (b) Machiavelli has been credited withformulating for the first time the “modern conceptof the state”. The term ‘lo state’ appears widelyin Machiavelli’s writings, especially in The Prince,in connection with the acquisition andapplication of power in a coercive sense.

65. (a) Narmada is the only river in India that flowsin a rift valley, flowing west between the Satpuraand Vindhya ranges. The Tapti River and MahiRiver also flow through rift valleys, but betweendifferent ranges.

66. (b) An Isthmus is a narrow strip of landconnecting two larger land areas, usually withwater on either side.

67. (a) Coral reefs are underwater structures madefrom calcium carbonate secreted by corals. Coralreefs are colonies of tiny animals found in marinewaters that contain few nutrients.

yoursmahboob.w

ordpress.com

245SSC CGL SolvedPaper

68. (b) The Apatanis area tr ibal group ofapproximately 60,000 people living in the Zirovalley in the Lower Subansiri district ofArunachal Pradesh. Their language belongs tothe Sino-Tibetan family.

69. (a) Phytochrome is a photoreceptor, a pigmentthat plants use to detect light. It is a protein witha bilin chromophore. It detects mainly red andfared region of the visible spectrum and regulatesgermination of seeds.

70. (c) Quenching is a process in which absorbedlight energy is dissipated as heat and does nottake part in photochemistry. The phenomenoninvolves quenching of chlorophylla (Chla)fluroescence, which is induced under steady-stateillumination.

71. (c) AIDS virus destroys the T-cells inside ofthe immune system. T cells or T lymphocytesbelong to a group of white blood cells known aslymphocytes, and play a central role in cell –mediated immunity.

72. (b) The Telangana Movement was the biggestpeasant guerrilla war of modern Indian historyaffecting 3000 villages and 3 million population.The uprising began in july 1946 and was at itsgreatest intensity between August 1947 andSeptember 1948.

73. (d) A topographic map is a type of mapcharacterized by large scale detail andquantitative representation of relief, usuallyusing contour lines in modern mapping, buthistorically using a variety of methods.

74. (a) Bragg Spectrometer is an instrument usedto analyze crystal structure by using X– rays. Init, a beam of collimated X–rays strikes the crystal,and a detector measures the angles andintensities of the reflected beam.

75. (a) Alpha particles consist of two protons andtwo neutrons bound together into a particleidentical to a helium nucleus, which is generallyproduced in the process of alpha decay.

76. (b) Mark Elliot Zuckerburg is an Americancomputer programmer and internet entrepreneur.He is best known as one of five co-founders ofthe social networking website Facebook.

77. (a) Looping is used in programming to savethe tedium of many repetitive tasks. One of themain uses of loops in programs is to carry outrepetitive tasks.

78. (c) The human body requires more calciumthan any other mineral. At least 99% of thecalcium is found in the bones and teeth, givingthem strength and rigidity.

79. (a)80. (a) The sigmoid colon is the part of the large

intestine. It forms a loop that averages about 40cm in length.

81. (d) Electrons carry current in a good conductorof electricity and they are negatively charged.This makes a good electrical conductornegatively charged.

82. (a) Magnetic declination is the angle betweencompass north (the direction the north end of acompass needle points) and true north (thedirection along the earth’s surface towards thegeographic North Pole.

83. (c) Containment of toxic or hazardousconstituents in industrial solid wastes iscurrently being accomplished using: (i) coatinggrains of waste material with an inert and non-reactive impervious material (microencapsulation),and (ii) coating blocks of waste material with aninert, non-reactive impervious material(microencapsulation).

84. (b) UV radiation can be an effective viricideand bactericide. Disinfection using UV radiationis commonly used in wastewater treatmentapplications and is finding an increased usagein drinking water treatment.

85. (b) When cells are exposed to sunlight, radiantenergy can damage the DNA. For example,ultraviolet irradiation cause covalent bondformation between adjacent thymines on thesame strand of DNA. Ultraviolet light is absorbedby a double bond in thymine and cytosine basesin DNA.

86. (b) World No. 1 Novak Djokovic of Serbiadefeated US open champion Andy Murray infour sets to win his fourth and third consecutiveAustralian open title on 28 January 2013. Djokovicalso became the first man in the Open era to winthree straight Australian titles.

87. (a) In order to manufacture silicones, alkyl-substituted chlorosilanes are used as startingmaterial. Since dimethylsilane contains – OHgroup at the end of the chain, polymerizationand chain length increases. However, thehydrolysis of alkyl trichlorosilane gives verycomplex cross linked polymer.

88. (b) Examples of natural coloids can be foundin our body itself : blood consists of colloidalsized red blood corpuscles (RBC) which providethe vital oxygen to all the body tissues.

yoursmahboob.w

ordpress.com

246 SSC CGL SolvedPaper

89. (a) Nickel silver, also known as German silver,is a copper alloy with nickel and often zinc. Theusual formulations is 60% copper, 20% nickeland 20% zinc. Nickel silver is named for its silveryappearance, but it contains no elemental silverunless plated.

90. (d) Atomic absorption spectroscopy is ananalytical chemistry technique used fordetermining concentration of particular metalelement and is widely used in pharmaceutics. Itwas used for the determination of cobalt inVitamin B12.

91. (b) Nitrosomonas is a genus comprising rodshaped chemoautotrophic bacteria. This rarebacteria oxidizes ammonia into nitrite as ametabolic process.

92. (b) Mark Tully is the author of No Full Stopsin India (1998). Its collection of journalisticessays, was published in the US as The Defeatof a Congress-man.

93. (b) Tahrir Square, also known as “ MartyrSquare”, is a major public town square in Downtown Cairo. Tahrir Square was the focal point ofthe 2011 Egyptain Revolution against formerpresident Hosni Mubarak.

94. (b) There are 10 non-permanent members in theUnited Nations Security Council, with fiveelected each year to serve two-year terms.

95. (b) Justice D.K Jain, Judge, Supreme Court ofIndia, is the Chairman of the Twentieth LawCommission of India. The Twentieth LawCommission was constituted through aGovernment Order with effect from 1st September,2012.

96. (b) Julia Eileen Gillard is the 27th and currentPrime Minister of Australia and the Leader of theAustralian Labor Party since 24 June 2010. Sheis the first women to hold the office.

97. (a) Chhattisgarh was the first state in thecountry to introduce the Food Security Act, aunique law so that it could become a right of thepeople to get sufficient quantity of food atsubsidized rates.

98. (b) Christopher Martin - Jen - kins was a Britishcricket journalist and a past President of theMCC. He was also a commentator for Test MatchSpecial (TMS) on BBC Radio from 1973 untildiagnosed with terminal cancer in January 2012.

99. (c) The Nobel Peace Prize 2012 was awardedto European Union (EU) “for over six decadescontributed to the advancement of peace andreconciliation, democracy and human rights inEurope”.

100. (b) Pratibha Ray is an Indian academic andwriter who won the Jnanpith Award in 2011. Shewas the first woman to win the MoortideviAwards in 1991.

101. (c) HCF of two-prime numbers = 1\ Product of numbers = their LCM = 117

102. (c) Side of square = x unitsDiagonal of square = 2x units

Radius of smaller circle =x2

units

Radius of larger circle

2x x units2 2

= =

\ Required ratio of areas2 2x x:

4 2p= p = 2 : 4 = 1 : 2

103. (c) Surface area of sphere = 4pr2

Þ r2 = 2 Þ r Þ 2 units\ Volume of sphere

( )334 4r 23 3= p = p´

8 2 cubic units3

= p

104. (b) Volume of water in conical flask 21 r h3= p

If the height of water level in cylindricalflask be H units, then

2 2 21m r H h3

p = p

2

2 2 21 r h hH .3 m r 3mpÞ = =p

105. (b) (A + B)’s 1 day’s work = 1

12

(B + C)’s 1 day’s work = 120

(C + A)’s 1 day’s work =1

15On adding all three,2 (A + B + C)’s 1 day’s work

1 1 112 20 15

= + +

yoursmahboob.w

ordpress.com

247SSC CGL SolvedPaper

5 3 4 160 15

+ += =

\ (A + B + C)’s 1 day’x work 1

10=

Hence, the work will be finished in 10 days.

106. (a) (A + B)’s 2 days’ work 23

=

Remaining work 2 113 3

= - =

Time taken by A in doing 13 work = 2 days

\ Time taken by A in completing the work= 6 days.

\ B’s 1 day’s work = 1 13 6- =

2 16-

=16

\ B alone will complete the work in 6 days.107. (a) Efficiency of A and B = 4 : 5

Ratio of respective time = 5 : 4\ Time taken by B

4 15 65 2= ´ = hours

108. (b)3 3 4 124 4 4 16

´= =´

3 68 16

=

6 7 8 9 10 11 12, , , , , ,16 16 16 16 16 16 16\

\ Required rational number 9

16=

109. (a) 7x – 5x = 200Þ 2x = 200 Þ x = 100\ Price of a pair of shoes = 5x = 500

110. (d) Q = P + 30 Þ Q – P = 30 andR – Q = 60 = 2 × 30\ Required ratio = 2 : 3 : 5Look : 3 – 2 = 1, 5 – 3 = 2

111. (a) The sixth number = 9 × 50 – 5 × 54 – 3 × 52= 450 – 270 – 156 = 24

112. (a) Required average( )3 1 2 3 ....... 9

9+ + + +=

9 10 152 3´= =´

113. (b) C.P. of the article100 300120= ´ = ` 250

On selling at ̀ 235,

Loss per cent15 100 6%250= ´ =

114. (a) After taking away respective balls,Number of balls in the box= 75 + 25 + 50 = 150\Percentage of black balls

50 100150= ´

100 133 %3 3

= =

115. (d) Q S.P. of a dozen pairs of socks

´= =

180 80

` 144

100

\ S.P. of 1 pair of socks

= =144

` 12

12

\ No of pairs available for

` 48 = 48 412

=

116. (a) Discount = 12000 – 10500 = ̀ 1500If the discount per cent be x,then

12000 x 1500100´ =

1500 100x 12.5%12000´Þ = =

117. (b) If the C.P. of radio be ` x,then

x 108 480 90 432100 100´ ´= =

´Þ = =

432 100

x ` 400

108

If no discount is allowed,Gain = 480 – 400 = ̀ 80Gain per cent

80 100 20%400= ´ =

yoursmahboob.w

ordpress.com

248 SSC CGL SolvedPaper

118. (b)

A

P

BN O

AB = 14 cm, PB = 12 cmÐAPB = 90°

2 2AP 14 12\ = -

( )( )14 12 14 12= + - 26 2 52= ´ =ON = x \ AN = 7 – x ; BN = 7 + x\ From D PAN, PN2 = AP2 – AN2

\ 52 – (7 – x)2

\ From D PNBPN2 = (12)2 – (7 + x)2

\ 52 – (7 – x)2 = 144 – (7 + x)2

Þ 52 – (49 – 14x + x2) = 144 – (49 + 14x + x2)Þ 52 – 49 + 14x – x2 = 144 – 49 – 14x – x2

Þ 28x = 144 – 52 = 92

92 23x28 7

Þ = =

\ BN = 7 + x

23 49 23 7277 7 7

+= + = = 210 cm7

=

119. (b)

Crossing

V1 40

V 250

If1 2

40 50V V

= then they will collide i,e. cars will

reach at the same time.

1

2

V 40 4V 50 5

\ ¹ =

120. (d) If the original speed be S1 units and time =t1 units and distance = D, then

21

D2 S

2t=

2 11 1

D DS and S4t t

\ = =

1 1

21

DS t 4

DS 14t

\ = =

121. (c) S.I.Pr incipal Time Rate

100´ ´=

4000 3 x100´ ´\

5000 2 12100´ ´=

5 2 12x4 3´ ´Þ =´

= 10% per annum122. (c) x2 – 3x + 1 = 0

Þ x2 + 1 = 3xDividing both sides by x,

1x 3x

Þ + =

22

1 1x xx x

\ + + +

22

1 1x xxx

æ ö æ ö= + + +ç ÷ç ÷ è øè ø21 1x 2 x

x xæ ö æ ö= + - + +ç ÷ ç ÷è ø è ø

= 9 – 2 + 3 = 10

123. (a) 4x 3 4y 3 4z 3 0x y z- - -+ + =

4x 3 4y 3 4z 3 0x x y y z z

Þ - + - + - =

3 3 3 4 4 4 12x y z

Þ + + = + + =

1 1 1 12 4x y z 3

Þ + + = =

124. (d) Expression

2 50 18 72= + -

2 2 5 5 3 3 2 2 2 2 3 3= ´ ´ + ´ ´ - ´ ´ ´ ´

10 2 3 2 6 2= + -

7 2 7 1.414 9.898= = ´ =

yoursmahboob.w

ordpress.com

249SSC CGL SolvedPaper

125. (b) a2 + b2 + 4c2 = 2a +2b – 4c – 3Þ a2 + b2 + 4c2 – 2a –2b + 4c + 3 = 0Þ a2 – 2a + 1 + b2 – 2b + 1 + 4c2 + 4c + 1 = 0Þ (a –1)2 + (b – 1)2 + (2c + 1)2 = 0\ a – 1 = 0 Þ a = 1;b – 1 = 0 Þ b = 1;

2c + 1 = 0 Þ c 12

= -

2 2 2 1a b c 1 14

\ + + = + +

124

=

126. (b) Here, the first divisor (289) is a multiple ofsecond divisor (17).\ Required remainder = Remainder obtained ondividing 18 by 17 = 1

127. (a) ax + by +c = 0When c = 0,ax + by = 0

aby ax y xb

= - Þ = -

When x = 0, y = 0 i,e. this line passes throughthe origin (0, 0).

128. (d) Original fraction x 4

x-=

In case II,8 (x – 4 – 2) = x + 1Þ 8x – 48 = x + 1Þ 7x = 49 Þ x = 7

Original fraction7 4 3

7 7-= =

129. (d) x2 = y + zÞ x2 + x = x + y + zÞ x (x + 1) = x + y + z

x y zx 1x

+ +Þ + =

Þ 1 x

x 1 x y z=

+ + +

Similarly, 1 y

y 1 x y z=

+ + +

1 zz 1 x y z

=+ + +

1 1 11 x 1 y 1 z\ + +

+ + +

x y zx y z x y z x y z

= + ++ + + + + +

x y z 1x y z

+ += =+ +

130. (b)

A B

C

D

ÐA = 90°, ÐC = 55°\ ÐB = 90° – 55° = 35°ÐADB = 90°\ ÐBAD = 90° – 35° = 55°

131. (a) A

B C

G

DBGC1 ABC3= ´D

1 48 16 sq. cm.3= ´ =

132. (d) A

B C

D

Here, AC and BD are chords of the circle.\ AP . CP = BP . DP

yoursmahboob.w

ordpress.com

250 SSC CGL SolvedPaper

133. (d)3tan4

q =

4cot3

\ q =

2 2cosec cot 1q - q =Q

2cosec 1 cotÞ q = + q

24 16 251 1

3 9 9æ ö= + = + =ç ÷è ø

53

=

134. (d) Expression

2 21 1

1 tan 1 cot= ++ q + q

2 21 1

sec cosec= +

q q= cos2q + sin2q = 1

135. (b) P

Q

O

R

ÐQOR = 110°ÐOPR = 25°\ ÐQPR = 110° ¸ 2 = 55°OR = OP\ ÐOPR = ÐPRO = 25°

\ ÐOQR = ÐORQ 70 352

= = °

\ ÐPRQ = 25° + 35° = 60°

136. (d) Height of towerLength of stick

Length of shadow of towerLength of shadow of stick

=

h 4012 8

Þ =

40 12h 60 metre8´Þ = =

137. (b)

A

B

E

D

C

\ ÐBEC = 130°\ ÐDEC = 180° – 130° = 50°\ ÐEDC = 180° – 50° – 20° = 110°\ ÐBAC = ÐEDC = 110°(Angles on the same arc)

138. (d) Triangle will be equilateral.139. (d)

A

B C

D

E

36 metre24

met

re

60°

DE = 36 – 24 = 12 metreFrom DADE,

DEsin 60AD

° =

3 122 AD

Þ =

12 2AD 8 3metre3´Þ = =

140. (a) tan a = n tan b

Þ tan b 1n

= tan a

Þ cot b n

tan=

a and

sin a = m sin b Þ sin b 1m

= sin a

Þ coses b m

sin=

a\ cosec2b – cot2b = 1

2

2 2m n2 1

sin tanÞ - =

a a

yoursmahboob.w

ordpress.com

251SSC CGL SolvedPaper

2 2 2

2 2m n cos 1

sin sinaÞ - =

a a2 2 2

2m n cos 1

sin- aÞ =

aÞ m2 – n2 cos2 a = sin2a= 1 – cos2aÞ m2 – 1 = n2 cos2 a – cos2a= (n2 – 1) cos2a

22

2m 1cosn 1-Þ a =-

141. (d) Expression1 1

cosec cot sin= -

q- q q2 2cosec cot cosec

coses cotq- q= - qq- q

= coses q – cot q – coses q= cot q

2 2 1cosec cot 1; cosecsin

é ùq = q = = qê úqë û

142. (b) cos q + sin q = 2 cos qOn squaring both sides,cos2q + sin2q + 2cos q. sin q= 2 cos2 qÞ cos2q – sin2q = 2cosq. cos qÞ (cos q + sin q) (cos q – sin q)= 2 sin q. cos qÞ 2 cos q (cosq – sinq)= sin q. cos qÞ cos q – sin q

2sin .cos 2 sin2 cosq q= = qq

143. (a) cos4q – sin4q 23

=

2 2 2 2 2(cos sin )(cos sin )3

Þ q+ q q - q =

2 2 2cos sin3

Þ q- q =

2 2 21 sin sin3

Þ - q- q =

2 21 2sin3

Þ - q =

144. (b) Total students in 2001 – 2002 = 1350\ Required percentage

400 100 29.61350= ´ =

145. (a) Total students in 2003 – 04 = 1600\ Required percentage

250 100 15.6%1600= ´ =

146. (d) Required percentage increase

600 400 100 50%400-= ´ =

147. (c)135 810

x 360=

x = 135 360

810´

60= ° which is in english148. (*) English + physics + Social Science = 200°

Maths + Chemistry = 160°Required percentage

40 100 25%160= ´ =

149. (c) Difference of corresponding angles :Physics and Chemistry = 85 – 70 = 15°Chemistry and Social science = 70 – 55 = 15°=150

150. (b) Sum of corresponding angles of Maths andChemistry

= 90 + 70 = 160°Sum of corresponding angles of Physics

and Social Science= 85 + 55 = 140°Difference = 20°

360 810° ºQ

81020 20 45360

\ ° º ´ =

151. (a) The sentence shows subjunctive mood ofVerb (desire). Hence, even a singular subjectagrees with a plural verb. Hence, If I were heshould be used.

152. (a) It is not proper to use ‘to’ after advise (Verb).153. (d)154. (a) Know is generally not used in progressive

Tenses. Hence, we known/we have known shouldbe used here.

yoursmahboob.w

ordpress.com

252 SSC CGL SolvedPaper

155. (b) Neither of/Either if agrees with a singularverb. Hence, is sensible enough should be used.

157. (b) Tie-up = an agreement between twocompanies to join together.

158. (a) Slip into = to go in earlier situation.159. (c) Here, Past Simple should be used.160. (a) Come of = to be the result of something.161. (c) The word Citadel (Noun) means : a castle

on high ground where people could go whenthe city was being attacked, fortress.

162. (d) The word Stern (Adjective) means : strict ;harsh’ serious and difficult.

163. (c) The word Aberration (Noun) means :deviation; unusual action.

164. (b) The word Dwindle (Verb) means : tobecome gradually less or smaller; decrease;diminish.

165. (a) The word Dormant (Adjective) meansinactive.

166. (a) The word Tranquility (Noun) means :peace, serenity.

Disturbance = a situation in which peoplebehave violently in a public place.

167. (a) Let sleeping dogs lie = to avoid mentioninga subject that happened in the past, in order toavoid any problems or argument

168. (d) Born with a silver spoon in your mouth =having rich parents

169. (c) A man of straw = a man of no substance;an ordinary man

170. (b) Look a gift horse in the mouth = to refuseor criticize something that is given to you fornothing

171. (d) A cakewalk means something that isextremely easy to do

172. (c) Here, Past Simple should be used.I wish I were taller.I only wish I knew!

173. (b) It is an error related to position of words.174. (c) Raise = to lift or move something to a higher

level.Rise = to move upwardSmoke was rising from the Chimney.

175. (a) Have = had (Past) = had (Past Participle)176. (b) 177. (d)178. (a) Here, contrast is evident.179. (d)180. (c) No need of double subjects.181. (c) Knock down = to destroy a building; hit

somebody.Put down = demolish; destroy a building

completely.182. (d) 183. (c) 184. (a) 185. (b) 186. (b)187. (a) 188. (c) 189. (c) 190. (a) 191. (b)192. (b) 193. (b) 194. (c) 195. (a) 196. (b)197. (d)198. (a) Fish out of water = A person who feels

uncomfortable or awkward.199. (d) 200. (d)

yoursmahboob.w

ordpress.com

253SSC CGL SolvedPaper

PART-A : GENERAL INTELLIGENCE & REASONING

Directions (Qs. 1-3) : In a given series, with one termmissing. Choose the correct alternative from the givenones that will complete the series.1. AKU, FPZ, ?, PZJ, UEO, ZJT

(a) JUE (b) KVE(c) KUE (d) JVE

2. MRS, LTU, KVW, ?(a) TQR (b) MOP(c) JXY (d) CDE

3. B, G, K, N, ?(a) P (b) O(c) H (d) L

Directions (Qs. 4-10) : Select the missing numberfrom the given responses.4. 1944, 108, ?, 6, 3

(a) 16 (b) 18(c) 11 (d) 12

5. 251 (12) 107381 (?) 125(a) 14 (b) 24(c) 11 (d) 16

6. 354 (110) 526297 (?) 2631(a) 128 (b) 116(c) 135 (d) 143

7. 0, 7, 26, 63, ?(a) 125 (b) 126(c) 217 (d) 124

8. 8 10 125 7 912 14 ?(a) 16 (b) 15(c) 18 (d) 17

9. 4 9 23 5 78 1 ?(a) 9 (b) 6(c) 15 (d) 14

10. 2, 5, 10, 19, 36, ?(a) 70 (b) 71(c) 68 (d) 69

11. If 2 = 5, 4 = 18, 6 = 39, 8 = 68 then 10 = ?(a) 100 (b) 105(c) 110 (d) 116

12. If the day to day-after-tomorrow is Sunday, whichday was day-before-yesterday?(a) Thursday (b) Wednesday(c) Tuesday (d) Monday

13. Asha drives 6 km towards West and turns to theright and drives 3 km. Then, she turns again anddrives towards right hand and drives 6 km. Howfar is she from her starting point? In whichdirection would she be driving?(a) 6 km East (b) 3 km West(c) 3 km East (d) 6 km North

14. Five coaches P, L, R, M, O are in a row. R is to theright of M and left of P. L is to the right of P andleft of O. Which coach is in the middle?(a) P (b) L (c) R (d) O

Directions (Qs. 15-17) From the given alternativesselect the word which cannot be formed using theletters of the given word.15. INVESTIGATION

(a) INSTIGATION (b) GESTATION(c) VEST (d) STIGMA

16. SANCTION(a) ACTION (b) NATION(c) NOTION (d) NION

17. RESEARCH(a) SEARCH (b) REACH(c) HEAR (d) READ

18. If each of the letter in the English alphabet isassigned odd numerical value beginning codingA = 1, B = 3 & so on, what will be the total valueof the letter of the word ‘SNAKE’?(a) 95 (b) 105 (c) 115 (d) 113

19. If DFIN is coded as WURM, then HJMO can becoded as(a) RPNO (b) SQNP (c) SQNL (d) TRPO

20. If RUMOUR can be written as QSJKPL, then howHERMIT can be written?(a) GEPKHR (b) GCOIDN(c) GCPIDM (d) GCPIEN

21. If ‘–’ stand for addition, ‘+’ stands for subtraction,‘÷’ stands for multiplication and ‘×’ stands fordivision, then which one of the followingequations is correct?(a) 25 × 5 ÷ 20 – 27 + 7 = 120(b) 25 + 5 × 20 – 27 ÷ 7 = 128(c) 25 + 5 – 20 + 27 × 7 = 95(d) 25 – 5 + 20 × 27 ÷ 7 = 100

SSC Combined Graduate Level (CGL) Solved PaperMORNING SHIFT 19.05.2013

yoursmahboob.w

ordpress.com

254 SSC CGL SolvedPaper

22. In the following question, some relationship havebeen expressed through symbols which are× = greater than q = not less than÷ = less than b = not greater than+ = equal to f = not equal to,then A q B × C implies

(a) B q C (b) A ÷ C(c) A f C (d) B b C

23. If 63 – 30 = 30, 72 – 10 = 40, then 81 – 60 = ?(a) 50 (b) 35(c) 15 (d) 20

24. Priya starts walking in the afternoon facing theSun. After some time, she turned to the right.Later again, she tunred to her left and again alsoleft. At what direction is Priya moving now?(a) East (b) West(c) North (d) South

25. A direction pole was situated on the crossing.Due to an accident, the pole turned in such amanner that the pointer which was showing Eaststarted showing South. One traveller went to thewrong direction thinking it to be West. In whatdirection actually was he travelling?(a) South (b) East(c) West (d) North

26. The digits are given as below:562, 871, 438, 753If the position of the first and the third digits ofeach of the numbers are interchanged, which ofthe following will be the sum of the first and thesecond digits of the third highest number?(a) 9 (b) 7 (c) 6 (d) 8

Directions (Qs. 27-30) Select the related words/numbers from the given alternatives.27. A + B + Y + Z = 54

C + D + W + X = ?(a) 45 (b) 54(c) 56 (d) 52

28. Petrology : Rocks : : Palaeontology : ?(a) Birds (b) Animals(c) Fossils (d) Soil

29. Melt : Liquid : : Freeze : ?(a) Crystal (b) Solid(c) Ice (d) Evaporate

30. 8 : 27 : : ? : 81(a) 24 (b) 34 (c) 44 (d) 54

31. Which one set of letters when sequentiallyplaced at the gaps in the given letter series shallcomplete it?- - b a a b a a - a a -(a) b b a a (b) b a a b(c) a a b b (d) a b a b

Directions (Qs. 32-35) Find the odd words/letters/number pair from the given alternatives.32. (a) Fervent (b) Enthusiastic

(c) Apathetic (d) Ardent33. (a) FAA (b) OFF

(c) ATT (d) EPP34. (a) 117 – 143 (b) 142 – 156

(c) 64 – 78 (d) 103 – 16935. (a) Radio (b) Television

(c) Transistor (d) Tube36. Find the number which does not fit in the following

series:49, 81, 123, 156, 221(a) 81 (b) 123(c) 156 (d) 49

Directions (Qs. 37-38) In the following questions two/three statements are given followed by two/threeconclusions respectively. You have to consider thestatements to be true even if they seems to be atvariance from commonly known facts. You have todecide which of the given conclusions, if any, followfrom the given statements.37. Statements:

I. Some towers are windows.II. All windows are houses.III. Some houses are temples.Conclusions:I. Some towers are temples.II. Some houses are towers.(a) Only conclusion I follows.(b) Only conclusion II follows.(c) Both conclusions I & II follow.(d) Neither conclusion I nor II follows.

38. Statements:I. Man can find ultimate reality by dedication

to God through meditation.II. Meditation is the process to enrich the peace

of mind.Conclusions:I. Meditation is the process to realize the God.II. Meditation does not help to find out the

Mukti of Soul.III. Perfect dedication to God is essential to

realize the existence of ultimate reality.(a) Only conclusion I follows(b) Only conclusion II & III follow.(c) Only conclusion I and III follow.(d) All conclusions follow.

yoursmahboob.w

ordpress.com

255SSC CGL SolvedPaper

39. Four views of a dice have been shown below,which of the following symbols is on the faceopposite to the face having the symbol ÷ ?

(a) $ (b) 0 (c) = (d) D40. Two positions of a dice are shown below. If 1 is

at the bottom, which number will be on top?

(a) 4 (b) 3 (c) 8 (d) 541. What comes next in the series?

Question Figures :

Answer Figures :

(a) (b) (c) (d)42. Which one of the following diagrams represents

the relationship among Delhi, Lucknow, UttarPradesh?

(a) (b)

(c) (d)

43. How many triangles are there in the given figure?

(a) 10 (b) 12 (c) 14 (d) 1144. Which one of the following diagrams represents

the correct relationship among day, week, year?

(a) (b)

(c) (d)

Directions (Qs. 45-47) : Which answer figure willcomplete the pattern in the question figure?45. Question Figures :

Answer Figures :

(a) (b) (c) (d)46. Question Figures :

Answer Figures :

(a) (b) (c) (d)47. Question Figures :

Answer Figures :

(a) (b) (c) (d)48. A piece of paper is folded and punched as shown

below in the question figures. From the givenanswer figures, indicate how it will appear whenopened.Question Figures :

Answer Figures :

(a) (b) (c) (d)

yoursmahboob.w

ordpress.com

256 SSC CGL SolvedPaper

49. Which of the answer figures is exactly the mirrorimage of the given figure when the mirror is heldat MN?Question Figures :

Answer Figures :

(a) (b) (c) (d)50. A word is represented by only one set of numbers

as given in any one of the alternatives. The setsof numbers given in the alternatives arerepresented by two classes of alphabets as intwo matrices given below. The columns and rowsof Matrix I are numbered from 0 to 4 and that ofMatrix II are numbered from 5 to 9. A letter fromthese matrices can be represented first by its rowand next by its column, e.g., 'G' can be representedby 04, 40 etc. and 'K' can be represented by 56, 75etc. Similarly, you have to identify the set for theword 'CHILD'.

Matrix I

0 1 2 3 40 C D E F G1 F G C D E2 D E F G C3 E F G C D4 G C D E F

Matrix II

5 6 7 8 95 H K L I N6 I N H K L7 K L I N H8 L I N H K9 N H K L I

(a) 24, 21, 99, 57, 01 (b) 12, 79, 99, 57, 01(c) 33, 57, 99, 57, 01 (d) 41, 79, 99, 57, 11

PART-B : GENERAL AWARENESS51. Reduction of nitrates to ammonia can be achieved

through one of the following methods :(a) in alkaline medium using Devarda’s alloy.(b) in neutral medium using Devarda’s alloy.(c) in acidic medium using Devarda’s alloy.(d) in neutral medium using Cupric oxide.

52. The Red Data Books published by theInternational Union for Conservation of Natureand Natural Resources enumerate(a) Biodiversity parks and wild life sanctuaries

in different countries.(b) Centres of origin of cultivated plants.(c) Centres of origin of all economically

important plants.(d) Threatened species of plants and animals.

53. Grits of sewage are removed in(a) Grit chamber (b) Detritus tank(c) Skimming tank (d) Trickling filter

54. Environmental impact assessment was firstformally established in 1969 in which country ?(a) United Kingdom (b) United States(c) France (d) Netherlands

55. The most affected sulphur containing amino acidby PAN is(a) Cysteine (b) Methonine(c) Proline (d) Globuline

56. Who won the Women Australian Open SinglesTitle in 2013?(a) Serena Williams (b) Victoria Azarenka(c) Li Na (d) Sharapova

57. The first non-Englishman elected as Chairman ofthe International Cricket Council was(a) Cydle Walcott (b) Gary Sobers(c) Imran Khan (d) Sunil Gavaskar

58. Which of the following has zero electron affinity ?(a) Oxygen (b) Fluorine(c) Nitrogen (d) Neon

59. For which language included in the IndianConstitution, the Jnanpith Award has not beengiven upto 2011?(a) Sanskrit (b) Sindhi(c) Kashmiri (d) Konkani

60. Who is the first British Author to win the ManBooker Prize for fiction twice ?(a) Peter Carey (b) J.M. Coetzee(c) Hilary Mantel (d) None of the above

61. Which one of the following is wrongly paired?Country Currency

(a) Japan Yen(b) Iran Rand(c) Bangladesh Taka(d) Bhutan Ngultrum

yoursmahboob.w

ordpress.com

257SSC CGL SolvedPaper

62. The two South American countries, which aremembers of the Organisation of PetroleumExporting Countries (OPEC) are(a) Ecuador and Brazil(b) Ecuador and Bolivia(c) Ecuador and Venezuela(d) Venezuela and Brazil

63. Who is the Chairperson of the National GreenTribunal ?(a) Markandey Katju (b) Swatantra Kumar(c) Satyananda Mishra (d) K.G. Balakrishnan

64. Which is the capital of Mali ?(a) Mopti (b) Barmako(c) Cairo (d) Nairobi

65. Which was the first film of Jaspal Bhatti ?(a) Power Cut(b) Mahaul Theek Hai(c) Thank You Jijaji(d) None of the three above

66. Who was the first recipient of the "Bharat Ratna"Award posthumously ?(a) K. Kamaraj (b) Lal Bahadur Shastri(c) M.G. Ramachandran(d) B.R. Ambedkar

67. An irrigation project is categorized as a majorproject if it covers a cultivable command area of(a) less than 2,000 hectares(b) 2,000 to 10,000 hectares(c) above 10,000 hectares(d) all the above

68. High powered money is(a) Banks reserves at Central Bank(b) All loans and advances of banks(c) Money held by banks(d) Currency held by public and reserves withthe Central Bank

69. Hardening interest rate means(a) interest rate is remaining sticky(b) interest rate is very low(c) interest rate is increasing(d) interest rate is falling

70. “Dumping” is a situation when the seller(a) supplies mote than the demand for products

in the World Market.(b) supplies more in the Domestic Market.(c) sells a commodity at a lower price in the

World Market and charges a higher price inthe Domestic Market.

(d) sells a commodity at a higher price in theWorld Market and charges a lower price inthe Domestic Market.

71. Which one of the following is not a function ofElection Commission ?(a) Allotment of symbols

(b) Fixation of election dates(c) Maintaining fairness of election(d) Selecting the candidates for election

72. ‘State is a necessary evil’ is associated with(a) Individualism (b) Idealism(c) Marxism (d) Constructivism

73. The programme of ‘Operation Flood’ wasconcentrated on(a) increasing irrigation facilities.(b) flood control.(c) increasing the milk production.(d) increase the flood grains production.

74. Which of the following is not associated withMarxism ?(a) Dialectical Materialism(b) Class Struggle(c) Welfare State(d) Surplus Value

75. Article 324 of the Indian Constitution deals withthe(a) imposition of President’s Rule in States.(b) appointment of Finance Commission.(c) powers and functions of the Chief Election

Commissioner.(d) functions of the Union Public Service

Commission.76. The founder of the Lodi Dynasty was

(a) Bahlol Lodi(b) Sikandar Shah Lodi(c) Jalal Khan Lodi(d) Ibrahim Lodi

77. The Gandharva School of art is also known asthe(a) Buddhist-Roman art(b) Dravidian-Roman art(c) Greco-Roman art(d) Greco-Roman-Buddhist art

78. The Bolshevik Revolution of Russia symbolizedfollowing:(a) coming of the Communist Rule(b) establishment of Republic of Russia(c) establishment of Democracy in Russia(d) overthrow of Romanov Dynasty

79. Which General, who commanded the Britishforces against the Americans in their War ofIndependence later became Governor-General ofIndia ?(a) Dalhousie (b) William Bentinck(c) Wellesley (d) Cornwallis

80. Who was the first President of the All India TradeUnion Congress (AITUC) ?(a) C.R. Das (b) V.V. Giri(c) Lala Lajpat Rai (d) Sarojini Naidu

yoursmahboob.w

ordpress.com

258 SSC CGL SolvedPaper

81. If there is a deadlock between Rajya Sabha andLok Sabha over an ordinary bill, it will be resolvedby(a) The President(b) The Council of Ministers(c) The Joint Session of Parliament(d) The Supreme Court

82. According to Ferrel’s law (Coriolis Force) windschange their direction(a) Towards left in Northern hemisphere and

towards right in Southern hemisphere.(b) Towards right in Northern hemisphere and

towards left in Southern hemisphere.(c) Towards right in both the hemisphere.(d) Towards left in both the hemisphere.

83. Which one of the following atmospheric layersabsorb ultraviolet rays of the sun?(a) Troposphere (b) Stratosphere(c) Ionosphere (d) Ozonosphere

84. The drainage pattern developed on foldedsedimentary rock is termed as(a) Trellis (b) Dendritic(c) Radial (d) Deranged

85. Which one of the following is not a line ofdemarcation between two countries ?(a) Durand Line (b) Mac Mahon Line(c) Plimsoll Line (d) Maginot Line

86. Water potential remains lowest in(a) Water plants (b) Woody plants(c) Succulents (d) Halophytes

87. The free living bacterium in the soil whichincreases the the yield of rice is(a) Rhizobium (b) Azotobacter(c) Acetobacter (d) Anabaena

88. The human body’s largest blood vessel is(a) Pulmonary artery (b) Aorta(c) Renal artery (d) Coronary artery

89. In human body, which one of the followinghormones regulates blood calcium and phosphate ?(a) Glucagon(b) Growth hormone(c) Parathyroid hormone(d) Thyroxine

90. Frontal cyclones occur characteristically in(a) Equatorial region(b) Tropical region(c) Mid-latitudinal region(d) Polar region

91. How do most insects respire ?(a) Through skin (b) Through gills(c) By tracheal system (d) By lungs

92. In nuclear reactions, there is conservation of(a) mass only(b) momentum only(c) energy only(d) mass, energy and momentum

93. When a particle and an antiparticle come incontact with each other, they(a) repell each other(b) annihilate each other(c) go undisturbed(d) spin about a common axis

94. Photoelectric effect is(a) an instantaneous process(b) delayed process(c) emission of protons(d) emission of neutrons

95. For a particle moving with a constant speed alonga straight line PQ, the hodograph is(a) a straight line parallel to PQ(b) a straight line perpendicular to PQ(c) a point(d) a circle

96. Which computer was the first to use the magneticdrum for memory ?(a) IBM - 650 (b) IBM - 7090(c) IBM - 701 (d) IBM - 360

97. Identify the FIFO (First In First Out) structureamong the following :(a) Stack (b) Queue(c) De-queue (d) Array

98. Aluminium is obtained by the electrolysis of pureAl2O3 dissolved in(a) Bauxite (b) Cryolite(c) Feldspar (d) Alumina

99. Complete hydrolysis of cellulose gives(a) D-fructose (b) L-glucose(c) D-glucose (d) L-fructose

100. Each body segment of Earthworm is called(a) Proglottid (b) Metamere(c) Scolex (d) Rostellum

PART-C : NUMERICAL APTITUDE

101. A, B, P are three points on a circle having centreO. If ÐOAP = 25° and ÐOBP = 35°, then themeasure of ÐAOB is(a) 120° (b) 60°(c) 75° (d) 150°

102. Side BC of DABC is produced to D. If ÐACD =140° and ÐABC = 3ÐBAC, then find ÐA.(a) 55° (b) 45°(c) 40° (d) 35°

yoursmahboob.w

ordpress.com

259SSC CGL SolvedPaper

103. The length of tangent (upto the point of contact)drawn from an external point P to a circle of radius5 cm is 12 cm. The distance of P from the centre ofthe circle is(a) 11 cm (b) 12 cm(c) 13 cm (d) 14 cm

104. ABCD is a cyclic quadrilateral, AB is a diameterof the circle. If ÐACD = 50°, the value of ÐBAD is(a) 30° (b) 40°(c) 50° (d) 60°

105. Two circles of equal radii touch externally at apoint P. From a point T on the tangent at P,tangents TQ and TR are drawn to the circles withpoints of contact Q and R respectively. Therelation of TQ and TR is(a) TQ < TR (b) TQ > TR(c) TQ = 2TR (d) TQ = TR

106. When two circles touch externally, the number ofcommon tangents are(a) 4 (b) 3(c) 2 (d) 1

107. D and E are the mid-points of AB and AC ofDABC.If ÐA = 80°, ÐC = 35°, then ÐEDB is equal to(a) 100° (b) 115°(c) 120° (d) 125°

108. If the inradius of a triangle with perimeter 32 cm is6 cm, then the area of the triangle in sq. cm is(a) 48 (b) 100(c) 64 (d) 96

109. The value of sin 53° cot 65cos 37° tan 25

°¸

° is

(a) 2 (b) l(c) 3 (d) 0

110. The value of cos 60° + sin 60°cos 60° – sin 60°

is

(a) – 1 (b) 3 + 2

(c) – (2 + 3 ) (d) 3 – 2111. The value of

2 2cot 5 .cot10 .cot15 .cot 60 .cot 75 .cot 80 .cot 85

(cos 20 cos 70 ) 2° ° ° ° ° ° °

° + ° + is

(a) 93

(b) 19

(c)13

(d) 39

112. In a triangle, the angles are in the ratio 2 : 5 : 3.What is the value of the least angle in the radian ?

(a)20p (b)

10p

(c)25p

(d)5p

113. If x = a cos q – b sin q, y = b cos q + a sin q, thenfind the value of x2 + y2.(a) a2 (b) b2

(c)2

2ab

(d) a2 + b2

114. If tan a + cot a = 2, then the value of tan7a +cot7a is(a) 2 (b) 16(c) 64 (d) 128

115. From 125 metre high towers, the angle ofdepression of a car is 45°. Then how far the car isfrom the tower ?(a) 125 metre (b) 60 metre(c) 75 metre (d) 95 metre

116. A can do a certain work in the same time in whichB and C together can do it. If A and B togethercould do it in 10 days and C alone in 50 days,then B alone could do it in(a) 15 days (b) 20 days(c) 25 days (d) 30 days

117. A can do a piece of work in 6 days. B can do thesame work in 12 days. How long would both ofthem take to do the same work ?(a) 2 days (b) 4 days(c) 6 days (d) 8 days

118. If a = 2, b = 3, then (ab + ba)–l is

(a) 131

(b) 117

(c) 121

(d) 113

119. The smallest positive integer which whenmultiplied by 392, gives a perfect square is(a) 2 (b) 3(c) 5 (d) 7

120. A square is inscribed in a circle of radius 8 cm.The area of the square is(a) 16 square cm (b) 64 square cm(c) 128 square cm (d) 148 square cm

121. The biggest possible circle is inscribed in arectangle of length 16 cm and breadth 6 cm. Thenits area is(a) 3p cm2 (b) 4p cm2

(c) 5p cm2 (d) 9p cm2

yoursmahboob.w

ordpress.com

260 SSC CGL SolvedPaper

122. 12 men construct 1.5 km of road in 7 days. 28 menwill construct 12 km of roads in(a) 20 days (b) 24 days(c) 28 days (d) 38 days

123. Kabir buys an article with 25% discount on itsmarked price. He makes a profit of 10% by sellingit at ` 660. The marked price is(a) ` 600 (b) ` 685(c) ` 700 (d) ` 800

124. On the eve of Gandhi Jayanti, Gandhi Ashramdeclared a 25% discount on silk. If selling priceof a silk saree is ̀ 525, what is its marked price ?(a) ` 700 (b) ` 725(c) ` 750 (d) ` 775

125. A shopkeeper marks an article at a price whichgives a profit of 25%. After allowing certain

discount, the profit reduces to 112 %

2. The

discount percent is(a) 12% (b) 12.5%(c) 10% (d) 20%

126. The prize money of ` 1,800 is divided among 3students A, B and C in such a way that 4 timesthe share of A is equal to 6 times the share of B,which is equal to 3 times the share of C. Then A’sshare is(a) ` 400 (b) ` 600(c) ` 700 (d) ` 800

127. Divide 81 into three parts so that 12

of 1st, 13

of

2nd and 14

of 3rd are equal.

(a) 36, 27, 18 (b) 27, 18, 36(c) 18, 27, 36 (d) 30, 27, 24

128. If the diagonal of a square is doubled, then itsarea will be(a) three times (b) four times(c) same (d) none of these

129. Out of 40 boys in a class, average weight of 30 is60 kg and the average weight of the remaining is56 kg. The average weight (in kilogram) of thewhole class is(a) 58.5 (b) 58(c) 57 (d) 59

130. A CD was sold at a profit of 112 %

2. If it had

been sold at a profit of 15%, it would have gainedhim ` 10 more. The cost prices of CD is (in `)(a) 450 (b) 500(c) 400 (d) 550

131. Rakesh got 273 marks in an examination andscored 5% more than the pass %. If Lokesh got312 marks, then by what % above the pass markdid he pass the examination?(a) 20% (b) 27%(c) 25% (d) 15%

132. Anil calculated that it will take 45 minutes to covera distance of 60 km by his car. How long will ittake to cover the same distance if the speed ofhis car is reduced by 15 km/hr?(a) 36 min (b) 55.38 min(c) 48 min (d) 40 min

133. A train 100 metres long moving at a speed of 50km/hr. crosses a train 120 metres long comingfrom opposite direction in 6 sec. The speed ofthe second train is(a) 60 km/hr. (b) 82 km/hr.(c) 70 km/hr. (d) 74 km/hr.

134. Two equal sums were borrowed at 8% simpleinterest per annum for 2 years and 3 yearsrespectively. The difference in the interest was `56. The sums borrowed were(a) ` 800 (b) ` 700(c) ` 560 (d) ` 350

135. The average of the first five multiples of 7 will be(a) 14 (b) 21(c) 17.5 (d) 24.5

136. The expression x4 – 2x2 + k will be a perfect squarewhen the value of k is(a) 1 (b) 2

(c) 12

(d) 14

137. If 3x – 1

4y = 6, then the value of 4x – 1

3y is

(a) 2 (b) 4(c) 6 (d) 8

138. If a + b + c = 0, find the value of2+ +

- ++

a b b b cc c a a

.

(a) 0 (b) l(c) – 1 (d) 2

139. If x + 4x

= 4, find the value of 33

4+x

x.

(a) 8 (b) 812

(c) 16 (d) 1612

yoursmahboob.w

ordpress.com

261SSC CGL SolvedPaper

140. If x = 3 + 2 2 , then the value of 1æ ö

-ç ÷è øx

x is

(a) 1 (b) 2(c) 2 2 (d) 3 3

141. If ‘a’ be a positive number, then the least value of

a + 1a

is

(a) 1 (b) 0

(c) 2 (d)12

142. If a = 0, b ¹ 0, c ¹ 0, then the equation ax + by +c = 0 represents a line parallel to(a) x + y = 0 (b) x-axis(c) y-axis (d) none of these

143. The sum of the ages of Puneet and his father is45 years and the product of their ages is 126.What is the age of Puneet ?(a) 3 years (b) 5 years(c) 10 years (d) 45 years

PART-D : ENGLISH COMPREHENSION

Directions (Qs. 144-146) : The following graph showsthe expenditure incurred in bringing a book, by amagazine producer. Study the graph and answerquestion.

Cost ofPrinting

35%

BindersCharge15%

Royalty15%

Adv.charges

15%

Cost ofPaper16%

Misc4%

144. What should be the central angle of the sectorfor the cost of the paper ?(a) 57.6° (b) 54.4°(c) 56.7° (d) 54.8°

145. If the miscellaneous charges are ̀ 6,000, the costof paper is(a) ` 12,000 (b) ` 18,000(c) ` 15,000 (d) ` 24,000

146. If 5500 copies are published, miscellaneousexpenditures amount to ̀ 1,848, find the cost priceof 1 copy.(a) ` 10.40 (b) ` 9.40(c) ` 12.40 (d) ` 8.40

Directions: (Qs. 147-148) : The pass percentage foran examination in a school is shown in the adjoiningbar diagram, for males and females separately forfour years. Study the diagram and answer thequestion.

Years

10

20

30

40

50

2007 2008 2009 2010

Perc

enta

ge o

f stu

dent

s Male

Female

147. The maximum percentage of students passed inthe year is(a) 2007 (b) 2008(c) 2009 (d) 2010

148. The year in which the difference of passpercentage between male and female is maximum,is(a) 2010 (b) 2009(c) 2008 (d) 2007

Directions (Qs. 149-150) : The adjacent histogramshows the average pocket money received by 60students for a span of one month. Study the diagramand answer the question.

4

8

12

16

20

050 80 110 140 170 200

Pocket Money (in ) `

No.

of s

tude

nts

yoursmahboob.w

ordpress.com

262 SSC CGL SolvedPaper

149. Maximum number of students received pocketmoney between(a) 50 – 80 (b) 140 –170(c) 80 – 110 (d) 110 – 140

150. The number of students who received pocketmoney upto ` 140 is(a) 20 (b) 32(c) 48 (d) 56

Directions (Qs. 151-153) : Choose the word oppositein meaning to the given word and mark it in theAnswer Sheet.151. Fabricate

(a) Unearth (b) Construct(c) Demolish (d) Renovate

152. Gregarious(a) Sociable (b) Societal(c) Unsociable (d) Solitary

153. Pragmatic(a) Indefinite (b) Vague(c) Optimistic (d) Idealistic

Directions (Qs. 154-158) : In the following questions,four alternatives are given for the Idiom/Phraseunderlined. Choose the alternative which bestexpresses the meaning of the Idiom/Phrase and markit in the Answer Sheet.154. To be above board.

(a) To have a good height(b) To be honest in any business deal(c) Having no debts(d) To try to be beautiful

155. To cry wolf.(a) To listen eagerly(b) To give false alarm(c) To turn pale(d) To keep off starvation

156. He is on the wrong side of seventy.(a) more than seventy years old(b) less than seventy years old(c) seventy years old(d) eighty years old

157. To have an axe to grind.(a) a private end to serve(b) to fail to arouse interest(c) to have no result(d) to work for both sides

158. To drive home.(a) To find one’s root(b) To return to place of rest(c) Back to original position(d) To emphasise

Directions (Qs. 159-168) : In the following questions,a sentence/part of the sentence is underlined. Beloware given alternatives to the underlined sentence/part of the sentence at (a), (b) and (c) which mayimprove the sentence. Choose the correct alternative.In case no improvement is needed, your answer is(d).159. The climate of Karnataka is cooler than Tamil Nadu.

(a) is cooler to(b) is cooler than of(c) is cooler than that of(d) No improvement

160. The Tsunami victims suffered of cholera.(a) suffered from (b) suffered under(c) suffered in (d) No improvement

161. I gave to Sana the keys.(a) I gave (b) I gave to the(c) I gave the (d) No improvement

162. If he smokes less he might get rid of his cough.(a) If he smoked less he would get rid of his

cough.(b) If he had smoked less he might get rid of his

cough.(c) If he smokes less he might have got rid of

his cough.(d) No improvement.

163. He compensated the loss to me.(a) He compensated the loss for me.(b) He compensated me to the loss(c) He compensated me for the loss.(d) No improvement.

164. As employees, we are accountable for ourstakeholders.(a) accountable with(b) accountable to(c) accountable against(d) No improvement

165. Recently he had insured for a mediclaim policy.(a) He had recently insured for(b) Recently he insured for(c) He insured recently for(d) No improvement

166. Everyday, we usually had lunch at 1.30 p.m.(a) we have had usually(b) we have usually(c) we usually have(d) No improvement

167. All nations must first become agricultural strong.(a) become agricultural strong(b) become strong agriculture(c) become agriculture strong(d) No improvement

yoursmahboob.w

ordpress.com

263SSC CGL SolvedPaper

168. An orangutan’s intelligence is as superior to thatof man.(a) is more superior to(b) is superior to(c) is superior than that of(d) No improvement

Directions (Qs. 169-175) : In the following questions,out of the four alternatives, choose the one whichcan be substituted for the given words/sentence.169. That which has a double meaning

(a) doubtless (b) uncertain(c) controversial (d) ambiguous

170. Incapable of making errors(a) infallible (b) incorrigible(c) impervious (d) inexplicable

171. Governed by a sense of duty(a) conscious (b) sensible(c) intelligent (d) conscientious

172. The depository where state records anddocuments are preserved(a) museum (b) library(c) emporium (d) archive

173. That which is no longer fashionable or in use(a) unused (b) ancient(c) obsolete (d) old

174. Murder of a king(a) homicide (b) fratricide(c) regicide (d) parricide

175. A place where birds are kept(a) Aviary (b) House(c) Aquarium (d) Apiary

Directions (Qs. 176 & 177) : In the followingquestions, four words are given in each question, outof which only one word is correctly spelt.176. (a) milennium (b) millenium

(c) milleneum (d) millennium177. (a) ocassion (b) occassion

(c) occasion (d) occation

Directions (Qs. 178-187) : Read the passage carefullyand choose the best answer to each question out ofthe four alternatives.The stunning Baltimore Oriole is a common summervisitor to eastern and mid western deciduous woodlands,neighbourhoods, and gardens. Baltimore Orioleswinter in the tropics. About 7 inches in length, themale Baltimore Oriole has a black head, throat, backand wings. Its breast, stomach, and rump are brightorange. It also has an orange patch on the top of eachwing and white wing bars. The tail is mostly black withorange fringes. The female is dull orange throughout.

Baltimore Orioles range throughout the easternand mid western United States, and can be found asfar west as the Dakotas. At the western edge of theirrange, Baltimore Orioles may breed with the Bullock’sOriole (They were once considered the same speciesunder the name Northern Oriole).

Baltimore Orioles build unusual pouch like neststhat hang down from branches. They usually nest highin the trees, but often come down to lower heights,flashing bright orange and black feathers to delightedobservers Active and acrobatic by nature, BaltimoreOrioles may even feed upside down at time.

Baltimore Orioles eat insects and berries. Theycan easily be attracted to gardens by nailing orangewedges to tree branches. Baltirmore Orioles are alsoknown to feed at hummingbird feeders and sapsuckerwells.l78. The other name of Baltimore Oriole was

_________ .(a) Bullock’s Oriole (b) Baltimore’s Oriole(c) Northern Oriole (d) Southern Oriole

179. The nest of the Baltimore Oriole _________ .(a) is in a tree cavity(b) stands upon a branch of a tree(c) hangs from a branch of a tree(d) is usually low in the branches

180. Which of the following is the closest in size to aBaltimore Oriole ?(a) The size of a half-scale(b) A little more than a half-scale(c) A little less than a half-scale(d) A foot ruler

181. The Baltimore Oriole spend the winters in the_______ .(a) Dakotas (b) Carolinas(c) Tropics (d) Deserts

182. What is the colour of the female Baltimore Oriole ?(a) Bright Orange (b) Light Orange(c) Dull Orange (d) White

183. Which of the following does not attract theBaltimore Oriole?(a) Oranges(b) Hummingbird feeders(c) Sapsueker wells(d) Sunflower seeds

184. The Baltimore Oriole can be found as far west as(a) North and South Dakota(b) The Carolinas(c) California(d) Baltimore

yoursmahboob.w

ordpress.com

264 SSC CGL SolvedPaper

185. Which of the following is not true about theBaltimore Oriole?(a) They feed upside down sometimes.(b) They may breed with the Bullock’s Oriole.(c) The Baltimore Oriole is uncommon in the

U.S.(d) The Baltimore Oriole has a black throat.

186. Where would I probably not find a BaltimoreOriole ?(a) High in the trees(b) In gardens and neighbourhoods(c) Deciduous woodlands(d) The Sahara desert

187. Which of these colours is not found on aBaltimore Oriole?(a) Purple (b) Orange(c) White (d) Black

Directions (Qs. 188 -192) : In the following questions,some parts of the sentences have errors and some arecorrect. Find out which part of a sentence has anerror. If a sentence is free from error, your answer is(d) i.e. No error.

188. Mohans' eyes(a)

/ reflect a hope(b)

/ for a better future in Microsoft.

(c) / No error.

(d)

189.He went to Mumbai

(a) /

with a view(b)

/

to secure a job.(c)

/ No error.

(d)

190. The Headmaster with all his senior teachers(a)

/ have come

(b) / to attend the meeting.

(c) /

No error.(d)

191.The teacher said that

(a) /

the building adjacent with his house(b)

/ needed repairs.

(c) / No error.

(d)

192.Grapes

(a) /cannot gathered

(b) /from thistles.

(c) /

No error.(d)

Directions (Qs. 193-197) : In the following questions,sentences are given with blanks to be filled with anappropriate word(s). Four alternatives are suggestedfor each question. Choose the correct alternative outof the four and indicate it.193. Mr. Murugan has been in this college ________

2010.(a) for (b) since(c) after (d) before

194. We attended a ________ discourse.(a) spiritual (b) spirituous(c) spirituality (d) spiritually

195. The valley is known for its ________ growth ofvegetation.(a) luxurious (b) luxury(c) luxuriant (d) luxuriously

196. Satyajitray’s films ________ all barriers of caste,creed and religion. They are universal.(a) transcend (b) transcends(c) trancend (d) transend

197. I could hardly recognize him ________ I sawhim.(a) after (b) but(c) and (d) when

Directions (Qs. 198-200) : In the following questions,out of the four alternatives, choose the one whichbest expresses the meaning of the given word.198. Condone

(a) Forgive (b) Support(c) Forget (d) Defend

199. Analogy(a) Difference (b) Comparison(c) Addition (d) Deletion

200. Allure(a) Extol (b) Excite(c) Entice (d) Elicit

yoursmahboob.w

ordpress.com

265SSC CGL SolvedPaper

1. (c)+5 +5 +5 +5 +5

+5 5 +5 +5 +5

+5 +5 +5 +5 +5

A F P U Z

K P Z E J

U Z J O T

+

¾¾® ¾¾® ¾¾® ¾¾® ¾¾®

¾¾® ¾¾® ¾¾® ¾¾® ¾¾®

¾¾® ¾¾® ¾¾® ¾¾® ¾¾®

K

U

E

2. (c) 1 1 1

+2 +2 +2

+2 +2 +2

M L K

R T V

S U W

¾¾® ¾¾® ¾¾®

¾¾® ¾¾® ¾¾®

¾¾® ¾¾® ¾¾®

J

XY

3. (a)

4. (d) 6 × 3 = 1818 × 6 = 108108 × 18 = 1944Hence, 18 is the missing number in thesequence.

5. (d) 251 – 107 = 144 = (12)2

\ 381 – 125 = 256 = (16)2

Hence, 16 is the missing number in the givenquestion.

6. (b) 354 + 526 = 880

8=110

\ 297 +631 = 9238 = 116

7. (d)

8. (a)

9. (b)10. (d)

11. (b) 2 = 5 Þ 22 + 14 = 18 Þ 42 + 26 = 39 Þ 62 + 38 = 68 Þ 82 + 410 = 102 + 5 = 105

12. (c) Tuesday was the day before yesterday.Below figure shows the days pattern:

13. (c)

Hence, Asha is 3km from starting point andin the east direction.

14. (a)

Hence, P coach is in the middle of the fivecoaches.

15. (d) STIGMA cannot be formed using word'INVESTIGATION' because letter M is notin the given reference word.

16. (c) NOTION cannot be formed because two Osare not in the reference word.

17. (d) READ cannot be fomed as letter D is not inthe reference word.

18. (a) S N A K E

37 + 27 + 1 + 21 + 9 = 9519. (c)

20. (b)

\

HINTS & SOLUTIONS

yoursmahboob.w

ordpress.com

266 SSC CGL SolvedPaper

21. (a) Solve by options, we can check all theoptions one by one.25 ̧ 5 × 20 + 27 – 7 Þ 5 × 20 + 27 – 7 Þ 100+ 27 – 7120 = 120

22. (a) A q B × CA q B ; B × C\ A ³ B ; B > CHence, option (a) implies the given equation.

23. (c) (6 + 3) – (3 + 0) = 6 × 5 = 30(7 + 2) – (1 + 0) = 8 × 5 = 40\ (8 + 1) – (6 + 0) = 3 × 5 = 15

24. (d)

Hence, Priya is moving in the South direction.25. (d)

As East started showing south then westwas actually north.

26. (d) Arranging digits according to question.265, 178, 834, 357Hence, third highest would be 265 and 8 bethe sum of first and second digits of thenumber.

27. (b) A + B + Y + Z = 1 + 2 + 25 + 26 = 54\ C + D + W + X = 3 + 4 + 24 + 23 = 54

28. (c) Petrology is the branch of geology thatdeals with the origin, composition, structureand alteration of rocks.Therefore, palacontology is the study offossils to determine the structue andevolution of extinct animals and plants andthe age and the conditions of deposition ofthe rock strata in which they are found.

29. (c) First is the process of formation of the second.30. (a) 27 × = 81 \ 8 × 3 = 2431. (c) a a b/ a a b/ aa b/ aa b32. (c) Fervent: having or displaying a passionate

intensity.Enthusiastic: having or showing intense andeager enoyment.Ardent: very enthusiastic or passionateApathetic: showing or feeling no interest,enthusiasm or concern.Hence, apathetic is odd one out.

33. (a) In all other groups, a vowel is followed by aconsonant repeated twice.

34. (a) Only 117-143 is divisible by 13. Therefore, itis odd one out.

35. (d) Radio, Transistor and television are the wayof broadcasting. Hence, tube is odd one out.

36. (c)37. (b)

By looking at above venn diagram, we canconcluded that only conclusions II follow.

38. (c)39. (a) ÷ + O $® ¹ D

All other symbols are adjacent to ¸.Therefore, $ symbol is opposite to it.

40. (b) By looking, the dice position, we can saythat 2, 4, 5 and 6 are adjacent faces of 3.therefore, if 1 number is at the bottom then 3will be on the top.

41. (c) The series represents continous alphabetsstarting from K. Hence, N is the right answer.

42. (c)

Delhi is separate state while Lucknow is partof Uttar Pradesh.

yoursmahboob.w

ordpress.com

267SSC CGL SolvedPaper

43. (c)

O

C

BA H

D

E

F G

There 14 triangles in the given figure. TheseareAHO, ACB, BHO, BAD, ABE, ABD, BAF,ABG, AOF, AFD, BOG, BGC, ADO and BOC.

44. (a) Year

Week

Day

45 (d)

46. (d)47. (c)

48. (c) 49. (b)50. (b) C = 00, 12, 24, 33, 41

H = 55, 67, 79, 88, 96I = 58, 65, 77, 86, 99L = 57, 69, 76, 85, 98D = 01, 13, 20, 34, 42\ CHILD = 12, 79, 99, 57, 01

51. (a) Devarda's alloy , is an alloy of aluminium(44% - 46%), copper (49% - 51%) and zinc(4% - 6%). Devarda's alloy is used asreducing agent in analytical chemistry forthe determination of nitrates after theirreduction to ammonia under alkalineconditions. It owes its name to the Italian

chemist Arturo Devarda (1859-1944), whosynthezised it at the end of the 19th centuryto develop a new method to analyze nitratein Chile saltpeter.

52. (d) The Red Data Book is the state documentestablished for documenting rare andendangered species of animals, plants andfungi as well as some local sub-species thatexist within the territory of the state orcountry. This book provides centralinformation for studies and monitoringprogrammes on rare and endangered speciesand their habits.

53. (a) Grit chambers are long narrow tanks that aredesigned to slow down the flow so thatsolids such as sand, coffee grounds, andeggshells will settle out of the water. Gritcauses excessive wear and tear on pumpsand other plant equipment. Its removal isparticularly important in cities withcombined sewer systems, which carry agood deal of silt, sand, and gravel that washoff streets or land.

54. (b) Environmental impact assessmentscommenced in the 1960s, as part ofincreasing environmental awareness. EIAsinvolved a technical evaluation intended tocontribute to more objective decisionmaking. In the United States, environmentalimpact assessments obtained formal statusin 1969, with enactment of the NationalEnvironmental Policy Act.

55. (a) Cysteine (abbreviated as Cys or C) is an a-amino acid with the chemical formulaHO2CCH(NH2)CH2SH. It is a semi-essentialamino acid, which means that it can bebiosynthesized in humans. The thiol sidechain in cysteine often participates inenzymatic reactions, serving as anucleophile.

56. (b) Victoria Azarenka successfully defended hertitle, defeating Li Na in the final 4-6, 6-4, 6-3.All of the top three seeds (Victoria Azarenka,Maria Sharapova and Serena Williams) werein contention for the World No. 1 ranking atthe start of the tournament.

57. (a) Sir Clyde Leopold Walcott, KA, GCM (17January 1926 - 26 August 2006) was a WestIndian cricketer. Walcott was a member ofthe "three W's", the other two being EvertonWeekes and Frank Worrell: all were verysuccessful batsmen from Barbados, bornwithin a short distance of each other in

yoursmahboob.w

ordpress.com

268 SSC CGL SolvedPaper

Bridgetown, Barbados in a period of 18months from August 1924 to January 1926;all made their Test cricket debut againstEngland in 1948. In the mid-1950s, Walcottwas arguably the best batsman in the world.In later life, he had an active career as acricket administrator, and was the first non-English and non-white chairman of theInternational Cricket Council.

58. (d) Neon is a chemical element with symbol Neand atomic number 10. It is in group 18 (noblegases) of the periodic table. Neon is acolorless, odorless, inert monatomic gasunder standard conditions, with about two-thirds the density of air.

59. (b) The Jnanpith Award is a literary award inIndia. Along with the Sahitya AkademiFellowship. It is one of the two mostprestigious literary honours in the country.The award was instituted in 1961. AnyIndian citizen who writes in any of theofficial languages of India is eligible for thehonour. It is presented by the BharatiyaJnanpith, a trust founded by the Sahu Jainfamily, the publishers of the The Times ofIndia newspaper.

60. (c) Hilary Mary Mantel is an English writerwhose work ranges in subject from personalmemoir and short story to historical fictionand essay. She has twice been awarded theBooker Prize. She won her first Booker Prizefor the 2009 novel, Wolf Hall, a fictionalaccount of Thomas Cromwell's rise to powerin the court of Henry VIII. She won hersecond Booker Prize for the 2012 novel,Bring Up the Bodies, the second instalmentof the Thomas Cromwell trilogy. Mantel wasthe first woman to receive the award twice.

61. (b) The currency of Iran is Rial.62. (c) OPEC is an intergovernmental organization

that was created at the Baghdad Conferenceon September 10-14, 1960, by Iraq, Kuwait,Iran, Saudi Arabia and Venezuela. Later itwas joined by nine more governments:Libya, United Arab Emirates, Qatar,Indonesia, Algeria, Nigeria, Ecuador, Angola,and Gabon. OPEC was headquartered inGeneva, Switzerland before moving toVienna, Austria, on September 1, 1965.

63. (b) Swatantra Kumar is the Chairperson ofNational Green Tribunal.

64. (b) Barmako is the capital of Mali.

65. (b) Jaspal Singh Bhatti (3 March 1955 - 25October 2012) was an Indian televisionpersonality famous for his satirical take onthe problems of the common man. He is mostwell known for his television series FlopShow and mini capsules Ulta Pulta whichran on Doordarshan, India's nationaltelevision network, in the late 1980s and early1990s. In 2013, he was honoured with thePadma Bhushan (posthumously), India'sthird highest civilian award

66. (b) Lal Bahadur Shastri was the second PrimeMinister of the Republic of India and a leaderof the Indian National Congress party.Shastri joined the Indian independencemovement in the 1920s.

67. (c) Above 10,000 hectares68. (a) Bank's reserves at Central Bank69. (a) Interest rate is remaining sticky70. (c) Sells a commodity at lower price in the world

market and charges a higher price in thedomestic market

71. (d) Selecting the candidate of election is thefunction of the political party

72. (a) This quote is given by the theory ofIndividualism

73. (c) Operation Flood in India, a project of theNational Dairy Development Board (NDDB)was the world's biggest dairy developmentprogram which made India, a milk-deficientnation, the largest milk producer in the world,surpassing the USA in 1998, with about 17percent of global output in 2010-11, whichin 30 years doubled the milk available perperson, and which made dairy farming India'slargest self-sustainable rural employmentgenerator. All this was achieved not merelyby mass production, but by production bythe masses.

74. (c) Marxism is a socio-economic and politicalworldview or inquiry based on a materialistinterpretation of historical development, adialectical view of social transformation, ananalysis of class-relations and conflictwithin society. Marxist methodologyinforms an economic and sociopoliticalenquiry applying to the analysis andcritique of the development of capitalism andthe role of class struggle in systemiceconomic change. In the mid-to-late 19thcentury, the intellectual tenets of Marxismwere inspired by two German philosophers:Karl Marx and Friedrich Engels.

yoursmahboob.w

ordpress.com

269SSC CGL SolvedPaper

75. (c) powers and functions of the chief ElectionCommissioner

76. (a) Bahlul Khan Lodi was the founder of Lodidynasty of the Delhi Sultanate in India uponthe abdication of the last claimant from theprevious Sayyid rule.

77. (d) Greco-Roman-Buddhist Art78. (a) the Bolshevik Revolution, was a seizure of

state power instrumental in the largerRussian Revolution of 1917. It took placewith an armed insurrection in Petrogradtraditionally dated to 25 October 1917.

79. (b) Lieutenant-General Lord William HenryCavendish-Bentinck, GCB, GCH, PC, knownas Lord William Bentinck, was a Britishsoldier and statesman. He served asGovernor-General of India from 1828 to 1835.

80. (c) The All India Trade Union Congress is theoldest trade union of India, established in1920, History of AITUC is coterminous withthe history of organised labour movementin India. Since its birth, AITUC has had amajor role to play in mass movement phasein India's freedom struggle.

81. (c) The Joint Session of Parliament resolves thedeadlock between Lok Sabha and RajyaSabha over an ordinary bill.

82. (b) the law explains that wind is deflected to theright in the Northern Hemisphere and to theleft in the Southern Hemisphere, derived fromthe application of the Coriolis effect to airmasses.

83. (d) ozone layer, also called ozonosphere, regionof the upper atmosphere, between roughly15 and 35 km (9 and 22 miles) above Earth'ssurface, containing relatively highconcentrations of ozone molecules (O3).

84. (b) A dendritic drainage pattern refers to thepattern formed by the streams, rivers, andlakes in a particular drainage basin. It usuallylooks like the branching pattern of tree rootsand it mainly develops in regions underlainby homogeneous material.

85. (c) Plimsol line is not a line of demarcationbetween two countries.

86. (d) A halophyte is a plant that grows in watersof high salinity, coming into contact withsaline water through its roots or by saltspray, such as in saline semi-deserts,mangrove swamps, marshes and sloughs,and seashores. An example of a halophyteis the salt marsh grass Spartina alterniflora(smooth cordgrass).

87. (b) Azotobacter is a genus of usually motile,oval or spherical bacteria that form thick-walled cysts and may produce largequantities of capsular slime.

88. (b) The aorta is the largest artery in the humanbody, originating from the left ventricle ofthe heart and extending down to theabdomen, where it bifurcates into two smallerarteries (the common iliac arteries). The aortadistributes oxygenated blood to all parts ofthe body through the systemic circulation.

89. (c) Parathyroid hormone (PTH), parathormoneor parathyrin, is secreted by the chief cellsof the parathyroid glands as a polypeptidecontaining 84 amino acids. It acts to increasethe concentration of calcium (Ca2+) in theblood, whereas calcitonin (a hormoneproduced by the parafollicular cells (C cells)of the thyroid gland) acts to decreasecalcium concentration.

90. (c) Extratropical cyclones, sometimes calledmid-latitude cyclones or wave cyclones, area group of cyclones defined as synopticscale low pressure weather systems thatoccur in the middle latitudes of the Earth(outside the tropics) not having tropicalcharacteristics, and are connected withfronts and horizontal gradients intemperature and dew point otherwise knownas "baroclinic zones".

91. (c) by tracheal system92. (c) In nuclear physics and nuclear chemistry, a

nuclear reaction is semantically consideredto be the process in which two nuclei, orelse a nucleus of an atom and a subatomicparticle (such as a proton, neutron, or highenergy electron) from outside the atom,collide to produce one or more nuclides thatare different from the nuclide(s) that beganthe process.

93. (b) annihilate each other94. (a) In the photoelectric effect, electrons are

emitted from solids, liquids or gases whenthey absorb energy from light. Electronsemitted in this manner may be calledphotoelectrons.

95. (d) a circle96. (a) The IBM 650 Magnetic Drum Data-

Processing Machine was one of IBM's earlycomputers, and the world's first mass-produced computer. It was announced in1953 and almost 2000 systems wereproduced, the last in 1962. Support for the650 and its component units was withdrawnin 1969.

yoursmahboob.w

ordpress.com

270 SSC CGL SolvedPaper

97. (a) stack98. (b) Cryolite (Na3AlF6, sodium

hexafluoroaluminate) is an uncommonmineral identified with the once large depositat Ivigtût on the west coast of Greenland,depleted by 1987.

99. (c) Glucose (C6H12O6, also known as D-glucose, dextrose, or grape sugar) is a simplemonosaccharide found in plants. It is one ofthe three dietary monosaccharides, alongwith fructose and galactose, that areabsorbed directly into the bloodstreamduring digestion.

100. (b) Any of the homologous segments, lying ina longitudinal series, that compose the bodyof certain animals, such as earthworms andlobsters. Also called somite.

101. (a) In DOBP.OB = OP (Q radius)

25º

35º

O

PB

A

\ Ð OBP = Ð OPB = 35ºIn DD AOPOA = OP (Q radius)\ Ð OAP = Ð OPA = 25ºNow, Ð APB = Ð OPA + ÐOPB= 25º + 35º = 60ºHence, ÐAOB = 2ÐAPB

(Angle be substended by are at centre istwice)

= 2 × 60º = 120º

102. (d)

140º

DCB

A

ÐACB + ÐACD = 180º (linear pair)\ ÐACB = 180º – 140º = 40ºIn D ABCÐ BAC + Ð ABC + Ð ACB = 180ºÐ BAC + 3 Ð BAC + 40º = 180º

4 ÐBAC = 180º – 40º

Ð BAC = 140 35º

4=

103. (c) 12 cmP

O

A

5 cm

AP is a tangent and OA is a radius.Therefore, OA is ^ at AP.So, In D OAPOP2 = 52 + 122

OP2 = 25 + 144 = 169OP = 13 cm

104. (b) In D ABC, Ð ACB = 90º

50º

O B

CD

A

\ ÐACB + ÐACDÞ 90º + 50º = 140ºAs angle mode by trianglein semicircle is equal to 90º.\ In quad. ABCD ÐBAD + ÐBCD = 180ºangle of (opp. pair of quad is equal to 180º)Ð BAD = 180º – 140º = 40º

105. (d) The relation of TQ and TR is TQ = TR.

106. (b)

107. (b) DE is parallel to BCSo Ð AED = ÐC = 35º

80º

D E

35ºCB

A

yoursmahboob.w

ordpress.com

271SSC CGL SolvedPaper

Since ÐA = 80ºThen Ð ADE = 65ºÐ EDB is supplement to ÐADE.So, Ð EDB = 180º – Ð ADE= 180º – 65º = 115º

108. (d) Area of triangle = Inradius × Semi-perimeter= 6 ×16 = 96 sq. cm

109. (b)sin 53º cot 65ºcos37º tan 25º

¸

sin 53º tan 25ºcos37º cot 65º

´

sin53º tan 25ºcos(90º –53º ) cot(90º –25º )

Þ ´

Þsin53º tan 25º 1sin53º tan 25º

´ =

[Qcos (90º – q) = sin q and cot (90º – q)= tan q]

110. (c)

1 3cos 60º sin 60º 2 2cos60º – sin 60º 1 3–

2 2

++ =

1 3 1 31 – 3 1 3

+ += ´+

Þ 2

2 2(1 3) 1 3 2 3 4 2 3

1– 3 –21 – ( 3)+ + + += =

Þ –2(2 3) (2 3)

2+ = - +

111. (d) 2 2cot 5º .cot10º.cot15º.cot 60º .cot 75º .cot80º .cot 85º

(cos 20 cos 70º ) 2+

+ +

Þ 2 2cot(90º –85º ).cot(90º –80º ).cot(90º –75º ).cot 60º.cot 75º .cot 80ºcot85º

(cos (90º –70º ) cos 70º ) 2+ +

Þ

1cot 60º 1 3 33(1 2) 3 93 3 3

= = ´ =+

112. (d) Let angles are 2x, 5x and 3x.2x + 5x + 3x = 180º(sum of interior angle of triangles is 180º)

10x =18ºx = 18º\ Least angle in degree = 2x = 2 × 18 = 36º

In radian = 36º180º 5

p p´ =

113. (d) x = a cos q – b sin qy = b cos q + a sin qx2 + y2 = (a cos q – b sin q)2 + (b cos q + a sin q)2

Þ a2 cos2 q + b2 sin2 q – 2 ab cos q sin q+ b2 cos2 q + a2 sin2 q + 2ab cos q sin qÞ (a2 + b2) cos2 q + (a2 + b2) sin2 qÞ a2 + b2 (cos2 q + sin2q)Þ a2 + b2. (1) Þ a2 + b2

114. (a) tan a + cot a = 2

tan a + 1 2

tan=

a Þ tan2 a + 1 = 2 tan a

Þ tan2 a – 2 tan a + 1 = 0Þ tan2 a – tan a – tan a + 1 = 0Þ tan a (tan a – 1) – 1 (tan a – 1) = 0

(tan a – 1) (tan a – 1) = 0\ tan a = 1Now, tan7 a + cot7 a Þ (tan a)7

71 1 1 2

(tan )+ = + =

a

115. (a) q = 45º

q = 45ºB Car

C

ATower

125 m

In DABC

tanABBC

q = Þ tan 125 12545º 1BC BC

= Þ =

BC = 125 mHence, car is 125 m from the tower.

116. (c) (A + B)’s 1 day’s work 1 ;

10=

C’s 1day’s work 150

=

(A + B + C)’s 1 day’s work1 1 6 3

10 50 10 25æ ö= + = =ç ÷è ø

...(1)

Also, A’s 1 day’s work = (B + C)’s 1 day’swork ...(2)From (1) and (2), we get : 2 × (A’s 1 day’s

work) 325

=

yoursmahboob.w

ordpress.com

272 SSC CGL SolvedPaper

Þ A’s 1 day’s work 325

=

\ B’s 1 day’s work1 3 2 1–

10 50 50 25æ ö= = =ç ÷è ø

So, B alone could do the work in 25 days.

117. (c) A’s 1 day’s work 1

10= and B’s 1 day’s work

115

=

\ (A + B)’s 1 day’s work 1 1 1

10 15 6æ ö= + =ç ÷è ø

So both together will finish the work in 6days.

118. (b) (ab + ba)–1 = (23 + 32)–1 = (8 + 9)–1 = (17)–1

117

=

119. (a) 392 × 2 = 784 Þ (27)2

Hence, 2 can be multiplied by 392 whichgives perfect square.

120. (c)D C

BA

8 cm

8 cmO

Diagonal of square = Diameter of circle2 ´ side of square = 16 cm

Squaring on both sides

( )2 22 sides of square 16´ =

Þ (side of square)2 = 16 16

Þ Area of square = 128 sq. cm121. (d) The area of circle is 9p cm2.122. (b) Let the required number of days be x.

Then, more men, more km (Directproportion)more days, more km (Direct proportion)menMen 12 : 28

::1.5 :12Days 7 : x

üýþ

\ 12 × 7 × 12 = 28 × x × 1.512 7 12 24

28 1.5x ´ ´= =

´123. (d) Let the marked price be ̀ x.

Q C.P. = (x – 25% of x) = 34

x

3x 3x 33S.P. 10% of x4 4 40

æ öÞ = + =ç ÷è ø

But, 33 66040

x = Þ x = 800.

124. (a) Let the marked price be ̀ x.

Q S.P. = (x – 25% of x) = 34

x

But, S.P = ` 525

\ 3 525 7004

x x= Þ =

125. (c) Short cut method :Net profit = Profit + Discount +Profit Discount

100´

25 25×Discount25 – Discount–2 100

=

(‘–’ to represent discount)25 –5– 252 4

= Discount

\ Discount % = 10%126. (b) 4A = 6B Þ 2A = 3B Þ A : B = 3 : 2

B = 3C Þ 2 B = C Þ B : C = 1 : 2A : B : C3 : 2 1 : 23 : 2 : 4

A’s share3 31800 1800 600

(3 2 4) 9= ´ = ´ =

+ +127. (c) Let lst, 2nd and 3rd part represented by x, y, z

Let 1 1 12 3 4

x y z k= = =

\ x = 2k, y = 3k, = 4kAccording to questionx + y + z = 81Þ 2k + 3k + 4k = 81 Þ 9k = 81 Þ k = 9Hence, parts are 18, 27, 36.

yoursmahboob.w

ordpress.com

273SSC CGL SolvedPaper

128. (b) Diagonal of a square (d) = 2 × side ofsquare (a).

d = 22

da aÞ =

Area of square Þ a2 = 2

2d

Now, diagonal gests doubled(2 )

2da =

Area of square 22 4

22d d 2æ öæ ö= = ç ÷ç ÷ ç ÷è ø è ø

2

2d

is area of square

Therefore, Area will be four temes.129. (d) Average weight of 30 = 60 kg

Þ Sum of weight of 30 boys = 1800Average weight of 10 =56 kgÞ Sum of weight of 10 boys = 560Average weight of the whole class

= Sum of weight of all boys

40

sum of weight of 30 boys + sum of weight of 10 boys=

4060×30+56×10= 59kg

40=

130. (c) Ist case :100 Profit%S.P C.P.

100´= ´

25100 C.P2S.P.

100

+ ´Þ =

Þ 112.5S.P CP100

= ...(1)

IInd case :100 Profit %S.P C.P.

100+= ´

100 15(S.P 10) C.P.

100+

Þ + = ´

Þ115(S.P+10)= C.P100 ...(2)

Dividing equation (1) by (2)S.P 112.5 100(C.P)

S.P+10 100 115(C.P)= ´

112.5S.P (S.P + 10)115æ ö= ç ÷è ø

115 S.P = 112.5 SP + 1125S.P = 450

\S.P 100 450 100C.P 400

112.5 112.5´ ´= = =

131. (a) Let passing marks be represented by p.p × 1.05 = 273p = 260

Lokesh passing312 – 260% 100

260= ´

= 20%132. (b) D = S × T

60 = 45S hr60æ ö´ç ÷è ø

S = 60 60

45´Þ 80km/hr

Now, new speed = 80 – 15 = 65 km/hr.

\ Time Distance 60 hr.Speed 65

= =

or 60 60 min 55.38min.65´ =

Hence, Time to taken by car to travel samedistance is 55.38 min.

133. (b) Let speed of the second train = x km/hr.Relative speed of trains = (50 + x) km/hr.Distance travelled by trains = (100 + 120) =220 metresDistance = Speed × Time

220 6km (50 ) km/hr. hr1000 3600æ ö æ ö= + ´ç ÷ ç ÷è ø è ø

x

50 + x = 220 36001000 6´´

50 + x = 132x = 132 – 50 = 82 km/hr

134. (b) Let principal be represented by P.Ist Case :

S.I. P R T P 8 3

100 100´ ´ ´ ´= =

yoursmahboob.w

ordpress.com

274 SSC CGL SolvedPaper

IInd Case :P R T P 8 3S.I

100 100´ ´ ´ ´= =

According to questionP 8 3 P 8 2– 56

100 100´ ´ ´ ´ =

P 8 56 10056 700100 8

P´ ´= Þ = =

135. (b) Average 7 14 21 28 35 21

5+ + + += =

136. (a) x4 – 2x2 + k(x2)2 – 2x2 + k Þ (x2)2 – 2.1. x2 + kFor above expression to make a perfectsquare, the k value is equal to 1.

137. (d)13 – 6

4x

y=

13 64

xy

= +

Taking 3 common on both sides6 1 123 4.3 12

x xy y

= + Þ = +

Dividing equation by 4 on both sides1 14 8 4 – 8

3 3x x

y y= + Þ =

138. (a) a + b + c = 0i.e. a = – (b + c); b = – (c + a); c = – (a + b)

Now, 2–a b b b c

c c a a+ ++

+

Þ 2[–( )]–

– ( ) – ( )a b c a b ca b c a b c+ + +++ + +

Þ – 1 + 2 – 1 = 0

139. (b) x + 4 4x

=

x2 + 4 = 4x Þ x2 – 4x + 4 = 0 Þ (x – 2)2 = 0x = 2

3 33 3

4 4(2)(2)

xx

+ = +

Þ 4 1 18 8 88 2 2

+ Þ + Þ

140. (b) x = 3 + 2 2x = 2 + 1 + 2 2

x = 2 2( 2) (1) 2.1. 2+ +

x = 2( 2 1)+

( 2 1)x = + ...(1)

1 1 2 –1 2 –1 2 –12 –12 1 2 –1x

= ´ = =+

Now, 1

– 2 1– ( 2 –1)xx

= +

2 1– 2 1= + +1– 2xx

=

141. (c) The least value of 1aa

+ is 2 where a = 1.

142. (b) It a = 0, b ¹ 0, c ¹ 0, then equation ax + by+ c = 0represents a line parallel to x-axis.

143. (a) Let Puneet’s age = x yr.Let Puneet’s father age = y yr.x + y = 45 Þ y = (45 – x)xy = 126Putting the value of y.(x) (45 – x) = 12645x – x2 = 126x2 – 45x + 126 = 0x2 – 42x – 3x + 126 = 0x (x – 42) – 3 (x – 42) = 0x = 3, x = 42Hence, Puneet’s age in 3yr.

144. (a) Central angle of the sector for cost of thepaper

Cost of paper % 360º100

= ´16 360º 57.6º100

= ´ =

145. (d) If the cost of paper = ` x, then from thegiven pie-chart

Cost of paper 16%Miscellaneous charges 4%

=

Þ16 16 6000

6000 4 4x x ´= Þ = = ̀ 24000

147. (c) By observing the graph, we can say that yr.2009 has maximum percentage of studentspassed in the year.

148. (d) Year 2007.150. (c) 20 + 12 + 16 = 48151. (a) Fabricate means make up something

artificial or untrue while demolish meansdestroy completely which is just opposite.

yoursmahboob.w

ordpress.com

275SSC CGL SolvedPaper

152. (c) Gregarious denotes tending to form a groupwith others of the same species andunsociable is the opposite.

153. (a) Pragmatic means concerned with practicalmatters while indefinite denotes vague ornot clearly defined or stated.

154. (b) If somebody is above board, he/she ishonest in any business deal.

155. (b) To cry wolf means that someone is givingfalse alarm.

156. (a) If somebody is on the right/ wrong side of30/ 40 etc that means he/she is younger/older than 30/ 40 etc.

157. (a) If you have an axe to grind; that means youhave a private end to serve.

158. (d) If you drive something home, that meansyou are making something completely clearto someone. She didn't have to drive thepoint home. The movie had done that.

159. (c) Here we compare the climate of Karnatakawith the climate of Tamil Nadu and not withthe Tamil Nadu itself; hence we use 'thanthat of'.

160. (a) If someone suffers from an unpleasant ordifficult experience or situation, then we use'suffer from.' Ex: Shiela is suffering from illhealth. Lately factories are suffering from adesperate shortage of labours.

161. (a) The correct arrangement of sentence is - Igave Sana the keys.

162. (a) When you are using if to talk aboutsomething that is unlikely to happen or isimpossible, use the past tense in the if-clause, not present. Ex: If someone gaveme (NOT gives or would give me) themoney, I'd buy a car tomorrow.

163. (c) 'He compensated me for the loss.' is thecorrect answer.

164. (b) The proposition 'to' is complementary withaccountable.

165. (a) Recently denotes not long ago and thususually takes perfect tense.

166. (c) Present tense shows what exits orhappening now. It also denotes a habitwhich in this case is the timing of havinglunch.

167. (a) Here 'agriculturally' is the adverb that addsto the meaning of the adjective 'strong'.

Other examples are 'slowly' in 'He ran slowly','very' in 'It's very hot', or 'naturally' in'Naturally, we want you to come.'

168. (b) Superior always takes preposition 'to'. Ex:Your computer is far superior to mine.

169. (d) Ambiguous means having more than onepossible meaning.

170. (a) If someone or something is infallible, thatmeans they are incapable of failure or error.

171. (d) Conscientious is the one who is guided byor in accordance with conscience or senseof duty and right and wrong.

173. (c) Obsolete means no longer in use174. (c) regicide means the act of killing a king.175. (a) 0Aviary is a building where birds are kept.176. (d) 177. (c) 178. (a) 179. (c)180. (b) Baltimore Oriole is of 7 inches in length.181. (c) 182. (c) 183. (d) 184. (a)185. (c) 186. (d) 187. (a)188. (b) Here Mohan's eyes reflect means that it was

Mohan's habit which is not the case. Hence,it should be as Mohan's eyes reflected .....

189. (c) With a view to (doing) something becauseyou are planning to do something in thefuture. Ex: We bought the house with a viewto retiring there.

190. (b) 'has come' is the correct option because theverb will agree with the first subject.

191. (b) adjacent always takes the preposition 'to'.192. (b) Grapes cannot be the subject of the

sentence; hence it should be 'Grapescannot be gathered......'

193. (b) Since is usually followed by a timeexpression ('last year', 'this morning', '4o'clock' etc) or by a clause in the simplepast tense. Use the present perfect or thepast perfect in the other clause. Use forwhen you state the length of time thatsomething has been or had beenhappening. Ex: We have known each otherfor ten years (NOT since ten years).

194. (c) 195. (a) 196. (a) 197. (d)198. (a) Condone and forgive are similar in meaning.199. (b) Analogy denotes comparison.200. (c) Allure and entice both denote provoking

someone to do something through (oftenfalse or exaggerated) promises or persuasion.

yoursmahboob.w

ordpress.com

276 SSC CGL SolvedPaper

PART-A : GENERAL INTELLIGENCE & REASONING

Directions (1-4) : In each of the following questions,select the related number/word/letter from givenalternatives.

1. 9 : 7 : : 80 : ?(a) 48 (b) 50(c) 78 (d) 82

2. Defer : Put off: : Deduce : ?(a) Infer (b) Protect(c) Delate (d) Oppose

3. Fish : Gills : : Human : ?(a) Heart (b) Lungs(c) Nose (d) Mouth

4. CARD : IGXJ : : POST : ?(a) UTXY (b) VUYZ(c) UTYZ (d) VUXY

Directions (5-6) : In each of the following questions,which one set of letters when sequentially placed atthe gaps in the given letter series shall complete it ?

5. a _ _ b abba _ abb a _ ba(a) abab (b) abba(c) aabb (d) aaab

6. AZB–, AZ – Y, A – BY, – ZBY(a) YBZA (b) BYAZ(c) BZYA (d) AZBY

7. Find out the wrong number in the sequence102, 101. 98, 93, 86, 74, 66, 53(a) 101 (b) 66(c) 74 (d) 93

Directions (8-11) : In each of the following questions,select the one which is different, from other threeresponses.

8. (a) Soldier – Barrack (b) Principal – School(c) Artist – Troupe (d) Singer – Chorous

9. (a) Detach (b) Affix(c) Append (d) Fasten

10. (a) Man = 13114 (b) Ram = 18113(c) Jug = 10217 (d) Hub = 82110

11. (a) 74, 7 (b) 97, 9(c) 41, 4 (d) 63, 6

12. The set which resembles the given set 1, 5, 12 is(a) 7, 11, 35 (b) 4, 8, 24(c) 10, 14, 44 (d) 9, 13, 42

Directions (13-16) : In each of the following questions,a series is given, with one term missing. Choose thecorrect alternative from the given ones that willcomplete the series.

13. ELFA, GLHA, ILJA, ? MLNA(a) OLPA (b) KLMA(c) LLMA (d) KLLA

14. 4, 18, 48, ? , 180(a) 80 (b) 100(c) 105 (d) 125

15. FGHJKLNOPRS ?(a) T (b) M(c) Q (d) U

16. b – 0,y – 3, c – 8, x – 15, d – 24, ? ,(a) e – 48 (b) w – 35(c) w – 39 (d) v – 30

17. The average age of father and his son is 22 years.The ratio of their ages is 10 : 1 respectively. Whatis the age of the son ?(a) 24 (b) 4(c) 40 (d) 14

18. Veni is an year older than Smith. Smith is twoyears older than Salim. Raju is an year older thanSalim. Who is the youngest of all ? .(a) Raju (b) Salim(c) Veni (d) Smith

19. Rachel starts walking towards North. Afterwalking 15 metres, she turns towards South andwalks 20 metres. She then turns towards Eastand walks 10 metres. Then, again, she walks5 metres towards North. How far is she from herstarting point and in which direction ?(a) 10 metres, West (b) 5 metres, East(c) 5 metres. North (d) 10 metres, East

SSC Combined Graduate Level (CGL) Solved PaperMORNING SHIFT 21 MAY 2013

yoursmahboob.w

ordpress.com

277SSC CGL SolvedPaper

Directions (20-21): In each of the following questions,from the given alternative words, select the wordwhich cannot be formed using the letters of the givenword.

20. HETEROGENOUS(a) REGENT (b) GENERATE(c) STORE (d) ROUTE

21. UNEQUIVOCAL(a) EQUAL (b) VOCAL(c) QUAINT (d) NAIVE

22. A group of alphabets are given with each beingassigned a number. These have to beunscrambled into a meaningful word and correctorder of letter may be indicated from the givenresponses.YMLOSBCI(a) 47685321 (b) 51264387(c) 21645387 (d) 56241387

23. In a certain code, LONDON is coded as24 – 30 – 28 – 8 – 30 – 28. How will FRANCEbe coded?(a) 10 – 24 – 6 – 28 – 6 – 12(b) 12 – 26 – 6 – 28 – 8 – 10(c) 12 – 36 – 2 – 28 – 6 – 10(d) 12 – 26 – 2 – 28 – 8 – 10

24. If each of the letters in the English alphabet isassigned odd numerical value beginning A = 1,B = 3 and so on, what will the total value of theletters for the word 'HOTEL' ?(a) 95 (b) 115(c) 125 (d) 105

25. In a certain code, MAARK is written as KRAAM.How PAS- SI can be written in that code?(a) ISSAP (b) ISSPA(c) SSIPA (d) ASS1P

26. If 29 × 48 = 576, 35 × 16 = 90, 22 × 46 = 96, then42 × 17 = ?(a) 56 (b) 286(c) 48 (d) 64

27. If 'P'denotes' 'multiplied by', 'T' denotes 'subtractedfrom', 'M' denotes 'added to' and 'B' denotes'divided by' then : what should be the correctresponse of12P6M 15 T 16 B 4 ?(a) 70 (b) 75(c) 83 (d) 110

28. If + = Greater than, f = Not greater than, – = Notless than, × = Equal to, | = Less than and L = Notequal to, then of A | B × C which of the followingis true ?(a) B + C | A (b) C – B + A(c) B | A | C (d) A f B | C

29. Identify one response which would be a correctinference from the given premises statedaccording to the following symbols :'A' stands for not greater than'B' stands for equal to'C' stands for less than'D' stands for not less than'E' stands for not equal to'F' stands for greater than Premises (2 M B N)and (2N A 3K)(a) 2M D 3K (b) 2M B 3K(c) 2M C 3K (d) 2K B 3N

Directions (30-34) : In each of the following questions,select the missing number from the given responses:30. 54 30 112 42 ? 28 24 70 38

(a) 176 (b) 166(c) 116 (d) 66

31. 216 209 202522 515 508633 626 ?(a) 620 (b) 608(c) 602 (d) 619

32. 15 12 44 28 64 53 3 8 ?(a) 30 (b) 13(c) 70 (d) 118

33. 95, 115, 145, 155, ?(a) 215 (b) 175(c) 185 (d) 165

34. 1, 3, 8, 19, 42, _?(a) 65 (b) 71(c) 89 (d) 93

35. Rohan walks a distance of 3 km towards North,then turns to his left and walks for 2 km. He againturns left and walks for 3 km. At this point heturns to his left and walks for 3 km. How manykilometres is he from the starting point ?(a) 1 km (b) 2 km(c) 3 km (d) 4 km

yoursmahboob.w

ordpress.com

278 SSC CGL SolvedPaper

36. Rani and Sarita started from a place X. Rani wentWest and Sarita went North, both travelling withthe same speed. After sometimes, both turnedtheir left and walked a few steps. If they againturned to their left, in which directions' the facesof Rani and Sarita will be with respect to X ?(a) North and East (b) North and West(c) West and North (d) East and South

Directions (37-38) : In each of the following questions,one/two statements are given followed by threeconclusions. You have to consider the statements tobe true even it they seem to be at variance fromcommonly known facts. You have to decide which ofthe given conclusions, if any, follow from the givenstatements.37. Statement 1 : Water has no shape, has volume.

Statement 2 : The knowledge is, like water, flowedfrom one side to other.Conclusion 1 : The knowledge is interdisciplinaryConclusion 2 : The knowledge is bound within aspecific area.Conclusion 3 : The knowledge influences thecore of mental activity directly.(a) Only Conclusion 1 follows.(b) Only Conclusion 2 follows.(c) Both Conclusions 1 and 2 follow.(d) Both conclusions 1 and 3 follow.

38. Statements.:Some ladies are beautiful.Some beautifuls are honest.All honest are sensitives.Conclusion 1 : Some sensitives are beautifuls.Conclusion 2 : Some honest are ladies.Conclusion 3 : Some sensitives are ladies.(a) None of the Conclusion follows.(b) Only Conclusion 1 follows.(c) Only Conclusions 1 and 2 follow.(d) All Conclusions follow.

39. From the given blocks when 10 is at the bottom,which number will be at the top ?

10

2

412

68

(a) 8 (b) 12(c) 6 (d) 4

40. Which one of the answer figures shall completethe given question figure ?Question Figure:

Answer Figures:

(a) (b) (c) (d)41. Which of the four cubes pictured below are

correct view when the six squares are folded (inclockwise direction) into a cube?Question Figure:

W 3O

1

Answer Figures:

3 3 1 1wo

w

(a) (b) (c) (d)

ow

42. Which one of the following diagrams bestdepicts the relationship among pen, pencils,stationery?

(a) (b)

(c) (d)

43. Indicate which figure will best represent therelationship amongst the three :Legumes Seeds, Peas, Kidney Beans

(a) (b)

(c) (d)

yoursmahboob.w

ordpress.com

279SSC CGL SolvedPaper

44. Which one of the following diagrams bestdepicts the relationship among.Black eyed people, Brown haired people, Indian

(a) (b)

(c) (d)

45. Find out which of the answer figures will exactlymake up the question figure?Question Figure:

Answer Figures:

=

(a) (b) (c) (d)

46. Find out which of the following answer figureswill exactly make up the question figure ?Question Figure :

Answer Figures.

(a) (b) (c) (d)

47. Which one of the answer figures is hidden in thefollowing question figure?Question Figure :

Answer Figures.

(a) (b) (c) (d)48. A piece of paper is folded and punched as shown

below in the question figures. From the givenanswer figures, indicate how it will appear whenopened.Question Figures :

Answer Figures.

(a) (b) (c) (d)49. From the answer figures, find out the figure which

is the exact mirror image of the question figure,when the mirror is placed on the line MN.Question Figures :

M

N

Answer Figures.

(a) (b) (c) (d)

yoursmahboob.w

ordpress.com

280 SSC CGL SolvedPaper

50. In the following question, a word is representedby a set of numbers as given in any one of thealternatives. The sets of numbers given in thealternatives are represented by alphabets as inthe matrices given below. The columns and rowsof matrix are numbered from 1 to 6. A letter fromthese matrices can be represented first by its rowand next by its column number. e.g., 'A' can berepresented by 42. 'S' can be represented by 21,etc. Similarly. you have to identify the set for theword 'PLAY'.

1 2 3 4 5 6J H R E I P S2 S G N D Z I3 B U F T K L4 V A P C Y A5 H W C O X N6 B A E F L Q

(a) 43, 36, 42, 23 (b) 43, 32, 33, 33(c) 15, 12, 42, 45 (d) 43, 65, 62, 45

PART-B : GENERAL AWARENESS51. Who established the Indian Civil Liberties Union

in 1936?(a) Subhash Chandra Bose(b) Bal Gangadhar Tilak(c) Jawahar Lal Nehru(d) Rajendra Prasad

52. Which of the following was established first?(a) Banaras Hindu University(b) University of Bombay(c) Aligarh Muslim University(d) University of Allahabad

53. "Yosemite" is a(a) River (b) Peak(c) Waterfall (d) Dam

54. The first Indian Satellite Aryabhatta waslaunched in(a) 1972 (b) 1975(c) 1977 (d) 1979

55. Where is the shore based steel plant located?(a) Tuticorin (b) Salem(c) Vishakhapatnam (d) Mangalore

56. Which two of the following are connected bythe North South corridor?(a) Srinagar and Kanyakumari(b) Mumbai and Chennai(c) Amritsar and Kolkata(d) Hyderabad and Bhopal

57. The pollutants which move downward withpercolating ground water are called(a) Leachates (b) Pollutates(c) Earthites (d) Percolates

58. Lungs are located in the(a) abdominal cavity (b) pericardial cavity(c) peritoneal cavity (d) pleural cavity

59. Which one of the following is the ideal food fornewborn babies?(a) Water (b) Sugar(c) Honey (d) Milk

60. Transcription means the synthesis of(a) Lipids (b) Protein(c) DNA (d) RNA

61. Hydrochloric acid is secreted by the cells liningthe(a) Oral cavity (b) Stomach(c) Ileum (d) Colon

62. Emulsification is(a) breaking fats into small globules(b) digestion of fats(c) absorption of fats(d) storage of fats

63. Taxonomy is a science that deals with(a) Morphology (b) Anatomy(c) Classification (d) Economic uses

64. Which of the following is responsible for theworking of Newton's colour disc experiment?(a) Formation of pure spectra(b) Formation of impure spectra(c) Persistence of vision(d) Principle of complementary colour

65. The dimension MLT–2 corresponds to(a) force (b) work done(c) acceleration (d) velocity

66. Who is the founder of quantum theory ofradiation?(a) Einstein (b) Bohr(c) Plank (d) S.N. Bose

67. Fiber optics cable used in communication, workson the principle of(a) regular reflection of light(b) diffuse reflection of light(c) refraction of light(d) total internal reflection of light

68. Which was the first electronic computerconstructed at the Moore School of Engineering?(a) EOVAC (b) ONIVAC(c) ENIAC (d) EDSAC

yoursmahboob.w

ordpress.com

281SSC CGL SolvedPaper

69. Which among the following standard protocolsis the most widely used by the Internet?(a) HTTP (b) TCP/IP(c) SMTP (d) SLIP

70. The outer skin most of the crustaceans are madeup of a carbohydrate. This carbohydrate is(a) cellulose (b) galactose(c) chitin (d) starch

71. Rutherford's scattering experiment proved thepresence of(a) atoms in all matter(b) electrons in atoms(c) neutrons in atoms(d) nucleus in atoms

72. When a metal is heated in a flame, the electronsabsorb energy and jump to higher energy state.On coming back to the lower energy state, theyemit light, which we can observe in(a) Raman spectra (b) Absorption spectra(c) Emission spectra (d) Fluorescence

73. Blood pressure may be increased by theexcessive secretion of(a) Thyroxine (b) Testosterone(c) Estradiol (d) Estrol

74. The Concept of 'Green House Gases" waspostulated by(a) Joseph Furier (b) Abdul Kalam(c) M. S. Swaminathan (d) Richael Carlson

75. "Bhopal gas tragedy" 1984 is related to(a) Aluminium Phosphide(b) Methyl bromide(c) Methyl isocyanate(d) Carbon dioxide

76. The Particulate Matter (PM–10) exhaled from thepolluted atmosphere is often filtered out duringthe process of(a) Coughing (b) Sneezing(c) A and B (d) Urination

77. Sarus crane is the state bird of(a) Rajasthan (b) Uttar Pradesh(c) Madhya Pradesh (d) West Bengal

78. The Tongario volcano erupted on November 21,2012 is in(a) Australia (b) Indonesia(c) Papua New Guinea (d) New Zealand

79. The Daocheng Yading Airport is located in(a) Thailand (b) Philippines(c) China (d) Tibet

80. BCCI named the "Indian Cricketer of the year2011–12" to honour(a) Sunil Gavaskar (b) V V S Laxman(c) Virat Kohli (d) Yuvaraj

81. "Martyr's Day" is marked on(a) January 1 (b) January 15(c) January 30 (d) January 9

82. Who won the World Carrom Championship 2012?(a) Rashmi Kumari(b) Mirabai Chanu(c) Nishantha Fernando(d) Nuthaki Priyanka

83. The animal who can consume more salt amongthe following is(a) Sheep (b) Camel(c) Donkey (d) Dog

84. Which of the following is only domestic Airport?(a) Dabolin Airport, Goa(b) Srinagar Airport(c) Devi Ahilyabai Holkar Airport(d) None of the above

85. Tick the correct option with regards to thecontribution towards GDP (Gross DomesticProduct) from Agriculture(a) During 1950–51 (GDP 51-88%) and 2011–12

(GDP 14-01%)(b) During 1950–51 (GDP 11-00%) and 2011–12

(GDP 25%)(c) During 1990–91 (GDP 29-53%) and 2011–12

(GDP 66-77%)(d) During 1980–81 (GDP 35-69%) and 2011–12

(GDP 20-69%)86. Second India–Africa Forum Summit–2011 was

held in(a) Eretrea (b) Ethiopia(c) Sudan (d) Nigeria

87. Where did Aurangzeb die?(a) Pune (b) Aurangabad(c) Ahmad Nagar (d) Mumbai

88. The principle of maximum social advantage is thebasic principle of(a) Micro Economics(b) Macro Economics(c) Fiscal Economics(d) Environmental Economics

89. Which Five Year Plan is not correct among thefollowing?(a) First 1951–56 (b) Second 1956–61(c) Third 1961–66 (d) Fourth 1966–71

yoursmahboob.w

ordpress.com

282 SSC CGL SolvedPaper

90. The ordinary and maximum tolerance limit ofsound by human being is(a) 50 db to 70 db (decibel)(b) 60 db to 80 db (decibel)(c) 65 db to 75 db (decibel)(d) 70 db to 85 db (decibel)

91. An economic theory is a/an(a) Axion(b) Proposition(c) Hypothesis(d) Tested hypothesis

92. Indian Special Economic Rules amendment camein the year(a) 2000 (b) 2002(c) 2004 (d) 2006

93. Provisions of citizenship in Indian Constitution,became applicable in(a) 1950 (b) 1949(c) 1951 (d) 1952

94. Who gave the title of "Sardar" to Ballabh BhaiPatel?(a) Mahatma Gandhi (b) Vinoba Bhave(c) Women of Bardoli (d) Peasants of Gujrat

95. According to Marx, the source of value is(a) Capital (b) Land(c) Labour (d) None of the above

96. The National Emergency in India declared by thePresident of India due to the external aggressionor armed revolt through(a) Article–352 (b) Article–356(c) Article–360 (d) Article–368

97. The Community Development Programme waslaunched in the year(a) 1950 (b) 1952(c) 1951 (d) 1953

98. What Satyagraha was held at Nagpur in 1923?(a) Salt Satyagraha(b) Individual Satyagraha(c) Ryots Satyagraha(d) Flag Satyagraha

99. Which one of the following is not a sect ofBuddhism?(a) Mahayana (b) Hinayana(c) Digambar (d) Theravad

100. Who was the viceroy when Delhi became thecapital of British India?(a) Lord Curzon (b) Lord Minto(c) Lord Hardinge (d) Lord Waveli

PART-C : QUANTITATIVE APTITUDE101. The fourth root of 24010000 is

(a) 7 (b) 49(c) 490 (d) 70

102. The greatest 4 digit member which is a perfectsquare,is(a) 9999 (b) 9909(c) 9801 (d) 9081

103. A piece of work can be done by Ram and Shyamin 12 days, by Shyam and Hari in 15 days and byHari and Ram in 20 days. Ram alone will completethe work in(a) 130 days (b) 32 days(c) 36 days (d) 42 days

104. 3 men or 5 women can do'a work in 12 days. Howlong will 6 men and 5 women take to finish thework? .(a) 4 days (b) 5 days(c) 6 days (d) 7 days

105. A and B can do a job in 12 days. B and C in 15days and C and A in 20 days. How long would Atake to do that work ?(a) 20 days (b) 60 days(c) 30 days (d) 40 days

106. The difference of perimeter and diameter of acircle is X unit. The diameter of the circle is

(a)X

1p -unit (b)

X1p +

unit

(c)Xp

unit (d)X

1æ ö-ç ÷è øp unit

107. The perimeter of the base of a right circularcylinder is 'a' unit. if the volume of the cylinder isV cubic unit. then the height of the cylinder is

(a)24a Vp

unit (b)24 a

Vp unit

(c)2a V4p unit (d) 2

4 Vap

unit

108. A sphere of diameter 6 cm is dropped in a rightcircular cycindrical vessel partly filled with water.The diameter of the Cylindrical vessel is 12 cm. Ifthe sphere is just completely submerged in water,then the rise of water level in the cylindrical vessel is(a) 2 cm (b) 1 cm(c) 3 cm (d) 4 cm

yoursmahboob.w

ordpress.com

283SSC CGL SolvedPaper

109. A shopkeeper marks his goods 20% above hiscost price and gives 15% discount on the markedprice. His gain percent is(a) 5% (b) 4%(c) 2% (d) 1%

110. A shopkeeper earns a profit of 12% on selling abook at 10% discount on printed price. The ratioof the cost price to printed price of the book is(a) 45 : 56 (b) 50 : 61(c) 90 : 97 (d) 99 : 125

111. The list price of an article is ̀ 160 and a customerbuys it for `122.40 after two successivediscounts. If the first discount is 10%, thensecond discount is(a) 12% (b) 10%(c) 14% (d) 15%

112. In a school, 10% of number of girls is equal to20th of number of boys. Ratio between the numberof boys to number of girls is(a) 1 : 2 (b) 2 : 1(c) 1 : 4 (d) 4 : l

113. The ratio of 252.5 : 53 is same as(a) 5 : 3 (b) 5 : 6(c) 1 : 25 (d) 25 : l

114. If a, b, c, d, e are five consecutive odd numbers,their average is

(a) 5 (a + 4) (b)5

abcde

(c) 5 (a + b + c + d + e) (d) a + 4

115. The average of 20 numbers is 15 and the averageof first five is 12. The average of the rest is(a) 16 (b) 15(c) 14 (d) 13

116. A tradesmen sold an article at a loss of 20%. ifthe selling price had been increased by ̀ 100. therewould have been a gain of 5%. The cost price ofthe article (in ̀ ) was(a) 100 (b) 200(c) 400 (d) 500

117. The price of an article is first decreased by 20%and then increased by 30%. if the resulting priceis ` 416, the original price of the article is.(a) ` 350 (b) ` 405(b) `400 (d) ` 450

118. A man performs 2

15 of the total journey by train,

220

by bus and the remaining 10 km on foot.

His total journey in km is(a) 15.6 (b) 24(c) 16.4 (d) 12.8

119. By walking at 34

of his usual speed, a man reaches

his oflice 20 minutes later than usual. His usualtime is(a) 30 min. (b) 75 min.(c) 90 min. (d) 60 min.

120. If the compound interest on a certain sum fortwo years at 12% per annum is ̀ 2,544, the simpleinterest on it at the same rate for 2 years will be(a) ` 2,400 (b) ` 2,500(3) ` 2,480 (d) ` 2.440

121. The total cost of 8 buckets and 5 mugs is ` 92and the total cost of 5 buckets and 8 mugs is` 77. Find the cost of 2 mugs and 3 buckets.(a) ` 35 (b) ` 70(c) ` 30 (d) ` 38

122. If a b a

1 a 1 b 1 c+ +

- - - = 1, then the value of

1 1 11 a 1 b 1 c

+ +- - -

is

(a) 1 (b) 3(c) 4 (d) 0

123. If (x – 3)2 + (y – 5)2 + (z – 4)2 = 0, then the value of2 2 2x y z

9 25 16+ + is

(a) 12 (b) 9(c) 3 (d) 1

124. If 4x3

+ 2P = 12 for what value of P, x = 6 ?

(a) 6 (b) 14(c) 2 (d) 1

125. The value of 4 3 37 4 3

++

is

(a) 5 3 8- (b) 5 3 8+(c) 8 3 5+ (d) 8 3 5-

yoursmahboob.w

ordpress.com

284 SSC CGL SolvedPaper

126. If 2 3

x 3x x

æ ö- =ç ÷è ø , then the value of 2

21

xx

+ is

(a)1

29

(b)4

29

(c)139

(d)439

127. What number must be added to the expression16a2 –12a to make it a perfect square ?

(a)94

(b)132

(c)112 (d) 16

128. The straight line 2x + 3y = 12 passes through :(a) lst, 2nd and 3rd quadrant(b) lst, 2nd and 4th quadrant(c) 2nd, 3rd and 4th quadrant(d) lst, 3rd and 4th quadrant

129. The sum of three altitudes of a triangle is(a) equal to the sum of three sides(b) less than the sum of sides(c) greater than the sum of sides(d) twice the sum af sides

130. In DABC, ÐA + ÐB = 65°, ÐB + ÐC = 140°, thenfind ÐB.(a) 40° (b) 25°(c) 35° (d) 20°

131. The length of the tangent drawn to a circle ofradius 4 cm from a point 5 cm away from the centreof the circle is(a) 3 cm (b) 4 2 cm

(c) 5 2 cm (d) 3 2 cm132. A cyclic quadrilateral ABCD is such that

AB = BC, AD = DC, AC ̂ BD, ÐCAD = q. Thenthe angle ÐABC =

(a) q (b)2q

(c) 2 q (a) 3 q133. The height of an equilateral triangle is 15 cm.

The area of the triangle is(a) 50 3 sq. cm. (b) 70 3 sq. cm.

(c) 75 3 sq. cm. (d) 150 3 sq. cm.

134. Two parallel chords of a circle, of diameter 20 cmlying on the opposite sides of the centre are oflengths 12 cm and 16 cm. The distance betweenthe chords is(a) 16cm (b) 24cm(c) 14 cm (d) 20 cm

135. In DABC, DE || AC. D and E are two points on ABand CB respectively. If AB = 10 cm and AD 2 4cm, then BE: CE is(a) 2 : 3 (b) 2 : 5(c) 5 : 2 (d) 3 : 2

136. A, B and C are the three points on a circle suchthat the an gles subtended by the chords ABand AC at the centre O are 90° and 1 10°respectively. ÐBAC is equal to(a) 70° (b) 80°(c) 90° (d) 100°

137. 1f the angles of elevation of a balloon from twoconsecutive kilometre–stones along a road are30° and 60" respectively, then the height of theballoon above the ground will be

(a)3

2 km (b)

12

km

(c)23 km (d) 3 3 km

138. Evaluate : 3 cos 80° cosec 10° + 2 cos 59° cosec31°(a) 1 (b) 3(c) 2 (d) 5

139.35pæ ö

ç ÷è ø radians is equal to

(a) 100° (b) 120°(c) 108° (d) 180°

140. If tan q + cot q = 2, then the value of tan2q + cot2qis(a) 2 (b) 1(c) 2 (d) 0

141. The eliminant of q from xcos q – y sinq = 2 andx sin q + y cos q = 4 will give(a) x2 + y2 = 20 (b) 3x2 + y2 = 20(c) x2 – y2 = 20 (d) 3x2 – y2 = 10

142. sin2q – 3 sin q + 2 = 0 will be true if(a) 0 £ q < 90 (b) 0 < q < 90(c) q = 0° (d) q – 90°

yoursmahboob.w

ordpress.com

285SSC CGL SolvedPaper

143. The value of

2 2

2 2cos A(sin A cosA) sin A(sin A cosA)

cosec A(sin A cos A) sec A(sin A cos A)

é ù+ -+ê ú- +ê úë û

(sec2 A – cosec2 A)(a) l (b) 3(c) 2 (d) 4

Directions (144 – 148) : The sub divided bar diagramgiven below depicts H.S. Students of a school forthree years. Study the diagram and answer thequestions.

250

200

150

100

50

02008 2009 2010

Year

Num

bero

fStu

dent

s

Fail 3rd Division2nd Division1st Division

144. The percentage passed in 1st division in 2008was(a) 27% (b) 32%

(c)315 %8

(d)1311 %17

145. The pass percentage in 2008 was(a) 67% (b) 73%

(c)279 %3

(d)682 %

17146. In which year the school had the best result for H.S.

in respect of percentage of pass candidates ?

(a) 2008(b) 2009(c) 2010(d) The percentage of pass candidates are samefor the three years.

147. The number of students passed in third divisionin the year 2008 was(a) 50 (b) 60(c) 70 (d) 80

148. The percentage of the students passed in 2nddivision in the year 2010 was(a) 30% (b) 40%(c) 50% (d) 60%

Directions (149–150) : Following figure is Pie–chartrepresenting itemwise cost of manufacturing certainproduct. Study the chart and answer the questions.

Cost ofLabour151.2°

Cost ofMaterials

144°

Overhead

Expenses

57.6°

DirectExpenses

43.2°

149. Total manufacturing cost is ̀ 96.000. Then, costof labour is(a) ̀ 30,720 (b) ` 38,400(c) ̀ 11,520 (d) ` 15,000

150. The difference of cost of material and directexpenses is(a) ` 26,000 (b) ` 10,000(c) ` 26,500 (d) ` 26,880

PART-D : ENGLISH COMPREHENSION

Directions (151–155): In the following questions,some parts of the sentences have errors and some arecorrect. Find out which part of a sentence has anerror and blacken the oval [ ] corresponding tothe appropriate letter (a, b, c). If a sentence is freefrom error, blacken the ovan corresponding to (d) inthe Answer Sheet.

yoursmahboob.w

ordpress.com

286 SSC CGL SolvedPaper

151.In India

(a)

working woman lead a life of dual responsibilities

(b)

if they are married and have a family(c)

No error(d)

152. Greatly to our surprise

(a) We find the ringleader

(b)

was lame(c)

Noerror(d)

153.They have

(a)

played a game last week(b) (c)

Noerror(d)

154.The teacher made the boys

(a)

to to the sum(b)

all over again(c)

Noerror(d)

155.Many overseas students

(a) attend colleges

(b)

in the Great Britain(c)

Noerror(d)

Directions (156–160): Sentences are given withblanks to be filled in with an appropriate andsuitable word(s). Four alternatives are suggestedfor each question. Choose the correct alternative outof the four.

156. Student-parking should be __________;students should not be charged to buy parkingstickers.(a) fined (b) free(c) costly (d) cheap

157. If you have roses growing in your garden, youcan make a lovely __________ of flowers athome.(a) bouqutte (b) bucquete(c) bouquete (d) bouquet

158. The __________ of the middle school is a womanof __________.(a) Principles, Principal(b) Principals, Principal(c) Principal, Principle(d) Principle, Principals

159. With the changing times, most of the studentshave become business-like they are __________and want to take only those courses which theyfind rewarding.(a) idealistic (b) pragmatic(c) enthusiastic (d) partial

160. 1. John's at __________ institute studyingFrench.2. They're building __________ school at theend of our street.3. Do they live in __________ UnitedKingdom or somewhere else?(a) a, the, an (b) the, a, an(c) an, a, the (d) the, an, a

Directions (161–163): Out of the four alternatives,choose the one which best expresses the meaning ofthe given .

161. Parsimony(a) expenditure (b) bankruptcy(c) bribery (d) miserliness

162. Tribulation(a) palpitation (b) suffering(c) weakness (d) stimulation

163. The Prime Minister goes on the ramparts of theRed Fort to hoist the National Flag.(a) rompway (b) staircase(c) parapet (d) scaffold

Directions (164–166): Choose the word opposite inmeaning to the given word and mark it in the AnswerSheet.

164. Lunacy(a) sanity (b) stupidity(c) sensibility (d) insanity

yoursmahboob.w

ordpress.com

287SSC CGL SolvedPaper

165. Obtuse(a) sharp-witted (b) transparent(c) timid (d) blunt

166. Inadvertently(a) secretly (b) accidentally(c) completely (d) deliberately

Directions (167–171): Four alternatives are givento the Idiom/Phrase underlined in the sentence.Choose the alternative which best expresses themeaning of the Idiom/Phrase and mark it in theAnswer Sheet.167. What egged you on to become a social worker?

(a) urged (b) dampened(c) hindered (d) discouraged

168. Many politicians in India are not fit to hold acandle to Mahatma Gandhi.(a) superior (b) equal(c) inferior (d) indifferent

169. She must be paying through the nose for theface left.(a) paying less than necessary(b) paying too much(c) paying the right amount(d) paying reluctantly

170. He is putting the cart before the horse bypurchasing furniture before buying a house.(a) doing a thing in the wrong way(b) doing a thing in the right way(c) committing a great crime(d) doing things meticulously

171. Casting pearls before swine.(a) speaking nice words and convincing them(b) offering good things to underserving

people(c) uplifting the needy for their welfare(d) doing worthwhile things to unknown

people

Directions (172–181): A sentence/ part of thesentence is underlined. Below are given alternativesto the underlined sentence/part of the sentence at(a), (b) and (c) which may improve the sentence.Choose the correct alternative. In case noimprovement is needed, your answer is (d).172. It became clear that the strangers were heading

into a serious disaster.(a) along (b) towards(c) on (d) No improvement

173. Twenty kms are not a great distance in these daysof fast moving vehicles.(a) is not a great distance(b) are not too great a distance(c) aren't proving a great distance(d) No improvement

174. I adapted a new method to solve the problem.(a) I have been adopted(b) I adopted(c) I was adapted(d) No improvement

175. Hoping not to be disturbed, I sat down in myeasy chair to read the book, I won as a prize.(a) I had won as a prize(b) I have won as prize(c) I had to win as a prize(d) No improvement

176. If you are living near a market place you shouldbe ready to bear the disturbances caused bytraffic.(a) to bear upon (b) to bear with(c) to bear away (d) No improvement

177. The more they earn, more they spend on luxuryitems.(a) more they should spend(b) the more they spend(c) the more they ought to spend(d) No improvement

178. You have come here with a view to insult me.(a) to insulting me (b) of insulting me(c) for insulting me (d) No improvement

179. A little rail-road engine was employed by a stationyard for doing small pieces of work.(a) was made by a station yard(b) was used at the station yard(c) was employed at the station yard(d) No improvement

180. From an aesthetic point of view, the painting didnot appeal to me.(a) From the viewpoint of aesthetics, the

painting did not appeal to me(b) The painting had no aesthetic appeal to me(c) From an aesthetic point of view, the painting

had a little appeal to me(d) No improvement

181. The child tossed in bed burning with fever.(a) The child in bed, burning with fever tossed(b) The child burning with fever, tossed in bed(c) The child burning in bed tossed with fever(d) No improvement

yoursmahboob.w

ordpress.com

288 SSC CGL SolvedPaper

Directions (182–188): out of the four alternatives,choose the one which can be substituted for the givenwords/sentence.182. An apartment building in which each apartment

is owned separately by the people living in it,but also containing shared areas.(a) condominium (b) multiplex(c) duplex (d) caravan

183. A group of three powerful people.(a) trio (b) tritium(c) trivet (d) triumvirate

184. Operation of the body after death.(a) post-mortem (b) obituary(c) homage (d) mortuary

185. Not allowing the passage of light.(a) oblique (b) opaque(c) optique (d) opulant

186. Science regarding principles of classification.(a) taxidermy (b) taxonomy(c) toxicology (d) classicology

187. A political leader appealing to popular desiresand prejudices(a) dictator (b) tyrant(c) popularist (d) demagogue

188. Enclosed in a small closed space.(a) closophobia (b) clusterophobia(c) claustrophobia (d) liftophobia

Directions (189–190): Four words are given in eachquestion, out of which only one word is correctlyspelt. Find the correctly spelt word and mark youranswer in the Answer Sheet.189. (a) conivance (b) connivanse

(c) connivance (d) conivence190. (a) maintennance (b) manteinance

(c) maintenance (d) mentenance

Directions (191–200): You have a passage with 10questions. Read the passage carefully and choosethe best answer to each question out of the fouralternatives.

PASSAGECYBER BOGEYS

The cyber–world is ultimately ungovernable. This isalarming as well as convenient; sometimes, convenientbecause alarming. Some Indian politicians use this togreat advantage. When there is an obvious failure ingovernance during a crisis they deflect attention fromtheir own incompetence towards the ungovernable.

So, having failed to prevent nervous citizens fromfleeing their cities of work by assuring them of properprotection, some national leaders are now busy tryingto prove to one another, and to panic-prone Indians,that a mischievous neighbour has been using theinteraet and social networking sites to spreaddangerous rumours. And the Centre's automaticreaction is to start blocking these sites and beginelaborate and potentially endless negotiations withGoogle, Twitter and Facebook about access toinformation. If this is the official idea of prompt actionat a time of crisis among communities, then Indianshave more reason to fear their protectors than thenebulous mischief-makers of the cyber world. Wastingtime gathering proof, blocking vaguely suspiciopuswebsites, hurling accusations across the border andworrying about bilateral relations are ways of keepingbusy with inessentials becuase one does not quiteknown what to do about the essentials of a difficultsituation. Besides, only a fifth of the 245 websitesblocked by the Centre mention the people of theNortheast or the violence in Assam. And if a fewmorphed images and spurious texts can unsettle anentire nation, then there is something deeply wrongwith the nation and with how it is being governed. Thisis what its leaders should be addressig immediately,rather than making a wrongheaded display of theirpowers of censorship.It is just as absurd, and part of the same syndrome, totry to ban Twitter accounts that parody despatchesfrom the Prime Minister's Office. To describe suchforms of humour and dissent as "misrepresenting" thePMO–as if Twitter would take these parodies forgenuine despatches from the PMO — makes the PMOlook more ridiculous than its parodists manage to. Withthe precedent for such action set recently by the chiefminister of West Bengal, this is yet another proof thatwhat Bengal thinks today India will think tomorrow.Using the cyber–world for flexing the wrong musclesis essentially not funny. It might even prove to be quitedangerously distracting.191. According to the passage, the cyber-world is

(a) beyond the imagination of people(b) outside the purview of common people(c) not to be governed(d) ungovernable

yoursmahboob.w

ordpress.com

289SSC CGL SolvedPaper

192. The author is of the opinion that(a) the centre should start negotiations with

Google, Twitter and Facebook(b) the centre should help the citizens evacuate

their city(c) the centre should not block the sites(d) the centre should arrest the guilty

193. Which of the following is closest to the meaningof 'nebulous'?(a) confused (b) vague(c) iridescent (d) glowing

194. The author's seriousness regarding the situationcan best be described in the following sentences.Pick the odd one out.(a) Our leaders should display their powers of

censorship when needed(b) If this is the official idea of prompt action at

a time of crisis among communities, thenIndians have more reason to fear theirprotectors than the nebulous mischief makerof the cyber-world

(c) The politicians deflect attention from t heirown incompetence

(d) If a few morphed images and spurious textscan unsettle an entire nation, then there issomething deeply wrong with the nation

195. The word 'spurious' means(a) genuine (b) authentic(c) substantial (d) fake

196. The author warns us against(a) not playing false with the citizens(b) dangers inherent in the cyber-world(c) not using the cyber-world judiciously(d) not protecting the citizens from dangerous

politicians

197. 'Parody' means(a) twist (b) jeopardize(c) ridicule (d) imitate

198. What is the opposite of 'wrong headed'?(a) silly (b) sane(c) insane (d) insensible

199. The passage suggests different ways of keepingthe public busy with 'inessentials'. Pick the oddone out.(a) By blocking websites which are vaguely

suspicious(b) By blaming neighbouring countries across

the border(c) By turning the attention of the people to

violence in Assam(d) By getting involved in a discourse on

bilateral relations200. The following is a list of statements made by the

author of the above passage. Pick the odd oneout.(a) It is absurd to ban Twitter accounts that

parody despatches from the Prime Minister'sOffice

(b) Twitter take these parodies for genuinedespatches from the PMO

(c) To describe such forms of humour as'misrepresenting" the PMO makes the PMOlook more ridiculous

(d) The precedent for such action was setrecently by the chief minister of West Bengal

yoursmahboob.w

ordpress.com

290 SSC CGL SolvedPaper

HINTS & SOLUTIONS

1. (c) (3)2 = 9; (3)2 – 2 = 7Similarly,(3)4 –1 = 81 – 1 = 80(3)4 – 3 = 81 – 3 = 78In simple way, 9 – 2 = 7 and 80 – 2 = 78

2. (a) Defer means 'to delay something until a latertime', 'to postpone'.Put off means 'to put something to a latertime or date'. ' to dalay'.Thus, Defer and Put off are synonymous toone another.Deduce means 'to infer', 'to arrive at facts'.Infer means 'to arrive at a conclusion'.

3. (c) 'Gill' is the opening on the side of a fish'shead through which it breathes.'Nose' is the part of the face above themouth, used for breathing and smelling.

4. (b) C A R D I G X J+6+6+6+6

Similarly,P O S T V U Y Z

+6+6+6+6

5. (a) a a b b a b b a / a a bb a b ba

6. (a) A Z B Y / A Z B Y / A A Z BY / A ZBY7. (c) 102 – 1 = 101; 101 – 3 = 98; 98 – 5 = 93;

93 – 7 = 86

86 – 9 = 7777 – 11 = 6666 – 13 = 53Therefore, the number 74 is wrong in thesequence.

8. (b) Barrack is a large building or group ofbuildings where soldiers live.Principal is the head of school. Troupe is agroup of artists.Chorous is a large group of singers.

9. (a) 'Detach' is different from the other threewords.Detach means 'to separate or removesomething from a larger or longer part'.Affix means 'to stick, fasten or attachsomething'.Append means 'to attach or add something'.

10. (d) M a n

13 1 14¯ ¯ ¯

The position number of letters in the Englishalphabet.

R a m

18 21 13¯ ¯ ¯

J u g

10 21 7¯ ¯ ¯

H u b

8 21 2¯ ¯ ¯

11. (b) The second number in each number is thefirst digit of the first number.The difference between the two digits ofthe first number is 3 in all the number pairsexcept 97, 9.

12. (b) 1 + 5 = 6 and 6 × 2 = 12Similarly, 4 + 8 = 12 and 12 × 2 = 24

13. (d) 2 2 2

2 2 2

E G I

L L L

F H J

A A A

+ + +

+ + +

¾¾® ¾¾® ¾¾®

¾¾® ¾¾® ¾¾®

¾¾® ¾¾® ¾¾®

¾¾® ¾¾® ¾¾®

KLLA

2

2

M

L

N

A

+

+

¾¾®

¾¾®

¾¾®

¾¾®

14. (b) 4 18 48 100 180

+14 +30 +52 +80

+16 +22 +28

+6 +6

15. (a) FGH : H 2 J+¾¾® ; JKL : L 2 N+¾¾®

NOP : P 2 R+¾¾® ; RS : T

yoursmahboob.w

ordpress.com

291SSC CGL SolvedPaper

16. (b) 0 3 8 15 24 35

+3 +5 +7 +9 +11

+2 +2 +2 +2The two consecutive letters are pairs ofopposite letters.b « y; c « x : d « wTherefore, ? = w – 35.

17. (b) Suppose the age of son is x yearsTherefore, age of father = 10x yearsAccording to question10x x

2+

= 22

Þ 11x = 44 \ x = 4411

= 4 years

Age of father = 10 × 4 = 40 years18. (b) Suppose the age of Salim is x years

Age of Raju = x + 1 yearAge of Smith = x + 2 yearsAge of Veni = x + 3 yearsTherefore, Salim is the youngest of all.

19. (d) B

A

C D

E

North

South

West East

10 m

5 m5 m

Startingpoint

15 m

Required distance = AE = 10mRachel is 10 metres towardsEast from the starting point.

20. (b) There is no 'A' letter in the given word.Therefore, the word GENERATE cannot beformed.

21. (c) There is no 'T' letter in the given word.Therefore, the word QUAINT cannot beformed.

22. (b) Y M L O S B C I

1 2 3 4 5 6 7 8¯ ¯ ¯ ¯ ¯ ¯ ¯ ¯

Meaningful word

S Y M B O L I C

5 1 2 6 4 3 8 7¯ ¯ ¯ ¯ ¯ ¯ ¯ ¯

23. (c) L Þ 12; 12 × 2 = 24O Þ 15; 15 × 2 = 30N Þ 14; 14 × 2 = 28D Þ 04; 04 × 2 = 08O Þ 15; 15 × 2 = 30N Þ 14; 14 × 2 = 28Therefore,F Þ 06; 06 × 2 = 12R Þ 18; 18 × 2 = 36A Þ 01; 01 × 2= 02N Þ 14; 14 × 2 = 28C Þ 03; 03 × 2 = 06E Þ 05; 05 × 2 = 10

24. (b) A Þ 1 × 2 – 1 = 1B Þ 2 × 2 – 1 = 3Therefore,H Þ 8 × 2 – 1 = 15O Þ 15 × 2 – 1 = 29T Þ 20 × 2 – 1 = 39E Þ 5 × 2 – 1 = 09

L Þ 12 × 2 – 1 = 23

11525. (a) The letters have been written in the reverse

order.M A A R K Þ K R A A MTherefore,P A S S I Þ I S S A P

26. (a) 29 × 48Þ 2 × 9 × 4 × 8 = 57635 × 16Þ 3 × 5 × 1 × 6 = 9022 × 46Þ 2 × 2 × 4 × 6 = 96Therefore,42 × 17

Þ 4 × 2 × 1 × 7 = 56

27. (c) P TM B

Þ ´ Þ -Þ + Þ ¸

12 P 6 M 15 T 16 B 4 = ?Þ ? = 12 × 6 + 15 – 16 ¸ 4Þ ? = 72 + 15 – 4 = 83

yoursmahboob.w

ordpress.com

292 SSC CGL SolvedPaper

28. (b)

| L+ Þ> f Þ£ - Þ³´ Þ= Þ< Þ¹

A|B × C Þ A < B = C Option (a)B + C ? A Þ B > C < A Option (b)C – B + A Þ C ³ B > A Option (c)B | A | C Þ B < A < C Option (d)A f B | C Þ A £ B M< C

29. (c) A B CD E F

Þ£ Þ= Þ<Þ³ Þ¹ Þ>

2 M B N

Þ 2 M = N Þ M = N2

2 N A 3 KÞ 2 N £ 3K Þ 4M £ 3KOption (a)2 M D 3KÞ 2 M ³ 3K : Not TrueOption (b)2 M B 3 KÞ 2 M = 3 K : Not TrueOption (c)2 M C 3 KÞ 2 M < 3 K : TrueOption (d)2 K B 3 NÞ 2 K = 3 N : Not True

30. (d) 54 30

2430 + 24 = 54112 42

7042 + 70 = 112Therefore, ? 28

38? = 28 + 38 = 66

31. (d) First Row216 – 7 = 209 ; 209 – 7 = 202Second Row522 – 7 = 515 ; 515 – 7 = 508Third Row

633 – 7 = 626; 626 – 7 = 619

32. (b) 15 + 29 = 4444 + (29 – 9) = 6412 + 16 = 2828 + (16 + 9) = 53

3 + 5 = 8 ; 8 + 5 = 13

33. (b) 95 115 145 155 175

+20 +30 +10 +20

–10–10

34. (c) 1 3 8 19 42 89

+2 +5 +11 +23 +47

+3 +6 +12 +24

×2 ×2 ×2

35. (a)

3 km

2 km

3 km

3 km

Required distance = 1 km

36. (d)

Rani

Sarita

Rani is facing towards East and Sarita isfacing towards South.

37. (d) From the statements it is clear thatknowledge is interdisciplinary. Mentalactivity is related to acquired knowledge.Thus, Conclusions 1 and 3 follow.

38. (b)

Ladies

Beautiful

Honest

Sensitive

yoursmahboob.w

ordpress.com

293SSC CGL SolvedPaper

39. (b) From the two views of blocks it is clear thatwhen 10 is at the bottom, number 12 will beat the top.

40. (b)

41. (a) 3 lies opposite .lies opposite W.1 lies opposite .In option (b) 3 is on teh adjacent face ofdot.In option (c) 1 is on the adjacent face of W.In option (d) is on the top and hence thepositions of dot ( ) and W are not correct.

42. (c) Pen is different from Pencil. But both arestationery items.

Pen Pencil

Stationery

43. (b) Pea is different from kidney bean. But bothare Leguminous seeds.

LeguminousSeeds

Pea KidneyBean

44. (a) Some black eyed people may be brownhaired and vice-versa.Some black eyed people may be Indians andvice-versa.Some brown haired people may be Indiansand vice-versa.Some black eyed and brown haried peoplemay be Indians and vice–versa.

Black eyed

people

Brownhairedpeople

Indian

45. (a) All the components of the question figureare present in the Answer Figure (a).

=

46. (b) All the components of the Question Figureare present in the answer figure (b).

47. (c)

48. (a)

49. (d)

50. (d) P Þ 15, 43;L Þ 36, 65;A Þ 42, 46, 62 :Y Þ 45

Option

(a)

(b)

(c)

(d)

P L A Y

43 36 42 23

43 32 33 33

15 12 42 45

43 65 62 45

yoursmahboob.w

ordpress.com

294 SSC CGL SolvedPaper

51. (c) 52. (b) 53. (c) 54. (b)55. (c) 56. (a) 57. (a) 58. (d)59. (d) 60. (c) 61. (b) 62. (a)63. (c) 64. (c) 65. (a) 66. (c)67. (d) 68. (c) 69. (b) 70. (c)71. (d) 72. (c) 73. (a) 74. (a)75. (c) 76. (b) 77. (b) 78. (d)79. (d) 80. (c) 81. (c) 82. (c)83. (b) 84. (d) 85. (a) 86. (b)87. (c) 88. (c) 89. (d) 90. (b)91. (b) 92. (d) 93. (a) 94. (a)95. (c) 96. (a) 97. (b) 98. (d)99. (c) 100. (c)101. (d) 24010000 = 4900

Again, 4900 = 70

\ 4 24010000 = 70102. (c) 99 × 99 = 9801

103. (a) (Ram's + Shyam's) 1 day's work = 1

12

(Shyam's + Hari's) 1 day's work = 1

15

(Hari's + Ram's) 1 day's work = 120

Adding all three,(2 Ram's + Shyam's + Hari's)1 day's work

= 1 1 1

12 15 20+ + =

5 4 3 160 5

+ + =

\ (Ram's + Shyam's + Hari's)

1 day's work = 1

10

\ Ram's 1 day's work = 1 1 3 2 1

10 15 30 30-- = =

\ Ram alone will do the work in 30 days.104. (a) 3 men = 5 women

6 men + 5 women = 15 women\ By M1D1 M2D25 × 12 = 15 × D2

D2 = 5 12

15´

= 4 days

105. (c) (A + B)'s 1 day's work = 1

12

(B + C)' 1 day's work = 1

15

(C + A)' 1 days' work = 120

\ (A + B + C)'s 1 day's work = 1

10

\ A' 1 day's work = 1 1

10 15- =

3 2 130 30-

=

106. (a) If the diameter of the circle be d units, thenp d – d = xÞ d (p –1) = x

Þ d = x

1p - units

107. (d) If the radius of base of cylinder be r unitsand its height be h units, then

2p r = a Þ r = a

2p units

\ Volume of cylinder = pr2h

Þ V = p × 2

2a h

p Þ h = 2

4 Vap

units

108. (b) Volume of sphere

= 34r

3p =

43 3 3

3p ´ ´ ´

= 36 p cu. cm.If the water level rises by h cm, thenpR2h = 36pÞ 6 × 6 × h = 36Þ h = 1 cm

109. (c) If the C.P. of goods be `100, thenMarked price = ̀ 120

\ S.P. = 120 85

100´

= ̀ 102

\ Profit per cent = 2%110. (a) C.P. of the book = ` x

Printed price = ̀ y

\ y 90 112

x100 100´

= ´

Þ x 90 45y 112 56

= =

111. (d) S.P. after a discount of 10%

= 160 90

100´

= ̀ 144

Second discount = 144 – 122.40 = ̀ 21.6If the second discount be x %, then

144 x100

´ = 21.6 Þ x =

21.6 100144

´ = 15%

yoursmahboob.w

ordpress.com

295SSC CGL SolvedPaper

112. (b) If boys = x and girls = y, then

y × 10 x

100 20= Þ

y x10 20

=

Þ x 20 2y 10 1

= =

113. (d) (52)2.5 : 53 252.5 : 53

= 55 : 53 = 52 : 1 = 25 : 1114. (d) b = a + 2

c = b + 2 = a + 4d = c + 2 = a + 6e = d + 2 = a + 8\ Required average

= a a 2 a 4 a 6 a 8

5+ + + + + + + +

= 5a 20

5+

= a + 4

115. (a) If the average of remaining numbers be x,then20 × 15 = 5 × 12 + 15xÞ 300 = 60 + 15xÞ 15x = 300 – 60 = 240

Þ x = 24015

= 16

116. (c) C.P. of article = ` x

\ First S.P. = 80x100

= ̀ 4x5

Case II,4x x 105 21x1005 100 20

´+ = =

Þ 21x 4x20 5

- = 100

Þ 21x 16x

20-

= 100

Þ 5x = 2000

Þ x = 2000

5 = ̀ 400

117. (c) If the original price of article be ̀ x, then

x × 80 130

100 100´ = 416

Þ x = 416 100 100

80 130´ ´

´ = ̀ 400

118. (b) If the total journey be of x km, then2x 9x 10 x15 20

+ + =

Þ x – 2x 9x15 20

- = 10

Þ 60x 8x 27x

60- -

= 10 Þ 25x60

= 10

Þ x = 60 10

25´

= 24 km

119. (d) New speed = 34

× usual speed

\ New time = 43

× usual time

\ 13

× usual time= 20 minutes

Þ Usual time = 3 × 20 = 60 minutes

120. (a) C.I. = P TR1 1

100

é ùæ ö+ -ê úç ÷è øê úë û

Þ 2544 = 212P 1 1

100

é ùæ ö+ -ê úç ÷è øê úë û

Þ 2544 = P 228 1

25

é ùæ ö -ê úç ÷è øê úë û

Þ 2544 = P 784

1625

æ ö-ç ÷è ø

Þ 2544 = P 784 625

625-æ ö

ç ÷è ø = P 159

625´

Þ P = 2544 625

159´

= ̀ 10000

\ S.I. = P R T

100´ ´

= 10000 2 12

100´ ´

= ̀ 2400

121. (a) C.P. of 1 bucket = ̀ xC.P. of 1 mug = ̀ y\ 8x + 5y = 92 ...(i)5x + 8y = 77 ...(ii)By equation (i) × 5 – equation (ii) × 8,40x + 25y – 40x – 64y = 460 – 616Þ –39y = –156Þ y = 4

yoursmahboob.w

ordpress.com

296 SSC CGL SolvedPaper

From equation (i),8x + 20 = 92Þ 8x = 92 – 20 = 72Þ x = 9\ C.P. of 2 mugs and 3 buckets= 2 × 4 + 3 × 9= 8 + 27 = ̀ 35

122. (c)a b c

1 a 1 b 1 c+ +

- - - = 1

Þ a b c

1 1 11 a 1 b 1 c

+ + + + +- - -

= 4

Þ a 1 a b 1 b c 1 c

1 a 1 b 1 c+ - + - + -

+ +- - -

= 4

Þ 1 1 1

1 a 1 b 1 c+ +

- - - = 4

123. (c) (x – 3)2 + (y – 5)2 + (z – 42)= 0Þ x – 3 = 0 Þ x = 3y – 5 = 0 Þ y = 5z – 4 = 0 Þ z = 4

\ 2 2 2x y z

9 25 16+ +

= 9 25 169 25 16

+ + = 1 + 1 + 1 = 3

124. (c) When x = 6.4 6

+ 2P = 12

Þ 8 + 2P = 12Þ 2P = 12 – 8 = 4 Þ P = 2

125. (a) Expression = 4 3 37 4 3

++

Rationalising the denominator.

= (4 3 3)(7 4 3)(7 4 3)(7 4 3)+ -+ -

= 28 16 3 21 3 12 3

49 48- + - ´

-= 28 5 3 36 5 3 8+ - = -

126. (b) 3x – 2 = 3x

Þ 3x – 3x

= 2

Þ x – 1 2x 3

=

On squaring both sides

21 4xx 9

æ ö- =ç ÷è ø

Þ 2

21 4

x 29x

+ - =

Þ 2

21

xx

+

= 4 22 4

2 29 9 9

+ = =

127. (a) a2 – 2ab + b2 = (a – b)2

\ 16a2 – 12a

= (4a)2 – 2 × 4a × 32

Hence, on adding 23 9

2 4æ ö =ç ÷è ø ,

Expression will be a perfect square.128. (b) Putting y = 0 in 4x + 3y = 12, we get x = 3

Putting x = 0 in 4x + 3y = 12, we get, y = 4

Y1

X1

OX

Y

II quadrant

III quadrant IV quadrant

I quadrant

129 (b) A

R Q

BP

C

AP ÐABBQ Ð BCCR ÐAC\ AP + BQ + CR < AB + BC + AC

130. (b) ÐA + ÐB = 65°\ ÐC = 180° – 65° = 115°ÐB + ÐC = 140°\ ÐB = 140° – 115° = 25°

yoursmahboob.w

ordpress.com

297SSC CGL SolvedPaper

131. (a) B

O A

ÐOBA = 90°OA = 5, OB = 4

\ AB = 2 2OA OB-

= 25 16 9- = = 3 cm

132. (c) A

B C

D

90°

ÐB + ÐD = 180°ÐA + ÐC = 180°ÐBAC + ÐBCAÐDAC = ÐDCA\ ÐDAB = ÐDCB = 90°ÐDAC = q\ ÐADE = 90° – q = ÐCDE\ ÐABC = 180° – 2 (90° – q) = 2q

133. (c) A

B D CAB = BC = CA = 2a cm,AD ̂ BC

AD = 2 2AB BD-

= 2 24a a 3 a- = \ 3 a 15=

Þ a = 5 3\ 2a = side = 10 3 cm\ Area of triangle

= 23(10 3)

4´ = 75 3 sq. cm.

134. (c) C A

Q

D B

PO

OA = OC = 10 cmAB = 12 cmAP = PB = 6 cmCD = 16 cmCQ = QD = 8 cmFrom DOCQ,

OQ = 2 210 8-= 18 2´ = 6 cmFrom DOAP

OP = 2 210 6- = 16 4´ = 8 cm\ PQ = 6 + 8 = 14 cm

135. (d) A

D

B E CDE | | AC

DABC – DBDE \ AB ACBD BE

=

Þ AB AC

1 1BD BE

- = -

Þ AD CEBD BE

= Þ BD BEAD CE

=

Þ 10 4 BE

4 CE- = Þ

BE 3CE 2

=

136. (b) B

OA

C

yoursmahboob.w

ordpress.com

298 SSC CGL SolvedPaper

ÐAOB = 90° ; OA = OB = r\ ÐBAO = ÐABO = 45°\ ÐAOC = 110°; OA = OC = r

\ ÐOAC = ÐOCA = 702

= 35°

\ ÐBAC = 45° + 35° = 80°137. (a)138. (d) 3 cos 80°. cosec 10° + 2 cos 59°. cosec 31°

= 3 cos (90° – 10°). cosec 10° + 2 cos(90° – 31°). cosec 31°= 3 sin 10°. cosec 10° + 2 sin 31°. cosec 31°= 3 + 2 = 5

[Qcos (90° – q) = sinq; sinq.cosecq = 1]139. (c) pQ radian = 180°

\ 35p

radian = 180 3

5p´

p= 108°

140. (a) tanq + cot q = 2On squaring both sides,(tan q + cot q)2 = 4Þ tan2q + cot2 q + 2tan q cot q = 4Þ tan2q + cot2q = 4 – 2 = 2

[tan q. cotq = 1]141. (a) x cos q – y sin q = 2

x sin q + y cos q = 4On squaring both the equations and addingx2cos2 q + y2 sin2 q – 2xysin q. cos q + x2sin2 q + y2 cos2q + 2xy sinq. cosq= 4 + 16Þ x2 (cos2 q + sin2 q) + y2 (sin2 q + cos2 q)= 20Þ x2 + y2 = 20

142. (d) sin2 q – 3 sin q + 2 = 0Þ sin2 q – 2 sin q – sin q + 2 = 0Þ sin q (sinq – 2) – 1 (sin q – 2) = 0Þ (sin q – 1) (sin q – 2) = 0Þ sin q = 1 = sin 90°Þ q = 90° and sin q ¹ 2

143. (c) Expression

= 2

2cos A(sin A cos A)

cosec A(sin A cos A)

é +ê

-êë

2

2sin A(sin A cos A)sec A(sin A cosA)

ù-+ ú+ úû

2 21 1

cos A sin Aæ ö-ç ÷è ø

2 2cos A.sin A(sin A cos A)sin A cos A

é +ê

-êë

2 2sin A.cos A(sin A cos A)(sin A cos A)

ù-+ ú+ úû

2 2

2 2sin A cos Asin A.cos A

æ ö-ç ÷è ø

sin A cos A sin A cosAsin A cos A sin A cosA

+ -æ ö+ç ÷è ø- +

2 2(sin A cos A)-

= 2 2(sin A cos A) (sin A cos A)

(sin A cos A)(sin A cos A)

é ù+ + -ê ú

- +ê úë û

(sin2A – cos2A)= 2 (sin2A + cos2 A) = 2

144. (d) Total students in 2008 = 170Students passed in 1st division = 20\ Required percentage

= 22 100

170´ =

200 1311 %17 17

=

145. (d) 146. (a) 147. (b) 148. (a)149. (a) 150. (d) 151. (b) 152. (b)153. (a) 154. (b) 155. (b) 156. (b)157. (d) 158. (c) 159. (b) 160. (c)161. (b) 162. (b) 163. (c) 164. (a)165. (a) 166. (d) 167. (a) 168. (c)169. (b) 170. (a) 171. (b) 172. (b)173. (a) 174. (b) 175. (a) 176. (b)177. (b) 178. (b) 179. (b) 180. (c)181. (b) 182. (a) 183. (a) 184. (a)185. (b) 186. (b) 187. (d) 188. (c)189. (c) 190. (c) 191. (d) 192. (d)193. (b) 194. (c) 195. (d) 196. (b)197. (c) 198. (b) 199. (b) 200. (d)

yoursmahboob.w

ordpress.comPART-A : GENERAL INTELLIGENCE & REASONING

Directions (Qs. 1-6 ): Select the related word/letter/number from the given alternatives.1. Voyage : Sea sickness : : Heights : ?

(a) Ship (b) Travel(c) Giddiness (d) Motion

2. Waitress : Restaurant : : ?(a) Doctor : Nurse (b) Driver : Truck(c) Teacher : School (d) Actor : Role

3. AROUND : RAUODN : : GROUND : ?(a) RGUODN (b) NDOOGR(c) OUNDGR (d) DNUURG

4. APPROACHED : ROACHEDAPP : :BARGAINED : ?(a) AINEDBARG (b) GAINEDBAR(c) GAINEDRAB (d) RABGAINED

5. 8 : 256 : : ?(a) 7 : 343 (b) 9 : 243(c) 10 : 500 (d) 5 : 75

6. 21 : 3 : : 574 : ?(a) 23 (b) 82(c) 97 (d) 113

Directions (Qs. 7-12) : Find the odd word/number/letters/number pair from the given alternatives.7. (a) Obstetrician (b) Podiatrist

(c) Pulmonologist (d) Prosthetist8. (a) Century (b) Decadent

(c) Year (d) Month9. (a) FEDC (b) STUV

(c) LKJI (d) RQPO10. (a) LPXOY (b) RQST

(c) FBDLX (d) MPONL11. (a) 14 - 16 (b) 56 - 64

(c) 77 - 88 (d) 80 - 9312. (a) 13 - 21 (b) 19 - 27

(c) 15 - 23 (d) 16 - 2413. Which one of the given responses would be a

meaningful order of the following ?1. apartment 2. town3. street 4. building5. complex(a) 1, 5, 4, 3, 2 (b) 4, 5, 3, 2, 1(c) 2, 1, 3, 4, 5 (d) 1, 4, 5, 3, 2

14. If the following words are arranged in reversedictionaryorder, which word comes second ?(a) Explosion (b) Express(c) Exploit (d) Expulse

15. Which one set of letters when sequentiallyplaced at the gaps in the given letter series shallcomplete it ?ac__c_cb_acbcacbca_bc(a) abbb (b) bacc (c) babc (d) bbcc

Directions (Qs. 16-18) : A series is given, with oneterm missing. Choose the correct alternative fromthe given ones that will complete the series.16. AZ, CX, FU, ?

(a) IR (b) IV (c) JQ (d) KP17. 1, 2, 6, 24, ? ,720

(a) 3 (b) 5 (c) 120 (d) 818. 156, 506, ?, 1806

(a) 1056 (b) 856 (c) 1456 (d) 150619. Suket has three daughters and each daughter

has a brother. How many male members are therein the family ?(a) 4 (b) 2 (c) 3 (d) 1

20. Dinesh and Ramesh start together from a certainpoint in the opposite direction on motorcycles.The speed of Dinesh is 60 km per hour andRamesh 44 km per hour. What will be the distancebetween them after 15 minutes ?(a) 20 km. (b) 24 km. (c) 26 km. (d) 30 km.

21. An insect is walking in a straight line. It covers adistance of 15 cm per minute. It comes back 2.5cm after every 15 cm. How long will it take tocover a distance of 1 metre ?(a) 6.5 min (b) 8 min(c) 10 min (d) 12 min

22. A group of alphabets are given with each beingassigned a number. These have to beunscrambled into a meaningful word and correctorder of letters may be indicated from the givenresponses.T M H R E O5 4 3 2 1 0(a) 025314 (b) 315402(c) 405312 (d) 504231

SSC Combined Graduate Level (CGL) Solved Paper19 OCTOBER, 2014

yoursmahboob.w

ordpress.com

300 SSC CGL SolvedPaper

23. From the given alternative words, select the wordwhich cannot be formed using the letters of thegiven word :TRIVANDRUM(a) RAIN (b) DRUM(c) TRAIN (d) DRUK

24. How many meaningful English words can bemade with the letters ' OEHM ' using each letteronly once in each word ?(a) FOUR (b) THREE(c) TWO (d) ONE

25. A shopkeeper quotes the rate on the price tagby replacing numbers with letters as follows :0 1 2 3 4 5 6 7 8 9B R O W N S T I C KIf a customer purchases two items whose pricetags read ` IIT and ` NICK, what is the totalamount he has to Pay?(a) ` 4776 (b) ` 4765(c) ` 5565 (d) ` 5665

26. If CASUAL is coded as SACLAU, then whatwould be the code of MATRIC ?(a) CIRTAM (b) TMAICR(c) TAMCIR (d) ATMCIR

27. If ‘S’ is written as ‘H’ ‘R’ as ‘@’ ‘A’ as ‘Ñ’ ‘M’as ‘#’ ,‘T’ as ‘$’ and ‘E’ as ‘%’ then how is‘MASTER’ written in that code ?(a) #ÑH$%@ (b) #HÑ$%@(c) #Ñ$H%@ (d) #ÑH%@$

28. If 1 candle in box number 1 is placed in box number2, then box-2 has twice the number of candlesthat box 1 has.If 1 candle from box-2 is placed in box-1, the box-2 and box-1 have the same number of candles.How many candles were there in box-1 and box-2 ?

Box--1 Box-2 Box-1 Box-2

(a) 5 : 3 (b) 7 : 5

(c) 6 : 4 (d) 5 : 729. Which of the following interchange of signs

would make the equation correct ?6 × 4 + 2 = 16(a) + and ×, 2 & 4 (b) + and ×, 4 & 6(c) + and ×, 2 & 6 (d) + and ×, 3 & 4

30. Select the correct combination of mathematicalsings to replace the * signs and to balance thefollowing equation :45 * 3 * 6 * 2 * 16(a) + × ¸ = (b) + ¸ × =(c) + × – = (d) + + – =

31. Select the correct combination of mathematicalsigns to replace * signs and to balance thefollowing equation :8 * 5 * 10 * 2 * 25(a) + × ¸ = (b) + ¸ – =(c) × + = × (d) × – = ×

Directions (Qs. 32-34): Select the missing numberfrom the given responses.32. 7 6 6

8 6 ?3 4 5168 144 120(a) 8 (b) 10 (c) 5 (d) 4

33. 8 5 63 7 51 4 274 90 ?(a) 65 (b) 85 (c) 52 (d) 76

34. 22 46 2427 58 3132 68 ?(a) 46 (b) 36 (c) 32 (d) 38

35. On one side of a street are even numbers and onthe other side are odd numbers. No. 1 is exactlyin front of No. 2. My House is No. 9. From myhouse , a man comes up from No. 2 and knocksat the door, five doors beyond the house infrontof me. What is the No. of that house ?(a) 18 (b) 20 (c) 22 (d) 26

36. Four players P, Q, R and S are standing a playfield in such a way that Q is to East of P, R is tothe South of P and S is to the North of P. Inwhich direction of Q is S Standing ?(a) North (b) South(c) North-West (d) South-East

37. One statement is given followed by twoConclusions I and II. You have to consider thestatement to be true, even if it seems to be atvariance from commonly known facts. You areto decide which of the given conclusions candefinitely be drawn from the given statement.Indicate your answer.Statement : The rich must live more simply. Allpoor people are simple.Conclusions : I. Rich people waste money.II. Poor people save money.(a) Only I follows.(b) Only II follows.(c) Neither I nor II follow.(d) Both I and II follow.

yoursmahboob.w

ordpress.com

301SSC CGL SolvedPaper

38. (1) All roses in Sita's garden are red.(2) All marigold flowers in Sita's garden are

orange.(3) All flowers in Sita's garden are either red

or orange.If 1st two statements are true, the third is(a) True (b) False(c) Uncertain (d) Vague

39. Find out which answer figure will exactly makeup the question figure.Question figure :

Answer figures :

(a) (b) (c) (d)40. How many triangles are there in the give figure ?

(a) 48 (b) 60(c) 56 (d) 52

41. Choose the cube that will be formed by foldingthe sheet of paper shown in the problem figure.Question figure :

Answer figures :

(a) (b) (c) (d)42. In a group of persons, 11 persons speak

Kannada, 20 persons speak Tamil and 11 personsspeak Telugu. In that group, if two personsspeak two languages and one person speak allthe languages, then how many persons are therein the group ?

19 111

10

10

(a) 40 (b) 41(c) 42 (d) 43

43. Which one of the following diagrams bestdepicts the relationship among Human Society- Youth Club, Political Party and Youths ?

(a) (b)

(c) (d)

44. Which one of the following diagrams representcorrect relationship among pigeon, birds, dogs ?

(1) (2) (3) (4)(a) 1 (b) 2(c) 3 (d) 4

Directions (Qs. 45-46) : Which answer figure willcomplete the pattern in the question figure ?45. Question figure :

Answer figures :

(a) (b) (c) (d)46. Question figure :

Answer figures :

(a) (b) (c) (d)

yoursmahboob.w

ordpress.com

302 SSC CGL SolvedPaper

47. From the given answer figures, select the figurewhich is hidden/embedded in the questionfigure.Question figure :

Answer figures :

(a) (b) (c) (d)48. A piece of paper is folded and cut as shown

below in the question figures. From the givenanswer figures, indicate how it will appear whenopened.Question figures :

Answer figures :

(a) (b) (c) (d)49. If a mirror is place on the line MN, then which of

the answer figures is the right image of the givenfigure ?Question figure :M

N

JACK

Answer figures :

(a) (b) (c) (d)50. A word is represented by only one set of

numbers as given in any one of the alternatives.The sets of numbers given in the alternativesare represented by two classes of alphabets asin two matrices given below. The columns androws of Matrix I are numbered from 0 to 4 andthat of Matrix II are numbered from 5 to 9. A

letter from these matrices can be representedfirst by its row and next by its column, e.g., 'F'can be represented by 30, 22, etc. and 'N' can berepresented by 97, 89, etc. Similarly, you have toidentify the set for the given word."DAKU"

Matrix-I

0 1 2 3 44 A F K P U3 F K A U P2 P U F K A1 K P U A F0 U A P F K

Matrix-II

5 6 7 8 99 D I N S X8 X S I D N7 N X S I D6 S D X N I5 I N D X S

(a) 95, 40, 04, 42 (b) 24, 95, 20, 27(c) 88, 24, 10, 34 (d) 57, 13, 23, 21

PART-B : GENERAL AWARENESS

51. Prof. Milton Friedman was the leader of(a) Ohio school(b) Chicago school(c) Cambridge school(d) London school

52. Which one of the following is not a qualitativecontrol of credit by the Central Bank of a country?(a) Rationing of credit(b) Regulation of consumer credit(c) Variation of the reserve ratio(d) Regulation of margin requirements

53. The market in whcih loans of money can beobtained is called(a) Reserve market(b) Institutional market(c) Money market(d) Exchange market

54. If the marginal return increases at a diminishingrate, the total return(a) increases (b) decreases(c) remains constant (d) becomes zero

yoursmahboob.w

ordpress.com

303SSC CGL SolvedPaper

55. The law of Increasing Returns means(a) increasing cost(b) decreasing cost(c) increasing production(d) increasing income

56. The most important feature of Cabinet systemof Government is(a) Individual responsibility(b) Collective responsibility(c) Responsibility to none(d) Non-responsibility

57. Direct legislation in Switzerland has(a) a natural growth(b) a haphazard growth(c) an artificial growth(d) None of the above

58. Who gave the idea of "Cabinet Dietatorship"?(a) Muir (b) Lowell(c) Marriot (d) Laski

59. In which of the following countries are thejudges of the federal court elected by the twoHouses of the Federal Legislature?(a) Switzerland (b) Germany(c) Canada (d) Both (a) and (b)

60. The President of the USA appoints SupremeCourt Judges(a) with Senate's consent(b) at his discretion(c) with consent of the House of

Representatives(d) None of these

61. Multan was named by the Arabs as(a) City of beauty (b) City of wealth(c) City of gold (d) Pink city

62. Which one of the following was the book writtenby Amoghvarsha, the Rashtrakuta King?(a) Adipurana(b) Ganitasara Samgraha(c) Saktayana(d) Kavirajamarga

63. Who built the Kailasanatha Temple at Ellora?(a) Rajendra I(b) Mahendra Varman I(c) Krishna I(d) Govinda I

64. The land measures of the Second PandyanEmpire was mentioned in(a) Thalavaipuram Copper Plates(b) Uttirameru Inscription(c) Kudumiyammalai Inscription(d) Kasakudi Copper Plates

65. Who was the greatest ruler of the Satavahanas?(a) Satkarni I(b) Gautamiputra Satkarni(c) Simuka(d) Hala

66. Cactus is referred to as(a) Hydrophyte (b) Mesophyte(c) Xerophyte (d) Epiphyte

67. Which of the following is not a renewableresource?(a) Thorium (b) Geothermal heat(c) Tidal power (d) Radiant energy

68. Which of the following statements is correct?(a) Mahadeo hills are in the west of Maikala

hills.(b) Mahadeo hills are the part of Karnataka

Plateau.(c) Mahadeo hills are in the east of

Chhotanagpur Plateau.(d) Mahadeo hills are the part of Aravalli ranges.

69. Which one of the following pairs is not correctlymatched?(a) Hevea Tree—Brazil(b) Sumatra Storm—Malaysia(c) Kajan River—Borneo(d) Dekke Toba fish—Brazil

70. Which of the following resources is renewableone?(a) Uranium (b) Coal(c) Timber (d) Natural Gas

71. How many neck canal cells are found in thearchegonium of a fern?(a) One (b) Two(c) Three (d) Four

72. Which angiosperm is vesselless?(a) Hydrilla (b) Trochodendron(c) Maize (d) Wheat

73. Who was the first child born after operativeprocedure?(a) Caesar (b) Huxley(c) William (d) Pasteur

74. Myrmecology is study of(a) Insects (b) Ants(c) Crustaceans (d) Arthropods

75. NIN (National Institute of Nutrition) CentralOffice is located at(a) Hyderabad (b) Mumbai(c) Bengaluru (d) Kolkata

76. HIV often changes is shapes due to the presenceof an enzyme called(a) Reverse Transcriptase(b) Enterokinase(c) Nucleotidase(d) Nucleoditase

yoursmahboob.w

ordpress.com

304 SSC CGL SolvedPaper

77. Fleming's right hand rule is used to find thedirection of the(a) Alternate current (b) Direct current(c) Induced current (d) Actual current

78. The unit of electrical power is(a) Volt (b) Watt(c) Kilowatt hour (d) Ampere

79. The resistance of the human body (drycondition) is of the order of(a) 101 Ohm (b) 102 Ohm(c) 103 Ohm (d) 104 Ohm

80. Certain substances loose their electricalresistance completely at super low temperature.Such substances are called(a) super conductors(b) semi conductors(c) dielectrics (d) perfect conductors

81. The section of the CPU that selects, interpretsand monitors the execution of programinstructions is(a) Memory (b) Register unit(c) Control unit (d) ALU

82. Who among the following introduced theworld's first laptop computer in the market?(a) Hewlett-Packard(b) Epson(c) Laplink travelling software Inc(d) Microsoft

83. Brass contains(a) Copper and Zinc(b) Copper and Tin(c) Copper and Silver(d) Copper and Nickel

84. Which is the purest commercial form of iron?(a) Pig iron (b) Steel(c) Stainless steel (d) Wrought iron

85. In galvanization, iron is coated with(a) Copper (b) Zinc(c) Tin (d) Nicked

86. Which one of the following is also known assolution?(a) A compound(b) A homogeneous mixture(c) A heterogeneous mixture(d) A suspension

87. The cells which are closely associated andinteracting with guard cells are(a) Transfusion tissue(b) Complementary cells(c) Subsidiary cells(d) Hypodermal cells

88. Conversion of starch to sugar is essential for(a) Stomatal opening(b) Stomatal closing(c) Stomatal formation(d) Stomatal growth

89. Soil erosion can be prevented by(a) Increasing bird population(b) Afforestation(c) Removal of vegetation(d) Overgrazing

90. Natural sources of air pollution are(a) Forest fires(b) Volcanic eruptions(c) Dust storm(d) Smoke from burning dry leaves

91. Which of the following Genetically Modifiedvegetable is recently being made available inIndian market?(a) Carrot (b) Radish(c) Brinjal (d) Potato

92. "Bull's eye" is used in the game of(a) Boxing (b) Basketball(c) Polo (d) Shooting

93. As per newspapers report what percent ofGovernment stake will be disinvested inRashtriya Ispat Nigam Ltd. (RINL)?(a) 5% (b) 50%(c) 10% (d) 12%

94. Pablo Picasso, the famous painter was(a) French (b) Italian(c) Flemish (d) Spanish

95. Which of the following is the Regulator of thecredit rating agencies in India?(a) RBI (b) SBI(c) SIDBI (d) SEBI

96. Which is the first Indian Company to be listedin NASDAQ?(a) Reliance (b) TCS(c) HCL (d) Infosys

97. RRBs are owned by(a) Central Government(b) State Government(c) Sponsor Bank(d) Jointly by all of the above

98. The Monetary and Credit Policy is announcedby which of the following?(a) Ministry of Finance of Centre(b) Reserve Bank of India(c) State Bank of India(d) Planning Commission of India

yoursmahboob.w

ordpress.com

305SSC CGL SolvedPaper

99. Which of the following method is not used indetermining National Income of a country?(a) Income Method(b) Output Method(c) Input Method(d) Investment Method

100. What does the letter 'e' denotes in the term 'e-banking'?(a) Essential Banking(b) Economic Banking(c) Electronic Banking(d) Expansion Banking

PART-C : QUANTITATIVE APTITUDE101. Arrange the following in ascending order

334, 251, 717, we get(a) 334 > 251 > 717 (b) 717 > 251 > 334

(c) 334 > 717> 251 (d) 251 > 334> 717

102. If the product of first fifty positive consecutiveintegers be divisible by 7n, where n is an integer,then the largest possible value of n is(a) 7 (b) 8(c) 10 (d) 5

103. A, B and C together can do a piece of work in 40days. After working with B and C for 16 days, Aleaves and then B and C complete the remainingwork in 40 days more. A alone could do the work in(a) 80 days (b) 90 days(c) 100 days (d) 120 days

104. Three pipes A, B and C can fill a tank in 6 hours.After working it together for 2 hours, C is closedand A and B can fill the remaining part in 7 hours.The number of hours taken by C alone to fill thetank is(a) 10 (b) 12(c) 14 (d) 16

105. Pratibha is thrice as efficient as Sonia and istherefore able to finish a piece of work in 60days less than Sonia. Pratibha and Sonia canindividually complete the work respectively in(a) 30, 60 days (b) 60, 90 days(c) 30, 90 days (d) 40, 120 days

106. The base of a right pyramid is an equilateraltriangle of side 4 cm each. Each slant edge is 5cm long. The volume of the pyramid is

(a)4 8

3cm3 (b)

4 603

cm3

(c)4 59

3cm3 (d)

4 613

cm3

107. There are two cones. The curved surface area ofone is twice that of the other. The slant height ofthe latter is twice that of the former. The ratio oftheir radii is(a) 4 : 1 (b) 4 : 3(c) 3 : 4 (d) 1 : 4

108. A wire is bent into the form of a circle, whosearea is 154 cm2. If the same wire is bent into theform of an equilateral triangle, the approximatearea of the equilateral triangle is(a) 93.14 cm2 (b) 90.14 cm2

(c) 83.14 cm2 (d) 39.14 cm2

109. A shopkeeper allows 10% discount on goodswhen he sells without credit. Cost price of hisgoods is 80% of his selling price. If he sells hisgoods by cash, then his profit is(a) 50% (b) 70%(c) 25% (d) 40%

110. A dealer of scientific instruments allows 20%discount on the marked price of the instrumentsand still makes a profit of 25%. If his gain overthe sale of an instrument is ̀ 150, find the markedprice of the instrument.(a) ` 938.50 (b) ` 940(c) ` 938 (d) ` 937.50

111. Ram bought a T.V. with 20% discount on thelabelled price. Had he bought it with 30%discount he would have saved ̀ 800. The valueof the T.V. set that he bought is(a) ` 5,000 (b) ` 8,000(c) ` 9,000 (d) ` 10,000

112. A vessel full of pure acid contains 10 litres of it,of which 2 litres are withdrawn. The vessel isthen filled with water. Next 2 litres of the mixtureare withdrawn, and again the vessel is filled upwith water. The ratio of the acid left in the vesselwith that of the original quantity is(a) 1 : 5 (b) 4 : 5(c) 4 : 25 (d) 16 : 25

113. Gold is 19 times as heavy as water and copper is9 times as heavy as water. In what ratio shouldthese be mixed to get an alloy 15 times as heavyas water?(a) 1 : 1 (b) 1 : 2(c) 2 : 3 (d) 3 : 2

114. The average age of boys in the class is twice thenumber of girls in the class. The ratio of boysand girls in the class of 50 is 4 : 1. The total of theages (in years) of the boys in the class is(a) 2000 (b) 2500(c) 800 (d) 400

yoursmahboob.w

ordpress.com

306 SSC CGL SolvedPaper

115. There are 100 students in 3 sections A, B and Cof a class. The average marks of all the 3 sectionswas 84. The average of B and C was 87.5 and theaverage marks of A is 70. The number ofstudents in A was(a) 30 (b) 35(c) 20 (d) 25

116. A sold an article to B at 20% profit and B sold itto C at 15% loss. If A sold it to C at the sellingprice of B, then A would make(a) 5% profit (b) 2% profit(c) 2% profit (d) 5% loss

117. The monthly salaries of A and B togetheramount to ̀ 40,000. A spends 85% of his salaryand B, 95% of his salary. If now their savings arethe same, then the salary (in `) of A is(a) 10,000 (b) 12,000(c) 16,000 (d) 18,000

118. It takes 8 hours for a 600 km journey, if 120 km isdone by train and the rest by car. It takes 20minutes more if 200 km is down by train and therest by car. The ratio of the speed of the train tothat of the car is(a) 2 : 3 (b) 3 : 2(c) 3 : 4 (d) 4 : 3

119. If a train runs at 70 km/hour, it reaches itsdestination late by 12 minutes. But if it runs at80 km/hour, it is late by 3 minutes. The correcttime to cover the journey is(a) 58 minutes (b) 2 hours(c) 1 hour (d) 59 minutes

120. A man borrowed some money from a privateorganisation at 5% simple interest per annum.He lended 50% of this money to another personat 10% compound interest per annum andthereby the man made a profit of ` 3205 in 4years. The man borrowed(a) ` 80,000 (b) ` 1,00,000(c) ` 1,20,000 (d) ` 1,50,000

121. If a2 + b2 + c2 = 2a – 2b – 2, then the value of 3a– 2b + c is(a) 0 (b) 3(c) 5 (d) 2

122. If a + b + c = 3, a2+ b2 + c2 = 6 and 1 1 1a b c

+ + = 1,

where a, b, c are all non-zero, then 'abc' is equal to

(a)23 (b)

32

(c)12

(d)13

123. If a2 – 4a – 1 = 0, a ¹ 0, then the value of a2 + 3a

+ 21

a– 3

ais

(a) 24 (b) 26 (c) 28 (d) 30124. The total area (in sq. unit) of the triangles formed

by the graph of 4x + 5y = 40, x-axis, y-axis and x= 5 and y = 4 is(a) 10 (b) 20 (c) 30 (d) 40

125. For what value of k, the system of equations kx+ 2y = 2 and 3x + y = 1 will be coincident?(a) 2 (b) 3 (c) 5 (d) 6

126. If x = 2 + 3 , then x2 + 21

x is equal to

(a) 10 (b) 12 (c) –12 (d) 14127. If a = 4.965, b = 2.343 and c = 2.622, then the

value of a3 – b3 – c3 – 3abc is(a) –2 (b) –1(c) 0 (d) 9.932

128. If x + y + z = 0, then the value of 2 2 2

2x y z

x yz+ +-

is

(a) –1 (b) 0(c) 1 (d) 2

129. The value of sin21° + sin22° + sin23° + .... +sin289° is(a) 22 (b) 44

(c)1222

(d) 4421

130. The value of 3 3cos sin

cos sinq+ qq+ q

+ 3 3cos sin

cos sinq- qq- q

is equal to(a) – 1 (b) 1(c) 2 (d) 0

131. The shadow of a tower standing on a level planeis found to be 30 m longer when the Sun's altitudechanges from 60° to 45°. The height of the tower is

(a) ( )15 3 3+ m (b) ( )15 3 1+ m

(c) ( )15 3 1- m (d) ( )15 3 3- m

132. If sin 17° = xy then sec 17° – sin 73° is equal to

(a)2 2

y

y x-(b)

2

2 2

y

x y xæ ö-ç ÷è ø

(c) 2 2

x

y y xæ ö-ç ÷è ø(d)

2

2 2

x

y y xæ ö-ç ÷è ø

yoursmahboob.w

ordpress.com

307SSC CGL SolvedPaper

133. If q is a positive acute angle and cosec q + cot q= 3 , then the value of cosec q is

(a)13

(b) 3

(c)23

(d) 1

134. If cos a + sec a = 3 , then the value of cos3 a+ sec3 a is(a) 2 (b) 1(c) 0 (d) 4

135. If sin q + cos q = 2 cos q, then the value of cotq is(a) 2 1+ (b) 2 1-(c) 3 1- (d) 3 1+

136. In a quadrilateral ABCD, the bisectors of ÐAand ÐB meet at O. If ÐC = 70° and ÐD = 130°,then measure of ÐAOB is(a) 40° (b) 60°(c) 80° (d) 100°

137. In DABC, E and D are points on sides AB andAC respectively such that ÐABC = ÐADE. IfAE = 3 cm, AD = 2 cm and EB = 2 cm, then lengthof DC is(a) 4 cm (b) 4.5 cm(c) 5.0 cm (d) 5.5 cm

138. In a circle with centre O, AB is a chord, and AP isa tangent to the circle. If ÐAOB = 140°, then themeasure of ÐPAB is(a) 35° (b) 55°(c) 70° (d) 75°

139. In DABC, ÐA < ÐB. The altitude to the basedivides vertex angle C into two parts C1 and C2,with C2 adjacent to BC. Then(a) C1 + C2 = A + B (b) C1 – C2 = A – B(c) C1 – C2 = B – A (d) C1 + C2 = B – A

140. If O is the in-centre of DABC; if ÐBOC = 120°,then the measure of ÐBAC is(a) 30° (b) 60°(c) 150° (d) 75°

141. Two parallel chords of a circle of diameter 20 cmare 12 cm and 16 cm long. If the chords are in thesame side of the centre, then the distancebetween them is(a) 28 cm (b) 2 cm(c) 4 cm (d) 8 cm

142. The interior angle of a regular polygon is 140°.The number of sides of that polygon is(a) 9 (b) 8(c) 7 (d) 6

143. If two circles of radii 9 cm and 4 cm touchexternally, then the length of a common tangentis(a) 5 cm (b) 7 cm(c) 8 cm (d) 12 cm

Study the following table and answer Question Nos.144 to 146:

School No. of Percentage No. ofstudents of students studentsscoring scoring appearedmarks marksless than more than50% 50%

A 240 55 600B 220 40 400C 300 20 375D 280 10 350E 210 25 300

144. The ratio of the total number of students scoringmarks less than 50% to that of scoring marksexactly 50% is(a) 50 : 3 (b) 25 : 2(c) 25 : 4 (d) 35 : 2

145. Which school has the highest number ofstudents scoring exactly 50% marks?(a) D (b) E(c) B (d) A

146. The total number of students scoring 50% ormore marks is(a) 1250 (b) 875(c) 775 (d) 675

Study the following graph which shows income andexpenditure of a company over the years 2005-2009and answer questions 147 to 150.

010

20

30

40

50

60

70

2005 2006 2007 2008 2009

Income Expenditure

( in

Cro

res)

`

Years

yoursmahboob.w

ordpress.com

308 SSC CGL SolvedPaper

147. The difference in profit (` in crores) of thecompany during 2006 and 2007 is(a) 10 (b) 15(c) 20 (d) 25

148. In how many years was the income of thecompany less than the average income of thegiven years?(a) 4 (b) 3(c) 2 (d) 1

149. The percentage increase in expenditure fo thecompany from 2007 to 2008 is(a) 20 (b) 25(c) 30 (d) 35

150. Profit of the company was maximum in the year(a) 2009 (b) 2008(c) 2006 (d) 2005

PART-D : ENGLISH COMPREHENSION

DIRECTIONS (Qs. 151-155): Some parts of thesentences have errors and some are correct. Findout which part of a sentence has an error and blackenthe oval [•] corresponding to the appropriate letter(A, B, C). If a sentence is free from error, blacken theoval corresponding to (D) in the Answer Sheet.

151.If I would have realised

(a) /

what a bad shape our library is in(b) /

I would have done something,(c)

to arrest the deterioration./

No error(d)

152.He has been

(a) / enhanced in position

(b) /

as a result of his diligence(c)

and integrity. /

No error(d)

153.It is I(a) /

who is responsible(b) /

for the delay.(c) /

No error(d)

154.There is only one cure

(a) /

to the evils which newly(b) /

acquired freedom produces and that cure is freedom.

(c)/

No error(d)

155.He flew

(a) / over extensively

(b) /

the Pacific last winter.(c) /

No error(d)

DIRECTIONS (Qs. 156-160):Sentences are givenwith blanks to be field in with an appropriateword(s). Four alternatives are suggested for eachquestion. Choose the correct alternative out of thefour and indicate it by blackening the appropriateoval [•] in the Answer Sheet.156. The student was punished for his _____.

(a) impudence (b) prudence(c) modesty (d) elemency

157. My father was too _____ to push the heavy door.(a) faint (b) feeble(c) fragile (d) faltering

158. The flood damaged the books so much that itwas impossible to _____ them.(a) retrieve (b) retrace(c) retract (d) retreat

159. His bungalow went through a make _____.(a) up (b) out(c) over (d) for

160. This auspicious beginning _____ well for asuccessful completion of our project.(a) attunes (b) argues(c) augurs (d) answers

DIRECTIONS (Qs. 161-163): Out of the fouralternatives, choose the one which best expresses themeaning of the given word and mark it in the AnswerSheet.161. Persist

(a) Resist (b) Leave(c) Quit (d) Insist

162. Eventually(a) previously (b) briefly(c) finally (d) successfully

163. Impeccable(a) remarkable (b) unbelievable(c) flawless (d) displeasing

yoursmahboob.w

ordpress.com

309SSC CGL SolvedPaper

DIRECTIONS (Qs. 164-166): Choose the wordopposite in meaning to the given word and mark itin the Answer Sheet.164. Predilection

(a) Predicament (b) Afterthought(c) Aversion (d) Postponement

165. Pompous(a) Uppish (b) Humble(c) Meek (d) Grandiose

166. Serene(a) Calm (b) Angry(c) Ruffled (d) Bitter

DIRECTIONS (Qs. 167-171): Four alternatives aregiven for the Idiom/Phrase underlined in the sentense.Choose the alternative which best expresses themeaning of the Idiom\Phrase and mark it in theAnswer Sheet.167. With great difficulty, he was able to carve out a

niche for himself.(a) became a sculptor(b) did the best he could do(c) destroyed his career(d) developed a specific position for himself

168. You will succeed if you follow my advice to theletter.(a) about writing letters(b) written in the letter(c) in every detail(d) very thoughtfully

169. A critic's work is to read between the lines.(a) to comprehend the meaning(b) to appreciate the inner beauty(c) to understand the inner meaning(d) to read carefully

170. Where discipline is concerned I put my foot down.(a) take a firm stand(b) take a light stand(c) take a heavy stand(d) take a shaky stand

171. The convict claimed innocence and stood hisground in spite of the repeated accusations.(a) knelt (b) surrendered(c) kept standing (d) refused to yield

DIRECTIONS(Qs. 172-181): A sentence/a part of thesentence is underlined. Below are given alternativesto the underlined part at (a), (b), (c) which mayimprove the sentence. Choose the correct alternative.In case no improvement is needed your answer is (d).Mark your answer in the Answer Sheet.

172. Why should you be despaired of your successof your undertaking?(a) you despair of the success of your

undertaking(b) you despair of success of undertaking(c) you be despaired of the success of your

undertaking(d) No improvement.

173. As Rees was going to town in the High Street asavage dog attacked him and bit him.(a) going to town a savage dog attacked him

and bit him in the High Street(b) in the High Street a savage dog attacked

him and bit him in the town(c) going to town in the High Street a savage

dog bit him and attacked him(d) No improvement

174. Something is pretty here that Vineeta can wearto the party.(a) Something here is pretty(b) Something is here pretty(c) Here is something pretty(d) No improvement

175. I have dreamt all my life to own a beautiful marooncoloured car.(a) of owning (b) to owning(c) at owning (d) No improvement

176. Sitting on the hill top, the sun went downwatching before him.(a) he watched the sun go down.(b) the sun went down with him watching(c) the sun went down when he watched(d) No improvement

177. The office is soon to be closed.(a) just to (b) about to(c) immediately to (d) No improvement

178. He has achieved nothing out of his way worthmentioning.(a) out of the way (b) by the way(c) in a big way (d) No improvement

179. I prevailed on him to vote for you.(a) to (b) at(c) upon (d) No improvement

180. Eager to pass his final exams, studying was thestudents top priority.(a) the student's top priority was studying.(b) the student made studying his top priority.(c) the top priority of studying was made by

the student.(d) No improvement.

yoursmahboob.w

ordpress.com

310 SSC CGL SolvedPaper

181. Mr. Dev will not go to the wedding receptionwithout being called.(a) if he is not invited(b) till he is invited(c) unless he is invited(d) No improvement

DIRECTIONS (Qs. 182-188): Out of the fouralternatives, choose the one which can be substitutedfor the given words/sentences and indicate it byblackening the appropriate oval [•] in the AnswerSheet.182. Belief in many gods

(a) pantheism (b) monotheism(c) polytheism (d) atheism

183. A cluster of flowers on a branch(a) bouquet (b) inflorescence(c) wreath (d) incandescence

184. A person who believes that only selfishnessmotivates human actions(a) agnostic (b) cynic(c) sceptic (d) misogynist

185. A highly skilled musician(a) artiste (b) virtuoso(c) performer (d) diva

186. A method of boiling briefly to cook food slightly(a) steam (b) bake(c) saute (d) parboil

187. The group, especially in the arts, regarded asbeing the most experimental(a) avant-garde (b) iconoclast(c) revolutionary (d) nerd

188. One who helps people by giving them money orother aid(a) benefactor (b) beneficiary(c) tycoon (d) patriot

DIRECTIONS (Qs. 189-190): Four words are givenin each question, out of which only one word iscorrectly spelt. Find the correctly spelt word andmark your answer in the Answer Sheet.189. (a) Plebeian (b) Plibeian

(c) Plebian (d) Plebiean190. (a) Suroundings (b) Surroundings

(c) Sarroundings (d) Surondings

DIRECTIONS (Qs. 191-195): You have two briefpassages with 5 questions following each passage.Read the passages carefully and choose the bestanswer to each question out of the four alternativesand mark it by blackening the appropriate oval [•]in the Answer Sheet.

PASSAGE - I (Q. NOS. 191-195)As I stepped out of the train I felt unusually solitarysince I was the only passenger to alight. I wasaccustomed to arriving in the summer, when holiday-makers throng coastal resorts and this was my firstvisit when the season was over. My destination was alittle village which was eight miles by road. It tookonly a few minutes for me to come to the foot of thecliff path. When I reached the top I had left all signs ofhabitation behind me. I was surprised to notice thatthe sky was already a flame with the sunset. It seemedto be getting dark amazingly quickly. I was at a loss toaccount for the exceptionally early end of daylightsince I did not think I had walked unduly slowly. ThenI recollected that on previous visits I had walked inhigh summer and how it was October.All at once it was night. The track was grassy andeven in daylight sghowed up hardly at all. I was terrifiedof hurtling over the edge of the cliff to the rocks below.I felt my feet squelching and sticking in somethingsoggy. Then I bumped into a little clump of trees thatloomed up in front of me. I climbed up the nearesttrunk and managed to find a tolerabley comfortablefork to sit on. The waiting was spent by my attemptsto identify the little stirrings and noises of animal lifethat I could hear. I grew colder and colder and managedto sleep only in uneasy fitful starts. At last when themoon came up I was on my way again.191. The writer felt unusually solitary because

(a) he was feeling very lonely without his family.(b) he was missing the company of other

holiday-makers.(c) his destination was a little village eight

miles away.(d) there was no one to meet him.

192. "I left all signs of habitation behind me." Thismeans that he(a) came to a place where there were very few

houses.(b) was in front of a large collection of

cottages.(c) had come very far from places where

people lived.(d) had just passed a remote village.

193. I became darker than the writer expected because(a) the nights are shorter in autumn than in

summer.(b) the nights are longer in October than mid

summer.(c) the train arrived later than usual.(d) he had walked unduly slowly.

yoursmahboob.w

ordpress.com

311SSC CGL SolvedPaper

194. The writer found it difficult to keep to the pathbecause of(a) the darkness and narrowness of the path.(b) poor visibility and grassy track.(c) the darkness and his slow pace.(d) poor visibility and dew on grass.

195. When he settled himself on the fork of the treethe writer ___________(a) had a sound sleep.(b) was disturbed by noises of animals.(c) was too afraid to sleep.(d) tried to sleep but without much success.

PASSAGE - II (Q. NOS. 196-200)It is sad that in country after country, progress shouldbecome synonymous with an assault on nature. Wewho are a part of nature and dependent on her forevery need, speak constantly about 'exploiting' nature.When the highest mountain in the world was climbedin 1953, Jawaharlal Nehru objected to the phraseconquest of Everest' which he thought was arrogant.Is it surprising that this lack of consideration and theconstant need to prove one's superiority should beprojected on to our treatment of our fellowmen? Iremember Edward Thompson, a British writer and agood friend of India, once telling Mr. Gandhi thatwildlife was fast disappearing. Remarked Mr. Gandhi:‘It is decreasing in the jungles but it is increasing inthe towns’

On the one hand the rich look askance at ourcontinuing poverty; on the other they warn us againsttheir own methods. We do not wish to impoverish theenvironment any further and yet we cannot forget thegrim poverty of large numbers of people. Are notpoverty and need the great polluters? For instance,unless we are in a position to provide employmentand purchasing power for the daily necessities of thetribal people and those who live in and around our

jungles, we cannot prevent them from combing theforest for food and livelihood, from poaching and fromdespoiling the vegetation.196. At the beginning of the passage, the writer

expresses her opinion that in many countriesprogress is synonymous with(a) development.(b) utmost care for nature.(c) a balanced treatment of nature.(d) utmost cruelty to nature.

197. In the passage the term 'exploiting' naturesuggests(a) regretfulness.(b) sarcasm.(c) destructive urge of man.(d) greed of man.

198. Nehru objected to the phrase 'conquest ofEverest' since(a) it carries a war-like connotation.(b) it sounds pompous and boastful.(c) it depicts Everest as a victim.(d) Everest is unconquerable.

199. Gandhi's statement 'It is decreasing in thejungles but it is increasing in the towns.!'(a) Refers to wild animals' decrease in the

jungle.(b) Refers to flora and fauna.(c) Refers to man's selfishness.(d) Is a satirical comparison of man's

callousness to the animals.200. The writer is of opinion that tribal people can be

prevented from combing forest for food(a) to provide employment(b) to increase purchasing power(c) by deterring them from poaching and

despoiling vegetation(d) to provide employment and purchasing

power for daily necessities.

yoursmahboob.w

ordpress.com

312 SSC CGL SolvedPaper

1. (c) Sea sickness is to feel ill when you aretravelling on a ship or boat.Similarly, Giddiness is to feel that you are goingto fall.

2. (c) Waitress is a person whose job is to servecustomers in a restaurant.Similarly, A teacher teaches students in a school.

3. (a) A O NR U D R A U O D N

Similarly,

G O NR U D R G U O D N

4. (b)A P P R O A C H E D R O A C H E D A PP

Similarly,B A R G A I N E D G A I N E D BAR

5. (c) 8 8 64 4 256´ = ´ =

10 10 100 5 500´ = ´ =

6. (b)21 73

=

Similarly, 574 7x

= , 574 82

7x = =

7. (d) An Obsterician is a medical doctor whospecializes in the management of pregnancy,labor and birth.A Podiatrist is a doctor who diagonose and treatconditions of the foot, ankle, and relatedstructures of the leg.A Pulmonologist is a physician who possessesspecilised knowledge and skill in the diagonosisand treatment of lung conditions and disease.A Prothesis is a device designed to replace amissing part of the body or to make a part ofbody work better. Hence (d) is odd one out.

8. (b) Except (b), all other are related to a period.While decadent is a person who has interest onlyin pleasure and enjoyment.

9. (b) Except (b) all others have consecutive lettersin reverse order.While (b) has consecutive letters in forward order.

10. (b) Except (b) all others have five letters.11. (d) Except (d) in both number in all others pairs

are divided by same number.12. (d) All others have odd numbers.

13. (d) 2. Town3. Street

5. Complex

4. Building

1. Apartment

14. (b) Arrangement in Reverse dictionary order—Expulse

1 2 3 4Express Explosion Exploit

15. (b) /acbc /acbc /acbc /acbc acbc

16. (c)

A Z C X

–2

+2 +3 +4

–3 – 4

F U J Q

17. (c) 1 2 22 3 66 4 2424 5 120120 6 720

´ =´ =´ =´ =´ =

18. (a) 156, 506, ?, 1806156 350 506506 550 10561056 750 1806

+ =+ =+ =

19. (b) Suket has three daughters and eachdaughter has a brother.

D1 = B = D2 = D3(F) (M) (F) (F)

Suket

Hence, there are 2 male members in a family.

HINTS & SOLUTIONS

yoursmahboob.w

ordpress.com

313SSC CGL SolvedPaper

20. (c) 60km/hr

44km/hrRamesh

Dinesh

Relative speed of Dinesh and Ramesh'smotorcycles = (60 + 44) = 10 4 km/hrDistance travelled by them = Relative speed ×Time covered,

15 104×15104km / hr hr = 26km

60 60æ ö´ =ç ÷è ø

21. (b)22. (c) 4

M O T H E R

0 5 3 1 2

23. (d) DRUK cannot be formed using TRIVANDROM as it does not contain letter 'K'.

24. (d) Home, only one meaningful word is formed.25. (c) Price tag ` IIT = ̀ 776

Price tag ̀ NICK = ̀ 4789Total amount = ̀ 4789 + ̀ 776 = ̀ 5565

26. (c) CS L

UAA A

ASC U

L

Similarly, M AT A M C I R

RA IT C

27. (a) SLetter

code H @ # $ %Ñ

R A M T E

# Ñ $ % @HMASTER = 28. (d) Going by options; Box 1 Box 2

5 7:If 1 cande in box number is placed in box number2 then

Box 1 Box 24 8:

Therefore, Box 2 has twice the number of candlesthan box 1.If 1 candle from box 2 is placed in box-1Then- Box 1 Box 2

6 6:: Hence, Both boxes have

the same numbers of candles.29. (b) 4 6 2 16+ ´ =30. (*) Going by options:-

(a) 45 3 6 2 16+ ´ ¸ = 54 16¹(b) 45 3 6 2 16+ ¸ ´ =

46 16¹(c) 45 3 6 2 16+ ´ - =

61 16¹

(d) 45 3 6 2 16+ + - = 52 16¹

None of option matching, Hence question is wrong.31. (c) 8 × 5 + 10 = 2 × 25

50 = 5032. (d) 7 8 3 168

6 4 4 1446 5 120x

´ ´ =´ ´ =´ ´ =\ 30 120x =

120 430

x = =

33. (a) 2 2 2

4 2 2

2 2 2

8 3 1 74

5 7 4 90

6 5 2 65

+ + =

+ + =

+ + =34. (b) 46 – 22 = 24

58 – 27 = 31

68 – 32 = 3635. (b)

1 3 5 7 9 11 13 15 17 19 21

2 4 6 8 10 12 14 16 18 20 22

My Home

Man from no.25 houses beyond my house.

Hence, 20 is the number of that house.

36. (c)North

South

R

P

S

QEast

North west N

W E

SHence, Q is in North west direction of S.

37. (c) Neither I nor II38. (c) Uncertain

Statement 1:

Red

Roses True ( )P

Statement 2:

Yellow

True ( )PMarigold

39. (c)

yoursmahboob.w

ordpress.com

314 SSC CGL SolvedPaper

40. (c) A E DI K

F

J HG

C

B

D ABC, DADC, DDBC, DAEG, DBFHD EIG, DEID, DIGJ, DIDJ,DDKJ, DDFK, DKJH, DKFHDEDG, DDJG, DEGJ, DDJGDDFJ, DFHJ, DDHJ, DDFHDADG, DDGH, DDBH, DJEFDGJC, DHJC, DGHC = 28 Triangles28 × 2 = 56 Triangles.

41. (d) 42. (c)

43. (b) Political Party Youth club

Youths

44. (a)

BirdsPigeon Dogs

45. (d) 46. (b) 47. (d)48. (c) 49. (c) 50. (d)51. (b) Milton Friedman (July 31, 1912 - November

16, 2006) was an American economist,statistician, and writer who taught at theUniversity of Chicago for more than threedecades. He was a recipient of the 1976 NobelPrize in Economic Sciences, and is known forhis research on consumption analysis, monetaryhistory and theory, and the complexity ofstabilization policy.

52. (c) The qualitative or selective methods ofcredit control are adopted by the Reserve Bankin its pursuit of economic stabilization and aspart of credit management. The four importantmethods are Margin Requirements, CreditRationing, Regulation of Consumer Credit, MoralSuasion.

53. (c) A segment of the financial market in whichfinancial instruments with high liquidity and veryshort maturities are traded. The money market isused by participants as a means for borrowingand lending in the short term, from several daysto just under a year.

54. (a) In economics, diminishing returns (alsocalled law of diminishing returns, law of variableproportions, principle of diminishing marginalproductivity, or diminishing marginal returns isthe decrease in the marginal (incremental) outputof a production process as the amount of a singlefactor of production is incrementally increased,while the amounts of all other factors ofproduction stay constant.

55. (b) The law of increasing returns is theopposite of the law of decreasing returns. Wherethe law of diminishing returns operates, everyadditional investment of capital and labour yieldsless than proportionate returns. But, in the caseof the law of increasing returns, the return ismore than proportionate.

56. (b) Cabinet collective responsibility isconstitutional convention in governments isthat members of the Cabinet must publiclysupport all governmental decisions made inCabinet, even if they do not privately agree withthem. This support includes voting for thegovernment in the legislature. Cabinet collectiveresponsibility is related to the fact that, if a voteof no confidence is passed in parliament, thegovernment is responsible collectively, and thusthe entire government resigns.

57. (a) Switzerland is a small country located inthe heart of western Europe, at the intersectionof German, French and Italian language andculture. Switzerland has been multicultural in itsown way for centuries. Direct Democracy inparticular, has a long, but not undisputedtradition in this country. Switzerland's uniquepolitical system is today world's most stabledemocratic system, offering a maximum ofparticipation to citizens.

58. (a) John Muir was a Scottish-Americannaturalist, author, and early advocate ofpreservation of wilderness in the United States.

59. (a) The Federal Assembly is bicameral, beingcomposed of the 200-seat National Council andthe 46-seat Council of States. The houses haveidentical powers. Members of both housesrepresent the cantons, but, whereas seats in theNational Council are distributed in proportionto population, each canton has two seats in theCouncil of States, except the six 'half-cantons'which have one seat each. Both are elected infull once every four years, with the last electionbeing held in 2011.

60. (a) The Court consists of the Chief Justice ofthe United States and eight associate justices

yoursmahboob.w

ordpress.com

315SSC CGL SolvedPaper

who are nominated by the President andconfirmed by the Senate. Once appointed,justices have life tenure unless they resign, retire,take senior status, or are removed afterimpeachment (though no justice has ever beenremoved).

61. (c) During the early period, Multan was knownas the city of gold for its large and wealthytemples. The Sun temple, Suraj Mandir, wasconsidered one of the largest and wealthiesttemples in the entire sub-continent.

62. (d) Amoghavarsha I was a follower of theDigambara branch of Jainism. His own writingKavirajamarga is a landmark literary work in theKannada language and became a guide bookfor future poets and scholars for centuries tocome.

63. (c) The Kailasa temple is a famous rock cutmonument, one of the 34 monasteries andtemples known collectively as the Ellora Caves,extending over more than 2 km.The temple wascommissioned and completed between dated757-783 CE, when Krishna I ruled theRashtrakuta dynasty. It is designed to recallMount Kailash, the home of Lord Shiva. It is amegalith carved out of one single rock. It wasbuilt in the 8th century by the Rashtrakuta kingKrishna I.

64. (a) the Thalavaipuram copper plate, belongingto the period between 1018 and 1054 broughtout by the Pandya kings, describes giant waves,most possibly a tsunami.

65. (b) Gautamiputra Satakarni was the greatestof the Satavahana rulers. His reign period isnoted by some scholars as 80 to 104 and byothers from 106 to 130; in any case he is creditedwith a rule of 24 years.

66. (c) Succulent plants store water in their stemsor leaves. They include the Cactaceae family,which has round stems and can store a lot ofwater. The leaves are often vestigial, as in thecase of cacti, wherein the leaves are reduced tospines, or they do not have leaves at all. Wateris stored in the bulbs of some plants, at or belowground level. They may be dormant duringdrought conditions and are, therefore, knownas drought evaders.

67. (a) Thorium is an element which are used inradioactive chemicals where all other threeoptions are power generating systems which areregenerated.

68. (a) The Mahadeo Hills are a range of hills inMadhya Pradesh state of central India. The hillsform the central part of the Satpura Range.

69. (d) Dekke Toba fish in found in Indonesia.Lake Toba (Indonesian: Danau Toba) is a lakeand supervolcano. The lake is 100 kilometreslong, 30 kilometres wide, and up to 505 metres(1,666 ft) deep. The fauna includes severalspecies of zooplankton and benthic animals.Since the lake is oligotrophic (nutrient-poor), thenative fish fauna is relatively scarce, and theonly endemics are Rasbora tobana.

70. (c) Timber means wood that we obtain fromplants are called renewable in the sense that aftercutting a tree if we plant another tree then it willgrow up and again give us wood and timberwhereas all other things given are non-renewable,once the stock is finished we cannot get more of it.

71. (a) chegonium, the female reproductive organin ferns and mosses. An archegonium alsooccurs in some gymnosperms, e.g., cycads andconifers. A flask-shaped structure, it consists ofa neck, with one or more layers of cells, and aswollen base-the venter-which contains the egg.

72. (b) Trochodendron is a genus of floweringplants with one living species, Trochodendronaralioides, and six extinct species known fromthe fossil record.

73. (a) A Caesarean section (often C-section, alsoother spellings) is a surgical procedure in whichone or more incisions are made through amother's abdomen (laparotomy) and uterus(hysterotomy) to deliver one or more babies. Thefirst modern Caesarean section was performedby German gynecologist Ferdinand Adolf Kehrerin 1881. But in ancient medical history JuliusCaesar was the first person to be borne by thismethod and thus the operation named after him.

74. (b) Study of ants is called Myrmecology.75. (a) The National Institute of Nutrition (NIN)

is an Indian Public health, Biotechnology andTranslational research center located inHyderabad, India. The institute is one of the oldestresearch centers in India, and the largest center,under the Indian Council of Medical Research,located in the vicinity of Osmania University.

76. (a) A Reverse transcriptase (RT) is an enzymeused to generate complementary DNA (cDNA)from an RNA template, a process termed reversetranscription. RT is needed for the replication ofretroviruses (e.g., HIV), and RT inhibitors arewidely used as antiretroviral drugs.

77. (c) Fleming's right-hand rule (for generators)shows the direction of induced current when aconductor moves in a magnetic field. The righthand is held with the thumb, first finger andsecond finger mutually perpendicular to eachother (at right angles).

yoursmahboob.w

ordpress.com

316 SSC CGL SolvedPaper

78. (b) The watt (symbol: W) is a derived unit ofpower in the International System of Units (SI),named after the Scottish engineer James Watt(1736-1819).

79. (d) The NIOSH states "Under dry conditions,the resistance offered by the human body maybe as high as 100,000 Ohms. Wet or broken skinmay drop the body's resistance to 1,000 Ohms,"adding that "high-voltage electrical energyquickly breaks down human skin, reducing thehuman body's resistance to 500 Ohms."

80. (a) The critical temperature forsuperconductors is the temperature at which theelectrical resistivity of a metal drops to zero. Thetransition is so sudden and complete that itappears to be a transition to a different phase ofmatter; this superconducting phase is describedby the BCS theory.

81. (c) A central processing unit (CPU) is theelectronic circuitry within a computer that carriesout the instructions of a computer program byperforming the basic arithmetic, logical, controland input/output (I/O) operations specified bythe instructions.

82. (b) The Epson HX-20 (also known as the HC-20) is generally regarded as the first laptopcomputer, announced in November 1981,although first sold widely in 1983. Hailed byBusinessWeek magazine as the "fourthrevolution in personal computing", it is generallyconsidered both the first notebook and handheldcomputer.

83. (a) Brass is an alloy made of copper and zinc;the proportions of zinc and copper can be variedto create a range of brasses with varyingproperties.

84. (d) Wrought iron is an iron alloy with a verylow carbon content, in comparison to steel, andhas fibrous inclusions, known as slag. This iswhat gives it a "grain" resembling wood, whichis visible when it is etched or bent to the pointof failure. Wrought iron is tough, malleable,ductile and easily welded.

85. (b) Galvanization, or galvanisation, is theprocess of applying a protective zinc coating tosteel or iron, to prevent rusting. The most commonmethod is hot-dip galvanization, in which partsare submerged in a bath of molten zinc.

86. (b) A homogeneous mixture is a type of mixturein which the composition is uniform and everypart of the solution has the same properties. Ahomogeneous mixture in which there is both asolute and solvent present is also a solution.

87. (c) The plant epidermis consists of three maincell types: pavement cells, guard cells and theirsubsidiary cells that surround the stomata.

88. (a) As sugar concentration increases in theguard cells, as a result water enters the guardcells. The guard cells become turgid (swollenwith water). The thin outer walls bulge out andforce the inner wall into a crescent shape. In thisway a stoma or pore is formed between eachpair of guard cell.

89. (a) Revegetation is often used to join uppatches of natural habitat that have been lost,and can be a very important tool in places wheremuch of the natural vegetation has been cleared.It is therefore particularly important in urbanenvironments, and research in Brisbane hasshown that revegetation projects cansignificantly improve urban bird populations.The Brisbane study showed that connecting arevegetation patch with existing habitatimproved bird species richness, while simplyconcentrating on making large patches of habitatwas the best way to increase bird abundance.

90. (c) A dust storm or sand storm is ameteorological phenomenon common in arid andsemi-arid regions. Dust storms arise when a gustfront or other strong wind blows loose sand anddirt from a dry surface.

91. (c) Mahyco, an Indian seed company basedin Jalna, Maharashtra, has developed the Btbrinjal. The genetically modified brinjal event istermed Event EE 1 and Mahyco have also appliedfor approval of two brinjal hybrids.

92. (d) Bullseye, also known as conventionalpistol, is a shooting sport in which participantsshoot handguns at paper targets at fixeddistances and time limits.

93. (c) Rashtriya Ispat Nigam Ltd, (abbreviatedas RINL), also known as Vizag Steel, is an Indiangovernment-owned steel producer. Based inVisakhapatnam, RINL has been awarded theNavaratna status, with accompanying autonomyof operations, due to its strong financialperformance.

94. (d) Pablo Ruiz y Picasso, also known as PabloPicasso, was a Spanish painter, sculptor, printmaker,ceramicist, stage designer, poet and playwrightwho spent most of his adult life in France.

95. (d) The Securities and Exchange Board ofIndia (SEBI) is the regulator for the securitiesmarket in India. It was established in the year1988 and given statutory powers on 12 April1992 through the SEBI Act, 1992.

yoursmahboob.w

ordpress.com

317SSC CGL SolvedPaper

96. (d) Infosys Ltd is an Indian multinationalcorporation that provides business consulting,information technology, software engineeringand outsourcing services. It is headquartered inBangalore, Karnataka.

97. (d) Regional Rural Banks are local levelbanking organizations operating in differentStates of India. They have been created with aview to serve primarily the rural areas of Indiawith basic banking and financial services. Themain purpose of RRB's is to mobilize financialresources from rural / semi-urban areas and grantloans and advances mostly to small and marginalfarmers, agricultural labourers and rural artisans.

98. (b) The Reserve Bank of India is India'sCentral Banking Institution, which controls theMonetary Policy of the Indian Rupee. Itcommenced its operations on 1 April 1935 duringthe British Rule in accordance with the provisionsof the Reserve Bank of India Act, 1934.

99. (d) The national income of a country can bemeasured by three alternative methods: (i)Product Method (ii) Income Method, and (iii)Expenditure Method.

100. (c) For many people, electronic bankingmeans 24-hour access to cash through anautomated teller machine (ATM) or DirectDeposit of paychecks into checking or savingsaccounts. But electronic banking involves manydifferent types of transactions, rights,responsibilities - and sometimes, fees.

101. (b) 334 = (32)17 = 917

251 = (23)17 = 817

Clearly, 717 > 817 > 917

or 717 < 251 < 334

102. (b) Product of first fifty positive consecutiveintegers = 1 × 2 × .... × 50 = 50 !Largest possible value of n

= 250 507 7

é ùé ù + ê úê úë û ë û= 7 + 1 = 8

103. (c) (A + B + C)'s 1 day's work = th1

40æ öç ÷è ø part

of whole work

(A + B + C)'s 16 days work = 16 240 5

= of wholework (B + C) completes remaining work in 40 days

(B + C) completes th3

5æ öç ÷è ø part of work in 40 days.

(B + C) completes whole work in 40 5 200

3 3´ =

days.

1 1 1 1A B C 40

+ + =

1 3 1A 200 40

+ =

1 1 1 4A 40 200 200= - =

1 1A 50

=

A alone can complete whole work in 50 days.

104. (c)1 1 1 1A B C 6

+ + =

(A + B + C) can do 2 16 3

= part of work in 2 days.

Remaining work = 1 2

13 3- =

In one hour (A + B) can do 2

3 7´ part of work

1 1 1 1C 6 B C

æ ö= - +ç ÷è ø1 1 2 3C 6 21 42= - =

C = 14 hours105. (c) Let Pratibha can finish the work in x days

then, Sonia can finish the same work in 3x daysAccording to question3x – x = 602x = 60 Þ x = 30Pratibha and Sonia can individually complete

the work in 30 days and 90 days respectively.

106. (c) Height of base = 3

2a where a = 4

=3 42´ = 2 3

Ao = 23 ×

32

× 4 = 43

cm

A

B C

D

O

Slant edge BD = 5cm

Verticle height Do2 =(5)2 –24

3æ öç ÷è ø

= 25 –163

yoursmahboob.w

ordpress.com

318 SSC CGL SolvedPaper

= 75 16

3-= =

593

= Do

Volume of Pyramid = 13 ar of base × height

=13 ×

34

× 4 × 4 × 593

= 4 59

3107. (a) C1 = 2C2

pr1l1 = 2pr2l2also, l2 = 2l1pr1l1 = 2 × 26 pr2l1

1

2

r 4r 1

=

108. (b) Let r be the radius of circle.pr2 = 154 cm2

r2 = 154

722´ = 49

r = 7 cmlength of wire = circumference of circle

= 22

2 77

´ ´ = 44 cm

Now, Perimeter of equilateral triangle = 44 cm

side = 443

cm

Area of equilateral triangle = 23 44

4 3æ ö´ ç ÷è ø

= 484 3

9 = 91.42 cm2

Area of equilateral triangle is nearly equal to90.14 cm2

Hence, option (b) is correct.109. (c) Let marked price of goods be ̀ 100.

Selling price of goods = 10100 100

100- ´ = ̀ 90

Cost price of goods is 80% of its selling price

C.P. = 80 90

100´ = 72

Profit on goods = (90 – 72) = ̀ 18

Profit % = 18 10072´ = 25%

110. (a) Let marked price of the instrument be ̀ x

Selling price, S.P. = x – 20

100x = 0.8x

Cost price, C.P. = C.P. + 25

100 C.P. = 0.8x

C.P. = 0.8 100 16

125 25´ = x

x = 2516

C.P..

Given that 25

100 C.P = 150

Þ C.P. = 150 100

25´

= 600

Marked price x = 25 6,00016´ = ` 938.50

111. (b) Let labelled price of T.V. be ̀ x

Price after 20% discount, x – 20

x100

= 0.8x

Price after 30% discount, x – 30

x100

= 0.7xAccording to question0.8x – 0.7x = 800x = 800 × 10 = 8000

112. (d) Quantity of acid left = 2210 1

10æ ö-ç ÷è ø =

325

Required ratio = 32 16

5 10 25=

´ = 16 : 25

113. (d) By rule of alligation, we haveGold

19 timesCopper9 times

15 – 9 = 6 19 – 15 = 4

15 times

\ Required ratio = 64

= 3 : 2

114. (c) Number of boys = 4 505´ = 40

Number of girls = 1

505´ = 10

Average age of boys = 2 × 20 = 40Total ages of the boys = 40 × 20 = 800

115. (c) Total marks of all three sections = 84 × 100= 8400Total marks of (B + C) = 87.5 (n2 + n3)total marks of A = 70 × n1

yoursmahboob.w

ordpress.com

319SSC CGL SolvedPaper

n1 + n2 + n3 = 100 ...(1)70n1 + 87.5 n2 + 87.5 n3 = 8400 ...(2)Multiplying equation (1) by 87.5 and subtract

from equation (2)We get 17.5 n1 = 350n1 = 20

116. (b) Let ` 100 be the cost price for A.S.P. for A = 100 + 20% of 100 = 120S.P. for B = 120 – 15% of 120 = 102

Profit % = 102 100 100

100- ´ = 2%

117. (a) Let the monthly salary of A be x,, monthlysalary of B is (40000 – x).Savings of A = (100 – 85)% of x = 0.15xSavings of B = (100 – 95)% of (40000 – x) = 0.05 (40000 – x)0.15 x = 0.05 (40000 – x)0.15x + 0.05x = 40000 × 0.050.2x = 2000x = 10000

118. (c) Let T be the speed of train and C be thespeed of car.120 480T C

+ = 8 Þ 1 4 1T C 15

+ = .....(1)

200 400T C

+ = 20860

+ Þ 1 2 1T C 24

+ = ....(2)

Subtracting (2) from (1)2 1 1

(2 1)C 15 24- = -

2C

= 140

Þ C = 80

1T

= 1

15 – 4

801T

= 1

60 Þ T = 60

Required ratio = 60 : 80 = 3 : 4119. (c) Let correct time to cover journey be t hours

1270 t

60æ ö+ç ÷è ø =

380 t

60æ ö+ç ÷è ø

70t + 14 = 80t+ 410t = 10t = 1 hour

120. (b) Let the required amount = p4P 10 P 4 51 1 3205

2 100 100

é ù ´ ´æ ö+ - - =ê úç ÷è øê úë û

4P 11 P1 32052 100 5

é ùæ ö - - =ê úç ÷è øê úë ûP 14641 P1 32052 10000 5é ù- - =ê úë û

P 4641 P 32052 10000 5é ù - =ê úë û

4641P P 320520000 5

- =

641P 320520000

=

3205P 20000641

= ´ = ̀ 10,0000

121. (c) a2 + b2 + c2 = 2a – 2b – 2(a2 – 2a + 1) + (b2 + 2b + 1) + c2 = 0(a – 1)2 + (b + 1)2 + c2 = 0This equation is possible ifa – 1 = 0, b + 1 = 0 and c = 0a = 1, b = –1, c = 03a – 2b + c = 3 × 1 – 2 × (–1) + 0 = 3 + 2 = 5

122. (b) a + b + c = 3Squaring both sidesa2 + b2 + c2 + 2 (ab + bc + ac) = 96 + 2 (ab + bc + ca) = 9

ab + bc + ca = 32

...(1)

given 1 1 1a b c

+ + = 1

Þ ab + bc + ac = abc = 32

[from (1)]

123. (d) a2 – 4a – 1 = 0a2 – 4a = 1a (a – 4) = 1

a – 4 = 1a

1aa- = 4 ...(1)

We have a2 + 3a + 21 3

aa-

22

1 1a 3 aaa

æ ö æ ö+ + -ç ÷ç ÷ è øè ø21 1a 3 a 2

a aæ ö æ ö- + - +ç ÷ ç ÷è ø è ø42 + 3 × 4 + 2 = 30

yoursmahboob.w

ordpress.com

320 SSC CGL SolvedPaper

124. (a)

A (5, 4)

4x + 5y = 40

C (10, 0)B(5, 0)

Area of DABC = 12

× BC × AC

= 12

× (10 – 5) × 4 = 12

× 5 × 4

Area = 10 sq unit.125. (d) Kx + 2y = 2 ...(1)

3x + y = 1 ...(2)divide eqn (1) by (2)

K y2

+ = 1

for system of equation to be coincident

K2

= 3

K = 6126. (d) x = 2 3+

1x

= 1 2 3

2 3 2 3-´

+ - = 2 3-

22

1x

x+ =

21x

xæ ö+ç ÷è ø – 2

= ( )22 3 2 3 2+ + - -

= 16 – 2 = 14127. (c) a = 4.965 » 5, b = 2.343 » 2

c = 2.622a – b = ctaking cube both sidesa3 – b3 – 3a2b + 3ab2 = c3

a3 – b3 – c3 – 3ab(a – b) = 0a3 – b3 – c3 – 3abc = 0

128. (d) x + y + z = 0y + z = –xy2 + z2 + 2yz = x2

Þ y2 + z2 = x2 – 2yz ...(1)2 2 2

2x y z

x yz+ +-

= 2 2 2

2 2x 2yz x 2(x yz)

x yz x yz- + -=- -

= 2

129. (d) (sin2 1° + sin2 89°) + (sin2 2° + sin2 88°) + ...+ (sin2 44° + sin2 48°) + sin2 45°

= (sin2 1° + cos2 1°) + (sin2 2° + cos2 2°) + ...+ (sin2 44° + cos2 44°) + sin2 45°

= 1 + 1 + .... + 1 (44 times) + 12

= 1442

130. (c)2 2(cos sin )(cos sin sin cos )

(cos sin )q + q q + q - q q

q+ q

+ 2 2(cos sin )(cos sin sin cos )

(cos sin )q - q q + q + q q

q - q= 2 cos2 q + 2sin2 q – sin q cos q + sinq cosq = 2

131. (a)

60°45°

h

xCD

A

B30 m

In DABC, tan60° = hx

x = h3 ...(1)

In D ABD, tan 45° = h

30 x+

1 = h

30 x+ or h = 30 + x

Putting value of x from (1)

h = h

303

+

or ( 3 1)h

3-

= 30 Þ h = 15 (3 3)+ m

yoursmahboob.w

ordpress.com

321SSC CGL SolvedPaper

132. (d) sin17° = xy

cos 17° = 2

2x1y- =

2 2y xy-

sec 17° – sin 73°= sec 17° – cos 17°

= 2 2

2 2

y xyyy x

--

-

= 2 2 2

2 2

y y x

y y x

- +

- = 2 2

x

y y x

2

-

133. (c) cosecq + cotq = 31

sinq +

cossinqq

= 3

1 cossin+ qq

= 3

22cos2

2sin cos2 2

q

q q = 3

cot2q

= 3

tan2q

= 13 ;

2q

= 30°; q = 60°

cosec q = cosec 60° = 23

134. (c) cosa + sec a = 3taking cube both sidescos3a + sec3 a + 3 cosa seca (cosa + seca)

= 3 3cos3a + sec3 a + 3 3 = 3 3cos3a + sec3 a = 0

135. (a) sinq + cos q = 2 cos q

sinq = ( 2 1)- cos q

cotq = 1

2 1-

cotq = 1 2 1 2 1

2 1 2 1+´ = +

- +

136. (d) A B

CD

O

130° 70°

A + B + C + D = 360A + B = 360 – (130 + 70) = 160°A B2 2

+ = 80° ...(1)

In D AOB,A B

02 2

+ + = 180°

0 = 180° – 80° = 100°137. (d) In D ADE and D ABC

A DÐ = Ð A

D

B

C

EB DÐ =Ð

By AA ADE ABCD D:

AE ACAD AB

=

3 2 DC2 3 2

+=+

15 = 4 + 2 DC11 = 2 DC

5.5 = DC138. (c)

O

A B

P

60°

In D AOB, ÐA + ÐB + ÐO = 180°ÐA + Ð B = 180 – 140° = 40°ÐA = ÐB = 20° {AO = BO}ÐPAO = 90°ÐPAB + ÐBAO = 90°ÐPAB = 90° – 20° = 70°

139. (c) C

A BD

C1 C2

yoursmahboob.w

ordpress.com

322 SSC CGL SolvedPaper

In D ADC,A + D + C1 = 180°; A + C1 = 180° – 90° = 90°In D BDC,B + D + C2 = 180°; B + C2 = 180° – 90° = 90°A + C1 = B + C2C1 – C2 = B – A

140. (b)190 BAC 1202+ Ð =

1 BAC 302Ð =

BAC 60Ð = °

A

B C

O

While BOCÐ is in circle then

BOCÐ = 190 BAC2+ Ð

141. (b)

A

C B

O 10 cm

6 cm

8 cm

6 cm

8 cmD

In DADO,

OD = 2 2(AO) AD-

= 2 2100cm 64cm- = 6 cmIn DBCO,

OC = 2 2OB CB-

= 2 2100cm 36cm- = 8 cmdistance between chords = OC – OD

= 2CM142. (a) Let n be the number of sides.

(n – 2) × 180° = 140° × n180n – 360 = 140 n

40n = 360

n = 36040

= 9

143. (d)

9 cm

9 cm 4 cm

C

O

A

B

O'4 cm

In figure, AC = AO – CO= 9 cm – 4 cm = 5 cm {CO = BO'}Also, CB = OO¢ = 13 cmIn D ABC

AB = 2 2CB AC-

= 2 2(13cm) (5cm)-= 12 cm

144. (b) Number of students scoring less than50%= (240 + 220 + 300 + 280 + 210) = 1250Number of student scoring exact 50%= (30 + 20 + 0 + 35 + 15) = 100Ratio = 1250 : 100 = 25 : 2

145. (a)146. (c) Number of students scoring 50% or more

marks= (600 – 240) + (400 – 220) + (375 – 300) + (350

– 280) + (300 – 210)= 360 + 180 + 75 + 70 + 90 = 775

147. (b) Profit during 2006 = (60 – 35) = 25Profit during 2007 = (50 – 40) = 10Difference = 25 – 10 = 15

148. (c) Average Income =40 60 50 65 70 285

5 5+ + + +

= = 57

Income during 2005 and 2007 is less thanaverage

149. (b) Required % = 50 40

10040-

´ = 25%

150. (c) It is clear from the graph.151. (a) Part 'A' of the statement is wrong. It will be

as 'If I had realized'. The Past perfect tense suitshere and it denotes the sentence to be in Activevoice.

yoursmahboob.w

ordpress.com

323SSC CGL SolvedPaper

152. (b) Part 'B' of the statement is wrong. The word'enhanced' given here is wrong. The correct wordthat will suit the statement is 'topper'. Correctapplication of word enriches the meaning of thesentence.

153. (b) Part 'B' of the sentence is wrong. The cor-rect form of sentence is 'It is I am who should beresponsible for the delay'.

154. (b) Part 'B' of the sentence is wrong. The prepo-sition 'of' is missing in that part. 'Of' is applicableafter the word 'evils'.

155. (b) Part 'B' of the sentence is wrong. It will bearranged as 'over extensively'. In the part 'B' 'ex-tensively over' is denoting wrong application ofwords which is acting as a breach to the meaningof the sentence.

156. (a) The correct word that will fit the blank is'impudence'. 'Impudence' means 'not to show theproper respect'. Hence, the word suits the mean-ing of the sentence.

157. (b) The correct word that will fit the blank is'feeble'. 'Feeble' means 'weak'. Hence, the wordsuits the meaning of the sentence.

158. (a) The correct word that will fit the blank is'retrieve'. 'Retrieve' means to 'get or bring back'.Hence, the word aptly suits the meaning of thesentence.

159. (c) The correct word that will fit the blank is'over'. As it is given in the sentence, 'makeover'means to 'renew or renovate something'. Hencethe preposition 'over' will suit the meaning of thesentence.

160. (c) The correct word that will fit the blank is'augurs'. 'Augurs' means 'something that will pro-ceed well'. Hence, the word suits the meaning ofthe sentence.

161. (d) 'Persist' means 'to insist'.162. (c) 'Eventually' means 'finally'.163. (c) 'Impeccable' means 'flawless'.164. (c) The opposite of 'predilection' is 'aversion'.165. (b) The opposite of 'pompous' is 'humble'.166. (c) The opposite of 'serene' is 'ruffled'.167. (d) In the sentence, 'to carve out a niche' means

'to develop a specific position for him'. The word'niche' means 'a specialized area or sector'.

168. (c) The alternative meaning of the phrase 'tothe letter' is 'in every detail'.

169. (c) The meaning of the idiom 'to read betweenthe lines' is 'to understand the inner meaning'.So, the critic's work is to understand the innermeanings.

170. (a) The meaning of the idiom 'put my footdown' is 'to take a firm stand'. So, as the meaningof the sentence implies that where disciplinematters, one should take a firm stand.

171. (d) The idiom 'stood his ground' means 'refuseto yield'. So, as the meaning of the sentence im-plies, the convict claimed his innocence and re-fused to yield for the blames that were laid uponhim.

172. (a) The sentence requires an improvement. Theunderlined portion must be rectified to 'you de-spair of the success of your undertaking'.

173. (d) The statement requires no improvement.The underlined portion of the statement suitsbest with the meaning of the statement.

174. (c) The underlined portion of the statementrequires an improvement. It may be rectified as'here is something pretty that Vinita can wear tothe party.

175. (a) The underlined portion requires an im-provement. It may be rectified as 'of owning'. So,the sentence will be as 'I have dreamt all my lifeof owning a beautiful maroon colored car'.

176. (a) The underlined portion in the sentence re-quires an improvement. It may be rectified as 'hewatched the sun go down'. So, the sentence willbe 'sitting on the top of the hill he watched thesun go down'.

177. (b) The underlined portion of the sentence hasto be rectified. 'Soon to' may be replaced with'about to'. So, the sentence will be as 'the office isabout to close'.

178. (a) The underlined portion of the sentenceneeds to be rectified. It may be replaced with 'outof the way'. So, the sentence will be as 'He hasachieved nothing out of the way worth mention-ing'.

179. (c) 'On' may be replaced with 'upon'. So, thesentence will be as 'I prevailed upon him to votefor you'.

180. (b) The underlined portion may be replacedwith 'the student made studying his top priority'.So, the sentence will be as 'Eager to pass his finalexams the student made studying his top prior-ity'.

181. (c) The underlined portion in the sentencemay be replaced with 'unless he is invited'. So,the sentence will be as 'Mr.Dev will not go to thewedding reception unless he is invited'.

182. (c) Belief in many Gods-Polytheism.183. (b) Cluster of flowers on a branch-Inflores-

cence.

yoursmahboob.w

ordpress.com

324 SSC CGL SolvedPaper

184. (b) A person who believes that only selfish-ness motivates human action-Cynic.

185. (b) A highly skilled musician-Virtuoso.186. (d) A method of boiling briefly to cook food

slightly Parboil.187. (a) The group, especially in arts, regarded as

being the most experimental Avant-garde.188. (a) One who helps people by giving them

money or other aid Benefactor.189. (a) Among the four options 'plebeian' is cor-

rectly spelt word.190. (b) Among the four options 'surroundings' is

the correctly spelt word.191. (b) The writer felt unusually solitary because

he was missing the company of other holidaymakers.

192. (c) "I left all signs of habitation behind me"This means that he had come very far from places

where people lived.193. (b) It became darker than the writer expected

because the nights are longer in October thanmidsummer.

194. (d) The writer found it difficult to keep to thepath because of the poor visibility and dew ongrass.

195. (d) When he settled himself on the fork of thetree the writer tried to sleep but without muchsuccess.

196. (d) At the beginning of the passage the writerexpresses her opinion that in many countriesprogress is synonymous with utmost cruelty tonature.

197. (b) In the passage the term 'exploiting' naturesuggests 'sarcasm'.

198. (b) Nehru objected to the phrase 'conquest ofEverest' since it sounds pompous and boastful.

199. (c) Gandhiji's statement 'It is decreasing in thejungles but it is increasing in the towns!' refersto man's selfishness.

200. (d) The writer is of opinion that tribal peoplecan be prevented from combing forest or food toprovide employment and purchasing power fordaily necessities.

yoursmahboob.w

ordpress.com

PART-A : GENERAL INTELLIGENCE & REASONING

Directions (Qs. 1-2): In the following two Questions,which one set of letters when sequentially placed atthe gaps in the given letter series shall complete it?1. ccbab _ caa _ bccc _ a _

(a) babb (b) bbba (c) baab (d) babc2. a_ _ __dba_ _bcad__ _da__ _cd

(a) bccdbcab (b) abcddcba(c) cbcddcba (d) aabbccdd

Directions (Qs. 3-4): In the following two Questions,a series is given, with one term missing. Choose thecorrect alternative from the given ones that willcomplete the series.3. 4, 6, 10, 16, 24, ?

(a) 28 (b) 30 (c) 34 (d) 404. 3, 5, 9, 17, _____?____ K ,

(a) 26 (b) 65 (c) 33 (d) 425. A train starts from station A and reaches B 15

minutes late when it moves with 40 km/hr and 24minutes late when it goes 30 km/hr. The distancebetween the two stations is(a) 16 km (b) 18 km (c) 21 km (d) 24 km

6. In a row of men, Manoj is 30th from the right andKiran is 20th from the left. When they interchangetheir position, Manoj becomes 35th from theright. What is the total number of men in therow?(a) 45 (b) 44 (c) 54 (d) 34

7. Seven persons A, B, C, D, E, F and G are standingin a straight line.D is to the right of G.C is between A and B.E is between F and D.There are three persons between G and B. Whois on the extreme left?(a) A (b) B (c) D (d) G

8. From the given alternative words, select the wordwhich cannot be formed using the letters of thegiven word:CUMBERSOME(a) MOUSE (b) SOBER(c) ROME (d) MERCY

9. Name a single letter, which can be prefixed to thefollowing words in order to obtain entirely newwords ?TILL TABLE PILE TAB PRING(a) S (b) B (c) H (d) C

10. Unscramble the following letters to frame ameaningful word. Then find out the correctnumerical position of the letters :

B C U S M E L R N A1 2 3 4 5 6 7 8 9 10(a) 6 1 4 3 2 5 8 7 9 10 (b) 3 1 5 7 10 4 2 6 9 8(c) 3 9 4 2 8 10 5 1 7 6 (d) 2 1 3 4 6 8 9 7 5 10

11. Using the following code and key decode thegiven coded word:

Code : L X P Z J Y Q M N BKey : b a e s p r h i g t

Coded word : Z B Y X M N Q B(a) height (b) struggle(c) straight (d) strength

12. In a certain code "MOUSE" is written as"PRUOC". How is "SHIFT" written in that code?(a) VJIDR (b) VIKRD(c) RKIVD (d) VKIDR

13. In a certain code, '253' means 'books are old'; '546'means 'man is old' and '378' means 'buy goodbooks'. What stands for "are" in that code?(a) 2 (b) 4 (c) 5 (d) 6

14. If, + stands for division; × stands for addition ; –stands for multiplication : ¸ stands forsubtraction, which of the following is correct ?A. 46 × 6 ̧ 4 – 5 + 3 = 74B. 46 – 6 + 4 × 5 ̧ 3 = 71C. 46 ̧ 6 × 4 – 5 + 3 = 75.5D. 46 × 6 – 4 + 5 ̧ 3 = 70.1(a) D (b) B (c) A (d) C

15. If + = ×, – = ¸, × = +, ¸ = –, then which is thecorrect equation out of the following?(a) 18 ¸ 6 + 4 – 2 ̧ 3 = 22(b) 18 + 6 – 4 × 2 ÷ 3 = 26(c) 18 × 6 – 4 + 7 × 8 = 47(d) 18 – 6 × 7 ̧ 2 + 8 = 63

SSC Combined Graduate Level (CGL) Solved PaperMORNING SHIFT 09 AUGUST, 2015

yoursmahboob.w

ordpress.com

326 SSC CGL SolvedPaper

Directions (Qs. 16-20): In the following Five Questions,select the missing number from the given responses.

16.?

8 2137

17

(a) 97 (b) 907 (c) 1097 (d) 9107

17.14

36

7 3

12 9 11

54

16 ?

(a) 12 (b) 17 (c) 18 (d) 1618. 9 11 13

13 15 1710 12 1414 16 1811 13 ?(a) 22 (b) 14 (c) 15 (d) 21

19. 3 2 2 1 6 54 24 2 4 0 ?

--

(a) 30 (b) 11 (c) 0 (d) 120. 7 5 3

8 4 92 8 ?112 160 162(a) 4 (b) 6 (c) 8 (d) 12

21. Pinky walks a distance of 600 mtr. towards east,turns left and moves 500 mtr, then turns left andwalks 600 mtr. and then turns left again and moves500 mtr and halts. At what distance in metres isthe from the starting point?(a) 2200 (b) 500 (c) 0 (d) 600

22. Sunita rode her scooty northwards, then turnedleft and then again rode to her left 4 km. Shefound herself exactly 2 kms west of her startingpoint. How far did she ride northwards initially ?(a) 2 km (b) 4 km (c) 5 km (d) 6 km

Directions (Qs. 23-24) : In the following TwoQuestions, one statement is given followed by twoconclusions, I and II. You have to consider thestatement to be true, even if it seems to be at variancefrom commonly known facts. You are to decide whichof the given conclusions can definitely be drawn fromthe given statement. Indicate your answer.

23. Statement : Every school should promotepartnerships that will increase parentalinvolvement and participation for promoting thegrowth of children.Conclusions I : For the growth of the children,parents should be involved in various schoolactivities.Conclusions II. Involvement of parents in schoolactivities has no influence on the growth of thechildren.(a) Only I follows(b) Only II follows(c) Neither I nor II follows(d) Both I and II follow

24. Statement : Aggressive animals can be trainedwith care and affection to behave as theoccassion demands.Conclusions I : Trained dogs cannot beaggressive.Conclusions II : Animals are always aggressiveunless care and affection is given to them.(a) Only I follows(b) Only II follows(c) Neither I nor II follows(d) Both I and II follow

Directions (Qs. 25-30): In the following SixQuestions, select the related word/letters/numberfrom the given alternatives.

25. Haematology : Blood : : Phycology : ?(a) Fungi (b) Fishes(c) Algae (d) Diseases

26. Pride of Lions : : ________ of cats(a) Herd (b) School(c) Clowder (d) Bunch

27. MAN : PDQ : : WAN : ?(a) ZDQ (b) NAW (c) YQD (d) YDQ

28. AEFJ : KOPT : : ? : QUVZ(a) GLKP (b) GKLP(c) HLKP (d) HKQL

29. 2 : 32 : : 3 : ?(a) 243 (b) 293 (c) 183 (d) 143

yoursmahboob.w

ordpress.com

327SSC CGL SolvedPaper

30. D × H : 4 × 8 as M × Q: ?(a) 12 × 17 (b) 12 × 16(c) 13 × 17 (d) 14 × 18

Directions (Qs. 31-36): In the following SixQuestions, find the old word/ number/ letters / numberpair from the given alternatives.

31. (a) Morning (b) Noon(c) Evening (d) Night

32. (a) Liberty (b) Society(c) Equality (d) Fraternity

33. (a) DWFU (b) EVHS(c) HSKP (d) KQNN

34. (a) CBEF (b) EDGH (c) IHKL (d) GFHJ35. (a) 4025 (b) 7202 (c) 6023 (d) 506136. (a) 96 : 80 (b) 64 : 48

(c) 80 : 60 (d) 104 : 7837. Choose the correct alternative to complete the

series.Lily, Daisy, Datura, ?(a) Sun Flower (b) Hibiscus(c) Marigold (d) Jasmine

38. Which one of the given responses would be ameaningful order of the following ?1. Elephant 2. Cat3. Mosquito 4. Tiger5. Whale(a) 5, 3, 1, 2, 4 (b) 1, 3, 5, 4, 2(c) 3, 2, 4, 1, 5 (d) 2, 5, 1, 4, 3

39. How many triangles are there in the figure ?

(a) 24 (b) 14 (c) 28 (d) 2040. The figure given on the left hand side is folded

to form a box. Choose from the alternatives (1),(2), (3) and (4) the boxes that is similar to the boxformed.

(1) (2) (3) (4)

(a) (2) and (3) only (b) (1), (3) and (4) only(c) (2) and (4) only (d) (1) and (4) only

41. Find out the number of circles in the given figure:

(a) 14 (b) 16 (c) 17 (d) 1842. Identify the diagram that best represents the

relationship among the classes given below :Animals and animals, sea animals

(a) (b)

(c) (d)

43. 1

5 6 24

3DoctorsProfessorsMarried People

Which number indicates doctors who are notmarried ?(a) 6 (b) 4 (c) 2 (d) 1

44.

P R K

Q

In the fig.Q represents all quadrilateralsK represents all KitesR represents all RhombusP represents all ParallelogramThe statement 'Rhombus is also a Kite' can bedescribed as(a) P and K is nothing but R(b) P or K is nothing but R(c) P and R is nothing but K(d) P or R is nothing but K

yoursmahboob.w

ordpress.com

328 SSC CGL SolvedPaper

DIRECTIONS (Qs. 45-46): In the following TwoQuestions, which answer figure will complete thepattern in the questiion figure ?45. Question figure

Answer figures :

(a) (b)

(c) (d)

46. Question figure

Answer figures :

(a) (b)

(c) (d)

47. From the given answer figures, select the onewhich is hidden/embedded in the question figure:Question figure :

Answer figures :

(a) (b)

(c) (d)

48. A piece of paper is folded and cut as shown belowin the question figures. From the given answerfigures indicate how it will appear when opened.Question figure :

Answer figures :

(a) (b)

(c) (d)

49. If a mirror is places on the line MN, then which of theanswer figures is the right image of the given figure?Question figure:

M

N Answer figures :

(a) (b)

(c) (d)

yoursmahboob.w

ordpress.com

329SSC CGL SolvedPaper

50. Direction : A word is represented by only oneset of numbers as given in any one of thealternatives. The sets of numbers given in thealternatiives are represented by two classes ofalphabets as in two matrices given below. Thecolumns and rows of Matrix I are numbered from0 to 4 and that of Matrix II are numbered from 5 to9. A letter from these matrices can be representedfirst by its row and next by its column, e.g.. 'A'can be represented by 01, 14 etc. and 'O' can berepresented by 59, 67 etc. similarly, you have toidentify the set for the word 'PEARL'.

Matrix – I

0 1 2 3 40 P A G R Z1 G R Z P A2 Z P A G R3 A G R Z P4 R Z P A G

Matrix – II

5 6 7 8 95 E M L N O6 L E O M N7 O N E L M8 N O M E L9 M L N O E

(a) 00, 55, 22, 11, 96 (b) 00, 66, 14, 32, 56(c) 13, 77, 30, 14, 88 (d) 12, 88, 43, 32, 89

PART-B : GENERAL AWARENESS51. To whom the line 'A thing of beauty is a joy for

ever' is attributed ?(a) John Keats(b) Dr. Charles Dickens(c) Dr. Jonathan Swift(d) William Wordsworth

52. The birthday of which of the following leaders iscelebrated as 'Teachers Day' in India?(a) Dr. Rajendra Prasad (b) S. Radhakrishnan(c) C. Rajgopalachari (d) Lala Lajpat Rai

53. The award given for outstanding performance insports is(a) Bharat Ratna(b) Padma Shri Award(c) Arjuna Award(d) Dronacharya Award

54. Which hill station's name means place of thethunderbolt'?(a) Shillong (b) Oottacamand(c) Darjeeling (d) Gangtok

55. The ship building yard––Mazgaon Dock islocated at –(a) Kochi (b) Kolkata(c) Mumbai (d) Vishakhapatnam

56. Electric current is measured using which of thefollowing instrument ?(a) Voltmeter (b) Anemometer(c) Wattmeter (d) Ammeter

57. 'Agha Khan Cup' is related with which of thefollowing sport event ?(a) Cricket (b) Hockey(c) Table Tennis (d) Football

58. Where was the first conference of SAARC (SouthAsian Association for Regional Cooperation) held?(a) Dhaka (b) New Delhi(c) Colombo (d) Kathmandu

59. Which among the following is not a BrettonWoods Institution ?(a) International Monetary Fund (IMF)(b) World Bank(c) Organisaiton of Economic Cooperation and

Development (O.E.C.D.)(d) None of these

60. Equilibrium price in the market is determined bythe(a) equality between total cost and total revenue(b) equality between average cost and average

revenue.(c) equality between marginal cost and marginal

revenue(d) equality between marginal cost and average

cost.61. In the national context which of the following

indicates Macro Approach ?(a) Sales of Bata Shoe Company(b) Exports of Mangoes to U.K.(c) Income from Railways(d) Inflation in India

62. Internal economies(a) arise in an economy as it makes progress(b) accrue to a firm when it expands its output(c) arise when there is expansion in internal

trade(d) arise when there is expansion in an industry

yoursmahboob.w

ordpress.com

330 SSC CGL SolvedPaper

63. One of the features of a free market economy is(a) public ownership of factors of production(b) rationing and price control(c) consumer's sovereignty(d) active state intervention

64. Gross National Product – Depreciation Allowance= ?(a) Gross Domestic Product(b) Personal Income(c) Net National Product(d) Per Capita Income

65. The Panchayat Samiti remains accountable forits functions to(a) The Gram Panchayats and Gram Sabhas(b) Zilla Parishads(c) Anchal Panchayats(d) Janpad Panchayats

66. The legislature gains a priority over the executivein(a) A Federal Government(b) An Authoritarian Government(c) A Parliamentary Government(d) A Presidential Government

67. The legislature in a democratic country caninfluence public opinion by(a) Granting rights(b) Enacting non controversial laws(c) Defining the duties of the citizens(d) Focusing attention on public issues

68. If the President wants to resign from his office,he may do so by writing to the(a) Vice President(b) Chief Justice of India(c) Prime Minister(d) Speaker of Lok Sabha

69. Which of the following is not a Union Territory ?(a) Lakshadweep(b) Puducherry(c) Nagaland(d) Dadra and Nagar Haveli

70. The greatest king of the Pratihara dynasty was(a) Bhoj (Mihir–Bhoj) (b) Dantidurga(c) Nagbhatta II (d) Vatsaraj

71. In 1939 Subhash Chandra Bose was elected asPresident of the Congress Party defeating(a) Jawaharlal Nehru(b) Maulana Abul Kalam Azad(c) V.B. Patel(d) Pattabhi Sitharamayya

72. Jallianwala incident took place at(a) Lucknow (b) Surat(c) Amritsar (d) Allahabad

73. Who was the founder of Lodhi dynasty ?(a) Sikandar Lodhi (b) Bahlol Lodhi(c) Ibrahim Lodhi (d) Daulat Khan Lodhi

74. Which one of the following pair is not correctlymatched ?(a) Akbar – Todarmal(b) Chanakya – Chandragupta(c) Vikramaditya – Chaitanya(d) Harshvardhan – Hiuen Tsang

75. The South East trade winds are attracted towardsthe Indian sub continent in the rainy season dueto(a) the effect of easterlies(b) the effect of Northern–East trade winds(c) the presence of low atmospheric pressure

over North–West India(d) the development of cyclone over the equator

76. The 'graded profile' of a river course is a(a) smooth curve in the upper course(b) smooth curve in the middle course(c) smooth curve in the lower course(d) smooth curve from source to mouth

77. Sink hole is a phenomenon of _______topography.(a) Desert (b) Tundra (c) Karst (d) Plain

78. Kerala is famous for the cultivation of1. Coconut 2. Black pepper3. Rubber 4. Rice(a) 1, 2 and 4 (b) 2, 3 and 4(c) 1 and 4 (d) 1, 2 and 3

79. The longest continental Railway in the world is(a) Trans Siberian Railway(b) Canadian Pacific Railway(c) Canadian National Railway(d) Trans Atlantic Railway

80. Photoperiodisrn affects(a) Flowering (b) Vegetative growth(c) Fruiting (d) All of these

81. Match the following : I II

A. Ascorbic acid 1. Photosyntheticpigment

B. Chlorophyll 2. QuencherC. Carotenoid 3. EnzymeD. Superoxide 4. Vitamin–C

dismutaseA B C D

(a) 4 2 1 3(b) 2 4 1 3(c) 4 1 3 2(d) 4 1 2 3

yoursmahboob.w

ordpress.com

331SSC CGL SolvedPaper

82. Allantois of Embryo helps in(a) respiration (b) excretion(c) protection (d) digestion

83. Which one of the following animals belongs tomollusca ?(a) Hare (b) Hydra(c) Hyla (d) Haliotis

84. Outside the nucleus DNA is found in(a) Mitochondria(b) Ribosome(c) Endoplasmic reticulum(d) Golgi bodies

85. Animal protein is called first class protein becauseit is(a) delicious in taste(b) cheaper in the market(c) rich in essential amino acids(d) easily digestible

86. It is easy to burst a gas filled balloon with a needlethan with a nail. It is because(a) nail exerts more pressure than needle on the

balloon(b) needle exerts more pressure than nail on the

balloon(c) gas is reactive with the needle(d) nail is more longer than needle

87. The velocity of sound in moist air is more than indry air because the moist air has(a) less pressure than dry air(b) more pressure than dry air(c) more density than dry air(d) less density than dry air

88. X–rays can be used(a) to detect heart diseases.(b) to detect defects in precious stones and

diamonds.(c) to detect gold under the earth.(d) for cutting and welding of metals.

89. Ice is packed in saw dust because(a) saw dust is poor conductor of heat.(b) saw dust is a good conductor of heat.(c) saw dust does not stick to the ice.(d) saw dust will not get melted easily.

90. What is used to identify whether a data wordhas an odd or even number of 1's ?(a) Sign bit (b) Zero bit(c) Parity bit (d) Carry bit

91. Rearranging and allocating space in memory toprovide for multiple computing tasks is called(a) Multiprogramming

(b) Multitasking(c) Memory Management(d) Networking

92. What happens when a drop of glycerol is addedto crushed KMnO4 spread of a paper ?(a) There is a violent explosion(b) There is no reaction(c) The paper ignites(d) There is a crackling sound.

93. Most commonly used bleaching agent is(a) Alcohol (b) Carbon dioxide(c) Chlorine (d) Sodium chloride

94. The least penetrating power ray is(a) a–Ray (b) b–Ray(c) g–Ray (d) X–Ray

95. Hydrogen peroxide is an effective sterilizingagent. Which one of the following productresults when it readily loses active oxygen ?(a) Water (b) Hydrogen(c) Ozone (d) Nasant Hydrogen

96. The maximum fixation of solar energy is done by(a) Bacteria (b) Fungi(c) Green plants (d) Protozoa

97. The term 'brown air' is used for(a) Photochemical smog(b) Sulfurous smog(c) Industrial smog(d) Acid fumes

98. Which of the following is FALSE with respect torain water harvesting?(a) It helps raising water table(b) It helps meet rising water demand(c) It increases run–off losses(d) It is a device of water conservation

99. Peroxyacetyl nitrate is a(a) Plant hormone(b) Vitamin(c) Secondary pollutant(d) Acidic dye

100. Which of the following river does not originatein Indian territory ?(a) Mahanadi (b) Brahmaputra(c) Satluj (d) Ganga

PART-C : NUMERICAL APTITUDE101. In DABC, a line through A cuts the side BC at D

such that BD : DC = 4 : 5. If the area of DABD = 60cm2 , then the area of DADC is(a) 50 cm2 (b) 60 cm2

(c) 75 cm2 (d) 90 cm2

yoursmahboob.w

ordpress.com

332 SSC CGL SolvedPaper

102. A tangent is drawn to a circle of radius 6cm froma point situated at a distance of 10 cm from thecentre of the circle. The length of the tangent willbe(a) 4 cm (b) 5 cm (c) 8 cm (d) 7 cm

103. A ship after sailing 12km towards south from aparticulari place covered 5 km more towards east.Then the straightway distance of the ship fromthat place is(a) 18 km (b) 15 km (c) 13 km (d) 11km

104. Two poles of height 7 m and 12 m stand on aplane ground. If the distance between their feetis 12 m, the distance between their top will be(a) 13 m (b) 19 m (c) 17 m (d) 15 m

105. The maximum value of sin4q + cos4q is

(a) 1 (b) 2 (c) 3 (d)13

106. Find the value oftan 4° tan 43° tan 47° tan 86°

(a) 1 (b)12

(c) 2 (d)23

107. The sum of four numbers is 48. When 5 and 1 areadded to the first two; and 3 & 7 are subtractedfrom the 3rd & 4th, the numbers will be equal.The numbers are(a) 4, 12, 12, 20 (b) 5, 11, 13, 19(c) 6, 10, 14, 18 (d) 9, 7, 15, 17

108. The least number that should be added to 2055so that the sum is exactly divisible by 27 :(a) 24 (b) 27 (c) 31 (d) 28

109. A and B together can do a piece of work in 6days. If A can alone do the work in 18 days, thenthe number of days required for B to finish thework is(a) 12 (b) 9 (c) 15 (d) 10

110. A pipe can fill a tank in x hours and another canempty it in y hours. They can together fill it in (y > x)(a) x – y (b) y – x

(c)xy

x y- (d)xy

y x-111. A tap can empty a tank in 30 minutes. A second

tap can empty it in 45 minutes. If both the tapsoperate simultaneously, how much time is neededto empty the tank?(a) 18 minutes (b) 14 minutes(c) 15 minutes (d) 30 minutes

112. The perimeter of one face of a cube is 20 cm. Itsvolume will be(a) 100 cm3 (b) 125 cm3

(c) 400 cm3 (d) 625 cm3

113. If the area of a circle is A, radius of the circle is rand circumference of it is C, then

(a) rC = 2A (b)C rA 2

=

(c) AC = 2r4

(d)Ar

= C

114. A square is inscribed in a quarter–circle in such amanner that two of its adjacent vertices lie on thetwo radii at an equal distance from the centre,while the other two vertices lie on the circulararc. If the square has sides of length x, then theradius of the circle is

(a)16x

4p +(b)

2xp

(c)5x2

(d) 2x

115. 10% discount and then 20% discount insuccession is equivalent to total discount of(a) 15% (b) 30% (c) 24% (d) 28%

116. The marked price of a watch was `720. A manbought the same for `550.80 after getting twosuccessive discounts, the first being 10%. Thesecond discount rate is(a) 12% (b) 14% (c) 15% (d) 18%

117. Allowing 20% and 15% successive discounts,the selling price of an article becomes `3,060;then the marked price will be(a) `4,400 (b) `5,000(c) `4,500 (d) `4,000

118. Eighteen years ago, the ratio of A's age to B'sage was 8 : 13. Their present ratio's are 5 : 7. Whatis the present age of A?(a) 70 years (b) 50 years(c) 40 years (d) 60 years

119. 729 ml of a mixture contains milk and water in theratio 7 : 2. How much more water is to be added toget a new mixture containing milk and water inthe ratio 7 : 3 ?(a) 60 ml (b) 71 ml (c) 52 ml (d) 81 ml

120. The average weight of 15 oarsmen in a boat isincreased by 1.6 kg when one of the crew, whoweighs 42 kg is replaced by a new man. Find theweight of the new man (in kg).(a) 65 (b) 66 (c) 43 (d) 67

yoursmahboob.w

ordpress.com

333SSC CGL SolvedPaper

121. What is the Arithmetic mean of the first 'n' naturalnumbers?

(a)n 1

2+

(b)2n (n 1)

2+

(c) 2 (n + 1) (d)n(n 1)

2+

122. A shopkeeper bought 30 kg of rice at the rate of`70 per kg and 20 kg of rice at the rate of `70.75per kg. If he mixed the two brand of rice and soldthe mixture at ̀ 80.50 per kg. his gain is(a) `510 (b) `525 (c) `485 (d) `450

123. The population of a town increases by 5% everyyear. If the present population is 9261, thepopulation 3 years ago was(a) 5700 (b) 6000 (c) 7500 (d) 8000

124. A farmer travelled a distance of 61 km in 9 hrs. Hetravelled partly on foot at the rate of 4 km/hr andpartly on bicycle at the rate of 9 km/hr. Thedistance travelled on foot is(a) 17 km (b) 16 km (c) 15km (d) 14 km

125. Walking at the rate of 4 kmph a man covers certaindistance in 2hrs 45 min. Running at a speed of16.5 kmph the man will cover the same distancein how many minutes ?(a) 35 min. (b) 40 min.(c) 45 min. (d) 50 min.

126. In certain years a sum of money is doubled itself

at 164

% simple interest per annum, then the

required time will be

(a)1122

years (b) 8 years

(c)2103

years (d) 16 years

127. The length of the portion of the straight line 3x +4y = 12 intercepted between the axes is(a) 3 (b) 4 (c) 7 (d) 5

128. The value of1 1 1 1 1

7 6 6 5 5 2 8 7 3 8- + - +

- - - - -is(a) 0 (b) 1 (c) 5 (d) 7

129. If m = –4, n = –2, then the value of m3 – 3m2 + 3m+ 3n + 3n2 + n3 is(a) 124 (b) –124 (c) 126 (d) –126

130. 2x – ky + 7 = 0 and 6x – 12y + 15 = 0 has nosolution for(a) k = – 4 (b) k = 4 (c) k = 1 (d) k = – 1

131. Choose the incorrect relation(s) from the following:(i) 6 2 5 3+ = +(ii) 6 2 5 3+ < +(iii) 6 2 5 3+ > +(a) (i) (b) (ii)(c) (i) and (iii) (d) (ii) and (iii)

132. If x = 332, y = 333, z = 335, then the value of x3 +y3 + z3 – 3xyz is(a) 7000 (b) 8000 (c) 9000 (d) 10000

133. If 12 x 3

2 3+ =

+, then the simplest value of

x is(a) 1 (b) –2 (c) 2 (d) –1

134. If 2 2 2

2 2 2 2 2 2m a m b m cb c c a a b- - -+ ++ + +

= 3, then the

value of m is(a) a2 + b2 (b) a2 + b2 + c2

(c) a2 – b2 – c2 (d) a2 + b2 – c2

135. The measure of an angle whose supplement isthree times as large as its complement, is(a) 30° (b) 45° (c) 60° (d) 75°

136. The sides of a triangle having area 7776 sq. cmare in the ratio 3:4:5. The perimeter of the triangleis(a) 400 cm (b) 412 cm(c) 424 cm (d) 432 cm

137. Two chords of length a unit and b unit of a circlemake angles 60° and 90° at the centre of a circlerespectively, then the correct relation is(a) b = 2 a (b) b = 2a

(c) b = 3a (d) b = 3 a2

138. In a parallelogram PQRS, angle P is four times ofangle Q, then the measure of ÐR is(a) 36° (b) 72° (c) 130° (d) 144°

139. If 2sin sinq + q = 1 then 2 4cos cosq + q isequal to

(a) 1 (b) 2sin

cosqq

(c)2cos

sinqq

(d) None

140. If a clock started at noon, then the angle turnedby hour hand at 3.45 PM is

(a)11042°

(b)1972°

(c)1

1122°

(d)11172°

yoursmahboob.w

ordpress.com

334 SSC CGL SolvedPaper

141. The numerical value of 2

2cos 45sin 60

°°

+ 2

2cos 60sin 45

°°

2

2tan 30cot 45

°°

– 2

2sin 30cot 30

°°

is

(a)34

(b)14

(c)12

(d)114

142. If x cos q – sin q = 1, thenx2 + (1 + x2) sin q equals(a) 1 (b) –1 (c) 0 (d) 2

143. A 10 m long ladder is placed against a wall. It isinclined at an angle of 30° to the ground. Thedistance (in m) of the foot of the ladder from thewall is (Given 3 = 1.732)(a) 7.32 (b) 8.26 (c) 8.66 (d) 8.16

Directions (Qs. 144-147) : Study the following bardiagram carefully and answer the following FourQuestions.The number of the production of electronic items (TVsand LCDs) in a factory during the period from 2009 to2013.

6,0007 ,0008,0009,0009,400

10,00011,00012,00013,000

2009 2010 2011 2012 2013

TVs

LCDs

Years

No.

of p

rodu

ctio

ns

144. The total number of production of electronicitems is maximum in the year(a) 2009 (b) 2010 (c) 2011 (d) 2013

145. The ratio of production of LCDs in the year 2011and 2013 is(a) 3 : 4 (b) 4 : 3 (c) 2 : 3 (d) 1 : 4

146. The difference between averages of productionof TVs and LCDs from 2009 to 2012 is(a) 600 (b) 700 (c) 800 (d) 900

147. The ratio of production of TVs in the years 2009and 2010 is(a) 7 : 6 (b) 6 : 7 (c) 2 : 3 (d) 3 : 2

Directions (Qs. 148-150) : The following pie–chartshows the sources of funds to collected by theNational Highways Authority of India (NHAI) for itsPhase II projects. Study the pie–chart and answerthe following Three Questions:

MarketBorrowing

29,952

SPVS5252

ExternalAssistance

11,486

Toll4910

Annuity6000

148. If the toll is to be collected through an outsourcedagency by allowing a maximum 10% commission,how much amount should be permitted to becollected by the outsourced agency, so that theproject is supported with ` 4,910 crores?(a) ` 6,213 crores (b) ` 5,827.crores(c) ` 5,401crores (d) ` 5,316 crores

149. If NHAI could receive a total of ̀ 9,695 crores asExternal Assistance, by what percent(approximately) should it increase the MarketBorrowing to arrange for the shortage of funds?(a) 4.5% (b) 7.5% (c) 6% (d) 8%

150. The central angle corresponding to MarketBorrowing is(a) 52° (b) 137.8° (c) 187.2° (d) 192.4°

PART-D : ENGLISH COMPREHENSION

DIRECTIONS (Qs. 151-155): In the following FiveQuestions, four alternatives are given for the Idiom/Phrase underlined in the sentence. Choose thealternative which best expresses the meaning of theIdiom\Phrase and mark it in the Answer Sheet.151. At his wit's end, he turned to his old trade.

(a) After careful thinking(b) Confidently(c) Not knowing what to do(d) Overjoyed

152. Then it comes to dancing, he is all–thumbs.(a) an expert (b) a trainer(c) clumsy (d) lazy

yoursmahboob.w

ordpress.com

335SSC CGL SolvedPaper

153. He usually goes to bed very early and rises withthe lark.(a) very late (b) very early(c) after sunrise (d) at midnight

154. Their attempt to get back the stolen necklaceproved to be a wild goose chase.(a) wise decision (b) useless search(c) timely action (d) delayed action

155. The day the new product was launched, peoplemade a beeline to purchase it.(a) rushed (b) were doubtful(c) refused (d) went online

DIRECTIONS (Qs. 156-165): In the following TenQuestions, a sentence/a part of the sentence isunderlined. Below are given alternatives to theunderlined part which may improve the sentence.Choose the correct alternative. In case noimprovement is needed blacken the circle [ ]corresponding to "No improvement".156. It's high time that you go home.

(a) are going (b) went(c) gone (d) No improvement

157. Drinking tea is an English habit.(a) tradition (b) convention(c) custom (d) No improvement

158. He is wanting ina little common sense.(a) lacks (b) is lacking in(c) needs some (d) No improvement

159. The second pigeon flew just at the first pigeonhad flown.(a) one had done(b) one had flown away(c) had done(d) No improvement

160. The old man has acquired experience throughage.(a) developed experience(b) experienced(c) got experience(d) No improvement

161. Water and soil pollutants find their entry into thebody through ingestion of contaminated wateror food.(a) digestion of contaminated(b) injection of contaminated(c) passage of contaminated(d) No improvement

162. He lives far from the station.(a) away from the station(b) a long way from the station(c) off the station(d) N o improvement

163. Mumbai is famous because of its textiles.(a) at (b) in(c) for (d) No improvement

164. They spoke about the weather.(a) said (b) talked(c) expressed (d) No improvement

165. Mr. Mukherjee knows ten languages, isn't it'?(a) doesn't Mr. Mukherjee(b) hasn't he(c) doesn't he(d) No improvement

DIRECTIONS (Qs. 166-172): In the following SevenQuestions, out of the four alternatives, choose theone which can be substituted for the given words/sentences and indicate it by blackening theappropriate circle [ ] in the Answer Sheet166. A recurrent Compulsive urge to steal.

(a) Pneumonia (b) Insomnia(c) Nymphomania (d) Kleptomania

167. Act of injuring another's reputation by anyslanderous communication.(a) Orchestration (b) Aberration(c) Misrepresentation (d) Defamation

168. A story in which animals or objects speak andgive wholesome moral lesson.(a) Fable (b) Parable(c) Allegory (d) Legend

169. Medical Study of skin and its diseases.(a) Dermatology (b) Endocrinology(c) Gynaecology (d) Orthopaedics

170. A process involving too much official formality.(a) Diplomacy (b) Bureaucracy(c) Red–tapism (d) Nepotism

171. A person who enters without any invitation.(a) Burglar (b) Intruder(c) Thief (d) Vandal

172. Not suitable for eating(a) Tasteless (b) Uneatable(c) Inedible (d) Spicy

DIRECTIONS (Qs. 173-174): In the following TwoQuestions, four words are given each question, outof which only one word is correctly spelt. Find thecorrectly spelt word and mark your answer, in theAnswer Sheet.173. (a) Acquariam (b) Aquarium

(c) Acquerium (d) Aquarim174. (a) Facsimile (b) Facsimilee

(c) Fasimile (d) Fasimmile

yoursmahboob.w

ordpress.com

336 SSC CGL SolvedPaper

Directions (Qs. 175-184): In the following TenQuestions, you have two passages with 5 questionsfollowing each passage. Read the passages carefullyand choose the best answer to each question out ofthe four alternatives and mark it by blackening theappropriate circle [ ] in the Answer Sheet.

PASSAGE – I (5 Questions)The instinctive, natural way to express anger is torespond aggressively. Anger is a natural, adaptiveresponse to threats; It inspires powerful, oftenaggressive, feelings and behaviours, which allow usto fight and to defend ourselves when we are attacked.On the other hand, we can't physically lash out atevery person or object that irritates or annoys us;laws, social norms and common sense place limits onhow far our anger can take us. People use a variety ofboth conscious and unconscious processes to dealwith there angry feelings. The three main approachesare expressing, suppressing and calming. Expressingyour angry feelings in an assertive – not aggressivemanner is the healthiest way to express anger. Beingassertive doesn't ' mean being 'pushy or demanding;It means being respectful of yourself and others. Angercan be suppressed and then converted or redirected.This happens when you hold in your anger, stopthinking about it and focus on something positive.175. How does a person naturally express anger?

(a) By inspiring powerful feelings(b) By responding aggressively(c) By defending oneself(d) By adapting strong behaviour.

176. Which one of the following places limits on howfar we can take our anger?(a) Behaviour (b) Feelings(c) Instinct (d) Law

177. According to the author, how should people dealwith their anger?(a) Express it assertively(b) Express it aggressively(c) Expressing consciously(d) Expressing unconsciously

178. What does the author mean by being assertive?(a) Being pushy (b) Being demanding(c) Being respectful (d) Being calm

179. How, accordmg to the author, can one suppressanger ?(a) By holding one's anger.(b) By thinking about one's anger(c) By converting anger.(d) By redirecting anger.

PASSAGE –II (5 Questions)The crowd surged forward through the narrow streetsof Paris. There was a clatter of shutters being closedhastily by trembling hands the citizens of Paris knewthat once the fury of the people was excited there wasno telling what they might do. They came to an oldhouse which had a workshop on the ground floor. Ahead popped out of the door to see what it was allabout "Get him! Get Thimonier! Smash his devilishmachines!" yelled the crowd.They found the workshop without its owner. M.Thimonier had escaped by the back door. Now thefury of the demonstrators turned against the machinesthat were standing in the shop, ready to be deliveredto buyers. They were systematically broken up anddestroyed – dozens of them. Only when the last wheeland spindle had "been trampled under foot did theinfuriated crowd recover their senses."That is the end of M'Sieur Thimonier and his sewingmachines," they said to one another and went homesatisfied. Perhaps now they would find work, for theywere all unemployed tailors and seamstresses whobelieved that their livelihood was threatened by thatnew invention.180. The passage throws light on

(a) why inventions should be avoided.(b) how a well meant invention can be

misunderstood(c) what mischief an inventor can do to ordinary

people.(d) how dangerous an invention can be.

181. The crowd was protesting against(a) the closings of workshops.(b) the misdoings of Thimonier.(c) the newly invented sewing machine(d) Thimonier for keeping the invention a secret

182. The aim of the crowd was to(a) kill Thimonier(b) drive Thimonier away(c) humiliate Thimonier(d) destroy the sewing machines

183. The people thought that(a) their lives were in danger.(b) Thimonier was mad.(c) the sewing machine was dangerous.(d) they would be deprived of their livelihood.

yoursmahboob.w

ordpress.com

337SSC CGL SolvedPaper

184. Shutters were being closed because theshopkeepers(a) wanted to attack the crowd.(b) wanted to protect Thimonier.(c) feared their shops would be destroyed.(d) wanted to support the crowd.

Directions (Qs. 185-189) : In the following FiveQuestions, some parts of the sentences have errorsand some are correct. Find out which part of asentence has an error and blacken the circle [ ]corresponding to the appropriate correct option. Ifa sentence is free from error, blacken the circle [ ]corresponding to "No Error" in the Answer Sheet.

185. I shall look forward to being with you sometimenext month.(a) I shall look forward(b) to being with you(c) sometime next month(d) No error

186. I really do regret not to learn to play the violinwhen I had so many opportunities to learn thepractice in school.(a) I really do regret not to learn to play the

violin.(b) when I had so many opportunities(c) to learn and practice in school.(d) No error

187. The government granted relief payments of`5,000 each to those who injured in the fireaccident.(a) The government granted(b) relief payments of ̀ 5,000 each(c) to those who injured in the fire accident.(d) No error

188. By May next year I have been working in thiscollege for twenty years.(a) By May next year(b) I have been working in this college(c) for twenty years.(d) No error

189. Everyone of us should realize that any act ofnegligence will cause a great harm to ourcountry's security.(a) Everyone of us should realize that(b) any act of negligence will cause(c) a great harm to our country's security(d) No error

DIRECTIONS (Qs. 190-194): In the following FiveQuestions, sentences given with blanks to be filled inwith an appropriate word(s). Four alternatives aresuggested for each questton. Choose the correctalternative out of the four and indicate it byblackening the appropriate circle [ ] in the AnswerSheet.190. You must ______ your career with all

seriousness.(a) follow (b) complete(c) direct (d) pursue

191. Making pies and cakes _____ Mrs. Reddy'sspeciality.(a) are (b) were (c) is (d) has

192. The deserted garden was infested ________weeds.(a) for (b) into (c) on (d) with

193. He went there _______ .(a) walking on foot (b) by foot(c) with foot (d) on foot

194. After your long illness I am happy to see you.(a) up and about (b) by and by(c) up and down (d) up and round

DIRECTIONS (Qs. 195-197): In the following ThreeQuestions, out of the four alternatives, choose theone which best expresses the meaning of the givenword and mark it in the Answer Sheet.195. Devout

(a) Solemn (b) Loyal(c) Dedicated (d) Pious

196. Predilection(a) Favour (b) Whim(c) Prejudice (d) Preference

197 Effigy(a) Dummy (b) Imagery(c) Reflection (d) Organ

DIRECTIONS (Qs. 198-200): In the following threeQuestions, choose the word opposite in meaning tothe given word and mark it in the answer sheet.198. Tremulous

(a) Steady (b) Obese(c) Young (d) Healthy

199. Fake(a) Fanciful (b) Real(c) Ideal (d) Wrong

200. Disconsolate(a) Joyous (b) Thankful(c) Unprejudiced (d) Prominent

yoursmahboob.w

ordpress.com

338 SSC CGL SolvedPaper

1. (a) c c b a/b b c a/a a b c/c c b a/b2. (a) a b c d/b a c d/b c a d/b c d a/a b c d.3. (c) 4 6

+2

10

+4

16

+6 +8

24 34

+10

4. (c) 3 5

+2

9

+4

17

+8 +16

33

5. (b) Let ‘D’ be the distance between A and Band T be the time taken by themThen,Distance = Speed × Time

D = 40 × 15T60

æ ö+ç ÷è ø...(1)

D = 30 × 24T16

æ ö+ç ÷è ø...(2)

Equating (1) and (2)

40 1T4

æ ö+ç ÷è ø = 30 2T

5æ ö+ç ÷è ø

43

= 4T = 1 = 3T + 65

T = 6 15-

T = 15

hourPutting ‘T’ value in equation (1), we get

D = 40 × 1 15 4æ ö´ç ÷è ø

= 40 9

20´

= 18 km.

Hence, the distance between the twostations is 18 km.

6. (c) Original Position:

20th from the left 30th from the right

Kiran Manoj

Position after Interchange:

35th from the right

Manoj

Total Number of Students = 35 + 20 – 1= 55 – 1 = 54

7. (d) Standing Arrangement:

G F E D B C AG is standing on the extreme left.

8. (d) MERCY cannot be formed from the givenword CUMBERSOME, as there is no ‘Y’ inthe word.

9. (a) Only ‘S’ can be prefixed to the given words.New words are:STILL, STABLE, SPILE, STAB, SPRING

10. (c) Meaningful word is:U N S C R A M B L E3 9 4 2 8 10 5 1 7 6

11. (c) Coded word: Z B Y X M N Q B¯ ¯ ¯ ¯ ¯ ¯ ¯ ¯

Key : s t r a i g h t12. (d) M O U S E

¯ +3 ¯ +3 ¯ +0 ¯ –2 ¯ –2Coded as: P R U Q CSimilarly,

S H I F T¯ +3 ¯ +3 ¯ +0 ¯ –2 ¯ –2

Coded as: V K I D R

13. (a) 2 5 3 Þ books are old

5 4 6 Þ man is old

3 7 8 Þ buy good booksCodes are :5 Þ old 4 Þ man or is8 Þ buy or good3 Þ books 6 Þ man or is2 Þ are 7 Þ buy or good2 stands for “are” in that code.

14. (b) 46 × 6 ̧ 4 + 5 – 3 = 46 × 1.5 + 5 – 3 = 69 + 5 –3 = 71

15. (b) 18 × 6 ̧ 4 + 2 – 3 = 18 × 1.5 + 2 – 3 = 27 + 2 –3 = 26

16. (c) 8 × 2 + 1 = 178 × 17 + 1 = 1378 × 137 + 1 = 1097

17. (c) 36 – 14 + 12 + 7 = 354 – 9 + 11 + 16 = 18

18. (c) 4 3 4 39 13 10 14 11+ - + -¾¾¾® ¾¾¾® ¾¾¾® ¾¾¾®4 3 4 311 15 12 16 13+ - + -¾¾¾® ¾¾¾® ¾¾¾® ¾¾¾®4 3 4 313 17 14 18 15+ - + -¾¾¾® ¾¾¾® ¾¾¾® ¾¾¾®

Hence, 15 is the correct answer.19. (c) 3 × 4 × 2 = 24

2 × –2 × –1 = 46 × 0 × 5 = 0

HINTS & SOLUTIONS

yoursmahboob.w

ordpress.com

339SSC CGL SolvedPaper

20. (b) 7 × 8 × 2 = 1125 × 4 × 8 = 1603 × 9 × ? = 16227 × ? = 162

? = 16227

= 621. (c)

D C

A B

500

mtr. 500 m

tr.

600 mtr.

600 mtr.Pinky’s

starting pointHence, Pinky is 0 metres from the startingpoint.

22. (b) BC

AD

4 km

2 km

4 km

Sunita’sstarting point

Hence, Sunita rode 4 km northwards initially.23. (a) Parental involvement and participation is

necessary for promoting the growth ofchildren.

24. (c) Conclusion I is not based upon the givenstatement. Conclusion II does not provethe given statement so, neither I nor IIfollows.

25. (c) Hematology is the branch of medicineconcerned with the study and preventionof diseases related to the blood.Similarly, phycology is the scientific studyof algae.

26. (c) A group of Lions is called a pride.A group of Cats is called a clowder.

27. (a) M A N W A N¯ +3 ¯ +3 ¯ +3 Similarly, ¯ +3 ¯ +3 ¯ +3P D Q Z D Q

28. (b) A E F J¯ +10 ¯ +10 ¯ +10 ¯ 10K O P TSimilarly,G K L P­ –10 ­ –10 ­ –10 ­ –10Q U V Z

29. (a) 25 = 3235 = 243

30. (c) D × H M × Q¯ ¯ Similarly, ¯ ¯4 × 8 13 × 17Respective place value of letters in Englishalphabet.Hence, option (c) in the correct answer.

31. (d) Except (d), all others belong to the period ofbefore sunset.

32. (b) Except (b), all others are principles ofsociety.

33. (a) Option (b): E V H S

– 3

+ 3Option (c):

H S K P

– 3

+ 3Option (d):

K Q N N

– 3

+ 3Option (a):

D W F U

– 2

+ 2Hence, option (a) is the group of odd letters.

34. (d) Option (a): 1 3 1C B E F- + +¾¾¾® ¾¾¾® ¾¾¾®Option (b): 1 3 1E D G H- + +¾¾¾® ¾¾¾® ¾¾¾®

Option (c): 1 3 1I H K L- + +¾¾¾® ¾¾¾® ¾¾¾®Option (d): 1 1 1G F H J- + +¾¾¾® ¾¾¾® ¾¾¾®Hence, option (d) is the odd group of letters.

35. (d) 4 + 0 + 2 + 5 = 116 + 0 + 2 + 3 = 117 + 2 + 0 + 2 = 115 + 0 + 6 + 1 = 12Hence, (d) is the odd one out.

36. (a) 64 : 48

(8 8) (8 6)¯ ¯´ ´80 : 60

(10 8) (10 6)¯ ¯´ ´

104 : 78

(13 8) (13 6)¯ ¯´ ´

Hence, option (a) is odd one out

yoursmahboob.w

ordpress.com

340 SSC CGL SolvedPaper

37. (d) Lily, Daisy, Datura all have outer part whiteand inner part yellow.Jasmine also has outer part white and innerpart yellow.

38. (c) Meaningful Order:Mosquito ® Cat ® Tiger ® Elephant ® Whale (3) (2) (4) (1) (5)

39. (c)

B F I

D E LGA

C H K

J

DABD, DADC, DBDE, DDEC,DABC, DEBC, DACE, DABE,DBEF, DFIG, DCEH, DHGK,DFGE, DEGH, DGIL, DILJ,DGLK, DLJK, DGIJ, DGKJ,DGIK, DIJK, DBCH, DIHK,DBFC, DFIK, DFCK, DHBITotal Triangles = 28

40. (b) The given figure can be numbered like this:123 4

56

In this figure:1 lies opposite 3; 2 lies opposite 5;4 lies opposite 6.When the sheet of question figure is foldedto form a cube, then the face bearing a dotlies opposite to one of the shaded faces.Therefore, figure (2) which has both theshaded faces adjacent to the face bearingthe dot, cannot be formed.Hence, the cubes shown in figures (1), (3)and (4) can be formed.

41. (c) There are 17 number of circles in the givenfigure.

42. (d)

Landanimals

Animals

Sea Animals

43. (d)2 3 4 5 61

Married

Doctors

Persons Numbers

Number 1 indicates doctors who are notmarried

44. (a) P and K is nothing but R.45. (a) 46. (c)47. (b)

48. (c)

49. (a)M

N Mirror image50. (a) P E A R L

¯ ¯ ¯ ¯ ¯00 55 22 11 96

51. (a) Endymion is a poem,written by John Keats.It begins with the line "A thing of beauty isa joy forever". The poem tells about hownature and its wonder mesmerize us and takeaway all the sorrow that surrounds us fromtime to time.

52. (b) The birthday of Sarvepalli Radhakrishnanis celebrated as Teacher's Day. The daycommemorates the birthday of Dr SarvepalliRadhakhrishnan, a philosopher and ateacher par excellence, and his contributiontowards Indian education system. As atribute to this great teacher, his birthday isobserved as Teacher's Day across India on5th September.

53. (c) The Arjuna Awards are given by theMinistry of Youth Affairs and Sports,government of India to recognizeoutstanding achievement in Nationalsports.

54. (c) "Place of the Thunderbolt" is associatedwith Darjeeling(west Bengal). The wordDarjeeling is a combination of two words'dorje,' which means 'thunderbolt' and 'ling',which means 'place'. Hence the wordDarjeeling means 'the Land of Thunderbolt'.

55. (c) The Ship building yard Mazagon DockLimited (MDL) is located inMumbai(Maharashtra).It is India's primeshipyard. It manufactures warships andsubmarines for the Indian Navy and offshoreplatforms and associated support vesselsfor offshore oil drilling.

yoursmahboob.w

ordpress.com

341SSC CGL SolvedPaper

56. (d) Electric current is measured using a devicecalled an ammeter.

57. (d) Aga Khan Cup is related to Football. WhenPrince Aga Khan IV of Iran visit in 1958,expressed his interest to start a majorinternational football club tournament in theregion. The football authorities of EastPakistan in collaboration with AsianFootball Confederation (AFC) decided tostart this event.

58. (a) The first summit was held in Dhaka,Bangladesh on 7-8 December 1985 and wasattended by president of Bangladesh,Maldives, Pakistan and Sri Lanka, the kingsof Bhutan and Nepal, and the prime ministerof India.

59. (c) The Bretton Woods Institutions are theWorld Bank, and theInternational MonetaryFund (IMF). They were set up at a meetingof 43 countries in Bretton Woods, NewHampshire, USA in July 1944.

60. (c) The price and output under monopoly aredetermined by equality between marginalcost and marginal revenue and not by theintersection of demand and supply curves.

61. (d) Macroeconomics is a branch of economicsdealing with the performance, structure,behavior, and decision-making of aneconomy as a whole, rather than individualmarkets. Macroeconomists develop modelsthat explain the relationship between suchfactors as national income, output,consumption, unemployment, inflation etc.

62. (d) Internal economies arise within the firmbecause of the expansion of the size of aparticular firm. They are called theeconomies of scale.

63. (c) In a free market economy there is a freedomof choice for the consumers to buy goodsand services which suit their tastes andpreferences. This is generally called theprinciple of consumer sovereignty. Thismeans in a market economy the consumersare just like a king or sovereign who dictatewhat goods and services and whatquantities of them are produced.

64. (c) Net national product (NNP) refers to grossnational product (GNP), i.e. the total marketvalue of all final goods andservicesproduced by the factors ofproduction of a country or other polityduring a given time period, minusdepreciation.

NNP = GNP – Depreciation

65. (b) Zilla Parishad is the apex body of thePR(Panchayati Raj) system located at thedistrict level. Chairpersons/Presidents ofPanchayat Samitis come within itsjurisdiction.

66. (c) A parliamentary system is a system ofdemocratic governance of a state in whichthe executive branch derives its democraticlegitimacy from, and is held accountable tothe legislature(parliament).

67. (d) Public opinion is important in a democracybecause the people are the ultimate sourceof political power. The legislature in ademocratic country can influence publicopinion byFocusing attention on Publicissues.

68. (a) Article 56 of the Indian Constitution saysthat the President shall hold office for a termof five years from the date he takes up hispost. He may resign from his office bywriting his resignation to the Vice-Presidentof India.

69. (c) Nagaland is a state in Northeast India. Thestate capital is Kohima. Nagaland becamethe 16th state of India from 1 December 1963.

70. (a) Mihira Bhoja was a ruler of the GurjaraPratihara dynasty of India. Bhoja's empireextended to Narmada River in the South,Sutlej River in the northwest, and up toBengal in the east.

71. (d) Bose appeared at the 1939 Congressmeeting and was elected president overGandhi's preferred candidate PattabhiSitaramayya.

72. (c) The Jallianwala Bagh Massacre happenedin Amritsar, in 1919. It is named after theJallianwala Bagh (Garden) atAmritsar. OnApril 13, 1919, British, Indian Army soldiersstarted shooting an unarmed gathering ofmen, women and children.

73. (b) Bahlol Lodi was the founder of the Lodidynasty.He ruled for long thirty-nine years(1451-89). He was the governor of Lahoreand Sirhind during the rule of MuhammadShah of Sayyid dynasty.

74. (c) Sri Chaitanya Mahaprabhu arrived in theempire at the time of Emperor Prataparudra(Gajapatis)and stayed for 18 long years at Puri.

75. (c) The instance heat that prevails in the IndianSub continent causes a low pressure regionover the northern plains. It is intense enoughto attract the moisture bearing winds fromthe Indian Ocean .Thus the south east tradewinds from the southern hemisphere areattracted towards India.

yoursmahboob.w

ordpress.com

342 SSC CGL SolvedPaper

76. (d) A river profile is a curve which shows theslop of a river from source to mouth.A streamflowing over irregular terrain may havewaterfalls, rapids and lakes along itscourse.Though the stream will wear awaythe irregularities to leave a smoothlycurving profile called a graded profile.

77. (c) Karst is a landscape formed from thedissolution of soluble rocks includinglimestone, dolomite and gypsum. It ischaracterized by sinkholes, caves, andunderground drainage systems.

78. (d) Kerala is famous for the cultivation ofcoconut, tea, coffee, cashew and spices.

79. (a) The Trans-Siberian Railway is the longestrailway line in the world. It has a length of9,289 km which connects Moscow toVladivostok.

80. (d) Photoperiodism is the physiological reactionof organisms to the length of day or night.It occurs in plants and animals. It affectsFlowering,Vegetative growth and fruiting inplants.

81. (d) The correct match is as follows:Ascorbic acid- Vitamin CChlorophyll- Photosynthetic pigmentCarotenoid- QuencherSuperoxide dismutase- Enzyme

82. (b) Allantois is a part of a developingamniote'sconceptus which primarily involved innutrition and excretion, and is webbedwithblood vessels.

83. (d) Haliotis belongs to Mollusca.84. (a) Although most DNA is packaged in

chromosomes within the nucleus,mitochondria also have a small amount oftheir own DNA. This genetic material isknown as mitochondrial DNA or mtDNA

85. (c) First class proteins contain all the essentialamino acids in sufficient amounts.Animalproteins are obtained from milk, egg, fish,meat etc. are first classproteins. These arealso called adequate proteins.

86. (b) Needle exerts more pressure than nail onthe balloon.

87. (c) The speed of sound is greater in moist airthan in dry air.

88. (b) X rays are frequently used to check thedefects in Diamonds and other preciousstones.

89. (a) When ice is kept on saw dust then it doesnot melt quickly as it's an insulator of heatand air does not circulate in goodinsulators. So, it prevents ice from meltingquickly .

90. (c) A parity bit, or check bit is a bit added tothe end of a string of binary code thatindicates whether the number of bits in thestring with the value one is even or odd.Parity bits are used as the simplest form oferror detecting code.

91. (c) Memory management is the process ofcontrolling and coordinating computermemory, assigning portions called blocksto various running programs to optimizeoverall system performance.It involvescomponents that physically store data, suchas RAM (random access memory) chips,memory caches, and flash-based SSDs(solid-state drives).

92. (a) When a drop of Glycerol is added to crushedKMnO4 spread on a paper there is a violentexplosion.

93. (c) A bleaching agent is a material that lightensor whitens a substrate through chemicalreaction. The most common bleachingagents generally fall into two categories:chlorine and its related compounds (suchas sodium hypochlorite) and the peroxygenbleaching agents, such as hydrogenperoxide and sodium perborate.

94. (a) Alpha particles are the least penetrating asthey are the most densely ionizing. Thepenetrating power of nuclear radiationdepends upon the ionizing power of theradiation. The more localised the ionizationthe less penetrating power it will possess.

95. (a) Hydrogen- peroxide is an effective sterilizingagent. Water results when it readily losesactive oxygen.

96. (c) The maximum fixation of solar energy isdone by green plants.The energy is storedin the plants as carbohydrates for theirmetabolic activities as growth,respirationetc.

97. (a) The term 'brown air ' is used forphotochemical smog. The brown color iscause by the presence of a various NitrousOxides.

98. (c) One of the Major objectives andadvantages of rainwater harvesting is toreduce run off loss.

99. (c) Peroxyacetyl nitrate is a secondary pollutantpresent in photochemical smog. It isthermally unstable and decomposes intoperoxyethanoyl radicals and nitrogendioxide gas.

yoursmahboob.w

ordpress.com

343SSC CGL SolvedPaper

100. (b) Brahmaputra River is one of the largest riversin the world. The origin of Brahmaputra Riveris in southwestern Tibet as the YarlungRiver.

101. (c)

4x 5xD B C

A

Area of DABD = 1 BD AD2´ ´

60 =1 4x AD2´ ´ ... (1)

Area of DADC = 1 DC AD2´ ´

Area of DADC = 1

5x AD2´ ´ ... (2)

Dividing eqn. (1) and (2)

1 4x AD60 21Area of ADC 5x AD2

´ ´=

D ´ ´

Þ Area of D ADC = 5x 60 75cm

4x´ =

102. (c)

BO

A

6 cm

10 cm

AB2 + OA2 = OB2

AB2 = (10)2 – (6)2

= 100 – 36 = 64AB = 8cm

103. (c) Given, AB = 12 km, BC = 5 kmA

B C

12 km

5 km

Straight way distance of ship

AC = 2 2AB BC+ = 2 212 5+ = 169 =13 km

104. (a) E

BA

CD

7 m 12 m

12 m

12 m

5 m

Let DE be the distance between the polesDistance between poles top

DE = 2 2DC CE+ = 2 212 5+

= 144 25+ = 13 m105. (a) The maximum value of sin4q + cos4q is 1.106. (a) tan 4° tan 43° tan47° tan 86°

tan (90° – 86°) × tan (90° – 47°) × tan 47° ×tan 86°Þ cot 86° × cot 47° × tan 47° × tan 86°Þ 1

107. (c) Let four numbers are a, b, c, d, thena + b + c + d = 48 ...(i)

and a + 5 = b + 1 ...(ii)or, a = b – 4 ...(iii)and c – 3 = d – 7 ...(iv)

c = d – 4 ...(v)Substituting equation (iii) and (v) inequation (i) we get

b – 4 + b + d – 4 + d = 48b + d = 28 ...(vi)

But we know,b + 1 = d – 7

\ b = d – 8Substituting in equation (vi) we get

d – 8 ¹ d = 28 d = 18

Solving this way we get a = 6, b = 10, c = 14and d = 18

108. (a) Number has to be less than 27. Let thenumber be x. On Dividing 2055 by 27, weget remainder as 3Now, 3 + x = 27\ x = 24

yoursmahboob.w

ordpress.com

344 SSC CGL SolvedPaper

109. (b) A and B can complete work in 6 daysA can complete in 18 daysLet B can complete in x days\ One day work be equal to1 1 16 18 x

= +

1 x 186 18x

+=

3x = x + 18x = 9 days

110. (d) Work done by A in one hour = 1x

Work done by B in one hour = 1y

Both A & B together in work one hour

= 1 1x y- =

y xxy-

Both A & B fill tank in xyhours.

y x-

111. (a) Work done by 1st tap in one minute = 130

Work done by 2nd tap in one minute = 145

Both tap one minute work = 1 130 45

+

= 45 301350

+

= 75 1

1350 18=

Both tap will empty the tank in 18 minutes.112. (b) Perimeter of one face, 4a = 20 cm

Therefore, side of cube, a = 5 cmVolume of cube= a3 = 53 = 125 cm3

113. (a) Area of circle, A = pr2 ...(i)Circumference of circle, C = 2pr ...(ii)Multiplying eq. (i) by 2, we get, 2A = 2pr2

Multiplying eq. (ii), by ‘r’, we get rC = 2pr2

\ rC = 2A114. (d)

A x O x B

xx

CD

2x 2x

OA = AD = x\ 2 2OD x x 2 x= + =So (d) option is correct.

115. (d) Successive discount can be given by =xyx y

100+ +

= ( 10 20)10 20

100- ´-- - +

= –30 + 2 = 28%Hence, the successive dicount in equal to 28%

116. (c) Let the second discount be x%. Then(100 – x)% of 90% of 720 = 550.80

100 x 90 55080720100 100 100-Þ ´ ´ =

55080 100(100 x) 85

90 720´

Þ - = =´

x 100 85 15%Þ = - =117. (c) S.P. of an article = 20% and 15% successive

discount× marked price of an article

3060 = 80 85

100 100´ ´marked price of an article

\ Marked of an article3060 100 100 4500

80 85´ ´= =´

118. (b) Let the A’s age and B’s age was 8x and 13xAccording to question8x 18 5

13x 18 7+ =+

Þ 56x + 18 ́ 7 = 65x + 18 ́ 5Þ 65x – 56x = 18 ´ 7 – 18 ́ 5Þ 9x = 18 ´ 2Þ x = 4Hence, the present age of A= 8 ´ 4 + 18 = 50 yrs.

119. (d) Quantity of milk = 7 7299´ = 567 ml

Quantity of water = 2 7299´ = 162 ml

Let ‘x’ be the quantity that should be addedto make the ratio 7 : 3According the question

567 7162 x 3

=+

Þ 1701 = 1134 + 7xÞ 7x = 1701 – 1134Þ x = 81 ml

120. (b) Let the average weight of 15 Oarsmen at thestart = x kgLet the new man’s weight = y kgAccording to question

15x – 42 = 15 (x + 1.6) – y15x – 42 = 15x + 24 – yy = 24 + 42 = 66 kg

yoursmahboob.w

ordpress.com

345SSC CGL SolvedPaper

121. (a) Arithmetic mean of first ‘n’ natural number

= Sum of 'n' natural numberNumber of observations

= (n)(n 1) n 1

2 n 2+ +=´

122. (a) Solving by alligation70 70.75

x

30 20

x = 70 30 70.75 20 2100 1415 70.3

50 50´ + ´ += =

Hence, cost price = 70.3 × 50 = ̀ 3515Selling price = 80.5 × 50 = ̀ 4025Required gain = 4025 – 3515 = 510

123. (d) Population 3 yrs. ago = 39261

51100

æ ö+ç ÷è ø

= 9261 20 20 20 8000

21 21 21´ ´ ´ =´ ´

124. (b) Let the distance travelled as foot be x km.Then, distance travelled by bicycle = (61 –x) km

So, x 61 x 94 9

-+ =

9x + 4 (61 –x) = 9 × 369x – 4x = 324 – 244

5x = 80x = 16 km

125. (b) When distance is contant, then speed isinversely proportional

S1 : S2 = T2 : T14 : 16.5 = T2 : 165

or 2T416.5 165

=

T2 = 165 4 40 min

16.5´

=

126. (d) Let x be the principal amount‘y’ be the time to double the money.Then interest will also be ‘x’.

\ x = x 25 y4 100´ ´´

400 = 25yy = 16 years

127. (d) Intercept can represent in the form of

1x ya b

+ =

To get x and y intercept, we have3x + 4y = 12

14 3x y+ =

So, triplets of 3, 4 and 5.Hence, 5 is the length of portion of straightline.

128. (c)1 1 1 1 1

7 6 6 5 5 2 8 7 3 8- + - +

- - - - -

Þ 1 7 6 1 6 5 1

7 6 7 6 6 5 6 5 5 2+ +´ - ´ +

- + - + -5 2 1 8 7 1 3 85 2 8 7 8 7 3 8 3 8

+ + +´ - ´ + ´+ - + - +

Þ ( )6 57 6 5 2

7 6 6 5 5 4

++ +- +- - -

( )8 7

8 7

+-

-3 89 8++-

Þ 7 6 6 5 5 2 8 7 3 8+ - - + + - - + + Þ 5129. (d) m3 – 3m2 + 3m + 3n + 3n2 + n3

Þ (– 4)3 – 3 (– 4)2 + 3 (– 4) + 3 (– 2) + 3 (– 2)2+ (– 2)3Þ – 64 – 48 – 12 – 6 + 12 – 8Þ – 126

130. (b) For no solution, a = b26 12

k-=-

k = 12 2

6- ´- = 4

131. (c) By squaring the given relations, we get (i)and (iii) are incorrect relations from thegiven statement.

132. (a) Using the formula,

( )3 3 3 132

x y z xyz x y z+ + - = ´ + +

( ) ( ) ( )2 2 2x y y z z xé ù- + - + -ë û

Þ ( )1 332 333 3352´ + + ´

( ) ( ) ( )2 2 2332 333 333 335 335 332é ù- + - + -ë û

Þ ( ) ( ) ( )2 2 21 1000 1 2 32

é ù´ ´ - + - + -ë û

Þ [ ]1 1000 1 4 92´ ´ + +

Þ 1 1000 14 70002´ ´ =

yoursmahboob.w

ordpress.com

346 SSC CGL SolvedPaper

133. (d) 12 32 3

x+ =+

( )( )2 3 2 3 1x+ + =

4 2 3 2 3 3 1x x+ + + =

2 3 3 1 4 2 3x x+ = - - 3 2 3= - -

( ) ( )2 3 3 3 2 3x + = - +

x = – 1

134. (b)2 2 2

2 2 2 2 2 2 3- - -+ + =+ + +

m a m b m cb c c a a b

2 2 2

2 2 2 2 2 21 1 1 0- - -- + - + - =+ + +

m a m b m cb c c a a b

2 2 2 2 2 2

2 2 2 2- - - - - -+

+ +m a b c m a b c

b c c a

2 2 2

2 2 0- - -+ =+

m c a ba b

2 2 22 2 2 2 2 2

1 1 1( ) é ù- - - + +ê ú+ + +ë ûm a b c

b c c a a b= 0

2 2 2 0- - - =m a b c2 2 2= + +m a b c

135. (b) Let ‘x’ be the measure of an angle.According to question3x + x = 180°4x = 180°

180 454

x = = °

136. (d) Let sides of D be 3x, 4x, 5x

2a b cs + += = 6x

Area of ( )( )( )s s a s b s cD = - - -

7776 6 .3 .2 .x x x x=7776 = 6x2

\ x = 36Sides of D will be 108, 144 and 180Perimeter of D is 108 +144 + 180 = 432 cm

137. (a)138. (d) P = 4Q

P + Q = 180°4Q + Q = 180°

180Q 365

= = °

So, R = 180° – 36° = 144°139. (a) Given sinq + sin2q = 1

1 – sin2q = sinqThen, cos2q + cos4q Þ cos2q + (cos2q)2

Þ (1– sin2q) + (1– sin2q)2

Þ sinq + sin2q = 1140. (c) Clock started at 12 pm

Angle turned by hour hand in one hour =36012

= 30°Angle turned by hour hand in one minute

30 1º60 2

= =

Angle turned by hour hand in 3 hour 45minutes

= 3 × 30° + 45 ×12

=1º1122

141. (a)2 2 2 2

2 2 2 2

cos 45 cos 60 tan 30º sin 30ºsin 60 sin 45º cot 45º cot 30º

° °+ - -°

Þ ( ) ( )

22 2 2

2 2 2 2

11 1 12 232

113 322

æ öæ ö æ ö æ öç ÷ç ÷ ç ÷ ç ÷è ø è ø è ø è ø+ - -

æ ö æ öç ÷ ç ÷ç ÷ è øè ø

1 4 1 2 1 1 12 3 4 1 3 4 3= ´ + ´ - - ´

Þ 2 1 1 13 2 3 12+ - -

Þ 8 6 4 1 9 3

12 12 4+ - - = =

142. (a)

143. (c)

A

10m

C

B30°

cos 30° = ABAC

yoursmahboob.w

ordpress.com

347SSC CGL SolvedPaper

AB = AC × cos 30°

= 310×

2= 8.66 m

144. (c) Production of electronic items is highest in2011i.e. 13,000 + 9,000 = 22,000

145. (a) Production of LCD in 2011 = 9,000Production of LCD in 2013 = 12,000

Ratio = 9,000 3

12,000 4=

146. (d) Total production of TV from 2009 to 2012 =39,000Average of TV production = 9,750Total production of LCD = 35,400Average of TV production = 8,850Their difference = 9,750 – 8,850 = 900

147. (c) Ratio of production of TV = 6,0009,000 = 2 : 3

148. (c) Amount of product = 4910 croresAmount to be collectd= 4910 crores + 10% of 4910 crores= 4910 + 491 = 5401 crores

149. (c) External assistance received = 9695 croresTotal required = 11486 croresShortage of external assistance = 11486 – 9695

= 1791 croresPrevious Market borrwings = 29952 crores% increase in market borrowing

= 1791 10029952

´ = 6%150. (c) Total of sources of funds

= 11486 + 5252 + 4910 + 6000 + 29952= 57600 croresCentral angle corresponding to marketborrowing

= 29952

36057600

´ = 187.2151. (c) At one's wit's end means at the limits of

one's mental resources. Example: I'm at mywit's end with choosing the subject in mygraduation. I cannot figure it out. Anju coulddo no more. She was at her wit's end.

152. (c) All thumbs means very awkward andclumsy, especially with one's hands. PoorSam can't play the piano at all. He's allthumbs. Pamela is all thumbs when it comesto gardening.

153. (b) To rise with the lark entails to rise very earlyin the morning.

154. (b) Wild-goose chase means a worthless huntor chase; a futile pursuit. I wasted allevening on a wild-goose chase. Raj wasangry because he was sent out on a wild-goose chase.

155. (a) Make a beeline for someone or somethingmeans heading straight toward someone orsomething. John came into the kitchen andmade a beeline for the cookies. After thegame, we all made a beeline for Dhoni, whojust played the winning run.

156. (b) If you say it's high time that somethinghappened, you mean that it should alreadyhave been done. His parents decided it washigh time he started behaving himself. (often+ that) It's high time that workers were givenbetter pay and conditions.

157. (d) Habit is a particular practice, custom, orusage: the habit of shaking hands.

158. (a) 'He lacks common sense' is a meaningfulsentence. Others are not.

159. (a) A single person or thing; a unit: This is theone I like best. Of his many movies, the bestones are the last two.

160. (c) Experience may be gotten, acquired orgathered; hence, option c is correct.

161. (d) Ingestion is the process of taking food intothe body through the mouth (as by eating);hence, option d.

162. (d) The adverb far showing distance indicatesat, to, or from a great distance in space or time:Ex: How far is it from Australia to NewZealand? He doesn't live far from here.

163. (c) If someone or something is famous, a lot ofpeople know their name or have heard aboutthem. Ex: The town of Moradabad is famousfor brassware. Alexander Fleming, the Scotfamous for discovering penicillin.

164. (b) If you talk, you are having a conversationwith other people. Ex: They were all talkingand laughing together. We were talkingabout you just last night.

165. (c) A tag question is one where a statement ismade, but the speaker wants a responsefrom the listener. The given sentence is insimple present with third person, soresponse will be 'doesn't he?'

166. (d) A compulsion to steal having no relation toneed or the monetary value of the object.Pneumonia is a disease; insomnia is aninability to sleep; chronic sleeplessnesswhile nymphomania is an abnormallyintense sexual desire in women.

yoursmahboob.w

ordpress.com

348 SSC CGL SolvedPaper

167. (d) A false accusation of an offence or amalicious misrepresentation of someone'swords or actions.

168. (a) A usually short narrative making an edifyingor cautionary point and often employing ascharacters animals that speak and act likehumans.

169. (a) The branch of medicine that deals with thediagnosis and treatment of diseases anddisorders of the skin.

170. (c) The practice of requiring excessivepaperwork and tedious procedures beforeofficial action can be considered orcompleted; also called red-tapery or red-tapist.

171. (b) Intruder is someone who intrudes,especially into a building with criminal intent.

172. (c) Inedible is something that is not suitablefor food or eating.

173. (b) Aquarium174. (a) Facsimile175. (b) A person naturally expresses his anger by

responding aggressively.176. (b) Our feelings place limits on how far we can

take our anger.177. (a) People should deal with their anger by

expressing it assertively.178. (c) According to author, being assertive means

to be respectful of yourself and others.179. (a) One, according to author, can suppress his

anger by holding his anger.180. (b) The passage throws light on how a well-

means invention can be misunderstood.181. (c) The crowd was protesting against the

newly invented sewing machine.182. (d) The aim of the crowd was to destroy the

sewing machines.183. (d) People thought they would be deprived of

their livelihood.184. (c) Shutters were being closed because the

shopkeepers feared their shops would bedestroyed.

185. (b) This part of the sentence should be 'to bewith you.'

186. (a) I really do regret not having learnt to playthe violin….. should be the correctstructure.

187. (c) Injure cannot be an intransitive verb. Youdo not say, for example, 'He injured in a caraccident'. You say 'He was injured in a caraccident'. Fifty workers were injured in thecollapse of the building.'

188. (b) 'I shall have been working in this college' isthe right construction.

189. (d) No error190. (d) Pursue means carry out or participate in an

activity; be involved in or go in search of orhunt for something. Ex: She pursued manyactivities.

191. (c) Making is in singular; hence, 'is' the correcthelping verb.

192. (d) If something is infected, it means the insectsor animals are present (in a place or site) inlarge numbers, typically so as to causedamage or disease. Ex: The house is infestedwith cockroaches.

193. (d) On foot means running or walking usingthe feet. My car won't work so I have totravel on foot. We go everywhere aroundthe campus on foot. He went there on foot.

194. (a) Up and about means no longer in bed (aftersleep or an illness).

195. (d) 'Devout' means deeply religious or pious.196. (d) 'Predilection' means a predisposition in

favour of something; A strong liking;preference.

197. (a) Effigy means a representation of a person(especially in the form of sculpture); hence,dummy.

198. (a) Tremulous means unsteady; hence, steadyis the opposite.

199. (b) The opposite of fake is real.200. (a) Disconsolate means sad beyond

comforting; incapable of being consoled;hence, Joyous is the opposite.

yoursmahboob.w

ordpress.com

PART-A : GENERAL INTELLIGENCE & REASONING1. Introducing a man, a woman said "His mother is

the only daughter of my father'. How is the manrelated to the woman?(a) Son (b) Father(c) Brother (d) Uncle

Directions (Qs. 2-7) : In questions, select the relatedword/letters number from the given alternatives.2. BLOCKED : YOLXPVW : : ? : OZFMMXS

(a) LABOURS (b) LAUNNCH(c) DEBATES (d) RESULTS

3. 12 : 72 : : 8 : ?(a) 36 (b) 32 (c) 38 (d) 40

4. FIG : EGHJFH : : BIN : ?(a) ACJHMO (b) CAHJMOC(c) CAJHOM (d) ACHJMO

5. 6 : 42 : : 7 : ?(a) 48 (b) 40 (c) 56 (d) 52

6. Garbage : House : : Gangue : ?(a) Factory (b) Drugs(c) Office (d) Ore

7. DBU : EEZ : : CJH : ?(a) DMN (b) DNM(c) DNN (d) DMM

Directions (Qs. 8-13): In questions, find the odd word/letters /numbers from the given alternatives.8. (a) Lawyer (b) Legislator

(c) Mayor (d) Governor9. (a) Acute (b) Parallel

(c) Right (d) Obtuse10. (a) 50 (b) 120 (c) 37 (d) 14511. (a) BS (b) EV (c) DX (d) IZ12. (a) OKHA (b) PERU

(c) ERKU (d) ANDI13. (a) 200 (b) 191 (c) 808 (d) 133114. Find out the number of circles in the figure.

(a) 18 (b) 20 (c) 16 (d) 19

Directions (Qs. 15-17): In questions, which one setof letters/numbers when sequentially placed at thegaps in the given letter series shall complete it ?

15. SH _ ELAS _ EELA _ HEELA SHEE _ A(a) HHSS (b) EEHS(c) ELHA (d) EHSL

16. 12_ 41 _ 4123 __ 234(a) 3212 (b) 2134 (c) 3241(d) 1432

17. _ _ aba _ _ ba _ ab(a) abbbb (b) baabb(c) bbaba (d) abbab

18. From the given answer figures, select the one inwhich the question figure is hidden / embedded.

(a) (b)

(c) (d)

19. A word is represented by only one set of numbersas given in any one of the alternatives. The setsof numbers given in the alternatives arerepresented by two classes of alphabets as intwo matrices given below. The columns and rowsof Matrix I are numbered from 0 to 4 and that ofMatrix II are numbered from 5 to 9. A letter fromthese matrices can be represented first by its rowand next by its column, e.g. 'A' can be representedby 01, 14 etc. and 'M' can be represented by 56,68 etc. Similarly, you have to identify the set forthe word 'EAGLE'.

SSC Combined Graduate Level (CGL) Solved PaperMORNING SHIFT 16 AUGUST, 2015

yoursmahboob.w

ordpress.com

350 SSCCGLSolvedPaper

Matrix – I

0 1 2 3 40 P A G R Z1 G R Z P A2 Z P A G R3 A G R Z P4 R Z P A G

Matrix – II

5 6 7 8 95 E M L N O6 L E O M N7 O N E L M8 N O M E L9 M L N O E

(a) 88, 22, 31, 89, 76 (b) 66, 43, 44, 79, 88(c) 99, 01, 44, 96, 77 (d) 55, 14, 11, 78, 66

20. In a certain code, FRACTION is written asFNAITCOR. How is QUANTITATIVE written inthat code ?(a) QTNAVIAITETU (b) QIATAETUTNVI(c) QTEATUIAVITN (d) QEAITATITNVU

21. If a mirror is placed on the line MN, then which ofthe answer figures is the right image of the givenfigure ?

22. Find the number that is common for all of the clue'sgiven below :(A) Virgo(B) Volleyball(C) Highest scoring shot of a particular sport(D) Extra sensory perceptions.(a) 8 (b) 4(c) 2 (d) 6

23. Directions : Study the diagram given below andanswer question.The Qualified and experienced doctors workingin villages are represented by :

1 2

34

56

Qualified Doctors

Experience Doctors

Doctors working invillage

(a) 4 (b) 5 (c) 2 (d) 624. In a certain coding system APPLE stands for

ETTPI. What is the code for 'DELHI'?(a) CQMND (b) ZAHDE(c) HIPLM (d) CQPLM

Directions (Qs. 25-29): In questions, select themissing number from the given responses.

25.2 2

12 3

2 3

30 5

5 1

–5 –1

4 3

? –1

(a) 12 (b) 7 (c) –12 (d) 926. I. 40 32 72 12

II. 30 24 54 9III. 54 ? 90 15(a) 49 (b) 36 (c) 46 (d) 48

27.3

1 60 45

10

3 ? 31

7

6 84 12

(a) 16 (b) 12 (c) 90 (d) 48

28. 4 7

? 11

64343

(a) 1332 (b) 1321 (c) 1231 (d) 133129. 6

15 3

0 4133

1

34 7

2 5?

2

46 3

8 5196

(a) 535 (b) 451 (c) 702 (d) 154

yoursmahboob.w

ordpress.com

351SSCCGLSolvedPaper

30. Name a single letter, which can be deleted fromthe body of the following words to form entirelynew words ?HOST POST COST LOST STOP(a) T (b) P(c) S (d) O

Directions (Qs. 31-32): Questions, one or twostatements is given followed by two conclusions/assumptions, I and II. You have to consider thestatement to be true, even if it seems to be at variancefrom commonly known facts. You have to decide whichof the given conclusions/assumptions can definitelybe drawn from the given statement.31. Statement : No rod is pod, No pod is flexible.

Inferences :I. No rod is flexible.II. Some pods are flexible(a) Both of them follows(b) Only inference I follows(c) Only inference II follows(d) Neither of them follows.

32. Statement : A good book even if costly is soldout in no time.Assumptions:I. Some books are better than othersII. All good books are costly(a) Only I follows(b) Both I and II follows(c) Neither I nor II follows(d) Only II follows

33. If CUSTOM is written as UCTSMO then howPARENT will be written in the same code ?(a) TNERAP (b) RAPTNE(c) ERAPTN (d) APERTN

34. In the question one part of the problem figure issubtracted. Select the option that shows thecorrect shape by the subtraction.

= ?

(a) (b)

(c) (d)

35. Two position of dice are shown below. Whenthree is at the top what number will be at thebottom.

(a) 5 (b) 1(c) 4 (d) 2

36. If 'air' is called 'green', green is called 'blue', 'blue'is called 'sky', 'sky' is called 'yellow', 'yellow' iscalled 'water' and water is called 'Pink' then whatis the colour of clear sky ?(a) Yellow (b) Water(c) Sky (d) Blue

37. A piece of paper is folded and cut as shown belowin the question figures. From the given answerfigures, indicate how it will appear when opened.

(a) (b)

(c) (d)

38. If the words are organised in reverse order ofwhat they appear in dictionary, which word willcome in the third place.(a) Occulist (b) Odium(c) Odious (d) Ordeum

Directions (Qs. 39-40): Which answer figure willcomplete the pattern in the question figure ?39.

?

yoursmahboob.w

ordpress.com

352 SSCCGLSolvedPaper

(a) (b)

(c) (d)

40.

(a) (b)

(c) (d)

Directions (Qs. 41-42) : identify the diagram thatbest represents the relationship among classes givenbelow :41. Vegetables, Potatoes, Eatables

(a) (b)

(c) (d)

42. Christians, Catholics, Pope

(a) (b)

(c) (d)

43. Veena and Veeru both start from a point towardsNorth, Veena turns to left after walking 10 Km.Veeru turns to right after walking the samedistance. Veena waits for some time and thenwalks another 5 Km. Whereas Veeru walks only 3Km. They both then return towards south andwalk 15 Km. forward. How far is Veena from Veeru?(a) 12 km. (b) 10 km. (c) 8 km. (d) 15 km.

44. Find the missing number ?2, 5, 10, 17, 26, ?(a) 36 (b) 49 (c) 37 (d) 47

45. To identify the correct response from the givenpremises stated according to following symbols.'A' stands for not less that ( )'B' stands for not equal to ( ¹ )'C' stands for not greater than ( )'D' stands for greater than (>)'E' stands for less than (<)'F' stands for equal to (=)Premises YF3x and 3xF6Z(a) 2 Y F 3 Z (b) 4 Y B 5 Z(c) 2 Y D 3 Z (d) 2 Y E 3 Z

46. Ravi is walking in the East direction. Aftercovering a distance of one kilometer, he turns45° left and then 90° right. In which direction ishe now ?(a) North (b) West(c) North West (d) North East

47. Which of the following jumbled words is not ananimal ?(a) LATHPEEN (b) TICRECK(c) FEFEOC (d) TAR

48. Arrange the following words as per order in thedictionary(1) Hale (2) Hake (3) Halt (4) Hamlet (5) Ham(a) 1, 3, 5, 2, 4 (b) 2, 1, 3, 5, 4(c) 2, 1, 4, 3, 5 (d) 2, 3, 5, 4, 1

49. If '+' means 'x', '–' means '¸', '×' means '–' and '¸'means '+', then what will be the value of 16 ̧ 64 –8 x 4 + 2 ?(a) 12 (b) 24 (c) 16 (d) 18

50. From the given alternative words, select the wordwhich cannot be formed using the letters of thegiven word.'BOOKBINDING'(a) DONDING (b) DOING(c) DIGGING (d) DINING

yoursmahboob.w

ordpress.com

353SSCCGLSolvedPaper

PART-B : GENERAL AWARENESS51. A computer executes programs in the sequence

of :(a) Decode, Fetch, Execute(b) Execute, Fetch, Decode(c) Fetch, Decode, Execute(d) Store, Fetch, Execute

52. What is 'Reformation'?(a) Revival of classical learning(b) The revolt against authority of pope(c) Rise of absolute monarchy(d) Change in attitude of man

53. Which of the following particles has the dualnature of particle–wave ?(a) Neutron (b) Electron(c) Meson (d) Proton

54. SIDBI stands for :(a) Small Industries Developmental Banker

Institute(b) Small Industrial Designed Bank of India(c) Small Innovations Development Banker's

Institute(d) Small Industries Development Bank of India

55. The metal ion present in vitamin B12 is :(a) nickel (b) cobalt(c) iron (d) zinc

56. Swaraj is my Birth Right and I shall have it. Thiswas advocated by :(a) Mahatma Gandhi (b) Lala Lajpat Rai(c) Sardar Patel (d) Lokmanya Tilak

57. Which of the following is called the Light houseof the Mediterrancan ?(a) Stromboli of sicily(b) Mount Pelee of West Indies(c) Paracutin of Mexico(d) Vesuvius of Italy

58. Who of the following has given the termrhizosphere :(a) Alexopolus(b) Garret(c) Hiltner(d) None of the given options

59. Which one among the following industries in themaximum consumer of water in India ?(a) Textile (b) Engineering(c) Paper and Pulp (d) Thermal Power

60. First Nobel Prize to India was given for :(a) Physics (b) Literature(c) Medicine (d) Chemistry

61. Gandhiji's Famous Quit India Movement call tothe British was given in :(a) 1940 (b) 1942(c) 1941 (d) 1943

62. Choose the correct option which represents thearrangement of atmospheric layers.(a) Troposphere, Stratosphere, Mesosphere,

Ionosphere, Exosphere(b) Mesosphere, Ionosphere, Exosphere,

Troposphere, Stratosphere(c) Ionosphere, Exosphere, Mesosphere,

Troposphere, Stratosphere(d) Exosphere, Troposphere, Ionosphere,

Mesosphere, Stratosphere63. Which of the following options correctly explains

the term ‘heat budget’?(a) It is the amount of heat which the surface of

earth receives form the sun.(b) It is the radiation from the earth in the form

of long waves(c) It is a mode of transfer of heat through matter

by molecular activity.(d) It is the balance between incoming and

outgoing radiation.64. Reverse transcription was discovered by :

(a) Beadle and Tatum(b) Watson and Crick(c) Temin and Baltimore(d) Har Govind Khorana

65. Burns caused by steam are much more severethan those caused by boiling water because:(a) Steam pierces through the pores of body

quickly(b) Temperature of steam is higher(c) Steam is gas and engulfs the body quickly(d) Steam has latent heat

66. Which among the following is the sweetestsugar?(a) lactose (b) maltose(c) glucose (d) fructose

67. Ultra purification of a metal is done by :(a) smelting (b) leaching(c) zone melting (d) slagging

68. The layer of atmosphere close to the earth'ssurface is called:(a) Exosphere (b) Ionosphere(c) Stratosphere (d) Troposphere

69. Microbial degradation of nitrates intoatmospheric nitrogen is known as :(a) Ammonification (b) Denitrification(c) Putrefacation (d) Nitrifcation

70. Which of the following is in the ascending orderof Data hierarchy ?(a) Bit–Byte – Record – Field – Database – File(b) Byte – Bit – File – Record – Database – Field(c) Bit– Byte – Field – Record – File – Database(d) Field – Byte – Bit – Record – File– Database

yoursmahboob.w

ordpress.com

354 SSCCGLSolvedPaper

71. The best milch breed in the world is :(a) Deoni(b) Holstein – Friesian(c) Sindhi(d) Chittagong

72. In which year was the Indian National Congressformed :(a) 1901 (b) 1835 (c) 1875 (d) 1885

73. Bangladesh was created in :(a) 1973 (b) 1970 (c) 1972 (d) 1971

74. Raja Ram Mohan Roy was the founder of :(a) Brahmo Samaj(b) Prathna Samaj(c) Ram Krishna Mission(d) Arya Samaj

75. Pulses are obtained from the family :(a) Liliaceae (b) Fungi(c) Cycadaceae (d) Leguminosae

76. Who was the Indian women president of theUnited Nations General Assembly ?(a) Margret Thatcher (b) Golda Mayer(c) Sarojini Naidu(d) Vijya Lakshmi Pandit

77. The one rupee note bears the signature of :(a) Governor, Reserver Bank of India(b) Finance Minister(c) Secretary, Ministry of Finance(d) None of these

78. What is the currency of Saudi Arabia ?(a) Riyal (b) Pound(c) Lira (d) Dinar

79. Reserve Bank of India was nationalised in :(a) 1951 (b) 1947(c) 1935 (d) 1949

80. Which among the following is a folk dance ofIndia.(a) Kathakali (b) Mohiniattam(c) Manipuri (d) Garba

81. NABARD stands for(a) National business for Accounting and

Reviewing(b) National Bank for Agriculture and Rural

Development(c) National Bank for Aeronautics and Radar

Development(d) National Bureau for Air and Road Transport

82. Surplus budget is recommended during :(a) Depression (b) Boom(c) War (d) Famines

83. Who was the first Speaker of the Lok Sabha :(a) B.R. Ambedkar (b) G.V. Mavalankar(c) N. Sanjeev Reddy (d) Dr S.P. Mukherjee

84. What is the plural volting system?(a) All the citizens caste three votes each(b) Eligible voter exercises one vote and some

voters with specific qualifications cast morethan one vote.

(c) Only the higher officials caste more thanone votes

(d) Candidates themselves caste more than onevote.

85. Which of the following plant shows chloroplastdimorphism?(a) Sugarcane (b) Sugar beet(c) Rice (d) Wheat

86. Day and Night are equal at the :(a) Prime Meridian (b) Poles(c) Equator (d) Antarctic

87. Economic profit or normal profit is the same as :(a) accounting profit (b) optimum profit(c) net profit (d) maximum profit

88. Evergreen type forests are found in :(a) Mediterranean region(b) Monsoon climatic area(c) Desert region(d) Equatorial region

89. The gene which exhibits multiple effects isknown as :(a) Pleiotropic (b) Pseudogene(c) Polygene (d) Complementary

90. The ash–grey soils of high latitude coniferousforests are known as :(a) Grey–Brown soils(b) Red and Yellow soils(c) Tundra soils(d) Podsols

91. Radio activity was discovered by :(a) Curie (b) Beequeral(c) Soddy (d) Rutherford

92. Muddy water is treated with alum in purificationprocess, it is termed as :(a) absorption (b) adsorption(c) coagulation (d) emulsification

93. An enzyme produced by HIV that allows theintegration of HIV DNA into the host cell's DNA is:(a) DNA gyrase (b) Ligase(c) Integrase (d) Helicase

94. Voting is :(a) The unit of area who constitute a unit for

electing representative(b) The process by which voters exercise their

right to vote(c) The process of selecting representatives(d) Universal adult franchise.

yoursmahboob.w

ordpress.com

355SSCCGLSolvedPaper

95. The two specific heats of gases are related by :(a) Cp / Cv = R (b) Cp – Cv = RJ(c) Cp – Cv = R/J (d) Cp + Cv = RJ

96. Who initiated the movement to form the IndianNational Congress :(a) Annie Besant (b) A.O. Hume(c) W.C. Banerjee (d) Gandhi ji

97. Best way to conserve our water resources :(a) All of the options mentioned here.(b) Encouragement of natural regeneration of

vegetation(c) Sustainable water utilization(d) Rain water harvesting

98. Constitutional Monarchy means :(a) The King is elected by the people(b) The King interprets the constitution(c) The King writes the constitution(d) The King exercises power as granted by

constitution99. What is popular sovereignty ?

(a) Sovereignty of the legal head(b) Sovereignty of the head of state(c) Sovereignty of the people(d) Sovereignty of peoples representative

100. Granite, quartzite areas have upstanding lookbecause(a) not easily worn(b) these rocks are resistant to all kinds of

erosion(c) these rocks are not easily eroded(d) mechanically weathered faster

PART-C : NUMERICAL APTITUDE

101. Let C1 and C2 be the inscribed and circumscribedcircles of a triangle with sides 3 cm, 4 cm and 5 cmthen area of C1 to area of C2 is

(a)9

16(b)

925

(c)425

(d)1625

102. If x = 1

2 1+then (x + 1) equals to(a) 2 (b) 2 1-(c) 2 1+ (d) 2

Directions : Questions nos. 103 to 105, The pie–chartgiven here shows expenditure incurred by a family onvarious items and their savings. Study the chart andanswer the questions based on the pie–chart

EDUCATION

70°36°120°

60°54°

20°

FOOD

HOUSING

SAVINGS

CLOTH

OTHERS

103. If the monthly income is ̀ 36000 then the yearlysavings is:(a) ` 72000 (b) ` 60000(c) ` 74000 (d) ` 70000

104. If the expenditure on education is `1600 morethan that of housing then the expenditure on food is:(a) `6000 (b) `12000(c) `7000 (d) `3333

105. The ratio of expenditure on food to savings is :(a) 2 : 1 (b) 3 : 1(c) 3 : 2 (d) 10 : 9

106. The average marks obtained by a student in 6subjects is 88. On subsequent verification it wasfound that the marks obtained by him in a subjectwas wrongly copied as 86 instead of 68. Thecorrect average of the marks obtained by him is–(a) 85 (b) 87 (c) 84 (d) 86

Directions (Qs. 107-110) : Given here a multiple bardiagram of the scores of four players in two innings.Study the diagram and answer the questions.

1009080706050403020100

MahendraSingh Dhoni

Virat Kohli

ChatehswarPujara

ShikharDhawan

RU

NS

AC

CU

RED

1st Innings2nd Innings

yoursmahboob.w

ordpress.com

356 SSCCGLSolvedPaper

107. The average runs of two innings of the playerwho scored highest in average is :(a) 75 (b) 85 (c) 80 (d) 70

108. The average runs in two innings of the player whohas scored minimum at the second innings is :(a) 50 (b) 60 (c) 40 (d) 30

109. The average score in second innings contributedby the four players is :(a) 30 (b) 60 (c) 40 (d) 50

110. The total scores in the first innings contributedby the four players is :(a) 220 (b) 200 (c) 210 (d) 190

111. If the volume of a sphere is numerically equal toits surface area then its diameter is :(a) 4 cm (b) 2 cm (c) 3 cm (d) 6 cm

112. A train runs at an average speed of 75 km/hr. Ifthe distance to be covered is 1050 Kms. Howlong will the train take to cover it ?(a) 13 hrs (b) 12 hrs (c) 14 hrs (d) 15 hrs

113. G is the centroid of DABC. The medians AD andBE intersect at right angles. If the lengths of ADand BE are 9 cm and 12 cm respectively; then thelength of AB (in cm) is ?(a) 10 (b) 10.5 (c) 9.5 (d) 11

114. The minimum value of 2 sin2Q + 3 cos2Q is(a) 1 (b) 3 (c) 2 (d) 4

115. If the three angles of a triangle are :6x 2x

(x 15 ), 6 and 305 3

æ ö æ ö+ ° + ° + °ç ÷ ç ÷è ø è ø then the

triangle is :(a) scalene (b) isosceles(c) right angled (d) equilateral

116. If the number of vertices, edges and faces of arectangular parallelopiped are denoted by v, eand f respectively, the value of (v – e + f) is(a) 4 (b) 2 (c) 1 (d) 0

117. 5 persons will live in a tent. If each person requires16m2 of floor area and 100 m3 space for air thenthe height of the cone of smallest size toaccomodate these persons would be ?(a) 18.75 m (b) 16 m(c) 10.25 m (d) 20 m

118. If the altitude of an equilateral triangle is 12 3cm, then its area would be :(a) 12 cm2 (b) 72 cm2

(c) 36 3 cm2 (d) 144 3 cm2

119. The difference between successive discounts of40% followed by 30% and 45% followed by 20%on the maked price of an article is ̀ 12. The markedprice of the article is:(a) `400 (b) `200 (c) `800 (d) `600

120. The area of the triangle formed by the graphs ofthe equations x = 0, 2x + 3y = 6 and x + y = 3 is

(a) 3 sq. unit (b)1

12

sq. unit

(c) 1 sq. unit (d)142

sq. unit121. Among the equations

x + 2y + 9 = 0; 5x – 4 = 0; 2y – 13 = 0; 2x – 3y = 0,the equation of the straight line passing throughorigin is–(a) 2x – 3y = 0 (b) 5x – 4 = 0(c) x + 2y + 9 = 0 (d) 2y – 13 = 0

122. The HCF of x8 – 1 and x4 + 2x3 – 2x – 1 is:(a) x2 + 1 (b) x + 1 (c) x2 – 1 (d) x – 1

123. The least number which when divided by 6, 9, 12,15, 18 leaves the same remainder 2 in each case is:(a) 178 (b) 182 (c) 176 (d) 180

124. Internal bisectors of ÐQ and ÐR of DPQRintersect at O. If ÐROQ = 96° then the value ofÐRPQ is :(a) 12° (b) 6° (c) 36° (d) 24°

125. A certain sum will amount to ̀ 12,100 in 2 years at10% per annum of compound interest, interestbeing compounded annually. The sum is:(a) `12000 (b) `6000(c) `8000 (d) `10000

126. A's 2 days work is equal to B's 3 days work. If Acan complete the work in 8 days then to completethe work B will take :(a) 14 days (b) 15 days(c) 16 days (d) 12 days

127. If the measure of three angles of a triangle are inthe ratio 2 : 3 : 5, then the triangle is :(a) equilateral (b) isocsceles(c) Obtuse angled (d) right angled

128. What must be added to each term of the ratio 2 :5 so that it may equal to 5 : 6 ?(a) 12 (b) 78 (c) 65 (d) 13

129. If the sum and difference of two angles are 229

radian and 36° respectively, then the value ofsmaller angle in degree taking the value of p as227

is :(a) 60° (b) 48° (c) 52° (d) 56°

130. 4 men and 6 women complete a work in 8 days, 2men and 9 women also complete in 8 days. Thenumber of days 18 women complete the work is :

(a)243

days (b)253

days

(c)143

days (d)153

days

yoursmahboob.w

ordpress.com

357SSCCGLSolvedPaper

131. If x = 24

12x 1 7

x+ = then the value of

72

36x 1

x+

(a) 432 (b) 433(c) 343 (d) 322

132. If 5x + 9y = 5 and 125x3 + 729y3 = 120 then thevalue of the product of x and y is

(a) 135 (b)1

135(c)

19

(d) 45

133. If 4 men or 8 women can do a piece of work in 15 days,in how many days can 6 men and 12 women do thesame piece of work ?(a) 45 days (b) 20 days(c) 15 days (d) 30 days

134. The value of sin222° + sin268° + cot230° is

(a)34

(b) 4 (c)54

(d) 3

135. Find a simple discount equivalent to a discountseries of 10%, 20% and 25%(a) 45% (b) 55%(c) 52% (d) 46%

136. If q be acute angle and tan (4q – 50°) = cot(50° – q), then the value of q in degrees is :(a) 30 (b) 40(c) 20 (d) 50

137. Cost price of 100 books is equal to the sellingprice of 60 books. The gain or loss percentagewill be :

(a)266 %3

(b)166 %4

(c) 66% (d)366 %4

138. If 5sinq = 3, the numerical value of sec tansec tan

q - qq + q

sec tansec tan

q - qq + q

(a)13

(b)12

(c)14

(d)15

139. If 34

of a number is 7 more than 16

of the number,,

then 53

of the number is :

(a) 15 (b) 18 (c) 12 (d) 20140. What is the arithmetic mean of first 20 odd natural

numbers?(a) 17 (b) 19 (c) 22 (d) 20

141. A kite is flying at the height of 75 m from theground. The string makes an angle q (where cot

q = 8

15) with the level ground. Assuming that

there is no slack in the string, the length of thestring is equal to :(a) 75 m (b) 85 m (c) 40 m (d) 65 m

142. If a person travels from a point L towards east for12 km and then travels 5 km towards north andreaches a point M, then shortest distance from Lto M is :(a) 14 (b) 12 (c) 17 (d) 13

143. In an examination, a student must get 36% marksto pass. A student who gets 190 marks failed by35 marks. The total marks in that examination is :(a) 500 (b) 625(c) 810 (d) 450

144. A train 180 mt long is running at a speed of 90km/hr. How long will it take to pass a post ?(a) 8.2 secs (b) 7.8 secs(c) 8 secs (d) 7.2 secs

145. An article which is marked ̀ 975 is sold for ̀ 897.The discount % is ?(a) 6% (b) 10%(c) 12% (d) 8%

146. If sec q + tan q = p, (p ¹ 0) then secq s equal to

(a)1p ,p 0p

æ ö+ ¹ç ÷è ø (b)

1 1p ,p 02 p

æ ö+ ¹ç ÷è ø

(c)12 p ,p 0p

æ ö- ¹ç ÷è ø (d)

1p ,p 0p

æ ö- ¹ç ÷è ø

147. If p = 99 then the value of p (p2 + 3p + 3)(a) 999999 (b) 988899(c) 989898 (d) 998889

148. If x = 2 then the value of x3 + 27x2 + 243x + 631(a) 1233 (b) 1231(c) 1321 (d) 1211

149. If D, E and F are the mid points of BC, CA and ABrespectively of the DABC then the ratio of areaof the parallelogram DEFB and area of thetrapezium CAFD is :(a) 1 : 3 (b) 1 : 2(c) 3 : 4 (d) 2 : 3

150. If A and B are in the ratio 4 : 5 and the differenceof their squares is 81, what is the value of A?(a) 36 (b) 15(c) 45 (d) 12

yoursmahboob.w

ordpress.com

358 SSCCGLSolvedPaper

PART-A : ENGLISH COMPREHENSION

Directions (Qs. 151-155): In Question given belowsentences given with blanks to be filled in with anappropriate word(s). Four alternatives are suggestedfor each question. Choose the correct alternative outof the four and indicate it by blackening theeappropriate circle in the Answer Sheet.151. When the morning ______________the murder

was discovered.(a) came (b) happened(c) arrived (d) occurred

152. The smell of the Sea called ___________memories of her childhood.(a) on (b) black (c) up (d) for

153. He has the full facts ___ but is deliberately hidingthem.(a) under his sleeves (b) upon his sleeves(c) up his sleeve (d) in his sleeves

154. The appearance of the city _______ day by day.(a) could change (b) changed(c) had changed (d) is changing

155. The police fired on the mob when they ________(a) turned noisy (b) turned violent(c) became abusive (d) fizzled out

DIRECTIONS (Qs. 156-158): In questions, out of thefour alternatives, choose the one which best expressesthe meaning of the given word and mark it in theAnswer Sheet.156. BOARD

(a) Frame (b) Lodging(c) Food (d) Furniture

157. PERUSE(a) Follow (b) Argue (c) Reduce(d) Read

158. SPUME(a) foam (b) Lava (c) poison (d) spit

DIRECTIONS (Qs. 159-163): In question, you have apassage with 5 questions following. Read the passagescarefully and choose the best answer to each questionout of the four alternatives and mark it by blackeingthe appropriate circle in the Answer Sheet.

PASSAGETrue, it is the function of the army to maintain law andorder in abnormal times. But in normal times there isanother force that compels citizens to obey laws andto act with due regard to the rights of others. Theforce also protects the lives and properties of lawabiding men. Laws are made to secure the personalsafety of its subjects and to prevent murder and crimesof violence. They are made to secure the property ofthe citizens against theft and damage and to protect

the rights of communities and castes t carry out theircustoms and ceremonies, so long as they do notconflict with the rights of others. Now the good citizen,of his own free will obeys these laws and he takescare that everything he does is done with due regardto the rights and well being of others.But the bad citizen is only restrained from breakingthese laws by fear of the consequence of his actions.And the necessary steps to compel the bad citizen toact as a good citizen are taken by this force. The supremecontrol of law and order in a State is in the hands of aMinister, who iss responsible to the State Assemblyand acts through the Inspector General of Police.159. The expression "customs and ceremonies"

means :(a) habits and traditions(b) fairs and festivals(c) superstitions and formalities(d) usual practices and religious rites

160. "They are made to secure the property of citizensagainst theft and damage" means that the law :(a) Safeguards people's possessions against

being stolen or lost(b) Initiates process against offenders of law(c) helps in recovering the stolen property of

the citizens(d) Assists the citizens whose property has

been stolen or destroyed.161. Which one of the following statement is implied

in the passage ?(a) The police hardly succeed in converting bad

citizens into good ones.(b) Criminals, who flout the law, are seldom brought

to book(c) Peaceful citizens seldom violate the law(d) The police check the citizens, whether they

are good or bad, from violating the law.162. According to the writer, which one of the

following is not the responsibility of the police ?(a) To check violent activities of citizens.(b) To maintain peace during extraordinary

circumstances.(c) To protect the privileges of all citizens(d) To ensure peace among citizens by

safeguarding individual rights163. Which of the following statements is not implied in

the passage ?(a) Law protects those who respect it(b) A criminal is deterred from committing crimes

only for fear of the law(c) The forces of law help to transform

irresponsible citizens into responsible ones(d) Law ensures people's religious and social

rights absolutely ad unconditionally.

yoursmahboob.w

ordpress.com

359SSCCGLSolvedPaper

DIRECTIONS (Qs. 164-173): In Question, a sentence/a part of the sentence is underlined. Below are givenalternatives to the underlined part which mayimprove the sentence. Choose the correct alternative.In case no improvement is needed choose "NoImprovement". Mark your answer in the answer sheet.164. He does not laugh, nor he smiles.

(a) nor he does smile (b) No improvement(c) neither he does smile (d) nor does he, smile

165. If I will get an opportunity, I shall attend theseminar.(a) get (b) No improvement(c) shall get (d) got

166. The others shook their heads and made vaguenoises of approval.(a) nodded (b) No improvement(c) turned around (d) hung

167. The world is being faced with a crisis(a) No improvement (b) facing(c) confronted (d) in front of

168. She insisted on she was innocent.(a) No improvement (b) insisted on that(c) insisted that (d) insisted with

169. She could not help but laugh(a) but laughing (b) No improvement(c) laughter (d) laughing

170. I took my mother some grapes when she was inhospital.(a) I took some grapes for my mother(b) No improvement(c) I took for my mother some grapes(d) I brought my mother some grapes

171. We met yesterday, haven't we ?(a) isn't it ? (b) No improvement(c) didn't we ? (d) hadn't we ?

172. Both the teams played the game fairly.(a) played fairly the game (b) No improvement(c) fairly played the game (d) played a fair game

173. You must endure what you cannot cure.(a) suffer (b) No improvement(c) prevail (d) accept

DIRECTIONS(Qs. 174-176): In Question, choose theword opposite in meaning to the given word andmark it in the Answer sheet.174. VACILLATION

(a) inoculation (b) relief(c) steadfastness (d) remorse

175. Placid(a) dull (b) stormy(c) urgent (d) moving

176. Effeminacy(a) manliness (b) boorishness(c) aggressiveness (d) attractiveness

DIRECTIONS (Qs. 177-181): In Question, some partsof the sentences have errors and some are correct. Findout which part of a sentence has an error and blackenthe circle corresponding to the appropriate correctoption. If a sentence is free from error, blacken the circlecorresponding to "No error" option in the Answer sheet.177. A bird in the tree is worth two in the bush.

(a) No error (b) two in the bush(c) is worth (d) A bird in the tree

178. It is not advisable to take heavy luggages whileon a journey.(a) while on a journey(b) to take heavy luggages(c) It is not advisable(d) No error

179. Just outside my house are a playground forschool boys and girls(a) for school boys and girls(b) are a playground(c) No error(d) Just outside my house

180. He can be a basketball player since he is tall like a mule.(a) No error(b) He can be(c) He is tall like a mule(d) A basketball player since

181. Who do you think I met?(a) Who (b) I met ?(c) No error (d) Do you think

DIRECTIONS (Qs. 182-188): In Question, out of thefour alternatives, choose the one which can besubstituted for the given words / sentences andindicate it by blackening the appropriate circle inthe Answer sheet.182. Of one's own free will

(a) obligatory (b) mandatory(c) voluntary (d) compulsory

183. One who runs away from justice or the law.(a) Fugitive (b) Thief(c) Criminal (d) Smuggler

184. One who is skillful.(a) disciplined (b) different(c) diligent (d) dexterous

185. One who is too careless to plan for the future.(a) impotent (b) improvident(c) impractical (d) imprudent

yoursmahboob.w

ordpress.com

360 SSCCGLSolvedPaper

186. A person who deserves all praise(a) despicable (b) detestable(c) laudable (d) lovable

187. One who has long experience(a) expert (b) novice(c) practitioner (d) veteran

188. A study of ancient things.(a) Physiology (b) Archaeology(c) Ethnology (d) Zoology

DIRECTIONS (Qs. 189-190): In question, four wordsare given in each question, out of which only oneword is correctly spelt. Find the correctly spelt wordand mark your answer in the Answer Sheet189. (a) DIFFUSSION (b) DEFFUSION

(c) DIFFUSION (d) DEFUSION190. (a) Circumlocation (b) Circumlocution

(c) Circmlocution (d) Circumlocutien

DIRECTIONS (Qs. 191-195): In question, fouralternatives are given for the Idiom/phraseunderlined in the sentence. Choose the alternativewhich best expresses the meaning of the Idiom/Phraseand mark it in the Answer sheet.191. Unless you make amends for the loss, nobody is

prepared to excuse you.(a) improve (b) pay debt(c) confess (d) compensate

192. Instead of keeping his promise of helping me withoffice work, he just left me high and dry.(a) left me feeling like a fool(b) left me in a state of anger(c) left me without a drop of water(d) left me alone to do the work

193. Amit said to Rekha, "Don't make a mountain outof a molehill".(a) attempt an impossible task(b) start looking for molehills in mountains.(c) create problems(d) exaggerate a minor problem

194. Before the report reached the authority, the mediaspilled the beans.(a) dropped the charges(b) hinted at the consequences(c) revealed the secret information(d) spilled the content of the package

195. His friend turned out to be snake in the grass.(a) cowardly and brutal(b) low and mean(c) a hidden enemy(d) an unreliable and deceitful person.

Directions (Qs. 196-200): In question, you have apassage with 5 questions following. Read thepassages carefully and choose the best answer toeach question out of the four alternatives and markit by blackening the appropriate circle in the Answersheet.Journalists argue over functions of a newspaper. I feelthat a provincial paper's purpose is not only to presentand project the news objectively and imaginatively,but to help its readers to express themselves moreeffectively, canalizing their aspirations, making morearticulate their demands. A newspaper should reflectthe community it serves– warts and all. When the mirroriss held to society it reveals neglect, injustice, ignoranceor complacency. It should help to eradieate them. Itwould be pretentious to think that a newspaper canchange the course of world affairs but at the local limitit can exert influence, it can probe, it can help get thingsdone. The individual's voice must not be stifled.Instead, the readers should be encouraged to expresstheir opinions, fears, hopes, and or their grievances onthis platform.196. How can the readers air their grievances ?

(a) By being complacent.(b) By supporting the local newspaper(c) By writing to journalists(d) By writing to their local newspaper

197. What is the main purpose of a newspaper?(a) Project news objectively and imaginatively(b) To present facts in a blunt way(c) Exert influence on the individuals(d) Encourage the readers to be pretentious

198. The expression "wants and all" in the passagemeans :(a) hopes and fears(b) the reader's grievances(c) with no attempt to conceal blemishes and

inadequacies(d) the community's problems

199. How can a newspaper influence local affairs ?(a) By probing in the ills of society and rallying

support for change(b) By encouraging the readers to accept their

grievances(c) By focusing on world affairs(d) By influencing public opinion through half

truths.200. In this passage the writer highlights the fact that :

(a) A newspaper should reflect the communityit serves

(b) A newspaper should only concentrate onlocal affairs

(c) Journalists differ in their opinion on thefunction of a newspaper

(d) Newspaper can eradicate injustice

yoursmahboob.w

ordpress.com

361SSCCGLSolvedPaper

1. (a) Father

(Woman)

ManHence, man is son of the woman.

2. (b) B L O C K E D

Y O L X P V W¯ ¯ ¯ ¯ ¯ ¯ ¯

Above word follows reverse order ofalphatets.Similarly,

L A U N N C H

O Z F M M X S¯ ¯ ¯ ¯ ¯ ¯ ¯

3. (b) \ 1212 722

´ =

\ 88 322

´ =

4. (d)

–1 +1 –1 +1 –1 +1

E G

IF G

J HFHSimilarly,

–1 +1 –1 +1 –1 +1

A C

IB N

J OMH5. (c) 6 × 6 + 6 = 42

7 × 7 + 7 = 566. (d) The waste of the house is called garbage.

Similarly, the impurities in the ore are calledgangue.

7. (d)

D B U

E E Z

+1 +3 +5C J H

D M M

+1 +3 +5Similarly,

8. (a) All others except (a) have official rankings9. (b) All other except (b) are forms of angles10. (c) All others except (c) are the multiples of 5.11. (b) Only in option (b) first letter is 4th from the

left end and second letter is 4th from theright end. Others have no sequence

12. (b) All except ‘PERU’ have vowel in both ends.13. (d) 1331 is the cube of 1114. (d) There are 19 circles in the figure.15. (d) S H E E L A / S H E E L A / S H E E L A / S

H E E L A16. (c) 1 2 3 4 / 1 2 3 4 / 1 2 3 4 / 1 2 3 417. (d) a b a b a b a b a b a b18. (b)19. (c) E A G L E

99 01 44 96 77¯ ¯ ¯ ¯ ¯

20. (d) F R A C T I O N

F N A I T C O RSimilarly

Q U A N T I T A T I V E

Q E A I T A T I T N V U21. (d)22. (d) (A) Virgo is the sixth sign of zodiac.

(B) Volleyball is a team sport in which eachteam has six players

(C) A highest scoring short of a particularsport is six.

23. (b)

Qualified Doctors

1 2

3

46

Doctors working in Village

Experience Doctors

5

HINTS & SOLUTIONS

yoursmahboob.w

ordpress.com

362 SSCCGLSolvedPaper

Hence, shaded portion in above diagramrepresents.Qualified Experienced Dorctors working invillage i.e. 5.

24. (c) A P P L E

E T T P I

+4 +4 +4 +4 +4

Similarly,D E L H I

H I P L M

+4 +4 +4 +4 +4

25. (c) 2 × 2 × 3 = 122 × 3 × 5 = 305 × 1 × –1 = –5\ 4 × 3 × –1 = – 12

26. (b) 40 + 32 = 726

= 12

30 + 24 = 546 = 9

54 + 36 = 906 = 15

27. (c) 1 × 3 × 4 × 5 = 606 × 7 × 1 × 2 = 84therefore, 3 × 10 × 3 × 1 = 90.

28. (d)

11 = 13313

47

117 = 3433

4 = 643

29. (d) Figure follows the Rule(0 + 6 + 4 + 3 + 1 + 5) × 7 = 19 × 7 = 133(8 + 2 + 5 + 3 + 6 + 4) × 7 = 28 × 7 = 196

(2 + 1 + 5 + 7 + 3 + 4) × 7 = 22 × 7 = 15430. (c) ‘S’ can be deleted from the body of the

following words to form entirely new words.New words are :HOT, POT, COT, LOT, TOP

31. (d)Rod Pod Flexible

Inferences :I : (False)II : (False)

Both statements are False, so noconclusion follows.

32. (c) Neither I nor II assumption follows the givenstatement.

33. (d) C U S T O M U C T S M O

Every two letters get interchanged theirposition.Therefore,P A R E N T A P E R T N

34. (c) 35. (a)36. (c) The color of sky is blue. But blue is called

sky. Hence, option (c) is correct choice.37. (a)38. (c) Words in reverse order :

Ordeum ® Odium ® Odious ® Occulist (1) (2) (3) (4)

39. (a)

40. (a)

41. (a)

Potatoes

Vegetables

Eatables

yoursmahboob.w

ordpress.com

363SSCCGLSolvedPaper

42. (b)

Pope

Catholics Christians

43. (c)

A(Veena and Veeru

Starting point)

8 km

Veeru Destination

Point

Veena Destination

Point

3 km

Veena’s 10 km

15 km

5 kmleft right

By looking at the above diagram, it is clearthat Veena is 8 km for away form Veeru’sposition.

44. (c) 2 5 10 17 26 37

+3 +5 +7 +9 +1145. (a) A B C

D E FÞ< Þ¹ Þ>/ /Þ> Þ< Þ=

Premises :4Y = 3x and 3x = 6Z(a) 2Y = 3Z (ü) (b) 4Y ¹ 5Z (û)(c) 2Y > 3Z (û) (d) 2Y < 3Z (û)

46. (d)

45°

left

Destination90°Right

1 kmRavi’sStarting pointAfter walking towards 90° right, he is nowin north east direction.

47. (c) LATHPEEN ® ELEPHANTTICRECK ® CRICKETFEFEOC ® COFFEETAR ® RATHence, coffee is not animal.

48. (b) HAKE > HALE > HALT > HAM >HAMLOT

49. (c) 16 ÷ 64 – 8 × 4 + 2Þ 16 + 64 ÷ 8 – 4 × 2Þ16 + 8 – 4 × 2Þ 16 + 8 – 8 Þ 16

50. (c) DIGGING cannot be formed from the givenword.

51. (c) The first step the CPU carries out is to fetchsome data instructions from main memorythen store them in its own internal temporarymemory areas. The next step is for the CPUto make sense of the instruction it has justfetched this process is called Decode.Execute is the part of the cycle when dataprocessing actually takes place.

52. (b) Reformation was a 16th-century movementin Western Europe that aimed at reformingsome doctrines and practices of the RomanCatholic Church and resulted in theestablishment of the Protestant churches.

53. (b) Wave particle duality is a quantummechanics effect. It means the electronsometimes acts like a particle and sometimesit acts like a wave. It depends on thesituations.

54. (d) SIDBI stands for Small IndustriesDevelopment Bank of India. It is anindependent financial institution aimed toaid the growth and development of micro,small and medium-scale enterprises(MSME) in India. It was set up on April 2,1990 through an act of parliament.

55. (b) Vitamin B12 consists of a class of chemicallyrelated compounds (vitamers). It containsthe biochemically rare element cobalt.

56. (d) Bal Gangadhar Tilak is considered as"Father of Indian National Movement". Hewas a social reformer, freedom fighter,national leader and a scholar of Indianhistory, sanskrit, hinduism, mathematics andastronomy. During freedom struggle, hisslogan "Swaraj is my birthright and I shallhave it" inspired millions of Indians.

57. (a) Stromboli, a small island north of Sicily, isone of the most active volcanoes in theworld and famous for its normally small, butregular explosions throwing out glowinglava from several vents inside its summitcrater. This activity has been going on forat least 2000 years. Thus, it is called Lighthouse of the Mediterranean.

yoursmahboob.w

ordpress.com

364 SSCCGLSolvedPaper

58. (d) In 1904 the German agronomist and plantphysiologist Lorenz Hiltner first coined theterm "rhizosphere" to describe the plant-root interface, a word originating in part fromthe Greek word "rhiza", meaning root.Hiltner described the rhizosphere as the areaaround a plant root that is inhabited by aunique population of microorganismsinfluenced, he postulated, by the chemicalsreleased from plant roots.

59. (d) Thermal Power Plants consumes maximumindustrial water in India.

60. (b) Rabindranath Tagore was the first Indianever to receive a Nobel Prize. He was awardedthe Nobel Prize for Literature in recognitionof his work Geetanjali, a collection of poems,in 1913.

61. (b) The Quit India Movement(August Kranti),was a civil disobedience movementlaunched in India on 9 August 1942 byMohandas Karamchand Gandhi.

62. (a) The current sequence is Troposphere-Stratosphere-Mesosphere-Ionosphere-Exosphere

63. (d) The Earth and the atmosphere are heatedby energy from the sun. The atmosphericheat budget of the Earth depends on thebalance between insolation and outgoingterrestrial radiation.

64. (c) Reverse transcription were discovered byHoward Temin and independently isolatedby David Baltimore.

65. (d) Steam will produce more severe burns thanboiling water because steam has more heatenergy than water due to its latent heat ofvaporisation.

66. (c) Fructose is the sweetest of all natural sugartypes.

67. (c) Zone melting, any of a group of techniquesused to purify an element or a compound orcontrol its composition by melting a shortregion (i.e., zone) and causing this liquidzone to travel slowly through a relativelylong ingot, or charge, of the solid.

68. (d) The troposphere is the first layer above thesurface and contains half of the Earth'satmosphere. Weather occurs in this layer.This is closest to the Earth's surface,extending up to about 10-15 km above theEarth's surface.

69. (b) 'Dentrification' is the biological conversionof nitrate to nitrogen gas, nitric oxide ornitrous oxide. These compounds aregaseous compounds and are not readilyavailable for microbial growth; thereforethey are typically released to the atmosphere.

70. (c) · Bit (Binary digit) = a bit can onlyhave one of two possible values.

· Byte = 1 byte = 8 bits· Field = one or more bits/bytes holding

a single piece of information about asingle entity

· Record (Logical) = all the fields whichapply to a single entity (except for"Inverted" files)

· File = all the records which apply to acollection of entities

· Database = a related collection of Files.71. (b) Holstein Friesians are a breed of cattle

known today as the world's highest-production dairy animals. They are foundin Europe and America.

72. (d) The Indian National Congress was foundedon December 28, 1885, by members of theTheosophical Society. The foundersincluded a prominent member of theTheosophical Society, Allan Octavian Humeas well as Dadabhai Naoroji and DinshawWacha.

73. (d) Modern Bangladesh emerged as anindependent nation in 1971 after achievingindependence from Pakistan in theBangladesh Liberation War. The ProvisionalGovernment of Bangladesh was formed on17 April 1971.

74. (a) 'Brahmo Samaj' was founded by Raja RamMohan Roy in 1828.

75. (d) Pulses plants belong to the familyLeguminosae.

76. (d) Vijaya Lakshmi Pandit became the firstwoman to be elected president of the UNGeneral Assembly in 1953. She was anIndian diplomat, politician, and a sister ofIndia's first prime-minister, JawaharlalNehru. She was active in the Indian freedommovement and held high national andinternational positions.

77. (d) The one rupee note bears the signature ofsecretary ministry of Finance.

78. (a) The currency of Saudi Arabia is Saudi Riyal.

yoursmahboob.w

ordpress.com

365SSCCGLSolvedPaper

79. (d) After independence, the governmentpassed Reserve Bank (Transfer to PublicOwnership) Act, 1948 and took over RBIfrom private shareholders after payingappropriate compensation. Thus,nationalisation of RBI took place in 1949and from January 1, 1949, RBI startedworking as a government owned centralbank of India.

80. (d) Garba is a folk dance of state of Gujarat.81. (b) NABARD is set up as an apex Development

Bank with a mandate for facilitating creditflow for promotion and development ofagriculture, small-scale industries, cottageand village industries, handicrafts and otherrural crafts. It was established on 12 July1982 by a special act by the parliament

82. (b) Surplus budget is the order of the economiesin boom time.

83. (b) G. V Mavalankar was the first speaker of theLok Sabha.

84. (d) Plural voting is the practice whereby oneperson might be able to vote multiple timesin an election.

85. (a) Sugarcane shows chloroplast dimorphism.The presence of bundle sheath withchloroplast in the leaves is granulated andlacking in starch. Such feature is known aschloroplast dimorphism.

86. (c) The equator receives equal day and nightthroughout the year because it does not tilt inrelation to the sun's location. Because of thetilted axis of the Earth, the poles and locationsaway from the equator lean towards or awayfrom the sun as an orbit is completed, whilethe equator stays in essentially the samelocation relative to the sun.

87. (b) Economic profit is the difference betweentotal revenue and total opportunity cost. Ifa firm's total opportunity cost is less thanthe total revenues then the firm is makingeconomic profit. If a firm's total opportunitycost is greater than the total revenues thenthe firm is making economic losses. It issimilar to optimum profit.

88. (b) The tropical rainforest is earth's mostcomplex biome in terms of both structureand species diversity. It occurs underoptimal growing conditions, abundantprecipitation and year round warmth. Meanmonthly temperatures are above 64 ° F;

precipitation is often in excess of 100 inchesa year. There is usually a brief season ofreduced precipitation. In monsoonal areas,there is a real dry season but that is morethan compensated for with abundantprecipitation the rest of the year.

89. (a) pleiotropic gene is a gene that causes anumber of distinct but seemingly unrelatedphenotypic effects.

90. (a) podsol soil is a soil that develops intemperate to cold moist climatesunderconiferous or heath vegetation; anorganic mat over a grey leached layer.

91. (b) Antoine-Henri Becquerel (1852-1908) isknown for his discovery of radioactivity, forwhich he received the Nobel Prize forPhysics jointly with Marie Curie) and Pierrein 1903 and the contributions he made tothat field. Henri Becquerel discoveredradioactivity, and Mari Curie coined theterm.

92. (c) Coagulation removes dirt and other particlessuspended in water. Alum and otherchemicals are added to water to form tinysticky particles called "floc" which attractthe dirt particles. The combined weight ofthe dirt and the alum (floc) become heavyenough to sink to the bottom duringsedimentation.

93. (c) Retroviral integrase (IN) is an enzymeproduced by a retrovirus (such as HIV) thatenables its genetic material to be integratedinto the DNA of the infected cell.

94. (c) Voting is the process of selectingrepresentatives.

95. (b)96. (b) Allan Octavian Hume was a civil servant,

political reformer in British India whoinitiated the movement to form the IndianNational Congress.

97. (a) All are correct98. (d) Constitutional monarchy is a form of

government in which a king or queen actsas Head of State. The ability to make andpass legislation resides with an electedParliament, not with the Monarch.

99. (c) Popular sovereignty or the sovereignty ofthe people is the principle that the authorityof the government is created and sustainedby the consent of its people, through theirelected representatives (Rule by thePeople), who are the source of all politicalpower.

yoursmahboob.w

ordpress.com

366 SSCCGLSolvedPaper

100. (c) Granite and quartzite areas have upstandinglook because these rocks are not easilyeroded

101. (c) Here the side of triangle are3, 4 and 5. So triangle is right angletriangle.When triangle is circumscribed (C1)

3

4

5O

A

B C

Radius of circle = 52

When triangle is Inscribed (C2)

3–x

A

B

O

x E 4 – x

3 – x

5

4 – x

C

3

D

x

4Let BE = xThen DE = x (triangle)

CE = 4 – xAC = 3 – x + 4 – x

Þ 5 = 7 – 2xx = 1

Now,

1

2

Area ofArea of

CC =

21

22

rr

pp

2

2(1) 4

2552

p =æ öpç ÷è ø

102. (d) x = 1 2 1

2 12 1 2 1

-´ = -

+ -Now, x + 1 = 2 1 1- + = 2

103. (a) Monthly savings = 60 36000

360´

= ̀ 6000Yearly savings = 6000 × 12

= ̀ 72000104. (b) Let Total Expendition be ̀ x

(70 54 )360

x ´ ° - ° = 1600

Þ16 1600

360x ´ =

x = ̀ 36000

Expenditure on food = 120 36000360

° ´

= ̀ 12000

105. (a) Ratio of Expenditure = 120 260 1

= = 2 : 1

106. (a) Correct Average Marks

= (86 68)88

6--

= 88 – 3= 85

107. (d) Highest Average in two Innings

= 60 80 70.

2+ =

108. (c) Less score in second innings by playerpujara = 10

Average score in two innings

= 70 10 40

2+ =

109. (c) Average score in second Innings

= 80 50 10 20

4+ + +

= 160 40.

4=

110. (c) Total score in Ist Innings= 60 + 50 + 70 + 30 = 210.

111. (d) According to questionVolume of sphere= surfae area of shpere

3 24 r 4 r3

Þ p = p

r 3cmÞ =diameter = 6 cm

112. (c) Average speed = 75k/hDistance = 1050 kmsTime taken to cover the distance =1050

75=14 hrs.

113. (a) Given AD = 9 cmBE = 12 cm

yoursmahboob.w

ordpress.com

367SSCCGLSolvedPaper

A

B

O

D C

E

Here AD and BE Intersect at O (AD ̂ BE)\ ÐAOB = 90°

AO =2 2AD 9 6cm3 3

´ = ´ =

OB =2 2

BE 12 8cm3 3

´ = ´ =

AB = 2 2(AO) (OB)+

= 2 2(6) (8)+

= 36 64+= 10 cm

114. (c) Given that2sin2q + 3cos2q= 2(1 – cos2q) + 3cos2q= 2 – 2cos2q + 3 cos2q= 2 + cos2qFor minimum value cosq = 0Minimum value of 2sin2q + 3 cos2q = 2

115. (d) Angles are = (x + 15°),

6x 2x

6 and 305 3

æ ö æ ö+ ° + °ç ÷ ç ÷è ø è øWe know thatSum of the angles of a triange is 180°.

Þ 6x 2x

x 15 6 30 1805 3

+ ° + + ° + + ° =

Þ 15x 18x 10x

51 18015

+ ++ =

Þ 43x

180 51 12915

= - =

Þ 43x = 129 ́ 15 x = 45°Then angle are = (45+ 15°),6 45

6 and5

´æ ö+ °ç ÷è ø 2 45

303

´æ ö+ °ç ÷è ø= 60°, 60°, 60°So this is an equilateral triangle.

116. (b) The value of = v – e + f= 8 – 12 + 6 = 2.

117. (a) For surface Area of 5 persen2 5 16rp = ´

2 80r =p

Now, volume of cone = volume of air spaceof 5 person

Þ 21 5 1003

r hp = ´

Þ1 80 5 1003

hp´ ´ = ´p

\ h = 75 18.754

=

118. (d) Let ABC is equilateral triangle. Where sidesis a cm.

A

B D C

Þ Area of equilateral triangle = 23Va

4

21 39 AD a

2 4Þ ´ ´ =

1 312 3 a

2 4Þ ´ =

Þ a = 24 cm

Now, Area of triangle = 23a

4

= 3

24 244

´ ´

= 148 3 cm119. (d) 1st successive discount final rate

= xyx 4

100- - +

= 40 3040 30

100´- - +

= – 70 + 12= – 58%2nd successive discount final rate

= 95 2095 20

100´- - +

yoursmahboob.w

ordpress.com

368 SSCCGLSolvedPaper

= –65 + 9 = – 56%Let mare price be MP

then 58 56MP MP 12

100 100´ - ´ =

MP 2 12100´Þ =

MP = ` 600120. (b) x = 0 ...(1)

2x + 3y = 6 ...(2)2x + y = 3 ...(3)

AB

(0, 3)X = 0

Y – Axis

CO X Axis

(3, 0)

x + y = 3

2x + 3y = 6

2x + 3y = 6 ...(ii)x = 0, y = 2x = 3, y = 0

From eqn. (iii) x+ y = 3x = 0, y = 3x = 3, y = 0

Area made by these three lines= Area of triangle OBC – Area of OAC

= 1 13 3 2 32 2´ ´ - ´ ´

= 9 3 13 1 sq.2 2 2- = =

121. (a) 2x – 3y = 0 is passing through origionbecause its satisfy x = 0 and y = 0.

122. (c) Given fraction8 4 21 ( 1)( 1)( 1)( 1)x x x x x- = + + - +4 3 32 2 1 ( 1)( 1)x x x x x+ - - = - +

HCF of Given fractor

= 2( 1)( 1) 1x x x- + = -

123. (b) LCM of 6, 9, 12, 15 and 18

2 6,9,12,15,183 3,9,6,15,93 1,3,2,5,3

1,1,2,5,1

LCM = 2 × 3 × 3 × 2 × 5 = 180Least number = 180 + 2 = 182

124. (a) ÐQPR = P90

2Ð+

Þ 96° = P90

2а+

P

QR

96°

ÞP 6

2Ð = °

\ ÐP = 12°125 (d) Final rate of interest for two pens

= x + y + xy

100

= 10 1010 10 21%

100´+ + =

Let principal be P.121P 12100100

Þ ´ =

P = 100 × 100 = ̀ 10000126. (d) A’s 2 days work = B’s 3 days work.

A complete a work in 8 days

A’s 1 days work = 18

days

A’s 2 days work = 1 128 4´ = work.

Now, B’s 3 days work = 14

work

B’s 1 days work = 1 work

12\ B’s will take 12 days to complet the work.

127. (d) Sum of the angle of a triangle = 180°Þ 2x° + 3x° + 5x° = 180°Þ 10x° = 180°

x° = 18°Angle are = 36°, 54°, 90° So, this is rightangles triangle.

yoursmahboob.w

ordpress.com

369SSCCGLSolvedPaper

128. (d) According to question2 x 55 x 6

+=

+Þ 12 + 6x = 25 + 5x

x = 25 – 12 = 13.129. (c) Let angles are x and y rad.

x – y = 36° ... (i)

x + y = 229

x + y = 22 180 440 1409

°´ = = °p p

...(ii)

Now, on solving each (i) and (ii), we getx = 68°, and y = 52°So, smaller angle = 52°

130. (d) Now, M1d1 = M2d2Þ (4M + 6W) ́ 8 = (2M + 9W) ́ 8Þ 4M + 6W = 2M + 9WÞ 2M = 3W

1 M = 3

W2

Now, 4M + 6W = 34 W 6W 12W2

´ + =

Here 12W complete a work in 8 days.So, let 18 women complete a work in x days12W ´ 8 = 18W ´ x

x = 1612 8 days318

´ =

= 15 days3

131. (d) Given that24

121 7x

x+ =

Þ12

121 7x

x+ =

On cubing both sides we getÞ

36 12 1236 12 121 1 13. 343x x x

x x xæ ö+ + + =ç ÷è ø

Þ 36

361 3 7 343x

x+ + ´ =

Þ 72

361x

x+

= 343 – 21 = 322

132. (b) 5 9 5x y+ = ...(i)3 3125 729 120x y+ = ...(ii)

Now cube both sides of equation (i), we get

Þ 3 3(5 9 ) (5)x y+ =Þ

3 3125 729 135 (5 9 ) 125x y xy x y+ + + =

Þ 3 3125 729 135 5 125x y xy+ + ´ =Now, put the value of equation (i)120 + 135 xy × 5 = 125Þ 135 × 5xy = 125 – 120Þ 135 × 5xy = 5

xy = 5 1

135 5 135=

´133. (*) 4 M = 8w

1M 2w\ =

Now, 6M + 12 w = 6 × 2w + 12w= 12w + 12w= 24 w

Now, M1 d1 = M2 d2Þ 8w × 15 = 24 w × d2

Þ \ d2 = 8 15

24w´

= 5 days

None of the options is correct

134. (b) 2 2 2sin 22 sin 68 cot 30° + ° + °

= 2 2sin (90 68) sin 68 cot 230- ° + ° + °

= 2 2 2cos 68 sin 68 cot 30° + ° + °

= 21 ( 3)+ = 1 + 3 = 4

135. (c) 10 20 25%10 18 18

100 90 72 54¯ ¯ ¯- - -

¾¾¾® ¾¾¾® ¾¾¾®

Required single discount = (100 – 54)% = 46%136. (a) tan(4 50 ) cot(50 )q - ° = ° - q

Þ tan (4q – 50°) = tan (90 – (50° –q))Þ 4q – 50° = 90 – 50° + qÞ 3q = 90°

q = 30°.137. (a) Now according to question.

100 × CP = 60 × SP

Þ100 560 3

SPCP

= =

yoursmahboob.w

ordpress.com

370 SSCCGLSolvedPaper

Both sides substract 1

Þ5 3 2

3 3SP CP

CP- -= =

Percentage loss = 2 2100 66 %3 3´ =

138. (c) 5 sinq = 3

sinq = 35

b = 2 2(5) (3)- = 16 4=

3tan4

q = ,4cos5

q =

5sec4

Þ q =

Now,

5 3sec tan 2 14 4

5 3sec tan 8 44 4

-q- q = = =q + q +

139. (d) Let the number be x

Þ3 1 74 6

x x= +

Þ3 1 74 6

x x- =

= 9 2 7

12x x+ - =

7 712

x = \ x = 12

Now, 53 of x =

5 12 20.3´ =

140. (d) For 20 natural last odd number= 1 + (20 – 1) × 2 = 39

Arithmetic mean of odd number = 1

2n +

= 40 202

=

141. (b)

75 m

B C

A

q

cot BCAB

q =

Þ8

15 75BC=

Þ BC = 40

AC = 2 2(BC) AB+

AC = 2 2(75) (40)+

= 5625 1600+= 7225 = 85 cm.

142. (d)

M5 km.

OE

N

W

S

L12 km

LM = 2 2(OL) (MO)+

= 2 2(12) (5)+

= 144 25 13km.+ =143. (b) Let total mark of Examination be x.

Þ36

100x´ = 190 + 35

Þ36 225

100x´ =

x = 625

144. (d) Speed = 590

18´ = 25 m/sec

Time = 18025

dv

=

= 7.2 sec.145. (d) Now, Let discount be x%

(100 )975100

x-´ = 897

Þ 100 – x = 89700

975

Þ x = 97500 89700

975-

= 7800 8%975

=

yoursmahboob.w

ordpress.com

371SSCCGLSolvedPaper

146. (b) Secq + tanq = p ...(i)We know that sec2q – tan2q = 1Þ (secq – tanq) (secq + tanq) = 1Now, Put the value of secq + tanq = p

secq – tanq = 1p ...(ii)

Now, solving eqn. (i) and (ii)

1 1sec P2 p

æ öq = +ç ÷

è ø147. (a) Given P = 99

then P(P2 + 3P + 3)= P((P + 1)2 + P + 2)= 99 ((99 + 1)2 + 101)= 99 × (10000 + 101)= 999999

148. (a) Given x = 2then x3 + 27x2 + 243x + 631= 8 + 108 + 486 + 631= 1233

149. (d) A

B CD

EF

Area of DEFBW = BD × EDArea of trapezium

1 (AC FD) DE2

CAFD = + ´W

Here AC = 2 AE = 2FD

1 3FD DE2

CAFD = ´W

Now Area of ||

Area of trapeziumW

W

DEFBCAFD =

32

BD EF

FD DE

´

´

= 32

BD ED

BD DE

´

´ ´=

23

150. (d) RatioNow, 2 81B A2- =

45

AB

= Þ54

BA

=

Squaring both sides, we get

Þ2

22516

BA

=

both sides substract 1

Þ2 2

225 16 9

16 16B A

A- -= =

Þ 281 9

16A=

Þ A2 = 16 × 9

12A\ =151. (a) Arrive means to reach a destination by

movement or progress; happen denotes anevent coming to pass (in time, so that it isreal and actual at some time); occur alsomeans (of an event) coming to pass (in time,so that it is real and actual at some time);come means happen or arrive, example: Dawncomes early in June.

152. (c) The phrase call up here denotes having andbringing to mind a memory of something;bringing back knowledge from memory.

153. (c) Up (one's) sleeve means hidden but readyto be used: He still has a few tricks up hissleeve.

154. (d) The phrase day by day means gradually andprogressively; "his health is weakening dayby day."

155. (b) The police could only fire the mob when itturned violent.

156. (c) Board means food or meals considered as awhole: board and lodging.

157. (d) Peruse means examine or consider withattention and in detail. "Please peruse thisreport at your leisure."

158. (a) Spume means foam or froth on the sea. Lava,poison and spit do not correspond to thegiven word.

159. (d) 160. (a) 161. (d) 162. (b) 163. (d)164. (d) Nor doesn't necessarily have to appear in a

sentence with the word "neither." "Nor" canstart a sentence. Ex: if you've just mentionedthat you don't usually wake up at 6 a.m. andyou want to continue being negative, youcan start another sentence with "nor": "Nordo I like to wake up at 5 a.m."

yoursmahboob.w

ordpress.com

372 SSCCGLSolvedPaper

165. (a) In the first conditional sentencesthe structure is: If + simple present simplefutureIf this thing happens, that thing will happen.Ex: If you don't hurry, you will miss the train.If it rains today, you will get wet.

166. (a) The second part of the sentence clears theambiguity with 'approval.' Hence, noddedis the right answer.

167. (b) The world is facing a crisis is a meaningfulsentence.

168. (c) The verb insist may be used as follows: Sheinsisted on her being innocent or sheinsisted that she was innocent.

169. (b) She couldn't help but laugh does not needany improvement.

170. (a) I took some grapes for my mother (subject'I' + verb + immediate object grapes).

171. (c) If the verb is in the past simple we use did.Ex: They went to the theatre, didn't they?She studied in New York, didn't she?

172. (d) Both the teams played a fair game is thecorrect usage.

173. (b) Endure means put up with something orsomebody unpleasant. Ex: The new clerkhad to endure a lot of unprofessionalremarks.

174. (c) Vacillation means indecision in speech oraction while steadfastness means fixed orunchanging.

175. (b) Placid means calm and peaceful, with littlemovement or activity while stormy meanscharacterized by violent emotions orbehaviour.

176. (a) Effeminacy means the traits in a human boyor man that are more often associated withfeminine nature, behaviour, mannerisms,style or gender roles which are opposite tothe manliness.

177. (d) Something that you say which means it isbetter to keep what you have than to risklosing it by trying to get something better IfI were you I'd accept the money they'reoffering. After all, a bird in the hand is worthtwo in the bush.

178. (b) Luggage means suitcases, trunks, etc,containing personal belongings for ajourney; baggage. It is never used in pluralform.

179. (b) Just outside my house is a playground forschool boys and girls. Playground is insingular, hence, is.

180. (c) Mule means the offspring of a donkey anda horse (strictly, a male donkey and a femalehorse) which is not tall.

181. (a) Who should be replaced with 'whom' as itsignifies the objective case of 'who.'

182. (c) Voluntary means of your own free will ordesign; done by choice; not forced orcompelled.

183. (a) Fugitive is one who is sought by lawofficers; someone trying to elude justice.

184. (d) Dexterous is the one who is skilful inphysical movements; especially of thehands.

185. (b) Improvident means not provident; notproviding for the future.

186. (c) Laudable means worthy of high praise.187. (d) An experienced person who has been

through many battles; someone who hasgiven long service is called a veteran.

188. (b) The branch of anthropology that studiesprehistoric people and their cultures throughtheir material remains is called archaeology.

189. (c)190. (b)191. (d) Compensate means here make payment to;

compensate192. (d) Leave somebody high and dry means leave

him alone to do the work.193. (d) If you are making a mountain out of a

molehill, it means you are exaggeratingthings, blowing them out of proportion.

194. (c) The idiom spills the beans means revealingthe secret information.

195. (c) A snake in the grass means a hidden enemy.196. (d) 197. (a)198. (c) 199. (a)200. (a)

yoursmahboob.w

ordpress.comPART-A : GENERAL INTELLIGENCE & REASONING

Directions (Qs. 1-3): Select the related word/Letters/numbers from the alternatives.1. Pig : Sty : : Dog : ?

(a) Burrow (b) Hives(c) Shed (d) Kennel

2. BDGK : ACFJ : : ? : DFIM(a) FKGH (b) EGJN(c) EGIK (d) FGHI

3. 01 : 36 : : 02 : ?(a) 48 (b) 49 (c) 69 (d) 70

Directions (Qs. 4-6 ): Find the Odd Word/Letters/Numbers from the given alternatives4. (a) House (b) Stable

(c) Hut (d) Cottage5. (a) BDGK (b) JLOS

(c) NPSW (d) MORU6. (a) 484 (b) 529

(c) 625 (d) 5667. Amongst the given responses choose the

meaningful logical series.1. College 2. Infant3. Child 4. School5. Youth(a) 2,4,3,1,5 (b) 2,3,4,1,5(c) 2,4,3,5,1 (d) 2,3,4,5,1

Directions (Qs. 8-9): A series is given,with one termmissing. Choose amongst the given responses choosethe meaningful one.8. CUS, DVT, EWU, ____

(a) FXV (b) VXF(c) XFV (d) XVF

9. 206, 221, 251, 296, ?, 431(a) 326 (b) 356 (c) 311 (d) 341

10. If A is the mother of B and K, D is the husband ofA. E is the son of D’s brother. What is the relationof A with E(a) Mother in law (b) Sister in law(c) Aunt (d) Sister

11. Kathir is senior of Ganesh. Ganesh is senior ofApparu. Apparu is junior of Raju. Raju is juniorof Ganesh. Who is the most senior?(a) Ganesh (b) Raju(c) Kathir (d) Apparu

12. Select the word which cannot be formed usingthe letters of the given word REJUVENATION(a) JUTE (b) NATION(c) REVISION (d) NATIVE

13. If A = 1, HAT = 29, then PAN = ?(a) 21 (b) 31(c) 41 (d) 28

14. If ‘x’ means ‘addition’, ‘–’ means ‘division’, ‘/’means ‘subtraction’ and ‘+’ means ‘multiplication’,then which of the equation is correct(a) 25 + 10 – 5/ 10 × 3 = 43(b) 25 –10 × 5+ 10/3 = 72(c) 25 ×10/5 + 10 – 3 = 12(d) 25/10 + 5 × 10/3 = 18

15. In this question, some equations are solved onthe basis of a certain system. On the same basisfind out the correct answer from amongst thefour alternatives for the unsolved equation.53 – 34 = 533465 – 46 = 645675 – 24 = ?(a) 7542 (b) 7524 (c) 7452 (d) 7254

16. Select the missing images from the givenresponses:

6 5 264 7 32? 9 44

(a) 8 (b) 31 (c) 32 (d) 3617. Deepak starts walking straight towards east. After

walking 75 metres, he turns to the left and walks25 metres straight. Again he turns to the left,walks a distance of 40 metres straight, again heturns to the left and walks a distance of 25 metres.How far is he from the starting point?(a) 35 m (b) 50 m (c) 115 m (d) 140 m

SSC Combined Graduate Level (CGL) Solved Paper MORNING SHIFT 8 SEPT, 2016

yoursmahboob.w

ordpress.com

374 SSC CGL SolvedPaper

18. Consider the given statement/s to be true anddecide which of the given conclusions/assumptions can definitely be drawn from thegiven statement.Statements : 1. All fish are tortoise.

2. No tortoise is a crocodile.Conclusion : I. No crocodile is a fish.

II. No fish is a crocodile(a) Only conclusion I follows(b) Only conclusion II follows(c) Both conclusions I & II follows(d) Neither conclusion I nor II follows

19. How many triangles can be found out from thefollowing figure:

(a) 17 (b) 21 (c) 24 (d) 2520. Identify the diagram that best represents the

relationship among classes given below:Universe,Stars,Sun

(a) (b)

(c) (d)

21. Which answer figure will complete the pattern inthe question figure?

(a) (b)

(c) (d)

22. Select the one in which the question figure ishidden/embedded.

(a) (b)

(c) (d)

23. A piece of paper is folded and cut. From thefigures given, indicate how it will appear whenopened

(a) (b)

(c) (d)

yoursmahboob.w

ordpress.com

375SSC CGL SolvedPaper

24. Mirror is placed on the line MN, then which ofthe answer figures is the right image?

(a) (b)

(c) (d)

25. In the question, a word is represented by onlyone set of numbers as given in any one of thealternatives. The sets of numbers given in thealternatives are represented by two classes ofalphabets as in two matrices given below. Thecolumns and rows of Matrix I are numbered from0 to 4 and that of Matrix II are numbered from 5to 9.A letter from this matrix can be representedfirst by its row and next by its coloum ex “A” canbe represented by 03,10,etc and “P” can berepresented by 55, 67, etc. Similarly, you have toidentify the set for the word “REST”

(a) 96, 33, 44, 87 (b) 58, 21, 85, 75(c) 89, 40, 31, 56 (d) 77, 10, 55, 68

PART-B : GENERAL AWARENESS

26. CENVAT is related to which of the following ?(a) Sales Tax (b) Excise Duty(c) Custom Duty (d) Service Tax

27. Malthusian theory is associated with which ofthe following ?(a) Poverty (b) Employment(c) Diseases (d) Population

28. An amendment of the constitution may beinitiated.(a) by introduction by the President of India.(b) by introduction of a Bill in Rajya Sabha.(c) by the Governors of States.(d) by the introduction of a bill in either House

of Parliament.29. Kamarup is an ancient name of which region of

India?(a) Bihar (b) Rajasthan(c) Karnataka (d) Assam

30. Which is the parameter for the economicdevelopment ?(a) Per capita monetary income(b) National income(c) Per capita rural income(d) Population

31. Equinox occurs when the sun is vertically above(a) Tropic of Capricorn(b) Tropic of Cancer(c) Poles(d) Equator

32. Which battle led to the downfall of theVijayanagar empire?(a) Battle of Takkolam (b) Battle of Talikota(c) Battle of Kanwah (d) Battle of Panipat

33. The antibiotic penicillin is obtained from whichof the following ?(a) synthetic process(b) a bacterium(c) fungus(d) virus infected cells

34. Among the world oceans, which ocean is havingthe widest continental shelf ?(a) Antarctic ocean (b) Arctic Ocean(c) Indian Ocean (d) Atlantic ocean

35. Data in database at a particular point of time iscalled as?(a) Intension (b) Extension(c) Back up (d) Application

36. Which is largest peninsular river in India?(a) Krishna (b) Godavari(c) Cauvery (d) Mahanadi

37. Red soil is normally found in India in whichregions?(a) Eastern Region only(b) Southern Region only(c) Eastern & Southern part of the Deccan

Plateau(d) None of these

yoursmahboob.w

ordpress.com

376 SSC CGL SolvedPaper

38. Who introduced Mansabdari system in India?(a) Babur (b) Humayun(c) Akbar (d) Jahangir

39. Limestone is a raw material used by whichindustry?(a) Aluminium (b) Fertilizers(c) Cement (d) Petrochemicals

40. The Directive Principles of State Policy has beenadopted from which Constitution?(a) U.S. Constitution(b) British Constitution(c) Irish Constitution(d) French Constitution

41. Which of the following is indicated by the colourof a star ?(a) weight (b) distance(c) temperature (d) size

42. Atomic number of an atom gives the number ofwhich of the following ?(a) electrons(b) protons(c) neutrons(d) neutrons and protons

43. DPT vaccine is categorized as which of thefollowing ?(a) Anti viral vaccine(b) Anti protozoan vaccine(c) Anti rickettsial vaccine(d) A combined vaccine

44. Which is the highest quality of hard coal?(a) Anthracite (b) Bituminous(c) Lignite (d) Peat

45. For which one of the following, ‘Diodes’ aregenerally used for?(a) Rectification (b) Amplification(c) Modulation (d) Filtration

46. An anemometer measures which of the following ?(a) Speed of light(b) Speed of wind(c) Speed of water current(d) Speed of satellites

47. ’Mission Indradhanush’ Campaign in India isassociated with?(a) Nutrition to Pregnant Women(b) Awareness of Diabetes(c) Eradication of blindness(d) Vaccination of children

48. Who is the new Director of Film and TelevisionInstitute of India (FTII) ?(a) Prashant Pathrabe(b) Amol Palekar(c) Shabana Azmi(d) Bhupendra Kainthola

49. Which cell disorder in our body is responsiblefor colour blindness?(a) WBC (b) Cone cell(c) Rod Cell (d) Neuron

50. Who is the recipient of Man Booker InternationalPrize 2016 for the novel ‘The Vegetarian’?(a) Orhan Pamuk (b) Han Kang(c) Elena Ferrante (d) Tonkin

PART-C : QUANTITATIVE APTITUDE

51. A can finish a work in 7 days . B can finish thesame work in 9 days . The days required to finishthe work by both of them together.

(a)15116 (b)

15216 (c)

15316 (d)

15416

52. If the successive discounts be 20% , 10% and5%, then the single equivalent rate of discount is(a) 31.6% (b) 31.5%(c) 31% (d) 31.4%

53. The ratio of number of boys and girls in a schoolof 720 students is 7 : 5 . How many more girlsshould be admitted to make the ratio 1 : 1 ?(a) 90 (b) 120 (c) 220 (d) 240

54. The selling price of 6 bananas is equal to thecost price of 8 bananas. Then the percentage ofprofit is

(a) 20 (b) 1333

(c) 25 (d) 30

55. A basket contains 300 mangoes. 75 mangoes weredistributed among some students. Find thepercentage of mangoes left in the basket(a) 70% (b) 72% (c) 76% (d) 75%

56. A man goes to a place on bicycle at speed of16 km/hr and comes back at lower speed . If theaverage speed is 6.4 km/hr in total , then thereturn speed (in km/hr) is(a) 10 (b) 8 (c) 6 (d) 4

57. The simple interest on a certain sum of money atthe rate of 5% per annum for 8 years is Rs. 840.Rate of interest for which the same amount ofinterest can be received on the same sum after5 years is(a) 7% (b) 8% (c) 9% (d) 10%

yoursmahboob.w

ordpress.com

377SSC CGL SolvedPaper

58. If 1 3CC

+ = , then the value of 77

1( – 3) + isCC

(a) 2 (b) 0 (c) 3 (d) 1

59. If 12x + = 1,

4xthen the value of 2

2

1x + is64x

(a) 0 (b) 1 (c) 1/4 (d) 260. O is the orthocentre of ΔABC , and if ÐBOC = 110°

then ÐBAC will be(a) 110° (b) 70° (c) 100° (d) 90°

61. BE and CF are two altitudes of a triangle ABC.If AB = 6 cm , AC = 5 cm and CF = 4 cm , then thelength of BE is(a) 4.8 cm (b) 7.5 cm (c) 3.33 cm (d) 5.5 cm

62. The circular measure of the included angleformed by the hour hand and minute hand of aclock at 3 PM will be(a) p/4 (b) p/3(c) 5p/12 (d) p/2

63. The value of 3

3

(2.3) 0.027(2.3) – 0.69 0.09

++

(a) 2 (b) 2.27(c) 2.33 (d) 2.6

64. The average of 100 observations was calculatedas 35.It was found later, that one of theobservation was misread as 83 instead of 53. Thecorrect average is :(a) 32.7 (b) 34.7(c) 35.7 (d) 36.7

65. If – 1,x y = 17x y+ = then ?xy =

(a) 72 (b) 72 (c) 32 (d) 24

66. If 13 ,3

x = + then the value of 126–42

xæ öç ÷ç ÷è ø

1–2 3–

3

xx

æ öç ÷ç ÷ç ÷è ø

is

(a)35

6(b)

2 33

(c) 5/6 (d) 2/3

67. In a D ABC, BC is extended upto D :

120 ,ACDÐ = °1 .2

B AÐ = Ð Then AÐ is

(a) 60° (b) 75° (c) 80° (d) 90°

68. O is the centre of a circle and AB is the tangentto it touching at B . If OB = 3 cm. and OA = 5 cm,then the measure of AB in cm is(a) 34 (b) 2 (c) 8 (d) 4

69. If sin 31° = .xy The value of sec 31° – sin59° is

(a)2

2 2–x

y y x(b)

2

2 2–

–x

y y x

(c)2

2 2–

yy x

(d) –2

2 2–

x

y y x70. The area of the largest sphere ( in cm2 that can

be drawn inside a square of side 18 cm is(a) 972 p (b) 11664 p(c) 36 p (d) 288 p

71. A tower is 50 meters high.Its shadow is x metresshorter when the sun’s altitude is 45° than whenit is 30°. The value of x in metres is

(a) 50 3 (b) ( )50 3 –1

(c) ( )50 3 1+ (d) 50

Directions (Qs. 72-75): The following piechart showsthe study time of different subjects of a student in aday . Study the pie chart and answer the followingquestions.

Chemistry15% Mathematics

30%Physics

20%

Other10% History

15%

Geography10%

72. The time spent to study history and chemistry is4 hours 30 minutes, Then the student studiedphysics for(a) 1 hour 30 min (b) 2 .9 hour ( approx.)(c) 2 hours (d) 3 hours

73. If the student studied chemistry for 3 hours, thenhe / she studied geography for(a) 1 hour (b) 2 hours(c) 1 hour 30 minutes (d) 2 hour 30 minutes

yoursmahboob.w

ordpress.com

378 SSC CGL SolvedPaper

74. If the student studied 10 hours in a day , thenhe/ she studied mathematics for.(a) 3 hour (b) 10/3 hour(c) 1/3 hour (d) 3/10 hour

75. Instead of 10% , if the student spends 15% tostudy other subjects and the time is taken fromthe time scheduled to study mathematics and ifhe/ she used to study 20 hours per day , then thedifference of time for studying mathematics perday is(a) 30 minutes (b) 45 minutes(c) 1 hour (d) 1 hour 30 minutes

PART-D : ENGLISH COMPREHENSION76. In the following question, out of the four

alternatives, choose the word which bestexpresses the meaning of the given word andclick the button corresponding to it.CONSCRIPT(a) DRAFT (b) DRAW(c) ENCIRCLE (d) SUBSCRIBE

77. In the following question, out of the fouralternatives, choose the word which is oppositein meaning to the given word and click the buttoncorresponding to it.CAPTIVITY(a) SLAVERY (b) PERMISSION(c) FREEDOM (d) LIMITATION

78. Four words are given, out of which only oneword is spelt correctly. Choose the correctly speltword and click the button corresponding to it.(a) Vivacious (b) Vivascious(c) Vivasious (d) Vivacouse

Directions (79-81): In the following questions, onepart of the sentence may have an error. Find outwhich part of the sentence has an error and click thebutton corresponding to it. If the sentence is freefrom error, click the “No error” option.

79. The job for drawing water (A)/ from the villagewell is usually (B) / carried out by the womenand young girls. (C) / No Error (D).(a) A (b) B(c) C (d) D

80. I remember him as (A) / someone who was a lotnicer (B) / than circumstances warranted. (C) /No Error (D)(a) A (b) B(c) C (d) D

81. Several days passed before (A) / Jeff workedover enough (B) / courage to return to the house.(C) / No Error (D)(a) A (b) B(c) C (d) D

Directions (Qs. 82-83): The sentences given withblanks are to be filled with an appropriate word(s).Four alternatives are suggested for each question.For each question, choose the correct alternativeand click the button corresponding to it.

82. The starving man _____ able to walk.(a) can barely (b) were barely(c) are barely (d) was barely

83. The new railway counter at Dwarka should_____ all travellers.(a) benefit (b) reward(c) reserve (d) provide

84. My younger brother _____ my grandfather.(a) looks on (b) looks after(c) looks to (d) looks of

Directions (Qs. 85-87): In each of the questions, fouralternatives are given for the Idiom/Phrase. Choosethe alternative which best expresses the meaning of theIdiom/Phrase and click the button corresponding to it.

85. To keep in abeyance(a) In a state of permanence(b) In a state of emergency(c) In a state of suspension(d) In a state of revision

86. To be in a fix(a) In pain(b) In distress(c) Depressed(d) In a difficult situation

87. To break the ice(a) Made people angry(b) Made people laugh(c) Made people excited(d) Made people relaxed and comfortable

yoursmahboob.w

ordpress.com

379SSC CGL SolvedPaper

Directions (Qs. 88-90): Out of the four alternatives,choose the one which can be substituted for the givenwords/sentences and click the button correspondingto it.88. An abattoir is _____ .

(a) a place where animals are slaughtered(b) a place where abbots stay(c) a title of respect given to a priest or abbot(d) a place where animals are worshipped

89. A man with abnormal habits(a) Eccentric (b) Frantic(c) Idiotic (d) Sulky

90. Words inscribed on the tomb(a) Epigraph (b) Epigram(c) Epitaph (d) Elegy

Directions (Qs. 91-95): A sentence/a part of thesentence is underlined. Four alternatives are givento the underlined part which will improve thesentence. Choose the correct alternative and clickthe button corresponding to it . In case noimprovement is needed, click the buttoncorresponding to “No improvement”.91. Sunita’s sister had ribbons on her hair.

(a) in (b) over(c) through (d) No improvement

92. She has an ability for Mathematics.(a) attitude (b) aptitude(c) altitude (d) No improvement

93. I was impressed of it.(a) by (b) on(c) for (d) No improvement

94. Only high officials have access with the President.(a) to (b) for(c) from (d) No improvement

95. No other snake is as poisonous as this one.(a) are as poisonous as this one(b) is more poisonous as this one(c) is poisonous than this one(d) No improvement

Directions (Qs. 96-100): A passage is given with 5questions following it. Read the passage carefullyand choose the best answer to each question out ofthe four alternatives and click the buttoncorresponding to it.

The first working steam powered vehicle wasdesigned and most likely built by Ferdinand Verbies,a Flemish member of a Jesuit mission in China around1672. It was a 65 cm long scale-model toy for theChinese Emperor, that was unable to carry a driver ora passenger. It is not known if Verbiest’s model wasever built. Nicolas-Joseph Cugnot is widely creditedwith building the first full-scale, self-propelledmechanical vehicle or automobile in about 1769, healso created a steam-powered tricycle. He constructedtwo steam tractors for the French Army, one of whichis preserved in the French National Conservatory ofArts and Crafts. His inventions were howeverhandicapped by problems of water supply andmaintaining steam pressure. In 1801, RichardTrevithick built and demonstrated his Puffing Devilroad locomotive, believed by many to be the firstdemonstration of a steam-powered road vehicle. Itwas unable to maintain sufficient steam pressure forlong periods. Sentiment against steam-powered roadvehicles led to the Locomotive Acts of 1865. In 1807Nicephore Niepce and his brother Claude probablycreated the world’s first internal combustion enginewhich they called Pyreolophore.96. The first full-scale, working steam powered

tricycle was built by:(a) Verbiest (b) Cugnot(c) Trevithick (d) Niepce

97. Cugnot built steam tractors for:(a) The Chinese Emperor(b) The French Army(c) The Jesuit mission(d) The French Conservatory

98. The problem with Trevithick’s Puffing Devil was:(a) Its incapability to carry a driver or a

passenger(b) With the water supply(c) Its inability to maintain steam pressure(d) Its combustion engine

99. What is meant by “Sentiment” in the context ofthe given paragraph?(a) Depression (b) Fascination(c) Celebration (d) Resentment

100. The Pyreolophore was ?(a) A self-propelled mechanical vehicle(b) A steam-powered tricycle(c) A steam tractor(d) The name of the world’s first internal

combustion engine

yoursmahboob.w

ordpress.com

380 SSC CGL SolvedPaper

1. (d) Pig lives in Sty in the same way Dog livesin Kennel.

2. (b)

–1–1

–1–1

B D G K A C F J

–1–1

–1–1

E G J N D F I M:

3. (b) 01 : 36 : : 02 : ____(1 + 5)2 = 36 (2 + 5)2 = 49

4. (b) Stabble is the place where animal (Horse)lives where as all other are Home of people

5. (d) Only MORU does not follow the pattern of+2, +3, +4 after every Alphabet

6. (d) Only 566 is not a complete square from thegiven options

7. (d) Infant becomes Child and goes to Schoolthan grows up to Youth and goes to Collegeie, 2, 3, 4, 5, 1

8. (a)

+1

+1

+1

+1

+1

+1

+1

+1

+1

C EU WS, UD V T, F X V

9. (b) 206, 221, 251, 296, 356, 431+15 +30 +45 +60 +75

10. (c)Spouse

A is Aunt of E.11. (c) Kathir > Ganesh > Raju > Apparu

Kathir is the senior most12. (c) Revision is the word which cannot be

formed from letters of REJUVENATION as,S is not in it.

13. (c) A = 1, H A T = 29 So, P A N8 + 1 + 20 = 29 16 + 1 + 14 = 31

14. (a) If all the signs are changed as per given in

the question only 25 + 10 – 5×3 = 43

10 will

be satisfied 25 × 10 ÷ 5 – 10 + 3 = 4315. (d)

16. (a) 6 5 264 7 32? 9 44

5 × 4 + 6 = 267 × 4 + 6 = 349 × 4 + x = 44x = 44 – 36x = 8

17. (a)4040

25 25

75He is 75 – 40 = 35 m far from starting point

18. (c)

Fish

Tortoise

Crocodile

conc I: Trueconc II: True

19. (d) 25

20. (b)

SunStar

Universite

HINTS & SOLUTIONS

yoursmahboob.w

ordpress.com

381SSC CGL SolvedPaper

21. (c) 22. (b) 23. (d) 24. (b)25. (b) Code 58, 21, 85, 75 will resemble R E S T

when matched from given two Matrices.26. (c) Concept relating to Central Excise Duty and

Service Tax:- For. a proper understandingwhat is CENVAT, some basic knowledgeabout Central Excise duty, Service Tax etc,is necessary.

27. (d)28. (d) As per the procedure laid out by article 368

for amendment of the Constitution, anamendment can be initiated only by theintroduction of a Bill in either House ofParliament. The Bill, passed by the requiredmajority, is then presented to the Presidentwho shall give his assent to the Bill.

29. (c) 30. (b) 31. (d)32. (b) The Battle of Talikota (26 January 1565), a

watershed battle fought between theVijayanagara Empire and the Deccansultanates, resulted in a defeat ofVijayanagara, and ended in greatlyweakening one of the greatest IndianEmpires originating from Southern Indiabefore the Maratha Empire.

33. (d)34. (b) The largest shelf - the Siberian Shelf in the

Arctic Ocean - stretches to 1,500 kilometers(930 mi) in width. The South China Sea liesover another extensive area of continentalshelf, the Sunda Shelf, which joins Borneo,Sumatra, and Java to the Asian mainland.

35. (b) The data in the database at a particularpoint of time is known as database instanceor database sate or snapshot. The databasestate is also called an extension of theschema.

36. (b) In terms of length, catchment area anddischarge, the Godavari river is the largestin peninsular India and had been dubbedas the 'Dakshina Ganga' - the South Gangesriver.

37. (c)38. (c) The mansabdari system was of Central

Asian origin and it was first introduced byBabur in North India. But it was Akbar whoinstitutionalized it in Mughal military setup and civil administration.

39. (c)40. (c) The concept of Directive Principles of State

Policy was borrowed from the Ir ishConstitution. The makers of theConstitution of India were influenced bythe Irish nationalist movement. Hence, theDirective Principles of the Indianconstitution have been greatly influencedby the Directive Principles of State Policy.

41. (c) 42. (a) 43. (d)44. (a) High grade (HG) and ultra high grade (UHG)

anthracite are the highest grades ofanthracite coal.

45. (a)46. (b) Anemometer is an instrument for measuring

the speed of the wind, or of any current ofgas.

47. (d) Mission Indradhanush was launched byUnion Health Minister J.P Nadda on 25December 2014. It aims to immunize allchildren under the age of 2 years andpregnant women against seven vaccinepreventable diseases namely diphtheria,whooping cough, tetanus, poliomyelitis,tuberculosis, measles and Hepatitis B by2020. In addition to this, vaccines forJapanese Encephalitis (JE), rotavac andHaemophilus influenzae type B (HIB) arealso being provided in selected states.

48. (d) 49. (b) 50. (b)51. (c) A can do work in = 7 day

B can do work in = 9 day

Both can do work in = 9×7 63=9+ 7 16

153 days16

=

52. (a) For 3 discont 20%, 10% and 5%Now take 20% and 10%

20×10100

20 10 –+ = 30 – 2 = 28%

Now take 28 % and 5%

28×5100

28 5 –+ = 33 – 1.4 = 31.6%

yoursmahboob.w

ordpress.com

382 SSC CGL SolvedPaper

53. (b) Ratio of boy and girl = 7 : 5Number of student – 720

Number of Girl = 5 × 720 = 300

120Number of Boy = 720 – 300 = 420Number of girl needed = 420 – 300 = 120

54. (b) Let CP of 8 banana = ` 8CP of 6 banana = ̀ 6Sp of 6 banana = ` 8

Profit % = 2 1×100 = 336 3 %

55. (d) Total mango = 300Distribution = 75

Distributed % = 75 ×100 = 25%

300Percentage left in the basket = 75%

56. (d) Average speed when speed x and y are

given = 2xyx + y

2× × xx +16

166.4=

32 x = 6.4(x + 16)x = 0.2(x + 16)0.8x = 3.2x = 4

57. (b) When P = P, R = 5%, T = 8yr then SI = 840PRT = 840100P × 5×8

= 840100

P = 2100Case II: When P = 2100, R = ?, T = 5 SI = 840

P % 2100× 5× R

= 840100

R = 8%

58. (b)1C + = 3C

1C – 3 + = 0C

C – 3 = 1

–C

(C – 3)7 = 71

–Cæ öç ÷è ø

(C – 3)7 71

Cæ ö+ ç ÷è ø

= 0

59. (a)12x + =14x

dividing Eq. by 2

1 1x + =8x 2 Squaring on both sides

22

1 1 1x + + 2x =64x 8x 4

22

1 1 1x + = –64x 4 4

= 0

60. (b) For Orthocentre ÐBAC = 180 – ÐBOC= 180 – 110 = 70°

61. (a) A B = 6 cm, AC = 5 cm, CF = 4 B E = ?ans A B C

12 AB × F C =

12 A C × B E

6 × 4 = 5 × x245 = x

4.8 = x62. (d) Hour hand covered in 12 hr = 360°

3 hr = 360 × 312 = 90° or

2p

63. (d) 3(2.3)

3(2.3) 0.027– 0.69 0.09

++

Þ 3[(2.3)

2(2.3 0.3)[(2.3) – 0.69 0.09]– 0.69 0.09]

+ ++

Þ 2.3 + 0.3 = 2.664. (b) Average of difference

= 83 – 53 3

10030 0.

100= =

Correct Average = 35 – 0 × 3 = 34 × 7

yoursmahboob.w

ordpress.com

383SSC CGL SolvedPaper

65. (b) – 1x y =

17x y+ =By adding both equations2 18x = , 9x =

9 – 1y =

9 –1 y=

8 y=

x y× = 8 × 9 = 72

66. (c) x 133

= +

x3 1 4

3 3+

= =

Now

Þ126 1x – x –42 2 3x –

3

æ ö æ öç ÷ ç ÷ç ÷è ø ç ÷ç ÷è ø

Þ

4 4 1– 4 23 3 –3 3

3 42–

42

æ ö æ öç ÷ ç ÷ç ÷è ø ç ÷ç ÷è ø

Þ4 43 3

3– 3 –

2æ öæ öç ÷ç ÷è ø è ø

Þ4 – 3 8 – 3

3 32æ ö æ öç ÷ ç ÷è ø è ø

Þ 1 5 5× =

63 2 367. (c) Ð A + Ð B = Ð ACD

1A + A =120°

2Ð Ð

A =1202

Ð A = 80°68. (d) O A2 = OB2 + BA2

A B2 = 52 – 32

= 25 – 9 = 16A B = 4

69. (a) sin31° = xy cos31 =

2yy

2– x

sec31° – sin59sec31° – cos31°

2 2

2 2

––

y xyyy x

2 2 2

2 2

y – y + xy y – x

2

2 2

xy y – x

70. (a)

18cm

Side of square = Diameter of sphere

Radius of sphere cm18

92

=

Area of sphere43 × p × 9 × 9 × 9 = 972p

71. (b) In D A B C AB = tan45°AC

AB = 150 , A C = 50m

In D A B D 50 = tan30°50 + x

50 3 = 50 + x

50 3 – 50 = x

x = 50( 3 –1)

yoursmahboob.w

ordpress.com

384 SSC CGL SolvedPaper

72. (d) History and Chemistry = 15% + 15% = 30%

Time Spent = 14 hr2

= 92

Physics = 20%, hours-pent in physics = x30 20

=9 x220x = ×9 = 3hr60

73. (b) Chemistry% geography%=Chemistryhr geographyhr15 10=3 x

30x = = 215

74. (a)Total % Math %

=Total hr Math hr100 30=10 x

300x = = 3 hr100

75. (c) Other hours study = 15%Then in Math = 30 – 5 = 25%Difference = 5%Total % Difference %

=Total hr Difference hr100 5=20 x

x = 1 hr76. (a) 77. (c) 78. (a)79. (a) 80. (d) 81. (b)82. (d) 83. (a) 84. (b)85. (c) 86. (d) 87. (d)88. (a) 89. (a) 90. (c)91. (a) 92. (b) 93. (a)94. (a) 95. (d) 96. (b)97. (b) 98. (c) 99. (d)100. (d)

yoursmahboob.w

ordpress.com

385SSC CGL SolvedPaper

PART-A : GENERAL INTELLIGENCE & REASONING

Directon : (Qs. 1-3) : Select the related word/letters/numbers from the given atternatives :1. Length : Meter :: Power : ?

(a) Calories (b) Degree(c) Watt (d) Kilogram

2. ZXVT : YWUS : : RPNL : ?(a) XVTS (b) TSQP(c) SQPM (d) QOMK

3. 42 : 20 :: 64 : ?(a) 31 (b) 32(c) 33 (d) 34

Directions (Qs. 4-6) : Find the odd words/letter/numbers from the given alternatives:4. (a) A (b) U

(c) T (d) O5. (a) January (b) April

(c) July (d) May6. (a) 28-21 (b) 31-38

(c) 45-52 (d) 64-717. Arrange the following words as per order in the

dictionary:1. Zeal 2.Zebra 3.Zygote 4.Zinc 5.Zest(a) 1,2,5,3,4 (b) 1,2,5,4,3(c) 1,5,2,4,3 (d) 1,5,2,3,4

8. A series is given, with one term missing. Choosethe correct alternative from the given ones thatwill complete the series:ALZ,CJV,EHR,GFN,?(a) JDJ (b) IEK(c) IDJ (d) JEK

9. A series is given, with one term missing. Choosethe correct alternative from the given ones thatwill complete the series:8, 24, 12, ?, 18, 54(a) 48 (b) 36(c) 29 (d) 21

10. Chanda is the wife of Bharat. Mohan is the sonof Chanda. Ashish is the brother of Bharat andfather of Dhruv. How is Mohan related to Dhruv?

SSC Combined Graduate Level (CGL) Solved PaperMORNING SHIFT 9 SEPT, 2016

(a) Sister (b) Cousin(c) Brother (d) Mother

11. A,B,C,D,E,F are sitting on the round table withequal distances. F is sitting opposite to E andbetween A and D. C is sitting right side of E andopposite to A. Who are the neighbours of A?(a) F and D (b) E and F(c) E and C (d) B and F

12. From the given alternative words, select the wordwhich cannot be formed using the letters of thegiven word:CONCENTRATION(a) CONCERN (b) CONTAINER(c) CONCERT (d) CENTRAL

13. If ‘WZB’ stands for ‘DAY’, how will you code‘MONDAY’?(a) NLMWZB (b) PLOWZB(c) NMLWZB (d) PQRWZB

14. If ‘+’ means ‘/’, ‘/’ means ‘–’,’–’ means ‘x’, ‘x’means ‘+’, then 24 + 8/2–6 × 6 = ?(a) –10 (b) –3(c) 12 (d) 21

15. In this question, some equations are solved onthe basis of a certain system. On the same basisfind out the correct answer from amongst thefour alternatives for the unsolved equation.7 × 6 × 8 = 6788 × 9 × 7 = 9876 × 5 × 7 = 5675 × 4 × 6 = ?(a) 456 (b) 564(c) 645 (d) 654

16. Select the missing number from the givenalternatives:

7 9 88 9 ?4 9 6

60 90 70

(a) 9 (b) 8(c) 7 (d) 6

yoursmahboob.w

ordpress.com

386 SSC CGL SolvedPaper

17. A man starts from a point, walks 5 km towardsEast, turns right and walks 3 km, turns right againand walks. What is the direction he is facingnow?(a) North (b) South(c) West (d) South-west

18. Consider the given statement/s to be true anddecide which of the given conclusions/assumptions can definitely be drawn from thegiven statement.Statement 1: All animals are four-footed.

2: Dog has two legs.Conclusions I: Dog is not an animal.

II: Dog is an animal.(a) If only I follows(b) If only II follows(c) If neither I nor II follows(d) If both I and II follow

19. In the given cubes, which colour is opposite topurple?

Purple

BlueRed

Blue

YellowRed

Red

GreenPurple

Green

OrangePurple

(a) Violet (b) Red(c) Yellow (d) Blue

20. Which of the following diagrams correctlyrepresents elephants, wolves, animals?

(a) (b)

(c) (d)

21. Which answer figure will complete the pattern inthe question figure?

(a) (b)

(c) (d)

22. From the given answer figures, select the one inwhich the question figure is hidden/embedded.

(a) (b)

(c) (d)

23. A piece of paper is folded and cut as shownbelow in the question fifgures. From the givenanswer figures, indicate how it will appear whenopened.

(a) (b)

(c) (d)

yoursmahboob.w

ordpress.com

387SSC CGL SolvedPaper

24. If a mirror is placed on the line MN, then whichof the answer figures is the right image of thegiven figure?

(a) (b)

(c) (d)

25. In the question, a word is represented by onlyone set of numbers as given in any one of thealternatives. The sets of numbers given in thealternatives are represented by two classes ofalphabets as in two matrices given below. Thecolumns and rows of Matrix I are numbered from0 to 4 and that of Matrix II are numbered from5 to 9. A letter from these matrices can berepresented first by its row and next by its column,e.g., ‘A’ can be represented by 01, 13, etc., and‘S’ can be represented by 55, 66, etc. Similarlyyou have to identify the set for the word 'BOTH'.

0 1 2 3 4F A N O II O F A NA N O I FO F I N AN I A F O

01234

5 6 7 8 95 S E H B T6 BTH S E

BB

B

S

S

EE

E

T

TT

HH S

789

MATRIX IIMATRIX I

H(a) 69,67,68,59 (b) 75,22,76,79(c) 88,30,85,86 (d) 58,02,68,65

PART-B : GENERAL AWARENESS26. A ‘Market Economy’ is one which

(a) is controlled by the Government(b) is free from the Government control

(c) in influenced by international market forces(d) All of these

27. The novelist, poet and critic of which languagehas been conferred with Jnanpith Award for2015?(a) Bengali (b) Telugu(c) Odia (d) Gujarati

28. The ‘Cabinet Mission’ of 1946 was led by 1946(a) Lord Linlithgow(b) Lord Mountbatten(c) Sir Pethic Lawrence(d) Sir Mountford

29. Mount Abu is a hill station located in _____ranges.(a) Vindhya (b) Satpuda(c) Aravalli (d) Sahyadri

30. The only perennial river in Peninsular India is_____ .(a) Godavari (b) Kaveri(c) Krishna (d) Bhima

31. Which one of the following forms the base ofvegetable fibres?(a) Cellulose (b) Proteins(c) Fats (d) Oils

32. A particle is thrown vertically upward. When itreaches the highest point, it has __________.(a) a downward acceleration(b) an upward acceleration(c) a downward velocity(d) a horizontal velocity

33. CPU Scheduler is also known as _____ .(a) Job Scheduler(b) Resource Scheduler(c) Short-term Scheduler(d) Process Scheduler

34. During fermentation of sugar, the compoundwhich is always formed is(a) Methyl Alcohol (b) Ethyl Alcohol(c) Acetic Acid (d) Ethylene

35. Which type of democracy do we follow in India?(a) Direct (b) Presidential(c) Representative (d) Dictatorship

36. Social activist Irom Chanu Sharmila, who hasrecently announced to end 16-year old hungerstrike belongs to which State ?(a) Mizoram (b) Manipur(c) Uttar Pradesh (d) Maharashtra

yoursmahboob.w

ordpress.com

388 SSC CGL SolvedPaper

37. The quality or tone of a musical sound producedby a stringed instrument depends on(a) frequency of vibration(b) length of the strings in the instrument(c) Amplitude of vibration(d) waveform of the sound

38. What did Gandhiji meant by ‘Sarvodaya’?(a) Non-violence(b) Upliftment of untouchables or dalits(c) The birth of a new society based on ethical

values(d) Satyagraha

39. When does solar eclipse take place ?(a) When the sun is between the moon and

earth(b) When the earth is between the moon and

sun(c) When the moon is between the sun and

earth(d) When the moon does not lie on the line

joining the sun and earth40. Strait of Gibraltar connects which of the

following?(a) Red Sea-Mediterranean Sea(b) Red Sea-Arabian Sea(c) Atlantic Ocean-Mediterranean Sea(d) Mediterranean Sea-Black Sea

41. The largest solar power plant in India is locatedat(a) Nagercoil (b) Jaisalmer(c) Madhapur (d) Rann of Kutch

42. Regional Rural Banks are sponsored by(a) Nationalised Commercial Bank(b) Reserve Bank of India(c) State Bank of India(d) Government of India

43. Removal of carbon particles from air involvesthe principle of(a) Precipitation (b) Filteration(c) Electrophoresis (d) Sedimentation

44. Which light is least effective in photosynthesis?(a) Blue light (b) Green light(c) Red light (d) Sunlight

45. Which of the following acts as best adsorbent?(a) Charcoal(b) Activated Charcoal(c) Activated Coconut Charcoal(d) Carbon black

46. The maximum fixation of solar energy is done by(a) Bacteria (b) Protozoa(c) Fungi (d) Green plants

47. Harsha moved his capital from _____ to ____.(a) Thanesar, Kanauj (b) Delhi, Deogiri(c) Kamboj, Kanauj (d) Valabhi, Delhi

48. Who is the Chief Economic Advisor to theGovernment of India?(a) Raghu Ram Rajan(b) Arvind Subramanian(c) Rajiv Mahirishi(d) Arvind Pangariya

49. Which type of foreign investment is consideredas unsafe?(a) Foreign Direct Investment (FDI)(b) Portfolio investment(c) NRI deposits(d) External commercial borrowing

50. The Chairman of the Drafting Committee of theConstituent Assembly of India was(a) K.M. Munshi(b) D.P. Khaitan(c) Dr. B.R. Ambedkar(d) T.T. Krishnamachari

PART-C : QUANTITATIVE APTITUDE51. A can do 1/3rd of a work in 5 days and B can do

2/5th of this work in 10 days. Both A and B,together can do the work in

(a)37 days8 (b)

48 days5

(c)39 days8 (d) 10 days

52. The base area of a right pyramid is 57 sq. unitsand height is 10 units. Then the volume of thepyramid is(a) 190 c. units (b) 380 c.units(c) 540 c.units (d) 570 c.units

53. The marked price of a ceiling fan is ` 1200 andthe shopkeeper allows a discount of 5 % on it.Then selling price of the fan is(a) ` 1410 (b) ` 1400(c) ` 1140 (d) ` 1104

54. If 35% of A’s income is equal to 25% of B’sincome, then the ratio of A’s income to B’sincome is(a) 7 : 5 (b) 5 : 7 (c) 4 : 7 (d) 4 : 3

yoursmahboob.w

ordpress.com

389SSC CGL SolvedPaper

55. A and B invest ̀ 3000 and ̀ 2400 respectively ina business. If after one year there is a loss of ` 720,how much loss will B bear? (Loss or Profit is inproportion to their investments)(a) ` 72 (b) ` 320 (c) ` 400 (d) ` 360

56.1

64

% of 11600 +122

% of 800 equals

(a) 100 (b) 200 (c) 300 (d) 40057. At an average of 80 km/hr Shatabdi Express

reaches Ranchi from Kolkata in 7 hrs. Then thedistance between Kolkata and Ranchi is(a) 560 Km (b) 506 Km(c) 560 m (d) 650 m

58. If a3 – b3 = 56 and a – b = 2 then what is the valueof a2 + b2 ?(a) 12 (b) 20 (c) 28 (d) 32

59. If x2 – 3x + 1 = 0, (x 0),¹ then the value of x1x

+ is

(a) 1 (b) 0 (c) 3 (d) 260. X and Y are the mid points of sides AB and AC

of a triangle ABC. If BC + XY=12 units, thenBC – XY is(a) 8 units (b) 4 units(c) 6 units (d) 2 units

61. The length of the base of an isosceles triangle is2x – 2y + 4z, and its perimeter is 4x – 2y + 6z.Then the length of each of the equal sides is(a) x + y (b) x + y + z(c) 2(x + y) (d) x + z

62. If q > 0, be an acute angle, then the value of q in

degrees satisfying 2

2

cos – 3 cos 21

sinq q

q+

= is

(a) 90° (b) 30° (c) 45° (d) 60°

63. If the numbers 3 649 , 20 , 25 are arranged inascending order, then the right arrangement is

(a) 6 3425 < 20 < 9 (b) 3 649 < 20 < 25

(c) 6 34 20 < 25 < 9 (d) 6 3 425 < 9 < 20

64. Visitors to a show were charged ` 15 each on thefirst day, ` 7.50 on the second day, ̀ 2.50 on thethird day and total attendance on three days werein the ratio 2:5:13 respectively. The averagecharge per person for the entire three days is(a) ` 5 (b) ` 5.50 (c) ` 6 (d) ` 7

65. If 2 + a 2 + b 2 + c

+ + = 4,a b c

then the value of

ab + bc+ caabc

is

(a) 2 (b) 1 (c) 0 (d)12

66. If x + y + z = 1, 1 1 1+ + =1

x y z and xyz = – 1, then

x3 + y3 + z3 is equal to(a) –1 (b) 1 (c) –2 (d) 2

67. In ΔPQR, L and M are two points on the sidesPQ and PR respectively such that LM II QR. IfPL = 2cm; LQ = 6cm and PM = 1.5 cm, then MR incm is(a) 0.5 (b) 4.5 (c) 9 (d) 8

68. The length of the radius of a circle with centre Ois 5 cm and the length of the chord AB is 8 cm.The distance of the chord AB from the point O is(a) 2 cm (b) 3 cm (c) 4 cm (d) 15 cm

69. The upper part of a tree broke at a certain heightmakes an angle of 60° with the ground at adistance of 10 m. from its feet. The original heightof the tree was(a) 20Ö3 m. (b) 10Ö3 m.(c) 10 (2 +Ö(3) m. (d) 10(2–Ö (3)m.

70. What would be the compound interest of ̀ 25000for 2 yrs. at 5% per annum(a) 2500 (b) 2562.5(c) 2425.25 (d) 5512.5

71. The value of

cot17°(cot73° cos222° + 2

1cot17°sec 068

is

(a) 0 (b) 1 (c) 2 (d) Ö3Directon : (Qs. 72-75) : The bar graph given indicatesthe income of a firm. Study the graph and answer thequestions given.

02468

101214

Inco

me

in la

khs r

upee

s

Jan Feb MarMonths

Apr May

yoursmahboob.w

ordpress.com

390 SSC CGL SolvedPaper

72. Which period shows a steady increase ofincome?(a) March to May(b) February to April(c) February to May(d) Insufficient data to predict

73. During which month, the ratio of the income tothat of the previous month is the largest ?(a) February (b) March(c) April (d) May

74. The income of May is how many times to that ofFebruarey?(a) 3.25 (b) 4 (c) 3.5 (d) 5

75. The average monthly income of the firm (in lakhrupees) is(a) 7.6 (b) 6(c) 8.8 (d) None of these

PART-D : ENGLISH COMPRESHENSION

Directon : (Qs. 76) : In the following question, out ofthe four alternatives, choose the word which bestexpresses the meaning of the given word.76. PROFLIGATE

(a) Talkative (b) Intelligent(c) Unconventional (d) Wasteful

Directon : (Qs. 77) : In the following question, out ofthe four alternatives, choose the word which isopposite in meaning to the given word and click thebutton corresponding to it.

77. Contemplative(a) Wistful (b) Unreflective(c) Numbed (d) Aroused

78. Four words are given, out of which only oneword is spelt correctly. Choose the correctly speltword.(a) Reconaissance (b) Recconaisance(c) Reconnaissance (d) Reconnaisance

Directon : (Qs. 79-81) : In the following questions,one part of the sentence may have an error. Find outwhich part of the sentence has an error and click thebutton corresponding to it. If the sentence is freefrom error, click the “No error” option.

79. One (A) / should keep (B) / his word. (C) / NoError (D)(a) A (b) B (c) C (d) D

80. If you turn the corner, (A) / you will found ahouse (B) / built of stone. (C) / No Error (D)(a) A (b) B (c) C (d) D

81. His appearance bears (A) / a striking resemblance(B) / to your cousin. (C) / No Error (D)(a) A (b) B (c) C (d) D

Directon : (Qs. 82-84) : The sentences given withblanks are to be filled with an appropriate word(s).Four alternatives are suggested for each question.For each question, choose the correct alternativeand click the button corresponding to it.82. The director congratulated Mr. Varma _____ his

success.(a) on (b) for(c) at (d) about

83. The boys were _____ to hear that we were goingto build a bridge.(a) delight (b) delights(c) delighted (d) delighting

84. The revolting players and the game’sadministrators held _____ discussions to resolvethe payment dispute.(a) unnecessary (b) obligatory(c) preliminary (d) silent

Directon : (Qs. 85-87) : In each of the questions, fouralternatives are given for the Idiom/Phrase. Choosethe alternative which best expresses the meaning ofthe Idiom/Phrase.85. Wild goose chase

(a) An admirable enterprise(b) An honest effort(c) A foolish, unprofitable adventure(d) A powerful effort

86. Smell a rat(a) Detect bad smell(b) Suspect a trick or deceit(c) Misunderstand(d) See hidden meaning

87. A live wire(a) Industrious and brilliant(b) Lively and active(c) Sincere and intelligent(d) Sincere and efficient

yoursmahboob.w

ordpress.com

391SSC CGL SolvedPaper

Directon : (Qs. 88-90) : Out of the four alternatives,choose the one which can be substituted for the givenwords/sentences and click the button correspondingto it.88. The ceremony of crowning a sovereign

(a) Felicitation (b) Promotion(c) Coronation (d) Installation

89. One who tends to patronize, rebuff or ignorepeople regarded as social inferiors and imitate,admire people regarded as social superiors(a) Snob (b) Fob(c) Dandy (d) Freak

90. A room where dead bodies are kept until burial(a) Grave (b) Cemetery(c) Mortuary (d) Pyre

Directon : (Qs. 91-95) : A sentence/a part of thesentence is underlined. Four alternatives are givento the underlined part which will improve thesentence. Choose the correct alternative. In case noimprovement is needed, “No improvement” is theanswer.91. Any able-bodied man is eligible for the job.

(a) Each able-bodied man is(b) Any able-bodied men have been(c) Any able-bodied men are(d) No improvement

92. She found a wooden broken table in the room.(a) wooden and broken table(b) broken wooden table(c) broken and wooden table(d) No improvement

93. The truck rumbled to a halt and a man got outand ran.(a) shrieked at (b) screeched to(c) screamed at (d) No improvement

94. The Professor asked the intruder who was heand why was he in his lecture.(a) who he was and why he was(b) who he was and why was he(c) who he had been and why he had been(d) No improvement

95. My uncle goes to office on bicycle or by foot.(a) by bicycle or by foot(b) by bicycle or on foot(c) on bicycle or on foot(d) No improvement

Directon : (Qs. 96-100) : A passage is given with 5questions following it. Read the passage carefullyand choose the best answer to each question out ofthe four alternatives and click the buttoncorresponding to it.We set out for the gallows. Two warders marched oneither side of the prisoner, with their rifles at theslope; two others marched close against him, grippinghim by his arm and shoulder, as though, at oncepushing and supporting him. The rest of us,magistrates and the like, followed behind. Suddenly,when we had gone ten yards, the procession stoppedshort without any order or warning. A dreadful thinghad happened-a dog, come goodness knows whence,had appeared in the yard. It came bounding amongus with a loud volley of barks, and leapt round uswagging its whole body, wild with glee at finding somany human beings together. It was a large woollydog, half Airedale, half Pariah. For a moment, itpranced round us, and then, before anyone couldstop it, it had made a dash for the prisoner, andjumping up tried to lick his face. Everyone stoodaghast, too taken aback even to grab at the dog.96. What was the tone of the essay at the beginning?

(a) Celebrative(b) Emotionally charged(c) Gloomy(d) Light-hearted

97. How did the arrival of the dog change theatmosphere of the event?(a) It caused the people to scatter(b) It allowed the prisoner to escape(c) It saddened the prisoner even more(d) It stunned everyone present there

98. What was the emotion displayed by the dog?(a) Fear (b) Joy(c) Anger (d) Alarm

99. What was surprising about the actions of thedog?(a) It ran up and down the path(b) It bit the guards(c) It barked at the magistrates(d) It licked the prisoner’s face

100. How did the author respond to the appearanceof the dog?(a) He jumped at the dog and collared it(b) He ignored the dog and pretended it was

not there(c) He was taken aback as the others(d) He yelled at the dog to silence its barking

yoursmahboob.w

ordpress.com

392 SSC CGL SolvedPaper

HINTS & SOLUTIONS

1. (c) As length is measured in meter so power inwatt.

2. (d) –1 –1 –1 –1

R P N L : Q O M K–1–1–1–1

3. (a) 42 : 20 : : 64 : 3 220 × 2 + 2 = 42

\ 31 × 2 + 2 = 644. (c) Only T is consonant else other are vowels.5. (b) April contains 30 days else all contains 31

days.6. (a) 28 – 21 as 28 > 21 else 1st number is higher

than other.7. (b) Zeal, Zebra, Zest, Zinc, Zygote.8. (c)

9. (b) 8 24 12 36 18 54

×3 ×3 ×3¸2 ¸2

10. (b) Chanda Bharat = AshishSpouse

5mMohan Dhruv

Hence, Mohan is cousin of Dhruv.11. (d)

B and F are neighours of A12. (d) There is no L in concentration \ CENTRAL

will not be Formed13. (c) 23 26 2

W Z B

D A Y4 1 25

Forword

Reverse

Similarly

14 15 13 4 1 25M O N D A Y13 12 14 23 26 2N L M W Z B

14. (d) 24 + 8 ÷ 2 – 6 × 6

will be written as 248

– 2 6 6´ + = 3 – 12 + 6 = 3

15. (a) = 6 7 8

Similarly = 4 5 6

16. (b) Here R1 × R2 + R3 ® R4: : 8 × x + 6 = 70

8 x = 64x = 8

17. (c) StartingPoint

Right

Right

He is facing west direction now.18. (a) In statement I it is clearly said all animal

has Four leg and in II that Dog has 2 leg\ Dog is not an animal

19. (c) Purple ® Red, Blue, Yellow, Green, OrangeGreen, Orange, Red, Blue are adjacent topurpleHence, yellow colour is opposite to purple.

20. (a)

21. (a) 22. (b) 23. (d) 24. (c)25. (b) B ® 58, 69, 75, 88, 99

O ® 03, 11, 22, 30, 44T ® 59, 68, 76, 87, 95H ® 57, 65, 79, 86, 98

yoursmahboob.w

ordpress.com

393SSC CGL SolvedPaper

26. (b) 27. (d)28. (c) Cabinet Mission was composed of three

Cabinet Ministers of England Sir PethickLawrence, Secretary of State for India, SirStafford Cripps, President of the Board of Trade;Alexander, the First Lord of the Admiralty. Themission arrived on March 24, 1946. The objectiveof this mission was to Devise a machinery todraw up the constitution of Independent India.Make arrangements for interim Government.Thus the mission was like a declaration of India'sindependence.

29. (c) 30. (b) 31. (a) 32. (a)33. (c) The short-term scheduler (also known as

the CPU scheduler) decides which of the ready,in-memory processes is to be executed (allocateda CPU) after a clock interrupt, an I/O interrupt,an operating system call or another form of signal.

34. (b) 35. (c) 36. (b) 37. (d)38. (c) Sarvodaya is a Sanskrit term meaning

'universal uplift' or 'progress of all'. The term wasused by Mahatma Gandhi as the title of his 1908translation of John Ruskin's tract on politicaleconomy, Unto This Last, and Gandhi came touse the term for the ideal of his own politicalphilosophy.

39. (c) A solar eclipse occurs when the Moonpasses between Earth and the Sun, therebytotally or partly obscuring the image of the Sunfor a viewer on Earth.

40. (c) Approximately 9 miles (14 km) wide at itsnarrowest point, the Strait of Gibraltar is the entrypoint into the Mediterranean Sea from the AtlanticOcean. It is bordered by the continents of Africaand Europe, and the countries of Morocco,Spain, the British colony of Gibraltar, and theSpanish exclave of Ceuta.

41. (c) 42. (a) 43. (c) 44. (b) 45. (c)46. (d) 47. (a) 48. (b) 49. (b) 50. (c)51. (c) A can do work in = 5 × 3 = 15 days

B can do work in = 510 25 days2´ =

Together, 1 1 8 75 3 or 9 days15 25 75 8 8

+ = =

52. (a) Volume of Pyramid1 area of base height3= ´

1 57 103= ´ ´

= 190 c. units

53. (c) Marked price = 1200, Discount % =5% Selling price = ?

95Selling price 1200 1140100= ´ = `

54. (b) 35 % A’s Salary = 25% of B’s Salary

35 25A B100 100

=

A 5B 7

= or 5 : 7

55. (b) A invested = ̀ 3000B invested = ̀ 2400

A invested A's lossB invested B's loss

=

According to Question, (A + B) = ̀ 720A = 720 – B

3000 720 – B2400 B

=

720 B 5B 4- =

9 B = 2880B = 320

56. (b)16 %4

of 1600 + 112 %2

of 800

25 251600 800 200400 200´ + ´ =

57. (a) Distance = Speed × Time= 80 × 7 = 560 km

58. (b) (a – b) = 2, a3 – b3 = 56(a – b)3 = a3 – b3 – 3ab (a – b)8 = 56 – 3ab (2)– 48 = – 6ab\ ab = 8(a – b)2 = a2 + b2 – 2ab4 = a2 + b2 – 1620 = a2 + b2

59 (c) x2 – 3x + 1 = 0Dividing Equation by x

1x – 3 0x

+ =

1x 3x

+ =

yoursmahboob.w

ordpress.com

394 SSC CGL SolvedPaper

60 (b) A

B C

X Y

In DABC Q X and Y are midpoint of AB and AC1XY BC2

2XY BC

=

=

Q

BC + XY = 122XY + XY = 123XY = 12XY = 4BC = 8Hence, BC – XY = 8 – 4 = 4

61. (d) Base side of isosceles D = 2x – 2y + 4zPerimeter = 4x – 2y + 6z

Remaining two sides are P B

2-=

4x – 2y 6z 2x 2y 4z2

2x 2z x z2

+ - + -=

+Þ = +

62. (d)2

2cos 3cos 2 1

sinq- q+ =

q2

2cos cos 2cos 2 1

1 cosq - q - q + =- q

[ ] [ ]( )( )

cos cos 1 2 cos 11

1 cos 1 cosq q- - q-

=- q + q

( )( )( )( )cos 2 1 cos

11 cos 1 cosé ùq- - q- =ê ú- q + që û(2 – cos q) = (1 + cos q)2 cos q = 1

cos q = 12

q = 60°63. (d) 3 649, 20, 25

LCM of 3, 4, 6 = 2424 24 248 6 49 , 20 , 25

24 24 244 8 625 9 20< <i.e. 6 3 425 9 20< <

64. (*) Ratio of ` charged per day × Ratio of no. ofmen = Total ̀ RatioRatio of ` charged per day

= 15 : 7.50 : 2.50 = 6 : 3 : 1Attendance Ratio = 2 : 5 : 13Total ` Ratio = 6 × 2 : 5 × 3 : x 13 × 1

= 12 : 15 : 13Average charge per person

12 15 136 3 1+ +=+ +

40 410

= =

65. (d)2 + a 2 + b 2 + c+ + = 4

a b c2 2 2+ + = 4a b c

1+

2 2 2+ + = 1a b c

1 1 1+ +a b c

2 1é ù =ê úë ûab + bc + ac 1=

abc 2

66. (b) x + y + z = 1, 1 1 1+ + =1x y z

, xyz = –1

xy + yz + zx–1

1=

xy + yz + zx = – 1(x + y + z)2 = 1x2 + y2 + z2 + 2xy + 2yz + 2zx = 1x2 + y2 +z2 + 2(–1) = 1x2 + y2 +z2 = 3x3 + y3 + z3 – 3xyz= (x + y + z) (x2 + y2 +z2 + xy – yz – zx)x3 + y3 + z3 – 3(–1) = (1) [(3 – (–1)]x3 + y3 + z3 + 3 = 4

= 4 – 3 = 167. (b) By Thales theorem

2 1.5=6 x

x = 4.5 cm

yoursmahboob.w

ordpress.com

395SSC CGL SolvedPaper

68. (b) AB = 8, AD = 4, OA = 5 cmOD2 = 52 – 42 = 32

OD = 3

69. (c) Tree

Ð C = 60AB = tan 60°ACAB = 310AB =10 3AB = sin 60°BC10 3 = BC

32

BC = 20

Tree height =

( )20 +10 3

10 2 3+

70. (b) CI = 1 –100

tRP Pé ù+ê úë û

= 2525000 1+ – 25000

100é ùê úë û

= 10525000100

2

–1é ùæ öç ÷ê úè øë û

= 2500011025 –10000

10000é ùê úë û

10255× = 2562.52

71. (a) 2 1cot73° cos 22° +

cot 17°2 –cot 17 sec 68

æ ö°ç ÷ç ÷è ø

cot 17° (tan 17° sin268° + tan 17° cos268°)cot 17° tan17° (sin268° + cos268°)(1) (1) = 1

72. (a) 73. (b) 74. (a) 75. (c) 76. (d)77. (b) 78. (c) 79. (c) 80. (b) 81. (c)82. (a) 83. (c) 84. (c) 85. (c) 86. (b)87. (b) 88. (c) 89. (a) 90. (c) 91. (d)92. (b) 93. (b) 94. (a) 95. (b) 96. (c)97. (d) 98. (b) 99. (d) 100. (c)

yoursmahboob.w

ordpress.com

396 SSC CGL SolvedPaper

PART-A : GENERAL INTELLIGENCE & REASONING

Direction (Qs. 1-3) : Select the related word/letters/numbers from the given alternatives:1. Cytology : Cells :: ? : Birds

(a) Odontology (b) Mycology(c) Etymology (d) Ornithology

2. RIGT : WDBY :: FUSH : ?(a) PKJQ (b) BYWD(c) DXWB (d) QKJR

3. 42 : 56 :: 110 : ?(a) 18 (b) 132 (c) 136 (d) 140

Direction (Qs. 4-6) : Find the odd word/letters/numbers from the given alternatives:4. For the following questions

(a) Rupee (b) Pound(c) Yen (d) Currency

5. For the following questions(a) DAEH (b) KIMP (c) HEIL (d) FCGJ

6. For the following questions(a) 43-6 (b) 28-4 (c) 50-7 (d) 36-5

7. Arrange the following words as per order in thedictionary1. Direction 2. Directed3. Director 4. Directing(a) 1, 4, 3, 2 (b) 2, 4, 1, 3(c) 4, 2, 3, 1 (d) 4, 1, 2, 3

8. A series is given, with one term missing. Choosethe correct alternative from the given ones thatwill complete the series.IGT, JHS, KIR, LJQ, MKP, ?(a) LOP (b) NOP (c) NLO (d) LNO

9. A series is given, with one term missing. Choosethe correct alternative from the given ones thatwill complete the series.6, 10, 18, 34, ?, 130(a) 78 (b) 52 (c) 66 (d) 94

10. Introducing Reeta, Monica said, "She is the onlydaughter of my father's only daughter." How isMonica related to Reeta?(a) Aunt (b) Niece(c) Cousin (d) Mother

SSC Combined Graduate Level (CGL) Solved PaperMORNING SHIFT 10 SEPT, 2016

11. In a row of students, if John , who is 16th fromthe left, and Johnson, who is 8th from the right,interchange their positions, John becomes 33rdfrom left. How many students are there in a row?(a) 38 (b) 39 (c) 40 (d) 41

12. From the given alternative words, select the wordwhich cannot be formed using the letters of thegiven word:ADMISSION(a) MISSION (b) DISMISS(c) MASONS (d) NOMADS

13. If in a certain language TEACHER is coded asQBXZEBO, then how is STUDENT coded in thesame language?(a) PQRBAQK (b) PQRABKQ(c) PQRKBAQ (d) PRKQBAQ

14. If 'x' means addition, '-' means division, '÷' meanssubtraction and '+' means multiplication, thenwhich of the equations is correct?(a) 16 x 5 ÷ 10 + 4 - 3 = 19(b) 16 + 5 ÷ 10 x 4 - 3 = 9(c) 16 + 5 - 10 x 4 ÷ 3 = 9(d) 16 - 5 × 10 ÷ 4 + 3 = 12

15. If 4 × 5 × 2 = 524, 3 × 7 × 2 = 723 and 6 × 8 × 7 = 876then 9 × 4 × 5 = ?(a) 495 (b) 459 (c) 549 (d) 954

16. Select the missing numbers from the givenresponses

43 25 ?21 40 3535 34 28

(a) 36 (b) 46 (c) 40 (d) 2617. Johnson left for his office in his car. He drove

15km towards North and then 10km towards West.He then turned to the South and covered 5km.Further he turned to East & moved 8km. Finallyhe turned right and moved 10km. How far & inwhich direction is he from his starting point?(a) 2 km, West (b) 5 km, East(c) 3 km, North (d) 3 km, South

yoursmahboob.w

ordpress.com

397SSC CGL SolvedPaper

18. Consider the given statement/s to be true anddecide which of the given conclusions/assumptions can definitely be drawn from thegiven statement.Statements:1. All stenographers are lazy.2. Some men are stenographers.Conclusions:I. All lazy people are men.II. Some men are lazy.(a) Only conclusion I follows(b) Only conclusion II follows(c) Both conclusion I and conclusion II follow(d) Neither conclusion I nor conclusion II

follows19. Four positions of a cube are shown below. Which

color is opposite to white color in the givencubes?

RED

WH

ITE

ORANGE

ORANGE

WH

ITE

YELLOW

WHITE

GR

EEN

YELLOW

RED

WH

ITE

BLUE

(a) Orange (b) Blue(c) Red (d) Yellow

20. Which one of the following figures bestrepresents the relationship among Kitchen,House and Garden?

(a) (b)

(c) (d)

21. Which answer figure will complete the pattern inthe question figure?

(a) (b)

(c) (d)

22. From the given answer figures, select the one inwhich the question figure is hidden/embedded

(a) (b)

(c) (d)

yoursmahboob.w

ordpress.com

398 SSC CGL SolvedPaper

23. A piece of paper is folded and cut as shown belowin the question figures. From the given answerfigures, indicate how it will appear when opened.

(a) (b)

(c) (d)

24. If a mirror is placed on the line MN, then which ofthe answer figures is the right image of the givenfigure?

(a) (b)

(c) (d)

25. In the question, a word is represented by onlyone set of numbers as given in any one of thealternatives. The sets of numbers given in thealternatives are represented by two classes ofalphabets as in two matrices given below. Thecolumns and rows of Matrix I are numbered from0 to 4 and that of Matrix II are numbered from 5 to9. A letter from these matrices can be representedfirst by its row and next by its column, e.g., 'P' canbe represented by 02, 13, etc., and 'A' can berepresented by 57, 68, etc. Similarly you have toidentify the set for the word 'GUNS'

0 1 2 3 401234

SS

SS

SU

UU

U

UR

R

RR

R

PP

P

P EE

EEP

5 6 7 8 956789

GG

GG

GL

LL

LL

DD

DD

DNN

NN

NAA

A

AA

E

(a) 88, 23, 59, 33 (b) 66, 40, 67, 11(c) 55, 34, 77, 44 (d) 99, 12, 86, 22

PART-B : GENERAL AWARENESS26. Which one of the following countries is not a

member of the "BRICS" group?(a) Brazil (b) Russia(c) China (d) Indonesia

27. 'Do or Die' is associated with which of themovements in India's freedom struggle(a) Dandi March(b) Non-Cooperation Movement(c) Khilafat Movement(d) Quit India Movement

28. 'Parsec' is the unit measurement of(a) Density of stars(b) Astronomical distance(c) Brightness of heavenly bodies(d) Orbital velocity of giant stars

29. A collective term used by the Jains for their sacredbooks is(a) Prabandhas (b) Agamas(c) Nibandhas (d) Charits

30. Which one of the following is not an instrumentof Fiscal policy?(a) Open Market Operations(b) Taxation(c) Public borrowing(d) Public expenditure

31. At room temperature, the metal that remains liquidis:(a) Mercury (b) Platinum(c) Lead (d) Zinc

32. Which is known as carbolic acid?(a) Phenol (b) Ethanol(c) Acetic acid (d) Oxalic acid

33. On which river is the Tehri dam built?(a) Alakananda (b) Bhagirathi(c) Ganga (d) Hooghly

34. With which country, India has the longestinternational boundary?(a) Nepal (b) Pakistan(c) China (d) Bangladesh

yoursmahboob.w

ordpress.com

399SSC CGL SolvedPaper

35. Which State in India has the largest coastline?(a) Tamil Nadu (b) Andhra Pradesh(c) Gujarat (d) West Bengal

36. Jog falls in Karnataka is located over which river?(a) Kaveri (b) Godavari(c) Sharavati (d) Krishna

37. In which of the following market forms, a firmdoes not exercise control over price?(a) Monopoly(b) Perfect competition(c) Oligopoly(d) Monopolistic competition

38. Which one of the following areas of India iscovered by tropical evergreen forest?(a) Semi-arid areas of Gujarat(b) Eastern Ghats(c) Western Ghats(d) Madhya Pradesh

39. Which one of the following substances isnormally found in urine?(a) Blood proteins (b) Creatinine(c) Red blood cells (d) White blood cells

40. The thymus gland produces a hormone called(a) thyroxine (b) thymosin(c) thyronine (d) calcitonin

41. Blood group AB has(a) No antigen(b) No antibody(c) Neither antigen nor antibody(d) Both antigen and antibody

42. Which vitamins are those, if taken in excess canbe dangerous as they are stored in the body?(a) B Complex (b) E and C(c) B and C (d) A and D

43. Atomic explosion is triggered by(a) thermo nuclear reaction(b) chemical reaction(c) controlled chain reaction(d) uncontrolled chain reaction

44. The phenomenon of change in direction of lightwhen it passes from one medium to another iscalled(a) Propagation (b) Reflection(c) Refraction (d) Dispersion

45. Who has the right to decide whether a Bill is amoney bill or not?(a) Speaker of Lok Sabha(b) Prime Minister(c) President(d) Finance Minister

46. What is the name of portal launched by RBIrecently to check illegal money collection?(a) Sahyog (b) Sahayata(c) Sampark (d) Sachet

47. Which of the following is not an operating system?(a) Android (b) Vista(c) iOS (d) Opera

48. Which pair of the following Indians has beenselected for grant of Raman Magasaysay Award2016?(a) Bezwada Wilson and T.M. Krishna(b) T.M. Krishna and Arvind Adiga(c) Arvind Adiga and Bezwada Wilson(d) T.M. Krishna and Satish Gujral

49. The discretionary powers of a Governor is limitedin(a) Appointment of Chief Minister(b) Dismissal of the Ministry(c) Dissolution of the Legislative Assembly(d) Assent to Bills

50. Who is the first law officer of the country?(a) Chief Justice of India(b) Attorney General(c) Law Minister(d) Solicitor General

PART-C : QUANTITATIVE APTITUDE51. A is twice as good as B and together they finish

a piece of work in 16 days. The number of daystaken by A alone to finish the work is(a) 20 days (b) 21 days(c) 22 days (d) 24 days

52. The successive discount of 15%, 20% and 25%on an article is equivalent to the single discount of(a) 60% (b) 47% (c) 49% (d) 40%

53. The numbers x, y, z are respectively proportionalto 2, 3, 5 and the sum of x, y and z is 80. If thenumber z is given by the equation z = ax – 8, thena is(a) 6 (b) 3/2 (c) 3 (d) 5/2

54. If the ratio of cost price and selling price be 10:11,then the profit percentage is(a) 1% (b) 10% (c) 5% (d) 8%

55. The price of rice has increased by 60%. In orderto restore the original price, the new price mustbe reduced by

(a)1

33 %3 (b)

137 %

2 (c) 40% (d) 45%

yoursmahboob.w

ordpress.com

400 SSC CGL SolvedPaper

56. A man covers a total distance of 100 km onbicycle. For the first 2 hours, the speed was20 km/hr and for the rest of the journey, it camedown to 10 km/hr. The average speed will be

(a)1122 km/hr (b) 13 km/hr

(c) 15 1/8 km/hr (d) 20 Km/hr57. Alipta got some amount of money from her father.

In how many years will the ratio of the moneyand the interest obtained from it be 10:3 at 6%simple interest per annum?(a) 7 years (b) 3 years(c) 5 years (d) 4 years

58. If (x – 2) (x – p) = x2 – ax + 6, then the value of(a – p) is(a) 0 (b) 1 (c) 2 (d) 3

59. When 22x + = 3,x then value of 3

3

1x + + 2x is

(a) 2/7 (b) 7/8 (c) 7/2 (d) 8/760. In a triangle ABC, if ÐA + ÐC = 140° and ÐA +

3ÐB = 180°, then ÐA is equal to(a) 80° (b) 40° (c) 60° (d) 20°

61. If PA and PB are two tangents to a cirlce withcentre O such that ÐAPB = 80°. Then, ÐAOP =?(a) 40° (b) 50° (c) 60° (d) 70°

62. If tan (5x - 10°) = cot (5y + 20°), then the value ofx + y is

(a) 15° (b) 16° (c)1

222

° (d) 24°

63. Which one of the following is the minimum valueof the sum of two integers whose product is 24?(a) 25 (b) 11 (c) 8 (d) 10

64. The average marks obtained by a class of 60students is 65. The average marks of half of thestudents is found to be 85. The average marks ofthe remaining students is(a) 35 (b) 45 (c) 55 (d) 65

65. If x = Öa + 1aÖ

, y = aa

1–ÖÖ , (a > 0), then the

value of x4 + y4 – 2x2y2 is(a) 16 (b) 20 (c) 10 (d) 5

66. If 23x = x +11 – 2, then the value of x3 + 5x2 +

12x is(a) 0 (b) 3 (c) 7 (d) 11

67. Which of the set of three sides can't form atriangle?(a) 5 cm, 6 cm, 7 cm (b) 5 cm, 8 cm, 15 cm(c) 8 cm, 15 cm, 18 cm (d) 6 cm, 7 cm, 11 cm

68. AB is the diameter of a circle with centre O and Pbe a point on its circumference, If ÐPOA = 120°,then the value of ÐPBO is:(a) 30° (b) 60° (c) 50° (d) 40°

69. A pilot in an aeroplane at an altitude of 200 mobserves two points lying on either side of a river.If the angles of depression of the two points be45° and 60°, then the width of the river is

(a)200

200 + m3Ö

æ öç ÷è ø

(b)200

200 – m3Ö

æ öç ÷è ø

(c) 400Ö3 m (d)400

m3Ö

æ öç ÷è ø

70. A solid sphere of radius 9 cm is melted to form asphere of radius 6 cm and a right circular cylinderof same radius. The height of the cylinder soformed is(a) 19 cm (b) 21 cm (c) 23 cm (d) 25 cm

71. An arc of 30° in one circle is double an arc in asecond circle, the radius of which is three timesthe radius of the first. Then the angles subtendedby the arc of the second circle at its centre is(a) 3° (b) 4° (c) 5° (d) 6°

Direction (Qs. 72-75) : Study the Bar diagramcarefully and answer the questions. The Bar diagramshows the trends of foreign direct investment (FDI)into India from all over the World (in Rs. crores).

35

30

25

20

15

10

5

01992 1993 1994 1995 1996 1997

31.36

24.23

10.22

20.16

10.15

5.7

72. The sum of FDI of 1992 and 1993 is(a) 15.58 cr (b) 15.85 cr(c) 15.22 cr (d) 15.65 cr

yoursmahboob.w

ordpress.com

401SSC CGL SolvedPaper

73. The year which exhibited the 2nd highest growthpercentage in FDI in India over the period shownis(a) 1993 (b) 1994 (c) 1997 (d) 1996

74. The ratio of investment in 1997 to the averageinvestment is(a) 2 : 1 (b) 1 : 2 (c) 1:1 (d) 3:1

75. The absolute difference in FDI to India between1996 and 1997(a) 7 . 29 (b) 7. 13 (c) 7 . 16 (d) 7 . 22

PART-D : ENGLISH COMPREHENSIVE76. In the following question, out of the four

alternatives, choose the word which bestexpresses the meaning of the given word andclick the button corresponding to it.FORSWEAR(a) SWEAR (b) OATH(c) ABUSE (d) FORSAKE

77. In the following question, out of the fouralternatives, choose the word which is oppositein meaning to the given word and click the buttoncorresponding to it.INCLEMENT(a) RADICAL (b) MILD(c) HARSH (d) TYRANNICAL

78. Four words are given, out of which only one wordis spelt correctly. Choose the correctly spelt wordand click the button corresponding to it.(a) ANTICEPTIC (b) ANTECEPTIC(c) ANTICEPTIQUE (d) ANTISEPTIC

Direction (Qs. 79-81) : In the following questions,one part of the sentence may have an error. Find outwhich part of the sentence has an error and click thebutton corresponding to it. If the sentence is free fromerror, click the "No error" option.

79. The doctor says that (A)/the patient will recover(B)/in few days (C)/No Error (D)(a) A (b) B (c) C (d) D

80. I do not think that (A)/I can cope up (B)/with thisproblem (C)/No Error (D)(a) A (b) B (c) C (d) D

81. The mother (A)/yearns for (B)/her only child (C)/No Error (D)(a) A (b) B (c) C (d) D

Direction (Qs. 82-84) : The sentences given withblanks are to be filled with an appropriate word(s).Four alternatives are suggested for each question.For each question, choose the correct alternativeand click the button corresponding to it.82. Since last year his condition has been _____.

(a) deteriorate(b) deteriorating(c) severely deteriorated(d) deterioratingly

83. Most children remain _____ school between theages of seven and ten.(a) in (b) under (c) at (d) inside

84. The quake also _____ mudslides on the outskirtsof the city.(a) tempered (b) tampered(c) erupted (d) triggered

Direction (Qs. 85-87) : In each of the questions, fouralternatives are given for the Idiom/Phrase. Choosethe alternative which best expresses the meaning ofthe Idiom/Phrase and click the button correspondingto it.

85. Wet behind the ears(a) Skillful(b) Young and rich with experience(c) Young and without much expreience(d) Hearing impaired

86. Under a cloud(a) Sheltered from the sun(b) Under suspicion(c) Out in the open(d) Get wet

87. Get the sack(a) Look for a sack (b) Find a sack(c) Get a call back (d) Be dismissed

Direction (Qs. 88-90) : Out of the four alternatives,choose the one which can be substituted for the givenwords/sentences and click the button correspondingto it.

88. A person's peculiar habit(a) Trait (b) Idiosyncracy(c) Idiolect (d) Talent

89. Speech delivered without preparation(a) Rhetoric (b) Oration(c) Extempore (d) Maiden speech

yoursmahboob.w

ordpress.com

402 SSC CGL SolvedPaper

90. One who will do any job for anyone for money(a) Mercenary (b) Recruit(c) Hoodlum (d) Merchant

Direction (91-95) : A sentence/a part of the sentenceis underlined. Four alternatives are given to theunderlined part which will improve the sentence.Choose the correct alternative. In case noimprovement is needed, "No improvement" is theanswer.91. The more they earn, more they spend

(a) The more they earn, the more they spend(b) More they earn, the more they spend(c) More they earn, more they spend(d) No improvement

92. The offerings were placed on the alter(a) on the altar (b) on the steps(c) on the platform (d) No improvement

93. Really speaking, no man is perfect(a) strictly (b) slightly(c) vaguely (d) No improvement

94. Mankind does not know this(a) do (b) had(c) is (d) No improvement

95. Practically every part of the coconut tree is usedby man(a) each (b) most(c) any (d) No improvement

Direction (96-100) : A passage is given with 5questions following it. Read the passage carefully andchoose the best answer to each question out of thefour alternatives and click the button correspondingto it.Dyslexia is a perceptual disorder often occurring inpersons of normal, or even above average intelligence.The reader is unable to perceive correctly what is on apage. Letters and numbers often appear reversed: "b"seems to be "d", "quite" is "quiet" and "from" is "form.The reader tends to leave out letters or words or insertwords or letters that are not there. Vowel andconsonant sounds may be confused. Many dyslexicsare left-handed or able to write with either hand. Theyoften confuse left and right. Learning to speak may

also be delayed beyond infancy. The condition seemsto be inherited. It may persist into adulthood. Howere,with early recognition and specialized approaches toteaching reading, most dyslexics can learn to read.Some researchers believe that latent dyslexia may beaggravated by the way reading is taught. The modernwhole-word, or look-and-say, method seems to be moreof a hindrance to learning for dyslexics than it is forordinary pupils. The phonetic method of teachingstudents to learn letters and sound them out appearsto achieve better reading results. The problem of wordsthat cannot be sounded out such as rough, laugh orthrough-is not solved by phonetics. These words mustsimply be memorized. However, for children withdyslexia the problem can be compounded by the failureof parents or teachers to recognize the condition. Thiscan easily lead to emotional problems for dyslexicchildren, who cannot understand their failure to keepup with their classmates.96. Dyslexia, often occurring in persons of normal,

or even above average intelligence, is a_____(a) Conceptual disorder(b) Pathological disease(c) Perceptive disorder(d) Perceptual disorder

97. In Dyslexia, letters and figures often appear ____(a) Inverted (b) Blurred(c) Reversed (d) Clustered

98. People suffering from dyslexia are often _____(a) right handed (b) far sighted(c) ambidextrous (d) only left handed

99. Dyslexia may _______(a) be noticed during infancy(b) last till childhood(c) persist into adulthood(d) end when one goes to school

100. The problem of perception can be compoundedby the failure of parents and teachers to ____.(a) provide treatment(b) recognize the condition(c) correct the child at infancy(d) understand the child

yoursmahboob.w

ordpress.com

403SSC CGL SolvedPaper

1. (d) Cytology is study of Cell similarlyOrnithology is study of bird.

2. (*)

R I G T–5

W D B Y+5 –5 +5

F U S H+5 –5 –5 +5

K P N M¯ ¯ ¯ ¯

3. (b) 6 × 7 = 42; 7 × 8 = 56, : : 10 × 11 = 110,11 × 12 = 132.

4. (d) Rupee, Pound, Yen are currencies ofcountry.

5. (c) (a) D – 3 = A, A + 4 = E, E + 3 = H(b) K – 2 = I, I + 4 = M, M + 3 = P(c) H – 3 = E, E + 4 = I, I + 3 = L(d) F – 3 = C, C + 4 = G, G + 3 = J

6. (b) 43 – 6 = 37, 28 – 4 = 24, 50 – 7 = 43, 36 – 5 = 31Here except 24 all are prime no.

7. (b) Words arranged as per order in theDictionary, Directed, Directing, Direction,Director.

8. (c) Each alphabet in the term follows +1, +1and – 1 pattern of the next term’s alphabet.

9. (c)6 10 18 34 66 130

+ 4 + 8 + 16 + 32 + 64

10. (d)

Monica is the mother of Reeta.11. (c) When John interchange his position, new

Position will be 33 from left and 8 from RightTotal = 33 + 8 – 1 = 40

12. (b) DISMISS cannot be make fromADMISSION

13. (b) Each Alphabet of QBXZEBO is 3 less thanTEACHER

\ STUDENT will be written as PQRABKQ14. (c) Putting × = +, – = ÷ ÷ = – and + = ×

Only 16 + 5 – 10 × 4 ÷ 3 = 9

Satisfy the equation 16 × 5 ÷ 10 + 4 – 3 = 99 = 9

15. (b) = 524, = 723,

= 876

\ = 45916. (a) Each coloumn sums upto 99\ 99 – (35 + 28) = 36

17. (a)

18. (b) Possible venn diagram

Lazy LazyStenographer

Men

Stenographer

Men

Conc I FalseConc II True

19. (*)20. (d)

GardenHouse

Kitchen

As kitchen is in house and Garden isoutside the house.

21. (b) 22. (d) 23. (d) 24. (c)25. (d) 99, 12, 86, 22 are the codes matched from

two Matrices, for GUNS.26. (d) BRICS is the acronym for an association

of five major emerging national economies: Brazil,Russia, India, China and South Africa. Originallythe first four were grouped as “BRIC” (or “theBRICs”), before the induction of South Africa in2010.

HINTS & SOLUTIONS

yoursmahboob.w

ordpress.com

404 SSC CGL SolvedPaper

27. (d) "Do or Die" associated with the Quit IndiaMovement. On 9 August 1942, MohandasKaramchand Gandhi launched the Quit IndiaMovement. On the night of 8th August 1942addressing the Congress delegates MahatmaGandhi gave the Slogan "DO or Die". The meanof Do or Die is we shall free India or die in theattempt. You know that Prime Minister of WinstonChurchill at that time was pressured by theAmericans President Franklin D. Roosevelt togive into Indian demand. The Quit Indiacampaign was effective crushed.

28. (b) A parsec (symbol: pc) is a unit of lengthused to measure large distances to objectsoutside our Solar System.

29. (b) The collective term given by the Jainas totheir Sacred literature is called Agamas writtenin Prakrt.

30. (a) The major instruments of fiscal policy areas follows: A. Budget B. Taxation C. PublicExpenditure D. Public Works E. Public Debt.

31. (a)32. (a) Phenol, also known as carbolic acid, is an

aromatic organic compound with the molecularformula C6H5OH. It is a white crystalline solidthat is volatile. The molecule consists of a phenylgroup (–C6H5) bonded to a hydroxyl group (OH).

33. (b) Tehri Dam is a multi-purpose rock andearth-fill embankment dam on the BhagirathiRiver near Tehri in Uttarakhand, India.

34. (d) India has 15,106.7 km of land border and acoastline of 7,516.6 km including islandterritories. The length of our land borders withneighbouring countries is as follows :Bangladesh : 4,096.7 China : 3,488 Pakistan : 3,323Nepal : 1,751 Myanmar : 1,643 Bhutan : 699Afghanistan : 106.

35. (c) Gujarat is strategically located with largestshare in India’s coastline, followed by AndhraPradesh and Tamil Nadu.

36. (c) Jog Fall located at southern state ofKarnataka is India’s highest waterfall. Its heightis 830 feet and is located on Sharavati River.

37. (b) 38. (c) 39. (b)40. (b) It grows larger until puberty and then

begins to shrink. The gland produces thymosins,which are hormones that stimulate thedevelopment of antibodies.

41. (b) 42. (d) 43. (d) 44. (c)45. (a)46. (d) The Reserve Bank of India (RBI) Governor,

RaghuramRajan, on 4 August 2016 launched

Sachet portal, sachet.rbi.org.in, tocheck illegalmoney collection.

47. (d)48. (a) Bezwada Wilson and TM Krishna are

included in the list of the prestigious RamonMagsaysay Award winners for the year 2016.

49. (d) 50. (b)51. (d) Let B takes day = 2x

A takes = x1 1 1

2 16x x+ =

3 12 16x

= x = 2452. (c) Successive discount = 15, 20, 25

net discount when, 15, 20 taken together15× 20

15 + 20 –100

= 35 – 3 = 32

Now taking 22 and 2525× 32

32 + 25 –100

80057 –100

57 – 8 = 49%53. (c) Let x, y and z be 2m, 3m and 5m

According to Question2m + 3m +5m = 8010m = 80, m = 8z = a x – 85(8) = a(2)(8) – 840 + 8 = 16 a48 = 16 aa = 3

54. (b) Let CP = 10, SP = 11

P % = 1

10100´ = 10%

55. (b) by using x + y + xy

100 = 0Let Price be reduced by = x%

60 + x + 60x100

0=

160x100

– 60=

x = 6000160

1– 372

= (– shows reduction)

56. (a) Total Distance = 100 kmdistance travelled in first 2hr = 2 × 20 = 40Remaining distance = 100 – 40 = 60 km

Time taken = 6010 = 6 hr

yoursmahboob.w

ordpress.com

405SSC CGL SolvedPaper

Av speed = Total distance

total time = 100

8

= 112 Km/hr257. (c) Let principal = 10x

Interest = 3xPRT SI100

=

10x × 6 ×T3x

100=

T = 5 years58. (c) (x – 2) (x – p) = x2 – ax + b

x2 + ( – 2 – p)x + (– P) (–2) = x2 – ax + b

– (2 + p) = – ab( )

aa b -+ =Q

2 = a – p

59. (b)22x + = 3x

Dividing eq by 21 3

x + =x 2 Cubing both sides

1 27x + =

x 8

3æ öç ÷è ø

33

1 1 27x + + 3 x + =x x 8

æ öç ÷è ø

33

1 27x + =x 8

3– 32æ öç ÷è ø

33

1 27x + =

x 89

2 – 22

+ +

=27– 36 16

8+

=78

60. (c) In D A B CÐ A + Ð B + Ð C = 180

Ð B = 180 – [Ð A + Ð C)= 180 – 140 = 40°

Ð A + 3Ð B = 180Ð A = 180 – 3 (40)°

= 180 – 120= 60°

61. (b) Ð A P B = 80°Ð A O B= 180 – 80 = 100°

Ð A O P =100

2 = 50°

62. (b) tan (5x – 10°) = cot(5y + 20°)tan (5x – 10°) = tan (90° – 5y – 20°)

5x – 10° = –5y + 70°5x + 5y = 80°

x + y = 1663. (d) Product of 2 no is = 24

Possible pair of factor = (1, 24) (2, 12) (3, 8)(4, 6) i,e., 4 + 6 is minimum = 10

64. (b) Total students = 6030×85 + 30x

6065=

30x = 65 × 60 – 30 × 8530x = 30[65 × 2 – 85]

x = 30[130 – 85] = 4530

65. (a) x = a 1a

+ y = a 1–a

x4 + y4 – 2 x2y2

= (x2 – y2)2

x2 = a 12

a+ +

y2 = a 1–2

a+

(x2 – y2)2 = 21 1

a + + 2 – a – + 2a a

é ùê úë û

= (4)2 =16

66. (b) x = 23 11 – 2x +

(x + 2) = 23 11x + (Cubing both sides)(x + 2)3 = x2 + 11x3 + 8 + 6x2 + 12x = x2 + 11x3 + 5x2 + 12x = 3

67. (b) 5 + 8 < 15\ 5, 8, 15 cannot Form a D

68. (b) Ð P O A = 120°Ð P O A = Ð OPB + Ð PBO

yoursmahboob.w

ordpress.com

406 SSC CGL SolvedPaper

A B

P

O120

Q OP = OB\ Ð OPB = Ð PBO2 Ð PBO = 120°, Ð PBO = 60°

69. (b) Let BC be the width of river in D A B CABBC = tan 45°

200BC

1=

BC = 200

In D ABD 45°200

60°

A

BC DAB = tan60°BD

BD = 200

3

DC = 200200 +

370. (a) Radius of large sphere = 9 cm

Radius of smaller sphere = 6 cmRadius of cylinder = 6 cmRadius height of cylinder = h4

h3

3 3 249 6 (6)

3p p p= +

4h

33 3 2(9 – 6 ) 6=

h = 43

3 3

2

[9 – 6 ]6

h = 43

[729 – 216]6 6´

h = 4×5133× 6× 6

= 19 cm

71. (c) 1q = 30° 2q = 2qArc l1 = 2l l2 = lr1 = r r2 = 3r

Arc length = 2 r360°q

p

l1

l2 =

1 1

2 2

2 r / 3602 r / 360p qp q

2ll 2

r 30=3r q

2q = 30 = 5°6

72. (b) Total FDI of 1992 and 1993= 5.70 + 10.15= 15.85 cr

73. (d) 199674. (a) Average FDI

= [5.70 +10.15 + 20.16 +10.22 + 24.23+ 31.36)

6

= 101.72 16.95

6=

FDI in 1997 = 31.36FDI in 1997Average FDI =

31.36 2 :116.95

»

75. (b) Absolute Difference = [24.23 – 31.36] = 7.1376. (d) 77. (b) 78. (d) 79. (c)80. (b) 81. (d) 82. (b) 83. (c)84. (d) 85. (c) 86. (b) 87. (d)88. (b) 89. (c) 90. (a) 91. (a)92. (a) 93. (a) 94. (d) 95. (d)96. (d) 97. (c) 98. (c) 99. (c)100. (b)

yoursmahboob.w

ordpress.com

407SSC CGL SolvedPaper

PART-A : GENERAL INTELLIGENCE & REASONING

Directions (Qs. 1-3) : Select the related word/letters/numbers from the given alternatives:

1. Medicine : Patient : : Education : ?(a) Teacher (b) School(c) Student (d) Tuition

2. LAMP : IXJM :: FISH : ?(a) CGPF (b) CFQE (c) CFPE (d) CGQF

3. 13 : 20 : : 17 : ?(a) 25 (b) 26 (c) 27 (d) 28

Directions (Qs. 4-6) : Find the odd word/letter/number form the given alternative.4. (a) Rival (b) Opponent

(c) Foe (d) Ally5. (a) POCG (b) KLIZ

(c) BUDX (d) FQMV6. (a) 36 - 48 (b) 56 - 44 (c) 78 - 66 (d) 33 - 647. Arrange the following words as per order in the

dictionary1. Ambitions 2. Ambiguous3. Ambiguity 4. Animation5. Animal(a) 3 ,2 , 4 , 1 , 5 (b) 3 , 2 , 5 , 4 , 1(c) 3 , 2 , 1 , 5 , 4 (d) 3 , 2 , 4 , 5 , 1

8. A series is given, with one term missing. Choosethe correct alternative from the given ones thatwill complete the series.CAT , DBT , ECT , ?(a) DCT (b) FDT (c) FCT (d) FAT

9. A series is given , with one term missing . Choosethe correct alternative from the given ones thatwill complete the series .5 , 11 , 24 , 51 , 106 ,____?(a) 122 (b) 217 (c) 120 (d) 153

10. Showing a man on the stage , Rita said , " He isthe brother of the daughter of the wife of myhusband . How is the man on stage related toRita ?(a) SON (b) HUSBAND(c) COUSIN (d) NEPHEW

SSC Combined Graduate Level (CGL) Solved PaperMORNING SHIFT 11 SEPT, 2016

11. The age of Dr. Pandey is four times the age of hisson. After 10 years , the age of Dr. Pandey will betwice the age of his son . The present age ofDr. Pandey's son is(a) 4 years (b) 5 years(c) 6 years (d) 8 years

12. From the given alternative words , select the wordwhich cannot be formed using the letters of thegiven word:I N T E L L I G E N C E(a) CANCEL (b) NEGLECT(c) GENTLE (d) INCITE

13. If C is coded 3 , DASH is coded as 32 , thenDANCE will be coded as(a) 20 (b) 25 (c) 26 (d) 27

14. If ' + ' means minus , ' - ' means multiplication,' ÷ ' means plus , and ' x ' means division , then15 - 3 + 10 x 5 ÷ 5(a) 52 (b) 48 (c) 22 (d) 5

15. A certain system is followed to solve the problem.Accordingly find out the correct answer fromthe alternatives for the unsolved equations.7 × 4 × 9 = 4799 × 5 × 2 = 5926 × 9 × 5 = 9658 × 6 × 2 = ?(a) 286 (b) 682 (c) 628 (d) 268

16. Find the missing number from the givenalternatives

7 10 516 40 815 ? 9

(a) 75 (b) 45 (c) 20 (d) 3017. Sanmitra walked 8 m towards the north. He turned

to his right and walked 16 m, then he turned tohis left and walked 5 m and again he turned tohis left and walked 16 m. In which direction is hefrom his starting point?(a) South (b) East (c) North (d) West

yoursmahboob.w

ordpress.com

408 SSC CGL SolvedPaper

18. Consider the given statement/s to be true anddecide which of the given conclusions/assumptions can definitely be drawn from thegiven statement.Statements :1. Some teacher are followers.2. Some followers are famous.Conclusions :I. Some teachers are famous.II. Some followers are teachers.(a) Only conclusion I follows(b) Only conclusion II follows(c) Both conclusion I and II follows(d) Neither conclusion I nor II follows

19. How many triangles are there in the questionfigure?

(a) 18 (b) 24 (c) 28 (d) 3020. Identify the diagram that best represents the

relationship among classes given belowDelhi , Sri Lanka , Asia

(a) (b)

(c) (d)

21. Which answer figure complete the pattern in thequestion figure?

(a) (b)

(c) (d)

22. From the given figure, select the one in thequestion the figure is hidden / embedded

(a) (b)

(c) (d)

23. A piece of paper is folded and cut as shownbelow in the given answer figures , indicate howit will paper when opened.

(a) (b)

(c) (d)

yoursmahboob.w

ordpress.com

409SSC CGL SolvedPaper

24. If a mirror is place on the line MN , then which ofthe answer figures is the right image of the figure?

(a) (b)

(c) (d)

25. In the question, a word is represented by onlyone set of numbers as given in any one of thealternatives. The sets of numbers given in thealternatives are represented by two classes ofalphabets as in two matrices given below. Thecolumns and rows of Matrix I are numbered from0 to 4 and that of Matrix II are numbered from5 to 9. A letter these matrix can be representedfirst by row and next by its column , e. g., ' A ' canbe represented by 01 , 13 , etc. ., and ' B ' can berepresented by 58 , 69 , etc. . Similarly , you haveto identify the set for the word ' FINE '

MATRIX-I MATRIX-II0 1 2 3 4 5 6 7 8 9

56789

SHBET

ESTHS

HESTE

BTEBH

TBHSB

01234

FIAON

AONFI

NFOIA

OAINF

INFAO

(a) 00 , 04 , 02 , 56 (b) 12 , 10 , 13 , 67(c) 24 , 19 , 31 , 78 (d) 31 , 32 , 33 , 87

PART-B : GENERAL AWARENESS26. Who propounded the "Doctrine of Passive

Resistance"?(a) Balgangadhar Tilak(b) Aurobindo Ghosh(c) Lala Lajpat Rai(d) Bipin Chandra Pal

27. At Rio Olympics, what was the final position ofDipa Karmakar in her category?(a) 4 (b) 5 (c) 6 (d) 7

28. The 14th Finance Commission has recommendedincrease in States share in net proceeds from taxcollection from 32% to(a) 35% (b) 40% (c) 42% (d) 45%

29. Fiscal policy in India is formulated by?(a) Reserve Bank of India(b) Planning Commission(c) Finance Ministry(d) SEBI

30. Which of the following has pH value 7?(a) Pure Water (b) H2SO4(c) Sodium Carbonate (d) HCl

31. When ice cubes are made, the entropy of water(a) does not change(b) decreases(c) increases(d) may either increase or decrease depending

on the process used32. Two vectors are said to be equal if

(a) only their magnitudes are same(b) only their directions are same(c) both magnitude and direction are same(d) magnitudes are same but directions areopposite

33. The chemical component that is invariably foundin all viruses is(a) proteins (b) lipids(c) DNA (d) RNA

34. Which is used as an Air pollution indicator?(a) Algae (b) Fungi(c) Bacteria (d) Lichens

35. The study of mountains is known as(a) Oncology (b) Lithology(c) Orology (d) Ornithology

36. Salination of soil is caused by(a) Pesticides (b) soil erosion(c) excess irrigation (d) crop rotation

37. ICC World T20 in 2016 was won by(a) India (b) England(c) New Zealand (d) West Indies

yoursmahboob.w

ordpress.com

410 SSC CGL SolvedPaper

38. Which one of the following was established witha definite provision under an Article of theConstitution of India?(a) Union Public Service Commission(b) National Human Rights Commission(c) Election Commission(d) Central Vigilance Commission

39. The President can advance money to meetunforeseen expenses from the(a) Consolidated Fund of India(b) Grants of the Central Government(c) Aid from the Union Government(d) Contingency Fund

40. The famous Vishnu temple at Angkor Wat inCambodia was built by ?(a) Shrutavarman (b) Suryavarman II(c) Indravarman (d) Aniruddha

41. In IT, the technique of delaying outgoingacknowledgements temporarily is known as(a) AR Acknowledgement(b) AR request(c) Piggybacking(d) Piggyframing

42. If speed of rotation of the earth increases, theweight of the body(a) decreases(b) increases(c) remains the same(d) may increase or decrease

43. Which among the following countries is facingdebt crisis?(a) China (b) Canada(c) Germany (d) Greece

44. The new symbol of Indian currency is designedby?(a) Santosh Kumar (b) Y.V. Reddy(c) Udayakumar (d) Dr. Rangarajan

45. What is MUDRA?(a) Development and Refinance Agency(b) Scheme for Agricultural Insurance(c) New Planet Discovered(d) Development and Regulatory Authority for

Urban Township46. Which of the following is not correctly matched?

(a) Sania Mirza - Tennis(b) Sania Nehwal - Badminton(c) Mary Kom - Boxing(d) Pankaj Advani - Wrestling

47. Which of the following State is surrounded byBangladesh from three sides?(a) Nagaland (b) Assam(c) Arunachal Pradesh (d) Tripura

48. Which enzyme digests proteins in the stomach?(a) Trypsin (b) Pepsin(c) Salivary amylase (d) Pancreatic canal

49. Fermentation is a type of _____ _______process.(a) Aerobic Respiration(b) Anaerobic Respiration(c) Exothermic Reaction(d) Transpiration

50. The first Muslim to be elected President of'Indian National Congress' was?(a) Maulana Azad(b) Mohammed Ali(c) Badruddin Tyabji(d) Shah Wali-ullah

PART-C : QUANTITATIVE APTITUDE51. A,B and C contract a work for ` 440. Together A

and B are to do 9/11 of the work. The share of Cshould be:(a) 75 (b) 90 (c) 100 (d) 80

52. A dealer marks a washing machine for ` 7500,and allows a discount of 6% on it. Find the sellingprice(a) 6850 (b) 7050 (c) 7250 (d) 6950

53. The sum of the cubes of two numbers in theratio 3 : 4 is 5824. The sum of the numbers is:(a) (5824)1/3 (b) 28(c) 24 (d) 14

54. Loss of 20% on selling price is equal to x% lossin cost price. What is x?

(a) 20% (b) 20 (c)2

16 %3 (d) 16

55. In a motor of 120 machine parts, 5% parts weredefective. In another motor of 80 machine parts,10% parts were defective. For the two motorsconsidered together, the percentage of defectivemachine parts were(a) 7 (b) 6.5 (c) 7.5 (d) 8

56. If (23)2 = 4x then 3x is equal to(a) 3 (b) 6 (c) 9 (d) 27

57. A train, 200 m long, is running at a speed of54 km/hr. The time in seconds that will be takenby train to cross a 175 m long bridge is(a) 12.5 (b) 20 (c) 25 (d) 10

yoursmahboob.w

ordpress.com

411SSC CGL SolvedPaper

58. Which of the following ratios can be the ratio ofthe sides of a right angled triangle?(a) 9 : 6 : 3 (b) 13 : 12 : 5(c) 7 : 6 : 5 (d) 5 : 3 : 2

59. Number of circles that can be drawn throughthree non-colinear points is(a) exactly one (b) two(c) three (d) more than three

60. Value of (cos 53° – sin 37°) is(a) 0 (b) 1(c) 2 sin 37° (d) 2 cos 53°

61. If 14x + = 5,x

× 0,¹ then the value of

2

5x4x +10x +1

is

(a) 1/2 (b) 1/3 (c) 2/3 (d) 3

62. If .....

1 3,C

C + =Ö then the value of 3

1C

3C + is

equal to(a) 0 (b) 3Ö3(c) 1/Ö3 (d) 6Ö3

63. What least value must be assigned to '*' so thatthe numbers 451 * 603 is exactly divisible by 9?(a) 7 (b) 8 (c) 5 (d) 9

64. The average of 9 observations was found to be35. Later on, it was detected that an observation81 was misread as 18. The correct average of theobservations is(a) 28 (b) 42 (c) 32 (d) 45

65. If x = 222, y = 223, z = 225 then the value of x3 + y3

+ z3 – 3xyz is(a) 4590 (b) 4690 (c) 4950 (d) 4960

66. If x = 31/3 – 3–1/3 then 3x3 + 9x is equal to(a) 5 (b) 6 (c) 7 (d) 8

67. Two circles touch each other internally. Theradius of the smaller circle is 6 cm and thedistance between the centre of two circles is3 cm. The radius of the larger circle is(a) 7.5 cm (b) 9 cm (c) 8 cm (d) 10 cm

68. PQR is an equilateral triangle. MN is drawnparallel to QR such that M is on PQ and N is onPR. If PN = 6 cm, then the length of MN is(a) 3 cm (b) 6 cm (c) 12 cm (d) 4.5 cm

69. If cosec q + sin q = 5/2 then the value of cosecq– sinq is(a) –3/2 (b) 3/2 (c) – Ö3/2 (d) Ö3/2

70. Two men are on opposite sides of a tower. Theymeasure the angles of elevation of the top of thetower as 30° and 45° respectively. If the heightof the tower is 50 m, the distance between thetwo men is (Take Ö3 = 1.7)(a) 136.5 m (b) 50 Ö3 m(c) 100 Ö3 m (d) 135.5 m

71. The radius and the height of a cone are eachincreased by 20%. Then the volume of the coneincreases by(a) 20% (b) 20.5% (c) 62% (d) 72.8%

Directions (Qs. 72-75) : A bar graph showing theheights of six mountain peaks. Study the bar graphand answer questions

8200

6000

86007500

8800

6500

A B C D E F

9000

80007000

600050004000300020001000

0

Heightinmeters

Mountain peaks

72. The average height of all the peaks (in meters) is(a) 7601.5 (b) 7600 (c) 7599.5 (d) 7610

73. Which peak is the second highest?(a) B (b) C (c) A (d) E

74. Write the ratio of the heights of the highest peakand the lowest peak(a) 22 : 15 (b) 15 : 22 (c) 20 : 13 (d) 13 : 22

75. When the heights of the given peaks are writtenin ascending order, what is the average of themiddle two peaks?(a) 7950 m (b) 7560 m (c) 7650 m (d) 7850 m

PART-D : ENGLISH LANGUAGE

Directions (Qs. 76-77) : In the following question,out of the four alternatives, choose the word whichbest expresses the meaning of the given word.76. CREDULOUS

(a) FUNNY (b) SILLY(c) INNOCCENT (d) GULLIBLE

77. REASSURE(a) COMFORT (b) CONSOLE(c) INSPIRE (d) DISCOURAGE

yoursmahboob.w

ordpress.com

412 SSC CGL SolvedPaper

78. Four words are given, out of which only oneword is spelt correctly. Choose the correctly speltword and click the button corresponding to it.(a) Defianse (b) Defyance(c) Difiance (d) Defiance

Directions (Qs. 79-81) : In the following questions,one part of the sentence may have an error. Find outwhich part of the sentence has an error and click thebutton corresponding to it. If the sentence is freefrom error, click the "No error" option.

79. Too great a (A)/ variety of studies (B)/ distractthe mind (C)/ No Error (D)(a) A (b) B (c) C (d) D

80. The severe cyclonic storm (A)/ has left behind(B)/ a trial of misery (C)/ No Error (D)(a) A (b) B (c) C (d) D

81. My sister has been (A)/ interested in medicine(B)/ ever since she was a child (C)/ No Error (D)(a) A (b) B (c) C (d) D

Directions (Qs. 82-84) : The sentences given withblanks are to be filled with an appropriate word(s).Four alternatives are suggested for each question.For each question, choose the correct alternative.

82. It is __________ university of excellence.(a) a (b) an (c) also (d) one

83. They have not had much time for social activitiesand I have not __________ .(a) done so (b) either(c) also (d) gone

84. Food was packed, and they__________, amerry, delighted party.(a) went off (b) start off(c) cut off (d) set off

Directions (Qs. 85-87) : In each of the questions,four alternatives are given for the Idiom/Phrase.Choose the alternative which best expresses themeaning of the Idiom/Phrase and click the buttoncorresponding to it.

85. pull a fast one(a) as fast as lightning(b) play a trick(c) carry a heavy burden(d) take a deep breath

86. Grease the palm(a) dirty one's hands (b) work in a garage(c) slip and fall (d) bribe

87. turn turtle(a) slow like a turtle (b) turn like a turtle(c) over-turn (d) a game turtles play

Directions (Qs. 88-90) : Out of the four alternatives,choose the one which can be substituted for the givenwords/sentences and click the button correspondingto it.

88. Action that is likely to make people very angry(a) Inflationary (b) Inflammable(c) Commensurable (d) Inflammatory

89. A humorous drawing dealing with current eventsor politics.(a) Sketch (b) Illustration(c) Cartoon (d) Skit

90. Act of mercy killing(a) Suicide (b) Euthanasia(c) Immolation (d) Asphyxiation

Directions (Qs. 91-95) : A sentence/a part of thesentence is underlined. Four alternatives are givento the underlined part which will improve thesentence. Choose the correct alternative and clickthe button corresponding to it . In case noimprovement is needed, "No improvement" is theanswer.

91. They have had a real good time.(a) have had a (b) have had really(c) have had a really (d) No improvement

92. Electricity has modernized the march of modernevents.(a) revolutionised (b) developed(c) created (d) No improvement

93. The sales boy told the consumer not to touchthe products on display.(a) buyer (b) shopper(c) customer (d) No improvement

94. Please make it a point to send the letter at myaddress.(a) on (b) to(c) in (d) No improvement

95. Sunil and I helped to sell tickets.(a) we (b) us(c) me (d) No improvement

yoursmahboob.w

ordpress.com

413SSC CGL SolvedPaper

Directions (Qs. 96-100) : A passage is given with 5questions following it. Read the passage carefullyand choose the best answer to each question out ofthe four alternatives and click the buttoncorresponding to it.To know language is to be able to speak it; even achild who does not yet attend school can speak his orher language. In order to speak a language it isimportant to listen to it and to read a few pages in iteveryday. A child picks up language and learns to talkjust as (s) he learns to walk. Walking and talking comesnaturally to a child as it grows. In our country, a childmay grow up speaking more than one language, ifthese languages are spoken in the home and in theneighbourhood. we call this multilingualism. A childspeaks a language or languages much before (s)hestarts going to school. To know a language then isfirst of all to be able to speak it as easily and naturallyas a tree year old child does. Later on the child willlearn to read and write in that language. In order toread and write in a language, one has to speak it. Butit is possible to speak a language but not able to reador write in it. A baby does not speak until it is ninemonths old but it understands a few words at sixmonths of age. It has been listening ever since it was

born, and even a little before that. So the first strategyin speaking a language is to listen.96. One of the activities of a child before it is even

born is __________ .(a) seeing (b) listening(c) understanding (d) talking

97. It is necessary for one to __________ thelanguage before (s)he writes in that language.(a) sing (b) spell(c) speak (d) none of the above

98. Multi-lingualism means(a) speaking more than one language(b) speaking only one language(c) speaking any language(d) speech

99. A child has been __________ ever since it wasborn(a) speaking (b) reading(c) walking (d) listening

100. To know a language one must be able to?(a) Speak it as easily and naturally as a three

year old child.(b) Read it well all the time.(c) Write it quickly(d) Sing in the language

HINTS & SOLUTIONS

1. (c) Medicine is given to patient similarlyEducation is given to student.

2. (a) L A M P

I X J M–3 –3 –3 –3¯ ¯ ¯ ¯

F I S H

C F P E–3 –3 –3 –3¯ ¯ ¯ ¯

3. (d) As 20 = 13 × 2 – 6\ 28 = 17 × 2 – 6

4. (d) Ally is Antonym of Remaining three.5. (d) Only FQMV is without any Vowel6. (d) Difference between 36 – 48, 56 – 44 and

78 – 64 is 12 where as 33 – 64 is different.7. (c) Arrangement as per Dictionary rules is 3, 2,

1, 5, 4i.e Ambiguity, Ambiguous, Ambitions,Animation

8. (b)

9. (b) 5 × 2 + 1 = 1111 × 2 + 2 = 2424 × 2 + 3 = 5151 × 2 + 4 =106106 × 2 + 5 = 217

10. (a)

That man is son of Reeta11. (b) Age of Son = x, Age of doctor = 4x

4x + 10 = 2(x + 10)4x – 2x = 20 – 102x = 10x = 5Age of Son = 5 year

yoursmahboob.w

ordpress.com

414 SSC CGL SolvedPaper

12. (a) CANCEL cannot be Form fromINTELLIGENCE

13. (d) D A S H Þ 4 + 1 + 19 + 8 = 32D A N C E Þ 4 + 1 + 14 + 3 + 5 = 27

14. (b) Putting + = – , – = ×; ÷ = – and × = ÷in 5 – 3 + 10 × 5 ÷ 5Þ 15 × 3 – 10 ÷ 5 + 5

45 – 2 + 5 = 48

15. (b) = 4 7 9, = 5 9 2,

= 9 6 5 = 6 8 2

16. (d) 7 10 516 40 815 ? 9

(7 – 5) × 5 = 10(16 – 8) × 5 = 40(15 – 9) × 5 = 30

17. (c)

He is in north direction from starting point

18. (b)

Only Conclusion II follows.19. (c)

20. (b)

Delhi and Srilanka both are in Asia.21. (a) 22. (b) 23. (d) 24. (a) 25. (a)26. (b) Doctrine of Passive Resistance is authored

by Aurobindo Ghosh. It is based on a series ofarticles by Aurobindo Ghosh which werepublished in April 1907 in the journal BandeMatram. The articles were written when theBengal was burning with indignation after its

partition October 16, 1905 by Viceroy Curzon.The main theme of the articles is the methodsand ideology which could be adopted in face ofoppression and injustice as perpetrated by thecontemporary British rule in India.

27. (a) Dipa Karmakar returned empty handed fromthe Vaults final at the Gymnastics event of Rio2016 Olympics after she finished the event atfourth rank.

28. (c) 29. (c) 30. (a) 31. (b) 32. (c)33. (a)34. (d) Lichens can be used asair pollution

indicators, especially of the concentration ofsulfur dioxide in the atmosphere. Lichens areplants that grow in exposed places such as rocksor tree bark. They need to be very good atabsorbing water and nutrients to grow there.

35. (c) Orology is the study of mountains.Anexample of orology is research on how a particularmountain came to be formed.

36. (c) Soil salinity is the salt content in the soil;the process of increasing the salt content isknown as salinization. Salts occur naturally withinsoils and water. Salinization can be caused bynatural processes such as mineral weathering orby the gradual withdrawal of an ocean.

37. (d) 38. (c) 39. (d)40. (b) It was built by the Khmer King Suryavarman

II in the early 12th century.41. (c) 42. (a) 43. (d)44. (d) The Indian rupee sign is the currency sign

for the Indian rupee, the official currency of India.Designed by D. Udaya Kumar, it was presentedto the public by the Government of India on 15July 2010, following its selection through an"open" competition among Indian residents.

45. (a) Micro Units Development and RefinanceAgency Bank (or MUDRA Bank) is a publicsector financial institution in India.

46. (d) 47. (d) 48. (b) 49. (b)50. (c) Badruddin Taiyabji became the Ist Indian

Barrister in Bombay; became the 2nd Indian ChiefJustice; was the founding member of Bombaypresidency association and INC and alsopresided over the 3rd congress session in Madrasin 1887.

51. (d) Remaining work = 9 2

1–11 11

=

C will get = 2 × 440 = 2× 40

11 = 80

yoursmahboob.w

ordpress.com

415SSC CGL SolvedPaper

52. (b) Market Price = 7500, Discount = 6%

Selling Price = 94 × 7500

100 = 7050 ̀

53. (b) Let the number be 3x and 4x (3x)3 + (4x)3 = 5824 27x3 + 64x3 = 5824

91x3 = 5824 x3 = 64 x = 4

Sum of numbers are = x(4 + 3) = 4× 7 = 2854. (c) Let Selling Price = 100 Loss = 20%

Cost price = 120

Loss% of cost price = 20

×100120 =

2%

316

55. (a) Total deffective part = 5 10×120 + × 80

100 100 = 6 + 8 = 14

Deffective % = 14

×100 = 7%200

56. (d) (23)2 = 4x

26 = 22x

6 = 2xx = 333 = 27

57. (c) Time = DistanceSpeed

Time = 200 +175

554×18

= 37515

= 25 sec58. (b) As only 13, 12 and 5 follows Pythagorous

theorem59. (a)60. (a) cos 53° – sin 37° Þ cos 53° – sin (90° – 53°)

Þ cos53° – cos53°= 0

61. (b)1

4x + = 5x

Þ 4x2 + 1 = 5x

2

5x 5x4x +10x +1 10x + 5x

=

Þ5x 1

15x 3=

62. (a)1

3CC

+ =

Cubing both Sides

( )33

13 3 3 3C

C+ + =

33

10C

C+ =

63. (b) To divide 451 * 603 by 9(4 + 5 + 1 + * + 6 + 0 + 3) = (19 + *)(19 + *) must be multiple of 9\ 19 + * = 27* = 8

64. (b) Average of 9 observations = 35Total = 9 × 35 = 315Misread Difference = 81 – 18 = 63New total = 315 + 63 = 378

Average = 378

9 = 42

Shortcut Method:81 – 18 = 63

\ 639 = 7

35 + 7 = 42

65. (b) x3 + y3 + z3 – 3xyz = 1

(x + y + z)2

[(x – y)2 + (y – z)2 + (z – x)2]

= 1

(222 + 223 + 225)2

[(222 – 223)2 + (223 – 225)2 + (225 – 222)2]

Þ 1[670][1+ 9]

24 + = 335 [14] = 4690

66. (d) x = 31/3 – 3–1/3

Cubing on both sides

x3 = 1 3 –1 31 1 13 – – 3(3) 3 – 33 3 3

é ù- ë û

x3 = 19 – – 3(x)3

x3 + 3x = 9 –1

33x3 + 9x = 8

67. (b) O¢O = 3

OA = 6

O¢A = 6 + 3 = 9 cm

yoursmahboob.w

ordpress.com

416 SSC CGL SolvedPaper

68. (b) In Given equilateral D, MN || QR

PN MNPQ QR

=

PN = MN Q (PR = QR)

MN = 6 cm

P

Q R

M N

69. (b)1 5

cosecθ + =cosec θ 2

Squaring on both sides2

2

1 25cosec θ + =

cosec θ 42+

22

1 25cosec θ + =

cosec θ 42 – 4 – 4+

22

1 169cosec θ + =

cosec θ 4– 2

(cosec q – sin q) = 169

4

(cosec° q – sin° q) = 165

43

cosec θ – =2

qsin

70. (a) In D ADCAC = 50 mAC tan30DC

= °

50 1DC 3

=

D C = 50 3 in D ABCAC = tan45°BC

BC = 50

BD = 50 + 50 3

= ( )50 1+ 3= 50 (2.73) = 136.5 m

71. (d) Let radius and height of cone = 100New radius and height of cone = 120

Ratio = 120 6=100 5

Volume Ratio = 36 216

=5 125æ öç ÷è ø

Difference = 216 – 125 = 91

Difference % = 91

×100125 =

491×

5

= 364

5 = 72.8%

72. (b) Average

= 8200 + 6000 + 8600 + 7500 + 8800 + 6500

6= 7600

73. (b)74. (a) Highest peak = 8800

lowest peak = 6000

Ratio = 88006000 = 22 : 15

75. (d) Ascending order 6000, 6500, 7500, 8200,8600, 8800

Middle two average = 7500 + 8200

2

= 15700

2 = 7850 m

76. (d) Credulous- having or showing too great areadiness to believe things.Gullible- easily persuaded to believe something.

77. (d) Reassure-say or do something to removethe doubts and fears of (someone).Discourage- cause (someone) to lose confidenceor enthusiasm.

78. (d) Here variety, the main nation, is Singular inform. If the writer intends it for a collective Noun,he may reasonably use the Plural distract. But itseems better to treat variety as a quality andtherefore really Singular, in which case the verbshould be distract. At all events the Plural studies,being only part of a secondary Noun-phrase,does not excuse the Plural distract.

79. (c) 80. (b) 81. (d) 82. (a) 83. (b)84. (d)85. (b) pull a fast one- to succeed in an act of

deception.86. (d) grease somebody's palm also grease the

palm of somebody- to give someone money topersuade them to do what you want.

87. (c) turn turtle- to turn upside down.88. (d) 89. (c) 90. (b) 91. (b) 92. (a)93. (c) 94. (b) 95. (d) 96. (b) 97. (c)98. (a) 99. (d) 100. (a)